You are on page 1of 289

Vectors &

3D Geometry
With Sessionwise Theory & Exercises

Amit M. Agarwal
ARIHANT PRAKASHAN (Series), MEERUT
All Rights Reserved

© AUTHOR
No part of this publication may be re-produced, stored in a retrieval system or
by any means, electronic mechanical, photocopying, recording, scanning, web or
otherwise without the written permission of the publisher. Arihant has obtained
all the information in this book from the sources believed to be reliable and true.
However, Arihant or its editors or authors or illustrators don’t take any responsibility
for the absolute accuracy of any information published, and the damages or loss
suffered thereupon.
All disputes subject to Meerut (UP) jurisdiction only.

Administrative & Production Offices


Regd. Office
‘Ramchhaya’ 4577/15, Agarwal Road, Darya Ganj, New Delhi -110002
Tele: 011- 47630600, 43518550
Head Office
Kalindi, TP Nagar, Meerut (UP) - 250002 Tel: 0121-7156203, 7156204
Sales & Support Offices
Agra, Ahmedabad, Bengaluru, Bareilly, Chennai, Delhi, Guwahati,
Hyderabad, Jaipur, Jhansi, Kolkata, Lucknow, Nagpur & Pune.

ISBN : 978-93-25298-68-2

PO No : TXT-XX-XXXXXXX-X-XX
Published by Arihant Publications (India) Ltd.
For further information about the books published by Arihant, log on to
www.arihantbooks.com or e-mail at info@arihantbooks.com
Follow us on
PREFACE
“YOU CAN DO ANYTHING IF YOU SET YOUR MIND TO IT, I TEACH GEOMETRY TO JEE ASPIRANTS BUT
BELIEVE THE MOST IMPORTANT FORMULA IS COURAGE + DREAMS = SUCCESS”

It is a matter of great pride and honour for me to have received such an overwhelming response to
the previous editions of this book from the readers. In a way, this has inspired me to revise this
book thoroughly as per the changed pattern of JEE Main & Advanced. I have tried to make the
contents more relevant as per the needs of students, many topics have been re-written, a lot of
new problems of new types have been added in etc. All possible efforts are made to remove all the
printing errors that had crept in previous editions. The book is now in such a shape that the
students would feel at ease while going through the problems, which will in turn clear their
concepts too.
A Summary of changes that have been made in Revised & Enlarged Edition
— Theory has been completely updated so as to accommodate all the changes made in JEE Syllabus &
Pattern in recent years.
— The most important point about this new edition is, now the whole text matter of each chapter has
been divided into small sessions with exercise in each session. In this way the reader will be able to
go through the whole chapter in a systematic way.
— Just after completion of theory, Solved Examples of all JEE types have been given, providing the
students a complete understanding of all the formats of JEE questions & the level of difficulty of
questions generally asked in JEE.
— Along with exercises given with each session, a complete cumulative exercises have been given at
the end of each chapter so as to give the students complete practice for JEE along with the
assessment of knowledge that they have gained with the study of the chapter.
— Last 10 Years questions asked in JEE Main & Adv, IIT-JEE & AIEEE have been covered in all the chapters.
However I have made the best efforts and put my all teaching experience in revising this book.
Still I am looking forward to get the valuable suggestions and criticism from my own fraternity
i.e. the fraternity of JEE teachers.
I would also like to motivate the students to send their suggestions or the changes that they
want to be incorporated in this book. All the suggestions given by you all will be kept in prime
focus at the time of next revision of the book.

Amit M. Agarwal
CONTENTS

1. VECTOR ALGEBRA 1-62


LEARNING PART — Section Formula
Session 1 Session 3
— Scalar and Vector Quantities — Linear Combination of Vectors
— Representation of Vectors — Theorem on Coplanar &
— Position Vector of a Point in Space Non-coplanar Vectors
— Direction Cosines — Linear Independence and
— Rectangular Resolution of a Vector Dependence of Vectors
in 2D and 3D Systems
PRACTICE PART
Session 2
— JEE Type Examples
— Addition & Subtraction of Vectors
— Chapter Exercises
— Multiplication of Vector by Scalar

2. PRODUCT OF VECTORS 63-166


LEARNING PART — Area of Parallelogram and Triangle
Session 1 — Moment of a Force and Couple
— Product of Two Vectors — Rotation About an Axis
— Components of a Vector Along and Session 3
Perpendicular to Another Vector — Scalar Triple Product
— Application of Dot Product in
Session 4
Mechanics
— Vector Triple Product
Session 2
— Vector or Cross Product of Two
PRACTICE PART
Vectors — JEE Type Examples
— Chapter Exercises
3. THREE DIMENSIONAL
COORDINATE SYSTEM 167-282
LEARNING PART Session 3
Session 1 — Plane
— Introduction — Equation of Plane in Various Form
— Position Vector of a Point in Space — Angles between Two Planes
— Shifting of Origin — Family of Planes
— Distance Formula — Two Sides of a Plane
— Section Formula — Distance of a Point from a Plane
— Direction Cosines and Direction — Equation of Planes Bisecting the
Ratios of a Vector Angle between Two Planes
— Projection of the Line Segment — Line and Plane
Joining Two Points on a Given Line
Session 4
Session 2
— Sphere
— Equation of a Straight Line in Space
— Angle between Two Lines PRACTICE PART
— Perpendicular Distance of a Point — JEE Type Examples
from a Line — Chapter Exercises
— Shortest Distance between Two
Lines
SYLLABUS

JEE MAIN
Three Dimensional Geometry
Coordinates of a point in space, distance between two points, section formula,
direction ratios and direction cosines, angle between two intersecting lines. Skew
lines, the shortest distance between them and its equation. Equations of a line
and a plane in different forms, intersection of a line and a plane, coplanar lines.

Vector Algebra
Vectors and scalars, addition of vectors, components of a vector in two
dimensions and three dimensional space, scalar and vector products, scalar and
vector triple product.

JEE ADVANCED
Locus Problems
Three Dimensions Direction cosines and direction ratios, equation of a straight
line in space, equation of a plane, distance of a point from a plane.

Vectors
Addition of vectors, scalar multiplication, scalar products, dot and cross products,
scalar triple products and their geometrical interpretations.
CHAPTER

01
Vector Algebra
Learning Part
Session 1
● Scalar and Vector Quantities

● Representation of Vectors

● Position Vector of a Point in Space

● Direction Cosines

● Rectangular Resolution of a Vector in 2D and 3D Systems

Session 2
● Addition & Subtraction of Vectors

● Multiplication of Vector by Scalar

● Section Formula

Session 3
● Linear Combination of Vectors

● Theorem on Coplanar & Non-coplanar Vectors

● Linear Independence and Dependence of Vectors

Practice Part
● JEE Type Examples
● Chapter Exercises

Arihant on Your Mobile !


Exercises with the #L
symbol can be practised on your mobile. See inside cover page to activate for free.
Session 1
Scalar and Vector Quantities, Representation of Vectors,
Position Vector of a Point in Space, Direction Cosines,
Rectangular Resolution of a Vector in 2D and 3D Systems
Vectors represent one of the most important mathematical A directed line segment with initial point A and terminal
systems, which is used to handle certain types of problems →
point B is denoted by AB or AB . Vectors are also denoted
in Geometry, Mechanics and other branches of Applied
by small letters with an arrow above it or by small bold
Mathematics, Physics and Engineering. → → →
letters, e.g. a, b, c etc. or a , b , c etc.
B
Scalar and Vector Quantities a
Physical quantities are divided into two categories-Scalar
quantities and Vector quantities. Those quantities which A
have only magnitude and which are not related to any Here, in the figure a = AB and magnitude or modulus of
fixed direction in space are called scalar quantities or a is expressed as | a| = | AB| = AB (Distance between initial
briefly scalars. Examples of scalars are mass, volume, and terminal points).
density, work, temperature etc.
A scalar quantity is represented by a real number along Remarks
with a suitable unit. Second kind of quantities are those 1. The magnitude of a vector is always a non-negative real
number.
which have both magnitude and direction, such quantities
2. Every vector AB has the following three characteristics
are called vectors. Displacement, velocity, acceleration,
momentum, weight, force etc., are examples of vector Length The length of AB will be denoted by | AB | or AB.
quantities.
Support The line of unlimited length of which AB is a
y Example 1. Classify the following measures as segment is called the support of the vector AB.
scalars and vectors Sense The sense of AB is from A to B and that of BA is
(i) 20 m north-west (ii) 10 Newton from B to A. Thus, the sense of a directed line segment is
(iii) 30 km/h from its initial point to the terminal point.
(iv) 50m/s towards north
y Example 2. Represent graphically
(v) 10 −19 coloumb
(i) A displacement of 60 km, 40° east of north
Sol. (i) Directed distance -Vector
(ii) Force-Vector (ii) A displacement of 50 km south-east
(iii) Speed-Scalar Sol. (i) The vector OP represent the required vector.
(iv) Velocity-Vector N
(v) Electric charge-Scalar P

40° 60 km

Representation of Vectors W E
O 45°
Geometrically, a vector is represented by a directed line 50
segment. km
Q
For example, a = AB. Here, A is called the initial point and
S
B is called the terminal point or tip.
(ii) The vector OQ represent the required vector.
Chap 01 Vector Algebra 3

Types of Vectors 12. Equality of vectors Two vectors a and b are said to
be equal, if
1. Zero or null vector A vector whose magnitude is zero
(i) | a | = | b |
is called zero or null vector and it is represented by 0.
(ii) they have the same or parallel support.
The initial and terminal points of the directed line
segment representing zero vector are coincident and (iii) they have the same sense.
its direction is arbitrary. Two unit vectors may not be equal unless they have
2. Unit vector A vector whose modulus is unity, is the same direction.
called a unit vector. The unit vector in the direction of y Example 3. In the following figure, which of the
a vector a is denoted by a$ , read as ‘‘a cap’. Thus, vectors are:
| a$ | = 1.
(i) Collinear
a Vector
a$ = = (ii) Equal
| a| Magnitude of a
(iii) Co-initial
3. Like and unlike vectors Vectors are said to be like
(iv) Collinear but not equal
when they have the same sense of direction and
unlike when they have opposite directions.
4. Collinear or parallel vectors Vectors having the a c
same of parallel supports are called collinear vectors.
5. Coinitial vectors Vectors having the same initial
b
point are called coinitial vectors.
scale
6. Coplanar vectors A system of vectors is said to be unit
d
coplanar, if they lie in the same plane or their
supports are parallel to the same plane.
7. Coterminous vectors Vectors having the same Sol. (i) a,c and d are collinear vectors.
terminal points are called coterminous vectors. (ii) a and c are equal vectors
8. Negative of a vector The vector which has the same (iii) b,c and d are co-initial vectors
magnitude as the given vector a but opposite (iv) a and d are collinear but they are not equal, as their
direction, is called the negative of a and is denoted directions are not same.
by −a. Thus, if PQ = a, then QP = − a.
9. Reciprocal of a vector A vector having the same
direction as that of a given vector a but magnitude
Position Vector of
equal to the reciprocal of the given vector is known as
the reciprocal of a and is denoted by a −1 . Thus, if
a Point in Space
| a | = a, then | a −1 | = 1/a. Let O be the fixed point in space and X ′ OX , Y ′ OY and
Z ′ OZ be three lines perpendicular to each other at O.
Remark Then, these three lines called X-axis, Y-axis and Z-axis
A unit vector is self reciprocal.
which constitute the rectangular coordinate system. The
planes XOY, YOZ and ZOX, called respectively, the
10. Localised vector A vector which is drawn parallel to XY-plane, the YZ-plane and the ZX-plane.
a given vector through a specified point in space is Z
called a localised vector. For example, a force acting
k P (x, y, z)
on a rigid body is a localised vector as its effect
zk
depends on the line of action of the force. yj+
xi+ Z
11. Free vectors If the value of a vector depends only on j
O Y
its length and direction and is independent of its xi+
position in the space, it is called a free vector. x
i yj

Remark y M
Unless otherwise stated all vectors will be considered as free vectors. X
4 Textbook of Vector & 3D Geometry

Now, let P be any point in space. Then, position of P is Sol. (d) Let a = 3 $i + 4 $j +5 k
$ then a
given by triad (x , y , z ) where x , y , z are perpendicular
distance from YZ-plane, ZX-plane and XY-plane = 32 + 4 2 + 52 = 9 + 16 + 25 = 5 2
respectively. Thus, the length of a side of square = 5 2
The vector OP is called the position vector of point P Hence, area of square = (5 2 )2 = 25 × 2 = 50
with respect to the origin O and written as
OP = x$i + y$j + zk$
Direction Cosines
where $i , $j and k
$ are unit vectors parallel to X-axis, Y-axis
Let r be the position vector of a point P ( x , y , z ). Then,
and Z-axis. We usually denote position vector by r .
direction cosines of r are the cosines of angles α, β and γ
Remarks that the vector r makes with the positive direction of X , Y
and Z-axes respectively. We usually denote direction
1. If Aand B are any two points in space having coordinates
cosines by l, m and n respectively.
( x1, y1, z1 ) and ( x 2, y2, z2 ) respectively, then distance between
In the figure, we may note that ∆OAP is right angled
the points Aand B = ( x 2 − x1 ) 2 + ( y2 − y1 ) 2 + ( z2 − z1 ) 2 .
triangle and in it we have
2. Using distance formula, the magnitude of OP (or r) is given by
x
|OP| = ( x − 0 ) 2 + ( y − 0 ) 2 + ( z − 0 ) 2 = x 2 + y2 + z2 cos α = (r stands for | r |)
r
3. Two vectors are equal if they have same components. i.e. if
a = a1$i + a2$j + a3k$ and b = b1$i + b2$j + b3k$ are equal, then Similarly, from the right angled triangles OBP and OCP,
we get
a1 = b1, a2 = b2 and a3 = b3.
y z
cos β = and cos γ =
y Example 4. Find a unit vector parallel to the r r
vector −3$i + 4 $j. Thus, we have the following
x x x
Sol. Let a = − 3$i + 4 $j cos α = l = = =
x +y +z
2 2 2 | r| r
Then, | a| = ( −3)2 + ( 4 )2 = 5
y y y
∴ Unit vector parallel to a = a$ =
1
⋅a
cos β = m = = =
|a | x2 +y2 +z2 | r| r

−3$i + 4 $j −3 $ 4 $ z z z
= = i+ j and cos γ = n = = =
5 5 5 x +y +z
2 2 2 | r| r

y Example 5. Let a = 12$i + n$j and | a | = 13, find the Clearly, l 2 + m 2 + n 2 = 1.


value of n. Here, α = ∠ POX , β = ∠ POY , γ = ∠ POZ and $i, $j and k
$ are
Sol. Here, a = 12$i + n$j the unit vectors along OX , OY and OZ respectively.
⇒ | a | = 122 + n 2 = 13 Z

⇒ 144 + n 2 = 169 C
⇒ n 2 = 25 or n = ± 5
P(x, y, z)
y Example 6. Write two vectors having same z y
r
magnitude. b y
Sol. Let a = 2 $i + $j + k
$ and b = $i + $j + 2 k
$ O
B
Y
a 2 +2
P
x r
2 y
Then, | a | = | b | = 22 + 12 + 1 2 = 6 x+
(r)
A
O a
y Example 7. If one side of a square be represented by 90°
the vectors 3$i + 4 $j + 5k$ , then the area of the square is X X
A
(a) 12 (b) 13
(c) 25 (d) 50 X
Chap 01 Vector Algebra 5

Remarks The scalars x and y are called the components or resolved


1. The coordinates of point P may also be expressed as parts of r in the directions of X-axis and Y-axis,
( lr , mr , nr ). respectively and the ordered pair ( x , y ) is known as
2. The numbers lr , mr and nr , proportional to the direction coordinates of point whose position vector is r.
cosines, are called the direction ratios of vector r and are Y
denoted by a, b and c respectively.
3. If r = a $i + b$j + c k$ , then a, b and c are direction ratios of the P(x,y)
given vector.
Also, if a2 + b2 + c 2 = 1, then a, b and c will be direction
j
cosines of given vector. r

y Example 8. The direction cosines of the vector


3$i − 4 $j +5k$ are X
O
3 −4 1 3 −4 1 i
(a) , , (b) ,,
5 5 5 5 2 5 2 2 Also, the magnitude of r = x 2 + y 2 and if θ is the
3 −4 1 3 4 1 y 
(c) , , (d) , , inclination of r with the X-axis, then θ = tan −1   .
2 2 2 5 2 5 2 2 x
Sol. (b) r = 3$i − 4 $j + 5k$
⇒ | r| = 3 2 + ( −4 ) 2 + 5 2 = 5 2 In three Dimensional System
3 −4 5 Any vector r in three dimensional system can be
Hence, direction cosines are , ,
5 2 5 2 5 2 expressed as
3 −4 1
i.e. , , . r = x$i + y$j + zk
$
5 2 5 2 2
Y
y Example 9. Show that the vector $i + $j + k$ is
z)
equally inclined to the axes OX , OY and OZ . x ,y,
P(
Sol. Let a = $i + $j + k$ j r

If a makes angles α, β, γ with X , Y and Z -axes respectively, O X


then i
k
1 1
cosα = =
1 +1 +1
2 2 2 3 Z

1 The vectors x$i, y$j and zk


$ are called the right angled
cosβ =
3 components of r.
1 The scalars x , y and z are called the components or
and cos γ =
3 resolved parts of r in the directions of X-axis, Y-axis and
Thus, we have cos α = cos β = cos γ , i.e. α = β = γ Z-axis, respectively and ordered triplet ( x , y , z ) is known
Hence, a is equally inclined to the axes. as coordinates of P whose position vector is r. Also, the
magnitude or modulus of
r = | r| = x 2 + y 2 + z 2 .
Rectangular Resolution of a
y Example 10. Let AB be a vector in two
Vector in 2D and 3D Systems dimensional plane with the magnitude 4 units and
making an angle of 30° with X-axis and lying in the
In Two Dimensional System first quadrant.
Any vector r in two dimensional system can be expressed Find the components of AB along the two axes of
as r = x$i + y$j . The vectors x$i and y$j are called the coordinates. Hence, represent AB in terms of unit
perpendicular component vectors of r. vectors $i and $j.
6 Textbook of Vector & 3D Geometry

Sol. Let us consider A as origin. From the diagram, it can be = AB cos 30° = 4 cos 30°
seen that the component of AB along X-axis 3
Y = 4× =2 3
2
B
D and the component of AB along Y -axis
1
j 4 = AB sin 30° = 4 × = 2
2
30°
X Hence, AB = 2 3$i + 2$j
A i

Exercise for Session 1


1. Classify the following measures as scalars and vector :
(i) 20 kg weight (ii) 45°
(iii) 10 m south-east (iv) 50 m/sec 2

2. Represent the following graphically:


(i) A displacement of 40 km, 30° west of south (ii) a displacement of 70 km, 40° north of west

3. In the given figure, ABCDEF is a regular hexagon, which vectors are:

E d D

x c

F z C
y b

A a B

(i) Collinear (ii) Equal


(iii) Coinitial (iv) Collinear but not equal

4. Answer the following as true or false


(i) a and a are collinear.
(ii) Two collinear vectors are always equal in magnitude.
(iii) Zero vector is unique.
(iv) Two vectors having same magnitude are collinear.

5. Find the perimeter of a triangle with sides 3$i + 4$j + 5k$ , 4$i − 3$j −5k$ and 7$i + $j .

6. Find the angle of vector a = 6$i + 2$j − 3k$ with X-axis.

7. Write the direction ratios of the vector r = $i − $j + 2k$ and hence calculate its direction cosines.
Session 2
Addition & Subtraction of Vectors, Multiplication
of Vector by Scalar, Section Formula

Addition of Vectors 3. Polygon law of addition


(Resultant of Vectors) If the number of vectors are represented by the sides of a
polygon taken in order, the resultant is represented by the
1. Triangle Law of Addition closing side of the polygon taken in the reverse order.
If two vectors are represented by two consecutive sides of E D
a triangle, then their sum is represented by the third side
of the triangle, but in opposite direction. This is known as
the triangle law of addition of vectors. Thus, if AB = a, F C
BC = b and AC = c, then AB + BC = AC i.e. a + b = c.
C
A B

c =a+b
In the figure, AB + BC + CD + DE + EF = AF
b

a
4. Addition in Component Form
A B If the vectors are defined in terms of $i, $j and k, $ i.e. if
a = a 1 $i + a 2 $j + a 3 k
$ and b = b 1 $i + b 2 $j + b 3 k
$ , then their
2. Parallelogram Law of Addition sum is defined as
If two vectors are represented by two adjacent sides of a a + b = (a 1 + b 1 ) $i+ (a 2 + b 2 ) $j + (a 3 + b 3 ) k
$.
parallelogram, then their sum is represented by the
diagonal of the parallelogram whose initial point is the
same as the initial point of the given vectors. This is
Properties of Vector Addition
known as parallelogram law of vector addition. Vector addition has the following properties
Thus, if OA = a, OB = b and OC = c (i) Closure The sum of two vectors is always a vector.
Then, OA + OB = OC i.e. a + b = c, where OC is a diagonal (ii) Commutativity For any two vectors a and b,
of the parallelogram OACB. ⇒ a+ b= b+ a
B C (iii) Associativity For any three vectors a, b and c,
⇒ a + ( b + c ) = (a + b ) + c
b c=a+b (iv) Identity Zero vector is the identity for addition. For
any vector a.
⇒ 0+ a= a= a+ 0
O a A
(v) Additive inverse For every vector a its negative
Remarks vector −a exists such that a + ( −a ) = ( −a) + a = 0
1. The magnitude of a + b is not equal to the sum of the i.e. ( −a ) is the additive inverse of the vector a.
magnitudes of a and b.
2. From the figure, we have OA + AC = OC (By triangle law of y Example 11. Find the unit vector parallel to the
vector addition) resultant vector of 2$i + 4 $j − 5 k$ and $i + 2$j + 3 k$ .
or OA + OB = OC (Q AC = OB ), which is the parallelogram
law. Thus, we may say that the two laws of vector addition are Sol. Resultant vector, r = (2$i + 4 $j − 5k$ ) + ( $i + 2$j + 3k$ )
equivalent to each other. = 3$i + 6$j − 2k$
8 Textbook of Vector & 3D Geometry

1 E D
Unit vector parallel to r = r
r
1 F C
= (3$i + 6$j − 2k$ )
32 + 62 + ( − 2) 2
A B
1
= (3$i + 6$j − 2k)
$
= AB + O + O + AB + 2AB = 4 AB
7
(Q ED = AB, FC = 2AB )
y Example 12. If a, b and c are the vectors Hence proved.
represented by the sides of a triangle, taken in
order, then prove that a + b + c = 0.
Sol. Let ABC be a triangle such that
Subtraction of Vectors
BC = a , CA = b and AB = c If a and b are two vectors, then their subtraction a − b is
A defined as a − b = a + ( −b ), where −b is the negative of b
having magnitude equal to that of b and direction
c b opposite to b.
B

B a C b
a+b
Then, a + b + c = BC + CA + AB
= BA + AB (Q BC + CA = BA ) O
a
A
= − AB + AB
a+ b+ c = 0 Hence proved.

y Example 13. If S is the mid-point of side QR of a –b


a+(–b) = a–b
∆PQR, then prove that PQ + PR = 2 PS.
Sol. Clearly, by triangle law of addition, we have C
P
If a = a 1 $i + a 2 $j + a 3 k
$
and b = b 1 $i + b 2 $j + b 3 k
$
Then, a − b = (a 1 − b 1 ) $i + (a 2 − b 2 ) $j + (a 3 − b 3 ) k
$

Q S R

PQ + QS = PS …(i) Properties of Vector Subtraction


and PR + RS = PS … (ii) (i) a − b ≠ b − a
On adding Eqs. (i) and (ii), we get (ii) (a − b ) − c ≠ a − ( b − c )
( PQ + QS ) + ( PR + RS ) = 2PS (iii) Since, any one side of a triangle is less than the sum
⇒ ( PQ + PR ) + ( QS + RS ) = 2PS and greater than the difference of the other two sides,
so for any two vectors a and b, we have
⇒ PQ + PR + 0 = 2PS
(a) | a + b | ≤ | a| + | b | (b) | a + b | ≥ | a| − | b |
[QS is the mid-point of QR∴ QS = − RS]
Hence, PQ + PR = 2PS Hence proved. (c) | a − b | ≤ | a| + | b | (d) | a − b | ≥ | a| − | b |

y Example 14. If ABCDEF is a regular hexagon, prove Remark


that AD + EB + FC = 4AB. If Aand B are two points in space having coordinates ( x1, y1, z1 )
and ( x 2, y2, z2 ), then
Sol. We have, AB = Position Vector of B − Position Vector of A
AD + EB + FC = ( AB + BC + CD ) = ( x 2i$ + y2$j + z2k$ ) − ( x1$i + y1$j + z1 k$ )
+ ( ED + DC + CB ) + FC = ( x − x ) i$ + ( y − y ) $j + ( z − z ) k$
2 1 2 1 2 1
= AB + ( BC + CB ) + ( CD + DC) + ED + FC
Chap 01 Vector Algebra 9

y Example 15. If A = (0, 1), B = (1, 0), C = (1, 2) and A (a i + b j + g k)

D = (2, 1), prove that vector AB and CD are equal.


Sol. Here, AB = (1 − 0) $i + (0 − 1) $j = $i − $j
and CD = (2 − 1)$i + (1 − 2) $j = $i − $j B C
(bi + g j + ak) (g i + a j + bk)
Clearly, AB = CD Hence proved.

y Example 16. If the position vectors of A and B Now, | AB | = (β − α )2 + ( γ − β )2 + (α − γ )2


respectively $i + 3$j − 7k$ and 5$i − 2$j + 4k$ , then find AB. Similarly, BC = CA = (β − α )2 + ( γ − β )2 + (α − γ )2
Sol. Let O be the origin, then we have ∴ ∆ABC is an equilateral triangle.
OA = i$ + 3$j − 7 k$ y Example 20. If the position vectors of the vertices
and OB = 5$i − 2$j + 4 k$ of a triangle be 2$i + 4 $j − k$ , 4 $i + 5$j + k$ and
Now, AB = OB − OA = (5$i − 2$j + 4 k$ ) − ( $i + 3$j − 7 k$ ) 3$i +6 $j − 3k$ , then the triangle is
= 4 $i − 5$j + 11k$ (a) right angled (b) isosceles
(c) equilateral (d) None of these
y Example 17. Vectors drawn from the origin O to
Sol. (a, b) Let A , B, C be the vertices of given triangle with
the points A, B and C are respectively a,b and
4 a − 3b. Find AC and BC. position vectors, 2$i + 4 $j − k$ , 4 $i + 5$j + k$ and 3$i + 6$j − 3k$
respectively.
Sol. We have, OA = a, OB = b and OC = 4a − 3 b
Then, we have
Clearly, AC = OC − OA = ( 4a − 3b ) − (a )
OA = 2$i + 4 $j − k$ , OB = 4 $i + 5$j + k$
= 3a − 3b
and BC = OC − OB = ( 4a − 3b ) − ( b ) = 4a − 4 b and OC = 3$i + 6$j − 3k$

y Example 18. Find the direction cosines of the Clearly, AB = OB − OA = 2$i + $j + 2k$
vector joining the points A(1, 2, − 3) and B( −1, − 2, 1), BC = − i$ + $j − 4 k$
directed from A to B. and AC = $i + 2$j − 2k$
Sol. Clearly, Now, AB = | AB | = 22 + 12 + 22 = 3
AB = ( −1 − 1)$i + ( −2 − 2)$j + (1 + 3)k$ = − 2$i − 4 $j + 4 k$
BC = | BC| = ( −1)2 + (1)2 + ( −4 )2 = 3 2
Now, | AB| = ( −2) + ( −4 ) + ( 4 ) = 36 = 6
2 2 2
and AC = | AC| = 12 + 22 + ( −2)2 = 3
AB −2$i − 4 $j + 4 k$ AB = AC and BC 2 = AB 2 + AC 2
∴ Unit vector along AB = = Q
| AB| 6
∴ The triangle is isosceles and right angled.
1 2 2
= − $i − $j + k$
3 3 3 y Example 21. The two adjacent sides of a
parallelogram are 2$i + 4 $j − 5k$ and $i + 2$j +3k$ . Find the
y Example 19. Let α , β and γ be distinct real
unit vectors along the diagonals of the parallelogram.
numbers. The points with position vectors
Sol. Let OABC be the given parallelogram and let the adjacent
α i$ + β $j + γ k$ , β $i + γ $j + α k$ and γ $i + α $j + β k$ sides OA and OB be represented by a = 2$i + 4 $j − 5k$ and
(a) are collinear b = $i + 2$j + 3k$ respectively.
(b) form an equilateral triangle Now, the vectors along the two diagonals are
(c) form a scalene triangle B a C
(d) form a right angled triangle a+b
b–
Sol. (b) Let the given points be A , B and C with position vectors a
b
α$i + β$j + γk$ , β$i + γ$j + αk$ and γ$i + α$j + βk$ . b

As, α, β and γ are distinct real numbers, therefore ABC


form a triangle. O a A

Clearly, AB = OB − OA = (β$i + γ$j + αk$ ) − (α$i + β$j + γk$ ) d1 = a + b = 3$i + 6$j − 2k$
= (β − α )$i + ( γ − β )$j + (α − γ )k$
10 Textbook of Vector & 3D Geometry

The required unit vectors are It is given that, | a$ | = | b$ | = | a$ + b$ | = 1


d 3i$ + 6$j − 2k$ ⇒ | OA | + | AB| = | OB | = 1
n$ 1 = 1 =
| d1| 32 + 62 + ( − 2) 2 ∆OAB is equilateral triangle.
3$ 6$ 2 $ Since, | OA | =| a$ | = 1 = | − b$ | = AB ′
= i+ j− k
7 7 7 Therefore, ∆OAB ′ is an isosceles triangle.
d − $i − 2$j + 8k$ ⇒ ∠AB ′ O = ∠AOB ′ = 30°
and n$ 2 = 2 =
|d 2 | ( − 1) 2 + ( − 2) 2 + 8 2 ⇒ ∠ BOB ′ = ∠ BOA + ∠ AOB ′ = 60° + 30° = 90°
−1 $ 2 $ 8 $ (since, ∆BOB ′ is right angled)
= i− j+ k ∴ In ∆BOB ′ , we have
69 69 69
| BB ′ | 2 = | OB | 2 + | OB ′ | 2
y Example 22. If a and b are any two vectors, then 2 2
give the geometrical interpretation of the relation = a$ + b$ + a$ − b$
| a + b| = | a − b|. 22 = 12 + | a$ − b$ | 2
Sol. Let OA = a and AB = b. Completing the parallelogram | a$ − b$ | = 3 Hence proved.
OABC.
a
C B

b
a+b
a–b b
Multiplication of a
Vector by a Scalar
O A If a is a vector and m is a scalar (i.e. a real number), then
a
m a is a vector whose magnitude is m times that of a and
Then, OC = b and CB = a
whose direction is the same as that of a, if m is positive
From ∆OAB, we have and opposite to that of a, if m is negative.
OA + AB = OB ⇒ a + b = OB …(i)
∴ Magnitude of ma = |ma| ⇒ m (magnitude of a) =m | a |
From ∆OCA, we have
OC + CA = OA Again, if a = a 1 $i + a 2 $j + a 3 k,
$
⇒ b + CA = a ⇒ CA = a − b …(ii) then m a = (ma 1 ) $i + (ma 2 ) $j + (ma 3 ) k
$
Clearly, | a + b| = | a − b| ⇒ | OB| = | CA |
Diagonals of parallelogram OABC are equal.
OABC is a rectangle.
Properties of Multiplication
⇒ OA ⊥ OC ⇒ a ⊥ b of Vectors by a Scalar
y Example 23. If the sum of two unit vectors is a The following are properties of multiplication of vectors
unit vector, prove that the magnitude of their by scalars, for vectors a, b and scalars m, n
difference is 3. (i) m ( −a ) = ( −m ) a = − (ma )
Sol. Let a$ and b$ be two unit vectors represented by sides OA and (ii) ( −m ) ( −a ) = m a
AB of a ∆OAB. (iii) m (na ) = (mn ) a = n (ma )
B
(iv) (m + n ) a = ma + na
(v) m ( a + b) = m a + m b
a+b b
y Example 24. If a is a non-zero vector of modulus a
a 60° and m, is a non-zero scalar, then ma is a unit vector, if
O A
30°
120° (a) m = ± 1 (b) m = |a|
30° 1
–b (c) m = (d) m = ± 2
a–b B' |a|
$ AB = b$
Then, OA = a, Sol. (c) Since, ma is a unit vector, | ma | = 1
OB = OA + AB = a$ + b$ ⇒ |m | | a | = 1
(using triangle law of vector addition) 1 1
⇒ |m | = ⇒ m=±
| a| |a |
Chap 01 Vector Algebra 11

y Example 25. For a non-zero vector a, the set of Proof


real numbers, satisfying |( 5 − x ) a | < | 2a | consists of Let O be the origin. Then OA = a and OB = b. Let r be the
all x such that position vector of P which divides AB internally is the
(a) 0 < x < 3 ratio m : n . Then
(b) 3 < x < 7 AP m
=
(c) −7 < x < − 3 PB n
(d) −7 < x < 3 or nAP = mPB
Sol. (b) We have, |(5 − x ) a | < | 2a | or n(PV of P − PV of A) =m (PV of B − PV of P)
|5 − x | | a| < 2| a | or n( r − a ) = m( b − r)
⇒ |5 − x | < 2
or nr − na = mb − mr
⇒ −2 < 5 − x < 2
or r(n + m ) = mb + na
⇒ 3< x < 7
mb + na
or r=
y Example 26. Find a vector of magnitude (5/2) m +n
units which is parallel to the vector 3$i + 4 $j. mb + na
or OP =
Sol. Here, a = 3i$ + 4 $j m +n
Then, | a| = 32 + 4 2 = 5 External Division
∴ A unit vector parallel to
If P divides AB externally in the ratio m : n. Then, the
a 1
a = a$ = ⋅ = (3$i + 4 $j) …(i) position vector of P is given by
|a | 5
m
Hence, the required vector of magnitude (5/2) units and n
A P
parallel to a B
5 5 1
= ⋅ a$ = ⋅ (3$i + 4 $j) a b
2 2 5
1 $
= (3i + 4 $j)
2
O

mb − na
Section Formula OP =
m −n
Let A and B be two points with position vectors a and b Proof
respectively. Let P be a point on AB dividing it is the ratio Let O be the origin. Then OA = a, OB = b. Let r be the
m : n. position vector of point P dividing AB externally in the
ratio m : n .
Internal Division Then,
AP m
=
If P divides AB internally in the ratio m : n. Then the BP n
position vector of P is given by or nAP = mBP
m n or n AP = m BP
A P
B
or n (PV of P − PV of A) =m (PV of P − PV of B)
or n( r − a ) = m ( r − b )
a b
or nr − na = mr −mb
or r (m − n ) = mb − na
O mb − na
or r=
mb + na m −n
OP = mb − na
m +n or OP =
m −n
12 Textbook of Vector & 3D Geometry

Remarks y Example 28. Find the position vectors of the


1. Position vector of mid-point of AB is
a +b
. points which divide the join of points A (2a − 3b )
2 and B ( 3a − 2b ) internally and externally in the ratio
2. In ∆ ABC, having vertices A( a ), B( b) and C( c ) 2 : 3.
A (a)
Sol. Let P be a point which divide AB internally in the ratio 2 : 3.
Then, by section formula, position vector of P is given by
2(3a − 2b ) + 3(2a − 3b )
OP =
2+3
6a − 4 b + 6a − 9 b 12 13
= = a− b
5 5 5
B (b) C (c)
Similarly, the position vector of the point ( P ′ ) which
a + b+ c divides AB externally in the ratio 2 : 3 is given by
(i) Position vector of centroid is .
3 2(3a − 2b ) − 3(2a − 3b )
(ii) Position vector of incentre is
OP ′ =
2−3
BCa + ACb + ABc 6a − 4 b − 6a + 9 b 5b
.
AB + BC + AC = = = − 5b
−1 −1
(iii) Position vector of orthocentre is
tan A a + tan Bb + tan Cc y Example 29. The position vectors of the vertices
.
tan A + tan B + tan C A, B and C of a triangle are $i − $j − 3 k$ , 2$i + $j − 2 k$ and
(iv) Position vector of circumcentre is −5$i + 2$j − 6 k$ , respectively. The length of the bisector
sin 2 A a + sin 2Bb + sin 2Cc
. AD of the ∠BAC, where D is on the segment BC, is
sin 2 A + sin 2B + sin 2C
3 1
(a) 10 (b)
y Example 27. If D, E and F are the mid-points of the 4 4
11
sides BC , CA and AB respectively of the ∆ABC and O (c) (d) None of these
2
be any point, then prove that
OA + OB + OC = OD + OE + OF Sol. (b) A (i - j -3k)
Sol. Since, D is the mid-point of BC, therefore by section
formula, we have
A

B C
F E (2i + j - 2k) (-5i + 2j - 6k)

O (a) | AB| = |(2$i + $j − 2k$ ) − ( $i − $j − 3k$ )|


= | i$ + 2$j + k$ |
B D C
= 12 + 22 + 1 2 = 6
OB + OC
OD = | AC | = |( −5$i + 2$j − 6k$ ) − ( $i − $j − 3k$ )|
2
= | − 6$i + 3$j − 3k$ |
⇒ OB + OC = 2OD …(i)
Similarly, OC + OA = 2OE …(ii) = ( −6)2 + 32 + ( −3)2 = 54 = 3 6
and OB + OA = 2OF …(iii) 6 1
BD : DC = AB : AC = =
On adding Eqs. (i), (ii) and (iii), we get 3 6 3
2( OA + OB + OC) = 2( OD + OE + OF ) 1 ( −5i$ + 2$j − 6k$ ) + 3 (2$i + $j − 2k$ )
∴ Position vector of D =
⇒ OA + OB + OC = OD + OE + OF 1+3
Hence proved. 1 $
= ( i + 5$j − 12k$ )
4
Chap 01 Vector Algebra 13

∴ AD = Position vector of D −Position vector of A Magnitude of Resultant of Two Vectors


1 1
AD = ( $i + 5$j − 12k$ ) − ( $i − $j − 3k$ ) = ( −3$i + 9 $j) Let R be the resultant of two vectors P and Q. Then,
4 4
3 $ R= P+ Q
= ( − i + 3$j)
4 | R| = R = P 2 + Q 2 + 2 PQ cos θ
3 3
| AD | = ( − 1) 2 + 32 = 10 Q sin θ
4 4 where, | P | = P ,| Q | = Q , tan α =
P + Q cos θ
y Example 30. The median AD of the ∆ ABC is
Q
bisected at E .BE meets AC in F. Then, AF : AC is equal R
to
θ
(a) 3/4 (b) 1/3 α θ
(c) 1/2 (d) 1/4 P
Sol. (b) Let position vector of A w.r.t. B is a and that of C w.r.t. B Deduction When | P| = | Q|, i.e. P = Q
is c.
P sin θ
A (a) tan α =
P + P cos θ
l sin θ θ
F = = tan
m
1
1
1 + cos θ 2
E
θ
B (0 ) D C(c) ∴ α=
(C/2) 2
Position vector of D w.r.t. Hence, the angular bisector of two unit vectors a and b is
0+ c c along the vector sum a + b.
B= =
2 2
Remarks
Position vector of
1. The internal bisector of the angle between any two vectors is
c
a+ along the vector sum of the corresponding unit vectors.
2 a c
E= = + …(i) 2. The external bisector of the angle between two vectors is along
2 2 4 the vector difference of the corresponding unit vectors.
Let AF : FC = λ : 1 and BE : EF = µ : 1 B(b)
λc + a µ(a – b) µ(a + b)
Position vector of F =
1+ λ
External Internal
Now, position vector of bisector
bisector
 λc + a  A(a)
µ  + 1 ⋅0 O
 1+ λ 
E= …(ii)
µ +1 y Example 31. The sum of two forces is 18 N and
From Eqs. (i) and (ii), we get resultant whose direction is at right angles to the
a c µ λµ smaller force is 12 N. The magnitude of the two
+ = a+ c
2 4 (1 + λ ) (1 + µ ) (1 + λ ) (1 + µ ) forces are
1 µ (a) 13, 5 (b) 12, 6
⇒ =
2 (1 + λ )(1 + µ ) (c) 14, 4 (d) 11, 7
1 λµ Sol. (a) We have, | P| + | Q| = 18N; | R| = P + Q + 12 N
and =
4 (1 + λ ) (1 + µ ) α = 90°
1 P + Q cosθ = 0
⇒ λ=
2 ⇒ Q cosθ = − P
1 Now. R 2 = P 2 + Q 2 + 2PQ cosθ
AF AF λ 1
∴ = = =2 = ⇒ R 2 = P 2 + Q 2 + 2P ( − P ) = Q 2 − P 2
AC AF + FC 1 + λ 3 3
2
14 Textbook of Vector & 3D Geometry

π
d 2 = | 4a | 2 + | 5b | 2 + 2 | 4 a | | 5b | cos
Q 4
1
R
= 16 × 8 + 25 × 9 + 40 × 2 2 × 3 ×
2
= 593
∴ Length of the longer diagonal = 593
α θ

P y Example 33. The vector c, directed along the


⇒ 12 = ( P + Q ) (Q − P ) = 18 (Q − P )
2 internal bisector of the angle between the vectors
⇒ Q − P = 8 and Q + P = 18 a = 7 $i − 4 $j − 4 k$ and b = −2$i − $j + 2k$ with | c | = 5 6, is
⇒ Q = 13, P = 5 5 $ $ 5 $ $
(a) (i − 7j + 2 k$ ) (b) (5 i + 5 j + 2 k$ )
∴ Magnitude of two forces are 5 N and 13 N. 3 3
5 5
y Example 32. The length of longer diagonal of the (c) ($i + 7$j + 2 k$ ) (d) (−5 $i + 5 $j + 2 k$ )
3 3
parallelogram constructed on 5 a + 2 b and a − 3 b,
when it is given that | a| = 2 2, | b | = 3 and angle Sol. (a) Let a = 7i$ − 4j$ − 4k$
π
between a and b is , is and b = − 2i$ − $j + 2k$
4
a b
(a) 15 (b) 113 Now, required vector c = λ  + 
 | a | | b| 
(c) 593 (d) 369
 7 $i − 4 $j − 4 k$ −2$i − $j + 2k$ 
Sol. (c) Length of the two diagonals will be = λ  + 
d1 = |(5a + 2b ) + (a − 3b )|  9 3 
and d 2 = |(5a + 2 b) − ( a − 3 b)| λ $
= ( i − 7 $j + 2k$ )
⇒ d1 = | 6a − b |, d 2 = | 4 a + 5 b | 9
Thus, λ2
| c| 2 = × 54 = 150
d1 = | 6a | 2 + | − b | 2 + 2 | 6a | | − b | cos( π − π / 4 ) 81
⇒ λ = ± 15
 1
= 36(2 2 ) + 9 + 12 ⋅ 2 2 ⋅ 3 ⋅  −  = 15
2

5
c = ± ( i$ − 7 $j + 2k$ )
 2 3
Chap 01 Vector Algebra 15

Exercise for Session 2


1. If a = 2$i − $j + 2k$ and b = − $i + $j − k$ , then find a + b. Also, find a unit vector along a + b.

2. Find a unit vector in the direction of the resultant of the vectors $i + 2$j + 3k$ , − $i + 2$j + k$ and 3$i + $j.

3. Find the direction cosines of the resultant of the vectors ( $i + $j + k$ ), ( − $i + $j + k$ ), ( $i − $j + k$ ) and ( $i + $j − k$ ).

4. In a regular hexagon ABCDEF, show that AE is equal to AC + AF − AB

5. Prove that 3OD + DA + DB + DC is equal to OA + OB − OC.

6. In a regular hexagon ABCDEF, prove that AB + AC + AD + AE + AF = 3AD.

7. ABCDE is a pentagon, prove that AB + BC + CD + DE + EA = 0.

8. The position vectors of A, B, C, D are a, b, 2a + 3b and a − 2b, respectively. Show that DB = 3b − a and
AC = a + 3b.
9. If P(−1, 2) and Q (3, − 7) are two points, express the vector PQ in terms of unit vectors $i and $j. Also, find
distance between point P and Q. What is the unit vector in the direction of PQ ?
10. If OP = 2$i + 3$j − k$ and OQ = 3$i − 4$j + 2k$ , find the modulus and direction cosines of PQ .

11. Show that the points A, B and C with position vectors a = 3$j − 4$j − 4k$ , b = 2$i − $j + k$ and c = $i − 3$j − 5k$
respectively, form the vertices of a right angled triangle.

12. If a = 2$i + 2$j − k$ and | xa | = 1, then find x.

13. If p = 7$i − 2$j + 3k$ and q = 3$i + $j + 5k$ , then find the magnitude of p − 2q.

14. Find a vector in the direction of 5$i − $j + 2k$ , which has magnitude 8 units.

15. If a = $i + 2$j + 2k$ and b = 3$i + 6$j + 2k$ , then find a vector in the direction of a and having magnitude as | b | .

16. Find the position vector of a point P which divides the line joining two points A and B whose position vectors are
$i + 2$j − k$ and − $i + $j + k$ respectively, in the ratio 2 : 1.
(i) internally (ii) externally

17. If the position vector of one end of the line segment AB be 2$i + 3$j − k$ and the position vector of its middle point
be 3 ( $i + $j + k$ ), then find the position vector of the other end
Session 3
Linear Combination of Vectors, Theorem on Coplanar
& Non-coplanar Vectors, Linear Independence
and Dependence of Vectors

Linear Combination of Vectors Now,


y
is a scalar, because x and y are scalars.
A vector r is said to be a linear combination of vectors a, b x
and c… etc., if there exist scalars x , y and z etc., such that Hence, Eq. (ii) expresses a as product of b by a scalar, so
r = xa + yb + zc + … that a and b are collinear. Thus, we arrive at a
For examples Vectors r1 = 2 a + b + 3 c and contradiction because a and b are given to be
r2 = a + 3 b + 2 c are linear combinations of the vectors non-collinear.
a, b and c . Thus our supposition that x ≠ 0, is wrong.
Hence, x = 0. Similarly, y = 0
Collinearity and Coplanarity of Remarks
Vectors a = 0, b = 0
 or
1. xa + yb = 0 ⇒ x = 0, y = 0
Relation between Two Collinear Vectors  or
 a || b
(or Parallel Vectors)
2. If a and b are two non-collinear (or non-parallel) vectors, then
Let a and b be two collinear vectors and let x$ be the unit x1 a + y1 b = x 2 a + y2 b
vector in the direction of a. Then, the unit vector in the ⇒ x1 = x 2 and y1 = y2
direction of b is x$ or −x$ according as a and b are like or Proof x1 a + y1 b = x 2 b + y2 b
unlike parallel vectors. Now, a = | a| x$ and b = ± | b| x$ . ⇒ ( x1 − x 2 )a + ( y1 − y2 )b = 0
 |a |   |a |  ⇒ x1 − x 2 = 0 and y1 − y2 = 0
∴ a =   | b | x$ ⇒ a = ±   b [Qa and b are non-collinear]
 | b|  | b |
⇒ x1 = x 2 and y1 = y2
|a | If a = a1 $i + a2 $j + a3 k$ and b = b1 $i + b2 $j + b3 k,
$ then a || b
⇒ a = λb, where λ = ± a1 a2 a3
| b| ⇒ = =
b1 b2 b3
Thus, if a and b are collinear vectors, then a = λb or
b = λa for some scalar λ i.e, there exist two non-zero Test of Collinearity of Three Points
scalar quantities x and y so that xa + yb = O
(i) Three points A, B and C are collinear, if AB = λBC
An Important Theorem (ii) Three points with position vectors a, b and c are
Theorem : Vectors a and b are two non-zero, collinear iff there exist scalars x , y and z not all zero
non-collinear vectors and x , y are two scalars such that such that xa + yb + zc = 0, where x + y + z = 0
xa + yb = 0 Proof Let us suppose that points A, B and C are
collinear and their position vectors are a, b and c
Then, x = 0, y = 0 respectively. Let C divide the join of a and b in the
Proof It is given that xa + yb = 0 …(i) ratio y : x . Then,
Suppose that x ≠ 0, then dividing both sides of (i) by the xa + yb
c=
scalar x, we get x +y
y xa + yb − ( x + y )c = 0
a=− b …(ii) or
x
or xa + yb + zc = 0 , where z = − ( x + y )
Chap 01 Vector Algebra 17

Also, x + y + z = x + y − (x + y ) = 0 ∴ A , B and C are collinear.


Conversely, let xa + yb + zc = 0, where x + y + z = 0. Now, let C divide AB in the ratio k : 1, then
kOB + 1 ⋅ OA
Therefore, OC =
k +1
xa + yb = − zc = ( x + y )c (Q x + y = − z )
k (3i + 4 $j + 7 k$ ) + ( $i + 2$j + 3k$ )
$
xa + yb ⇒ −3$i − 2$j − 5k$ =
or c= k +1
x +y
 3k + 1 $  4k + 2 $  7k + 3 $
⇒ −3$i − 2$j − 5k$ =  i +   j+  k
This relation shows that c divides the join of a and b  k +1  k +1   k +1 
in the ratio y : x . Hence, the three points A, B and C 3k + 1 4k + 2 7k + 3
are collinear. ⇒ = − 3; = −2 and = −5
k +1 k +1 k +1
(iii) If a = a 1 $i + a 2 $j, b = b 1 $i + b 2 $j and c = c 1 $i + c 2 $j, then −2
From, all relations, we get k =
the points with position vector a, b and c will be 3
a1 a2 1 Hence, C divides AB externally in the ratio 2 : 3.
collinear iff b 1 b 2 1 = 0. y Example 36. If the position vectors of A, B, C and D
c1 c2 1 are 2$i + $j, $i − 3$j, 3$i + 2$j and $i + λ$j, respectively and
Proof The points with position vector a, b and c will AB || CD, then λ will be
be collinear iff there exist scalars x , y and z not all (a) −8 (b) −6
zero such that, (c) 8 (d) 6
x (a 1 $i + a 2 $j) + y (b 1 $i + b 2 $j) + z (c 1 $i + c 2 $j) = 0 and
Sol. (b) AB = ( $i − 3$j) − (2$i + $j) = − $i − 4 $j ;
x +y +z =0
CD = ( i$ + λ$j) − (3$i + 2$j) = − 2$i + ( λ − 2)$j ;
⇒ xa 1 + yb 1 + zc 1 = 0
AB ||CD ⇒ AB = x CD
xa 2 + yb 2 + zc 2 = 0
− $i − 4 $j = x { −2$i + ( λ − 2)$j}
x +y +z =0
⇒ −1 = − 2x, −4 = ( λ − 2)x
Thus, the points will be collinear iff the above system 1
of equation’s have non-trivial solution ⇒ x = and λ = − 6
2
Hence, the points will be collinear
a1 b1 c 1 a1 a2 1 y Example 37. The points with position vectors
iff a 2 b 2 c 2 = 0 or b 1 b 2 1 = 0. 60$i + 3$j, 40$i − 8 $j and a $i − 52$j are collinear, if a is
1 1 1 c1 c2 1
equal to
(a) −40 (b) 40
y Example 34. Show that the vectors 2$i − 3$j + 4k$ and (c) 20 (d) None of these
−4 $i + 6 $j − 8k$ are collinear. Sol (a) The three points are collinear if
60 3 1
Sol. Let a = 2$i − 3$j + 4 k$ and b = −4 $i + 6$j − 8k$
40 −8 1 = 0
Consider, b = −4 $i + 6$j − 8k$ = −2(2$i − 3$j + 4 k$ ) = − 2a a −52 1
∴ The vectors a and b are collinear.
⇒ 60 ( −8 + 52) − 3( 40 − a ) + ( −2080 + 8a ) = 0
y Example 35. Show that the points A(1, 2, 3), B( 3, 4, 7 ) ⇒ 2640 − 120 + 3a − 2080 + 8a = 0
11a = −440
and C( −3, − 2, − 5) are collinear. Find the ratio in
⇒ a = − 40
which point C divides AB.
Sol. Clearly, AB = (3 − 1)$i + ( 4 − 2)$j + (7 − 3)k$ y Example 38. Let a, b and c be three non-zero vectors
= 2$i + 2$j + 4 k$ such that no two of these are collinear. If the vector
a + 2b is collinear with c and b + 3c is collinear with a
and BC = ( −3 − 3)$i + ( −2 − 4 )$j + ( −5 − 7 )k$
(λ being some non-zero scalar), then a + 2b + 6c is
= 6$i − 6$j − 12k$ equal to
= − 3(2$i + 2$j + 4 k$ ) = −3AB (a) 0 (b) λb
Q BC = − 3AB (c) λc (d) λa
18 Textbook of Vector & 3D Geometry

Sol. (a) As a + 2b and c are collinear a + 2b = λc …(i)


Again, b + 3c is collinear with a.
Test of Coplanarity of Three Vectors
∴ b + 3c = µa …(ii) (i) Three vectors a, b, c are coplanar iff any one of them
Now, a + 2b + 6c = (a + 2b ) + 6c = λc + 6c
is a linear combination of the remaining two, i.e. iff
a = xb + yc where x and y are scalars.
= ( λ + 6) c …(iii)
Also, a + 2b + 6c = a + 2( b + 3c) = a + 2µa (ii) If three points with position vectors
= (2µ + 1)a …(iv)
$ , b = b 1 $i + b 2 $j + b 3 k
a = a 1 $i + a 2 $j + a 3 k $
From Eqs. (iii) and (iv), we get and c = c 1 $i + c 2 $j + c 3 k$ are coplanar,
( λ + 6)c = (2µ + 1)a
a1 a2 a3
But a and c are non-zero, non-collinear vectors,
∴ λ + 6 = 0 = 2µ + 1 then b 1 b2 b 3 = 0.
Hence, a + 2b + 6c = 0 c1 c2 c3
If vectors a,b and c are coplanar, then there exist
scalars x and y such that c = xa + yb.
Theorem of Coplanar Vectors Hence, c 1 $i + c 2 $j + c 3 k
$ = x (a 1 $i + a 2 $j + a 3 k
$)
Let a and b be two non-zero, non-collinear vectors. Then
+y (b 1 i + b 2 $j + b 3 k
$ $)
any vector r coplanar with a and b can be uniquely
expressed as a linear combination xa + yb; x and y being Now, $i, $j and k $ are non-coplanar and hence
scalars. independent.
Proof Let a and b be any two non-zero, non-collinear Then, c 1 = xa 1 + yb 1 , c 2 = xa 2 + yb 2
vectors and r be any vector coplanar with a and b. and c 3 = xa 3 + yb 3
We take any point O in the plane of a and b
The above system of equations in terms of x and y is
B
consistent. Thus,
b
P
a1 b1 c 1 a1 a2 a 3
N
r a 2 b 2 c 2 = 0 or b 1 b 2 b 3 = 0.
a3 b3 c3 c1 c2 c 3
O A
M a
Let OA = a, OB = b and OP = r Remark
Clearly, OA, OB and OP are coplanar. If vectors x1 a + y1 b + z1 c, x 2 a + y2 b + z2 c and x 3 a + y3 b + z3 c
are coplanar(where a, b and c are non-coplanar).
Through P, we draw lines PM and PN , parallel to OB and x1 y1 z1
OA respeetively meeting OA and OB at M and N Then, x 2 y2 z2 = 0
respectively. x 3 y3 z 3
We have, OP = OM + MP
= OM + ON [Q MP = ON and MP || ON ] …(i)
Test of Coplanarity of Four Points
Now, OM and OA are collinear vectors
(i) To prove that four points A(a ), B( b ), C (c ) and D(d )
OM = x OA = xa, where x is scalar. are coplanar, it is just sufficient to prove that vectors
Similarly, ON = yOB = yb, where y is a scalar. AB, AC and AD and coplanar.
Hence, from Eq. (i), OP = xa + yb or r = x ′a + y ′ b (ii) Four points with position vectors a, b , c and d are
Uniqueness: If possible, let r = xa + yb and r = x ′ a + y ′ b coplanar iff there exist scalars x , y , z and u not all
be two different ways of representing r. zero such that x a + y b + z c + u d = 0, where
x + y + z + u = 0.
Then, we have xa + yb = x ′ a + y ′ b
(iii) Four points with position vectors
⇒ ( x − x ′ )a + (y − y ′ ) b = 0
a = a 1 $i + a 2 $j + a 3 k
$,
But a and b are non-collinear vectors
∴ x − x ′ = 0 and y − y ′ = 0 b = b 1 $i + b 2 $j + b 3 k
$

⇒ x ′ = x and y ′ = y c = c 1 $i + c 2 $j + c 3 k
$
Thus, the uniqueness in established. and d = d 1 $i + d 2 $j + d 3 k$
Chap 01 Vector Algebra 19

a1 a2 a3 1 Here, the three lines OA, OB, OC are not coplanar. Hence,
b1 b2 b3 1 they determine three different planes BOC , COA and AOB
will be coplanar, iff =0 when taken in pairs.
c1 c2 c3 1
Through P, draw planes parallel to these planes BOC, COA
d1 d2 d3 1
and AOB meeting OA, OB and OC in L, E and N
d 1 − a1 d 2 − a2 d 3 −a3 respectively. Thus we obtain a parallelopiped with OP as
or b1 − a1 b2 − a2 b3 − a3 = 0 diagonal and three coterminous edges OL, OE and ON
along OA, OB and OC, respectively.
c 1 − a1 c 2 − a2 c3 −a3
B
b
E S
Theorem on Non-coplanar M P

Vectors r
L
Theorem 1 O
a
A
N
If a, b, c, are three non-zero, non-coplanar vectors and c R
x , y , z are three scalars such that C

xa + yb + zc = 0. ∴OL is collinear with OA.


Then x = y = z = 0. ∴ OL = xOA = xa, where x is a scalar.
Proof It is given that xa + yb + zc = 0 …(i) Similarly, OE = y b and ON = zc,
Suppose that x ≠ 0 where y and z are scalars.
Then Eq. (i) can be written as Now, OP = OR + RP = (ON + NR ) + RP
xa = −yb − zc = ON + OL + OE [Q NR = OL and RP = OE]

y
a=− b− c
z
…(ii) = OL + OE + ON = xa + yb + zc
x x Thus, r = xa + yb + zc
y z Hence, r can be expressed as a linear combination of a, b
Now, and are scalars because x , y and z are scalars.
x x and c.
Thus, Eq. (ii) expresses a as a linear combination of b and c.
Uniqueness If possible let
Hence, a is coplanar with b and c which is contrary to our
r = xa + yb + zc
hypothesis because a,b and c are given to be non-coplanar.
and r = x ′a + y ′ b + z′c
Thus, our supposition that x ≠ 0 is wrong.
be two different ways of representing r, then we have
Hence, x = 0
xa + yb + zc = x ′ a + y ′ b + z ′ c
Similarly, we can prove that y = 0 and z = 0
⇒ ( x − x ′ )a + (y − y ′ ) b + (z − z ′ )c = 0
Theorem 2 Now a, b and c are non-coplanar vectors
If a,b and c are non-coplanar vectors, then any vector r ∴ x − x ′ = 0, y − y ′ = 0 and z − z ′ = 0
can be uniquely expressed as a linear combination ⇒ x = x ′, y = y ′ and z =z′
xa + yb + zc; x , y and z being scalars.
Hence, the uniqueness is established.
or
Any vector in space can be expressed as a linear Remark
combination of three non-coplanar vectors. If a , b, c are any three non-coplanar vectors in space, then
x1 a + y1 b + z1 c = x 2 a + y2 b + z2 c
Proof Take any point O.
⇒ x1 = x 2, y1 = y2, z1 = z2
Let a, b, c be any three non-coplanar vectors and r be any Proof x1 a + y1 b + z1 c = x 2 a + y2 b + z2 c
vector in space. ⇒ ( x1 − x 2 )a + ( y1 − y2 )b + ( z1 − z2 )c = 0
Let OA = a, OB = b, ⇒ x1 − x 2 = 0, y1 − y2 = 0 and z1 − z2 = 0
OC = c, OP = r ⇒ x1 = x 2, y1 = y2 and z1 = z2
20 Textbook of Vector & 3D Geometry

y Example 39. Check whether the given three Since, a , b and c are non-coplanar vectors.
vectors are coplanar or non-coplanar. Therefore,
−2$i − 2$j + 4 k$ , − 2$i + 4 $j − 2 k,
$ 4 $i − 2$j − 2 k$ 3x + y = 3, −2x + y = − 7
and x + 2y = − 4
Sol. Let a = − 2$i − 2$j + 4 k$
Solving first two, we find that x = 2 and y = − 3. These
b = −2$i + 4 $j − 2k$ and c = 4 $i − 2$j − 2k$ values of x and y satisfy the third equation as well.
Now, consider So, x + 2 and y = − 3 is the unique solution for the above
system of equation.
−2 −2 4
⇒ α = 2β − 3γ
−2 4 −2 = − 2( −8 − 4 ) + 2( 4 + 8) + 4 ( 4 − 16)
Hence, the vectors α, β and γ are coplanar, because α is
4 −2 −2 uniquely written as linear combination of other two.
= 24 + 24 − 48 = 0 Trick For the vectors α, β, γ to be coplanar, we must have
∴ The vectors are coplanar. 3 −7 −4
3 −2 1 = 0, which is true
y Example 40. If the vectors 4 $i + 11$j + m k$ ,
1 1 2
7 $i + 2$j + 6k$ and $i + 5$j + 4 k$ are coplanar, then m is
Hence, α, β, γ are coplanar.
equal to
(a) 38 (b) 0 y Example 42. The value of λ for which the four
(c) 10 (d) −10 points 2$i + 3$j − k$ , $i + 2$j + 3k$ , 3$i + 4 $j − 2k$ and
Sol. (c) Since the three vectors are coplanar, one will be a linear $i − λ$j + 6k$ are coplanar
combination of the other two.
(a) 8 (b) 0
∴ 4 i$ + 11$j + mk$ = x (7 $i + 2$j + 6k$ ) + y ( $i + 5$j + 4 k$ )
(c) −2 (d) 6
⇒ 4 = 7x + y …(i)
Sol. (c) The given four points are coplanar.
11 = 2x + 5y …(ii)
2 1 3 1
m = 6x + 4y …(iii)
3 2 4 λ
From Eqs. (i) and (ii), we get =0
3 23 −1 3 −2 6
x= and y =
11 11 1 1 1 0
From Eq. (iii), we get 2 1 3 1
3 23 0 0 −( λ + 2 )
m =6× +4× = 10 ⇒
0
=0
11 11 −1 3 −2 6
Trick Since, vectors 4 $i + 11$j + mk$ , 7 $i + 2$j + 6k$ and 1 1 1 1
$i + 5$j + 4 k$ are coplanar.
Operating ( R 2 → R 2 − R1 − R 4 )
4 11 m 2 1 3
∴ 7 2 6 =0 ⇒ −( λ + 2 ) − 1 3 −2 = 0 ⇒ λ = − 2
1 5 4 1 1 1
⇒ 4(8 − 30) − 11(28 − 6) + m(35 − 2) = 0
⇒ −88 − 11 × 22 + 33m = 0 y Example 43. Show that the points P( a + 2b + c ),
⇒ − 8 − 22 + 3m = 0 Q( a − b − c ), R(3a + b + 2c ) and S( 5a + 3b + 5c ) are
⇒ 3m = 30 ⇒ m = 10 coplanar given that a,b and c are non-coplanar.
y Example 41. If a, b and c are non-coplanar Sol. To show that P , Q , R, S are coplanar, we will show that
vectors, prove that 3a − 7b − 4c , 3a − 2b + c and PQ, PR, PS are coplanar.
a + b + 2c are coplanar. PQ = − 3b − 2c
PR = 2a − b + c
Sol. Let α = 3a − 7 b − 4 c, β = 3a − 2b + c
PS = 4a + b + 4 c
and γ = a + b + 2c Let PQ = xPR + yPS
Also, let α = xβ + y − γ ⇒ −3b − 2c = x (2a − b + c) + y ( 4a + b + 4 c)
⇒ 3a − 7 b − 4 c = x (3a − 2b + c) + y (a + b + 2c) ⇒ −3b − 2c = (2x + 4y )a + ( − x + y )b + ( x + 4y )c
= (3x + y )a + ( −2x + y )b + ( x + 2y )c
Chap 01 Vector Algebra 21

As the vectors a, b, c are non-coplanar, we can equate their (vii) Three vectors a = a 1 $i + a 2 $j + a 3 k $ , b = b 1 $i + b 2 $j + b 3 k
$
coefficients.
and c = c 1 $i + c 2 $j + c 3 k
$ will be linearly dependent
⇒ 0 = 2x + 4y
a1 a2 a 3
⇒ −3 = − x + y
vectors iff b 1 b 2 b 3 = 0.
⇒ − 2 = x + 4y
x = 2, y = − 1 is the unique solution for the above system of c1 c2 c 3
equations. (viii) Any four vectors in 3-dimensional space are linearly
⇒ PQ = 2PR − PS dependent.
PQ ,PR, PS are coplanar because PQ is a linear combination
of PR and PS y Example 44. Show that the vectors
$i − 3$j + 2k$ , 2$i − 4 $j − k$ and 3$i + 2$j − k$ and linearly
⇒ The points P, Q, R, S are also coplanar.
Trick For the vectors PQ, PR and PS to be coplanar, we independent.
0 −3 −2
Sol. Let α = i$ − 3$j + 2k$
must have 2 −1 1 = 0 which is true
β = 2$i − 4 $j − k$
4 1 4
and γ = 3$i + 2$j − k$
∴ The PQ, PR, PS are coplanar.
Hence, the points P , Q , R, S are also coplanar. Also, let xα + yβ + zγ = 0
∴ x ( $i − 3$j + 2k$ ) + y (2$i − 4 $j − k$ ) + z (3$i + 2$j − k$ ) = 0
or ( x + 2y + 3z )$i + ( −3x − 4y + 2z )$j + (2x − y − z )k$ = 0
Linear Independence and Equating the coefficient of $i, $j and k,$ we get
Dependence of Vectors x + 2y + 3z = 0
−3x − 4y + 2z = 0
1. Linearly Independent Vectors 2x − y − z = 0
A set of non-zero vectors a 1 , a 2 …., a n is said to be linearly 1 2 3
independent, if Now, −3 −4 2 = 1( 4 + 2) − 2(3 − 4 ) + 3(3 + 8) = 41 ≠ 0
x 1a 1 + x 2a 2 + … + x n a n = 0 2 −1 −1
⇒ x 1 = x 2 = ... = x n = 0. ∴ The above system of equations have only trivial solution.
Thus, x = y = z = 0
2. Linearly Dependence Vectors Hence, the vectors α,β and γ are linearly independent.
A set of vector a 1 , a 1 ...., a n is said to be linearly Trick Consider the determinant of coefficients of $i, $j and k$
dependent, if there exist scalars x 1 , x 2 ,..., x n not all zero 1 −3 2
such that x 1 a 1 + x 2 a 2 + ....+ x n a n = 0 i.e. 2 −4 −1 = 1( 4 + 2) + 3( −2 + 3) + 2( 4 + 12)
3 2 −1
Properties of Linearly Independent
and Dependent Vectors = 6 + 3 + 32 = 41 ≠ 0
∴ The given vectors are non-coplanar. Hence, the vectors
(i) A super set of a linearly dependent set of vectors is are linearly independent.
linearly dependent.
(ii) A subset of a linearly independent set of vectors is y Example 45. If a = $i + $j + k$ , b = 4 $i + 3$j + 4k$ and
linearly independent. c = $i + α$j + βk$ are linearly dependent vectors and
(iii) Two non-zero, non-collinear vectors are linearly | c | = 3, then
independent. (a) α = 1, β = − 1 (b) α = 1,β = ± 1
(iv) Any two collinear vectors are linearly dependent. (c) α = − 1, β = ± 1 (d) α = ± 1, β = 1
(v) Any three non-coplanar vectors are linearly Sol. (d) The given vectors are linearly dependent, hence there
independent. exist scalars x , y and z not all zero, such that
(vi) Any three coplanar vectors are linearly dependent. xa + yb + zc = 0
i.e. x ( i + j + k ) + y ( 4 i + 3 j + 4 k ) + z ( $i + α$j + βk$ ) = 0
$ $ $ $ $ $
22 Textbook of Vector & 3D Geometry

i.e. ( x + 4y + z )i$ + ( x + 3y + αz )$j + ( x + 4y + βz )k$ = 0 Trick | c | = 1 + α2 + β2 = 3


⇒ x + 4y + z = 0, x + 3y + αz = 0, x + 4y + βz = 0 ⇒ α2 + β2 = 2
1 4 1
1 1 1
For non-trivial solution 1 3 α = 0 ⇒ β = 1
Q a, b and c are linearly dependent, hence 4 3 4 = 0
1 4 β
4 α β
| c |2 = 3 ⇒ 1 + α 2 + β 2 = 3 ⇒ β =1
⇒ α2 = 2 − β2 = 2 − 1 = 1 ∴ α2 = 1 ⇒ α = ±1
∴ α = ±1

Exercise for Session 3


1. Show that the points A (1, 3, 2), B (−2, 0, 1) and C(4, 6, 3) are collinear.

2. If the position vectors of the points A, B and C be a, b and 3a − 2b respectively, then prove that the points A, B
and C are collinear.

3. The position vectors of four points P, Q, R and S are 2a + 4c, 5a + 3 3b + 4c, − 2 3b + c and 2a + c
respectively, prove that PQ is parallel to RS.
4. If three points A, B and C have position vectors (1, x , 3), (3, 4, 7) and ( y , − 2, − 5), respectively and if they are
collinear, then find (x , y ).

5. Find the condition that the three points whose position vectors, a = a i$ + b$j + ck,
$ b = i$ + c$j and c = − i$ − $j are
collinear.

6. Vectors a and b are non-collinear. Find for what values of x vectors c = ( x − 2)a + b and d = (2x + 1)a − b are
collinear?
7. Let a, b, c are three vectors of which every pair is non-collinear. If the vectors a + b and b + c are collinear with c
and a respectively, then find a + b + c.
8. Show that the vectors i$ − $j − k$ , 2$i + 3$j + k$ and 7i$ + 3$j − 4k$ are coplanar.

9. If the vectors 2$i − $j +  k$ , $i + 2$j − 3k$ and 3$i + a$j + 5k$ are coplanar, then prove that a = 4.

10. Show that the vectors a − 2b + 3c, − 2a + 3b − 4c and − b + 2c are coplanar vector, where a, b,c are non-coplanar
vectors.

11. If a, b and c are non-coplanar vectors, then prove that the four points 2a + 3b − c, a − 2b + 3c, 3a + 4b − 2c and
a − 6b + 6c are coplanar.
JEE Type Solved Examples :
Single Option Correct Type Questions
l Ex. 1 The non-zero vectors a, b and c are related by ⇒ P 2 + 3 PQ − 2 PQ − 6Q 2 = 0
a = 8b and c = −7b angle between a and c is ⇒ P ( P + 3Q ) − 2Q( P + 3Q ) = 0
π π ⇒ ( P − 2Q )( P + 3Q ) = 0
(a) (b)
4 2 ⇒ P − 2Q = 0 or P + 3Q = 0
(c) π (d) 0 P
From P − 2Q = 0 ⇒ =2
Sol. (c) a and b vectors are in the same direction, b and c are in Q
the opposite direction.
⇒ a and c are in opposite directions.
l Ex. 4 A vector a has the components 2p and 1 w.r.t. a
∴ Angle between a and c is π. rectangular cartesian system. This system is rotated through
a certain angle about the origin in the counter clockwise
π sense. If with respect to a new system, a has components
l Ex. 2 A unit vector a makes an angle with Z-axis. If
4 ( p +1) and 1, then
a + $i + $j is a unit vector, then a is equal to (a) p = 0 (b) p = 1 or p = −
1
3
i$ j$ k$ i$ j$ k$ 1
(a) + + (b) + − (c) p = − 1 or p = (d) p = 1 or p = − 1
2 2 2 2 2 2 3
$i j$ k$ Sol. (b) We have, a = 2pi$ + $j
(c) − − + (d) None of these
2 2 2 On rotation, let b be the vector with components ( p + 1 ) and
1 so that,
Sol. (c) Let a = l $i + m$j + nk$ , where l 2 + m 2 + n 2 = 1. a makes an
π b = ( p + 1 ) $i + $j
angle with Z-axis.
4 Now, | a| = | b | ⇒ a 2 = b 2
1 2 1 ⇒ 4p 2 + 1 = (p + 1)2 + 1 ⇒ 4p 2 = (p + 1)2
∴ n= , l + m2 = …(i)
2 2
⇒ 2 p = ± ( p + 1 ) ⇒ 3 p = − 1 or p = 1
k$
∴ a = l $i + m$j + ∴
1
p = − or p = 1
2 3
k$
a + $i + $j = (l + 1 ) $i + (m + 1 ) $j +
2
l Ex. 5 ABC is an isosceles triangle right angled at A.
1 Forces of magnitude 2 2, 5 and 6 act along BC, CA and AB
Its magnitude is 1, hence (l + 1 ) 2 + (m + 1 ) 2 = …(ii)
2 respectively. The magnitude of their resultant force is
From Eqs. (i) and (ii), we get (a) 4 (b) 5
1 1 (c) 11+ 2 2 (d) 30
2lm = ⇒ l = m = −
2 2 Sol. (b) R cosθ = 6 cos 0 ° + 2 2 cos(180 ° − B ) + 5 cos270 °
$i $j $k
Hence, a=− − + R cosθ = 6 − 2 2 cos B …(i)
2 2 2
R sin θ = 6 sin 0 ° + 2 2 sin(180 ° − B ) + 5 sin 270 °
l Ex. 3 If the resultant of two forces of magnitudes P and Q C
acting at a point at an angle of 60° is 7Q, then P / Q is
5 2√2
3
(a) 1 (b)
2
(c) 2 (d) 4 A B
6
Sol. (c) R 2 = P 2 + Q 2 + 2 PQ cos θ
R sin θ = 2 2 sin B − 5 …(ii)
⇒ ( 7Q ) 2 = P 2 + Q 2 + 2 PQ cos 60 °
From Eqs. (i) and (ii), we get
⇒ 7Q 2 = P 2 + Q 2 + PQ
R 2 = 36 + 8 cos2 B − 24 2 cos B + 8 sin 2 B + 25 − 20 2 sin B
⇒ P + PQ − 6Q = 0
2 2
= 61 + 8 (cos2 B + sin 2 B ) − 24 2 cos B − 20 2 sin B
24 Textbook of Vector & 3D Geometry

Q ABC is a right angled isosceles triangle. CA = (2 − 1 ) $i + (1 − 4 ) $j + ( −1 + 3 ) k$


i.e. ∠B = ∠C = 45 ° = $i − 3 $j − 2 k$
1 1
∴ R 2 = 61 + 8(1 ) − 24 2 − 20 2 ⋅ = 25 | AB | = 1 + 9 + 4 = 14
2 2
| BC | = 4 + 36 + 16 = 56 = 2 14
∴ R =5
| CA | = 1 + 9 + 4 = 14
l Ex. 6 A line segment has length 63 and direction ratios So, | AB| + | AC | = | BC | and angle between AB and BC is 180°.
are 3, − 2 and 6. The components of line vector are So, points A, B and C cannot form an isosceles triangle.
(a) −27, 18, 54 (b) 27, − 18, 54 Hence, A, B and C are collinear.
(c) 27, − 18, − 54 (d) −27, − 18, − 54
Sol. (b) Let the components of line segment on axes are x, y l Ex. 9 The position vector of a point C with respect to B is
and z. $i + $j and that of B with respect to A is $i − $j. The position
So, x 2 + y 2 + z 2 = 63 2 vector of C with respect to A is
Now,
x
=
y z
= =k (a) 2$i (b) 2$j
3 −2 6
(c) − 2$j (d) − 2$i
Q (3k ) 2 + ( −2k ) 2 + (6k ) 2 = 63 2
Sol. (a) Since, position vectors of a point C with respect to B is
63
k=±
= ±9 BC = $i + $j …(i)
7
Similarly, AB = i − j
$ $ …(ii)
∴Components are (27, − 18, 54 ) or ( −27, 18, − 54).
Now, by Eqs. (i) and (ii),
l Ex. 7 If the vectors 6 $i − 2 $j + 3k$ , 2 $i + 3 $j − 6k$ and AC = AB + BC = 2 $i

3 $i + 6 $j − 2k$ form a triangle, then it is l Ex. 10 In a ∆ABC, if 2 AC = 3CB, then 2OA + 3OB is
(a) right angled (b) obtuse angled equal to
(c) equilateral (d) isosceles (a) 5OC (b) −OC
Sol. (b) AB = Position vectors of B Position vector of A (c) OC (d) None of these
= (2 i$ + 3 $j − 6 k$ ) − (6 $i − 2 $j + 3 k$ ) = − 4 $i + 5 $j − 9 k$ Sol. (a) 2 OA + 3 OB = 2( OC + CA ) + 3( OC + CB)
⇒ | AB | = 16 + 25 + 81 = 122 = 5 OC + 2 CA + 3 CB = 5 OC (Q 2 CA = −3 CB)
BC = $i + 3 $j + 4 k$
l Ex. 11 If a, b, c and d be the position vectors of the
⇒ | BC | = 1 + 9 + 16 = 26 and AC = − 3 $i + 8 $j − 5 k$ points A, B, C and D respectively, referred to same origin O
⇒ | AC | = 98 such that no three of these points are collinear and
Therefore, AB 2 = 122, BC 2 = 26 and AC 2 = 98 a + c = b + d , then quadrilateral ABCD is a
(a) square (b) rhombus
⇒ AB 2 + BC 2 = 26 + 122 = 148
(c) rectangle (d) parallelogram
Since, AC 2 < AB 2 + BC 2, therefore ∆ABC is an obtuse angled Sol. (d) Given, a+c=b+d
triangle. 1 1
⇒ (a + c ) = ( b + d )
2 2
l Ex. 8 The position vectors of the points A, B and C are
Here, mid-points of AC and BD coincide, where AC and BD
( 2i$ + $j − k$ ), (3 $i − 2 $j + k$ ) and ( $i + 4 $j − 3k$ ) respectively. These are diagonals. In addition, we know that, diagonals of a
points. parallelogram bisect each other.
(a) form an isosceles triangle Hence, quadrilateral is parallelogram.
(b) form a right angled triangle
(c) are collinear
l Ex. 12 P is a point on the side BC of the ∆ABC and Q is
(d) form a scalene triangle a point such that PQ is the resultant of AP, PB and PC.
Sol. (c) AB = (3 − 2 ) $i + ( −2 − 1 ) $j + (1 + 1 ) k$ Then, ABQC is a
= i$ − 3 $j + 2 k$ (a) square
(b) rectangle
BC = (1 − 3 ) $i + ( 4 + 2 ) $j + ( −3 − 1 ) k$
(c) parallelogram
= − 2 $i + 6 $j − 4 k$ (d) trapezium
Chap 01 Vector Algebra 25

Sol. (c) AP + PB + PC = PQ or AP + PB = PQ + CP A

⇒ AB = CQ
A C

B C B
P P
∴ PA + PB = 2PC
Q
l Ex. 16 If O be the circumcentre and O ′ be the
Hence, it is a parallelogram.
orthocentre of the ∆ABC, then O′ A + O′ B + O′ C is equal to
(a) OO′ (b) 2O ′ O (c) 2OO′ (d) 0
l Ex. 13 If ABCD is a parallelogram and the position
Sol. (b) O′ A = O′ O + OA
vectors of A, B and C are $i + 3 $j + 5k$ , $i + $j + k$ and 7 $i + 7 $j + 7k$ ,
O′ B = OO′ + OB
then the position vector of D will be
O′ C = O′ O + OC
(a) 7 $i + 5$j + 3k$ (b) 7 $i + 9$j + 11k$ ⇒ O′ A + O′ B + O′ C = 3 O′ O + OA + OB + OC
(c) 9$i + 11$j + 13k$ (d) 8 $i + 8 $j + 8k$ A
Sol. (b) Let position vector of D is xi$ + y$j + zk$ , then AB = DC.
⇒ −2 $j − 4 k$ = (7 − x ) $i + (7 − y ) $j + (7 − z ) k$
O′
⇒ x = 7, y = 9 and z = 11
Hence, position vector of D will be 7 $i + 9 $j + 11 k$ . O
B C
l Ex. 14 P is the point of intersection of the diagonals of
Since, OA + OB + OC = OO′ = − O′ O
the parallelogram ABCD. If O is any point, then
∴ O′ A + O′ B + O′ C = 2O′ O
OA + OB + OC + OD is equal to
(a) OP (b) 2OP l Ex. 17 Five points given by A, B, C, D and E are in a
(c) 3OP (d) 4OP plane. Three forces AC, AD and AE act at A and three forces
Sol. (d) We know that , P will be the mid-point of AC and BD. CB, DB and EB act B. Then, their resultant is
O (a) 2AC (b) 3AB
(c) 3DB (d) 2BC
D C Sol. (b) Points A, B, C , D and E are in a plane.
Resultant = ( AC + AD + AE ) + CB + BD + EB )
= ( AC + CB ) + ( AD + DB ) + ( AE + EB )
P
= AB + AB + AB = 3AB
A B
l Ex. 18 If the vectors represented by the sides AB and BC
∴ OA + OC = 2 OP …(i)
of the regular hexagon ABCDEF be a and b, then the vector
and OB + OD = 2OP …(ii) represented by AE will be
On adding Eqs. (i) and (ii), we get (a) 2b − a (b) b − a
OA + OB + OC + OD = 4 OP (c) 2a − b (d) a + b
Sol. (a) As in figure, AB = a, BC = b,
l Ex. 15 If C is the middle point of AB and P is any point
So, AD = 2b and ED = a
outside AB, then
E D
(a) PA + PB = PC
(b) PA + PB = 2PC O
(c) PA + PB + PC = 0 F C
(d) PA + PB + 2PC = 0
Sol. (b) PA + PB = ( PA + AC ) + ( PB + BC ) − ( AC + BC ) A B
= PC + PC − ( AC − CB ) = 2PC − 0 Now, AE + ED = AD
(Q AC = CB)
⇒ AE = AD − ED = 2 b − a
26 Textbook of Vector & 3D Geometry

l Ex. 19 If a + b + c = 0 and | a| = 3, | b | = 5, | c | = 7, then the Therefore, | CD | = 3 | OB |


angle between a and b is ⇒ CD = 3 b
2
π π ⇒ OD = OC + CD = a + 3 b
(a) (b) 3
2 3
2
π π Hence, AD = OD − OA = a + 3 b − a
(c) (d) 2
4 6
1
Sol. (b) A = 3b − a
3

7
l Ex. 22 If position vectors of a point A is a + 2b and a
5
divides AB in the ratio 2 : 3, then the position vector of B is
π−θ θ (a) 2a − b (b) b − 2a
B
3 C (c) a − 3b (d) b
Sol. (c) If x be the position vector of B, then a divides AB in the
Let θ be the angle between a and b. Then, ∠C = π − θ. ratio 2 : 3.
32 + 52 − 72 2 x + 3(a + 2 b )
∴ cos( π − θ ) = ∴ a=
2(3 )(5 ) 2+3
−1 ⇒ 5a − 3a − 6 b = 2 x
∴ − cosθ =
2 ⇒ x = a − 3b
π
∴ θ = 60 ° =
3 lEx. 23 If D , E and F are respectively, the mid-points of
AB, AC and BC in ∆ABC, then BE + AF is equal to
l Ex. 20 If a and b are the position vectors of A and B
1
respectively, then the position vector of a point C on AB (a) DC (b) BF
2
produced such that AC = 3 AB is
3
(a) 3a − b (b) 3b − a (c) 2BF (d) BF
2
(c) 3 a − 2b (d) 3b − 2a
Sol. (a) BE + AF = OE − OB + OF − OA
Sol. (d) Since, given that AC = 3 AB. It means that point C divides
AB externally. C
Thus, AC : BC = 3 : 2
A E F

a B
A B
D
b OA + OC OB + OC
C = − OB + − OA
O 2 2
3 ⋅ b − 2 ⋅a OA + OB
Hence, OC = = 3 b − 2a = OC − = OC − OD = DC
3 −2 2

l Ex. 21 Let A and B be points with position vectors a and


l Ex. 24 In a quadrilateral PQRS, PQ = a , QR = b,
b with respect to the origin O. If the point C on OA is such SP = a − b. If M is the mid-point of QR and X is a point of
that 2AC = CO, CD is parallel to OB and CD =3 OB , then 4
SM such that, SX = SM, then
AD is equal to 5
a a 1
(a) 3b − (b) 3b + (a) PX = PR
2 2 5
a a 3
(c) 3b − (d) 3b + (b) PX = PR
3 3 5
Sol. (c) Since, OA = a, OB = b and 2AC = CO 2
(c) PX = PR
2 5
By section formula, OC = a
3 (d) None of the above
Chap 01 Vector Algebra 27

Sol. (b) If we take point P as the origin, the position vectors of Q Sol. (c) Let G be centroid and I be incenter.
and S are a and b − a respectively. 2a + 3 b + 4 c a + b + c
| GI| = | OI − OG | = −
In ∆PQR, we have R 9 3
S −a + c 3 1
= = =
9 9 3
X M
®
b l Ex. 27 Let position vector of points A, B and C of trian-
gle ∆ABC respectively be $i + $j + 2k$ , $i + 2 $j + k$ and
®
Q
2 $i + $j + k$ . Let l1 , l 2 and l3 be the lengths of perpendiculars
P a
drawn from the orthocenter ‘O’ on the sides AB, BC and CA,
PR = PQ + QR ⇒ PR = a + b then (l1 + l 2 + l3 ) equals
∴ Position vector of R = a + b 2 3
(a) (b)
a + (a + b )  1  6 6
⇒ PV of M = = a + b
2  2  6 6
(c) (d)
4 2 3
Now, SX = SM
5 Sol. (c) A (1, 1, 2)
4 1
⇒ XM = SM − SX = SM − SM = SM
5 5
∴ SX : XM = 4 : 1 Ö2 Ö2
 1 
4 a + b + 1( b − a )
 2 
⇒ PV of X =
4+1 B C
Ö2
3a + 2 b 3 (1, 2, 1) (2, 1, 1)
= ⇒ PX = (a + b )
5 5
Clearly, triangle formed by the given points $i + $j + 2 k$ ,
3
⇒ PX = PR i$ + 2 $j + k$ and 2 i$ + $j + k$ is equilateral as AB = BC = AC = 2.
5
∴ Distance of orthcentre ‘O’ from the sides is equal to inradius
l Ex. 25 Orthocentre of an equilateral triangle ABC is the of the triangle.
origin O. If OA = a, OB = b, OC = c, then AB + 2BC + 3CA 3
( 2 )2
∆ 4 1
is equal to ∴ l1 = l 2 = l 3 = inradius = r = = =
s 3 6
(a) 3c (b) 3a ( 2)
2
(c) 0 (d) 3b
3 6
Sol. (b) For an equilateral triangle, centroid is the same as ⇒ (l1 + l 2 + l 3 ) = =
orthocentre 6 2
OA + OB + OC
∴ =0 l Ex. 28 ABCDEF is a regular hexagon in the XY -plane
3
∴ OA + OB + OC = 0 with vertices in the anticlockwise direction. If AB = 2 $i, then
Now, AB + 2 BC + 3 CA CD is
= OB − OA + 2OC − 2OB + 3OA − 3OC (a) $i + 3 $j (b) $i + 2j$
= − OB + 2OA − OC (c) − $i + 3 j$ (d) None of these
= − (OB + OA + OC ) + 3OA = 3OA = 3a Sol. Y
l Ex. 26 If a, b, and c are position vector of A, B and C E D
respectively of ∆ABC and if | a − b| = 4, | b − c| = 2,
|c − a| = 3, then the distance between the centroid and
incentre of ∆ABC is F M C
1
(a) 1 (b)
2 60°
X
1 2 A 2i B
(c) (d)
3 3
28 Textbook of Vector & 3D Geometry

AB is along the X -axis and BD is along the Y -axis. l Ex. 31 In a trapezium, the vector BC = λAD. We will then
AB = 2 i$ ⇒ AB = BC = CD = ............ = 2 find that p = AC + BD is collinear with AD. If p =µ AD, then
From the figure, BM = BC sin60 ° = 2 sin 60 ° = 3 (a) µ = λ +1 (b) λ = µ +1
∴ BD = 2 3 $j (c) λ + µ =1 (d) µ = 2 + λ
BC = BC cos60 ° $i + BC sin 60 ° $j = i$ + 3 $j Sol. (a) We have, p = AC + BD = AC + BC + CD
CD = BD − BC = 2 3 $j − ( $i + 3 j) = − i$ + 3 $j = AC + λAD + CD
= λAD + ( AC + CD ) = λ AD + AD = ( λ + 1 ) AD
l Ex. 29 The vertices of triangle are A(1, 1, 2 ), B( 4, 3, 1) and Therefore, p = µAD ⇒ µ = λ + 1
C( 2, 3, 5 ). A vector representing the internal bisector of the
∠A is
l Ex. 32 If the position vectors of the points A, B and C be
$i + $j, $i − $j and a $i + b$j + ck$ respectively, then the points A, B
(a) $i + j$ + 2k$ (b) 2$i − 2$j + k$
and C are collinear, if
(c) 2$i + 2$j + k$ (d) None of these
(a) a = b = c = 1
Sol. (c) From the figure, we have (b) a = 1, b and c are arbitrary scalars
b = AC = $i + 2 $j + 3 k$ (c) a = b = c = 0
and c = AB = 3 $i + 2 $j + k$ (d) c = 0, a = 1 and b is arbitrary scalars
A (1, 1, 2) Sol. (d) Here, AB = − 2 $j, BC = (a − 1 ) $i + (b + 1 ) $j + ck$
The points are collinear, then AB = k( BC )
−2 $j = k {(a − 1 ) $i + (b + 1 ) $j + ck$ }
On comparing, k(a − 1 ) = 0, k(b + 1 ) = − 2, kc = 0
Hence, c = 0, a = 1 and b is arbitrary scalar.

B C
l Ex. 33 Let a , b and c be distinct non-negative numbers
(4, 3, 1) (2, 3, 5)
and the vectors a $i + a$j + ck$ , $i + k$ , c $i + c$j + bk$ lie in a plane,
∴ Unit vector along the bisector of ∠A is given by then the quadratic equation ax 2 + 2cx + b = 0 has
b + c ( $i + 2 $j + 3 k$ ) + (3 $i + 2 $j − k$ )
= = (a) real and equal roots
2 14
(b) real and unequal roots
2 i$ + 2 $j + k$ (c) unreal roots
=
14 (d) both roots real and positive
∴ Any vector along the angle bisector of Sol. (a) a$i + a$j + ck$ , $i + k$ and c$i + c$j + bk$ are coplanar
∠A = 2 $i + 2 $j + k$ a a c
∴ 1 0 1 = 0 ⇒ c 2 − ab = 0
l Ex. 30 Let a = (1, 1, − 1), b = (5, − 3, − 3 ) and c = (3, − 1, 2 ). If c c b
| a + b|
r is collinear with c and has length , then r equals For, equation ax 2 + 2cx + b = 0
2
3 D = 4c 2 − 4ab = 0
(a) ±3c (b) ± c
2 So, roots are real and equal.
2
(c) ±c (d) ± c
3
l Ex. 34 The number of distinct real values of λ for which
Sol. (c) Let r = λc the vectors λ3 $i + k$ , $i − λ3 $j and $i + ( 2λ − sin λ ) $j − λk$ are
Given | r| = | λ || c| coplanar is
| a + b| (a) 0 (b) 1
∴ = | λ| | c |
2 (c) 2 (d) 3
∴ |6 $i − 2 $j − 4 k$| = 2| λ ||3 $i − $j + 2 k$ | Sol. (a) Put ∆ = 0 ⇒ λ7 + λ3 + 2 λ − sin λ = 0
∴ 56 = 2 | λ | 14 Let f ( λ ) = λ7 + λ3 + 2 λ − sin λ
∴ λ = ±1 ⇒ f ′( λ ) = (7 λ6 + 3 λ2 + 2 − cos λ ) > 0, ∀ ∈ R
∴ r=±c ∴ f ( λ ) = 0 has only one real solution λ = 0.
Chap 01 Vector Algebra 29

Ex. 35 The points A( 2 − x , 2, 2 ), B( 2, 2 − y ,2 ), 7 1


l
On solving these, we get x = 0, y = − , z =
5 5
C ( 2, 2, 2 − z ) and D (1, 1, 1) are coplanar, then locus of
( −7 q + r )
P ( x , y , z ) is ∴ −2a + 3 b − c =
5
1 1 1
(a) + + =1 (b) x + y + z = 1 Trick Check alternates one-by-one
x y z
i.e. (a) p − 4q = − 2 a + 5b − 4 c
1 1 1
(c) + + = 1 (d) None of these −7q + r
1− x 1− y 1− z (b) = − 2 a + 3b − c
5
Sol. (a) Here, AB = x $i − y $j
AC = xi$ − zk$ ; AD = ( x − 1 ) $i − $j − k$
l Ex. 37 If a 1 and a 2 are two values of a for which the unit
1
As, these vectors are coplanar vector a $i + b$j + k$ is linearly dependent with $i + 2 $j and
x −y 0 2
1 1 1 1 1
⇒ x 0 −z = 0 ⇒ + + = 1 $j − 2k$ , then + is equal to
x y z a1 a 2
x − 1 −1 −1
1 −16 −11
(a) 1 (b) (c) (d)
l Ex. 36 p = 2 a − 3b, q = a − 2b + c and r = − 3 a + b + 2c, 8 11 16
1$
where a, b, c being non-zero non-coplanar vectors, then the Sol. (c) ai$ + b$j + k = l ( $i + 2 $j) + m( $j − 2 k$ )
2
vector − 2a + 3 b − c is equal to −1
− 7q + r ⇒ a = l , b = 2l + m and m =
(a) p − 4q (b) 4
5 1$
(c) 2p − 3q + r (d) 4p − 2r ai$ + b$j + k is unit vector
2
Sol. (b) Let −2a + 3 b − c = xp + yq + zr 3 11
∴ a2 + b2 = ⇒ 5a 2 − a − =0
⇒ −2a + 3 b − c = (2 x + y − 3z ) a + ( −3 x − 2y + z ) b 4 16
+ (y + 2z ) c a1 and a 2 are roots of above equation
1 1 a1 + a 2 16
∴ 2 x + y − 3z = − 2, −3 x − 2y + z = 3 ⇒ + = =−
a1 a 2 a1a 2 11
and y + 2z = − 1

JEE Type Solved Examples :


More than One Correct Option Type Questions
l Ex. 38 The vector $i + x$j + 3k$ is rotated through an angle l Ex. 39 a, b and c are three coplanar unit vectors such
θ and is doubled in magnitude. It now becomes that a + b + c = 0. If three vectors p, q and r are parallel to
4 $i + ( 4 x − 2 ) $j + 2k$ . The values of x are a, b and c respectively, and have integral but different magni-
−2 tudes, then among the following options, | p + q + r| can take
(a) 1 (b) a value equal to
3
4 (a) 1 (b) 0 (c) 3 (d) 2
(c) 2 (d)
3 Sol. (c,d) Let a, b and c lie in the XY -plane.
Sol. (b,c) Let α = i$ + x$j + 3 k$ , 1 3$ 1 3$
Let a = $i, b = − $i + j and c = − $i − j
β = 4 i$ + ( 4 x − 2 ) $j + 2 k$ 2 2 2 2
Therefore, | p + q + r | = | λa + µb + vc |
Given, 2| α | = | β |
 1 3 $  1 3 $
⇒ 2 10 + x 2 = 20 + 4(2 x − 1 ) 2 = λ$i + µ  − $i + j + v  − $i − j
 2 2   2 2 
⇒ 10 + x 2 = 5 + ( 4 x 2 − 4 x + 1 )  µ v 3
=  λ − −  $i + (µ − v ) $j
⇒ 3x 2 − 4x − 4 = 0  2 2 2
2 2
⇒ x = 2, −  µ v 3
3 =  λ − −  + (µ − v ) 2
 2 2 4
30 Textbook of Vector & 3D Geometry

= λ2 + µ 2 + v 2 − λµ − λv − µv |b | |a | | a||a|  a b
Now, a+ b=  + 
1 |a | + | b | |a | + | b | |a| + |a| |a| | b| 
= ( λ − µ ) 2 + (µ − v ) 2 + (v − λ ) 2
2 |b | |a | | a||b|  a b
Also, a+ b=  + 
1 2| a | + | b | 2| a | + | b | 2|a| + |b| |a| | b| 
= 1+1+4= 3
2
 a b
⇒ | p + q + r | can take a value equal to 3 and 2. Other two vectors cannot be written in the from t  + 
| a| | b | 
l Ex. 40 A, B, C and D are four points such that l Ex. 42 The vectors x $i + ( x + 1) $j + ( x + 2 )k$ ,
AB = m ( 2 $i − 6 $j + 2k$ ), BC = ( $i − 2 $j ) and ( x + 3 ) $i + ( x + 4 ) $j + ( x + 5 )k$ and
CD = n( −6 $i + 15 $j − 3k$ ). If CD intersects AB at some point E, ( x + 6 ) $i + ( x + 7 ) $j + ( x + 8 )k$ are coplanar if x is equal to
then (a) 1 (b) −3 (c) 4 (d) 0
1 1
(a) m ≥ (b) n ≥ (c) m = n (d) m < n
2 3 Sol. (a, b, c, d)
Sol. (a, b) Let EB = p AB and CE = q CD x$i + ( x + 1 ) $j + ( x + 2 ) k$ , ( x + 3 ) $i + ( x + 4 ) $j + ( x + 5 ) k$ and
Then 0 < p and q ≤ 1 ( x + 6 ) $i + ( x + 7 ) $j + ( x + 8 ) k$ are coplanar. We have
B x x +1 x +2
D determinant of their coefficients as x + 3 x + 4 x + 5
x +6 x +7 x +8
Applying C 2 → C 2 − C1 and C 3 → C 3 − C1, we have
E
C x 1 2
x + 3 1 2 = 0. Hence, x ∈ R.
A
x +6 1 2
Since, EB + BC + CE = 0
pm(2 $i − 6 $j + 2 k$ ) + ( $i − 2 $j) + qn( −6 $i + 15 $j − 3 k$ ) = 0 l Ex. 43 Given three vectors a, b, and c are non-zero and
⇒(2 pm + 1 − 6qn ) $i + ( −6 pm − 2 + 15qn ) $j + (2 pm − 6qn ) k$ = 0 non-coplanar vectors. Then which of the following are
⇒ 2 pm − 6qn + 1 = 0,
coplanar.
(a) a + b, b + c, c + a (b) a − b, b + c, c + a
−6 pm − 2 + 15qn = 0
(c) a + b, b − c, c + a (d) a + b, b + c, c − a
2pm −6qn = 0
Sol. (b, c, d) c + a = ( b + c ) + (a − b )
Solving these, we get
a + b = (b − c) + (c + a )
1 1
p= and q= a + c = (a + b ) + ( c − a )
(2m ) (3n )
So, vectors in options (b), (c) and (d) are coplanar.
1 1
∴ 0< ≤1 and 0< ≤1
(2m ) (3n ) l Ex. 44 In a four-dimensional space where unit vectors
1 1
⇒ m≥ and n≥ along the axes are $i, $j, k$ and $i, and a 1 , a 2 , a 3 , a 4 are four
2 3
non-zero vectors such that no vector can be expressed as a
l Ex. 41 If non-zero vectors a and b are equally inclined to linear combination of others and ( λ − 1)(a 1 − a 2 ) +
coplanar vector c, then c can be µ(a 2 + a 3 ) + γ (a 3 + a 4 − 2a 2 ) + a 3 + δa 4 = 0, then
|a | |b |
(a) a+ b (a) λ = 1 (b) µ = −
2
(c) γ =
2
(d) δ =
1
| a | + 2|b | | a | + |b | 3 3 3
|b | |a | Sol. (a, b, d)
(b) a+ b
| a | + |b | | a | + |b| ( λ − 1 )(a1 − a 2 ) + µ(a 2 + a 3 ) + γ(a 3 + a 4 − 2a 2 ) + a 3 + δa 4 = 0
|a | |b | i.e. ( λ − 1 )a1 + (1 − λ + µ − 2 γ )a 2 + (µ + γ + 1 ) a 3 + ( γ + δ ) a 4 = 0
(c) a+ b
| a | + 2|b | | a | + 2|b | Since, a1, a 2, a 3 and a 4 are linearly independent, we have
|b | |a | λ − 1 = 0, 1 − λ + µ − 2 γ = 0,
(d) a+ b
2| a | + |b | 2| a | + |b | µ + γ + 1 = 0 and γ+δ=0
Sol. (b,d) Since, a and b are equally inclined to c, therefore c must i.e. λ = 1, µ = 2 γ, µ + γ + 1 = 0, γ + δ = 0
a b 2
Hence, λ = 1, µ = − , γ = − , δ =
1 1
be of the form t  +  3 3 3
|a| |b|
Chap 01 Vector Algebra 31

JEE Type Solved Examples :


Statement I and II Type Questions
Directions (Ex. Nos. 45-51) This section is based on u+v
between them, then x = .
Statement I and Statement II. Select the correct answer α
from the codes given below. 2 sin
2
(a) Both Statement I and Statement II are correct and Statement II If ABC is an isosceles triangles with
Statement II is the correct explanation of Statement I
(b) Both Statement I and Statement II are correct but AB = AC =1, then vectors representing bisector of angle A is
Statement II is not the correct explanation of AB + AC
Statement I
given by AB = .
2
(c) Statement I is correct but Statement II is incorrect Sol. (d) We know that the unit vector along bisector of unit
(d) Statement II is correct but Statement I is incorrect u+v
vectors u and v is , where θ is the angle between
θ
2 cos
l Ex. 45 Statement I If | a | = 3, | b | = 4 and | a + b | = 5, 2
then | a − b | = 5. vectors u and v.
Statement II The length of the diagonals of a rectangle is Also, in an isosceles ∆ABC in which
the same. AB = AC , the median and bisector from A must be same line.
Sol. (a) We have, adjacent sides of triangle | a| = 3, | b| = 4
The length of the diagonal is | a + b | = 5
l Ex. 49 Statement I If a = 2 $i + k$ , b = 3 $j + 4k$ and
Since, it satisfies the Pythagoras theorem, a ⊥ b c = λa + µb are coplanar, then c = 4 a − b.
So, the parallelogram is a rectangle. Statement II A set vectors a 1 , a 2 , a 3 ,… , a n is said to be
Hence, the length of the other diagonal is | a − b | = 5. linearly independent, if every relation of the form
l1a 1 + l 2 a 2 + l3 a 3 +.....+ln a n = 0 implies that
l Ex. 46 Statement I If | a + b | = | a − b |, then a and b l1 = l 2 = l3 =… = ln = 0 (scalar).
Sol. (b) a, b and c are coplanar c = λa + µb ⇒ λ = 4 and µ = − 1
are perpendicular to each other.
Statement II If the diagonals of a parallelogram are equal
in magnitude, then the parallelogram is a rectangle. l Ex. 50 Statement I Let A( a ), B(b ) and C(c ) be three
Sol. (a) a + b = a − b are the diagonals of a parallelogram whose points such that a = 2 $i + k$ , b = 3 $i − $j + 3k$ and
sides are a and b.
c = − $i + 7 $j − 5k$ . Then, OABC is a tetrahedron.
|a + b| = |a − b|
Statement II Let A(a ), B( b ) and C(c ) be three points such
Thus, diagonals of the parallelogram have the same length.
So, the parallelogram is a rectangle, i.e. a ⊥ b.
that vectors a, b and c are non-coplanar.
Then OABC is a tetrahedron.
Sol. (a) Given vectors are non-coplanar.
l Ex. 47 Statement I If I is the incentre of ∆ABC, then
Hence, the answer is (a).
| BC | IA + | CA | IB + | AB | IC = 0
Statement II The position vector of centroid of ∆ABC is l Ex. 51 Statement I Let a, b, c and a be the position
OA + OB + OC vectors of four points A, B, C and D and
.
3 3 a − 2b + 5c − 6d = 0. Then points A, B, C and D are
Sol. (b) We know that, coplanar.
| CB | OA + | CA | OB + | AB | OC Statement II Three non-zero linearly dependent co-initial
OI =
| BC | + | CA | + | AB | vectors ( PQ, PR and PS) are coplanar. Then
OA + OB + OC PQ = λPR + µPS, where λ and µ are scalars.
and OG =
3 Sol. (a)
3a − 2 b + 5 c − 6 d = (2a − 2 b ) + ( −5a + 5 c ) + (6a − 6 d )
l Ex. 48 Statement I If u and v are unit vectors inclined = − 2 AB + 5 AC − 6 AD = 0
at an angle α and x is a unit vector bisecting the angle Therefore, AB, AC and AD are linearly dependent.
Hence, by Statement II, Statement I is true.
32 Textbook of Vector & 3D Geometry

JEE Type Solved Examples :


Passage Based Questions
Passage I 53. (b) Hence, P divides AL in the ratio 3 : 1 and P divides DB in the
(Ex. Nos. 52 to 54) ratio 1 : 3 Similarly Q divides DB in the ratio 1 : 3.
1
ABCD is a parallelogram. L is a point on BC which divides Thus, DQ = DB
4
BC in the ratio 1 : 2. AL intersects BD at P . M is a point on
1
DC which divides DC in the ratio 1 : 2 and AM intersects and PB = DB
4
BD in Q.
1
54. (b)∴ PQ = DB ,
l Ex. 52 Points P divides AL in the ratio 2
(a) 1 : 2 (b) 1 : 3 i.e. PQ : DB = 1 : 2
(c) 3 : 1 (d) 2 : 1
Passage II
l Ex. 53 Point Q divides DB in the ratio (Ex. Nos. 55 to 56)
(a) 1 : 2 (b) 1 : 3 Let A, B , C , D, E represent vertices of a regular pentagon
(c) 3 : 1 (d) 2 : 1 ABCDE. Given the position vector of these vertices be
a, a + b, b, λa and λb respectively.
l Ex. 54 PQ : DB is equal to
AD
2 1 l Ex. 55 The ratio is equal to
(a) (b) BC
3 3
3π 3π 2π π
1 3 (a) 1 − cos : cos (b) 1 + 2 cos : cos
(c) (d) 5 5 5 5
2 4
π π
Sol. (Ex. Nos. 52-54) (c) 1 + 2 cos : 2 cos (d) None of these
5 5
52. (c) 2
D C
M l Ex. 56 AD divides EC in the ratio
b Q 2π 3π
2 (a) cos :1 (b) cos :1
5 5
P L π
(c) 1 : 2 cos (d) 1 : 2
1 5
A B
a Sol. (Ex. Nos. 55-56) Given ABCDE is a regular pentagon
1 A B
BL = b
3
1
∴ AL = a + b
3
Let AP = λAL and P divides DB in the ratio µ : 1 − µ 2p/5
O
λ E C
Then, AP = λa + b …(i) p/5
3
2p/5
Also, AP = µa + (1 − µ ) b …(ii)
From Eqs. (i) and (ii), D
λ
λa + b = µa + (1 − µ ) b Let position vector point A and C be a and b, respectively.
3
AD is parallel to BC and AB is parallel to EC.
∴ λ =µ
Therefore,
λ
and =1 −µ AOCB is a parallelogram and position vector of B is a + b.
3
3 The position vectors of E and D are λb and λa respectively.
∴ λ= Also, OA = BC = AB = OC = 1 (let)
4
Chap 01 Vector Algebra 33

Therefore, AOCB is rhombus. Sol. (Ex. Nos. 57-58)


3π C(c) 1E 2 B(b)
∠ABC = ∠AOC =
5
P
3π 2π
and ∠OAB = ∠BCO = π − =
5 5
Further, OA = AE = 1 and OC = CD = 1
Thus, ∆EAO and ∆OCD are isosceles. O A(a)
In ∆OCD, using sine rule we get.
Let the position vector of A and C be a and c respectively.
OC OD
= Therefore,
2π π
sin sin Position vector of
5 5
B = b =a + c …(i)
1
⇒ OD = = OE Also, position vector of
π
2 cos b + 2c a + 3c
5 E= = …(ii)
1 3 3
⇒ AD = OA + OD = 1 +
π Now, point P lies on angle bisector of ∠AOC. Thus,
2 cos
5 Position vector of point
π a b
1 + 2 cos P = λ +  …(iii)
AD 1 5
55. (c) =1+ = |a| |b|
BC π π
2 cos 2 cos Also, let P divides EA in ration µ : 1. Therefore, Position vector
5 5
OE 1 of P
56. (c) = a + 3c
OC 2 cos π µa +
3 (3µ + 1 )a + 3 c
5 = = …(iv)
µ+1 3(µ + 1 )
Passage III Comparing Eqs. (iii) and (iv), we get
(Ex. Nos. 57 to 58) a c  (3µ + 1 ) a + 3 c
λ +  =
In a parallelogram OABC vectors a, b, c respectively, the |a| |c| 3(µ + 1 )
position vectors of vertices A, B , C with reference to O as λ 3µ + 1 λ 1
⇒ = and =
origin. A point E is taken on the side BC which divides it in | a| 3(µ + 1 ) |c| µ + 1
the ratio of 2 :1. Also, the line segment AE intersects the 3| c | − |a|
⇒ =µ
line bisecting the angle ∠AOC internally at point P. If CP 3|a|
when extended meets AB in point F , then λ 1 3| a|| c|
⇒ = ⇒ λ=
| c | 3| c| −| a| + 1 3| c| + 2| a|
l Ex. 57 The position vector of point P is 3| a |
| a | |c |  a c
(a)  +  3 |a || c|  a c
3|c | + 2| a | |a| |c| 57. (b) So, position vector of P is  + .
3 | c | + 2|a | |a| |c|
3| a | | c |  a c
(b)  +  58. (d) Let F divides AB in ratio t : 1, then position vector of F is
3|c | + 2| a | |a| |c| tb+ a
2| a | |c |  a c t +1
(c)  +  Now, points C , P , F are collinear, Then, CF = mCP
3|c | + 2| a | |a| |c|
t(a + c )  3| a || c |  a c 
(d) None of the above ⇒ − c = m  +  − c
t +1 3| c | + 2| a | |a| |c| 
l Ex. 58 The ratio in which F divides AB is Comparing coefficients, we get
2 |a | |a | t 3| c |
(a) (b) =m
| a | − 3| c | | a | − 3| c | t +1 3| c | + 2| a |
−1 | a | − 3| c |
(c)
3| a |
(d)
3| c | and =m
| a | − 3| c | 3 | c | −| a | t +1 3| c | + 2| a |
3| c |
t=
3| c | − | a |
34 Textbook of Vector & 3D Geometry

JEE Type Solved Examples :


Matching Type Questions
l Ex. 59 In the Cartesian plane, a man starts at origin and (C) The position vector of Q is given by
walks a distance of 3 units of the North-East direction and 1 $
OP + PQ = OP + OR = ( − i + 7 $j)
reaches a point P. From P, he walks a distance of 4 units in 2
the North-West direction to reach a point Q. Construct the Y

parallelogram OPQR with PO and PQ as adjacent sides. Let Q


M be the mid-point of PQ.
3 M
Column I Column II 4
3 $ $ R P
A. The position vector of P is (p) ( i + j) T
2 4 3
45° 45°
B. The position vector of R is (q) 1 ( $i + 5 $j) X
O
2
C. The position vector of M is (r) 2 2( − i$ + $j)  3  $ $  1 
  ( i + j) +   ( − $i + 7 $j)
D. If the line OM meets the diagonal PR (s) 2 $  2  2
( i + 5 $j) ∴ OM =
in the point T , then OT equals 3 2
2 $i + 10 $j $i + 5 $j
Sol. A → p , B → r , C → q , D → s = =
2 2 2
(A) Let $i and $j be the unit vectors along OX and OY (D) Now, PT : RT = 1 : 2
respectively. 1( OR ) + 2(OP )
Therefore, OT =
Now, OP = 3 and ∠XOP = 45 ° implies that 3
3 $ $  4 $ $  3  
OP = (3 cos 45 ° ) i$ + (3 sin 45 ° ) $j = ( i + j)   ( − i + j) + 2   ( $i + $j) 
2  2   2 
=
(B) Again, ∠XOR = 135 ° and OR = 4 implies that 3
4 $ $ 2 $
OR = ( − i + j) = 2 2( − i$ + $j) = ( i + 5 $j)
2 3

JEE Type Solved Examples :


Single Integer Answer Type Questions
l Ex. 60 P and Q have position vectors a and b relative to l Ex. 61 If A(1, − 1, − 3 ), B( 2, 1, − 2 ) and C( −5, 2, − 6 ) are the
the origin O and X , Y divide PQ internally and externally position vectors of the vertices of ∆ABC. The length of the
respectively in the ratio 2 : 1. Vector XY is λa + µb, then the λ 10
value of | λ + µ| is bisector of its internal angle at A is , then value of λ is
4
Sol. (0) Since, X and Y divide PQ internally and externally in the Sol. (3) We have, AB = $i + 2 $j + k$ , AC = − 6 $i + 3 $j − 3 k$
2b + a
ratio 2 : 1, then X = and y = 2b − a ⇒ | AB| = 6 and |AC | = 3 6
3
Clearly, point D divides BC in the ratio AB : AC , i.e. 1 : 3
∴ XY = Position vector of y-Position vector of x
2 b + a 4 b 4a ( −5 $i + 2 $j − 6 k$ ) + 3(2 $i + $j − 2 k$ )
= 2b − a − = − ∴Position vector of D =
3 3 3 1+3
On comparing it with λa + µb, we get 1 $
= ( i + 5 $j − 12 k$ )
4 4 4
λ = − and µ = 1 $ 3
3 3 ∴ AD = ( i + 5 $j − 12 k$ ) − ( $i − $j − 3 k$ ) = ( − i$ + 3 $j)
4 4
−4 4 3
∴ | λ + µ| = + =0 ⇒ | AD| = AD = 10
3 3 4
∴ λ =3
Chap 01 Vector Algebra 35

l Ex. 62 Let ABC be a triangle whose centroid is G, l Ex. 64 Find the least positive integral value of x for
orthocentre is H and circumcentre is the origin ‘O’. If D is which the angle between vectors a = x $i − 3 $j − k$ and
any point in the plane of the triangle such that no three of b = 2x $i + x$j − k$ is acute.
O, A, C and D are collinear satisfying the relation
Sol. (2) Let a = x$i − 3 $j − k$ and b = 2x$i + x$j − k$ be the adjacent
AD + BD + CH + 3HG = λHD, then what is the value of the
sides of the parallelogram.
scalar ‘∆’
Now angle between a and b is acute, i.e. | a + b| > | a − b |
Sol. LHS = d − a + d − b + h − c + 3( g − h ) 2 2
⇒ 3 x$i + ( x − 3 ) $j − 2 k$ > − x$i − ( x + 3 ) $j
(a + b + c )
= 2 d − (a + b + c ) + 3 − 2h
3 or 9x 2 + (x − 3)2 + 4 > x 2 + (x + 3)3
= 2 d − 2 h = 2( d − h ) = 2 HD ⇒ λ = 2
a–b
b
l Ex. 63 Let a, b and c be unit vectors such that a+b

a + b − c = 0. If the area of triangle formed by vectors a and


a
b is A, then what is the value of 16 A 2 ?
or 8 x 2 − 12 x + 4 > 0 or 2 x 2 − 3 x + 1 > 0
Sol. (3) Given a + b = c
1
Now, vector c is along the diagonal of the parallelogram which or (2 x − 1 )( x − 1 ) > 0 ⇒ x < or x > 1
2
has adjacent side vectors a and b. Since, c is also a unit vector,
triangle formed by vectors a and b is an equilateral triangle. Hence, the least positive integral value is 2.

3 3
Then, Area of triangle = ⇒ A2 = ⇒ 16 A 2 = 3 l Ex. 65 If the points a(cos α + $i sin γ ), b(cos β + $i sin β)
4 10
and c(cos γ + $i sin γ ) are collinear, then the value of | z | is …
(where z = bc sin(β − γ ) + ca sin( γ − α ) + ab sin(α + β) + 3i) $
a cosα a sin α 1
b a+b
Sol. (3) b cosβ b sin β 1 = 0
60°
c cos γ c sin γ 10
60°
⇒ bc sin( γ − β ) + a sin(α − γ ) + ab sin(β − α ) = 0
a
⇒ |z | =3

Subjective Type Questions


l Ex. 66 A particle in equilibrium is subjected to four 4u 4v
⇒ − + w cosθ = 0 …(i)
forces 13 13
12 12
F2 = u  $i − $j + k$ 
4 12 3 − u − v + w sinθ = 0
F1 = − 10k$ , …(ii)
 13 13 13 
13 13
3 3
u + v − 10 = 0 …(iii)
F3 = v  − $i − $j + k$ 
4 12 3 13 13
 13 13 13  130
From Eq. (iii), we get u + v =
and F = w(cos θ $i + sin θ $j)
4
3
From Eq. (ii), we get
Find the values of u , v and w in terms of θ.
12
Sol. Since, the particle is in equilibrium. − (u + v ) + w sin θ = 0
13
F1 + F2 + F3 + F4 = 0
12  130
4 12 3   4 12 3  ⇒ −   + w sinθ = 0
−10 k$ + u  $i − $j + k$  + v  − $i − $j + k$  13  3 
 13 13 13   13 13 13 
40
+ w(cosθ$i + sin θ$j) = 0 ⇒ w= = 40 cosec θ
sinθ
 4u 4v   −12 12 
⇒  − + w cosθ i$ +  u − v + w sin θ $j On substituting the value of w in Eqs. (i) and (ii), we get
 13 13   13 13 
u − v = − 130 cot θ
3 3  130
+  u + v − 10 k$ = 0 and u+v=
 13 13  3
36 Textbook of Vector & 3D Geometry

On solving, we get On adding these


u+
65
− 65 cot θ AA′ + BB′ + CC′ = 3 GG′ + ( AG + BG + CG )
3 + ( G′ A′ + G′ B′ + G′ C′ )
65 = 3 GG′ + ( AG + 2 DG ) + ( G′ A′ + 2 G′ D′ )
v+ + 65 cot θ and w = 40 cosec θ
3 (using AD and A′ D′ as the medians of ∆ABC and
∆A′ B′ C ′, respectively)
l Ex. 67 Find all values of ‘ λ’ such that x , y , z ≠ (0, 0, 0 ) = 3GG′ + ( AG + GA ) + G′ A′ + A′ G′
and ( $i + $j + 3k$ ) x + (3 $i − 3 $j + k$ ) y + ( −4 $i + 5 $j ) z = 3GG′ + O + O
= λ( x $i + y$j + zk$ ), where $i, $j and k$ are unit vectors along the ∴ AA′ + BB′ + CC′ = 3GG ′
coordinate axes. Aliter
Sol. Here, We know by triangle law
( i$ + $j + 3 k$ ) x + (3 $i − 3 $j + k$ )y + ( −4 $i + 5 $j)z = λ ( x$i + y$j + zk$ ) AA′ = OA′ − OA
$ we get
On comparing the coefficients of $i, $j and k, BB′ = OB′ − OB
CC′ = OC′ − OC
x + 3y − 4z = λx
⇒ AA′ + BB′ + CC′ = ( OA′ + OB′ + OC′ )
⇒ (1 − λ ) x + 3y − 4z = 0 …(i)
− ( OA + OB + OC )
x − 3y + 5z = λy
= 3 OG′ − 3 OG′ = 3 GG′
⇒ x − (3 + λ )y + 5z = 0 …(ii)
3x + y = λz l Ex. 69 If D , E and F are the mid-points of the sides
⇒ 3 x + y − λz = 0 …(iii)
BC , CA and AB, respectively of a ∆ABC and O is any point,
The Eqs. (i), (ii) and (iii) will have a non-trivial solution, if
show that
1−λ 3 −4
(i) AD + BE + CF = 0
1 − (3 + λ ) 5 = 0
3 1 −λ
(ii) OE + OF + DO = OA
2 1 1
[∴ (x, y , z ) ≠ ( 0, 0, 0 ) ∴∆ = 0] (iii) AD + BE + CF = AC
3 3 2
⇒(1 − λ ){ λ (3 + λ ) − 5 } − 3 { − λ − 15 } − 4 {1 + 3( λ + 3 )} = 0
Sol. Consider the point O as origin, we have,
⇒ (1 − λ ){ λ2 + 3 λ − 5 } − 3 { − λ − 15 } − 4 {3 λ + 10 } = 0
A(a) O
⇒ λ3 + 2 λ2 + λ = 0
⇒ λ ( λ2 + 2 λ + 1 ) = 0
⇒ λ(λ + 1)2 = 0
F(f)
E(e)
∴ λ = 0 or λ = − 1

l Ex. 68 If G is the centroid of the ∆ABC and if G ′ is the


centroid of another ∆A ′ B ′ C ′, then prove that B(b) D(d) C(c)
AA ′ + BB ′ + CC ′ = 3GG ′ .
(i) AD + BE + CF = = ( d − a ) + ( e − b ) + ( f − c )
Sol. Here,
= ( d + e + f ) − (a + b + c ) = 0 [using Eq. (i)]
G is centroid of ∆ABC and G′ is centroid of ∆A′ B′ C ′, shown as
in figure. ⇒ AD + BE + CF = 0
A′ (ii) Here, OE + OF + OD = e + f − d
c+a a+ b b+ c
= + − = a = OA
G′ 2 2 2
A
C′
∴ OE + OF + OD = OA
G B′ 2 1 2 1
(iii) Here, AD + BE + CF = ( d − a ) + ( e − b ) + ( f − c )
3 3 3 3
B C b+ c 2c+a  1 a + b 
= −a +  − b +  − c
Clearly, AA′ = AG + GG′+ G′ A′ (polygon law) 2 3 2  3 2 

BB′ = BG + GG′ + G′ B′  1 1  1 2 1  1 1 1
= a  −1 + +  + b  − +  + c  + − 
CC′ = CG + CG′ + G′ C′  3 6   2 3 6   2 3 3
Chap 01 Vector Algebra 37

1 1 1
= − a + c = (c − a ) = c cos A cosec C
2 2 2 c
= ⋅ cos A = 2 R cos A
1 sin C
= AC
2  a b c 
Q = = = 2R
2 1 1  sin A sin B sin C 
AD + BE + CF = AC
3 3 2 AO′ = 2(OD ) …(iii)
Now, ∆AGO′ and ∆OGD are similar.
l Ex. 70 If A and B be two vectors and k be any scalar
OG GD OD 1
quantity greater than zero, then prove that ∴ = = = [using Eq. (iii)]
O ⋅ G GA AO′ 2
 1
| A + B | 2 ≤ (1 + k )| A| 2 1 +  | B | 2 ⇒ 2OG = O′ G
 k
(i) We have, SA + SB + SC = SA + ( SB + SC )
 1 = SA + 2 SD
Sol. We know, (1 + k )| A| 2 + 1 +  | B | 2 (Q D is the mid-point of BC)
 k
= (1 + 2 ) SG = 3 SG
1
= | A| + k| A| + | B| + | B| 2
2 2 2
…(i) (ii) On replacing S by O in Eq. (i), we get
k
OA + OB + OC = 3 OG
1
1 2  1 2 2 = 2OG + OG = GO′ + OG
Also, k | A| 2 + | B| ≥ 2 k| A| 2⋅ | B|  = 2| A|⋅| B| …(ii)
k  k  = OG + GO′ = OO′
(since, Arithmetic mean ≥ Geometric mean) (iii) O′ A + O′ B + O′ C = 3 O′ G [from Eq. (i)]
 1 = 2O′ G + O′ G
So, (1 + k )| A| 2 + 1 +  | B | 2 ≥ | A| 2 + | B| 2 + 2| A| ⋅ | B|
 k = 2 O′ G + 2 GO (Q 2OG = O′ G)
= (| A| + | B| ) 2 [using Eqs. (i) and (ii)] = 2O′ O
(iv) AO + O′ B + O′ C = 2 AO′ + ( O′ A + O′ B + O′ C )
And also, | A| + | B| ≥ | A + B|
= 2 AO′ + 2 O′ O [From Eq. (iii)]
 1
Hence, (1 + k )| A| 2 + 1 +  | B| 2 ≥ | A + B| 2 = 2( AO′ + O′ O) = 2AO = AP
 k
(Q AO is the circumradius of ∆ABC)
l Ex. 71 If O is the circumcentre and O’ the orthocenter of
∆ABC prove that
l Ex. 72 If c = 3 a + 4b and 2c = a − 3b, show that,
(i) SA + SB + SC = 3SG, where S is any point in the plane (i) c and a have the same direction and | c | > | a |.
of ∆ABC. (ii) c and b have opposite direction and | c | > | b |.
(ii) OA + OB + OC = OO ′ Sol. We have,
c = 3 a + 4b and 2 c = a − 3 b
(iii) O ′ A + O ′ B + O ′ C = 2O ′ O
⇒ 2(3 a + 4b ) = a − 3b
(iv) AO ′+O ′ B + O ′ C = AP
⇒ 5a = − 11 b
where, AP is diameter of the circumcircle. 11 5
Sol. Let G be the centroid of ∆ABC, first we shall show that ⇒ a = − b and b = − a
5 11
circumcentre O, orthocenter O′ and centroid G are collinear
and O′ G = 2OG.  5   5 
(i) c = 3a + 4 b = 3 a + 4  − a  using b = − a
A  11   11 
20 13
M = 3a − a = a
11 11
O′
which shows that c and a have the same direction.
A A 13
And c = a
B C 11
L D 13
⇒ | c | = |a | ⇒ | c | > |a |
Let AL and BM be perpendiculars on the sides BC and CA, 11
11
respectively. Let AD be the median and OD be the (ii) We have, c = 3a + 4 b and a = − b
perpendicular from O on side BC. If R is the circumradius of 5
circumcircle of ∆ABC, then OB = OC = R.  11  33
c = 3  − b + 4 b = − b + 4 b
In ∆OBD, we have OD = R cos A …(i)  5  5
In ∆ABM, AM = AB cos A = c cos A …(ii) 13
c=− b
Form ∆AO′ M, AO′ = AM sec(90 ° − C ) 5
38 Textbook of Vector & 3D Geometry

This shows c and b have opposite directions. where, k and l are scalars.
13 13 BD BF
Also, |c | = − b = |b | ⇒ |c | > |b | ∴ = l and =k …(i)
5 5 BC BA
i.e. BC : BD = 1 : l
l Ex. 73 A transversal cuts the sides OL, OM and diagonal BC 1 DC 1 − l
⇒ −1 = −1 ⇒ =
ON of a parallelogram at A, B and C respectively. BD l BD l
OL OM ON BD l BA 1
Prove that + = . ⇒ = and =
DC 1 − l BF k
OA OB OC
BA 1 AF k − 1
Sol. We have, ⇒ 1− =1 − ⇒ = …(ii)
BF k BF k
ON = OL + LN = OL + OM …(i)
Now, let E divide the line AC in the ratio of x : y
Let OL = xOA, OM = y OB …(ii)
BD BF
and ON = zOC x⋅ + y.
xc + ya l k
So, that BE = = …(iii)
So, | OL | = x | OA|, | OM | = y | OB| and | ON | = z | OC | x +y x +y
OL OM ON
∴ x= ,y = and z = ⇒ BE −
x
BD −
y
BF = 0
OA OB OC l (x + y ) k( x + y )
∴From Eqs. (i) and (ii), we have
Since, D, E and F are collinear.
M N
Sum of coefficients must be zero.
x y
Hence, 1 − − =0
l ( x + y ) k( x + y )
B x y x y
⇒ (x + y ) − − =0 ⇒ x+y = +
O A L
l k l k
 1 1   l − 1 1 − k
⇒ x 1 −  = y  − 1 ⇒ x   =y 
zOC = xOA + yOB  l k   l   k 
⇒ x OA + y OB − z OC = 0 l y k −1
⇒ ⋅ ⋅ =1
∴Points A, B and C are collinear, the sum of the coefficients of l −1 x k
their PV must be zero. BD CE AF
⇒ x + y −z = 0 ⇒ ⋅ ⋅ = 1 [using Eqs. (i), (ii) and (iii)]
DC AE BF
OL OM ON
i.e. + =
OA OB OC l Ex. 75 Let A(t ) = f 1 (i ) $i + f 2 (t ) $j and
B(t ) = g 1 (t ) $i + g 2 (t ) $j t ∈[0, 1], where f 1 , f 2 , g 1 and g 2 are
l Ex. 74 If D , E and F be three points on the sides BC , CA
continuous functions. Then show that A(t ) and B(t ) are
and AB, respectively of a ∆ABC. such that the points D , E
parallel for some t.
BD CE AF
and F are collinear then prove that ⋅ ⋅ =1 Sol. If A(t ) and B(t ) are non-zero vectors for all t
CD AE BF and A( 0 ) = 2 $i + 3 $j, A(1 ) = 6 $i + 2 $j, B( 0 ) = 3 $i + 2 $j,
(Menelau’s theorem) and B(1 ) = 2 $i + 6 $j.
Sol. Here, D, E and F be the points on the sides BC, CA and AB In order to prove that A(t ) and B(t ) are parallel vectors for
respectively of ∆ABC. Such that points D, E and F are some values of t. It is sufficient to show that A(t ) = λB(t ) for
collinear, be Shawn as the adjoining figuece. some λ.
Let B as the origin, BA = a and BC = c ⇔ { f1(t ) i$ + f 2(t ) $j} = λ{g1(t ) $i + g 2(t ) $j}
Then, BF = ka and BD = lc ⇔ f1(t ) = λg1t and f 2(t ) = λg 2(t )
f1(t ) g1(t )
F ⇔ =
f 2(t ) g 2(t )
⇔ f1(t ) g 2(t ) − f 2(t ) g1t = 0 for some t ∈[ 0, 1 ]
Let f (t ) = f1(t ) g 2(t ) − f 2(t ) g1(t ), t ∈ [ 0, 1 ]
A
Since, f1, f 2, g1 and g 2 are continuous functions.
x E ∴F (t ) is also a continuous function.
a
y Also, f ( 0 ) = f1( 0 ) g 2( 0 ) − g1( 0 ) f 2( 0 )
=2 ×2 −3 ×3 = 4 −9 = −5 < 0
B c D C
and f (1 ) = f1(1 ) g 2(1 ) − g1(1 ) f 2(1 )
Chap 01 Vector Algebra 39

= 6 × 6 − 2 × 2 = 32 > 0 ⇒ x = λ + µ, 1 = λy + µ, 1 = λ + µz
Thus, F (t ) is a continuous function on [0, 1] such that 1 −µ 1−λ
⇒ x = λ + µ, y = ,z =
F ( 0 ) ⋅ F (1 ) < 0. λ µ
∴By intermediate value theorem, there exists some t ∈( 0, 1 ) λ −1 + µ
such that ⇒ 1 − x = 1 − λ − µ, 1 − y = ,
λ
f (t ) = 0 µ −1 + λ
1 −z =
⇒ f1(t ) g 2(t ) − f 2(t ) g1t = 0 µ
⇒ A(t ) = λB(t ) for some λ. 1 1 1 1 λ µ
∴ + + = + +
Hence, A(t ) and B(t ) are parallel vectors. 1 − x 1 −y 1 −z 1 − λ −µ λ + µ − 1 λ + µ − 1
−1 + λ + µ
l Ex. 76 Prove that if cos α ≠1 , cos β ≠1 and cos γ ≠1, then = =1
λ + µ −1
the vectors a = $i cos α + $j + k$ , b = $i + $j cos β + k$ , 1 1 1
∴ + + =1
c = $i + $j + k$ cos γ can never be coplanar. 1 − x 1 −y 1 −z
Sol. Suppose that, a, b and c are coplanar. Aliter
cosα 1 1 Thus, above problem could also be solved as
⇒ 1 cosβ 1 =0 x 1 1 x −1 0 1 −z
1 1 cos γ 1 y 1 =0 ⇒ 0 y −1 1 −z = 0
1 1 z 1 1 z
On applying R2 → R2 − R1 and R3 → R3 − R1
cos α 1 1 (using R1 → R1 − R3 and R2 → R2 − R3)
⇒ 1 − cos α cos β − 1 0 =0
1 0 −1
1 − cos α 0 cos γ − 1
⇒( x − 1 )(y − 1 )(z − 1 ) 0 1 −1 = 0
⇒ cosα (cosβ − 1 ) (cos γ − 1 ) − (1 − cosα )(cos γ − 1 ) −1 −1 −z
− (1 − cosα )(cosβ − 1 ) = 0 1−x 1 −y 1 −z
On dividing throughout by (1 − cosα )(1 − cosβ )(1 − cos γ ), we  1 1 1 
get  using R1 → R1, R2 → R 2, R 3 → R 3
 x −1 y −1 z −1 
cosα 1 1 1 1 z
+ + =0 ⇒ − (1 ) + ( −1 ) − (1 ) = 0
1 − cosα 1 − cosβ 1 − cos γ (1 − x ) (1 − y ) (1 − z )
−(1 − cosα ) + 1 1 1 (expanding along R3)
⇒ + + =0 −1 1 (1 − z ) − 1
1 − cosα 1 − cosβ 1 − cos γ ⇒ − + =0
(1 − x ) (1 − y ) (1 − z )
1 1 1
⇒ −1 + + + =0 1 1 1
(1 − cosα ) (1 − cosβ ) (1 − cos γ ) ⇒ + + =1
1 − x 1 −y 1 − z
1 1 1
+ + =1
1 − cosα 1 − cosβ 1 − cos γ l Ex. 78 If a, b and c be any three non-coplanar vectors,
2α 2β γ
⇒ cosec + cosec + cosec = 2, which is not possible.
2
then prove that the points l1 a + m1b + n1c, l 2 a + m 2 b + n 2 c,
2 2 2
l3 a + m3 b + n3 c and l 4 a + m 4 b + n 4 c are coplanar, if
α β
As, cosec 2 ≥ 1, cosec 2 ≥ 1 l1 m1 n1 1
2 2
γ l 2 m2 n 2 1
and cosec 2 ≥ 1 =0
2 l3 m3 n3 1
Q They cannot be coplanar. l 4 m4 n 4 1
Sol. We know that, four points having position vectors, a, b, c and
l Ex. 77 If the vectors x $i + $j + k$ , $i + y$j + k$ and $i + $j + zk$ d are coplanar, if there exists scalars x, y , z and t such that
are coplanar where,x ≠1 , y ≠1 and z ≠1, then prove that xa + yb + zc + td = 0 where, x + y + z + t = 0
1 1 1
+ + =1 So, the given points will be coplanar, if there exists scalars
1− x 1−y 1− z x, y , z and t such that
Sol. The vectors are coplanar, if we can find two scalars λ and µ x(l1a + m1b + n1c ) + y (l 2a + m2b + n2c ) + z (l 3a + m3b + n3c)
such that + t (l 4a + m4 b + n4 c) = 0
( x$i + $j + k$ ) = λ ( $i + y$j + k$ ) + µ( $i + $j + zk$ ) where, x + y + z + t = 0
40 Textbook of Vector & 3D Geometry

⇒(l1x + l 2y + l 3z + l 4t ) a + (m1x + m2y + m3z + m4t ) b Now, if A, B and C are collinear points, then AB and AC are in
+ (n1x + n2y + n3z + n4t ) c = 0 the same line and BC = λ ( AC )
where, x+y +z +t=0 ⇒ ( r3 − r2 ) = λ ( r2 − r1 )
l1x + l 2y + l 3z + l 4t = 0 …(i) ⇒ r3 = − λr1 + ( λ + 1 ) r2
m1x + m2y + m3z + m4t = 0 …(ii) ⇒ r3 = − λr1 + mr2
n1x + n2y + n3z + n4t = 0 …(iii) where, l = − λ and m = λ + 1
and x+y +z +t=0 …(iv) ⇒ l + m = − λ + (λ + 1) = 1
Eliminating x, y , z and t from above equations, we get
l1 l 2 l 3 l 4
l Ex. 80 Show that points with position vectors
a − 2b + 3c, −2 a + 3b − c and 4 a − 7b + 7c are collinear. It is
m1 m2 m3 m4
=0 given that vectors a, b and c and non-coplanar.
n1 n2 n3 n4
Sol. The three points are collinear, if we can find λ 1, λ 2 and λ 3,
1 1 1 1
such that
λ 1 (a − 2 b + 3 c) + λ 2 ( −2 a + 3 b − c) + λ 3
l Ex. 79 If r1 , r2 and r3 are the position vectors of three ( 4 a − 7 b + 7 c) = 0 with λ 1 + λ 2 + λ 3 = 0
collinear points and scalars l and m exists such that On equating the coefficients a, b and c separately to zero, we
r3 = l r1 + mr2 , then show that l + m =1. get λ 1 − 2 λ 2 + 4 λ 3 = 0, − 2 λ 1 + 3 λ 2 − 7 λ 3 = 0 and
Sol. Let A, B and C be the three points whose position vectors 3λ1 − λ 2 + 7λ 3 = 0
referred to O are r1, r2 and r3, respectively. On solving we get λ 1 = − 2, λ 2 = 1, λ 3 = 1
AB = OB − OA = r2 − r1 So that, λ 1 + λ 2 + λ 3 = 0
BC = OC − OB = r3 − r2 Hence, the given vectors are collinear.
#L
Vector Algebra Exercise 1 :
Single Option Correct Type Questions
1. If a = 3 i$ − 2$j + k$ , b = 2$i − 4 $j − 3k$ and c = − $i + 2$j + 2k$ , 9. If O is the origin and the position vector of A is 4 $i + 5$j ,
then a + b + c is then a unit vector parallel to OA is
(a) 3 i$ − 4 $j (b) 3 i$ + 4 $j (a)
4 $
i (b)
5 $
i
(c) 4 i$ − 4 $j (d) 4 i$ + 4 $j 41 41
1 1
2. What should be added in vector a = 3 i$ + 4 $j − 2 k$ to get (c) ( 4 i$ + 5 $j) (d) ( 4 i$ − 5 $j)
41 41
its resultant a unit vector $i ?
(a) − 2 i$ − 4 $j + 2 k$ (b) − 2 i$ + 4 $j − 2 k$ 10. The position vectors of the points A, B and C are
$i + 2$j − k$ , $i + $j + k$ and 2$i + 3$j + 2k$ , respectively. If A is
(c) 2 $i + 4 $j − 2 k$ (d) None of these
chosen as the origin, then the position vectors of B and
3. If a = 2$i + $j − 8k$ and b = $i + 3$j − 4 k$ , then the magnitude
C are
of a + b is equal to (a) $i + 2 k$ , $i + $j + 3 k$ (b) $j + 2 k$ , $i + $j + 3 k$
13
(a) 13 (b) (c) − $j + 2 k$ , $i − $j + 3 k$ (d) − $j + 2 k$ , $i + $j + 3 k$
3
(b)
3
(d)
4 11. The position vectors of P and Q are 5$i + 4 $j + ak$ and
13 13
− $i + 2$j − 2k$ , respectively. If the distance between them
4. If a = 2$i + 5$j and b = 2$i − $j , then the unit vector along is 7, then the value of a will be
a + b will be (a) −5, 1 (b) 5, 1
i$ − $j (c) 0, 5 (d) 1, 0
(a) (b) i$ + $j
2
12. If position vector of points A, B and C are respectively i$, $j
i$ + $j
(c) 2( $i + $j) (d) and k$ and AB = CX , then position vector of point X is
2
(a) − i$ + $j + k$ (b) i$ − $j + k$
5. The unit vector parallel to the resultant vector of (c) $i + $j − k$ (d) $i + $j + k$
2$i + 4 $j − 5k$ and $i + 2$j + 3k$ is
1 13. The position vectors of A and B are 2$i − 9 $j − 4 k$ and
(a) (3 $i + 6 $j − 2 k$ )
7 6$i − 3$j + 8k$ respectively, then the magnitude of AB is
i$ + $j + k$ (a) 11 (b) 12
(b)
3 (c) 13 (d) 14
$i + $j + 2 k$
(c) 14. If the position vectors of P and Q are ( $i + 3$j − 7 k$ ) and
6
1 (5$i − 2$j + 4 k$ ), then | PQ | is
(d) ( − i$ − $j + 8 k$ ) (a) 158 (b) 160
69
(c) 161 (d) 162
6. If a = $i + 2$j + 3k$ , b = − $i + 2$j + k$ and c = 3$i + $j , then the
unit vector along its resultant is
15. If the position vectors of P and Q are $i + 2$j − 7 k$ and
3 i$ + 5 $j + 4 k$ 5$i − 3$j + 4 k$ respectively, the cosine of the angle between
(a) 3 $i + 5 $j + 4 k$ (b)
50 PQ and Z-axis is
3 i$ + 5 $j + 4 k$ (a)
4
(b)
11
(c) (d) None of these
5 2 162 162
5 −5
7. If a = (2, 5) and b = (1, 4 ), then the vector parallel to (c)
162
(d)
162
( a + b) is
(a) (3, 5) (b) (1, 1) 16. If the position vectors of A and B are i$ + 3$j − 7 k$ and
(c) (1, 3) (d) (8, 5) 5$i − 2$j + 4 k$ , then the direction cosine of AB along Y -axis
8. In the ∆ABC, AB = a, AC = c and BC = b , then is
4 5
(a) a + b + c = 0 (b) a + b − c = 0 (a) (b) −
162 162
(c) a − b + c = 0 (d) − a + b + c = 0
(c) − 5 (d) 11
42 Textbook of Vector & 3D Geometry

17. The direction cosines of vector a = 3$i + 4 $j + 5k$ in the 24. The position vector of the points which divides
direction of positive axis of X , is internally in the ratio 2 : 3 the join of the points 2a − 3 b
3 4 and 3a − 2 b , is
(a) ± (b) 12 13 12 13
50 50 (a) a+ b (b) a− b
3 4 5 5 5 5
(c) (d) − 3 2
50 50 (c) a − b (d) None of these
5 5
18. The direction cosines of the vector 3$i − 4 $j + 5k$ are
3 4 1 3 −4 1 25. If O is origin and C is the mid-point of A (2, − 1) and
(a) , − , (b) , , B ( −4, 3). Then, value of OC is
5 5 5 5 2 5 2 2
3 −4 1 3 4 1 (a) i$ + $j (b) i$ − $j
(c) , , (d) , , (c) − i$ + $j (d) − i$ − $j
2 2 2 5 2 5 2 2
19. The point having position vectors 2$i + 3$j + 4 k$ , 26. If the position vectors of the points A and B are
$i + 3$j − k$ and 3$i − $j − 3k$ , then what will be the position
3$i + 4 $j + 2k$ and 4 $i + 2$j + 3k$ are the vertices of
vector of the mid-point of AB
(a) right angled triangle
(a) i$ + 2 $j − k$ (b) 2 i$ + $j − 2 k$
(b) isosceles triangle
(c) equilateral triangle (c) 2 i$ + $j − k$ (d) i$ + $j − 2 k$
(d) collinear
27. The position vectors of A and B are $j − $j + 2k$ and
20. If the position vectors of the vertices A, B and C of a 3$i − $j + 3k$ . The position vector of the middle point of
∆ABC are 7 $j + 10k, − $i + 6$j + 6k$ and − 4 $i + 9 $j + 6k,
$
the line AB is
respectively. The triangle is 1 1 5$
(a) i$ − $j + k$ (b) 2 i$ − $j + k
(a) equilateral 2 2 2
(b) isosceles 3$ 1$ 3 $
(c) i − j + k (d) None of these
(c) scalene 2 2 2
(d) right angled and isosceles also
28. If the vector bis collinear with the vector a = (2 2, − 1, 4 )
21. If a, b and c are the position vectors of the vertices A, B and | b| = 10, then
and C of the ∆ABC, then the centroid of ∆ABC is (a) a ± b = 0 (b) a ± 2 b = 0
a + b+ c 1 b + c (c) 2 a ± b = 0 (d) None of these
(a) (b)  a + 
3 2 2 
29. If a = (1,−1) and b = ( −2, m ) are two collinear vectors,
b+ c a + b+ c
(c) a + (d) then m is equal to
2 2
(a) 4 (b) 3
22. If in the given figure, OA = a, OB = b and AP : PB = m : n, (c) 2 (d) 0
then OP is equal to 30. The points with position vectors 10$i + 3$j, 12$i − 5$j and
P
A B a$i + 11$j are collinear, if a is equal to
(a) −8 (b) 4
(c) 8 (d) 12
31. The vectors $i + 2$j + 3k$ , λ$i + 4 $j + 7 k$ , − 3$i − 2$j − 5k$ are
O collinear, if λ is equal to
ma + nb na + mb (a) 3 (b) 4
(a) (b) (c) 5 (d) 6
m+n m+n
(c) ma − nb (d)
ma − nb 32. If the points a + b, a − b and a + kb be collinear, then k is
m −n equal to
23. If a and bare position vector of two points A, B and C (a) 0 (b) 2
(c) −2 (d) Any real number
divides AB in ratio 2 : 1, then position vector of C is
a + 2b 2a + b 33. If the position vectors of A, B, C and D are
(a) (b)
3 3 2$i + $j, i$ − 3$j, 3$i + 2$j and $i + λ$j respectively and AB ||CD,
a+2 a+ b
(c) (d) then λ will be
3 2
(a) −8 (b) −6
(c) 8 (d) 6
Chap 01 Vector Algebra 43

34. If the vectors 3$i + 2$j − k$ and 6$i − 4 x$j + yk$ are parallel, 43. If OP = 8 and OP makes angles 45° and 60° with OX -axis
then the value of x and y will be and OY -axis respectively, then OP is equal to
(a) −1, − 2 (b) 1, − 2 (a) 8( 2 $i + $j ± k$ ) (b) 4( 2 $i + $j ± k$ )
(c) −1, 2 (d) 1, 2 1 1
(c) ( 2 $i + $j ± k$ ) (d) ( 2 $i + $j ± k$ )
4 8
35. If a and b are two non-collinear vectors and xa + yb = 0
(a) x = 0, but y is not necessarily zero 44. Let a, b and c be three units vectors such that
(b) y = 0, but x is not necessarily zero 3a + 4 b + 5c = 0. Then which of the following statements
(c) x = 0, y = 0 is true?
(d) None of the above (a) a is parallel to b
36. Four non-zero vectors will always be (b) a is perpendicular to b
(a) linearly dependent (c) a is neither parallel nor perpendicular to b
(b) linearly independent (d) None of the above
(c) either (a) or (b) 45. A, B, C , D and E are five coplanar points, then
(d) None of the above DA + DB + DC + AE + BE + CE is equal to
37. The vectors a, b and a + b are (a) DE (b) 3DE
(a) collinear (b) coplanar (c) 2DE (d) 4ED
(c) non-coplanar (d) None of these 46. If the vectors a and b are linearly independent satisfying
38. If ( x , y, z ) ≠ (0, 0, 0) and ( $i + $j + 3k$ )x + (3$i − 3$j + k$ )y ( 3 tan θ + 1) a + ( 3 sec θ − 2) b = 0, then the most
+ ( − 4 $i + 5$j)z = λ( x$i + y$j + zk$ ), then the value of λ general value of θ are
π 11π
will be (a) nπ − ,n ∈ Z (b) 2nπ ± ,n ∈ Z
6 6
(a) − 2, 0 (b) 0, − 2 π 11π
(c) − 1, 0 (d) 0, − 1 (c) nπ ± ,n ∈ Z (d) 2nπ + ,n ∈ Z
6 6
39. The number of integral values of p for which 47. The unit vector bisecting OY and OZ is
( p + 1)i$ − 3$j + pi$, pi$ + ( p + 1)$j − 3k$ and $i + $j + k$ $j − k$
(a) (b)
− 3i$ + p$j + ( p + 1)k$ are linearly dependent vectors is 3 2
$j + k$ − $j + k$
(a) 0 (b) 1 (c) (d)
(c) 2 (d) 3 2 2

40. The vectors AB = 3$i + 4 k$ and AC = 5$i − 2$j + 4 k$ are the 48. A line passes through the points whose position vectors
sides of a ∆ABC. The length of the median through A is are $i + $j − 2k$ and $i − 3$j + k$ . The position vector of a
(a) 18 (b) 72 point on it at unit distance from the first point is
(c) 33 (d) 288 1 $ $ 1 $
(a) (5 i + j − 7 k$ ) (b) ( 4 i + 9 $j − 15 k$ )
5 5
41. In the figure, a vectors x satisfies the equation x − w = v. 1
Then, x is equal to (c) ( i$ − 4 $j + 3 k$ ) (d) ( i$ − 4 $j + 3 k$ )
5
A
49. If D, E and F are the middle points of the sides BC , CA
a c and AB of the ∆ABC, then AD + BE + CF is
b
(a) a zero vector (b) a unit vector
(c) 0 (d) None of these
B w v C
D 50. If P and Q are the middle points of the sides BC and CD
(a) 2a + b + c (b) a + 2 b + c of the parallelogram ABCD, then AP + AQ is equal to
(c) a + b + 2 c (d) a + b + c 1
(a) AC (b) AC
2
42. Vectors a = $i + 2$j + 3k$ , b = 2$i − $j + k$ and c = 3$i + $j + 4 k$ 2 3
are so placed that the end point of one vector is the (b) AC (d) AC
3 2
starting point of the next vector. Then the vectors are
(a) not coplanar 51. The figure formed by the four points $i + $j − k$ , 2$i + 3$j ,
(b) coplanar but cannot form a triangle 3$i + 5$j − 2k$ and k$ − $j is
(c) coplanar and form a triangle (a) rectangle (b) parallelogram
(d) coplanar and can form a right angled triangle (c) trapezium (d) None of these
44 Textbook of Vector & 3D Geometry

52. A and B are two points. The position vector of A is (a) a = b = c = 0


6 b − 2a. A point P divides the line AB in the ratio 1 : 2. If (b) any two of a,b and c are zero
a − b is the position vector of P, then the position vector (c) any one of a,b and c is zero
of B is given by (d) a + b + c = 0
(a) 7 a − 15 b 61. If a and b are two non-zero and non-collinear vectors,
(b) 7 a + 15 b then a + b and a − b are
(c) 15a − 7 b (a) linearly dependent vectors
(d) 15a + 7 b (b) linearly independent vectors
53. If three points A, B and C are collinear, whose position (c) linearly dependent and independent vectors
(d) None of the above
vectors are $i − 2$j − 8k$ , 5$i − 2k$ and 11$i + 3$j + 7 k$
respectively, then the ratio in which B divides AC is
62. If | a + b | < | a − b |, then the angle between a and b can
(a) 1 : 2 (b) 2 : 3 lie in the interval.
(c) 2 : 1 (d) 1 : 1 (a) ( −π / 2, π / 2 ) (b) ( 0, π )
(c) ( π / 2, 3 π / 2 ) (d) ( 0, 2π )
54. If in a triangle, AB = a, AC = b and D, E are the
mid-points of AB and AC respectively, then DE is equal
63. The magnitudes of mutually perpendicular forces a, b
to and c are 2, 10 and 11 respectively. Then the magnitude
a b a b of its resultant is
(a) − (b) − (a) 12 (b) 15
4 4 2 2
b a b a (c) 9 (d) None of these
(c) − (d) −
4 4 2 2 64. If $i − 3$j + k$ bisects the angle between a and − $i + 2$j + 2k$ ,
55. If ABCD is parallelogram, AB = 2$i + 4 $j − 5k$ and where a is a unit vector, then
1
AD = $i + 2$j + 3k$ , then the unit vectors in the direction of (a) a = ( 41 $i + 88 $j − 40 k$ )
105
BD is 1
1 $ 1 $ (b) a = ( 41 $i + 88 $j + 40 k$ )
(a) ( i + 2 $j − 8 k$ ) (b) ( i + 2 $j − 8 k$ ) 105
69 69 1
1 1 (c) a = ( −41 $i + 88 $j − 40 k$ )
(c) ( − i$ − 2 $j + 8 k$ ) (d) ( − i$ − 2 $j + 8 k$ ) 105
69 69 1
(d) a = ( 41 $i − 88 $j − 40 k$ )
56. If A, B and C are the vertices of a triangle whose position 105

vectors are a, b and c and G is the centroid of the ∆ABC, 65. Let a = $i be a vector which makes an angle of 120° with a
then GA + GB + GC is unit vector b. Then, the unit vector ( a + b) is
(a) 0 (b) A + B + C 1 3$ 3 $ 1$
a + b+ c a + b− c (a) − $i + j (b) − i+ j
(c) (d) 2 2 2 2
3 3 1 3$ 3 $ 1$
(c) $i + j (d) i− j
57. If ABCDEF is regular hexagon, then AD + EB + FC is 2 2 2 2
equal to 66. Given three vectors a = 6$i − 3$j, b = 2$i − 6$j and
(a) 0 (b) 2AB
c = −2$i + 21$j such that α = a + b + c. Then, the resolution
(c) 3AB (d) 4 AB
of the vector α into components with respect to a and b
58. ABCDE is a pentagon. Forces AB, AE, DC and ED act at is given by
a point. Which force should be added to this system to (a) 3a − 2 b (b) 3 b − 2a
make the resultant 2AC? (c) 2a − 3 b (d) a − 2 b
(a) AC (b) AD 67. ‘ I ’ is the incentre of ∆ABC whose corresponding sides
(c) BC (d) BD
are a, b, c respectively. a IA + b IB + c IC is always equal to
59. If ABCDEF is a regular hexagon and (a) 0 (b) (a + b + c ) BC
AB + AC + AD + AE+ AF = λAD, then λ is equal to (c) (a + b + c ) AC (d) (a + b + c ) AB
(a) 2 (b) 3 68. If x and y are two non-collinear vectors and ABC is a
(c) 4 (d) 6
triangle with side lengths a, b and c satisfying (20a − 15b )x
60. Let us define the length of a vector a$i + b$j + ck$ as +(15b − 12c )y + (12c − 20a )( x × y ) = 0, then ∆ABC is
| a | + | b | + | c |. This definition coincides with the usual (a) an acute angled triangle (b) an obtuse angled triangle
definition of length of a vector a$i + b$j + ck$ if and only if (c) a right angled triangle (d) a scalene triangle
Chap 01 Vector Algebra 45

69. If x and y are two non-collinear vectors and a, b and c 76. If a, b and c are three non-coplanar vectors such that
represent the sides of a ∆ABC satisfying a + b + c = αd and b + c + d = βa, then a + b + c + d is
(a − b )x + (b − c )y + (c − a )( x × y ) = 0, then ∆ABC is equal to
(where x × y is perpendicular to the plane of x and y) (a) 0 (b) αa
(a) an acute angled triangle (c) βb (d) (α + β ) c
(b) an obtuse angled triangle 77. The position vectors of the points P and Q with respect
(c) a right angled triangle
(d) a scalene triangle to the origin O are a = $i + 3$j − 2k$ and b = 3$i − $j − 2k$ ,
respectively. If M is a point on PQ, such that OM is the
70. If the resultant of two forces is of magnitude P and equal bisector of POQ, then OM is
to one of them and perpendicular to it, then the other (a) 2( i$ − $j + k$ ) (b) 2 i$ + $j − 2 k$
force is
(c) 2( − $i + $j − k$ ) (b) 2( $i + $j + k$ )
(a) P 2 (b) P
(c) P 3 (d) None of these 78. ABCD is a quadrilateral. E is the point of intersection of
the line joining the mid-points of the opposite sides. If O
71. If b is a vector whose initial point divides the join of 5$i
is any point and OA + OB + OC + OD = xOE, then x is
and 5$j in the ratio k : 1 and whose terminal point in the
equal to
origin and | b | ≤ 37, then k lies in the interval (a) 3 (b) 9
(a) [ −6, − 1 / 6 ] (b) [ −∞, − 6 ] ∪ [ −1 / 6, ∞ ] (c) 7 (d) 4
(c) [ 0, 6 ] (d) None of these
79. In the ∆OAB, M is the mid-point of AB, C is a point on
72. If 4 $j + 7 $j + 8k$ , 2i$ + 3$j + 4 k$ and 2i$ + 5$j + 7 k$ are the OM, such that 2OC = CM. X is a point on the side OB
position vectors of the vertices A, B and C respectively such that OX = 2XB. The line XC is produced to meet
of ∆ABC. The position vector of the point where the OA in Y . Then,
OY
is equal to
bisector of ∠A meets BC is YA
1 2 1 2
(a) (6 i$ + 13 $j + 18 k$ ) (b) (6 i$ + 12 $j − 8 k$ ) (a) (b)
3 3 3 7
1 2 3 2
(c) ( −6 $i − 8 $j − 9 k$ ) (d) ( −6 $i − 12 $j + 8 k$ ) (c) (d)
3 3 2 5
73. If a and b are two unit vectors and θ is the angle 80. Points X andY are taken on the sides QR and RS,
between them, then the unit vector along the angular respectively of a parallelogram PQRS, so that QX = 4 XR
bisector of a and b will be given by and RY = 4 YS. The line XY cuts the line PR at Z.
a−b a+b Then, PZ is
(a) (b)
2 cos (θ / 2 ) 2 cos (θ / 2 ) 21 16
(a) PR (b) PR
a−b 25 25
(c) (d) None of these
cos (θ / 2 ) 17
(c) PR (d) None of these
25
74. A, B, C and D have position vectors a, b, c and d,
81. Find the value of λ so that the points P, Q, R and S on the
respectively, such that a − b = 2( d − c). Then,
(a) AB and CD bisect each other sides OA, OB, OC and AB, respectively, of a regular
(b) BD and AC bisect each other tetrahedron OABC are coplanar. It is given that
OP 1 OQ 1 OR 1 OS
(c) AB and CD trisect each other = , = , = and = λ.
(d) BD and AC trisect each other OA 3 OB 2 OC 3 AB
1
75. On the xy plane where O is the origin, given points, (a) λ = (b) λ = −1
2
A(1, 0), B(0, 1) and C(1, 1). Let P, Q and R be moving point (c) λ = 0 (d) for no value of λ
on the line OA, OB, OC respectively such that
82. OABCDE is a regular hexagon of side 2 units in the
OP = 45t( OA ), OQ = 60t( OB), OR = (1 − t )( OC) with
XY -plane. O being the origin and OA taken along the
t > 0. If the three points P, Q and R are collinear, then the
X -axis. A point P is taken on a line parallel to Z-axis
value of t is equal to
1 7 through the centre of the hexagon at a distance of
(a) (b) 3 units from O. Then, the vector AP is
106 187
1 (a) − $i + 3 $j + 5 k$ (b) $i − 3 $j + 5 k$
(c) (d) None of these
100 (c) − $i + 3 $j + 5 k$ (d) $i + 3 $j + 5 k$
46 Textbook of Vector & 3D Geometry

#L
Vector Algebra Exercise 2 :
More than One Option Correct Type Questions
83. If the vectors $i − $j, $j + k$ and a form a triangle, then a 1
(a) (3 i$ + 6 $j − 2 k$ )
1
(b) (3 i$ − 6 $j − 2 k$ )
may be 7 7
1 $ 1
(a) − i$ − k$ (b) i$ − 2 $j − k$ (c) ( i + 2 $j + 8 k$ ) (d) ( − i$ − 2 $j + 8 k$ )
69 69
(c) 2 $j + $j + k$ (d) $i + k$
87. If A( −4, 0, 3) and B (14, 2, − 5), then which one of the
84. If the resultant of three forces
following points lie on the bisector of the angle between
F1 = pi$ + 3$j − k$ , F2 = 6$i − k$ and F3 = − 5$i + $j + 2k$
OA and OB (O is the origin of reference)?
acting on a particle has a magnitude equal to 5 units, (a) (2, 2, 4 ) (b) (2, 11, 5 )
then the value of p is (c) ( −3, − 3, − 6 ) (d) (1, 1, 2 )
(a) −6 (b) −4
(c) 2 (d) 4 88. If point $i + $j, $i − $j and p$i + q $j + r k$ are collinear, then
(a) p = 1 (b) r = 0
85. Let ABC be a triangle, the position vectors of whose (c) q ∈ R (d) q ≠ 1
vertices are 7 $j + 10k$ , − $i + 6$j + 6k$ and − 4 $i + 9 $j + 6k.
$
89. If a, b and c are non-coplanar vectors and λ is a real
Then, ∆ABC is
(a) isosceles (b) equilateral number, then the vectors a + 2b + 3c, λb + µc and (2λ − 1)c
(c) right angled (d) None of these are coplanar when
1
86. The sides of a parallelogram are 2$i + 4 $j − 5k$ and (a) µ ∈ R (b) λ =
2
$i + 2$j + 3k$ . The unit vector parallel to one of the (c) λ = 0 (d) no value of λ
diagonals is

#L
Vector Algebra Exercise 3 :
Statement I and II Type Questions
n
Directions (Q. Nos. 90-92) This section is based on 91. Statement I a = $i + p$j + 2k$ and b = 2$i + 3$j + qk$ are
Statement I and Statement II. Select the correct answer 3
from the codes given below. parallel vectors, if p = and q = 4.
2
(a) Both Statement I and Statement II are correct and
Statement II is the correct explanation of Statement I Statement II a = a1 i + a 2 $j + a 3 k$ and b = b1 $i + b 2 $j + b 3 k$
$
(b) Both Statement I and Statement II are correct but a a a
are parallel 1 = 2 = 3 .
Statement II is not the correct explanation of Statement I b1 b 2 b 3
(c) Statement I is correct but Statement II is incorrect
92. Statement I If three points P, Q and R have position
(d) Statement II is correct but Statement I is incorrect
vectors a, b and c respectively, and 2 a + 3b − 5c = 0, then
90. Statement I In ∆ABC, AB + BC + CA = 0 the points P, Q and R must be collinear.
Statement II If OA = a, OB = b, then AB = a + b
Statement II If for three points A, B and C, AB = λ AC,
then points A, B and C must be collinear.
Chap 01 Vector Algebra 47

#L
Vector Algebra Exercise 4 :
Passage based Type Questions
Passage I 99. B divided AC in ratio
(Q. Nos. 93 and 94) (a) 2 :1 (b) 2 : 3
n Let OABCD be a pentagon in which the sides OA and CB (c) 2 : −3 (d) 1 : 2
are parallel and the sides OD and AB are parallel.
Also, OA : CB = 2 : 1 and OD : AB =1 : 3.
Passage IV
(Q. Nos. 100 and 101)
C B
n If two vectors OA and OB are there, then their resultant
D
OA + OB can be found by completing the parallelogram
X
OACB and OC = OA + OB. Also, If | OA| = | OB|, then the
A resultant will bisect the angle between them.
O

OX 100. A vector C directed along internal bisector of angle


93. The ratio is between vectors A = 7 $i − 4 $j − 4 k$ and B = − 2i$ − $j + 2k$
XC
(a) 3/4 (b) 1/3 with |C| = 5 6 is
(c) 2/5 (d) 1/2 5 5
(a) ( $i − $j + k$ ) (b) ( $i − 7 $j + 2 k$ )
AX 3 3
94. The ratio is 5 $ 5
XD (c) (5 i + 5 $j + 2 k$ ) (d) ( −5 $i + 5 $j + 2 k$ )
3 3
(a) 5/2 (b) 6
(c) 7/3 (d) 4 101. If internal and external bisectors of ∠A of ∆ABC meet
the base BC at D and E respectively, then (D and E lie on
Passage II same side of B)
(Q. Nos. 95 and 96) BD + BE
(a) BC = (b) BC 2 = BD × DE
n
Consider the regular hexagon ABCDEF with centre at O 4
(origin). 2 1 1
(c) = + (d) None of these
BC BD BE
95. AD + EB + FC is equal to
(a) 2AB (b) 3AB Passage V
(c) 4AB (d) None of these
(Q. Nos. 102 and 103)
96. Five forces AB, AC, AD, AE, AF act at the vertex A of a
regular hexagon ABCDEF. Then, their resultant is
n
Let C : r (t ) = x (t ) $i + y (t ) $j + z (t )k$ be a differentiable
(a) 3AO (b) 2AO r (t + h ) − r (h )
curve, i.e. lim exist for all t,
(c) 4AO (d) 6AO x→ 0 h
∴ r ′ (t ) = x ′ (t ) $i + y ′ (t ) $j + z ′ (t ) k$
Passage III
If r ′ (t ) , is tangent to the curve C at the point
(Q. Nos. 97 to 99)
P [ x (t ), y (t ), z (t )] and r ′ (t ) points in the direction of
n
Three points A, B and C have position vectors increasing t.
−2 a + 3b + 5c, a + 2b + 3c and 7a − c with reference to an
origin O. Answer the following questions.
102. The point P on the curve r (t ) = (1 − 2t )$i + t 2 $j + 2e 2 (t −1) k$
at which the tangent vector r ′ (t ) is parallel to the radius
97. Which of the following is true? vector r (t ) is
(a) AC = 2 AB (b) AC = −3 AB (a) ( − 1, 1, 2 ) (b) (1, − 1, 2 )
(c) AC = 3 AB (d) None of these (c) ( − 1, 1, − 2 ) (d) (1, 1, 2 )
98. Which of the following is true? 103. The tangent vector to r (t ) = 2t 2 $i + (1 − t )$j + (3t 2 + 2)k$
(a) 2 OA − 3 OB + OC = 0
(b) 2 OA + 7 OB + 9 OC = 0
at (2, 0, 5) is
(c) OA + OB + OC = 0 (a) 4 i$ + $j − 6 k$ (b) 4 i$ − $j + 6 k$
(d) None of the above (c) 2 $i − $j + 6 k$ (d) 2 $i + $j − 6 k$
48 Textbook of Vector & 3D Geometry

#L
Vector Algebra Exercise 5 :
Matching Type Question
104. a and b form the consecutive sides of a regular hexagon ABCDEF.

Column I Column II

a. If CD = xa + yb, then p. x = −2
b. If CE = xa + yb, then q. x = −1
c. If AE = xa + yb, then r. y=1
d. If AD = −xb, then s. y=2

#L
Vector Algebra Exercise 6 :
Single Integer Answer Type Questions
105. If the resultant of three forces F1 = p$i + 3$j − k$ , 109. Let p be the position vector of orthocentre and g is the
F2 = −5$i + $j + 2k$ and F3 = 6i$ − k$ acting on a particle has position vector of the centroid of ∆ABC, where
a magnitude equal to 5 units. Then, what is difference in circumcentre is the origin. If p = kg, then the value of k is
the values of p? 110. In a ∆ABC, a line is drawn passing through centroid
106. Vectors along the adjacent sides of parallelogram are dividing AB internally in ratio 2 : 1 and AC in λ : 1
a = $i + 2$j + k$ and b = 2$i + 4 $j + k$ . Find the length of the (internally). The value of λ is
longer diagonal of the parallelogram. 111. A vector a has component 2p and 1 with respect to a
107. If vectors a = $i + 2$j − k$ , b = 2$i − $j + k$ and c = λi$ + $j + 2 k$ rectangular cartesian system. The system is rotated
through a certain angle about the origin in the counter
are coplanar, then find the value of ( λ − 4 ). clockwise sense. If with to the new system, a has
108. If a + b is along the angle bisector of a and b, where components ( p + 1) and 1, where p take the values p 1 and
| a | = λ | b |, then the number of digits in value of λ is p 2 . Then, the value of 3 | p 1 + p 2 | is

Vector Algebra Exercise 7 :


Subjective Type Questions
112. A vector a has components a1 , a 2 and a 3 in a right 114. Let OACB be a parallelogram with O at the origin and
handed rectangular cartesian system OXYZ. The OC a diagonal. Let D be the mid-point of OA. Using
coordinate system is rotated about Z-axis through angle vector methods prove that BD and CO intersects in the
π same ratio. Determine this ratio.
. Find components of a in the new system.
2 115. ∆ABC is a triangle with the point P on side BC such that
113. Find the magnitude and direction of r1 − r2 when | r1 | = 5 3BP = 2PC, the point Q is on the line CA such that
4CO = QA. Find the ratio in which the line joining the
and points North-East while | r2 | = 5 but points
common point R of AP and BQ and the point S divides
North-West.
AB.
Chap 01 Vector Algebra 49

116. In a ∆ABC internal angle bisectors AI , BI and CI are 118. Let OABCD be a pentagon in which the sides OA and CB
produced to meet opposite sides in A ′ , B ′ and C ′, are parallel and the sides OD and AB are parallel as
respectively. Prove that the maximum value of shown in figure. Also, OA : CB = 2 : 1 and OD : AB = 1 : 3.
AI ⋅ BI ⋅ CI 8 If the diagonals OC and AD meet at x, find OX : OC .
is .
AA ′ ⋅ BB ′ ⋅ CC ′ 27 119. If u, v and w is a linearly independent system of vectors,
117. Let r1 , r2 , r3 ,....., rn be the position vectors of points examine the system p, q and r, where
p = (cos a )u + (cos b )v + (cos c )w
P1 , P2 , P3 ,....., Pn relative to an origin O. Show that if the
q = (sin a )u + (sin b )v + (sin c )w
vectors equation a1r1 + a 2 r2 + K + an rn = 0 holds, then a
r = sin( x + a )u + sin( x + b )v + sin( x + c )w for linearly
similar equation will also hold good with respect to any
dependent.
other origin O′, if a1 + a 2 + a 3 + .... + an = 0.

#L
Vector Algebra Exercise 8 :
Questions Asked in Previous Years Exam
120. If the vectors AB = 3$i + 4 k^ and AC = 5$i − 2$j + 4 k$ are 125. If a, b, c are non-coplanar vectors and λ is a real number,
the sides of a ∆ABC, then the length of the median then the vectors a + 2b + 3c, λb + 4 c and (2λ − 1)c are
through A is [JEE Main 2013, 2003] non-coplanar for [AIEEE 2004]
(a) all values of λ
(a) 18 (b) 72
(b) all except one value of λ
(c) 33 (d) 45 (c) all except two values of λ
121. Let a, b and c be three non-zero vectors which are (d) no value of λ
pairwise non-collinear. If a + 3b is collinear with c and 126. Consider points A, B, C and D with position vectors
b + 2c is collinear with a, then a + 3b + 6 c is [AIEEE 2011] 7 $i − 4 $j + 7 k$ , $i − 6$j + 10k$ , − $i − 3$j + 4 k$ and 5$i − $j + 5k$ ,
(a) a + c (b) a
respectively. Then, ABCD is a [AIEEE 2003]
(c) c (d) 0
(a) square (b) rhombus
122. The non-zero vectors a, b and c are related by a = 8b and (c) rectangle (d) None of these
c = − 7 b. Then, the angle between a and c is a a2 1+a3
[AIEEE 2008] 127. If b b 2 1 + b 3 = 0 and vectors (1, a, a 2 ), (1, b, b 2 ) and
(a) π (b) 0
π π c c2 1+c 3
(c) (d)
4 2 (1, c , c 2 ) are non-coplanar, then the product abc equal to
123. If C is the mid-point of AB and P is any point outside AB, [AIEEE 2003]
then [AIEEE 2005] (a) 2 (b) −1
(a) PA + PB + PC = 0 (b) PA + PB + 2 PC = 0 (c) 1 (d) 0
(c) PA + PB = PC (d) PA + PB = 2 PC
128. The vector $i + x$j + 3k$ is rotated through an angle θ and
124. If a, b and c are three non-zero vectors such that no two doubled in magnitude, then it becomes
of these are collinear. If the vector a + 2b is collinear 4 $i + ( 4 x − 2)$j + 2k$ . The value of x are [AIEEE 2002]
with c and b + 3c is collinear with a (λ being some
 2  1 
non-zero scalar), then a + 2b + 6 c is equal to (a) − , 2  (b)  , 2
 3  3 
(a) λa (b) λb [AIEEE 2004]
(c) λc 2 
(d) 0 (c)  , 0  (d) {2, 7}
3 
50 Textbook of Vector & 3D Geometry

Answers
Exercise for Session 1 Exercise for Session 3
1. (i) vector (ii) scalar (iii) vector (iv) scalar 4. (2,−3) 5. a − 2b = 1
2. 1
N 6. x = 7. 0
3
70
km
Chapter Exercises
40° 1. (c) 2. (a) 3. (a) 4. (d) 5. (a) 6. (c)
W E
O
7. (c) 8. (b) 9. (c) 10. (d) 11. (a) 12. (a)
km

30° 13. (d) 14. (d) 15. (b) 16. (b) 17. (c) 18. (b)
40

19. (c) 20. (d) 21. (a) 22. (b) 23. (a) 24. (b)
S
25. (c) 26. (b) 27. (b) 28. (c) 29. (c) 30. (c)
3. (i) a , d ; b, x , z; c, y (ii) b, x ; a , d; c, y
(iii) a , y , z (iv) b, z ; x , z 31. (a) 32. (d) 33. (b) 34. (a) 35. (c) 36. (a)

4. (i) True (ii) False 37. (b) 38. (d) 39. (b) 40. (c) 41. (b) 42. (b)

(iii) False (iv) False 43. (b) 44. (d) 45. (b) 46. (d) 47. (c) 48. (a)
6−1 49. (a) 50. (d) 51. (c) 52. (a) 53. (b) 54. (d)
5. 450 6. cos
7 55. (c) 56. (a) 57. (d) 58. (c) 59. (b) 60. (b)
1 −1 2
7. Direction ratios are 1, − 1, 2 and Direction cosines are , , . 61. (b) 62. (c) 63. (b) 64. (d) 65. (c) 66. (c)
6 6 6 67. (a) 68. (c) 69. (a) 70. (a) 71. (b) 72. (a)
73. (b) 74. (d) 75. (b) 76. (a) 77. (b) 78. (d)
Exercise for Session 2
1 $ 1 $ 3 $ 5 $ 4 $ 79. (b) 80. (a) 81. (b) 82. (c) 83. (a,b,d) 84. (b,c)
1. $i + k$ ; i+ k 2. i+ j+ k 85. (a,c) 86. (a,d) 87. (a,c,d) 88. (a,b,d) 89. (a,b,c) 90. (c)
2 2 5 2 5 2 5 2
3. 
1 1 1  91. (a) 92. (a) 93. (c) 94. (b) 95. (c) 96. (d)
, , 
 3 3 3 97. (c) 98. (a) 99. (d) 100. (b) 101. (c) 102. (a)
4 $ 9 $
9. 4 $i − 9$j, 97, i− j 103. (b) 104. a → q, r ; b → p, r
97 97
1 $ 7 $ 3 $ 1 105. (2, – 4) 106. (7) 107. (2)
10. 59; i− j+ k 12. ±
59 59 59 3 108. (1) 109. (3) 110. (2) 111. (2)

13. 66 14.
8
(5$i − $j + 2k$ ) 112. (a2 , − a1 , a3 ) 113. 5 5, West to East
30 114. 2 :1 115. 6 : 1 118. 2 : 5
15. 7 3 , ($i + 2$j + 2k$ ) 120. (c) 121. (d) 122. (a) 123. (d) 124. (d) 125. (c)
1 4 1
16. (i) − $i + $j + k$ (ii) −3$i + 3k$ 126. (d) 127. (b) 128. (a)
3 3 3
17. 4$i + 3$j + 7k$
Chap 01 Vector Algebra 51

15. PQ = OQ − OP = 4 $i − 5$j + 11k$

Solutions ∴
PQ
| PQ|

∴ cos γ =
=
4 $
162
11
162
i−
5 $
162
j+
11 $
162
k

, where γ is the angle of PQ with Z-axis.

1. a + b + c = (3 + 2 − 1)$i + (−2 − 4 + 2)$j + (1 − 3 + 2)k$ 16. AB = 4 $i − 5 $j + 11 k$


−5 −5
= 4 $i − 4 $j Direction cosine along Y -axis = =
16 + 25 + 121 162
2. Let b should be added, then a + b = $i 3 3
17. =
⇒ b = i$ − a = i$ − (3 $i + 4 $j − 2 k$ ) 32 + 42 + 52 50

= −2 i$ − 4 $j + 2 k$ 18. Vector A = 3i$ − 4$j + 5k.


$ We know that, direction cosines of
3 −4 5
3. | a + b| = |3i$ + 4$j − 12k$ | A= , ,
3 +4 +5
2 2 2
3 +4 +5
2 2 2
3 + 42 + 52
2

= | 3 2 + 4 2 + (12 ) 2 | = 13
3 −4 1
= , ,
4. a + b = 4$i + 4$j , 5 2 5 2 2
4 ( i$ + $j) $i + $j 19. Here, OA = 2 $i + 3 $j + 4 k$
Therefore, unit vector = =
32 2 OB = 3 i$ + 4 $j + 2 k$
5. Resultant vector = (2 i$ + 4 $j − 5 k$ ) + ( $i + 2 $j + 3 k$ ) and OC = 4 i$ + 2 $j + 3 k$
= 3 i$ + 6 $j − 2 k$ So, AB = i$ + $j − 2 k$ , BC = i$ − 2 $j + k$ , CA = 2 i$ − $j − k$
3 i$ + 6 $j − 2 k$ 1 $ Clearly, | AB| = | BC | = | CA| = 6
Unit vector = = (3 i + 6 $j − 2 k$ )
9 + 36 + 4 7
So, these points are vertices of an equilateral triangle.
6. R = 3 i$ + 5 $j + 4 k$ 20. Given, position vectors of A, B and C are 7$j + 10k$ ,
3 i$ + 5 $j + 4 k$ − i$ + 6 $j + 6 k$ and − 4 i$ + 9 $j + 6 k,
$ respectively.
⇒ R=
5 2 ∴ | AB| = | − i$ − $j − 4 k | = 18
$
7. a + b = 3i$ + 9$j = 3(i$ + 3$j). Hence, it is parallel to (1, 3). | BC | = | −3 $i + 3 $j | = 18
8. AB + BC + CA = 0 | AC | = | −4 $i + 2 $j − 4 k$ | = 36
⇒ a + b− c= 0 Clearly, AB = BC and ( AC ) 2 = ( AB ) 2 + ( BC ) 2
4 $i + 5 $j 1 Hence, triangle is right angled isosceles.
9. Unit vector parallel to OA = = ( 4 $i + 5 $j)
16 + 25 41
24. Position vectors of the points which divides internally is
10. OA = $i + 2$j − k$ , OB = $i + $j + k$ 3(2 a − 3 b) + 2(3 a − 2 b) 12 a − 13 b
=
and OC = 2 $i + 3 $j + 2 k$ 5 5
 2 − 4 −1 + 3 
Position vector of B w.r.t origin at A at 25. Coordinate of C is  ,  = ( −1, 1 )
 2 2 
AB = OB − OA = − $j + 2 k$
∴ OC = − $i + $j
Position vector of C w.r.t. origin at A is
3 i$ − $j − 3 k$ + $j + 3 $j − k$
AC = OC − OA = $i + $j + 3 k$ 26. = 2 $i + $j − 2 k$
2
11. 7 = (5 + 1)2 + (4 − 2)2 + (a + 2)2 a+ b 5
27. = 2 $i − $j + k$
⇒ a + 2 = ± 3 ⇒ a = −5, 1 2 2
12. AB = CX ⇒ $j − $i = position vector of point X − k$ 28. It is given that b is collinear with the vector a.
∴ b = λa …(i)
∴ Position vector of point X = − $i + $j + k$
= 2 2 λi$ − λ$j + 4 λk$
13. AB = (6 − 2)i$ + ( − 3 + 9)$j + (8 + 4)k$
Also, | b | = 10
= 4 i$ + 6 $j + 12 k$
⇒ (2 2 λ ) 2 + ( − λ ) 2 + ( 4 λ ) 2 = 10
| AB | = 16 + 25 + 144 = 14
⇒ 25 λ2 = 100
14. PQ = (5 − 1)$i + (− 2 − 3)$j + (4 + 7)k$ ⇒ λ = ±2 … (ii)
= 4 $i − 5 $j + 11 k$ From Eqs. (i) and (ii), we have
| PQ| = 16 + 25 + 121 = 162 b = ± 2a ⇒ 2a + b = 0
52 Textbook of Vector & 3D Geometry

29. Condition for collinearity, b = λ a 1 −3 p


⇒ ( −2i$ + m$j) = λ ( $i − $j) ⇒ 2( p − 1 ) 0 p + 4 −3 − p = 0
Comparison of coefficient, we get 0 p+3 1
⇒ λ = −2 and − λ = m ⇒ 2( p − 1 )( p + 4 + ( p + 3 ) 2 ) = 0
So, m =2
⇒ ( p − 1 )( p 2 + 7 p + 13 ) = 0
30. If given points be A, B and C, then AB = k(BC) or Roots of p 2 + 7 p + 13 = 0 are (imaginary)
2 $i − 8 $j = k[(a − 12 ) $i + 16 $j]
1 ∴ p =1
⇒ k=− Only integral value of p is 1.
2
Also, 2 = k (a − 12 ) (3 + 5 ) i$ + ( 0 − 2 ) $j + ( 4 + 4 ) k$
40. PV of AD =
⇒ a =8 2
1 2 3 = 4i − j + 4k
$ $ $
31. λ 4 7 =0 | AD| = 16 + 16 + 1 = 33
−3 −2 −5 A
⇒ λ =3
32. (a − b) − (a + b) = [(a + kb ) − (a − b) ]
⇒ − 2 b = (k + 1 ) b
Hence, k ∈ R
33. AB = − $i − 4 $j, CD = − 2 $i + ( λ − 2 ) $j B D C
Q AB || CD 41. v= b+ c
−1 −4
So, = ⇒ λ − 2 = −8 w = b+ a
−2 λ − 2
We have, x = v + w = a + 2b + c
⇒ λ = −6
3 2 1 42. Note that a + b = c
34. Obviously, = =−
6 −4 x y 43. Here is the only vector 4( 2i$ + $j ± k$ ), whose length is 8.
⇒ x = −1 and y = −2 44. 3a + 4b + 5c = 0
35. If a and b are two non-zero, non-collinear vectors and x and y Hence, a, b and c are coplanar.
are two scalars such that x a + y b = 0 then x = 0 and y = 0 No other conclusion can be derived from it.
because one will be a scalar multiple of the other and hence
collinear which is a contradiction. 45. A, B, C , D and E are five coplanar points.
36. Four or more than four non-zero vectors are always linearly DA + DB + AE + BE + CE
dependent. = ( DA + AE) + ( DB + BE) + ( DC + CE)
37. These are coplanar because 1 (a ) + 1 (b ) = a + b = DE + DE + DE = 3 DE
$ and the corresponding
38. Comparing the coefficients of i$, $j and k, 46. 3 tanθ + 1 = 0 and 3 secθ − 2 = 0
equations are 11 π
⇒ θ=
x + 3y − 4z = λx or (1 − λ ) x + 3y − 4z = 0 …(i) 6
x − ( λ + 3 )y + 5z = 0 11 π
… (ii) ⇒ θ = 2nπ + ,n ∈ Z
3 x + y − λz = 0 … (iii) 6
These Eqs. (i), (ii) and (iii) have a non-trivial solution, if 47. $j and k$ are unit vectors along Y and Z-axes, then unit vector
(1 − λ ) 3 −4 $j + k$
bisecting OY and OZ is .
1 − ( λ + 3 ) 5 = 0 ⇒ λ = 0,− 1 2
3 1 −λ 48. C
A B
39. The vectors are linearly dependent C
p+1 −3 p
⇒ p p+1 −3 = 0
−3 p p+1
1 −3 p
⇒ (2 p − 2 ) 1 p + 1 − 3 = 0 O

1 p P +1 We have, A( i$ + $j − 2 k$ ) and B( i$ − 3 $j + k$ )
Chap 01 Vector Algebra 53

On line AB points C and C′ are at distance 1 unit from A. A (6b–2a)


OC = OA + AC, where AC is unit vector in direction of AB 1
AB P (a, b)
∴ OC = OA +
| AB|
2
AB
Similarly, OC′ = OA −
| AB|
O B
49. AD = OD − OA
⇒ 3(a − b) = OB + 12 b − 4 a
b+ c b + c − 2a
= −a = ⇒ OB = 7 a − 15 b
2 2
[where, O is the origin for reference]
53. Let the B divide AC in ratio λ : 1, then
c+ a λ (11 $i + 3 $j + 7 k$ ) + $i − 2 $j − 8 k$
Similarly, BE = OE − OB = −b 5 $i − 2 k$ =
2 λ +1
c + a − 2b ⇒ 3λ − 2 = 0
= 2
2 ⇒ λ = , i.e. ratio = 2 : 3
a + b − 2b 3
and CF =
2 54. We know by fundamental theorem of proportionality that,
1
A (a) DE = BC
2
A
F E

D E

B (b) D C (c)

b + c − 2a B C
Now, AD + BE + CF =
2 In triangle, BC = b − a
c + a − 2 b a + b − 2c 1
+ + =0 Hence, DE = ( b − a)
2 2 2
1 1 55. Since, AB + BD = AD
50. AP = AB + BP = AB + BC = AB + AD …(i)
2 2
BD = AD − AB
1 1
AQ = AD + DQ = AD + DC = AD + AB … (ii) ⇒ = ( $i + 2 $j + 3 k$ ) − (2 $i + 4 $j − 5 k$ )
2 2
D Q C = − $i − 2 $j + 8 k$
Hence, unit vector in the direction of BD is
− $i − 2 $j + 8 k$ − $i − 2 $j + 8 k$
=
P | − i$ − 2 $j + 8 k|
$ 69
56. Position vectors of vertices A, B and C of the ∆ABC = a, b and
A B
c. We know that, position vector of centroid of the triangle,
By Eqs. (i) and (ii), we get a + b+ c
G= .
3 3
AP + AQ = ( AB + AD) Therefore, GA + GB + GC
2
 a + b + c  a + b + c  a + b + c
3 3 = a −  + b −  +c − 
= (AB + BC) = AC  3   3   3 
2 2
1
51. Let A ≡ (1, 1, − 1), B ≡ (2, 3, 0), C ≡ (3, 5, − 2) and D ≡ (0, − 1, 1). = (2 a − b − c + 2 b − a − c + 2 c − a − b) = 0
3
So, AB = (1, 2, 1 ), BC = (1, 2, − 2 ), CD = ( − 3, − 6, 3 )
57. A regular hexagon ABCDEF .
and DA = (1, 2, − 2 ).
E D
Clearly, BC || DA but AB in not parallel to CD.
So, it is a trapezium.
1 (OB) + 2 (6 b − 2 a) F C
52. OP =
1+2
A B
54 Textbook of Vector & 3D Geometry

We know from the hexagon that AD is parallel to BC or Let α = x a + y b ⇒ 6 x + 2y = 6


AD = 2 BC is parallel to FA or EB = 2 FA and FC is parallel to and −3 x − 6y = 12
AB or FC = 2 AB. ∴ x = 2, y = − 3
Thus, AD + EB + FC = 2 BC + 2 FA + 2 AB ∴ α = 2a − 3 b
= 2( FA + AB + BC) = 2( FC) = 2(2 AB) = 4 AB
67. Let the incentre be at the origin and be
58. AE + ED + DC + AB A( p ), B( q ) and C( r ) . Then,
= AD + DC + AB = AC + AB IA = p, IB = q and IC = r
Obviously, if BC is added to this system, then it will be ap + bq + cr
Incentre I is , where p = BC , q = AC and r = AB
AC + AB + BC = AC + AC = 2 AC. a +b +c
59. By triangle law, AB = AD − BD, AC = AD − CD Incentre is at the origin. Therefore,
ap +bq +cr
E D = 0,
a +b +c
or ab +bq +cr = 0
F C ⇒ a IA + b IB + c IC = 0
68. Since x, y and x × y are linearly independent, we have
A B 20a − 15b = 15b − 12c = 12c − 20a = 0
a b c
Therefore, ⇒ = = ⇒ c2 = a2 + b2
3 4 5
AB + AC + AD + AE + AF Hence, ∆ABC is right angled.
= 3 AD + ( AE − BD) + ( AF − CD) = 3 AD
69. As x, y and x × y are non-collinear vectors, vectors are
Hence, λ = 3 (Q AE = BD,AF = CD)
linearly independent.
60. | a | +| b | +| c | = a 2 + b 2 + c 2 Hence,
⇔ 2 | ab | + 2 | bc | + 2 | ca | = 0 a −b = 0 = b −c = c −a
⇔ ab = bc = ca = 0 ⇔ any two of a, b and c are zero or a = b = c
61. Since, a and b are non-collinear, so a + b and a − b will also Therefore, the triangle is equilateral.
be non-collinear. 70. | AB| = | Q| = P 2 + P 2 = P 2
Hence, a + b and a − b are linearly independent vectors.
71. The point the divides 5$i and 5$j in th ratio of
62. | a + b | < | a − b |
(5 $j)k + (5 $i )1
π 3π k : 1 is
⇒ <θ < k+1
2 2
5 $i + 5k$j
63. R = 4 + 100 + 121 = 15 ∴ b=
k +1
2 k$ + 2 $j − $i Also | b | ≤ 37
64. We must have λ( i$ − 3 $j + 5 k$ ) = a +
3 + +
Therefore, 3a = 3 λ( i − 3 j + 5 k ) − (2 k + 2 $j − $i )
$ $ $ $ –6 – –1/6

= $i(3 λ + 1 ) − $j(2 + 9 λ ) + k$ (15 λ − 2 ) 1


⇒ 25 + 25k 2 ≤ 37
k +1
or 3| a | = (3 λ + 1 ) 2 + (2 + 9 λ ) 2 + (15 λ − 2 ) 2
or 5 1 + k 2 ≤ 37(k + 1 )
or 9 = (3 λ + 1 ) 2 + (2 + 9 λ ) 2 + (15 λ − 2 ) 2
On Squaring both sides, we get
2
or 315 λ2 − 18 λ = 0 ⇒ λ = 0, 25(1 + k 2 ) ≤ 37 (k 2 + 2k + 1 )
35
If λ = 0, a = $i − 2 $j − 2 k$ (not acceptable) or 6k 2 + 37k + 6 ≥ 0 or (6k + 1 )(k + 6 ) ≥ 0
2 41 $ 88 $ 40 $  1 
λ = ,a = i− j− k ∈ ( −∞, − 6 ) ∪ − , ∞
For k  6 
35 105 105 105
65. b = cos120 ° i$ + sin 120 ° $j 72. Let the bisector of ∠A meets BC at D, then AD divides BC in
the ratio AB : AC .
1 3$
or b = − i$ + j ∴ Position vectors of D
2 2
| AB | (2 $i + 5 $j + 7 k$ )+| AC|(2 $i + 3 $j + 4 k$ )
1 3 $ 1$ 3$ =
Therefore, a + b = $i − i$ + j= i+ j | AB | + | AC |
2 2 2 2
Here, | AB| = | −2 i − 4 j − 4 k$ | = 6
$ $
66. α = a + b + c = 6 $i + 12 $j
Chap 01 Vector Algebra 55

and | AC| = | −2 $i − 2 $j − k$ | = 3 β+1


⇒ d= a
∴ Position vector of D α +1
6(2 $i + 5 $j + 7 k$ ) + 3(2 $i + 3 $j + 4 k$ ) ⇒ a + b + c = αd
=  β + 1
6+3 ⇒ a + b+ c =α  a
 α + 1
18 i$ + 39 $j + 54 k$
=  α (β + 1 ) 
9 ⇒ 1− a + b+ c= 0
 α + 1 
1 $
= (6 i + 13 $j + 18 k$ )
3 a, b and c are coplanar which is contradiction to the given
a+b condition.
73. Vector in the direction of angular bisector of a and b is . ∴ α = −1
2
a+b and so a + b + c + d = 0
Unit vector in this direction is
|a + b| 77. Since | OP | =| OQ| = 14, ∆OPQ is an isosceles.
D C Hence, the internal bisector OM is perpendicular to PQ and M
is the mid-point of P and Q. Therefore,
1
OM = ( OP + OQ) = 2 $i + $j − 2 k$
b 2
E O

q q x x
A 2 B
(o) a
a+b
From the figure, position vector of E is .
2 P M Q
θ
Now in triangle AEB, AE = AB cos 78. C
2 Q
D
a+b θ
⇒ = cos
2 2
R S
a+b E
Hence, unit vector along the bisector is .
2 cos (θ / 2 )
74. a − b = 2( d − c ) A P B
a + 2c b + 2d
∴ = Let OA = a, OB = b, OC = c
2+1 2+1 and OD = d
Hence, AC and BD trisect each other as LHS is the position Therefore, OA + OB + OC + OD
vector of a point trisecting A an C, and RHS that of B and D.
=a + b + c + d
75. Again, it is given that the point P, Q and R are collinear. a+b
P, the mid-point of AB, is .
⇒ PQ = λQR 2
⇒ 15t( 4 $j − 3 $i ) = λ [(1 − t ) ( $i + $j) − 60t $j] c+d
Q, the mid-point of CD, is .
⇒ = λ[(1 − t ) i$ + (1 − 61t ) $j] 2
45t 60t a+ b+ c+ d
⇒ = Therefore, the mid-point of PQ is .
t − 1 1 − 61t 4
3t 4t a+ b+ c+ d
⇒ = Similarly, the mid-point of RS is
t − 1 1 − 61t 4
⇒ 3 (1 − 61t ) = 4(t − 1 ) a+b+c+d
i.e. OE =
⇒ 3 − 183t = 4t − 4 ⇒ 187t = 7 4
7 ⇒ x=4
∴ t=
187 79. O
76. We have, a + b + c = αd
Y
and b + c + d = βa C X
∴ a + b + c + d = (α + 1 ) d
and a + b + c + d = (β + 1 ) a
⇒ (α + 1 ) d = (β + 1 ) a A(a) M B(b)
If α ≠ −1, then (α + 1 ) d = (β + 1 ) a
OA = a, OB = b
56 Textbook of Vector & 3D Geometry

a+b 21(a + b ) 21
∴ OM = = = PR
2 25 25
a+b 81. Let OA = a, OB = b and OC = c,
∴ OC =
6 1
2 then AB = b − a and OP = a.
OX = b 3
3 1 1
OY OQ = b, OR = c.
Let =λ 2 3
YA
Since P , Q, R and S are coplanar, then
λ
∴ OY = a PS = αPQ + βPR
λ +1
(PS can be written as a linear combination of PQ and PR)
Now points Y , C and X are collinear.
= α ( OQ − OP ) + β ( OR − OP )
∴ YC = mCX a α β
a+b λ 2b a+b i.e. OS − OP = −(α + β ) + b + c
∴ − a = m −m 3 2 3
6 λ +1 3 6 a α β
Comparing coefficients of a and b ⇒ OS = (1 − α − β ) + b + c … (i)
3 2 3
1 λ m
∴ − =− Given OS = λAB = λ( b − a ) …(ii)
6 λ +1 6 1 −α α
1 2m m From Eq. (i) and Eq. (ii), β = 0, = − λ and = λ
and = − 3 2
6 3 6
⇒ 2 λ = 1 + 3 λ or λ = −1
1 2
∴ m = and λ = 82.
3 7
P
 5a + 4 b  a + 5 b
4 +
 3  5 21(a + b ) 21
80. = = PR D 2
4+1 25 25
2 2
S(b) y R(a + b) Q
E B
2
2
Z 2 2
X 60°
O 2 A (2, 0, 2)

P(o) Q(a)

4(a + b ) + a 5a + 4 b Here, coordinate of Q are (2 cos60 °, 2 sin 60 ° )


PV of X is = ⇒ Q (1, 3, 0 )
5 5
4b + a + b a + 5b ∴ P (1, 3, z )
PV of Y is =
5 5 OP = 3
Now, PZ = mPR ⇒ 1 + 3 + z 2 = 3 or z 2 = 5
PZ = m(a + b )
z= 5
Let Z divided YX in the ratio λ :1
λOX + OY ∴ P(1, 3, 5 ) ⇒ OP = $i + 3 $j + 5 k$
PV of Z =
λ +1 Now, AP = OP − OA = i$ + 3 $j + 5 k$ − 2 $i
 5a + 4 b  a + 5 b = − i$ + 3 $j + 5 k$
 +
 5  5
∴ PZ = = m(a + b ) 83. a = [ ±($i − $j) ± ($j + k$ )]
λ +1
Comparing coefficients of a and b = ± ( i$ + k$ ), ± ( $i − 2 $j − k$ )
5λ + 1 84. Let R be the resultant. Then
m=
5( λ + 1 ) R = F1 + F2 + F3 = ( p + 1 ) $i + 4 $j
4λ + 5 Given, | R | = 5 . Therefore, ( p + 1 ) 2 + 16 = 25
and m=
5( λ + 1 )
or p + 1 = ±3 or p = 2,−4
∴ λ=4
85. We have, AB = − $i − $j − 4 k$ , BC = −3 $i + 3 $j
 5a + 4 b  a + 5 b
4 + CA = 4 $i − 2 $j + 4 k$ .
 5  5 and
∴ PZ =
4+1 Therefore, | AB | = | BC| = 3 2 and | CA | = 6
Chap 01 Vector Algebra 57

Clearly, | AB| 2 + | BC | 2 = | AC | 2 Now, X divides OC in the ratio λ : 1. Therefore,


λc
Hence, the triangle is right angled isosceles triangle. PV of X = …(iii)
λ +1
86. Let a = 2 $i + 4 $j − 5 k$ and b = i$ + 2 $j + 3 k$ .
X also divided AD in the ratio µ : 1. Therefore,
Then, the diagonals of the parallelogram are µd + a
p =a+ b PV of X = …(iv)
µ+1
and q = b − a,
From Eqs. (iii) and (iv), we get
i.e. p = 3 i$ + 6 $j − 2 k$ , q = − i$ − 2 $j + 8 k$ λc µd + a
=
So, unit vectors along the diagonals are λ +1 µ+1
1 $ $ 1
(3 i + 6 j − 2 k$ ) and ( − i$ − 2 $j + 8k$ )  λ   µ   1 
7 69 or  c =  d +  a
 λ + 1  µ + 1  µ + 1
87. OA = −4$i + 3k$ ; OB = 14$i + 2$j − 5k$  λ   µ   −b + 2c  1 
or  c =    +  2 ( b − c)
− 4 $i + 3 k$ 14 $i + 2 $j − 5 k$  λ + 1  µ + 1  3   µ + 1
a= ;b=
5 15 [using Eqs. (i) and (ii) ]
λ
r = [ −12 $i + 9 $j + 14 $i + 2 $j − 5 k$ ]  λ   6 −µ   2µ 2 
or  c =   b + − c
15  λ + 1  3(µ + 1 )  3(µ + 1 ) µ + 1
λ 2λ $ $
= [2 i$ + 2 $j + 4 k$ ] = [ i + j + 2 k$ ]  λ   6 −µ   2µ − 6 
15 15 or  c =  b+  c
 λ + 1  3(µ + 1 )  3(µ + 1 )
88. Points A( $i + $j ), B( $i − $j) and C ( p$i + q$j + rk$ ) are collinear
 6 −µ   2µ − 6 λ 
Now, AB = −2 $j or  b+  − c = 0
 3(µ + 1 )  3(µ + 1 ) λ + 1
and BC = ( p − 1 ) $i + (q + 1 ) $j + rk$
6 −µ
Vectors AB and BC must be collinear or =0
3(µ + 1 )
⇒ p = 1, r = 0 and q ≠ −1 2µ − 6 λ
1 2 3 and − =0
3(µ + 1 ) λ + 1
89. For coplanar vectors, 0 λ µ =0
(as b and c are non-collinear)
0 0 2λ − 1 2
or µ = 6, λ =
1 5
or (2λ −1)λ = 0 or λ = 0,
2 µ 6
Hence, OX : XC = 2 : 5 and AX / XD = =
90. In ∆ABC, AB + BC = AC = −CA 1 1
or AB + BC + CA = 0 93. (c) 94. (b)
OA + AB = OB is the triangle law of addition.
Solutions (Q.Nos. 95-96)
Hence, Statement 1 is true and Statement 2 is is false.
Consider the regular hexagon ABCDEF with centre at O
1 p 2 3
91. = = ⇒ p = and q = 4 (origin).
2 3 q 2 E D
92. 2a + 3 b − 5 c = 0
3( b − a ) = 5( c − a ) O
5 F C
⇒ AB = AC
3
Hence, AB and AC must be parallel since there is a common
point A. The points A, B and C must be collinear. A B

AD + EB + FC = 2 AO + 2 OB + 2 OC
Solutions (Q.Nos. 93-94)
= 2( AO + OB) + 2 OC
Let the position vectors of A, B, C and D be a, b, c and d,
respectively. Then, = 2 AB + 2 AB [Q OC = AB]
OA :CB = 2 : 1 = 4AB
⇒ OA = 2 CB ⇒ a = 2( b − c ) … (i) R = AB + AC + AD + AE + AF
and OD : AB = 1 : 3 = ED + AC + AD + AE + CD
3OD = AB [Q AB = ED and AF = CD]
⇒ 3 d = ( b − a ) = b − 2( b − c ) [using Eq. (i)] = ( AC + CD) + ( AE + ED) + AD
= −b + 2c … (ii) = AD + AD + AD = 3 AD = 6 AO
Let OX : XC = λ : 1 and AX : XD = µ : 1 95. (c) 96. (d)
58 Textbook of Vector & 3D Geometry

Solutions (Q.Nos. 97-99) 1


= 4αe 2(t −1) , α =
97. AB = OB − OA = 3a − b − 2c 2
AC = OC − OA = 9a − 3 b − 6 c = 3AB The only value of t which satisfies all three equations is t = 1.
So, r(1 ) is the required point ( −1, 1, 2 ).
98. 2OA − 3OB + OC
103. (2, 0, 5) corresponding to r(1 ) and r ′ (t ) = 4t$i − $j + 6tk$
= 2( −2a + 3 b + 5 c ) − 3(a + 2 b + 3 c ) + (7a − c ) = 0
So, the required tangent vector is r′ (1 ) = 4 $i − $j + 6 k.
$
99. Q 2OA − 3OB + OC = 0
A 104. E D

(1)

F C
B O

(2) b
O
A a B
C
2 OA + OC AB = a, BC = b
⇒ OB =
3 ∴ AC = AB + BC = a + b … (i)
⇒ B Divides AC in 1 : 2. AD = 2 BC = 2 b … (ii)
(because AD is parallel to BC and
Solutions (Q.Nos. 100-101) twice is length)
 7 $i − 4 $j − 4 k$ ( −2 $i − $j + 2 k$  CD = AD − AC = 2 b − (a + b ) = b − a
100. Here, c = t(a$ + $b) = t  + 
 9 3  FA = − CD = a − b … (iii)
DE = − AB = −a … (iv)
 i$ − 7 $j + 2 k$ 
⇒ c = t  EF = − BC = − b … (v)
 9  AE = AD + DE = 2 b − a … (vi)
t CE = CD + DE = b − a − a = b − 2a … (vii)
Also, | c| = 5 6 ⇒
. 1 + 49 + 4 = 5 6
9
105. Let R be the resultant. Then,
15
∴ t = 15 ⇒ c = ( $i − 7 $j + 2 k$ ) R = F1 + F2 + F3 = ( p + 1 ) $i + 4 $j
9
Given, | R | = 5, Therefore R 2 = 25
5 $
or = ( i − 7 j + 2 k$ )
$
Q ( p + 1 ) 2 + 16 = 25 or p + 1 = ±3 or p = 2, − 4
3
101. Here,
AB BD
= and
AB BE
= 106. Vectors along to sides are a = i$ + 2$j + k$ and b = 2i$ + 4$j + k$
AC DC AC CE Clearly the vector along the longer diagonal is
A a + b = 3 $i + 6 $j + 2 k$
Hence, length of the longer diagonal is
| a + b| = |3 $i + 6 $j + 2 k$ | = 7
107. Vector a = i$ + 2$j − k$ , b = 2i$ − $j + k$ , c = λ$i + $j + 2k$ are
coplanar.
1 2 −1
B D C E ⇒ 2 −1 1 = 0 or λ − 3 + 2( −5 ) = 0 or λ = 13
BD BE BD BE λ 1 2
⇒ = ⇒ =
DC CE BC − BD BE − BC Number of digits in value of λ is 2.
⇒ BD ⋅ BE − BD ⋅ BC = BC ⋅ BE − BD ⋅ BE 108. Since, angle bisector of a and b
⇒ 2BD ⋅ BE = ( BD + BE ) ⋅ BC  a b
2 1 1 ⇒ h ( a$ + $b) = h  +  … (i)
or = +  | a | | b |
BC BD BE
given, a + b is along angle bisector.
Solutions (Q.Nos. 102-103)
 a b
102. r ′ (t ) = −2i$ + 2t$j + 4e 2(t − 1) k$ ⇒ µ +  =a+ b
 | a| | b |
Since, r ′(t ) is parallel to r (t ),
only, when | a | = | b | = µ
so r (t ) = αr ′ (t )
∴ | a| = | b| ⇒ λ = 1
1 − 2t = −2α , t 2 = αt, 2e 2(t −1)
Chap 01 Vector Algebra 59

109. A π
Now, if it is turned through an angle . As shown in adjoining
2
figure.
∴ Now components are (a 2, − a1, a 3 ) .
O G H
113. Here, r1 = OA pointing North-East and r2 = OB pointing
B E North-West. Where | OA| = | OB| = 5.
Here, O is circum centre = 0, G is centroid = g As shown in figure,
H is orthocentre = p N
OG 1 B A
Since, =
GH 2 r2 45° 45° r1
g 1
⇒ = ⇒ 2g = p − g
p− g 2 W O E
or p = 3g
∴ k =3
110. XG = kGY S

b + c 2b  λc b + c ∴ ∠BOA = 90 °
− = k − , k = 1
3 3  1+λ 3  ⇒ r1 − r2 = BA (using triangle law)
Clearly, ∠BOA is right angled at O.
A(O)
∴ BA 2 = OA 2 + OB 2 = 5 2 + 5 2 = 50
X Y ⇒ | BA| = 5 5
G or | r1 − r2| = 5 5
i.e. r1 − r2 has magnitude 5 5 and points from West to East.
B(b) C(c) 114. Let OACB be a parallelogram shown as
2c λ B(b) C(c)
= c
3 1+ λ
λ 2
⇒ =
1+ λ 3 P
⇒ λ =2
111. We have, a = 2p$i + $j O D A(a)
Let b be the vector obtained from a by rotating the axes. Then,
1
the components of b are p + 1 and 1. Therefore, Here, OD = BC
2
b = ( p + 1 ) α$ + β$
1
⇒ OP + PD = ( BP + PC ) [using ∆ law]
where α$ and β$ are unit vectors along the new axes. 2
But | b| = | a| ⇒ 2 OP + 2 PD = BP + PC
⇒ 4p 2 + 1 = (p + 1)2 + 1 −2 PO + 2 PD = − PB + PC
1 ⇒ PB + 2 PD = PC + 2 PO
⇒ 3 p 2 − 2 p − 1 = 0 ⇒ p = 1, − PB + 2 PD PC + 2 PO
3 ⇒ =
1 1+2 1+2
⇒ p1 = 1 and p 2 = −
3 The common point P of BD and CO divides each in the ratio
1 2 : 1.
∴ 3 | p1 + p 2| = 3 1 − =2
3 115. Let S be the point of intersection of AB and CR . Let A be the
origin and the position vectors of the points B, C , P , Q, R and S
112. Here, a is rotated about Z-axis, the Z-component of a will be b, c, p, q, r and s respectively.
remain unchanged namely a 3
3b + 2c
y ′(x) ∴ p=
5
A′ A 4c
and q= …(i)
a2 5
a
a 5p − 3b 5q
⇒ = ⇒ 10 p − 6 b = 5 q
2 4
x′(y) O a1
60 Textbook of Vector & 3D Geometry

10 p 5 q + 6 b Let the position vectors of P1, P2, P3,......, Pn with respect to O′ as


i.e. 10 p = 5 q + 6 b ⇒ = =r
11 11 the origin be R 1, R 2....., R n respectively. Then, R i = O′
Pi = ri − α, i = 1, 2,....., n [using Eq. (i)]
Now, a1R 1 + a 2R 2 + ..... + an R n = 0
n n
⇒ ∑ ai Ri = 0 ⇒ ∑ ai (ri − α ) = 0
i =1 i =1
n n
⇒ ∑ airi − ∑ ai α = 0
i =1 i =1
11 r 3b + 2c  n   n 
⇒ =p= [using Eq. (ii)]
10 5 ⇒ 0 − α  ∑ ai  = 0 Q ∑ airi = 0 (given ) 
 
11 r = 6 b + 4 c i = 1   i = 1 
11 r − 4 c = 6 b n

11 r − 4 c 6 ⇒ ∑ ai = 0
= b = s , thus s divides AB in the ratio 6 : 1. i =1
7 7 n n
116. Since, angle bisectors divides opposite side in the ratio of sides Thus, ∑ ai Ri = 0 will hold good, if ∑ ai = 0.
containing the angle. i =1 i =1
ac ab
⇒ BA′ = and CA′ = 118. Let O be the origin of reference.
b +c a +c
Let the position vectors of A, B, C and D be a, b, c and d,
Now, BI is also angle bisector of ∠B for ∆ABA′. respectively.
AI b +c AI b +c
⇒ = ⇒ = Then, OA : CB = 2 : 1
AI ′ a AA′ a + b + c OA 2
A ⇒ = ⇒ OA + 2CB
CB 1
C B
C′ B′

I
B C D X
A′
BI a +c
Similarly, =
BB′ a + b + c
CI a+b A
and = O
CC ′ a + b + c
⇒ OA = 2 CB
AI ⋅ BI ⋅ CI (b + c )(a + c )(a + b )
⇒ = …(i) ⇒ a = 2( b − c) …(i)
AA′ ⋅ BB′ ⋅ CC′ (a + b + c )(a + b + c )(a + b + c )
and OD : AB = 1 : 3
As we know AM ≥ GM , we get OD 1
b +c c +a a +b 1 ⇒ = ⇒ 3OD = AB
+ + AB 3
a + b + c a + b + c a + b + c ≥ (a + b )(b + c )(c + a )  3
  ⇒ 3OD = AB
3  (a + b + c ) 3 
⇒ 3 d = ( b − a ) = b − 2( b − c) [using Eq. (i)]
2(a + b + c ) [(a + b )(b + c )(c + a )]1 3 ⇒ 3d = − b + 2c
⇒ ≥ …(ii)
3(a + b + c ) a +b +c Let OX : XC = λ : 1 and AX : XD = µ : 1
(a + b )(b + c )(c + a ) 8 Now, X divides OC in the ratio λ : 1. Therefore,
⇒ ≤ …(ii)
(a + b + c ) 27 λc
PV of X = …(iii)
From Eqs. (i) and (ii), we get λ+1
AI ⋅ BI ⋅ CI 8 X also divides AD in the ratio µ : 1

AA′ ⋅ BB′ ⋅ CC ′ 27 µd + a
PV of X =
117. Let the position vector of O′ with reference to O as the origin µ+1
be α. From Eqs. (iii) and (iv), we get
Then, OO′ = α λc µd + a
=
Now, O′ Pi = Position vector λ+1 µ+1
or Pi − Position vector of O′ = ri − α  λ   µ   1 
i = 1, 2,......, n ⇒  c= d+  a
 λ + 1  µ + 1  µ + 1
Chap 01 Vector Algebra 61

 λ   µ   − b + 2 c  1  Also, AB + BM + MA = 0
⇒  c=    +   2 (b − c)
 λ + 1  µ + 1  3   µ + 1 [by properties of a triangle]
AC − AB
[using Eqs. (i) and (iv)] ⇒ AB+ = AM [Q AM = − MA]
2
 λ   2 µ 
⇒   c= −  b AB + AC
 λ + 1  µ + 1 3 (µ + 1 ) ⇒ AM =
2
 2µ 2  3 i$ + 4 k$ + 5 $i – 2 $j + 4 k$
+ −  c =
 3 (µ + 1 ) µ + 1 2
 λ   6 −µ   2µ − 6  = 4 i − j + 4k
$ $ $
⇒   c= b+  c
 λ + 1  3 (µ + 1 )  3(µ + 1 ) ⇒ | AM | = 4 2 + 1 2 + 4 2 = 33
 6 −µ   2µ − 6 λ  121. As, a + 3b is collinear with c.
⇒  b+  − c = 0
 3 (µ + 1 )  3(µ + 1 ) λ + 1
∴ a + 3 b = λc …(i)
6 −µ 2µ − 6 λ Also, b + 2 c is collinear with a.
⇒ = 0 and − =0
3 (µ + 1 ) 3 (µ + 1 ) λ + 1 ⇒ b + 2 c = µa …(ii)
(since, b and c are non-collinear) From Eq. (ii), we get
2 a + 3b + 6c = (λ + 6)c …(iii)
⇒ µ = 6 and λ =
5 From Eq. (ii), we get
Hence, OX : XC = 2 : 5 a + 3 b + 6 c = (1 + 3µ )a …(iv)
119. let l, m and n be scalars such that From Eqs. (iii) and (iv), we get
lp + mq + nr = 0 ∴ ( λ + 6 ) c = (1 + 3µ ) a
⇒ {l (cosa ) u + (cosb ) v + (cosc ) w} + m {(sin a ) u Since, a is not collinear with c.
+ (sin b ) v + (sin c ) w } ⇒ λ + 6 = 1 + 3µ = 0
+ n {sin( x + a ) u + sin( x + b ) v + sin( x + c ) w } = 0 From Eq. (iv), we get
⇒ {l cosa + m sin a + n sin( x + a )} u + {l sin b + m sin( x + b )} v a + 3b + 6c = 0
+ {l cosc + m sin c + n sin ( x + c )} w = 0
122. Since, a = 8 b and c = − 7b
⇒ l cosa + m sin a + n sin( x + a ) = 0 …(i)
So, a is parallel to b and c is anti-parallel to b.
l cosb + m sin b + n sin( x + b ) = 0 …(ii)
⇒ a and c are anti-parallel.
l cosc + m sin c + n sin( x + c ) = 0 …(iii)
So, the angle between a and c is π.
This is a homogeneous system of linear equations in l, m and n.
The determinant of the coefficient matrix is 123. Let P be the origin outside of AB and C is mid-point of AB,
cosa sin a sin( x + a ) cosa sin a 0 then
∆ = cosb sin b sin( x + b ) = cosb sin b 0 = 0 C
A B
cosc sin c sin( x + c ) cosc sin c 0
(using C 3 → C 3 − sin x C1 − cos x C 2)
⇒ So, the above system of equations has non-trivial solutions
also. This means that l, m and n may attain non-zero values
also. P
Hence, the given system of vectors is a linearly dependent PA + PB
PC = ⇒ 2PC = PA + PB
system of vectors. 2
120. We know that, the sum of three vectors of a triangle is zero. 124. If a + 2b is collinear with c, then a + 2b = tc …(i)
A Also, b + 3 c is collinear with a, then
b + 3 c = λa
⇒ b = λa − 3 c …(ii)
From Eqs. (i) and (ii), we get
a + 2( λa − 3 c ) = tc
B M C ⇒ (a − 6 c ) = tc − 2 λa
On comparing the coefficients of a and c, we get
∴ AB + BC + CA = 0
1
⇒ BC = AC − AB [Q AC = − CA] 1 = –2λ ⇒ λ = –
AC − AB 2
⇒ AB = [Q M is a mid-point of BC] and –6 = t ⇒ t = –6
2
62 Textbook of Vector & 3D Geometry

From Eq. (i), we get and DA = (5 − 7 ) 2 + ( −1 + 4 ) 2 + (5 − 7 ) 2


a + 2b = − 6c = 4+9+ 4
⇒ a + 2b + 6c = 0
= 17
125. The three vectors (a + 2b + 3c ), (λb + 4c ) and (2λ − 1)c are Hence, option (d) is correct.
non-coplanar, if
1 2 3 a a2 1 + a3 a a2 1 a a2 a3
0 λ 4 ≠0 127. Since, b b 2 1 + b 3 = b b 2 1 + b b 2 b 3 = 0
0 0 2λ − 1 c c2 1 + c3 c c2 1 c c2 c3

⇒ (2 λ − 1 ) ( λ ) ≠ 0 a a2 1 a a2 1
1 ⇒ b b 1 = abc b b 2 1 = 0
2
⇒ λ ≠ 0,
2 c c2 1 c c2 1
So, these three vectors are non-coplanar for all except two
values of λ. a a2 1  a a2 1 
 
126. Given that, OA = 7 $i − 4 $j + 7 k$ ⇒ (1 + abc ) b b 2 1 = 0 Q b b2 1 ≠ 0
c c2 1  c c2 1 
OB = $i − 6 $j + 10 k$  
OC = − $i − 3 $j + 4 k$ ⇒ 1 + abc = 0
OD = 5 $i − $j + 5 k$ ⇒ abc = −1
128. Since, the vector i$ + x$j + 3 k$ is doubled in magnitude, then it
Now, AB = (7 − 1 ) 2 + ( −4 + 6 ) 2 + (7 − 10 ) 2
becomes
= 36 + 4 + 9 4 $i + ( 4 x − 2 ) $j + 2 k$
= 49 = 7
∴ 2 | i$ + x$j + 3 k$ | = 4 $i + ( 4 x − 2 ) $j + 2 k |
BC = (1 + 1 ) 2 + ( −6 + 3 ) 2 + (10 − 4 ) 2 ⇒ 2 1 + x 2 + 9 = 16 + ( 4 x − 2 ) 2 + 4
= 4 + 9 + 36 ⇒ 40 + 4 x 2 = 20 + ( 4 x − 2 ) 2
= 49 = 7 ⇒ 3x 2 − 4x − 4 = 0
CD = ( −1 − 5 ) 2 + ( −3 + 1 ) 2 + ( 4 − 5 ) 2 ⇒ ( x − 2 ) (3 x + 2 ) = 0
= 36 + 4 + 1 2
⇒ x = 2, −
= 41 3
CHAPTER

02
Product of Vectors
Learning Part
Session 1
● Product of Two Vectors

● Components of a Vector Along and Perpendicular to Another Vector

● Application of Dot Product in Mechanics

Session 2
● Vector or Cross Product of Two Vectors

● Area of Parallelogram and Triangle

● Moment of a Force and Couple

● Rotation About an Axis

Session 3
● Scalar Triple Product

Session 4
● Vector Triple Product

Practice Part
● JEE Type Examples
● Chapter Exercises

Arihant on Your Mobile !


Exercises with the #L
symbol can be practised on your mobile. See inside cover page to activate for free.
Session 1
Product of Two Vectors, Components of a Vector
Along and Perpendicular to Another Vector,
Application of Dot Product in Mechanics

Product of Two Vectors B


Product of two vectors is processed by two methods. M
When the product of two vectors results in a scalar b
quantity, then it is called scalar product. It is also known
as dot product because we denote it by putting a dot (.) q
O A
between two vectors. a L
When the product of two vectors results in a vector From triangles OBL and OAM, we have OL = OB cos θ
quantity, then this product is called vector product. It is and OM = OA cos θ. Here, OL and OA are known as
also known as cross product because we denote it by projection of b on a and a on b respectively.
putting a cross (×) between two vectors.
Now, a ⋅ b = | a || b | cos θ
= | a| (OB cos θ ) = | a | (OL )
Scalar or Dot Product of Two Vectors = (Magnitude of a) (Projection of b on a) …(i)
If a and b are two non-zero vectors and θ be the angle Again, a ⋅ b = | a || b | cos θ
between them, then their scalar product (or dot product) is
= | b|(| a| cos θ ) = | b| (OA cos θ ) = | b| (OM )
denoted by a ⋅ b and is defined as the scalar | a|| b| cos θ,
where | a| and | b| are modulii of a and b respectively and a ⋅ b = (Magnitude of b)
0 ≤ θ ≤ π. (Projection of a on b) …(ii)
B Thus, geometrically interpreted, the scalar product
of two vectors is the product of modulus of
b
either vector and the projection of the other in
its direction.
q
O a A Remarks
a ⋅b
1. Projection of a on b = = a ⋅b
|b|
Remarks
a ⋅b
1. a ⋅b ∈ R 2. Projection of b on a = = a ⋅b
|a |
2. a ⋅ b ≤|a||b|
3. a ⋅ b > 0 ⇒ Angle between a and b is acute. 3. Angle between two vectors it a and b be two vectors inclined at
an angle θ, then a , b = |a| ⋅|b| cos θ
4. a ⋅ b < 0 ⇒ Angle between a and b is obtuse.
a ⋅b
5. The dot product of a zero and non-zero vector is a scalar zero. ⇒ cos θ =
|a| ⋅|b|
 a ⋅b 
Geometrical interpretation of scalar product ⇒ θ = cos −1  
|a| ⋅|b|
Let a and b be two vectors represented by OA and OB
If a = a1 $i + a2 $j + a3k$ and b = b1 i$ + b 2 $j + b 3k$
respectively. Let θ be the angle between OA and OB. Draw
BL ⊥ OA and AM ⊥ OB.  a1 b1 + a2 b 2 + a3 b3 
θ = cos −1  
 a + a + a b + b + b 
2 2 2 2 2 2
 1 2 3 1 2 3
Chap 02 Product of Vectors 65

Sol. (a) Angle between a and b is given by


Properties of Scalar Product a ⋅b
(i) Commutativity The scalar product of two vector is cos θ =
| a | | b|
commutative i.e., a ⋅ b = b ⋅ a.
(2i$ + 2$j − k$ ) ⋅ (6$i − 3$j + 2k$ )
(ii) Distributivity of scalar product over vector =
addition The scalar product of vectors is distributive 22 + 22 + ( − 1) 2 ⋅ 6 2 + ( − 3) 2 + 2 2
over vector addition i.e., 12 − 6 − 2 4
= =
(a) a ⋅ ( b + c ) = a ⋅ b + a ⋅ c (Left distributivity) 3 ⋅7 21
(b) ( b + c ) ⋅ a = b ⋅ a + c ⋅ a (right distributivity)
y Example 2. ( a ⋅ $i ) $i + ( a ⋅ $j ) $j + ( a ⋅k$ ) k$ is equal to
(iii) Let a and b be two non-zero vectors a ⋅ b = 0 ⇔ a ⊥ b.
(a) a (b) 2a
As $i, $j and k $ are mutually perpendicular unit vectors
(c) 3a (d) 0
along the coordinate axes, therefore
$i ⋅ $j = $j ⋅ $i = 0, $j ⋅ k
$ =k
$ ⋅ $j = 0; k
$ ⋅ $i = $i ⋅ k
$ =0 Sol. (a) Let a = a1i$ + a 2 $j + a 3 k$
∴ a ⋅ $i = (a1i$ + a 2 $j + a 3 k$ ) $i = a1,
(iv) For any vector a, a ⋅ a = | a |2 .
a ⋅ $j = a 2 ,a ⋅ k$ = a 3
As $i, $j and k
$ are unit vectors along the coordinate
So, (a ⋅ $i )$i + (a ⋅ $j ) $j + (a ⋅ k$ ) k$
axes, therefore $i ⋅ $i = | $i|2 = 1, $j ⋅ $j = | $j |2 = 1 and
$ ⋅k
$ =|k$|2 = 1. = a i$ + a $j + a k$ = a
1 2 3
k
(v) If m is a scalar and a and b be any two vectors then y Example 3. If | a| = 3,| b| = 4, then a value of λ for
(ma ) ⋅ b = m(a ⋅ b ) = a ⋅ (mb ). which a + λb is perpendicular to a − λb.
(vi) If m and n are scalars and a and b be two vectors, then (a) 9 / 16 (b) 3 / 4
(ma ) ⋅ nb = mn (a ⋅ b ) = (mna ) ⋅ b = a ⋅ (mnb ) (c) 3 / 2 (d) 4 / 3
(vii) For any vectors a and b, we have Sol. (b) a + λb is perpendicular to a − λb.
(a) a ⋅ ( −b ) = − (a ⋅ b ) = ( − a ) ⋅ b ∴ ( a + λb ) ⋅ ( a − λb ) = 0
(b) ( −a ) ⋅ ( −b ) = a ⋅ b ⇒ | a 2 | − λ ( a ⋅ b ) + λ ( b ⋅ a ) − λ 2 | b| 2 = 0
(viii) For any two vectors a and b, we have ⇒ | a | 2 − λ 2 | b| 2 = 0
(a) | a + b|2 = | a|2 + | b|2 + 2a ⋅ b |a | 3
⇒ λ=± =±
(b) | a − b| = | a| + | b| − 2a ⋅ b
2 2 | b| 4
(c) (a + b ) ⋅ (a − b ) = | a|2 − | b|2
y Example 4. The projection of a = 2$i + 3$j − 2k$ on
(d) | a + b| = | a| + | b | ⇒ a || b
b = $i + 2$j + 3k$ is
(e) | a + b|2 = | a|2 + | b|2 ⇒ a ⊥ b
1 2
(f) | a + b| = | a − b| ⇒ a ⊥ b (a) (b)
14 14
−2
Scalar Product in Terms of Components (c) 14 (d)
If a = a 1 $i + a 2 $j + a 3 k
$ and b = b 1 $i + b 2 $j + b 3 k
$ , then 14
a ⋅ b = a 1b 1 + a 2b 2 + a 3b 3 . Sol. (b) Projection of a on b
Thus, scalar product of two vectors is equal to the b (2$i + 3$j − 2k$ ) ⋅ ( $i + 2$j + 3k$ )
=a⋅ =
sum of the products of their corresponding | b| | i$ + 2$j + 3k$ |
components. In particular, a ⋅ a = | a| 2 = a 12 + a 22 + a 32 . =
2+6−6
=
2
14 14
y Example 1. If θ is the angle between the vectors
a = 2$i + 2$j − k$ and b = 6 $i − 3$j + 2k$ , then y Example 5. If a = 5$i − $j + 7k$ and b = $i − $j + λk$ , then
4 3 find λ such that a + b and a − b are orthogonal.
(a) cosθ = (b) cosθ =
21 19 Sol. Clearly, a + b = (5$i − $j + 7 k$ ) + ( $i − $j + λk$ )
2 5
(c) cosθ = (d) cosθ = = 6$i − 2$j + (7 + λ )k$
19 21
66 Textbook of Vector & 3D Geometry

and a − b = (5$i − $j + 7 k$ ) − ( $i − $j + λk$ ) y Example 8. If the vectors a = (c log 2 x ) $i − 6 $j + 3 k$


= 4 $i + (7 − λ ) k$ and b = (log x ) $i + 2 $j + (2c log x ) k$ make an obtuse
2 2
Since, a + b and a − b are orthogonal angle for any x ∈ (0, ∞ ). Then, determine the interval to
∴ (a + b ) ⋅ (a − b ) = 0 which c belongs.
⇒ [6i$ − 2$j + (7 + λ ) k$ ] ( 4 $i + (7 − λ ) k ) = 0 Sol. For the vectors a and b to be inclined at an obtuse angle, we
⇒ 6 × 4 − 2 × 0 + (7 + λ ) (7 − λ ) = 0 must have
⇒ 24 + 49 − λ 2 = 0 a ⋅ b < 0, ∀ x ∈ (0, ∞ )
⇒ c (log 2 x )2 − 12 + 6c log 2 x < 0 , ∀ x ∈ (0, ∞ )
⇒ λ 2 = 73
⇒ cy 2 + 6cy − 12 < 0, ∀ y ∈ R, where y = log 2 x
⇒ λ = ± 73
⇒ c < 0 and 36c 2 + 48c < 0,
y Example 6. If a , b and c are unit vectors such that (using ax 2 + bx + c < 0,
a + b + c = 0, then find the value of a ⋅b + b ⋅c + c ⋅ a . ∀ x ∈ R iff a < 0 and D < 0)
Sol. Consider, a + b + c = 0  4 
⇒ c < 0 and c (3c + 4 ) < 0 ⇒ c ∈  − , 0
On squaring both sides, we get  3 
( a + b + c ) 2 = 02
y Example 9. If a + 2b + 3c = 4, then find the least
⇒ (a + b + c) ⋅ (a + b + c) = 0 ⋅ 0
⇒ | a | 2 + | b | 2 + | c| 2 + 2a ⋅ b+ 2b ⋅ c + 2c ⋅ a = 0
value of a 2 + b 2 + c 2 .
⇒ 1 + 1 + 1 + 2 (a ⋅ b + b ⋅ c + c ⋅ a ) = 0 Sol. Consider vectors p = a$i + b$j + ck$ and q = $i + 2$j + 3k$
⇒ a ⋅ b + b ⋅ c + c ⋅a = −
3 a + 2b + 3c
Now, cosθ =
2 a + b + c 2 12 + 22 + 32
2 2

y Example 7. If a, b and c are mutually perpendicular (a + 2b + 3c )2


⇒ cos 2 θ = ≤1 [Q cos 2 θ ≤ 1]
vectors of equal magnitudes, then find the angle 14 (a 2 + b 2 + c 2 )
between the vectors a and a + b + c . (a + 2b + 3c )2 16 8
⇒ a2 + b2 + c 2 ≥ = =
Sol. Let θ be the angle between the vectors a and a + b + c. 14 14 7
a ⋅ (a + b + c) 8
Then, cos θ = Hence, least value of a 2 + b 2 + c 2 is .
|a | |a + b + c 7
a ⋅a + a ⋅ b + a ⋅ c
= y Example 10. Find the unit vector which makes an
| a | | a + b + c|
angle of 45° with the vector 2$i + 2$j − k$ and an angle
|a |2
=
| a | | a + b + c| of 60° with the vector $j − k$ .
[Qa ⋅ b = a ⋅ c = 0 as a , b, c are mutually Sol. Let the unit vector be c = c 1$i + c 2 $j + c 3 k$ so that; it makes
perpendicular vectors]
an angle of 45° with 2$i + 2$j − k$ .
|a |
= …(i) 2c 1 + 2c 2 − c 3
| a + b + c| ⇒ = cos 45° (Q| c$| = 1)
3
Now consider, 3
| a + b + c | 2 = (a + b + c) ⋅ (a + b + c) ⇒ 2c 1 + 2c 2 − c 3 = …(i)
2
= | a | 2 + | b| 2 + | c| 2 + 2(a ⋅ b + b ⋅ c + c ⋅ a ) Also, it makes an angle of 60° with $j − k$ .
= 3| a | 2 + 2(0) = 3| a | 2 c2 − c3
⇒ = cos 60° (Q| $j − k$ | = 2 and | c$ | = 1)
[Q| a | = | b| = | c| and a ⋅ b = b ⋅ c = c ⋅ a = 0] 2
| a + b + c| = 3 | a | 2
⇒ c2 − c3 = …(ii)
From Eq. (i), we get 2
1 ⇒ c 12 + c 22 + c 32 = 1 …(iii)
cosθ =
3 (using | c| = | c 1$i + c 2 $j + c 3 k$ | = 1)
1
⇒ θ = cos −1 1 −c 1
3 From Eq. (ii), c 2 = + c 3 and from Eq. (i), c 1 = 3 +
2 2 2 2
Chap 02 Product of Vectors 67

On substituting in Eq. (iii), we get y Example 13. Using vector method, prove that in a
c 32 1 2c 3 1
+ +
c
+ − 3 + c 32 + c 32 =1 triangle, a = b cos C + c cos B (projection formula).
4 2 2 8 2 2 Sol. In a ∆ ABC,
9 2 3c 3 5 1 1
⇒ c3 + + = 1 ⇒ c3 = − , Let AB = c, BC = a , CA = b
4 2 2 8 2 3 2
Q a+b + c=0
 1 1
Hence, the required vectors are  , 0, −  and A
 2 2
 1 4 1 
 , , . c b
3 2 3 2 3 2

y Example 11. Show that the median to the base of an


B C
isosceles triangle is perpendicular to base. a

Sol. Let ABC be an isosceles triangle in which AB = AC . We have, | a | = | − ( b + c)|


Let A be the origin of reference and let ∴ a ⋅ a = − ( b + c) ⋅ a
AB = b, AC = c
A or |a |2 = − b ⋅ a − c ⋅ a
Let D be the middle point of BC. = −| b | | a | cos ( π − C ) − | c| | a | cos ( π − B )
b+c b c Since, the angle between b and a = ( π − C ) and angle
Then, AD =
2 between c and a = ( π − B )
Now, BC = c − b ∴ a 2 = ab cos C + ac cos B
B C
 b + c D ⇒ a = b cos C + c cos B
∴ AD ⋅ BC =   ⋅( c − b )
 2 
1 1
= (| c| 2 − | b | 2 ) = (| AC | 2 − | AB| 2 )
2 2
Components of a Vector Along and
1
= ( 0) = 0
Perpendicular to Another Vector
2 Let a and b be two vectors represented by OA and OB and
Hence, median to the base of an isosceles triangle is let θ be the angle between a and b. Draw MB⊥OA,
perpendicular to base. shown as
y Example 12. Using vector method, prove that in a ∴ b = OM + MB
triangle, a 2 = b 2 + c 2 − 2bc cos A (cosine law). ⇒ OM = (OM )a = (OB cos θ ) a = (| b|cos θ ) a
 (a ⋅ b ) 
Sol. In a ∆ABC, =  | b| a
Let AB = c, BC = a , CA = b  | a| 2 
Q a+b+c=0
B
B
b

a
c

q q
q p= q
O A
C M a
b A

We have, a = − ( b + c) a ⋅ b a ⋅ b a  a − b 
∴ | a | = | − ( b + c)| ⇒ | a | 2 = | b + c| 2 = a= = a
 a  a a  a 2 
⇒ | a | 2 = | b | 2 + | c | 2 + 2b ⋅ c
⇒ | a 2 | = | b| 2 + | c| 2 + 2| b| | c|| cos( π − A )|
b = OM + MB
Since, angle between b and c = The angle between CA a ⋅ b
MB = b − OM = b −  2  a
produced and AB  | a| 
∴ a 2 = b 2 + c 2 − 2bc cos A
68 Textbook of Vector & 3D Geometry

Thus, the components of b along and perpendicular to a Work done = | f | | OA| cos θ = f ⋅ OA
a ⋅ b a ⋅ b = f ⋅ d, where d = OA
are  2  a and a −  2  a respectively.
 | a|   | a|  ∴ Work done = (Force) ⋅ (Displacement)

y Example 14. If a = 4 $i + 6 $j and b = 3$j + 4 k$ , then find Remarks


1. The work done by the resultant of a number of forces
the component of a along b. f1f2, f3, K fn in a displacement d of a particle is equal to the
(a ⋅ b ) sum of work done by the forces separately
Sol. The component of vector a along b is
| b| 2 i.e. Work done = f1 ⋅ d + f2 ⋅ d + ....+ fn ⋅ d
18 $ = ( f1 + f2 + ...+ fn ) ⋅ d
= (3 j + 4 k$ ) = R⋅d where, R = f1 + f2 + ...+ fn
25
2. The work done by a force f when its point of application
experiences a number of consecutive displacements
y Example 15. Express the vector a = 5$i − 2$j + 5 k$ as d1, d2, d3K,dn , is equal to the work done by the forces in single
sum of two vectors such that one is parallel to the displacement from the beginning to end.
i. e., Work done = f ⋅ ( d1 + d2 + ....+ dn )
vector b = 3$i + k$ and the other is perpendicular to b.
= The work done by the force f in the single displacement from
Sol. Required vectors are the beginning to end
a ⋅ b a ⋅ b
 2  b and a −  2  b y Example 16. Two forces f1 = 3$i − 2$j + k$ and
 | b|   | b| 
f2 = $i + 3$j − 5 k$ acting on a particle at A move it to B.
a ⋅ b
Clearly,  2  b = 2(3i$ + k$ ) = 6$i + 2k$ and so, Find the work done if the position vector of A and B
 b 
  are −2$i + 5 k$ and 3$i − 7 $j + 2 k$ .
a − b Sol. Let R be the resultant of two forces f1 and f 2 and d be the
a−  b = (5i$ − 2$j + 5k$ ) − (6$i + 2k$ )
 b2 displacement.
 
Then, R = (3$i − 2$j + k$ ) + ( $i + 3$j − 5k$ )
= − $i − 2$j + 3k$
= 4 i$ + $j − 4 k$
Note that (6$i + 2k$ ) + ( − $i − 2$j + 3k$ )
and d = (3$i − 7 $j + 2k$ ) − ( −2$i + 5k$ ) = 5$i − 7 $j − 3k$
= 5$i − 2$j + 5k$ = a
∴ The total work done = The work done by resultant
= R ⋅ d = ( 4 $i + $j − 4 k$ ) ⋅ (5$i − 7 $j − 3k$ )

Application of Dot Product in = 20 − 7 + 12 = 25 units

Mechanics (Work done) y Example 17. Forces of magnitudes 5 and 3 units


acting in the directions 6 $i + 2$j + 3k$ and 3$i − 2$j + 6k$ ,
A force acting on a particle is said to do work, if the
particle is displaced in a direction which is not respectively act on a particle which is displaced from
perpendicular to force. the point (2, 2, − 1) and (4, 3, 1). Find the work done by
the forces.
Let a particle be placed at O and a force f represented by
OB be acting on the particle at O. Due to the application of Sol. Let R be the resultant of two forces and d be the
displacement.
force f, the particle is displaced in the direction of OA. Let
Then,
OA be the displacement.
(6$i + 2$j + 3k$ ) (3$i − 2$j + 6k$ )
B R =5 +3
f
36 + 4 + 9 9 + 4 + 36
1
= (39 $i + 4 $j + 33k$ )
7
q and d = ( 4 $i + 3$j + k$ ) − (2$i + 2$j − k$ ) = 2$i + $j + 2k$
O A
1
∴ Total work done = R ⋅ d = (78 + 4 + 66)
Then, the component of OA in the the direction of force f 7
is, | OA | cos θ =
148
units
7
Chap 02 Product of Vectors 69

Exercise for Session 1


1. Find the angle between the vectors $i − 2$j + 3k$ and 3$i − 2$j + k$ .

2. Find and angle between two vectors a and b with magnitudes 3 and 2 respectively such that a ⋅ b = 6

3. Show that the vectors 2$i − $j + k$ and $i − 3$j − 5k$ are at right angles.

4. If r ⋅ $i = r ⋅ $j = r ⋅ k$ and | r | = 3, then find vector r.

5. Find the angle between the vectors a + b and a − b, if a = 2$i − $j + 3k$ and b = 3$i + $j − 2k$ .

6. Find the projection of the vector $i + 3$j + 7k$ on the vector 7$i − $j + 8k$ .
1
7. If the projection of vector x$i − $j + k$ on vector 2$i − $j + 5k$ is , then find the value of x.
30

8. If | a | + | b | = | c | and a + b = c, then find the angle between a and b.

9. If three unit vectors a, b and c satisfy a + b + c = 0, then find the angle between a and b.

10. If a = x$i + ( x − 1) $j + k$ and b = ( x + 1) $i + $j + ak$ make an acute angle, ∀ x ∈ R, then find the values of a.

11. Find the component of $i in the direction of the vector $i + $j + 2k$ .

12. Find the vector components of a vector 2$i + 3$j + 6k$ along and perpendicular to the non-zero vector 2$i + $j + 2k$ .

13. A particle is acted upon by constant forces 4$i + $j − 3k$ and 3$i + $j − k$ which displace it from a point $i + 2$j + 3k$ to
the point 5$i + 4$j + k$ . Find the work done by the forces in standard units.
Session 2
Vector or Cross Product of Two Vectors, Area of
Parallelogram and Triangle, Moment of a Force
and Couple, Rotation About an Axis
Vector or Cross Product (viii) The vector product of two non-zero vectors is zero
vector iff they are parallel (collinear) i.e. a × b = 0
of Two Vectors ⇔ a | | b, a and b are non-zero vectors.
Let a and b be two non-zero, non- parallel vectors. Then It follows from the above property that a × a = 0 for
the vector product a × b, in that order, every non-zero vector a, which in turn implies that
is defined as a vector whose magnitude $i × $i = $j × $j = k
$ ×k
$ = 0.
is
(ix) Vector product of orthonormal triad of unit vectors
| a || b | sin θ b
$i, $j and k
$ using the definition of the vector product
where θ is the angle between a and b, obtain
q
whose direction is perpendicular to the $i × $j = k
$ , $j × k
$ = $i, k
$ × $i = $j,
plane of a and b in such a way that a, b a
$j × $i = − k$, k$ × $j = − $i, $i × k$ = − $j
and this direction constitute a right
handed system. (x) Lagrange’s identity If a, b are any two vectors, then
In other words, a × b = | a | | b | sinθ n, where θ is the angle | a × b |2 = | a|2 | b |2 − (a ⋅ b ) 2
between a and b, n$ is a unit vector perpendicular to the or | a × b |2 + (a ⋅ b ) 2 = | a |2 | b |2
plane of a and b such that a, b and n form a right handed
system.
Vector Product in Terms
Properties of Vector Product of Components
(i) Vector product is not commutative i.e., if a and b are If a = a 1 $i + a 2 $j + a 3 k
$ and b = b 1 $i + b 2 $j + b 3 k $
any two vectors, then a × b ≠ b × a, however Then, a × b = (a 2 b 3 − a 3 b 2 ) $i − (a 1 b 3 − a 3 b 1 ) $j
a × b = − ( b × a ).
 $i $j $
k
(ii) Vector product is not associative, $ = a 1
i.e. a × ( b × c ) ≠ (a × b ) × c + (a 1 b 2 − a 2 b 1 ) k a2 a 3
 
(iii) If a and b are two vectors and m is a scalar, then b1 b2 b 3
ma × b = m (a × b) = a × mb
y Example 18. If a = 2$i + 3$j − 5k$ and b = m $i + n$j + 12k$
(iv) If a and b are two vectors and m and n are scalars,
then ma × nb = mn (a × b ) = m (a × b ) = n (ma × b ) and a × b = 0. Then, find the values of m and n.
(v) Distributivity of vector product over vector addition.  i$ $j k$ 
Let a, b and c be any three vectors. Then, Sol. Clearly, a × b = 2 3 − 5
 
(a) a × ( b + c ) = a × b + a × c (left distributivity) 
m n 12

(b) ( b + c ) × a = b × a + c × a (right distributivity) = i$ (36 + 5n ) − $j (24 + 5m ) + k$ (2n − 3m )
(vi) For any three vectors a, b and c, we have Since, a × b = 0
a × ( b − c ) = a × b − a × c. ∴(36 + 5n ) $i − (24 + 5m ) $j + (2n − 3m ) k$ = 0$i + 0$j + 0k$

(vii) The vector product of any vector (zero or non-zero) On comparing the coefficients of $i, $j and k, $ we get

with zero vector is a zero vector i.e. 36 + 5n = 0, − (24 + 5m ) = 0 and 2n − 3m = 0


36 24
a × 0 = 0 ×a = 0 ⇒ n=− and m = −
5 5
Chap 02 Product of Vectors 71

y Example 19. Show that ( a − b ) × ( a + b ) = 2 ( a × b ) y Example 23. If a , b and c are three non-zero vectors
Sol. Consider, (a − b ) × (a + b ) = ( a − b ) × a + (a − b ) × b
such that a ⋅ (b × c ) = 0 and b and c are not parallel
vectors, prove that a = λb + µc where λ and µ are
[By distributivity of vector product over vector addition]
scalar.
=a ×a − b ×a +a × b − b × b
Sol. We have, a ⋅ ( b × c) = 0
[Again, by distributivity of vector product over vector
addition] ⇒ a = 0 or b × c = 0 or a ⊥ ( b × c)
= 0 + a × b + a × b − 0 [Qa × b = − ( b × a )] ⇒ a = 0 or b | | c or a ⊥ ( b × c)
= 2 (a × b ) Hence Proved But a ≠ 0 and b ≠ c
∴ a ⊥ ( b × c)
y Example 20. If a is any vector, then ⇒ a lies in the plane of b and c.
( a × $i ) 2 + ( a × $j ) 2 + (a × k$ ) 2 is equal to ⇒ a , b and c are coplanar.
(a)| a |2 (b) 0 ⇒ a = λb + µc

(c) 3 | a | 2
(d) 2 | a |2 y Example 24. If a × b = a × c , a ≠ 0, show that
Sol. (d) Let a = a1i$ + a 2 $j + a 3 k$ b = c + ta for some scalar t.
∴ a × $i = (a1$i + a 2 $j + a 3 k$ ) × $i = − a 2 k$ + a 3 $j Sol. We have, a×b =a×c
⇒ a × b −a × c = 0
(a × $i )2 = (a × $i ) ⋅ (a × $i )
⇒ a × ( b − c) = 0
= ( − a 2 k$ + a 3 $j) ⋅ ( − a 2 k$ + a 3 $j) = a 22 + a 32 ⇒ a = 0 or ( b − c) = 0 or a | | ( b − c)
Similarly, (a × $j)2 = a 32 + a12 ⇒ a | | ( b − c) (Qa ≠ 0 and b ≠ 0)
and (a × k$ )2 = a 2 + a 2 ⇒ b − c = ta (for some scalar t)
1 2
⇒ b = c + ta
∴ (a × $i )2 + (a × $j)2 + (a × k$ )2
= 2 (a12 + a 22 + a 32 ) = 2 | a | 2 y Example 25. For any two vector u and v, prove that
(i) (u ⋅ v ) 2 + | u × v | 2 = |u | 2 | v | 2
y Example 21. If a ⋅ b = 0 and a × b = 0, prove that (ii) (1 + | u | 2 ) (1 + | v | 2 ) = (1 − u ⋅ v ) 2 + | u + v + (u × v )| 2
a = 0 or b = 0.
Sol. (i) To show ( u ⋅ v )2 + | u × v 2 | = | u | 2 | v | 2
Sol. Given, a ⋅ b = 0 and a × b = 0
Let θ be the angle between u and v.
Now, a ⋅ b = 0 ⇒ a = 0 or b = 0 or a ⊥ b
⇒ u ⋅ v = uv cos θ
and a × b = 0 ⇒ a = 0 or b = 0 or a | | b
and | u × v | = uv sin θ
Since, a ⊥ b and a || b can never hold simultaneously.
⇒ ( u ⋅ v )2 + | u × v | 2 = u 2v 2 cos 2 θ + u 2v 2 sin 2 θ
∴ a ⋅ b = 0 and a × b = 0
⇒ a = 0 or b = 0 ⇒ ( u ⋅ v )2 + | u × v | 2 = u 2v 2
⇒ ( u ⋅ v )2 + | u × v | 2 = | u | 2 | v | 2
y Example 22. If a , b and c are vectors such that
(ii) Taking RHS (1 − u ⋅ v )2 + | u + v + ( u × v ) | 2
a ⋅ b = a ⋅ c , a × b = a × c and a ≠ 0, then show that
b =c ⇒ 1 + ( u ⋅ v )2 − 2u ⋅ v + | [ u + v + ( u × v )]
Sol.a ⋅ b = a ⋅ c and a ≠ 0 ⋅ [ u + v + ( u × v )] |
⇒ a ⋅ b − a ⋅ c = 0 and a ≠ 0 ⇒ 1 + | u | 2 | v | 2 cos 2 θ − 2 | u | | v | cos θ + u ⋅ u
⇒ a ⋅ ( b − c) = 0 and a ≠ 0 + u ⋅ v + u (u × v)
⇒ a ⊥ ( b − c) or b = c …(i) + v ⋅ u + v ⋅ v + v ⋅ (u × v) + (u × v) + (u × v)⋅ u
+ (u × v) ⋅ v + (u × v) ⋅ (u × v)
Again, a × b = a × c and a ≠ 0
⇒ 1 + | u | 2 | v | 2 cos 2 θ − 2 | u | | v | cos θ + | u | 2 + | u | |
⇒ a × ( b − c) = 0 and a ≠ 0
⇒ a | | ( b − c) = 0 and b = c …(ii)
v | cosθ + 0
+ | u | v | cos θ + | v 2 + 0 + 0 + 0 + | u + v | 2
∴ From Eqs. (i) and (ii), we have
b=c ⇒ 1 + | u | 2 | v| 2
cos 2 θ + | u | 2 + | v | 2 + | u | 2 | v | 2 sin 2 θ
[as a cannot be both parallel and ⇒ 1 + | u |2 | v |2 + | u |2 + | v |2
perpendicular to ( b − c)]
= (1 + | u | 2 ) (1 + | v | 2 ) = LHS
72 Textbook of Vector & 3D Geometry

a×b
Angle between Two Vectors $ =
Thus, n
|a × b|
is a unit vector perpendicular to the
|a × b|
If θ is the angle between a and b , then sin θ = . a×b
|a || b| plane of a and b. Note that − is also a unit vector
|a × b|
Expression for sin θ perpendicular to the plane of a and b. Vectors of
If a = a 1 $i + a 2 $j + a 3 k
$ , b = b 1` $i + b 2 $j + b 3 k
$ and θ be magnitude ‘λ’ normal to the plane of a and b are given by
angle between a and b, then λ (a × b )
±
(a 2 b 3 − a 3 b 2 ) + (a 1 b 3 − a 3 b 1 ) + (a 1 b 2 − a 2 b 1 )
2 2 2
|b × b|
sin2 θ =
(a 12 + a 22 + a 23 ) (b 12 + b 22 + b 23 )
y Example 28. The unit vector perpendicular to the
y Example 26. The sine of the angle between the vectors 6 $i + 2$j + 3k$ and 3$i − 6 $j − 2k,
$ is
vector a = 3$i + $j + k$ and b = 2$i − 2$j + k$ is 2 $i − 3$j + 6 k$ 2 $i − 3$j − 6 k$
(a) (b)
74 25 7 7
(a) (b)
99 99 2 i + 3$j − 6 k$
$ 2 i + 3$j + 6 k$
$
(c) (d)
37 5 7 7
(c) (d)
99 41
Sol. (c) Let a = 6i$ + 2$j + 3k$ and b = 3$i − 6$j − 2k$
 $i $j k$  $i $j k$ 
Sol. (a) a × b = 3 1 1 = 3$i − $j − 8k$ a × b = 6 2 3
   
2 −2 1 3 −6 −2
   
|a × b | 74 74 = 14 i$ + 21$j − 42k$ = 7 (2i$ + 3$j − 6k$ )
sin θ = = =
|a || b | 11 ⋅ 9 99
| a × b | = 7 | 2$i + 3$j − 6k$ | = 7 ⋅ 7
y Example 27. If | a | = 2,| b | = 5 and | a × b | = 8, then a×b 1
∴ = (2$i + 3$j − 6k$ )
find the value of a ⋅ b. |a × b | 7
which is a unit vector perpendicular to a and b.
Sol. We have, | a | = 2, | b | = 5
and | a × b | = 8. y Example 29. Find unit vectors perpendicular to the
Let θ be the angle between a and b. plane determined by the points
|a × b | 8 4 P (1, − 1, 2), Q (2 , 0, − 1) and R (0, 2 , 1)
Then, sin θ = = =
|a || b | 2×5 5
Sol. Clearly, required unit vector is a unit vector perpendicular
Now, cos θ = ± 1 − sin 2 θ to the plane of PQ and PR.
16 3 Now,
= ± 1− =±
25 5 PQ = (2$i − k$ ) − ( $i − $j + 2k$ )
 3 = $i + $j − 3k$
a ⋅ b = | a | b | cosθ = ± 2 ⋅ 5 ⋅  = ± 6
 5
PR = (2$j + k$ ) − ( $i − $j + 2k$ )
= − $i + 3$j − k$
Vector Normal to the Plane  $i $j k$ 
of Two Given Vectors and PQ × PR = 1 1 − 3 = 8$i + 4 $j + 4 k$
 
If a and b are two non-zero, non-parallel vectors and let θ −1 3 −1
 
be the angle between them. a × b = | a| | b | sin θ n,
$ where n
$
is a unit vector perpendicular to the plane of a and b such ∴ Required unit vectors
$ form a right handed system.
that a, b, n PQ × PR (8$i + 4 $j + 4 k$ )
=± =±
⇒ (a × b ) = | a × b | n
$ | PQ × PR | 4 6
a×b 1 $ $ $
=± ( 2i + j + k )
⇒ $ =
n 6
|a × b|
Chap 02 Product of Vectors 73

a
y Example 30. Let A, B and C be unit vectors. b
Suppose A ⋅B = A ⋅C = 0 and the angle between B
π
and C is . Then,
4
(a) A = ± 2 ( B × C ) (b) A = ± 2 ( B × C ) c
(c) A = ± 3 ( B + C ) (d) A = ± 3 ( B × C )
y Example 31. The vectors c , a = x $i + y $j + z k$ and
Sol. (b) Since, A ⋅ B = 0
b = $j are such that a , c and b form a right handed
⇒ A ⊥ B and A ⋅ C = 0
⇒ A⊥C system, then c is
B×C (a) z i − x k$
^
∴ A=±
(b) 0
| B × C|
(c) y $j (d) − z $i + x k$
[Q A is a unit vector perpendicular
to both B and C] Sol. (a) a , c and b form a right handed system.
π Hence, b ×a = c
Here, | B × C | = | B | | C |sin
4 ⇒ c = $j × (x i$ + y $j + z k$ )
1 1
= 1⋅1⋅ = = − x k$ + z i$ = z i$ − x k$
2 2
( B × C) y Example 32. If a , b and c are three non-zero vectors
So, A=± = ± 2 ( B × C)
1
such that a × b = c and b × c = a , prove that a , b and c
2 are mutually at right angles and | b | = 1 and | c | = | a |.
Sol. a × b = c and a = b × c
Right Handed System and Left ⇒ c ⊥a, c ⊥ b and a ⊥ b, a ⊥ c ⇒ a ⊥ b, b ⊥ c and c ⊥ a
⇒ a , b and c are mutually perpendicular vectors.
Handed System of Vectors Again,a × b = c and b × c = a
(i) Right handed system of vectors Three mutually ⇒ |a × b | = | c | and | b × c | = | a |
perpendicular vectors a, b and c from a right handed π π
system of vector iff a × b = c, b × c = a, c × a = b. ⇒ | a | | b | sin = | c | and | b | | c | sin = | a |
2 2
b ⇒ | a | | b |= | c |
and | b || c| = |a | (Qa ⊥ b and b ⊥ c)
⇒ | b |2 | c | = | c |
a
⇒ | b |2 = 1 ⇒ | b | = 1
c On putting in | a | | b | = | c | ⇒ | a | = | c |

For example, the unit vectors $i, $j and k $ form a right


handed system, $i × $j = k
$ , $j × k
$ = $i, k
$ × $i = $j Geometrical Interpretation
Y of Vector Product
j
If a and b are two non-zero, non-parallel vectors
represented by OA and OB respectively and let θ be the
angle between them. Complete the parallelogram OACB.
X
i Draw BL ⊥ OA.
k
n
Z B C

(ii) Left handed system of vectors The vectors a, b and


c mutually perpendicular to one another form a left b
handed system of vectors iff
c × b = a, a × c = b, b × a = c. q
O L a A
74 Textbook of Vector & 3D Geometry

BL 1
In ∆OBL, sin θ = Sol. Area of ∆ABC = | AB × AC|
OB 2
Now, AB = Position vector of B − Position vector of A
⇒ BL = OB sin θ = | b | sin θ
AB = b − a
Now, a × b = | a | | b | sin θ n A a O
= (OA ) ( BA ) n
= (Base × Height) n = (Area of parallelogram OACB) n b c
= Vector area of the parallelogram OACB
Thus, a × b is a vector whose magnitude is equal to the
area of the parallelogram having a and b as its adjacent B C
sides and whose direction n is perpendicular to the plane
AC = Position vector of C − Position vector of A
of a and b such that a, b and n form a right handed system.
AC = c − a
Hence, a × b represents the vector area of the
AB × AC = ( b − a ) × ( c − a )
parallelogram having adjacent sides along a and b.
= b × c− b ×a −a × c+a ×a (Qa × a = 0)
= a × b + b × c+ c×a

Area of Parallelogram 1
Hence, area of ∆ABC = | AB × AC |
2
and Triangle 1
= |a × b + b × c+ c×a | = 0
2
(i) The area of a parallelogram with adjacent sides a and
is | a × b |. If the points A , B and C are collinear, then area of ∆ABC = 0
1
(ii) The area of a parallelogram with diagonals d 1 and d 2 ⇒ |a × b + b × c+ c × a | = 0
2
1
is | d 1 × d 2 |. ⇒ |a × b + b × c+ c×a | = 0
2
⇒ a × b + b × c+ c×a = 0
(iii) The area of a plane quadrilateral ABCD is Thus, a × b + b × c+ c×a = 0
1
| AC × BD |, where AC and BD are its diagonals. is the required condition of collinearity of three points a , b
2 and c.
(iv) The area of a triangle with adjacent sides a and b is
1 y Example 34. Show that the perpendicular distance of
| a × b |. the point c from to the joining a and b is
2
1 |b × c + c × a + a × b |
(v) The area of a ∆ABC is | AB × AC | |b − a |
2
1
or | BC × BA | Sol. Let ABC be a triangle and let a , b and c be the position of
2 its vertices A , B and C respectively. Let CM be the
1 perpendicular from C on AB.
or | CB × CA |
2 1 1
Then, area of ∆ABC = ( AB ) ⋅ (CM ) = | AB | | CM |
(vi) If a, b and c are position vectors of a ∆ABC, then its 2 2
1 1
area = | (a × b ) + ( b × c ) + (c × a ) | Also, area of ∆ABC = | a × b + b × c + c × a |
2 2
C(c)
Remark
Three points with position vectors a , b and c are collinear, if
( a × b) + ( b × c ) + ( c × a ) = 0

y Example 33. If a , b and c are position vectors of the


vertices A, B and C of ∆ABC, show that the area of A(a) M B(b)
1 1 1
∆ABC is | a × b + b × c + c × a |. ∴ | AB | | CM | = | a × b + b × c + c × a |
2 2 2
Deduce the condition for points a , b and c to be | b × c+ c×a +a × b |
⇒ CM =
collinear. | b −a |
Chap 02 Product of Vectors 75

r×F
y Example 35.
(i) Find the area of the quadrilateral whose diagonals F
O
are given by
3 $i + $j − 2k$ , $i − 3 $j + 4k$ r
Q

(ii) A1 , A 2 , ..., An are the vertices of a regular plane q q


polygon with n sides. O is the centre. Show that P
n −1 H
∑ (OAi × OA i + 1 ) = (1 − n) (OA 2 × OA 1 )
If several forces are acting through the same point P
i =1
then the vector sum of the moment of the separate
1 forces about O is equal to the moment of their
Sol. (i) Area of the quadrilateral = | d1 × d 2 |
2 resultant force about O.
 i$ $j k$ 
1
1 − 2 = | − 2i$ − 14 $j − 10k$ |
1 Remark
= 3
2  2 Moment of a force F about a point A = AB × F, where B is any
1 −3 4
  point on F.
1 10 3
= 4 + 196 + 100 = =5 3 (ii) About a line The moment of a force F acting at a
2 2
point P about a line L is a scalar given by ( r × F ) ⋅ a$ ,
(ii) A1, A 2 , ..., An are the vertices of a regular plane polygon
where a is a unit vector in the direction of the line
of n sides and centre O .
and OP = r, where O is any point on the line.
Let | OA i | = k , ∀ i = 1 < 2, 3 ,...., n
Let e$ i be the unit vector along OA i Thus, the moment of a force F about a line is the
OA i = ke$ i resolved part (component) along this line, of the
moment of F about any point on the line.
OA i × OA i + 1 = ke$ i × ke$ i + 1 = k 2 x$ i
where x$ i is a unit vector in the direction perpendicular to Remark
the plane of the polygon and x$ i = x$ i + 1 for The moment of a force about a point is a vector while the moment
i = 1, 2, 3, ....., n − 1 about a straight line is a scalar quantity.
n −1 n −1
∴ LHS = ∑ ( OA i × OA i + 1 ) = k 2 ∑ x$ i y Example 36. Find the moment about (1, − 1, − 1) of the
i =1 r =1

= k 2 (n − 1) x$ i = (n − 1) ( OA i × OA 2 )
force 3$i + 4 $j − 5 k$ acting at (1, 0, − 2).
= (1 − n ) ( OA 2 × OA 1 ) Sol. Let A ≡ (1, − 1, − 1), B ≡ (1, 0, − 2)
and F = 3$i + 4 $j − 5k$

Moment of a Force and Couple A F

Moment of a Force B
(i) About a point Let a force F be applied at a point P of
Then, moment of force F about A is given by AB × F.
a rigid body then, the moment of F about a point O
measures the tendency of F to turn the body about Here, AB = ( $i − 2k$ ) − ( $i − $j − k$ ) = $j − k$
point O. If this tendency of rotation about O is in  $i $j k$ 
anti-clockwise direction, the moment is positive, ∴ AB × F = 0 1 − 1
otherwise it is negative.  
3 4 −5
 
Let r be the position vector of P relative to O. Then,
the moment or torque of F about the point O is = $i ( − 5 + 4 ) − $j (0 + 3) + k$ (0 − 3)
defined as the vector M = r × F. = − $i − 3$j − 3k$
76 Textbook of Vector & 3D Geometry

y Example 37. Three forces $i + 2$j − 3 k$ , 2$i + 3$j + 4 k$ =


30 $
( 4 i + 48$j − 8k$ )
13
and $i − $j + k$ acting on a particle at the point (0, 1, 2). F
The magnitude of the moment of the forces about the 2i – 2j + k M
point (1, − 2, 0) is 12i – 4j – 3k
(a) 2 35 (b) 6 10
(c) 4 7 (d) None of these (0, 0, 0)
O r P
Sol. (b) Total force F = ( $i + 2$j − 3k$ ) + (2$i + 3$j + 4 k$ )
+ ( $i − $j + k$ ) = 4 $i + 4 $j + 2k$
Let a be unit vector in the direction of 2$i − 2$j + k$ . Then,
Moment of the forces about
P = r × F = PA × F 2$i − 2$j + k$ 1
a= = (2$i − 2$j + k$ )
PA = (0 − 1) $i + (1 + 2) $j + (2 − 0) k$ 4+ 4 +1 3

= − $i + 3$j + 2k$ Thus, the moment of F about the given line


30
∴ Moment about P = ( − $i + 3$j + 2k$ ) × ( 4 $i + 4 $j + 2k$ ) = M ⋅ a = (14 $i + 48$j − 8k$ )
13
 $i $j k$  1 760
⋅ (2i$ − 2$j + k$ ) = −
= − 1 3 2 = − 2$i + 10$j − 16k$ 3 13
 
4 4 2
 
F
Moment of a Couple
A system consisting of a pair of equal unlike parallel
forces is called a couple. The vector sum to two forces of a
(0, 1, 2) couple is always zero vector.
A
r q A N
(1, –2, 0) F q
P
r
Magnitude of the moment
= | − 2$i + 10$j − 16k$ |
O
= 2 12 + 52 + 82 = 2 90 = 6 10
B –F
y Example 38. Find the moment about a line through
the origin having the direction of 2$i − 2$j + k$ due to The moment of a couple is a vector perpendicular to the
30 kg force acting at a point ( − 4, 2, 5) in the direction plane of couple and its magnitude is the product of the
of 12$i − 4 $j − 3k$ . magnitude of either force with perpendicular distance
between the lines of the forces.
Sol. Let F be the force. Then,
M = r × F, where r = BA
30 (12$i − 4 $j − 3k$ ) 30
F= = (12$i − 4 $j − 3k$ ) | M | = | BA × F | = | F | | BA | sin θ
144 + 16 + 9 13
where θ is the angle between BA and F
Suppose the force F acts at point P ( − 4, 2, 5) the moment of
= | F | ( BN ) = | F|α
F acting at P about a line in the direction 2$i − 2$j + k$ is
equal to the resolve part along the line of moment of F where, α = BN is the arm of the couple and + ve or − ve
about a point on the line. sign is to be taken accordingly as the forces indicate a
∴ r = OP = ( − 4 $i + 2$j + 5k$ ) − (0) counter clockwise rotation or clockwise rotation.
= − 4 $i + 2$j + 5k$ y Example 39. The moment of the couple formed by
Let M be the moment F about O. Then, the forces 5$i + k$ and − 5$i − k$ acting at the points
 $i $j k$  (9, − 1, 2) and ( 3, − 2, 1) respectively, is
30
M = r×F = −4 2 5 (a) − $i + $j + 5 k$ (b) $i − $j − 5 k$
13 
12 − 4 − 3
  (c) 2 i$ − 2 $j − 10 k$ (d) − 2 i$ + 2 $j + 10 k$
Chap 02 Product of Vectors 77

Sol. (b) Moment of the couple, and AP = (3i$ + 6$j + 4 k$ ) − ( $i + $j + 2k$ )


F (5i + k) = 2$i + 5$j + 2k$
A
(9, –1, 2)
(3, –2, 1) w
B (1, 2, –2) B w
–F (–5i–k) v

= BA × F = {(9 − 3) $i + ( − 1 + 2) $j P (3, 6, 4)
+ (2 − 1) k$ } × (5$i + k$ ) r
A (1, 1, 2)
 i$ $j k$
= (6i + j + k ) × (5i + k ) = 6 1 1
$ $ $ $ $ 3 $
∴ ω= ( j − 4 k$ ) 2 and r
  17
5 0 1
  3 $
$ $ $ Now, v = ω × r = ( j − 4 k$ ) × (2$i + 5$j + 2k$ )
= i − j − 5k 17
3
= (22i$ − 8$j − 2k$ )
17
Rotation About an Axis y Example 41. A rigid body is spinning about a fixed
When a rigid body rotates about a fixed axis ON with an
point ( 3, − 2, − 1) with an angular velocity of 4 rad/s,
angular velocity ω, then velocity v of a particle P is given
the axis of rotation being in the direction of (1, 2, − 2) .
by
Find the velocity of the particle at the point (4, 1, 1).
v = ω × r,
 $i + 2$j − 2k$  4 $
were, r = OP Sol. ω = 4   = ( i + 2$j − 2k$ )
 1 + 4 + 4 3
and ω = | ω | (unit vector along ON )
r = OP − OA
N = ( 4 $i + $j + k$ ) − (3$i − 2$j − k$ )
w
= $i + 3$i + 2k$
4 $
v =ω × r= ( i + 2$j − 2k$ ) × ( $i + 3$j + 2k$ )
P 3
4
r ⇒ (10$i − 4 $j + k$ )
O
3

y Example 40. A particle has an angular speed of w

3 rad/s and the axis of rotation passes through the w


v
points (1, 1, 2) and (1, 2, − 2). Find the velocity of the
particle at point P ( 3, 6, 4 ). P (4, 1, 1)
Sol. Clearly, OA = i$ + $j + 2k$ r
(3, –2, –1) A
OB = i$ + 2$j − 2k$
AB = $j − 4 k$ = AB = 17
78 Textbook of Vector & 3D Geometry

Exercise for Session 2


1. Find | a × b |, if a = $i − 7$j + 7k$ and b = 3$i − 2$j − 2k$ .

2. Find the values of λ and µ for which (2$i + 6$j + 27k$ ) × ( $i + λ$j + µk$ ) = 0

3. If a = 2$i + 3$j − k$ , b = − $i + 2$j − 4k$ , c = $i + $j + k$ , then find the value of ( a × b) ⋅ ( a × c).

4. Prove that ( a ⋅ $i ) ( a × $i ) + ( a ⋅ $j) ( a × $j) + ( a × k$ ) ( a × k$ ) = 0.

5. If a × b = c × d and a × c = b × d, then show that a − d is parallel to b − c.

6. If ( a × b)2 + ( a ⋅ b)2 = 144 and | a | = 4, then find the value of | b |.

7. If | a | = 2, | b | = 7 and ( a × b) = 3$i + 2$j + 6k$ , find the angle between a and b.


2
8. Let the vectors a and b be such that | a | = 3, | b | = and a × b is a unit vector, then find the angle between a
3
and b.

9. If | a | = 26, | b | = 7, and | a × b | = 35, find a ⋅ b.

10. Find a unit vector perpendicular to the plane of two vectors a = $i − $j + 2k$ and b = 2$i + 3$j − k$ .

11. Find a vector of magnitude 15, which is perpendicular to both the vectors 4$i − $j + 8k$ and − $j + k.
$

12. Let a = $i + 4$j + 2k$ , b = 3$i − 2$j + 7k$ and c = 2$i − $j + 4k$ .
Find a vector d which is perpendicular to both a and b and c ⋅ d = 15.
π
13. Let a, b and c be unit vectors such that a ⋅ b = 0 = a ⋅ c. If the angle between b and c is , then find a.
6

14. Find the area of the triangle whose adjacent sides are determined by the vectors
a = −2$i − 5k$ and b = $i − 2$j − k$ .

15. Find the area of the parallelogram whose adjacent sides are represented by the vectors
3$i + $j − 2k$ and $i − 3$j + 4k$ .

16. Show that the area of the parallelogram having diagonals 3$i + $j − 2k$ and $i − 3$j + 4k$ is 5 3.

17. A force F = 2$i + $j − k$ acts at point A whose position vector is 2$i − $j. Find the moment of force Fabout the origin.

18. Find the moment of Fabout point (2, − 1, 3), when force F = 3$i + 2$j − 4k$ is acting on point (1, − 1, 2).

19. Forces 2$i + $j, 2$i − 3$j + 6k$ and − $i + 2$j − k$ act at a point P, with position vector 4$i − 3$j − k$ . Find the moment of
the resultant of these force about the point Q whose position vector is 6$i + $j − 3k$ .
Session 3
Scalar Triple Product

Scalar Triple Product Height of parallelopiped


The scalar triple product is defined for three vectors and it Volume of parallelopiped
=
is defined as the dot product of one of the vectors with the Area of base
cross product of the other two.
If a, b, c are any three vector, then their scalar product is
defined as (a × b ) ⋅ c. Properties of Scalar Triple Product
We denote it by [a, b, c ]. (i) If a, b and c are given by
It is also called the mixed or box product. a = a 1 $i + a 2 $j + a 3 k
$
b = b 1 $i + b 2 $j + b 3 k
$
Remark
Result of scalar triple product is always a scalar.
c = c 1 $i + c 2 $j + c 3 k
$
a1 a2 a3
Geometrical Interpretation of Scalar Then, (a × b ) ⋅ c = b 1 b2 b3
c1 c2 c3
Triple Product
Let a, b and c be three vectors. Consider a parallelopiped (ii) (a × b ) ⋅ c = a ⋅ ( b × c ) i.e. position of dot and cross can
having coterminous edges OA, OB and OC such that be interchanged without altering product. Hence, it is
OA = a, OB = b and OC = c. Then, a × b is a vector also represented by [a b c ].
perpendicular to the plane of a and b. Let φ be the angle (iii) [a b c ] = [ b c a ] = [c a b ]
between c and a × b. (iv) [a b c ] = − [ b a c ]
If n is a unit vector along a × b, then φ is the angle (v) [k a b c ] = k [a b c ], [k 1 a k 2 b k 3 c ], = k 1 k 2 k 3 [a b c ]
between n and c.
(vi) [a + b c d ] = [a c d ] + [ b c d ]
Now, [a b c ] = ( a × b ) ⋅ c
(vii) a, b and c in that order form a right handed system, if
C F [a b c ] > 0.
n c a
E G
f

b
B b
O c
a
A D

= (Area of parallelogram OADB) n ⋅ c (viii) The necessary and sufficient condition for three
= (Area of parallelogram OADB) ( n ⋅ c ) non-zero, non-collinear vector a, b and c to be
= (Area of parallelogram OADB) (| c | | n | cos φ ) coplanar is that [a b c ] = 0 i.e., a, b and c are coplanar
⇔ [a b c ] = 0.
= (Area of parallelogram OADB) (| c | cos φ )
(ix) [ x 1 a + y 1 b + z 1 c, x 2 a + y 2 b + z 2 c, x 3 a + y 3 b + z 3 c ]
= (Area of parallelogram OADB ) (OL )
x 1 y1 z1
= Area of base × height
= x 2 y 2 z 2 [a b c ]
= Volume of parallelopiped
x3 y3 z3
80 Textbook of Vector & 3D Geometry

Remarks y Example 43. Let a = x $i + 12$j − k$ , b = 2$i + 2x$j + k$ and


1. Four points A, B, C, D are coplanar if [ AB, AC, AD] = 0
c = $i + k$ . If b, c,a in that order form a left handed
2. Four points a , b, c and d are coplanar, if
[ d b c ] + [ d c a ] + [ d a b] = [ a b c ] system, then find the value of x.
or [ a b c ] + [ a c d] + [ a d b] = [ d b c ] [x 1 a + y 1b + z1 c , x 2 a + y 2b + z 2 c , x 3 a + y 3 b + z 3 c]
3. [ a a b] = [ b b a ] = [ c c b] = 0 x 1 y 1 z1
i.e., if any two vectors are same, then vectors are coplanar.
= x 2 y 2 z 2 [ a b c]
Volume of Tetrahedron x 3 y 3 z3
(A pyramid having a triangular base)
Sol. Since, b, c,a form a left handed system, three fore
If OABC is a tetrahedron as shown in figure, where
[ b, c,a ] < 0
OA = a, OB = b, and OC = c, then volume of
2 2x − 1
1
tetrahedron = [a b c ] ⇒ 1 0 1 <0
6
x 12 −1
O
⇒ 2(0 − 12) − 2x ( −1 − x ) + 1(12 − 0) < 0
⇒ − 24 + 2x + 2x 2 + 12 < 0
a ⇒ 2x 2 + 2x − 12 < 0 ⇒ x 2 + x − 6 < 0
b c
⇒ ( x − 2) ( x + 3) < 0 ⇒ x ∈ ( −3, 2)
A
y Example 44. For any three vectors a , b and c prove
B C
that [ a + b b + c c + a ] = 2 [ a b c ]
Sol. We have, [a + b b + c c + a ]
Remarks = {(a + b ) × ( b × c)} ⋅( c + a )
1. The six mid-points of the six edges of a tetrahedron lie in a = {a × b + a × c + b × b + b × c} ⋅ (c + a )
sphere, if the pair of opposite edges are perpendicular to each
other. {Q b × b = 0}
2. Centre of the sphere is the centroid of the tetrahedron. = {a × b + a × c + b × c} ⋅ ( c + a )
3. GA2 + GB2 + GC2 + GO 2 = 12r 2, G being the centroid.
= (a × b ) ⋅ c + (a × c) ⋅ c + ( b × c) ⋅ c
4. The angle between any two plane faces of a regular
1 + (a × b ) ⋅ a + (a × c) ⋅ a + ( b × c) ⋅ a
tetrahedron is cos −1 .
3 = [a b c] + 0 + 0 + 0 + 0 + [ b c a ]
5. Angle between the any edge and a face not containing the [Q [a c c] = 0, [ b c c] = 0,
1
angle is cos −1 (for regular tetrahedron). [a b a ] = 0, [a c a ] = 0]
3
6. Any two edges of regular tetrahedron are perpendicular to = [a b c] + [a b c] = 2[a b c]
each other.
7. The distance of any vertex from the opposite face of regular y Example 45. If a , b and c are coplanar show
2
tetrahedron is k, k being the length of any edge. [ a + b b + c c + a ] are coplanar.
3
Sol. Since, [a b c] are coplanar
y Example 42. Find the volume of the parallelopiped [a b c] = 0 …(i)
whose edges are represented by a = 2$i − 3$j + 4k$ , and shown in above example
[a + b b + c c + a ] = 2 [a b c ] = 0 [using Eq. (i)]
b = $i + 2$j − k$ and c = 3$i − $j + 2k$ .
which shows [a + b b + c c + a ] are coplanar, if [a b c ] are
2 −3 4 coplanar.
Sol. Here, [a b c] = 1 2 − 1 = 2( 4 − 1) + 3(2 + 3) + 4( − 1 − 6)
y Example 46. For any three vectors a , b and c prove
3 −1 2
a ⋅ a a ⋅b a ⋅c
= 6 + 15 − 28 = − 7
that [ a b c ] = b ⋅ a b ⋅b b ⋅c
2
∴ The volume of the parallelopiped = [a b c] = 7.
c ⋅ a c ⋅b c ⋅c
Chap 02 Product of Vectors 81

Sol. Let a = a1$i + a 2 $j + a 3 k$ y Example 49. If a , b and c are any three vectors in
b = b1$i + b 2 $j + b 3 k$ space, then show that
c = c 1$i + c 2 $j + c 3 k$
(c + b ) × (c + a ) ⋅(c + b + a ) = [ a b c ]
a1 a 2 a 3 a1 a 2 a 3 Sol. Here, ( c + b ) × ( c + a ) ⋅ ( c + b + a )
Then, [a b c ] [a b c ] = b1 b 2 b 3 b1 b 2 b 3 ⇒ ( c × c + c × a + b × c + b × a ) ⋅ (c + b + a )
⇒ (c × a + b × c + b × a) ⋅ (c + b + a) (Q c × c = 0)
c1 c 2 c 3 c1 c 2 c 3
⇒ ( c × a ) ⋅ c + ( c × a ) ⋅ b + ( c × a ) ⋅ a + ( b × c) ⋅ c + ( b × c) ⋅ b
On multiplying row-by-row, we get + ( b × c) ⋅ a + ( b × a ) ⋅ c + ( b × a ) ⋅ b + ( b × a ) ⋅ a
[a b c ]2 = ⇒ 0 + [ c a b ] + 0 + 0 + 0 + [ b c a ] + [ b a c] + 0 + 0
a1a1 + a 2a 2 + a 3a 3 a1b1 + a 2b 2 + a 3b 3 a1c 1 + a 2c 2 + a 3c 3 ⇒ [a b c] + [a b c] − [a b c] (Q c a b = a b c)
b1a1 + b 2a 2 + b 3a 3 b1b1 + b 2b 2 + b 3b 3 b1c 1 + b 2c 2 + b 3c 3 ⇒ [a b c] (Q [ b a c] = − [a b c])
c 1a1 + c 2a 2 + c 3a 3 c 1b1 + c 2b 2 + c 3b 3 c 1c 1 + c 2c 2 + c 3c 3 y Example 50. If u, v and w are three non-coplanar
a ⋅a a ⋅b a ⋅c vectors, then (u + v − w ) ⋅ [(u − v ) × ( v − w )] is equal to
= b ⋅a b⋅b b⋅c (a) 0 (b) u ⋅ (v × w)
c ⋅a c⋅ b c⋅ c (c) u ⋅ (w × v) (d) 3u ⋅ (v × w)
Sol. (b) ( u + v − w ) ⋅ [ u − v × ( v − w )]
y Example 47. If a , b, c, l and m are vectors, prove that
⇒ ( u + v − w ) ⋅ [( u × v ) × ( u × w ) − 0 + ( v × w )]
a b c
= [u u v ] + [v u v ] − [w u v ] − [u u w ] − [v u w ]
[ a b c] (l × m) = a ⋅ l b ⋅ l c ⋅ l + [w u w ] + [u v w ] + [v v w ] − [w v w ]
a ⋅m b ⋅m c ⋅m = 0 + 0 − [u v w ] − 0 + [u v w ] + 0 + [u v w ] + 0 − 0
= [u v w ] = u ⋅ ( v × w )
Sol. Let a = a1$i + a 2 $j + a 3 k$ , b = b1$i + b 2 $j + b 3 k$ ,
c = c1$i + c 2$j + c 3k$ , l = l1$i + l 2$j + l 3k$ y Example 51. If a, b and c are non-coplanar vectors
and
^
m = m1$i + m2$j + m3 k. and λ is a real number, then the vector a + 2b + 3c ,
a1 a 2 a 3 $i $j k$ λb + 4c and (2λ − 1) c are non-coplanar for
⇒ [a b c ]( l × m ) = b1 b 2 b 3 l1 l 2 l 3 (a) no value of λ
c 1 c 2 c 3 m1 m 2 m 3 (b) all except one value of λ
(c) all except two values of λ
On multiplying row-by-row, we get (d) all values of λ
a1$i + a 2 $j + a 3 k$ a1l1 + a 2l 2 + a 3l 3 a1m1 + a 2m 2 + a 3m 3 Sol. (c) Since, a, b and c are non-coplanar vectors.
= b1$i + b 2 $j + b 3 k$ b1l1 + b 2l 2 + b 3l 3 b1m1 + b 2m 2 + b 3m 3 ∴ [a b c] ≠ 0
c 1$i + c 2 $j + c 3 k$ b1l1 + b 2l 2 + b 3l 3 c 1m1 + c 2m 2 + c 3m 3 Now, a + 2b + 3c, λb + 4 c and (2λ − 1) c will be
non-coplanare iff
a a ⋅l a ⋅m a b c
(a + 2b + 3c) ⋅ {( λb + 4 c) × (2λ − 1) c} ≠ 0
= b b⋅l b⋅m = a ⋅l b⋅l c⋅ l i.e., ( a + 2b + 3c ) ⋅ { λ ( 2λ − 1) ( b × c ) ≠ 0
c c⋅ l c⋅ m a ⋅m b⋅m c⋅ m i.e., λ (2λ − 1) [a b c]} ≠ 0
1
∴ λ ≠ 0,
y Example 48. If a and b are non-zero and non- 2
collinear vectors, then show that Thus, given vectors will be non-coplanar for all values of λ
a × b = [a b i] $i + [a b j] $j + [a b k] k$ 1
except two values 0 and .
2
Sol. Let a × b = x$i + y$j + zk$ …(i)
(a × b ) ⋅ $i = ( x $i + y $j + z k$ ) ⋅ $i y Example 52. If x , y and z are distinct scalars such
(a × b ) ⋅ $i = x
that [ xa + yb + zc , xb + yc + za , xc + ya + zb ] = 0]
where a, b and c are non-coplanar vectors, then
Also, (a × b ) ⋅ $j = y …(ii)
(a) x + y + z = 0 (b) xy + yz + zx = 0
(a × b ) ⋅ k$ = z (c) x3 + y3 + z 3 = 0 (d) x 2 + y 2 + z 2 = 0
a × b = [a b i ] $i + [a b j] $j + [a b k ] k$ Sol. (a) a, b and c are non-coplanar.
∴ [a b c] ≠ 0
82 Textbook of Vector & 3D Geometry

Now, consider [ xa + yb + zc, xb + yc + za , xc + ya + zb ] = 0 Sol. (a) Since, the volume of tetrahedron with edges a , b and c is
x y z x y z [a b c].
⇒ z x y [a b c] = 0 ⇒ z x y = 0 [Q[a b c] ≠ 0] Where, a ⋅a = b ⋅ b = c ⋅ c = 1
y z x y z x 3
and a ⋅c = b⋅c = a ⋅c = (given)
2
⇒ ( x + y + z ) ( x 2 + y 2 + z 2 − xy − yz − zx ) = 0
1
1 ∴ V = [a b c]
⇒ ( x + y + z ) {( x − y )2 } + (y − z )2 + (z − x )2 } = 0 6
2 a ⋅a a ⋅ b a ⋅ c
⇒ x + y + z = 0 or x = y = z 1 1
⇒ V 2 = [a b c]2 = b ⋅a b ⋅ b b ⋅ c
But x , y and z are distinct 36 36
c ⋅a c ⋅ b c ⋅ c
∴ x + y + z = 0.
3 3
y Example 53. If a , b and c are three non-coplanar 1
2 2
uni-modular vectors, each inclined with other at an 1 3 3 1 3 3 5
= 1 =  − 
angle 30°, then volume of tetrahedron whose edges are 36 2 2 36  4 4
a , b and c is 3 3
1
3 3−5 2 2
3 3−5
(a) (b) 1
12 12 ∴ V = 3 3 −5
5 2 +3 12
(c) (d) None of these
12

Exercise for Session 3


1. If a and b are two vectors such that a × b = 2, then find the value of [ a b a × b].

2. If the vectors 2$i − 3$j , $i + $j − k$ and 3$i − k$ form three concurrent edges of a parallelopiped , the find the volume
of the parallelopiped.

3. If the volume of a parallelopiped whose adjacent edges are a = 2$i + 3$j + 4k$ , b = $i + α$j + 2k$ , c = $i + 2$j + αk$ is
15, then find the value of α, where α > 0.
4. The position vector of the four angular points of a tetrahedron are A ( $j + 2k$ ), B (3$i + k$ ), C (4$i + 3$j + 6k$ ) and
D = (2$i + 3$j + 2k$ ). Find the volume of the tetrahedron ABCD.

5. Find the altitude of a parallelopiped whose three coterminous edges are vectors A = $i + $j + k$ , B = 2$i + 4$j − k$
and C = $i + $j + 3k$ with A and B as the sides of the base of the parallelopiped.

6. Examine whether the vectors a = 2$i + 3$j + 2k$ , b = $i − $j + 2k$ and c = 3$i + 2$j + 4k$ from a left handed or a right
handed system.
7. Prove that the four points 4$i + 5$j + k$ , − ( $j + k$ ) , (3$i + 9$j + 4k$ ) and 4( − $i + $j + k)
$ are coplanar.

a ⋅ u b⋅ u c ⋅ u
8. Prove that [ a b c] [ u v w ] = a ⋅ v b ⋅ v c ⋅ v
a ⋅ w b⋅ w c ⋅ w

9. If [ a b c] = 2, then find the value of [( a + 2b − c) ( a − b) ( a − b − c)].

10. If a, b and c are three non-coplanar vectors, then find the value of
a ⋅ ( b × c) b ⋅ ( c × a) c ⋅ ( a × b)
+ +
b ⋅ ( c × a) c ⋅ ( a × b) a ⋅ ( b × c)
Chap 02 Product of Vectors 83

Session 4
Vector Triple Product

Vector Triple Product Remarks


It is defined for three vectors a, b and c as the vector 1. a × ( b × c ) is a linear combination of those two vectors which
a × ( b × c ). are with in brackets.
2. If r = a × ( b × c ), then r is perpendicular to a and lie in the plane
This vector being perpendicular to a and b × c. But b × c is of b and c.
a vector perpendicular to the plane of b and c. 3. a × ( b × c ) = ( a ⋅ c ) b − ( a ⋅ b) c
( a × b) × c = ( c ⋅ a ) b − ( c ⋅ b)a
Aid to memory
b×c I × (II × III) = (I ⋅ III) II − (I ⋅ II) III
a
4. ( a × b) × ( c × d) = (( a × b) ⋅ d) c − (( a × b) ⋅ c ) d
c = [ a b d] c − [ a b c ] d
⇒ The vector ( a × b) × ( c × d) lies in the plane of c and d.
a×(b×c)
Also, ( a × b) × ( c × d) = − ( c × d) × ( a × b)
b
= − {(( c × d) ⋅ b) a − (( c × d)⋅ ( a b)}
= − [ c d b] a + [ c d a ] b
∴a × ( b × c ) lie in the plane of b and c, i.e., it is coplanar Which shows that ( a × b) × ( c × d) lies in the plane of a and b.
with b and c. Thus, the vector ( a × b) × ( c × d) lies along the common
section of the plane of c,d and that of the plane of a , b.
i.e., a × ( b × c ) = lb + mc …(i)
Taking the scalar product of this equation with a, we get Lagrange’s Identity
Qa × ( b × c ) is ⊥ to a  a ⋅c a ⋅d
0 = l(a ⋅ b ) + m(a ⋅ c )  ∴ (a × ( b × c ) ⋅ a = 0  (a × b ) ⋅ (c × d ) =
  b ⋅c b ⋅d
⇒ l(a ⋅ b ) = − m(a ⋅ c ) Proof LHS = (a × b ) ⋅ (c × d ) = u ⋅ (c × d )
l m where u = a × b = ( u × c) ⋅ d
⇒ =− =λ (say)
a ⋅c a⋅b = ((a × b ) × c ) ⋅ d
⇒ l = λ(a ⋅ c ) = ((c ⋅ a ) ⋅ b − (c ⋅ a ) ⋅ a ) ⋅ d
and m = − λ(a ⋅ b ) = (c ⋅ a ) ( b ⋅ d ) − (c ⋅ b) (a ⋅ d )
Substituting the value of l and m in Eq. (i), we get = (a ⋅ c ) ( b ⋅ d ) − ( b ⋅ c ) (a ⋅ d )
a × ( b × c ) = λ [(a ⋅ c ) b − (a ⋅ b ) c ] a ⋅c a ⋅d
=
Here, the value of λ can be determined by taking specific b ⋅c b ⋅d
values of a, b and c.
If we choose the coordinate axes in such a way that, y Example 54. If a = $i + $j + k$ , b = $i + $j , c = $i and
a = a 1 $i, b = b 1 $i + b 2 $j ( a × b ) × c = λa + µb, then λ + µ is equal to
(a) 0 (b) 1
and c = c 1 $i + c 2 $j + c 3 k $,
(c) 2 (d) 3
it is easy to show that λ = 1.
Sol. (a) (a × b ) × c = λa + µb
Hence, a × ( b × c ) = (a ⋅ c ) b − (a ⋅ b ) c
⇒ (a ⋅ c) b − ( b ⋅ c) a = λa + µb
and a × ( b × c ) = (a × b ) × c, if some of all a, b and c are ⇒ λ = − b ⋅ c, µ = a ⋅ c
zero vectors or a and c are collinear. ∴ λ + µ = a ⋅ c − b ⋅ c = (a − b ) ⋅ c
= {( $i + $j + k$ ) − ( i + $j)} = k$ ⋅ $i = 0
84 Textbook of Vector & 3D Geometry

y Example 55. If a , b and c are three non-parallel unit y Example 58. Show that the vectors
1 a × ( b × c ), b × (c × a ) and c × ((a × b ) are coplanar.
vectors such that a × (b × c ) = b, then find the angles
2 Sol. Let p = a × ( b × c), q = b × ( c × a ) and r = c × (a × b ), then
which a makes with b and c. p + q + r = a × ( b × c) + b × ( c × a ) + c × (a × b ) = 0
1 ⇒ p = ( −1), q = ( −1) r
Sol. We have, a × ( b × c) = b
2 which shows p is linear combination of q and r.
1
⇒ (a ⋅ c) b − (a ⋅ b ) c = b So, p, q are coplanar.
2 Hence, a × ( b × c), b × ( c × a ) and c × (a × b ) are coplanar.
1
⇒ a ⋅ c = and a ⋅ b = 0 (comparing c and b)
2 y Example 59. Prove that [a × b b × c c × a ] = [a b c ] 2
1
⇒ a ⋅ c = and a ⊥ b Sol. We have, [a × b b × c c × a ]
2
1 = {(a × b ) × ( b × c)} ⋅ ( c × a )
Suppose a makes angle θ with c. Then, a ⋅ c = = {d × ( b × c)} ⋅ ( c × a ) [where, d = (a × b )]
2
1 1 = [(d ⋅ c) b − (d ⋅ b )c] ⋅ ( c × a )
⇒ | a | | c | cos θ = ⇒ cosθ = (Q| a | | c | ≠ 0) = [{(a × b ) ⋅ c) b − (a × b ) b} c]⋅ ( c × a )
2 2
π = {[a b c ] b − 0}. ( c × a ) [Q(a b b) = 0]
⇒ θ= = {[a b c] { b ⋅ ( c × a )}
3
π = [a b c] [ b c a ] {Q[a b c] = [ b c a ]}
Thus, a is perpendicular to b and makes an angle with c.
3 = [a b c]2

y Example 56. If a = − $i + $j + k$ and b = 2$i + k$ , then y Example 60. If a , b and c are coplanar show
find the vector x satisfying the conditions. [ a × b b × c c × a ] are coplanar.
(i) that it is coplanar with a and b. Sol. Since, [a b c] are coplanar.
(ii) it is perpendicular to b. ⇒ [a b c] = 0
(iii) a ⋅ x = 7 and [a × b b × c c × a ] = [a b c]2 = 0
Sol. Since, x is in the plane of a and b and is perpendicular to b. ∴[a × b b × c c × a ] are coplanar, if a, b and c are
coplanar.
∴ x = λ { b × (a × b )}
⇒ x = λ {( b ⋅ b ) a − (b ⋅ a )b} y Example 61. If A, B and C are vectors such that
= λ {5( − $i + $j + k$ ) − ( −1) (2$i + k$ )} | B | = | C |, prove that
= λ { −5$i + 5$j + 5k$ + 2i$ + k}
$ {( A + B ) × ( A + C )} × (B × C ) ⋅ (B + C ) = 0.
Sol. Let R1 = A + B, R 2 = A + C, R 3 = B + C
= λ { −3i$ + 5$j + 6k}
$
∴ LHS = {( A + B ) × ( A + C)} × ( B × C)} × ( B × C) ⋅ ( B + C)
Now, a ⋅ x = 7
⇒ {( R1 × R 2 ) × ( B × C)} ⋅ R 3
⇒ − 3λ + 5λ + 6λ = 7
⇒ [{ R1 ⋅ ( B × C)} R 2 − { R 2 ⋅ ( B × C) R1 }] ⋅ R 3
7
⇒ 8λ = 7 ⇒ λ = ⇒ [ A + B B C ] [ R 2 ⋅ R 3 ] − [ A + C B C ] ( R1 ⋅ R 3 )
8
7 ⇒ {[ A B C] + [ B B C]} [( A + C) ⋅ ( B + C)] − {[ A B C]
Hence, x = ( −3i + 5$j + 6k$ )
$
+ [ C B C]} [( A + B) ⋅ ( B + C)]
8
⇒ [ A B C] ( A ⋅ B + A ⋅ C + C ⋅ B + C ⋅ C) − [ A B C]
y Example 57. Prove that (A ⋅ B + A ⋅ C + B ⋅ B + B ⋅ C)
a × (b × c ) + b × (c × a ) + c × ( a × b ) = 0 ⇒ [ A B C] ( A ⋅ B + A ⋅ C + C ⋅ B + | C | 2 − A ⋅ B
Sol. We have, a × ( b × c) + b × ( c × a ) + c × (a × b ) − A ⋅ C + B ⋅ B + B ⋅ C)
= {(a ⋅ c ) b − (a ⋅ b ) c} + {( b ⋅ a ) c − ( b ⋅ c ) a} ⇒ [ A B C] (| C | − | B | )
2 2

+ {( c ⋅ b ) a − ( c ⋅ a ) b}
⇒ [ A B C] ( 0 ) (Q| B | = | C | )
= [(a ⋅ c ) b − (a ⋅ b ) c + (a ⋅ b ) c − ( b ⋅ c ) a
⇒ 0 = RHS
+ ( b ⋅ c ) a − (a ⋅ c ) b ] = 0
Chap 02 Product of Vectors 85

y Example 62. If b and c are two non-collinear y Example 63. Find the set of vectors reciprocal to
vector such that a || (b × c ), then prove that the set of vectors 2$i + 3$j − k$ , $i − $j − 2k$ , − $i + 2$j + 2k$
( a × b ) ⋅ ( a × c ) is equal to | a | 2 (b ⋅ c ). Sol. Let the given vector be a , b, c.
Sol. Since, a || ( b × c), therefore a ⊥ b and a ⊥ c 2 3 −1
⇒ a ⋅ b = 0 and a ⋅ c = 0 Now, [a b c ] = 1 − 1 −2
a ⋅a a ⋅c a ⋅a 0 −1 2 2
Now, consider (a × b ) ⋅ (a × c) = =
b ⋅a b⋅c 0 b⋅c = 2( −2 + 4 ) − 3(2 − 2) − 1 (2 − 1)
= (a ⋅ a ) ( b ⋅ c) = | a | ( b ⋅ c).
2 = 4 −1=3
$i $j k$
b × c = 1 −1 −2 = 2$i + k$
Reciprocal System of Vectors −1 2 2
The two system of vectors are called reciprocal system of
$i $j k$
vectors if by taking dot product, we get unity.
Thus, if a, b and c be three non-coplanar vectors and if. c × a = −1 2 2 = − 8$i + 3$j − 7 k$
b ×c c ×a a×b 2 3 −1
a′ = , b′ = and c ′ =
a bc a bc a bc $i $j k$
Then a ′ , b ′ and c′ are said to be reciprocal system of a × b = 2 3 −1 = − 7 $i + 3$j − 5k$
vectors for the vector a, b and c. 1 −1 −2

Remarks b × c 2$i + k$
Hence, a′ = =
1. If a , b, c and a ′, b′, c ′ are reciprocal system of vectors, then [a b c] 3
a ⋅ (b × c ) [a b c ]
a ⋅a′ = = =1 c×a − 8$i + 3$j − 7 k$
(a b c ) [a b c ] b′ = =
Similarly, b ⋅ b′ = c ⋅ c ′ = 1
[a b c] 3
2. a ⋅ b′ = a ⋅ c ′ = b ⋅ a ′ = b ⋅ a ′ = c ⋅ a ′ = c ⋅ b′ = 0 a×b −7 i + 3$j − 5k$
$
and c′ = =
3. [ a b c ] ⋅ [ a ′ b′ c ′ ] = 1 [a b c] 3
Proof : We have,
 b×c c ×a a ×b  y Example 64. Find a set of vector reciprocal to the
[ a ′ b′ c ′ ] =  = = 
[a b c ] [a b c ] [a b c ]  vectors a , b and a × b.
1 1
= [ b × c c × a a × b] = [a b c ] 2 Sol. Let the given vectors be denoted by a , b and c where
[a b c ] 3 [a b c ] 3
c = a × b.
1
= ∴ [a b c ] = (a × b ) ⋅ c = (a × b ) = (a × b )2 …(i)
[a b c ]
∴ [ a ′ b′ c ′ ] ⋅ [ a b c ] = 1 and let the reciprocal system of vector be a ′ , b ′ and c′.
4. The orthogonal triad of vectors $i , $j and k$ is self reciprocal. b×c b × (a × b )
∴ a′ = =
Let $i , $j and k$ be the system of vectors reciprocal to the system [a b c ] [a × b ]2
$i , $j and k$ then, we have, c ×a (a × b ) × a
b′ = =
$ $
$i = j × k = $j [a b c ] [a × b ]2
[ $i $j k$ ] a×b a×b
c′ = =
Similarly, $j ′ = $j and k$ ′ = k$ [a b c ] [a × b ]2
5. a, b and c are non-coplanar iff a ′, b′, c ′ are non-coplanar. ∴a ′ , b ′ and c′ are required reciprocal system of vectors for
Q [ a b c ] [ a ′ b′ c ′ ] = 1 a , b and a × b.
1
∴ [ a ′ b′ c ′ ] =
[a b c ] b×c c×a a ×b
y Example 65. If a ′ = , b′ = , c′ = ,
So, [ a b c ] ≠ 0 ⇔ [ a ′ b′ c ′ ] ≠ 0 [a b c] [a b c] [a b c]
Thus, a , b and c are non-coplanar iff [ a ′ b′ c ′ ] are non-coplanar. then show that
6. If a , b, c are non-coplanar vectors, then a × a ′ + b × b ′ + c × c ′ = 0, where a , b and c are
[ x b c ] a + [ x c a ] b + [ x a b] c
r= non-coplanar.
[a b c ]
86 Textbook of Vector & 3D Geometry

a × ( b × c)
Sol. Here, a × a ′ = Solving of Vector Equations
[a b c]
(a ⋅ c ) b − (a ⋅ b ) c Solving a vector equation means determining an unknown
a × a′ = …(i) vector (or a number of vectors satisfying the given
[a b c ]
conditions)
Similarly,
( b ⋅a ) c − ( b ⋅ c) a Generally, to solve vector equations we express the
b × b′ = …(ii) unknown as the linear combination of three non-coplanar
[a b c]
vector as; r = xa + yb + z(a × b); as a, b and a × b are
( c ⋅ b )a − ( c ⋅ a ) b
c × c′ = …(iii) non-coplanar and find x , y and z using given conditions.
[a b c]
Sometimes, we can directly solve the given condition it
a × a ′ + b × b′ + c × c′
would be more clear from some examples.
(a ⋅ c ) b − (a ⋅ b ) c + ( b ⋅ a )c − ( b ⋅ c ) a
+ (c ⋅ b )a − (c ⋅ a ) b y Example 67. Solve the vector equation r × b = a × b,
=
[a b c ] r ⋅c = 0 provided that c is not perpendicular to b.
(Qa ⋅ b = b ⋅ a ) Sol. We are given,
=0 r× b=a × b
y Example 66. If (e 1 , e 2 , e 3 ) and (e 1 ′ , e 2 ′ , e 3 ′ ) are two ⇒ (r − a ) × b = 0
Hence, ( r − a ) and b are parallel.
sets of non-coplanar vectors such that $i = 1, 2, 3, ⇒ r − a = tb
1, if $i = $j and we known r ⋅ c = 0, …(i)
we have e i ⋅ e j′ =  $ $ , then show that ∴ Taking dot product of Eq. (i) by c, we get
0, if i ≠ j
r ⋅ a − a ⋅ c = t( b ⋅ c )
[e 1 , e 2 , e 3 ], [e 1′ e 2′ e 3′ ] = 1. ⇒ 0 − a ⋅ c = t( b ⋅ c )
Sol. We have, e 1 ⋅ e 2′ = 0, e 1 ⋅ e 3′ = 0  a ⋅c
⇒ t =−  …(ii)
 b ⋅c
⇒ e 1 ⊥ e 2′ and e 1 ⊥ e 3′
∴ e 1 || (e 2′ × e 3′ ) ∴ From Eqs. (i) and (ii) solution of r is
 a ⋅c
∴ e 1 = λ (e 2′ × e 3′ ) …(i) r=a −   b
 b ⋅c
e 1 ⋅ e 1 ′ = λ (e 2′ × e 3′ ) ⋅ e 1′
y Example 68. Solve for x, such that A ⋅ X = C and
1 = λ [e 1′ e 2′ e 3′ ] (Qe 1 e 1 ′ = 1, given)
A × X = B with C ≠ 0.
1
λ= Sol. We have, A × X = B
[e 1′ e 2′ e 3′ ]
From Eq. (i), Taking vector product of both sides with A, we get
e2′ × e3′ A × B = A × (A × X)
e1 = = (A ⋅ X) A − (A ⋅ A ) X
[e 1′ e 2′ e 3′ ]
= CA − | A | 2 X
e 3 ′ × e1 ′
Similarly, e2 =
[e 1′ e 2′ e 3′ ] (using A ⋅ X = C and A ⋅ tA = | A | 2 )
e1 ′ × e 2 ′ ⇒ | A | 2 X = CA − A × B
and e3 =
[e 1′ e 2′ e 3′ ] CA + B × A
or X=
[e 2′ × e 3′e 3′ × e 1′ e 1′ × e 2′ ] | A |2
∴ [e 1 e 2 e 3 ] =
[e 1′ e 2′ e 3′ ]3
y Example 69. Solve the vector equation r × a + kr = b,
⇒ [e 1, e 2 ,e 3 ][e 1′ e 2′ e 3′ ]3 = [e 2′ × e 3′ e 3′ × e 1′ e 1′ × e 2′ ] …(ii)
where a and b are two given vector and k is any scalar.
Now, [e 2′ × e 3′ e 3′ × e 1′ e 1′ × e 2′ ] = [e 1′ e 2′ e 3′ ]2 …(iii) Sol. Since, a , b and a × b are two non-coplanar vectors.
∴ From Eqs. (ii) and (iii), we get r = xa + yb + z( a × b ) …(i)
[e 1 e 2 e 3 ] [e 1′ e 2′ e 3′ ]3 = [e 1′ e 2′ e 3′ ]2 (where, x , y and z scalars)
[e 1 e 2 e 3 ] [e 1′ e 2′ e 3′ ] = 1
Chap 02 Product of Vectors 87

On putting r in r × a + kr = b, we get y Example 70. Solve for vectors A and B, where


{ xa + yb + z ( a × b )} × a + k { xa + yb + z ( a × b )} = b A + B = a , A × B = b, A ⋅ a = 1
⇒y ( b × a) + z {( a ⋅ a) b − ( a ⋅ b ) a} + k { xa + yb
Sol. We have, A + B=a
+ z ( a × b )} = b
⇒ A ⋅a + B ⋅a = a ⋅a
⇒ {kx − z ( a ⋅ b )} a + {ky + z ( a ⋅ a)} b + {(y + zk )} ( a × b ) = b
⇒ 1 + B ⋅a = a 2 (given A ⋅ a = 1)
⇒ kx − z ( a ⋅ b ) = 0, ky + z ( a ⋅ a) = 1
⇒ − y + zk = 0 ⇒ B ⋅a = a − 1
2
…(i)
On solving these equations, we get Also, A×B=b
1 ⇒ a × ( A × B) = a × b
z= 2 ,
k + | a |2 ⇒ (a ⋅ B )A − (a ⋅ A )B = a × b
a⋅ b ⇒ ( a 2 − 1) A − B = a × b
x=
k(| a | 2 + k 2 ) [using Eq. (i) and a ⋅ A = 1] …(ii)
k and A +B=a
and y =
k 2 + | a |2 From Eqs. (i) and (ii), we get
(a × b ) + a  (a × b ) + a 
On putting theses value in Eq. (i), we get the solution, A= and B = a −  
(a ⋅ b ) k 1 a2  a2 
r= + (b ) + 2 (a × b )
k(k 2 + | a | 2 ) k 2 + | a | 2 k + | a |2 ( b × a ) + a ( a 2 − 1)
⇒ B=
1 (a ⋅ b ) a  a2
r= + (k ) b + ( a × b ) is required solution.
k + | a | 2  k
2
 (a × b ) + a ( b × a ) a ( a 2 − 1)
Thus, A = and B =
a2 a2

Exercise for Session 4


1. Find the value of α × (β × γ ), where α = 2$i − 10$j + 2k,
$ β = 3$i + $j + 2k,
$ γ = 2$i + $j + 3k.
$

2. Find the vector of length 3 unit which is perpendicular to $i + $j + k$ and lies in the plane of $i + $j + k$ and 2$i − 3$j.

3. Show that ( b × c). a × d) + ( c × a) ( b × d) + ( a × b) ( c × d) = 0

4. Prove that $i × ( a + $i ) + $j × ( a × $j) + k$ × ( a × k$ ) = 2a.

5. Prove that [ a × b a × c d] = ( a ⋅ d) [ a b c]
b+ c
6. If a, b and c are non-coplanar unit vector such that a × ( b × c) = , b and c are non-parallel, then prove that
2

the angle between a and b is .
4

7. Find a set of vectors reciprocal to the set of vectors − $i + $j + k$ , $i − $j + k$ , $i + $j + k$ .

8. If a, b, c and a′ , b′ , c′ are reciprocal system of vectors, then prove that


a+ b+ c
a′ × b′ + b′ × c′ + c′ × a′ = .
[ a b c]

9. Solve r × b = a, where a and b are given vectors such that a ⋅ b = 0.

10. Find vector r, if r ⋅ a = m and r × b = c, where a ⋅ b ≠ 0.


JEE Type Solved Examples :
Only One Option Correct Type Questions
l Ex. 1 If | a | = 5, | a − b | = 8 and | a + b | =10, then | b | is $i 4 $j
l Ex. 4 Let a , b > 0 and α = + + bk$ and
equal to a b
(a) 1 (b) 57 1 10
β = b $i + a$j + k$ , then the maximum value of is
(c) 3 (d) None of these b 5 + α ⋅β
Sol. (b) We know that for any two vectors (a) 1 (b) 2
| a + b | 2 + | a − b | 2 = 2 (| a | 2 + | b | 2 ) (c) 4 (d) 8
b 4a
⇒ (10 ) 2 + (8 ) 2 = 2[(5 ) 2 + | b | 2 ] Sol. (a) α ⋅β = + + 1 ≥5 (Q AM ≥ GM )
a b
⇒ 100 + 64 = 50 + 2 | b | ⇒ | b | = 57
2 2
 10 
So,   = 1.
∴ | b | = 57  5 + α ⋅ β max

l Ex. 2 Angle between diagonals of a parallelogram whose l Ex. 5 If the unit vectors e1 and e 2 are inclined at an
sides are represented by a = 2 $i + $j + k$ and b = $i − $j − k$ angle 2θ and | e1 − e 2 | < 1, then for θ ∈[0, π ], θ may lie in the
 1  1 interval
(a) cos − 1   (b) cos − 1  
3  2  π π π
(a)  0,  (b)  , 
 4
(c) cos − 1  
 5
(d) cos − 1    6 6 2
 9  9  5π   π 5π 
(c)  , π (d)  , 
Sol. (a) Let c and d be the diagonals of parallelogram. 6  2 6 
Then, c = a + b and d = a − b Sol. (a) It is given that e1 and e2 are two unit vectors inclined at
⇒ c = 3 i$ and d = $i + 2 $j + 2 k$ an angle 2θ and | e1 − e2 | < 1.
Let θ be the angle between c and d. ∴ | e1 − e2 | < 1 ⇒ | e1 − e2 | 2 < 1
c ⋅d 3 $i ⋅ ( i$ + 2 $j + 2 k$ ) ⇒ 4 sin 2 θ < 1 [Q | e1 − e2 | 2 = 4 sin 2 θ]
Then, cosθ = =
| c||d| (3 ) 2 1 2 + 2 2 + 2 2 1
⇒ sin 2 θ <
3 1 4
= =
3 ×3 3  π
∴ θ ∈ 0, 
 6 
 1
∴ θ = cos− 1  
 3
l Ex. 6 If a = 3 $i − $j + 5k$ and b = $i + 2 $j − 3k$ are given
l Ex. 3 Let a, b, c, be vectors of length 3, 4, 5 respectively vector. A vector c which is perpendicular to Z-axis satisfying
and a be perpendicular to (b + c ), b to (c + a ) and c to c ⋅ a = 9 and c ⋅ b = − 4. If inclination of c with X-axis and
( a + b ), then the value of ( a + b + c ) is Y-axis is α and β respectively, then which of the following is
not true?
(a) 2 5 (b) 2 2
π π
(c) 10 5 (d) 5 2 (a) α > (b) β >
4 2
Sol. (d) We have, | a| = 3, | b | = 4 and | c | = 5. It is given that π π
(c) α > (d) β <
a ⊥ ( b + c), b ⊥ ( c + a ) and c ⊥ ( a + b ) 2 2
⇒ a ⋅ (b + c) = 0, b ⋅ ( c + a ) = 0 and c ⋅ ( a + b ) = 0 Sol. (c) c lies in XY -plane
⇒ a ⋅ b + a ⋅ c = b ⋅ c + b ⋅ a = c ⋅ a + c ⋅b = 0 ∴ c = x $i + y $j
or a ⋅ b + b ⋅ c + c ⋅ a = 0 (adding all the above equations) From the given conditions 3 x − y = 9
Now, | a + b + c |2 = | a |2 + |b |2 + | c |2 and x + 2y = − 4
+ 2( a ⋅ b + b ⋅ c + c ⋅ a ) Solving, we get c = 2 i − 3 j
$ $
= 3 + 4 + 5 = 50
2 2 2  2  −1  − 3 
∴ α = cos−1   , β = cos  
 13   13 
∴ | a+ b+ c | = 5 2
Chap 02 Product of Vectors 89

l Ex. 7 If A is 3 × 3 matrix and u is a vector. If Au and u are A


orthogonal for all real u, then matrix A is a
(a) singular (b) non-singular c b
(c) symmetric (d) skew-symmetric
Sol. (a) Au ⋅ u = 0 B
a c
⇒ | A | | u | = 0, Since | u | # 0 ⇒ | A | = 0
2
  3   3
∴ A is singular. = 5 ⋅ 3 − cos cos− 1   = 5 ⋅ 3 ⋅  −  = − 9
  5    5

l Ex. 8 Let the cosine of angle between the vectors p and q ∴ a ⋅ b + b ⋅ c + c ⋅ a = 0 − 16 − 9 = − 25


Trick Qa + b + c = 0
be λ such that 2p + q = $i + $j and p + 2q = $i − $j, then λ is
On squaring both the sides, we get | a + b + c | 2 = 0
equal to
5 4 ⇒ | a |2 + | b | 2
+ | c | 2 + 2(a ⋅ b + b ⋅ c + c ⋅ a ) = 0
(a) (b) −
9 5 ⇒ 2(a ⋅ b + b ⋅ c + c ⋅ a ) = − (9 + 16 + 25 )
(c)
3
(d)
7 ⇒ a ⋅ b + b ⋅ c + c ⋅ a = − 25
9 9
Sol. (b) It is given that 2p + q = $i + $j l Ex. 10 Let u, v and w be such that | u| =1, | v | = 2,
and p + 2q = $i − $j | w | = 3. If the projection v along u is equal to that of w
1 1 along u and v, w are perpendicular to each other, then
⇒ p = $i + $j and q = i$ − $j
3 3 | u − v + w | equals.
Let θ be the angle between p and q. Then (a) 14 (b) 7
 1 $ $  1 $ $ (c) 2 (d) 14
 i + j ⋅  i − j
p⋅q 3  3  Sol. (a) We have,
cosθ = =
| p|| q|  1
2
 1
2
Projection of v and u = Projection of w along u
  + (1 )   + ( − 1 )
2 2
 3  3 v⋅u w⋅u
⇒ = ⇒ v⋅u= w⋅u … (i)
1 8 |u| |u|
−1 −
9 8 4 Also, v and w are perpendicular to each other
= = 9 =− =−
1 1 10 10 5 ∴ v⋅w = 0 … (ii)
+1 +1
9 9 9
Now, | u − v + w | 2 = | u | 2 + | v | 2 + | w | 2 − 2( u ⋅ v )
4
∴ λ = cosθ = − − 2 ( v ⋅ w ) + 2( u ⋅ w )
5
⇒ | u− v+ w | =1 + 4 + 9
2

l Ex. 9 Let a, b and c be vectors with magnitudes 3, 4 and ⇒ | u − v + w | = 14


5, respectively and a + b + c = 0, then the values of
a ⋅ b + b ⋅ c + c ⋅ a is l Ex. 11 If a, b and c are unit vectors, then
(a) 47 (b) 25 | a − b | 2 + | b − c | 2 + | c − a | 2 does not exceed
(c) 50 (d) − 25 (a) 4 (b) 9
Sol. (d) We observe, | a | 2 + | b | 2 = 3 2 + 4 2 = 5 2 = | c | 2 (c) 8 (d) 6
a⋅b= 0 Sol. (b) We have, | a − b |2 + | b − c |2 + | c − a |2
 4 = | a | + | b |2 −2 (a ⋅ b) + | b |2 + | c | 2 − 2 ( b ⋅ c ) + | c | 2
b ⋅ c = | b | | c |⋅ cos π − cos− 1 
 5
+ | a | 2 − 2 (c ⋅ a )
  4 
= 4 × 5 − cos cos− 1   = 2[| a | 2 + | b | 2 + | c | 2 − (a ⋅ b + b ⋅ c + c ⋅ a )]
  5 
= 2[3 − (a ⋅ b + b ⋅ c + c ⋅ a )]
 4
= 4 × 5 ×  −  = − 16 = 6 − 2(a ⋅ b + b ⋅ c + c ⋅ a )
 5 …(i)
Now, | a + b + c | ≥ 0 2
 3
c ⋅ a = | c | | a |⋅ cos  π − cos− 1 
 5 ⇒ | a | 2 + | b | 2 + | c | 2 + 2(a ⋅ b + b ⋅ c + c ⋅ a ) ≥ 0
90 Textbook of Vector & 3D Geometry

⇒ 3 + 2(a ⋅ b + b ⋅ c + c ⋅ a ) ≥ 0 Sol. (b) Let P( r ) be a point on the locus.


3 ∴ AP = BP
⇒ a ⋅ b + b ⋅ c + c ⋅a ≥ −
2 ⇒ | r − a | = | r − b | ⇒| r − a | 2 = | r − b | 2
⇒ − 2(a ⋅ b + b ⋅ c + c ⋅ a ) ≤ 3 …(ii)
⇒ (r − a ) ⋅ (r − a ) = (r − b) ⋅ (r − b)
From Eqs. (i) and (ii), we obtain
P(r)
| a − b |2 + | b − c |2 + | c − a |2 ≤ 6 + 3
⇒ | a − b |2 + | b − c |2 + | c − a |2 ≤ 9

l Ex. 12 The vectors a = 2 λ 2 $i + 4 λ$j + k$ and


b = 7 $i − 2 $j + λk$ make an obtuse angle whereas the angle A(a) B(b)

between b and k$ is acute and less than π / 6, ⇒ 2r ⋅ (a − b ) = a ⋅ a − b ⋅ b


1 1
(a) 0 < λ < (b) λ > 159 ⇒ r ⋅ (a − b) = (a + b ) ⋅ (a − b )
2 2
1  1 
(c) − < λ < 0 (d) null set ∴ r − (a + b ) ⋅ (a − b ) = 0
2  2 
Sol. (d) As angle between a and b is obtuse, a ⋅ b < 0 This is the locus of P.
⇒ (2 λ i + 4 λ$j + k$ ) ⋅ (7 $i − 2 $j + λk$ ) < 0
2$
l Ex. 14 In cartesian coordinates the point A is ( x 1 , y 1 ),
⇒ 14 λ2 − 8 λ + λ < 0
where x 1 = 1 on the curve y = x 2 + x + 10. Then tangent at A
⇒ λ (2 λ − 1 ) < 0
1 cuts the X-axis at B. The value of the dot product OA ⋅ AB is
⇒ 0<λ< …(i) 520
2 (a) − (b) − 148
π 3
Angle between b and k is acute and less than . (c) 140 (d) 12
6
b ⋅ k = | b |⋅ | k | cosθ Sol. (b) Given curve is y = x 2 + x + 10 …(i)

⇒ λ = 53 + λ2 1 ⋅ cosθ When x = 1, y = 1 + 1 + 10 = 12
2

λ ∴ A = (1, 12 )
⇒ cosθ =
53 + λ2 ∴ OA = $i + 12 $j
dy
π π From Eq. (i), = 2x + 1
θ< ⇒ cosθ > cos dx
6 6
 dy 
3 λ 3 Equation of tangent at A is y − 12 =   (x − 1)
⇒ cosθ > ⇒ >  dx  (1, 12)
2 53 + λ2 2
⇒ y − 12 = (2 × 1 + 1 ) ( x − 1 )
⇒ 4 λ2 − 3(53 + λ2 ) > 0
⇒ y − 12 = 3 x − 3
⇒ λ2 > 159 ∴ y = 3( x + 3 )
⇒ λ < − 159 …(ii) This tangent cuts X-axis (i.e. y = 0) at ( − 3, 0 )
From Eqs. (i) and (ii), λ = φ ∴ B = ( − 3, 0 )
∴ Domain of λ is null set. OB = − 3 $i + 0 ⋅ $j = − 3 $i, OA ⋅ AB = OA ⋅ ( OB − OA )
( i$ + 12 $j) ⋅ ( − 3 $i − $i − 12 $j) = ( i$ + 12 $j) ⋅ ( − 4 $i − 12 $j)
l Ex. 13 The locus of a point equidistant from two given = − 4 − 144 = − 148
points whose position vectors are a and b is equal to
 1  l Ex. 15 In a tetrahedron OABC, the edges are of lengths,
(a) r − ( a + b ) ⋅ ( a + b ) = 0
 2  | OA | = | BC | = a, | OB | = | AC | = b, | OC | = | AB | = c . Let G 1
and G 2 be the centroids of the triangle ABC and AOC such
 1 
(b) r − ( a + b ) ⋅ ( a − b ) = 0 a2 +c2
 2  that OG 1 ⊥ BG 2 , then the value of is
 1  b2
(c) r − ( a + b ) ⋅ a = 0 (a) 2 (b) 3
 2 
(c) 6 (d) 9
(d) [r − ( a + b )] ⋅ b = 0
Chap 02 Product of Vectors 91

Sol. (b) OG1 ⋅ BG2 = 0. Sol. (b) Let the required vector be r = x$i + zk$ , since r is a unit
a + b + c a + c − 3b vector.
⇒ ⋅ =0
3 3 ∴ x2 + y 2 = 1
⇒ a 2 + c 2 − 3b 2 + 2a ⋅ c − 2b ⋅ c − 2a ⋅ b = 0 It is given that r makes 45° and 60° angles with a and b
respectively.
Now, | c − a| = b , | c − b | = a and | a − b | = c
2 2 2 2 2 2
r ⋅a r⋅b
∴ 2a ⋅ c = a 2 + c 2 − b 2, 2b ⋅ c = b 2 + c 2 − a 2, ∴ cos 45° = and cos60° =
| a || r | | r|| b|
2a ⋅ b = a 2 + b 2 − c 2 1 2x − y 1 −y
⇒ = and = −
Putting in the above result, we get 2a 2 + 2c 2 − 6b 2 = 0 2 3 2 2
3 1
a2 + c2 ⇒ 2x − y = and y =
⇒ =3 2 2
b2
1
⇒ x=
2
l Ex. 16 If OABC is a tetrahedron such that OA 2 + BC 2
1 $ $
= OB 2 + CA 2 = OC 2 + AB 2 , then which of the following is ∴ r= ( i − k)
2
not true?
(a) OA ⊥ BC (b) OB ⊥ AC l Ex. 19 A unit vector perpendicular to the vector
(c) OC ⊥ AB (d) AB ⊥ AC − $i + 2 $j + 2k$ and making equal angles with X and Y -axes
Sol. (d) Let OA = a, OB = b, OC = c can be
Then from the given conditions 1 $ $ $ 1 $ $
(a) ( 2i + 2j − k ) (b) ( 2i − 2j − k$ )
a ⋅ a + (b − c) ⋅ (b − c) = b ⋅ b + (c − a ) ⋅ (c − a ) 3 3
⇒ − 2b ⋅ c = − 2c ⋅ a 1 1
(c) ( 2$i + 2$j + k$ ) (d) ( 2$i − 2$j + k$ )
⇒ c ⋅ ( b − a ) = 0 ⇒ BA ⋅ OC = 0 3 3
Hence AB ⊥ OC . Similarly, Sol. (a) Let the required vector be r = li$ + m$j + nk$ , where l, m, n
BC ⊥ OA and CA ⊥ OB are the direction cosines of r such that l = m.
It is given that r is perpendicular to − i$ + 2 $j + 2 k$ . Therefore,
l Ex. 17 If a , b, c and A, B, C ∈R − {0 } such that r ⋅ ( − $i + 2 $j + 2 k)
$ =0
aA + bB + cC + (a + b + c )( A + B + C ) = 0, then
2 2 2 2 2 2
⇒ − l + 2m + 2n = 0
aB bC cA ⇒ l + 2n = 0 [Q l = m ]
value of = + is
bA cB aC ⇒ l = − 2n
(a) 3 (b) 4 Now, l 2 + m2 + n2 = 1
(c) 5 (d) 6
⇒ 4n 2 + 4n 2 + n 2 = 1
Sol. (a) Let r1 = a$i + b$j + ck$ and r2 = Ai$ + B$j + Ck$
1
⇒ r1 ⋅ r2 = aA + bB + cC ⇒ n=±
3
⇒ | r1 | | r2 | = (a 2 + b 2 + c 2 )( A 2 + B 2 + C 2 ) 2 2 1
∴ l = m ,m = m ,n = m
∴ r1r2 = − | r1 | | r2 | 3 3 3
1 $ $ $
⇒ r1 and r2 are anti-parallel Hence, r = m (2 i + 2 j − k )
a b c 3
⇒ = = = k, where k is any constant
A B C
aB bC cA
l Ex. 20 If ( a + 3b ) ⋅ (7 a − 5b ) = 0 and
⇒ + + =3 ( a − 4b ) ⋅ (7 a − 2b ) = 0. Then, the angle between a and b is
bA cB aC
(a) 60° (b) 30°
l Ex. 18 The unit vector in ZOX plane making angles 45° (c) 90° (d) None of these
and 60° respectively, with a = 2 $i + 2 $j − k$ and b = $j − k$ , is Sol. (a) We have, (a + 3 b ) ⋅ (7a − 5 b ) = 0
1 1 $ $ ⇒ 7 | a | 2 + 16a ⋅ b − 15 | b | 2 = 0 …(i)
(a) ( − i$ + k$ ) (b) (i − k)
2 2 and (a − 4 b ) ⋅ (7a − 2 b ) = 0
(c)
3 $ $
(i + k) (d) None of these ⇒ 7 | a | 2 − 30a ⋅ b + 8 | b | 2 = 0 …(ii)
2
92 Textbook of Vector & 3D Geometry

From Eqs. (i) and (ii), we get l Ex. 22 If a, b, c be non-zero vectors such that a is
15 | b | 2 − 7 | a | 2 perpendicular to b and c and | a | =1, | b | = 2, | c | =1, b ⋅ c =1
a⋅b =
16 and there is a non-zero vector d coplanar with a + b and
1
a ⋅ b = (7| a |2 + 8 | b |2 ) 2b − c and d ⋅ a =1, then minimum value of | d | is
30 2 3
(a) (b)
15 | b | 2 − 7 | a | 2 1 13 13
⇒ = (7 | a | 2 + 8 | b | 2 )
8 15 4 4
(c) (d)
⇒ 15(15 | b | 2 − 7 | a | 2 ) = 8(7 | a | 2 + 8 | b | 2 ) 5 13
⇒ 225 | b | 2 − 105 | a | 2 = 56 | a | 2 + 64 | b | 2 Sol. (d) a ⋅ b = a ⋅ c = 0, | a | = | c | = 1, | b | = 2 and b ⋅ c = 1

⇒ 161 | b | = 161 | a |
2 2 let d = x(a + b ) + y (2 b − c )
But d ⋅a = 1
⇒ | b |2 = | a |2
⇒ x(1 + 0 ) + 0 = 1
From Eq. (i), we get
⇒ x =1
16a ⋅ b = 15 | b | 2 − 7 | a | 2 = 15 | b | 2 − 7 | b | 2
⇒ d = a + b + y(2 b − c )
⇒ 16a ⋅ b = 8 | b | 2 ⇒ | d | 2 = | a | 2 + | b | + 2a ⋅ b + y 2
1 (2 b − c ) 2 + 2y (a + b ) ⋅ (2 b − c )
⇒ a⋅b =| b |2
2
1 ⇒ | d | 2 = 1 + 4 + y 2(16 + 1 − 4 ) + 2y (8 − 1 )
⇒ | a | | b | cosθ = | b | 2
2 = 13y 2 − 14y + 5
1 4 × 13 × 5 − 14 × 14 4
⇒ cosθ = ∴ | d | min = =
2 4 × 13 13
∴ θ = 60 °
l Ex. 23 A groove is in the form of a broken line ABC and
l Ex. 21 Let two non-collinear vectors a and b inclined at the position vectors of the three points are respectively

an angle be such that | a | = 3 and | b | = 2. If a point P 2 $i − 3 $j + 2k$ , 3 $i + 2 $j − k$ and $i + $j + k$ . A force of magnitude
3
moves so that at any time t its position vector OP (where O is 24 3 acts on a particle of unit mass kept at the point A and
 1  1 moves it along the groove to the point C. If the line of action
the origin) is given as OP =  t +  a +  t −  b, then least of the force is parallel to the vector $i + 2 $j + k$ all along, the
 t  t
distance of P from the origin is number of units of work done by the force is
(a) 144 2 (b) 144 3
(a) 2 133 − 10 (b) 2 133 + 10
(c) 72 2 (d) 72 3
(c) 5 + 133 (d) None of these $i + 2 $j + k$ 24 3 $
2 2 Sol. (c) F = (24 3 ) = ( i + 2 $j + k$ )
 1  1 | i + 2j + k |
$ $ $ 6
Sol. (b) We have, | OP | 2 = t +  | a | 2 + t −  | b | 2
 t  t = 12 2 ( $i + 2 $j + k$ )
 1  2π  Displacement, r = Position Vector of C − Position Vector of A
+ 2 t 2 − 2  | a | | b | cos 
 t   3
= ( i$ + $j + k$ ) − (2 $i − 3 $j + 2 k$ )
2 2
 1  1  1  − 1 = ( − i$ + 4 $j − k$ )
∴ | OP | 2 = 9 t +  + 4 t −  + 2 t 2 − 2  3 ⋅ 2 ⋅  
 t  t  t   2 
Work done by the force
 1   1   1
= 9 t 2 + 2 + 2 + 4 t 2 + 2 − 2 − 6 t 2 − 2  W = r ⋅ F = ( − $i + 4 $j − k$ ) ⋅ 12 2 ( $i + 2 $j + k$ )
 t   t   t 
19 = 12 2 ( − 1 + 8 − 1 ) = 72 2
= 7t 2 + 2 + 10
t
19
l Ex. 24 For any vectors a, b; | a × b | 2 + ( a ⋅ b ) 2 is equal
⇒ | OP | ≥ 2 ⋅ 7t ⋅ 2 + 10
2 2
(Q AM ≥ GM )
t to
∴ Minimum value of | OP | = 10 + 2 133 (a) | a |2 | b |2 (b) | a + b |
(c) | a | − | b |
2 2
(d) 0
Chap 02 Product of Vectors 93

Sol. (a) We have, | a × b | = | a | | b |sin θ l Ex. 28 If a = 2 $i − 3 $j + k$ , b = − $i + k$ , c = 2 $j − k$ , then the


⇒ | a × b | 2 = | a | 2 | b | 2 sin 2 θ area (in sq units) of parallelogram with diagonals a + b and
b + c will be
= | a | 2 | b | 2 (1 − cos2 θ )
(a) 21 (b) 2 21
= | a | 2 | b | 2 − | a | 2 | b | 2 cos2 θ 1
(c) 21 (d) None of these
= | a | 2 | b | 2 − (| a || b | cosθ ) 2 2
= | a | 2 | b | 2 − (a ⋅ b ) 2 Sol. (c) We have, a = 2 $i − 3 $j + k$

∴ | a × b | 2 + (a ⋅ b ) 2 = | a | 2 | b | 2 b = − $i + k$ and c = 2 $j − k$
Since, (a + b ) and ( b + c ) are the diagonals of the parallelogram
l Ex. 25 If a = $i + $j + k$ , b = $i + $j − k$ , then vectors perpen- Now, a + b = $i − 3 $j + 2 k$ and b + c = − $i + 2 $j
dicular to a and b is/are 1
∴ Area of parallelogram = | (a + b ) × ( b + c )|
(a) λ( $i + j$ ) (b) λ( $i + j$ + k$ ) 2
1 $ $
(c) λ( $i − $j ) (d) None of these = | ( i − 3 j + 2 k$ ) × ( − $i + 2 $j) |
2
Sol. (c) Any vector perpendicular to both a and b = λ(a × b ) $j k$
i$
 $i $j k$  1 1
= 1 − 3 2 = ( − 4 i$ − 2 $j − k$ )
Now, a × b =  1 1 1  2
−1 2 0
2
 
1 1 − 1  1 21
= (− 4)2 (− 2)2 + (− 1)2 = sq units.
= − 2 $i + 2 $j = − 2( $i − $j) 2 2
= − 2( i$ − $j)
∴Required vector = λ( $i − $j)
l Ex. 29 The coordinates of the mid-points of the sides of
∆PQR are (3a , 0, 0 ), (0, 3b, 0 ) and (0, 0, 3c ) respectively, then
l Ex. 26 If a × b = b × c ≠ 0, then the correct statement is the area of ∆PQR is
(a) b | | c (b) a | | b (a) 18 b 2c 2 + c 2a 2 + a 2b 2 (b) 9 b 2c 2 + c 2a 2 + a 2b 2
(c) ( a + c ) | | b (d) None of these 9 2 2
(c) b c + c 2a 2 + a 2b 2 (d) 18 ab + bc + ca
Sol. (c) We have, 2
a× b= b× c Sol. (a) Let L, M, N be the mid-points of the sides of ∆PQR.
⇒ a× b− b× c = 0 P
⇒ a× b+ c× b = 0
⇒ (a + c ) × b = 0
∴ (a + c ) | | b (3a, 0, 0) L N (0, 0, 3c)
[Q if vector product of two vectors is zero, then
both vectors are parallel to each other]

l Ex. 27 If a = $i + 2 $j + 3k$ , b = − $i + 2 $j + k$ and c = 3 $i + $j. If Q M R


( a + b ) ⊥ c, then t is equal to (0, 3b, 0)
1
(a) 5 (b) 4 Area of ∆LMN = | MN × ML |
(c) 3 (d) 2 2
1
Sol. (a) We have, a = $i + 2 $j + 3 k$ = | ( − 3b$j + 3ck$ ) × (3a$i − 3b$j) |
2
b = − $i + 2 $j + k$ 1
= | 9(bc$i + ca$j + abk$ ) |
and c = 3 $i + $j 2
Since, (a + tb ) is perpendicular to c 9
= (bc ) 2 + (ca ) 2 + (ab ) 2
∴ (a + tb ) ⋅ c = 0 2
[(1 − t ) i$ + (2 + 2t ) $j + (3 + t ) k$ ] ⋅ (3 i$ + $j] = 0 Now, area of ∆PQR = 4 × (Area of ∆LMN)
9
⇒ 3 (1 − t ) + (2 + 2t ) = 0 =4× b 2c 2 + c 2a 2 + a 2b 2
2
⇒ t =5
= 18 b 2c 2 + c 2a 2 + a 2b 2
94 Textbook of Vector & 3D Geometry

l Ex. 30 In a parallelogram ABCD, AB = $i + $j + k$ and lEx. 32 If a 2 + b 2 + c 2 = 1 where a , b, c ∈R, then the


diagonal AC = $i − $j + k$ and area of parallelogram is 8 sq maximum value of ( 4a − 3b ) 2 + (5b − 4c ) 2 + (3c − 5a ) 2 is
units, then ∠BAC is equal to (a) 25 (b) 50
π π (c) 144 (d) None of these
(a) (b)
6 3 Sol. (b) Let r1 = ai$ + b$j + ck$ , r2 = 3 $i + 4 $j + 5 k$ …(i)
 8  8 | r1 × r2 | ≤ | r1 | | r2 |
2 2
(c) sin− 1   (d) cos − 1  
 3   3  $i $j k$
Sol. (c) We have, AB = i$ + $j + k$ and AC = $i − $j + k$ Now, r1 × r2 = a b c
D C 3 4 5

= $i(5b − 4c ) + $j (3c − 5a ) + k$ ( 4a − 3b )
So, from Eq. (i), we get

q
(5b − 4c ) 2 + (3c − 5a ) 2 + ( 4a − 3b ) 2 ≤ 50
A B

Let θ be the ∠BAC. Then,


l Ex. 33 If a, b and c are pth, qth, rth terms of HP and
| AB × AC | $i $j k$
sin θ = u = (q − r ) $i + (r − p ) $j + ( p − q )k$ , v = + + , then
| AB | | AC | a b c
$i $j k$ (a) u and v are parallel vectors
Now, AB × AC = 1 1 1 = 2 $i − 2 k$ (b) u and v are orthogonal vectors
1 −1 1 (c) u ⋅ v = 1
(d) u × v = $i + $j + k$
∴ | AB × AC | = 8
Sol. (b) Let A be the first term and D be the common difference of
8
Hence, sinθ = the corresponding AP. Then,
3× 3 1 1 1
= A + ( p − 1 ) D, = A + (q − 1 ) D, = A + (r − 1 ) D
 8 a b c
⇒ θ = sin − 1  
 3  ⇒ a − 1(q − r ) + b − 1(r − p ) + c − 1( p − q ) = 0
⇒ v⋅u = 0
l Ex. 31 Let ∆ABC be a given triangle. If ⇒ u⊥ v
| BA − tBC | ≥ | AC | for any t ∈R, then ∆ABC is Hence, u and v are orthogonal vectors.
(a) Equilateral (b) Right angled
(c) Isosceles (d) None of these l Ex. 34 If the vector product of a constant vector OA with
Sol. (b) | BA | 2 + t 2 | BC | 2 − 2 BA ⋅ BC t − | AC | 2 ≥ 0, ∀ t ∈ R …(i) variable vector OB in a fixed plane OAB be a constant vector,
Discriminante of the quadratic equation ≤ 0 then the locus of B is
⇒ 4( BA ⋅ BC ) 2 − | BC | 2 | BA | 2 + 4 BC AC ≤ 0
2 2
…(ii) (a) a straight line perpendicular to OA
2 2
(b) a circle with centre O radius equal to | OA |
Using ( BA ⋅ BC ) 2 − BC BA (c) a straight line parallel to OA
= − | BA × BC | 2 (d) None of the above
= − | ( BC + CA ) × BC | 2 Sol. (c) Let A(α , β ) point be given and O be taken as the origin

= − | CA × BC | 2 B (x, y)
Using Eq. (ii) in Eq. (i). O
| BC | 2 | AC | 2 ≤ | AC × BC | 2 (0, 0)
A (a, b)
But | AC × BC | = | AC | | BC |sin C
⇒ sin 2 C ≥ 1 We have, OA = α$i + β$j
and OB = x$i + y$j
⇒ sinC = ± 1
π Now, | OA × OB | = | (αy − βx ) k$ | = constant
⇒ ∠C =
2
Chap 02 Product of Vectors 95

⇒ αy − βx = constant Since, r makes an obtuse angle with Y -axis. Therefore,


∴Locus of B( x, y ) is a line parallel to OA because slope of λ=−
2
β 17
OA =
α Hence, r = − 2 (2 $i + 3 $j − 6 k$ )
or r = 2 ( − 2 $i − 3 $j + 6 k$ )
l Ex. 35 Unit vector perpendicular to the plane of ∆ABC
with position vectors a, b, c of the vertices A, B, C is l Ex. 37 Let a , b, c denote the lengths of the sides of a
a× b+ b× c+ c× a
(a) triangle such that

a× b+ b× c+ c× a (a − b) u + (b − c) v + (c − a) (u × v ) = 0
(b)
2∆ for any two non-collinear vectors u and v, then the
a× b+ b× c+ c× a triangle is
(c)
4∆ (a) right angled (b) equilateral
(d) None of the above (c) isosceles (d) scalene
Sol. (b) The required vector is given by Sol. (b) Since, u, v and u × b are non-coplanar vectors.
AB × AC ∴ (a − b ) u + (b − c ) v + (c − a ) ( u × v ) = 0
n$ =
| AB × AC | ⇒ a −b = 0=b −c =c −a
AB × AC = ( b × a ) × ( c − a ) ⇒ a =b =c
= b× c− b× a− a× c+ a× a So, the triangle is equilateral.
= b× c+ a× b+ c× a [Q a × a = 0 ]
We also know that, l Ex. 38 The value of $i ⋅ ( $j × k$ ) + $j ⋅ (k$ × $i ) + k$ ⋅ ( $i × $j ) is
1 (a) 3 (b) 2
Area of ∆ABC = | a × b + b × c + c × a |
2 (c) 1 (d) 0
a× b+ b× c+ c× a Sol. (a) We have, $i ⋅ ( $j × k$ ) + $j ⋅ ( k$ × $i ) + k$ ⋅ ( $i × $j)
∴ n$ =
| a× b+ b× c+ c× a |
= $i ⋅ $i + $j ⋅ $j + k$ ⋅ k$ [Q $i × $j = k$ , $j × k$ = $i, k$ × i$ = $j]
a× b+ b× c+ c× a
= = | $i | 2 + | $j | 2 + | k$ | 2 = 1 + 1 + 1 = 3 [Q $i, $j, k$ are unit vectors]
2∆
 1 
Q ∆ = | a× b+ b× c+ c× a |
 2  l Ex. 39 For non-zero vectors a, b, c;
| ( a × b ) ⋅ c | = | a | | b | | c | holds if and only if
l Ex. 36 The vector r satisfying the conditions that (a) a ⋅ b = 0, b ⋅ c = 0 (b) b ⋅ c = 0, c ⋅ a = 0
(c) c ⋅ a = 0, a ⋅ b = 0 (d) a ⋅ b = b ⋅ c = c ⋅ a = 0
I. it is perpendicular to 3 $i + 2 $j + 2k$ and 18$i − 22 $j − 5k$
Sol. (d) We have, | (a × b ) ⋅ c | = | a || b || c |
II. it makes an obtuse angle with Y-axis. ⇒ || a || b || c |sin θ cosα | = | a || b || c |
III. | r | = 14 ⇒ | sin θ | | cosα | = 1
(a) 2( − 2$i − 3 $j + 6k$ ) π
⇒ θ = and α = 0
2
(b) 2( 2i$ − 3 j$ + 6k$ )
⇒ a ⊥ b and c | | n$
(c) 4 $i + 6 $j − 12k$ ⇒ a ⊥ c and c ⊥ both a and b
(d) None of the above ⇒ a, b, c are mutually perpendicular.
Sol. (a) Let a = 3 $i + 2 $j + 2 k$ and b = 18 $i − 22 $j − 5 k$ ⇒ a ⋅ c = b ⋅ c = c ⋅a = 0
Then, the required vector r is given by
r = λ(a × b )
l Ex. 40 The position vectors of three vertices A, B, C of a
⇒ r = λ(34 $i + 51 $j − 102 k$ ) $
tetrahedron OABC with respect to its vertex O are $i, 6 $j, k,
= 17 λ(2 i$ + 3 $j − 6k$ ) then its volume (in cu units) is
1
Now, | r | = 14 ⇒ 119 | λ | = 14 (a) 3 (b)
2 3
⇒ |λ|= 1
17 (c) (d) 6
6
96 Textbook of Vector & 3D Geometry

Sol. (d) We have, A(6 $i + 0 $j + 0 k$ ), B( 0 $i + 6 $j + 0 k$ ), C( 0 $i + 0 $j + k$ ) l Ex. 44. Let r = ( a × b ) sin x + (b × c ) cos y + (c × a ),


and O( 0 $i + 0 $j + 0 k$ ) where a , b and c are non-zero non-coplanar vectors. If r is
∴ OA = 6$i + 0 $j + 0k$ orthogonal to 3 a + 5b + 2c,then the value of
sec 2 y + cosec 2 x + sec y cosec x is
OB = 0 $i + 6 $j + 0k$
(a) 3 (b) 4
and OC = 0 $i + 0 $j + k$
(c) 5 (d) 6
∴Volume of tetrahedron Sol. (a) r ⋅ (3 a + 5b + 2 c) = 0
1 1
= [ OA OB OC] = [6 $i 6 $j k$ ] ⇒ a ⋅ (b × c) [2 sin x + 3 cosy + 5 ] = 0
6 6
⇒ 2 sin x + 3 cosy + 5 = 0
6 0 0
1 1 ⇒ 2 sin x + 3 cosy = − 5 [Q a ⋅ (b × c) ≠ 0 ]
= 0 6 0 = [6(6 − 0 )] = 6 cu units
6 6 ⇒ sin x = − 1, cosy = 1
0 0 1
⇒ cosec x = − 1, secy = − 1
l Ex. 41 A parallelopiped is formed by planes drawn
l Ex. 45 Let a, b, c be distinct non-negative numbers. If the
parallel to coordinate axes through the points A = (1, 2, 3 )
vectors a $i + a$j + ck$ , $i + k$ and c $i + c$j + bk$ lie in a plane, then
and B = ( 9, 8, 5 ). The volume of that parallelopiped is equal
to (in cubic units) c is
(a) 192 (b) 48 (a) HM of a and b (b) 0
(c) 32 (d) 96 (c) AM of a and b (d) GM of a and b
Sol. (d) Translating the axes through A(1, 2, 3 ). Sol. (d) Since, the given points lie in a plane
a a c
A changes to ( 0, 0, 0 ), B changes to (8, 6, 2).
∴ 1 0 1 =0
∴Coterminous edges are of lengths 8, 6, 2.
c c b
Volume of parallelopiped = 8 ⋅ 6 ⋅ 2 = 96 cu units.
On applying C1 → C1 − C2
l Ex. 42 If | a | =1, | b | = 3 and | c | = 5, then the value of 0 a c
[ a − b b − c c − a ] is ⇒ 1 0 1 =0
(a) 0 (b) 1 0 c b
(c) − 1 (d) None of these ⇒ − 1(ab − c 2 ) = 0
 1 −1 0  ⇒ c 2 = ab
Sol. (a) [a − b b − c c − a ] +  0 1 − 1  [abc ]
  Hence, c is GM of a and b.
− 1 0 1 
= [1(1 − 0 ) + 1( 0 − 1 ) + 0( 0 + 1 )] [abc ] l Ex. 46 If a, b and c are non-coplanar vectors and λ is a
= 0 × [abc ] = 0 real number, then [ λ( a + b ) | λ 2 b| λc ] = [a a + c b ] for
(a) exactly two values of λ
l Ex. 43 If a, b, c are three non-coplanar vectors, then
(b) exactly one value of λ
3 a − 7b − 4c, 3 a − 2b + c and a + b + λc will be coplanar, if (c) no value of λ
λ is (d) exactly three values of λ
(a) − 1 (b) 1
Sol. (c) Given, [ λ (a + b ) λ2 bλc] = [a b + c b ]
(c) 3 (d) 2
λ (a1 + b1 ) λ (a 2 + b2 ) λ (a 3 + b3 )
Sol. (d) Let α = 3a − 7b − 4c, β = 3 a − 2b + c and γ = a + b + λc
⇒ λ2b1 λ2b2 λ2b3
For α, β and γ to be coplanar [α β γ ] = 0
λc1 λc 2 λc 3
3 −7 − 4
a1 a2 a3
⇒ 3 − 2 1 [a b c ] = 0
= b1 + c1 b2 + c 2 b3 + c 3
1 1 λ
b1 b2 b3
[3( −2 λ − 1 ) + 7(3 λ − 1 ) − 4(3 + 2 ) (abc )] = 0
a1 + b1 a 2 + b2 a 3 + b3
⇒ (15 λ − 30 ) [a b c ] = 0
⇒ λ4 b1 b2 b3
Since, a, b, c are non-coplanar
c1 c2 c3
∴ 15 λ − 30 = 0 ⇒ λ = 2
Chap 02 Product of Vectors 97

a1 a2 a3 Minimum value occurs when AB = BC = AD = 6


= b1 + c1 b2 + c 2 b3 + c 3 Hence, AC = AB 2 + BC 2 − 2 AB ⋅ BC ⋅ cos30 °
b1 b2 b3
=6 2 − 3
[applying R1 → R1 − R2 in LHS and R2 → R2 − R3 in RHS]
a1 a 2 a3 a1 a 2 a 3 l Ex. 49 If a = $i + $j + k$ , b = $i − $j + k$ , c = $i + 2j$ − k$ , then
⇒ λ4 b1 b2 b3 = − b1 b2 b3
a ⋅ a a ⋅b a ⋅c
c1 c 2 c3 c1 c 2 c 3
the value of b ⋅ a b ⋅ b b ⋅ c is
⇒ λ4 = − 1
c ⋅ a c ⋅b c ⋅c
Hence, no real value of λ exists.
(a) 2 (b) 4
l Ex. 47 In a regular tetrahedron, let θ be angle between (c) 16 (d) 64
any edge and a face not containing the edge. The value of Sol. (c) We have, a = i$ + $j + k$
cos 2 θ is b = $i − $j + k$ and c = $i + 2j$ − k$
(a) 1/6 (b) 1/9 a ⋅a a ⋅ b a ⋅ c
(c) 1/3 (d) None of these We know that, b ⋅ a b ⋅ b b ⋅ c = [a b c ]2
Sol. (c) Let OABC be the tetrahedron. Let G be the centroid of the c ⋅a c ⋅ b c ⋅ c
1
face OAB, then GA = AC . 2
3 1 1 1
C = 1 −1 1
1 2 −1
= [1(1 − 2 ) − 1( − 1 − 1 ) + 1(2 + 1 )]2
q A
= [ − 1 + 2 + 3 ]2 = [ 4 ]2 = 16
G B
O
l Ex. 50 The value of a so that the volume of
GA 1
Then, cosθ = = parallelopiped formed by $i + a$j + k$ , $j + ak$ and a $i + k$
CA 3
1 becomes minimum is
∴ cos2 θ = (a) − 3 (b) 3
3
(c) 1 / 3 (d) 3
l Ex. 48 DABC be a tetrahedron such that AD is perpen- Sol. (c) Volume of the parallelopiped
dicular to the base ABC and ∠ABC = 30 °. The volume of V = [ $i + a$j + kj
$ $ + a k$ a $i + k$ ]
tetrahedron is 18. If value of AB + BC + AD is minimum, = ( i$ + a$j + k$ ) ⋅ {( $j + a k$ ) × (a $i + k$ )}
then the length of AC is
= ( i$ + a$j + k$ ) ⋅ ( $i + a 2$j − a k$ ) = 1 + a 3 − a
(a) 6 2 − 3 (b) 3( 6 − 2 )
dV d 2V dV
(c) 6 2 + 3 (d) 3( 6 + 2 ) = 3a 2 − 1, 2 = 6a, =0
da da da
1 1  1
Sol. (a) Volume = AD  AB ⋅ BC sin 30 ° ⇒ 3a 2 − 1 = 0 ⇒ a = ±
3 2  3
1 1 d 2V 6
⇒ 18 = ( AD ⋅ AB ⋅ BC ) At a=, 2 = >0
12 3 da 3
⇒ AD ⋅ AB ⋅ BC = 216 1
∴V is minimum at a =
Now, AB + BC + AD ≥ 3( AD ⋅ AB ⋅ BC )1/3 3
B
D
l Ex. 51 If a, b and c be any three non-zero and
30°
non-coplanar vectors, then any vector r is equal to
(a) za + xb + yc (b) xa + yb + zc
A C (c) ya + zb + xc (d) None of these
[r b c] [r c a ] [r a b ]
⇒ AB + BC + AD ≥ 18 where, x = ,y= ,z=
[ a b c] [ a b c] [ a b c]
98 Textbook of Vector & 3D Geometry

Sol. (b) Since, a, b and c are three non-coplanar vectors, we may The greatest value of [a b c ] = 3, which is obtained when θ = 0.
assume r = αa + βb + γc b×c 2 2 1
So, a= = i$ + $j − k$
[ r b c ] = (αa + βb + γc) ⋅ (b × c) = α { a ⋅ (b × c)} |b × c| 3 3 3
= α[a b c ]
[ r b c] l Ex. 55 The vectors
⇒ α=
[a b c ] u = (al + a 1l1 ) $i + (am + a 1m1 ) $j + (an + a 1n1 )k$
[ r b c]
But x= v = (bl + b1l1 ) $i + (bm + b1m1 ) $j + (bn + b1n1 )k$ and
[a b c ]
w = (cl + c 1l1 ) $i + (cm + c 1m1 ) $j + (cn + c 1n1 )k$
∴ α=x
Similarly, β = y, γ = z (a) form an equilateral triangle
(b) are coplanar
∴ r = xa + yb + zc
(c) are collinear
l Ex. 52 The position vectors of vertices of ∆ABC are a, b, (d) are mutually perpendicular
c and a ⋅ a = b ⋅ b = c ⋅ c = 3. If [ a b c ] = 0, then the position Sol. (b) We have,
vectors of the orthocentre of ∆ABC is al + a1l1 am + a1m1 an + a1n1
1 [ u v w ] = bl + b1l1 bm + b1m1 bn + b1n1
(a) a + b + c (b) ( a + b + c )
3 cl + c1l1 cm + c1m1 cn + c1n1
(c) 0 (d) None of these a a1 0 l l1 0
Sol. (a) Hence, [a b c ] = 0 ⇒ [ u v w ] = b b1 0 m m1 0 =0
So, the points O, A, B and C are coplanar. Also, c c1 0 n n1 0
OA = OB = OC = 3, hence origin O is the circumcentre.
Hence, the given vectors are coplanar.
a+ b+ c
Position vector of the centroid G is .
3 l Ex. 56 Let a, b, c be three vectors such that [ a b c ] = 2.
Now, orthocentre divides OG in the ratio of 3 : 2 externally.
So, position vectors of orthocentre is a + b + c. If r = l (b × c ) + m(c × a ) + n( a × b ) is perpendicular to
a + b + c, then the value of (l + m + n ) is
l Ex. 53 If α and β are two mutually perpendicular unit (a) 2 (b) 1
vectors {rα + rβ + s (α × β}, [α + (α × β)] and (c) 0 (d) None of these
{ s α + s β + t (α × β)} are coplanar, then s is equal to Sol. (c) It is given that r perpendicular (a + b + c )
(a) AM of r and t (b) GM of r and t ∴ r ⋅ (a + b + c ) = 0
(c) HM of r and t (d) None of these ⇒ l [a b c ] + m[ c a b ] + n[a b c ] = 0
Sol. Since, α and β are two mutually perpendicular vectors and ⇒ 2(l + m + n ) = 0 [Q [a b c ] = 2 ]
(rα + r β + s (α × β ), [α + (α × β )], {sα + sβ + t(α × β )} are ⇒ l +m+n=0
coplanar
r r s l Ex. 57 If a, b and c are three mutually perpendicular
⇒ 1 0 1 =0 vectors, then the projection of the vectors
s s t a b (a × b)
l +m +n along the angle bisector of the
⇒ s 2 = rt |a| |b | | a× b|
vectors a and b is
l Ex. 54 Let b = − $i + 4 $j + 6k$ and c = 2 $i − 7 $j − 10 k.
$ If a be l +m
(a) (b) l 2 + m 2 + n 2
a unit vector and the scalar triple product [ a b c ] has the 2
greatest value, then a is equal to l2 + m2
(c) (d) None of these
1 $ $ $ 1 l2 + m2 + b2
(a) (i + j + k) (b) ( 2 $i − $j − 2k$ )
3 5
Sol. (a) A vector parallel to the bisector of the angle between the
1 1
(c) ( 2 i$ + 2 j$ − k$ ) (d) (3 i$ − 7 j$ − k$ ) vectors a and b is
3 59 a b
+ = a$ + b$
Sol. (c) b × c = 2 $i + 2 $j − k$ | a| |b|
[a b c ] = a ⋅ (2 i$ + 2 $j − k$ ) = 1 ⋅ 3 ⋅ cosθ ≤ 3 ∴Units vector along the bisector
Chap 02 Product of Vectors 99

a$ + b$ 1 l Ex. 60 Let r, a, b and c be four non-zero vectors such


= = ($a + b$ )
| a$ + b$ | 2 that r ⋅ a = 0,
($a + b$ ) 2 = | a | 2 + | b$ | + 2a$ ⋅ b$ = 1 + 1 + 0 = 2 | r × b | = | r | | b |, | r × c | = | r | | c |, then [ a b c ] is
∴ Required projection (a) | a | | b | | c | (b) − | a | | b | c |
(c) 0 (d) None of these
 a b a ×b  1
= l +m +n ⋅ ($a + b$ ) Sol. (c) Given, r ⋅ a = 0
 |a| |b| | a × b |  2
| r× b| =| r || b|
1
= (l + m ) and | r × c| =| r|| c|
2
This shows r is perpendicular to both b and c.
 a ⋅ a$ b ⋅ b$ 
∴ | a | = | b | = 1 and a$ ⋅ b = a$ ⋅ (a × b ) = b ⋅ ($a × b) = 0 
$ $ ⇒ r is perpendicular to a, b and c
 
∴ [a b c ] = 0
l Ex. 58 If the volume of parallelopiped formed by the l Ex. 61 If a, b and c are any three vector forming a
vectors a, b, c as three coterminous edges is 27 cu units, then linearly independent system, then θ ∈R,
the volume of the parallelopiped have α = a + 2b − c,  2π   2π 
β = a − b and γ = a − b − c as three coterminous edges is [a cos θ + b sin θ + c cos 2θ, a cos  + θ  + b sin  + θ
 3   3 
(a) 27 (b) 9
 2π 
(c) 81 (d) None of these + c cos 2  + θ
Sol. (c) We have, |[a b c ]| = 27 cu units  3 
1 2 −1  2π   2π   2π 
a cos  θ −  + b sin  θ −  + c cos 2  θ −  is
Now, [α β γ ] = 1 − 1 0 [a b c ] = 3[a b c ]  3   3   3 
1 −1 −1 (a) [ a b c ]cos θ
∴Required volume = |[α β γ ]| (b) [ a b c ]cos 2θ
= 3|[a b c ]| (c) [ a b c ]cos 3θ
(d) None of the above
= 3 × 27 = 81 cu units
Sol. (d) Since a, b and c are linearly independent,
l Ex. 59 If V is the volume of the parallelopiped having [a b c ] ≠ 0
three coterminous edges as a, b and c, then the volume of We know that,
parallelopiped having three coterminous edges as [ax1 + bx 2 + cx 3ay1 + by 2 + cy 3 az1 + bz 2 + cz 3 ]
α = (a ⋅ a ) a + ( a ⋅ b ) b + (a ⋅ c )c, β = ( a ⋅ b ) a + (b ⋅ b ) b + (b ⋅ c )c x1 x 2 x3
and γ = ( a ⋅ c ) a + (b ⋅ c ) b + (c ⋅ c ) c is = [a b c ] ⋅ y1 y 2 y3
(a)V 3 (b) 3V z1 z 2 z3
(c)V 2 (d) 2V Hence, the given system can be written as
Sol. (a) We have, |[a b c ]| = V
Let V1 be the volume of the parallelopiped formed by the cosθ sin θ cos2θ
vectors α, β and γ. Then,  2π   2π   2π 
[a b c ] cos  + θ sin  + θ cos2 θ + 
V1 = |[α β γ ]|  3   3   3
a ⋅a a ⋅ b a ⋅ c  2π   2π   2π 
cos θ −  sin θ −  cos2 θ − 
 3  3  3
Now, [α β γ ] = a ⋅ b b ⋅ b b ⋅ c [a b c ]
a⋅ c b⋅ c c⋅ c On applying R3 → R3 + R2 + R1, we get

⇒ [α β γ ] = [a b c ]2 [a b c ] cosθ sin θ cos2θ


 2π   2π   2π 
[a b c ] cos  + θ sin  + θ cos2 θ + 
⇒ [α β γ ] = [a b c ]3  3   3   3
0 0 0
Q V1 = |[α β γ ]
= |[a b c ]3| = V 3 ⇒[a b c ] ( 0 ) = 0
100 Textbook of Vector & 3D Geometry

l Ex. 62 Let a, b, c be three non-coplanar vectors and d be where, θ is an acute angle between a and c.
a non-zero vector, which is perpendicular to a + b + c. Now, ⇒ cosθ =
3
if d = (sin x )( a × b ) + (cos y ) (b × c ) + 2(c × a ), then the 2
minimum value of ( x 2 + y 2 ) is π
∴ θ=
6
π2
(a) π 2 (b)
2 l Ex. 64 Let a = 2 $i + $j + k$ , b = $i + 2 $j − k$ and c is a unit
π2 5π 2
(c) (d) vector coplanar to them. If c is perpendicular to a, then c is
4 4 equal to
Sol. (d) Given, d ⋅ (a + b + c ) = 0 1 1 $ $ $
(a) ( − $j + k$ ) (b) − (i + j + k)
and d = sin x (a × b ) + cosy ( b × c ) + 2 ( c × a ) …(i) 2 3
Q a ⋅ d = cosy [a b c ] … (ii) 1 $ 1 $ $ $
(c) ( i − 2$j ) (d) (i + j + k)
b ⋅ d = 2 [b c a] … (iii) 5 3
c ⋅ d = sin x [a b c ] … (iv) Sol. (a) a × (a × b ) = (a ⋅ c ) a − (a ⋅ a ) b
On adding Eqs. (ii), (iii) and (iv), we get
= 3( i$ + $j + k$ ) − 6( $i + 2 $j − k$ ) = − 9 $j + 9 k$
a ⋅ d + b ⋅ d + c ⋅ d = (cosy + 2 + sin x ) [a b c ]
a × (a × b )
Q sin x + cosy + 2 = 0 ∴Required unit vector =
| a × (a × b )|
⇒ sin x + cosy = − 2
1
⇒ sin x = − 1 =± ( − $j + k$ )
2
[Q −1 < sin x ≤ 1 and − 1 ≤ cosy ≤ 1]
and cosy = − 1 l Ex. 65 Let a = 2 $i + $j − 2k$ and b = $i + $j. If c is a vector
Since, we have to find the minimum value of
such that a ⋅ c = | c |, | c − a | = 2 2 and the angle between
π
x 2 + y 2; x = − ,y = π a × b and c is 30°, then | ( a × b ) × c | is equal to
2
2 3
π2 5π 2 (a) (b)
∴ x2 + y 2 = + π2 = 3 2
4 4
(c) 2 (d) 3
l Ex. 63 Let a, b, c be three vectors of magnitude 1, 1 and Sol. (b) | c − a | = 2 2
2 respectively. If a × ( a × c ) + b = 0, then the acute angle ⇒ | c | 2 + | a | 2 − 2a ⋅ c = 8
between a and c is ⇒ | c | 2 + ( 9 ) 2 − 2| c | = 8
π π
(a) (b) ⇒ | c |2 − 2 | c | + 1 = 0
3 4
π ⇒ (| c | − 1 ) 2 = 0 ⇒ | c | = 1
(c) (d) None of these
6 Now, a × b = 2 $i + 2 $j + k$
Sol. (c) Given, | a | = 1, | b | = 1, and | c | = 2 ∴ |a × b| = 4 + 4 + 1 =3
Also, a × (a × c ) + b = 0
∴ | (a × b ) × c| = | a × b| | c | sin 30 °
⇒ (a ⋅ c ) a − (a ⋅ a ) c + b = 0 1 3
=3 ×1 × =
⇒ (a ⋅ c )a − c + b = 0 [Q a ⋅ a = | a | 2 = 1 ] 2 2
⇒ (a ⋅ c )a − c = − b
⇒ | (a ⋅ c )a − c | = | − b |
1
l Ex. 66 Let a$ and b$ be two unit vectors such that a$ ⋅ b$ =
⇒ | (a ⋅ c )a − c | = | b |
2 2 3
and a$ × b = c$. Also F = αa$ + βb + γc,
$ $ $ where α, β, γ are
⇒ | (a ⋅ c )a | 2 + | c | 2 − 2 {(a ⋅ c )a ⋅ c } = | b | 2
scalars. If α = k 1 (F ⋅ a$ ) − k 2 (F ⋅ b ), then the value of
$ $ $
⇒ (a ⋅ c ) | a | 2 + | c | 2 − 2(a ⋅ c ) (a ⋅ c ) = | b | 2
2(k 1 + k 2 ) is
⇒ (a ⋅ c ) 2 {| a | 2 − 2 } + | c | 2 = | b | 2
(a) 2 3 (b) 3
⇒ − (a ⋅ c ) 2 + 4 = 1 (c) 3 (d) 1
⇒ a⋅c = ± 3 Sol. (c) F =αa$ + βb$ + γc$
⇒ | a |⋅| c | cosθ = 3 ∴ F ⋅ ( b$ × c$ ) = α [$a b$ c$ ]
Chap 02 Product of Vectors 101

F$ ⋅ ( b$ × c$ ) b×c
∴ α= Sol. (a) We have, a × ( b × c ) =
[$a b$ c$ ] 2
1 1
Now, b$ × c$ = b$ × ($a × b$ ) = ( b$ ⋅ b$ )$a − ( b$ ⋅ a$ ) b$ ⇒ (a ⋅ c ) b − (a ⋅ b ) c = b+ c
2 2
1
= a$ − b$ 1
3 ⇒ (a ⋅ c ) − =0 [Qa, b, c are non-coplanar]
2
$ $ ] = ($a × b$ ) 2 = | a$ | 2 | b$ | 2 − ($a ⋅ b$ ) 2
[$abc 1
and a⋅b = −
1 8 2
=1 − =
9 9 1 1
⇒ | a| | b | cosθ = − ⇒ cosθ = −
9 1  2 2
∴ α = F$ ⋅ a$ − F$ ⋅ b$ 
8 3  3π
∴ θ=
9 3 4
∴ k1 = , k2 =
8 8
⇒ k1 + k2 =
12 3
=
l Ex. 69 The unit vector which is orthogonal to the vector
8 2 3 $i + 2 $j + 6 k$ and is coplanar with the vectors 2 $i + $j + k
$ and
∴ 2 (k1 + k2 ) = 3 $i − $j + k $ is
2 $i − 6 $j + k$ 2 $i − 3 $j
l Ex. 67 Let a = $i − $j , b = $j − k$ and c = k$ − $i. If d is a unit (a) (b)
41 13
vector such that a ⋅ d = 0 = (b c d), then d is equal to $ $
3j− k 4 i + 3 $j − 3 k$
$
( $i + j$ − 2k$ ) ( $i + $j − k$ ) (c) (d)
(a) ± (b) ± 10 34
6 3
Sol. (c) Let a = 3 i$ + 2 $j + 6 k$ , b = 2 i$ + $j + k$ , c = $i − $j + k$
( i + j + k$ )
$ $
(c) ± (d) ± k$ Then, by definition, a vector orthogonal to a and coplanar to b
3 and c is given by
Sol. (a) We have, a ⋅ d$ = 0 and [ b c d$ ] = 0 ⇒ a × (b × c)
⇒a ⊥ d$ and b, c, d$ are coplanar. ⇒ a × ( b × c ) = (a ⋅ c ) b − (a ⋅ b ) c
⇒ d$ ⊥ a and d$ lies in the plane of b and c, we know that the = 7(2 i$ + $j + k$ ) − (14 )( $i − $j + k$ ) = 21 $j − 7 k$
vector r = a × ( b × c ) is perpendicular to a and lies in the plane a × (b × c) 3 $j − k$
of b and c Hence, a unit vector = =
| a × (b × c) | 10
r
∴ d$ = ±
|r| l Ex. 70 Let a, b and c be non-zero vectors such that
Now, r = a × (b × c) 1
( a × b ) × c = | b | | c | a. If θ is the acute angle between the
⇒ r = (a ⋅ c ) b − (a ⋅ b ) c 3
⇒ r = − ( $j − k$ ) + ( k$ − $i ) vector b and c, then sin θ is equal to
= − $i − $j + 2 k$ 2 2 2
(a) (b)
r ( − $i − $j + 2 k$ ) 3 3
∴ d$ = ± =± 2 1
|r| 1+1+4 (c) (d)
3 3
i$ + $j − 2 k$ 1
=± Sol. (a) (a × b ) × c = | b || c | a
6 3
1
⇒ (a ⋅ c ) b − ( b ⋅ c ) a = | b || c | a
l Ex. 68 If a, b and c are non-coplanar unit vectors such 3
1 1
that a × (b × c ) = (b + c ), then the angle between a and b ⇒ (a ⋅ c ) b = {( b ⋅ c ) + | b || c | }a
2 3
 1
is ⇒ (a ⋅ c ) b = | b || c | cosθ + a
3π π  3
(a) (b) 1
4 4 As a and b are not parallel, a ⋅ c = 0 and cosθ + =0
3
π
(c) (d) π 1 2 2
2 ⇒ cosθ = − ⇒ sinθ =
3 3
102 Textbook of Vector & 3D Geometry

Ex. 71 The value for [ a × (b + c ), b × (c − 2 a ), ( b × c ) (a ⋅ c ) b − (a ⋅ b ) c


l
a × p =a × =
[a b c ] [a b c ]
c × ( a + 3b )] is equal to
(a ⋅ b ) c − ( b ⋅ a )a
(a) [ a b c ]2 Similarly, b × q =
[a b c ]
(b) 7[ a b c ]2 ( b ⋅ c )a − (a ⋅ c ) b
and c×r=
(c) − 5[ a × b b × c c × a ] [a b c ]
(d) None of the above ∴ a×p+b×q+c×r
Sol. (b) Let a × b = l, b × c = m and c × a = n, 1
= {(a ⋅ c ) b − (a ⋅ b ) c + (a ⋅ b ) c − ( b ⋅ c )a
∴ [a × ( b + c ), b × ( c − 2a ), c × (a + 3 b )] [a b c ]
= [l − n, m + 2l , n − 3m ] + ( b ⋅ c )a − (a ⋅ c ) b }
1 0 −1 1
= ×0=0
= 2 1 1 [lmn ] [a b c ]
0 −3 1
= 7[lmn ] = 7 [a × b b × c c × a ]
l Ex. 73 Solve a ⋅ r = x, b ⋅ r = y, c ⋅ r = z, where a, b, c are
= 7[a b c ]2 given non-coplanar vectors.
Sol. Given a ⋅ r = x, b ⋅ r = y, c ⋅ r = z
l Ex. 72 If a, b, c and p, q, r are reciprocal system of Let a′, b′, c′, be the reciprocal vectors of a, b, c, respectively.
vectors, then a × p + b × q + c × r is equal to b ×c c ×a a×b
Then, a = , b′ = ,c=
[a b c ] [a b c ] [a b c ]
(a) [ a b c ] (b) [p + q + r ]
(c) 0 (d) a + b + c Now, r = ( r ⋅ a )a′ + ( r ⋅ b ) b′ + ( r ⋅ c ) c′
b×c c ×a a×b = xa′ + yb′ + zc′
Sol. (c) p = ,q= ,r=
[a b c ] [a b c ] [a b c ]

JEE Type Solved Examples :


More than One Option Correct Type Questions
l Ex. 74 If z = a $ i + b$j and z 2 = c $i + d $j are two vectors in l Ex. 75 If unit vectors $i and $j are at right angles to each
1
$i and $j system, where | z1 | = | z 2 | = r and z1 ⋅ z 2 = 0, then other and p = 3 $i + 4 $j, q = 5i, 4r = p + q and 2s = p − q, then
w 1 = a $i + c$j and w 2 = b $i + d$j satisfy (a) | r + ks | = | r − ks | for all real k
(a) | w1 | = r (b) r is perpendicular to s
(b) | w 2 | = r (c) r + s is perpendicular to r − s
(c) w1 ⋅ w 2 = 0 (d) | r | = | s | = | p | = | q |
(d) None of the above Sol. (a, b, d) We have, p = 3 $i + 4 $j and p = 5 $i
Sol. (a, b, c) | z1| = | z 2 | = r and z1, z 2 = 0 Also, 4r = p + q = 3 $i + 4 $j + 5 $i
⇒ a2 + b2 = c2 + d 2 = r … (i) = 8 $i + 4 $j ⇒ r = 2 i$ + $j
and ac + bd = 0 and 2s = p − q = 3 $i + 4 $j + 5 $i = − 2 $i + 4 $j
as, ac = − bd
⇒ s = − i$ + 2 $j
a c
⇒ = =λ …(ii) Now, | r + ks | = | r − ks |
b −d
⇒ | 2 i$ + $j − ki
$ $ + 2k$j | 2 = | 2 $i + $j + k$i − 2k$j | 2
From Eqs. (i) and (ii),
⇒ (2 − k ) 2 + (1 + 2k ) 2 = (2 + k ) 2 + (1 − 2k ) 2
a 2(1 + λ2 ) = d 2(1 + λ2 )
Which is true for all values of k.
⇒ a 2 = d 2 and b 2 = c 2
Now, r ⋅ s$ = (2 $i + $j) ( − $i + 2 $j)
Now, | w1 | = a 2 + c 2 = a 2 + b 2 = h = | w 2 |
= −2 + 2 = 0
w1 ⋅ w 2 = ab + bd = 0 ∴ r ⊥s
Chap 02 Product of Vectors 103

Also, ( r + s ) ⋅ ( r − s ) = ( $i + 3 $j) (3 $i − $j) = 3 − 3 = 0 l Ex. 78 If vectors b = (tan α, − 1, 2 sin α / 2 ) and


∴ (r + s ) ⊥ (r − s )
 3 
Also, | r | = (2 ) 2 + 1 = 5 c =  tan α, tan α, −  are orthogonal and vectors
 sin α / 2 
| s | = ( − 1 ) 2 + (2 ) 2 = 5 a = (1, 3, sin 2α ) makes an obtuse angle with the Z -axis, then
| p | = 32 + 42 = 5 the value of α is
(a) α = ( 4n + 1)π + tan− 1 2
| q | = 52 = 5
(b) α = ( 4n + 1)π − tan− 1 2
∴ | r | = | s | and | p | = | q |
(c) α = ( 4n + 2)π + tan− 1 2
l Ex. 76 a, b and c are three vectors such that (d) α = ( 4n + 2)π − tan −1 2
a ⋅ a = b ⋅ b = c ⋅ c = 3 and | a − b | 2 + | b − c | 2 + | c − a | 2 = 27, Sol. (b, d), Since, a = (1, 3, sin 2α ) makes an obtuse angle with the
then Z-axis, its z-component is negative.
(a) a, b and c are necessarily coplanar. Thus, − 1 ≤ sin 2α < 0 …(i)
(b) a, b and c represent sides of a triangle in magnitude But b ⋅ c = 0 (Q orthogonal)
and direction tan 2 α − tan α − 6 = 0
(c) a ⋅ b + b ⋅ c + c ⋅ a has the least value − 9 / 2
∴ (tan α − 3 ) (tan α + 2 ) = 0
(d) a, b and c represent orthogonal triad of vectors
⇒ tanα = 3,− 2
Sol. (a, b, c) Here,
Now, tanα = 3.
| a − b | 2 +| b − c | 2 + | c − a | 2
2 tan α 6 3
Therefore, sin 2α = = =
= 2 (| a | 2 + | b | 2 + | c | 2 − a ⋅ b − b − b ⋅ c − c ⋅ c) 1 + tan 2 α 1 + 9 5
9 (not possible as sin2α < 0)
⇒ a ⋅ b + b ⋅ c + c ⋅a = −
2 Now, if tanα = − 2,
Now, | a + b + c | 2 = | a | 2 + | b | 2 + 2(a ⋅ b + b ⋅ c + c ⋅ a ) 2 tan α −4 −4
⇒ sin 2α = = =
 9 1 + tan 2 α 1 + 4 5
= 3 + 3 + 3 − 2  = 0
 2 ⇒ tan2α > 0
∴ a+b+c=0 …(i) Hence, 2α is the third quadrant. Also, sin α / 2 is meaningful.
Also, | a + b + c |2 ≥ 0 If 0 < sin α / 2 < 1, the
9 α = ( 4n + 1 ) π − tan − 1 2
⇒ a ⋅ b + b ⋅ c + c ⋅a ≥ − …(ii)
2 and α = ( 4n + 2 ) π − tan − 1 2
Thus, least value is − 9 / 2
l Ex. 79 If a and b are any two unit vectors, then the
l Ex. 77 If a and b are non-zero vectors such that 3|a +b |
| a + b | = | a − 2b |, then possible integers in the range of + 2 | a − b |, is/are
2
(a) 2 a ⋅ b = | b |2 (a) 2 (b) 3
(b) a ⋅ b = | b | 2
(c) 4 (d) 5
1 Sol. (b, c, d) We have, | a | = | b | = 1
(c) least value of a ⋅ b + is 2
|b| + 2
2
Let θ be the angle between a and b.
1 θ
(d) least value of a ⋅ b + is 2 − 1 | a + b | = 2 cos
|b| + 2 2
θ
Sol. (a, d) | a + b | = | a − 2b | and | a − b | = 2 sin
2
| b |2 θ θ
⇒ a⋅b = Q 3 cos + 4 sin : θ ∈ [ 0, π ]
2 2 2
1 | b |2 + 2 1 θ θ
Also, a⋅b + = + −1 − 5 ≤ 3 cos + 4 sin ≤ 5
| b |2 + 2 2 | b |2 + 2 2 2
≥ 2 − 1 (using AM ≥ GM ) The possible range are 3, 4 or 5.
104 Textbook of Vector & 3D Geometry

l Ex. 80 Which of the following expressions are meaning- Sol. (a, d) Given, a ⋅ b = 0
ful? and pr + ( r ⋅ b ) a = c … (i)
(a) u ⋅ ( v × w ) (b) (u ⋅ v ) ⋅ w On taking dot product by b, we get
(c) (u ⋅ v ) w (d) u × ( v ⋅ w ) p( r ⋅ b ) + ( r ⋅ b )a ⋅ b = b ⋅ c
Sol. (a, c) (i) Since, v × w is a vector, therefore, u ⋅ ( v × w ) is a ⇒ p( r ⋅ b ) = b ⋅ c
scalar quantity.  c − pr 
⇒ p  = b⋅ c
∴ (a) is meaningful.  a 
(ii) ( u ⋅ v ) is scalar. ⇒ pc − p 2r = ( b ⋅ c )a
∴( u ⋅ v ) ⋅ w is not meaningful.
∴ p 2r = pc − ( b ⋅ c )a
(iii) ( u ⋅ v ) is a scalar.
c (b ⋅ c)
So, ( u ⋅ v ) w is a scalar multiple of w. ⇒ r= − a
p p2
∴( u ⋅ v ) w is meaningful.
(iv) ( v ⋅ w ) is a scalar. ⇒ [ r a c] = 0
So, u × ( v ⋅ w ) is not meaningful as cross product is taken for
two vector quantity and not for a vector and scalar.
l Ex. 84 If α( a × b ) + β(b × c ) + γ (c × a ) = 0, then
(a) a, b, c are coplanar if all of α, β, γ ≠ 0
l Ex. 81 If a + 2b + 3c = 0, then a × b + b × c + c × a = (b) a, b, c are coplanar if any one of α, β, γ ≠ 0
(a) 2( a × b ) (b) 6(b × c ) (c) a, b, c are non-coplanar for any α, β, γ ≠ 0
(c) 3(c × a ) (d) 0 (d) None of the above
Sol. (a, b, c) a = − (2 b + 3 c ) Sol. (a, b) We have, α (a × b ) + β( b × c ) + γ( c × a ) = 0
a × b + b × c + c ×a Taking dot product with c, we have
= − (2 b + 3 c ) × b + b × c + c × { − (2 b + 3 c )} α[a b c ] = 0
= − 3 c × b + b × c − 2 c × b = 6( b × c ) Similarly, taking dot product with b and c, we have
Similarly, putting the values of b and c in terms of a and a, b γ[a b c ] = 0, β[a b c ] = 0
respectively in a × b + b × c + c × a, we get the desired results. Now, even if one of α, β, γ ≠ 0, then we have [a b c ] = 0
⇒ a, b, c are coplanar.
l Ex. 82 Let α = a $i + b$j + ck, $ β = b $i + c$j + ak$ and
γ = c $i + a$j + bk$ be three coplanar vectors with a ≠ b and l Ex. 85 If a = $i + $j + k$ and b = $i − $j, then the vectors
v = $i + $j + k$ . Then v is perpendicular to ( a ⋅ $i ) $i + ( a ⋅ $j + ( a ⋅ k$ )k$ , (b ⋅ $i ) $i + (b ⋅ $j ) $j + (b ⋅ k$ )k$ and
(a) α (b) β $i + $j − 2k$
(c) γ (d) None of these (a) are mutually perpendicular
Sol. (a, b, c) It is given that α,β and γ are coplanar vectors. (b) are coplanar
a b c (c) form a parallelopiped of volume 6 units
Therefore, [α β γ ] = 0 ⇒ b c a = 0 (d) form a parallelopiped of volume 3 units
c a b Sol. (a, c) Given a = $i + $j + k$ and b = $i − $j
or 3abc − a 3 − b 3 − c 3 = 0 (a ⋅ $i ) $i + (a ⋅ $j) $j + (a ⋅ k$ ) k$ = i$ + $j + k$ = x (say)
or a + b + c − 3abc = 0
3 3 3
and (b ⋅ i$ ) i$ + (b ⋅ $j) $j + (b ⋅ k$ ) k$ = i$ − $j = y (say)
or (a + b + c ) (a + b + c − ab − bc − ca ) = 0
2 2 2
and $i + $j − 2 k$ = z$ (say)
or a + b + c = 0 [Qa + b + c − ab − bc − ca ≠ 0]
2 2 2
Clearly, x ⋅ y = yz = z ⋅ x = 0
⇒ v ⋅α = v ⋅β = v ⋅ γ = 0 ∴ x, y and z are mutually perpendicular
Hence, v is perpendicular to α, β and γ 1 1 1
Volume of parallelopiped = 1 − 1 0
l Ex. 83 If a is perpendicular to b and p is non-zero scalar 1 1 −2
such that pr + (r ⋅ b ) a = c, then r satisfy = 1(2 − 0 ) − 1( − 2 − 0 ) + 1(1 + 1 )
(a) [r a c ] = 0 (b) p 2r = pa − (c ⋅ a ) b =2 + 2 + 2 =6
(c) p 2r = pb − ( a ⋅ b )c (d) p 2r = pc − (b ⋅ c ) a Q x, y and x are not coplanar, i.e., [ x y z ] ≠ 0
∴Volume of parallelopiped formed by x, y and z is 6 cu units.
Chap 02 Product of Vectors 105

l Ex. 86 The volume of the parallelopiped whose cotermi- lEx. 88 If a, b and c are three non-zero vectors, then
nous edges are represented by the vectors 2b × c, 3c × a and which of the following statement(s) is/are true?
4a × b where a = (1 + sin θ ) $i + cos θ$j + sin 2θk
$ (a) a × ( b × c), b × ( c × a ),c × (a × b ) from a right handed
system.
 2π   2π   4π $
b = sin θ +  $i + cos θ +  $j + sin 2θ +  k, (b) c, (a × b ) × c, a × b from a right handed system
 3  3  3 
(c) a ⋅ b + b ⋅ c + c ⋅ a < 0, if a + b + c = 0
 2π   2π   4π (a × b ) ⋅ ( b × c)
c = sin θ −  $i + cos θ −  $j + sin 2θ −  k$ is 18 (d) = − 1, if a + b + c = 0
 3  3  3  ( b × c) ⋅ (a × c)
 π
cubic units, then the value of θ in the interval 0,  , is/are Sol. (b, c, d)
 2
(a) a × ( b × c ) + b × ( c × a ) + c × (a × b ) = 0
π 2π ⇒ vector are coplanar, so do not form RHS.
(a) (b)
9 9 (b) (a × b ) × c, a × b, c in that order form RHS.
π 4π
(c) (d) ⇒ c, (a × b ) × c, a × b also form RHS as they are in same cyclic
3 9 order.
Sol. (a, b, d) Volume = |[2 b × c 3 c × a 4a × b ]| = 18 (c) a + b + c = 0 ⇒| a + b + c | 2 = 0
⇒ 24[a b c ] = 18
2
⇒ a 2 + b 2 + c 2 = − 2(a ⋅ b + b ⋅ c + c ⋅ a )
3 ⇒ a ⋅ b + b ⋅ c + c ⋅a ⋅ < 0
⇒ |[a b c ]| =
2 (d) a + b + c = 0
⇒ a ×b = b × c = c ×a
(1 + sin θ ) cosθ sin 2θ
 2π   2π   4π 
Now, [a b c ] = sin θ +  cosθ +  sin θ +  l Ex. 89 Let the unit vectors a and b be perpendicular and
 3  3  3 
unit vector c is inclined at angle α to a and b. If
 2π   2π   4π 
sin θ − 

cosθ −  sin 2θ −  c = la + mb + n( a × b ), then
3  3  3 
(a) l = m (b) n 2 = 1 − 2l 2
Applying R1 → R1 + R2 + R3 and expanding
1 + cos 2α
3 (c) n 2 = − cos 2α (d) m 2 =
|[a b c ]| = 3 | cos3θ | = 2
2
Sol. (a, b, c, d)
1 π 2π 4π
⇒ cos3θ = ± ⇒3θ = , , a ⋅ b = 0, c ⋅ a = c ⋅ b = cosα
2 3 3 3
π 2π 4π Take dot products with a , b and c respectively.
⇒ θ= , , ⇒ l = m, l 2 + m 2 + n 2 = 1
9 9 9
n 2 = − cos2α,
l Ex. 87 If a = x $ i + y$j + zk$ , b = y $i + z$j + xk$ and 1 + cos2α
m2 =
c = z $i + x$j + yk$ , then a × (b × c ) is/are 2

(a) parallel to (y − z )i$ + ( z − x )$j + ( x − y )k$ lEx. 90 If a × (b × c ) is perpendicular to ( a × b ) × c, we


(b) orthogonal to $i + j$ + k$ may have
(c) orthogonal to (y + z )i$ + ( z + x )$j + ( x + y )k$ (a) ( a ⋅ c ) | b |2 = ( a ⋅ b ) (b ⋅ c ) (b) a ⋅ b = 0
(d) parallel to $i + $j + k$ (c) a ⋅ c = 0 (d) b ⋅ c = 0
Sol. (a, b, c) a × ( b × c ) = (a ⋅ c ) b − (a ⋅ b ) c Sol. (a, c)
a × ( b × c ) = (a ⋅ c ) b − (a ⋅ b ) c
= ( xz + yx + yz ) (y$i + z$j + xk$ )
and (a × b ) × c = − ( c ⋅ b )a + (a ⋅ c ) b
− ( xy + yz + zx ) (zi$ + x$j + yk$ )
We have been given
= ( xy + yz + zx ) [(y − z ) $i + (z − x ) $j + ( x − y ) k$ ] (a × ( b × c )) ⋅ ((a × b ) × c ) = 0
Clearly, 1 to i$ + $j + k$ and also to ∴ ((a ⋅ c ) b − (a ⋅ b ) c ) ⋅ ((a ⋅ c ) b − ( c ⋅ b )a ) = 0
(y + z ) i$ + (z + x ) $j + ( x + y ) k,
$ as dot product are zero, clearly
or (a ⋅ c ) 2| b | 2 − (a ⋅ c )( b ⋅ c )(a ⋅ b )
parallel to
− (a ⋅ b )(a ⋅ c )( b ⋅ c ) + (a ⋅ b )( b ⋅ c )( c ⋅ a ) = 0
(y − z ) $i + (z − x ) $j + ( x − y ) k$
or (a ⋅ c ) 2| b | 2 = (a ⋅ c )(a ⋅ b )( b ⋅ c )
106 Textbook of Vector & 3D Geometry

or (a ⋅ c )((a ⋅ c )( b ⋅ b ) − (a ⋅ b )( b ⋅ c )) = 0 (a) tan− 1( 3 ) (b) tan− 1(1 / 3 )


a⋅c = 0 (c) cot − 1(0) (d) tan− 1(1)
or (a ⋅ c )| b | = (a ⋅ b )( b ⋅ c )
2
Sol. (a, b, c) Consider V1 ⋅ V2 = 0
⇒ A = 90 °
l Ex. 91 If ( a × b ) × (c × d) ⋅ ( a × d) = 0, then which of the
b − ($a ⋅ b )$a 3 | a$ × b$ |
following may be true? Using the sine law, =
sin θ cosθ
(a) a, b, c and d are necessarily coplanar
(b) a lies in the plane of c and d A

(c) b lies in the plane of a and d v1=Ö3(a×b) v2=b –(a . b)a


(d) c lies in the plane of a and d
q (p/2)–q
Sol. (b, c, d) (a × b ) × ( c × d ) ⋅ (a × d ) = 0 B C
or [a c d ] b − [ b c d ]a ) ⋅ (a × d ) = 0 1 | b − ($a ⋅ b$ )$a |
or tan θ =
or [a c d ] [ d a d ] = 0 3 | a$ × b$ |
Hence, either c or b must lie in the plane of a and d. 1 | ($a × b$ ) × a$ |
=
3 | a$ × b$ |
l Ex. 92 The angles of a triangle, two of whose sides are
represented by vectors 3( a$ × b$ ) and b$ − ( a ⋅ b$ ) a$ , where b$ is 1 | a$ × b$ | | a$ |sin 90 ° 1
= =
3 | a$ × b |
$ 3
a non-zero vector and a$ is a unit vector in the direction of a$ ,
are π
or θ=
6

JEE Type Solved Examples :


Statement Type I & II Questions
n
Directions (Q. Nos. 93-96) This section is based on l Ex. 94 p, q and r are three vectors defined by
Statement I and Statement II. Select the correct answer
p = a × (b + c ), q = b × (c + a ) and r = c × ( a + b )
from the codes given below.
(a) Both Statement I and Statement II are correct and
Statement I p, q and r are coplanar.
Statement II is the correct explanation of Statement I Statement II Vectors p, q, r are linearly independent.
(b) Both Statement I and Statement II are correct but Sol. (c) Statement I p + q + r = a × ( b + c ) + b
Statement II is not the correct explanation of × ( c + a ) + c × (a + b )
Statement I = a × b+ a × c+ b × c+ b × a + c × a + c × b
(c) Statement I is correct but Statement II is incorrect = a × b+ a × c+ b × c − a × b − a × c − b × c
(d) Statement II is correct but Statement I is incorrect ∴p = − q − r (a linear combination of q and r)
Therefore, p, q, r are coplanar and hence statement I is true.
l Ex. 93 Let the vectors PQ, OR, RS, ST, TU and UP
Statement II p + q + r = 0 ⇒ p, q, r are not linearly
represent the sides of a regular hexagon. independent.
Statement I PQ × (RS + ST ) ≠ 0 Therefore, statement II is not true.
Statement II PQ × RS = 0 and PQ × ST ≠ 0
Sol. (c) Clearly, RS + ST = RT, which is not parallel to PQ. p q
l Ex. 95 Statement I If in a ∆ABC, BC = − and
∴ PQ × ( RS + ST ) ≠ 0
| p| |q|
2p
So, Statement I is correct. AC = ; | p| ≠| q |, then the value of cos 2 A + cos 2B + cos 2C
Also, PQ is not parallel to RS. |p|
∴ PQ × RS ≠ 0 is −1.
So, Statement II is not correct. Statement II If in ∆ABC, ∠C = 90 °, then
cos 2 A + cos 2B + cos 2C = −1
Chap 02 Product of Vectors 107

Sol. (b) Statement II In ∆ABC , ∠C = 90 ° = 2 cos(180 °− B ) cos( A − C ) − 1


∴ cos2 A + cos2 B + cos2C = − 2 cos B cos( A − C ) − 1 = − 1 (Q B = 90 ° )
= 2 cos( A + B ) cos( A − B ) + cos180 ° Therefore, Statement I is also true.
= cos (180 ° − C ) cos ( A − B ) − 1 Thus both Statements are true but Statement II is not the
= −2 cosC cos( A − B ) − 1 correct explanation of Statement I.
= 0 − 1 = −1 [Q cosC = cos90 °= 0]
Therefore, Statement II is true. l Ex. 96 Statement I If a is perpendicular to b and c,
Statement I BC = p$ − q$, AC = 2 p$ then a × (b × c ) = 0
∴ AB = AC + CB = 2 p$ − ( p$ − q$) = p$ + q$ Statement II If b is perpendicular to c, then b × c = 0
Now, AB ⋅ BC = ( p$ + q$).( p$ − q$) =| p$| 2 −| q$| 2 = 1 − 1 = 0 Sol. (c) If a is perpendicular to b and c, then a ||( b × c )
∴ ∠B = 90 ° ∴ a × (b × c) = 0
Now, cos2 A + cos2 B + cos2C Therefore, Statement I is true.
= cos 2 A + cos2C + cos 2 B But, if b ⊥ c, then b × c ≠ 0
= 2 cos( A + C ) cos( A − C ) + cos180 ° Therefore, Statement II is not true.

JEE Type Solved Examples :


Passage Based Type Questions
Passage I 41 $
Sol. (a) a1 ⋅ b = − (2 i − 3 $j + 6 k$ ) ⋅ (2 $i − 3 $j + 6 k$ ) = − 41
(Ex. Nos. 97-99) 49
Let a = 2 $i + 3 $j − 6 k
$ , b = 2 $i − 3 $j + 6 k
$ and c = − 2 $i + 3 $j + 6 k
$.
l Ex. 99. Which of the following is true?
Let a 1 be the projection of a on b and a 2 be the projection
(a) a and a 2 are collinear
of a 1 and c. Then
(b) a 1 and c are collinear
l Ex. 97 a 2 is equal to (c) a , a 1 and b are coplanar
(d) a , a 1 and a 2 are coplanar
943 $ 943
(a) ( 2i − 3 j$ − 6k$ ) (b) ( 2$i − 3 j$ − 6k$ ) Sol. (c) a, a1 and b are coplanar because a1 and b are collinear.
49 492
943 943
(c) ( − 2$i + 3 $j + 6k$ ) (d) 2
( −2$i + 3 j$ + 6k$ ) Passage II
49 49 (Ex. Nos. 100-102)
Sol. (b)
Let a, b be two vectors perpendicular to each other and
 (2 $i − 3 $j + 6 k$ )  2 $i − 3 $j + 6 k$ | a| = 2, | b| = 3 and c × a = b.
a1 = (2 i$ + 3 $j − 6 k$ ). 
 7  7
−41 $ l Ex. 100 The least value of | c − a| is
= (2 i − 3 $j + 6 k$ )
49 1
(a) 1 (b)
−41  $ ( −2 $i + 3 $j + 6 k$ ) ( −2 $i + 3 $j + 6 k$ ) 2
a2 = (2 i − 3 $j + 6 k$ ). ×
49  7  7 1 3
(c) (d)
−41
4 2
= ( − 4 − 9 + 36 ) ( − 2 i$ + 3 $j + 6 k$ )
( 49 ) 2 l Ex. 101 When | c − a| is least the value of α (when α is
943
= 2 (2 i$ − 3 $j − 6 k$ ) angle between a and c) equals
49
 1 3
(a) tan−1   (b) tan−1  
 2  4
l Ex. 98 a 1 . b is equal to
 2
(a) − 41 (b) −
41 (c) cos −1   (d) None of these
3
7
(c) 41 (d) 287
108 Textbook of Vector & 3D Geometry

l Ex. 102 When | c − a| attains least value, then the value l Ex. 105 The length of the perpendicular from vertex D
of | c| is on the opposite face is
1 7 14 2
(a) (b) (a) (b)
2 2 6 6
5 3
(c) (d) 4 (c) (d) None of these
2 6
Sol. For (Ex. Nos. 100-102) Sol. For (Ex. Nos. 103-105)
Here, | a| = 2, | b| = 3  4 1 8
Point G is  , ,  . Therefore,
c × a = b ⇒ | c | | a |sin α = | b |  3 3 3
3  5
2
1  5 51
2
⇒ | c | = cosec α | AG| 2 =   + +   =
2  3 9  3 9
Consider, | c − a| 2 = | c| 2 − 2(a . c ) + | a | 2 51
or | AG| =
9  3 3
= cosec 2α + 4 − 2(2 ) .   . cosec α . cosα
4  2 AB = − 4 i$ + 4 $j + 0 k$
25 9
= + . cot 2 α − 6 cot α AC = 2 $i + 2 $j + 2 k$
4 4
2
25  3  D (0, –5, 4)
= +  cot α − 2 − 4
4 2  4, 1, 8
3 3 3
2 h 2
9 3  9
= +  cot α − 2 ≥
4 2  4 G
3 A C (5, 2, 3)
| c − a | ≥ and least possible when (3, 0, 1)
2 1
3 3 3
cot α = 2 ⇒ tanα = ⇒ α = tan −1
2 4 4
3 3 5 B (–1, 4, 1)
Also, |c | = = =
2 sin α  3 2
2  $i $j k$
 5
∴ AB × AC = − 8 1 − 1 0
100. (d) 101. (b) 102. (c)
1 1 1
Passage III = 8( $i + $j − 2 k$ )
(Ex. Nos. 103-105)
1
Consider a triangular pyramid ABCD the position vectors Area of ∆ABC = | AB × AC | = 4 6
2
of whose angular points are A (3, 0, 1), B ( − 1, 4, 1), C (5, 2, 3 ) AD = − 3 $i − 5 $j + 3 k$
and D(0, − 5, 4 ). Let G be the point of intersection of the
medians of triangle BCD. The length of the perpendicular from the vertex D on the
opposite face
= | Projection of AD on AB × AC |
l Ex. 103 The length of vector AG is
51 ( − 3 i$ − 5 $j + 3 k$ ) ( $i + $j − 2 k$ )
(a) 17 (b) =
3 6
3 59 −3 −5 −6
(c) (d) =
6 4 6
14
l Ex. 104 Area of triangle ABC in sq units is =
6
(a) 24 (b) 8 6
103. (b) 104. (c) 105. (a)
(c) 4 6 (d) None of these
Chap 02 Product of Vectors 109

Passage IV 1 1
Sol. (b) ∆ = | AN × BP | = | {(1 − α ) b + αc } × {(1 − α ) c − b } |
(Ex. Nos. 106-108) 2 2
1
Let A, B, C represent the vertices of a triangle, where A is = |(1 − α ) 2( b × c ) + α ( b × c )|
2
the origin and B and C have position vectors b and c
1
respectively. Points M, N and P are taken on sides AB, BC = | b × c|(α 2 − α + 1 )
2
AM BN CP
and CA respectively, such that = = =α 1  1
2
3
AB BC CA = | b × c| α −  + 
2  2 4

A (Origin)
1
∴∆ is least, if α =
P 2
M

Passage V
B(b) N C(c) (Ex. Nos. 109-110)
If AP, BQ and CR are the altitudes of acute ∆ABC and
[Now answer the following questions]
9AP + 4BQ + 7CR = 0.
l Ex. 106. AN + BP + CM is l Ex. 109. ∠ACB is equal to
(a) 3α (b + c) (b) α(b + c) π π
(c) (1 − α ) (b + c) (d) 0 (a) (b)
4 3
AM
Sol. (d) Since = α,  1   1
AB (c) cos − 1   (d) cos − 1  
3 7   7
∴ P. V. of M = α b
BN
Since, =α l Ex. 110. ∠ABC is equal to
BC
 2  π
BN α (a) cos −1   (b)
∴ =  7 2
NC 1 − α
∴Position vector of N = (1 − α ) b + αc  7 π
(c) cos −1   (d)
CP  3  3
Since, =α
CA Sol. For (Ex. Nos. 109-110)
AP 1 − α
∴ = Since, sum of three vectors 9 AP, 4 BQ and 7CR is zero, there is
PC α a ∆ whose sides have lengths 9| AP|, 4| BQ, 7| CR | and are
∴ P.V. of p = (1 − α ) c parallel to the corresponding vectors.
Now, AN = (1 − α ) b + αc H is orthocentre.
BP = (1 − α ) c − b ⇒ ∠BHP = 90 °−∠QBC = ∠ACB
CM = αb − c ⇒ Angle between AP and BQ equal to ∠ACB, similarly angle
∴ AN + BP + CM = 0 between BQ and CR be ∠BAC.
AB BC AC
⇒ = =
l Ex. 107. The vectors AN, BP and CM are 7| CR | 9| AP| 4| BQ|

(a) concurrent (b) sides of a triangle 2. ar. ( ∆ABC ) = | AB × CR | = | BC × AP| = | CA × BQ|


(c) non-coplanar (d) None of these c2 a2 b2
⇒ = =
Sol. (b) Since AN + BP + CM = 0 7 9 4
Hence, AN, BP and CM from the sides of a triangle. ∴ a :b :c =3:2: 7
a2 + b2 − c2 1
l Ex. 108. If ∆ represents the area enclosed by the three
⇒ cosC = =
2ab 2
vectors AN, BP and CM, then the value of α for which ∆ is π
⇒ ∠C = 60 ° =
least 3
1 a2 + b2 − c2 2
(a) does not exist (b) and cos B = =
2 2ac 7
1 109. (b) 110. (a)
(c) (d) None of these
4
110 Textbook of Vector & 3D Geometry

Passage VI Now, we have


(Ex. Nos. 111 to 113) a × b + b × c = pa + q b + r c
⇒ a . (a × b ) + a . ( b × c ) = p (a . a ) + q (a . b ) + r (a . c )
Let a, b, c are non-zero unit vectors inclined pairwise with
the same angle θ. p, q, r are non-zero scalars satisfying ⇒ [a a b ] + [a b c ] = p| a| 2 + q| a||b|cosθ + r | a|| c| cosθ
a × b + b × c = pa + qb + r c ⇒ [a b c ] = p + q cosθ + r cosθ = p + (q + r ) cosθ
∴Volume of parallelopiped = p + (q + r ) cosθ
l Ex. 111 Volume of parallelopiped with edges a, b and c is Taking dot products with a, b, c respectively with given
equation
(a) p + (q + r )cosθ (b) ( p + q + r )cos θ [a b c ] = p + (q + r ) cosθ …(i)
(c) 2p − (q + r )cosθ (d) None of these 0 = ( p + r ) cosθ + q …(ii)
[a b c ] = ( p + q ) cosθ + r …(iii)
q  1 cosθ cosθ
l Ex. 112 The value of  + 2 cos θ  is
p  Also, [a b c ]2 = cosθ 1 cosθ
(a) 1 (b) 0 cosθ cosθ 1
(c) 2[ a b c ] (d) None of these
⇒ [a b c ]2 = (1 − cosθ ) 2(1 + 2 cosθ )
l Ex. 113 The value of |( p + q ) cos θ + r | is ∴ v = |[a b c ]| = |1 − cosθ|| 1 + 2 cosθ |

(a) (1 + cos θ ) 1 − 2 cos θ θ


= 2 sin 2. | 1 + 2 cosθ |
2
θ
(b) 2 sin2 | 1 + 2 cos θ | From Eqs. (i) and (iii), p = r substituting in Eq. (ii), we get
2
2 p cosθ + q = 0
(c) (1 − sinθ ) 1 + 2 cos θ
q
(d) None of the above ⇒ + 2 cosθ = 0
p
Sol. For (Ex. Nos. 111-113)
111. (a) 112. (c) 113. (b)
Volume of parallelopiped = [a b c ] = a . ( b × c )

JEE Type Solved Examples :


Matching Type Questions
lEx. 114 Match the items of Column I with items of a 2 + b 2 + 2a ⋅ b = a 2 + 4b 2 + 4a ⋅ b
Column II. or 2a ⋅ b = − 3b 2 < 0
Column I Column II Hence, angle between a and b is obtuse.
A. If | a + b | = |a + 2b | , then angle between a p. 90° (B) | a + b| = | a − 2 b|
and b is or a 2 + b 2 + 2a ⋅ b = a 2 + 4b 2 − 4a ⋅ b
B. If |a + b | = |a − 2b | , then angle between a q. obtuse or 6a ⋅ b = 3b 2 > 0
and b is
Hence, angle between a and b is acute.
C. If |a + b | = |a − b | then angle between a and r. 0°
b is (C) | a + b| = | a − b|
⇒ a⋅b = 0
D. Angle between a × b and a vector s. acute
perpendicular to the vector c × (a × b ) is Hence, a is perpendicular to b.
(D) c × (a × b ) lies in the plane of vectors a and b.
Sol. A → q; B → s; C → p; D → r A vector perpendicular to this plane is parallel to a × b
(A) | a + b| = | a + 2 b | Hence, angle is 0°.
Chap 02 Product of Vectors 111

lEx. 115 Match the items of Column I with items of lEx. 116 Match the items of Column I with items of
Column II. Column II.
Column I Column II Column I Column II
A. Let |a | = | b | = 2, x = a + b , y = a − b . If p. 4 A. Given two vectors a = 2$i − 3$j + 6k$ , p. 0
1
b = − 2$i + 2$j − k$ and
| x × y | = 2 {λ − (a ⋅ b)2}2 , then the value of
Projection of a on b
λ is λ= , then the value of
Projection of b on a
B. The non-zero value of λ for which angle q. 42
π 3λ is
between λ$i + $j + k$ and $i + λ$j + k$ is , is B. If a = $i + 2$j + 3k$ , b = − $i + 2$j + k$ , q. 7
3
$ $
c = 3i + j and a + pb is normal to c, then p is
C. If | a | = | b | = 1and | c | = 2, then the maximum r. 16
value of | a − 2 b |2 + | b − 2c |2 + | c − 2a |2is C. Let a, b, c be three non-zero vectors such that r. 5
s. 7 a + b + c = 0, then λ (a × b) + c × b
+ a × c = 0, where λ is equal to
Sol. A → (r), B → (p), C → (q) D. The points whose position vectors are s. 2
(A) x × y = (a × b ) × (a − b ) p$i + q$j + r k$ , q$i + r $j + pk$ and r $i + p$j + qk$
= a × a − a × b + b × a − b × b = − 2 (a × b ) are collinear, then the value of
LHS = | x × y| = | − 2 (a × b )| = 2| a|| b|sin θ (p2 + q2 + r2 − pq − qr − rp) is
1 1
RHS = 2 ( λ − (a ⋅ b ) 2 ) 2 = 2( λ − | a| 2 | b| 2 cos2 θ ) 2 Sol. A → (q), B → (r), C → (s), D → (p)
1 (A) Given, a = 2 i$ − 3 $j + 6 k$
Q 8 sin θ = 2( λ − 16 cos2 θ ) 2 and b = − 2 i$ + 2 $j − k$
⇒ 16 sin θ = λ − 16 cos θ
2 2
 a ⋅ b
 
⇒ 16(sin θ + cos2 θ ) = λ
2
Projection of a on b  | b |  | a |
Now, λ = = =
∴ λ = 16 Projection of b on a  b ⋅ a  | b |
 
π 1  |a | 
(B) We have, cos =
3 2
22 + (− 3)2 + 62 4 + 9 + 36 7
( λ$i + $j + k$ ) ⋅ ( $i + λ$j + k$ ) = = =
= (− 2)2 + 22 + (− 1)2 4+4+1 3
λ2 + 2 λ2 + 2
∴ 3λ = 7
1 λ+λ+1
Q = (B) Given, a = $i + 2 $j + 3 k$ ,
2 λ2 + 2
b = − $i + 2 $j + k$ and c = 3 i$ + $j
⇒ λ2 + 2 = 2 (2 λ + 1 )
Since, a + pb is normal to c
⇒ λ2 − 4 λ = 0
⇒ (a + pb ) ⋅ c = 0
⇒ λ(λ − 4) = 0 ⇒ [(1 − p ) i + (2 + 2 p ) $j + (3 + p ) k$ ] ⋅ (3 $i + $j) = 0
$
⇒ λ = 0 and 4
⇒ 3(1 − p ) + 2 + 2 p = 0
∴ λ=4
⇒ 3 − 3p + 2 + 2p = 0
(C) We have, | a − 2 b| 2 + | b − 2 c| 2 + | c − 2a| 2
∴ p =5
= | a| 2 + 4| b| 2 − 4a ⋅ b + | b| 2 + 4| c| 2 − 4 b ⋅ c (C) Given, a + b + c = 0
+ | c| 2 + 4| a| 2 − 4 c ⋅ a ⇒ a ×a + a × b + a × c =a × 0
⇒ 1 + 4 + 1 + 16 + 4 + 4 − 4(a ⋅ b + b ⋅ c + c ⋅ a) ⇒ a×b+a×c=0 …(i)
⇒ 30 − 4(a ⋅ b + b ⋅ c + c ⋅ a ) Also, b ×a + b × b + b × c = b × 0
We know that, ⇒ b ×a + b × c = 0
(a + b + c ) 2 = Σa 2 + 2 Σa ⋅ b ≥ 0 ⇒ −a × b − c × b = 0
⇒ a×b+c×b=0 …(ii)
i.e. 1 + 1 + 4 + 2 (a ⋅ b + b ⋅ c + c ⋅ a ) ≥ 0
On adding Eqs. (i) and (ii), we get
⇒ a ⋅ b + b ⋅ c + c ⋅a ≥ − 3
2(a × b ) + c × b + a × c = 0
⇒ ≥ 30 − 4( − 3 ) ≥ 30 + 12 ≥ 42
On comparing, we get λ = 2
112 Textbook of Vector & 3D Geometry

p q r (A) Given a and b are two unit vectors, i.e., | a | = | b | = 1 and


(D) q r p = 0 π
angle between them is .
r p q 3
|a × b | π
sin θ = ⇒ sin = | a × b |
⇒ ( p + q + r )( p 2 + q 2 + r 2 − pq − qr − rp ) = 0 | a| | b| 3
∴ p 2 + q 2 + r 2 − pq − qr − rp = 0 3
= |a × b|
2
l Ex. 117 Match the items of Column I with items of Now, [a b + a × b b ] = [a b b ] + [a a × b b ]
Column II. = 0 + [a a × b b ]
Column I Column II
= (a × b ) ⋅ ( b × a ) = − (a × b ) ⋅ (a × b )
3
A. If a and b are two unit vectors inclined at p. − 12 = − |a × b |2 = −
π 4
, then 16[ a b + a × b b] is (B) If b and c are orthogonal b ⋅ c = 0
3
Also, it is given that b × c = a.
B. If b and c are orthogonal unit vectors and q. 1
b × c = a , then [a + b + c a + b b + c] is Now [a + b + c a + b b + c ]
C. If | a | = | b | = | c | = 2 and r. 3 = [a a + b b + c ] + [ b + c a + b b + c ]
a ⋅ b = b⋅ c = c ⋅ a = 2, then [ a b c ]cos 45° is = [a b c ] = a ⋅ ( b × c )
equal to = a ⋅ a = | a|2 = 1 (because a is a unit vector)
D. a = 2$i + 3$j − k$ , b = − $i + 2$j − 4 k$ , s. 4 (C) We know that, [a × b b × c c × a ] = [a b c ]2
$ $ $ $ $ $
c = i + j + k and d = 3i + 2 j + k, then a ⋅a a ⋅ b a ⋅ c 4 2 1
1
(a × b) ⋅ (c × d) is equal to and = [a b c ]2 = b ⋅ a b ⋅ b b ⋅ c = 2 4 2 = 32
7
c⋅a c⋅ b c⋅ c 2 2 4
Sol. A → (p); B → (q); C → (s); D → (r) ∴ [a b c ] = 4 2
(D) (a ⋅ c )( b ⋅ d ) − ( b ⋅ c )(a ⋅ d ) = 21

JEE Type Solved Examples :


Single Integer Answer Type Questions
l Ex. 118 Given that u = $i − 2 $j + 3k$ ; v = 2 $i + $j + 4k$ ; l Ex. 119 The position vector of a point P is
w = $i + 3 $j + 3k$ and r = x $i + y $j + zk$ , where x , y , z ∈N and a = $i + 2 $j + k$ . If
(u ⋅ R − 15 ) $i + ( v ⋅ R − 30 ) $j + ( w ⋅ R − 20 )k$ = 0. Then, the r ⋅ a = 20 and the number of possible of P is 9λ, then the
value of λ is
greatest integer less than or equal to | R| is
Sol. (9) r ⋅ a = 20 ⇒ x + 2y + z = 20, x, y , z ∈ N
Sol. (6) Let R = x$i + y$j + zk$
∴ Number of non-negative integer solution are
u = $i − 2 $j + 3 k$ ; v = 2 $i + $j + 4 k$ ; w = $i + 3 $j + 3 k$
⇒ 17
C1 + 15C1 + 13C1 + ... + 1C1
( u ⋅ R − 15 ) $i + ( v ⋅ R − 30 ) $j + ( w ⋅ R − 20 ) k$ = 0 (given)
= 81 = 9λ ⇒ λ = 9
So, u ⋅ R = 15 ⇒ x − 2y + 3z = 15 …(i)
v ⋅ R = 30 ⇒2 x + y + 4z = 30 …(ii) l Ex. 120 Let u be a vector on rectangular coordinate
w ⋅ R = 25 ⇒ x + 3y + 3z = 25 …(iii) system with sloping angle 60°. Suppose that | u − $i| is
Solving, we get geometric mean of | u| and | u − 2 $i|, where $i is the unit vector
x=4 along the X-axis. Then, the value of ( 2 + 1)| u | is
y =2 Sol. (1) Since, angle between u and $i is 60°, we have
z =5 | u|
u ⋅ i$ = | u|| $i| cos60 ° =
Now, | R | = 4 2 + 2 2 + 5 2 = 45 2
[| R | ] = [ 45 ] = 6 Given that, | u |, | u − i|, |, | u − 2 i| are in GP.
$ $
So, | u − i$| 2 = | u | | u − 2 $i|
Chap 02 Product of Vectors 113

Squaring both sides, Sol. (1) We have, a × b = 3a × c


[| u| 2 + | i$| 2 − 2 u ⋅ $i ]2 = | u| 2 [| u| 2 + 4| i$| 2 − 4 u ⋅ $i ] ⇒ a × (b − 3c) = 0
 2 2| u| 
2
2 | u|  ⇒a is parallel to b − 3 c.
| u| + 1 −  = | u| | u| + 4 − 
2
 2   2 Now, b − 3 c = λa
or | u| + 2| u| − 1 = 0 ⇒ | b − 3 c | 2 = λ2| a | 2
2±2 2 ⇒ | b | 2 + 9| c| 2 − 6( b ⋅ c ) = λ2| a| 2
⇒ | u| 2 = −
2 1 1 1
⇒ 1+9× − 6 × 1 × × = λ2(1 ) 2
or | u| = 2 − 1 9 3 2
⇒ ( 2 + 1) | u | = ( 2 + 1) ( 2 − 1) = 2 − 1 = 1 ⇒ 1 + 1 − 1 = λ2 ⇒ λ2 = 1
∴ λ=±1
l Ex. 121 Let A( 2 $i + 3 $j + 5k$ ), B( − $i + 3 $j + 2k$ ) and
C( λ $i + 5 $j + µk$ ) are vertices of a triangle and its median l Ex. 123 If a, b, c are unit vectors such that
through A is equally inclined to the positive directions of the π
a ⋅ b = 0 = a ⋅ c and the angle between b and c is , then the
axes, the value of 2λ − µ is 3
Sol. (4) Median through A is value of | a × b − a × c |.
A Sol. a . b = 0 ⇒ a ⊥ b
a . c = 0 ⇒a ⊥ c
⇒ a⊥b−c
B
D ∴ | a × b − a × c | = | a × ( b − c )|
C
= | a || b − c | = | b − c |
AB + AC π
AD = Now, | b − c | 2 = | b | 2 + | c | 2 − 2 | b || c | cos
2 3
AB = − 3 $i − 3 k$ 1
=2 −2× =1
AC = ( λ − 2 ) $i + 2 $j + (µ − 5 ) k$ 2
1 | b − c| = 1
AD = [( λ − 5 ) $i + 2 $j + (µ − 8 ) k$ ]
2
We have, AD is equally inclined to the positive direction of
l Ex. 124 If the area of the triangle whose vertices are
axes A( − 1, 1, 2); B(1, 2, 3) and C (t ,1, 1) is minimum, then the
1 absolute value of parameter t is
[( λ − 5 ) $i + 2 $j + (µ − 8 ) k$ ] ⋅ $i
Sol. AB = 2 $i + $j + k$ , AC = (t + 1 ) $i + 0 $j − k$
cosθ = 2
| AD|| $i| $i $j k$
1 AB × AC = 2
[( λ − 5 ) i$ + 2 $j + (µ − 8 ) k$ ] ⋅ $j 1 1
= 2
t + 1 0 −1
| AD|| $j|
1 = − $i + (t + 3 ) $j − (t + 1 ) k$
[( λ − 5 ) $i + 2 $j + (µ − 8 ) k$ ] ⋅ k$
=2 = 1 + (t + 3 ) 2 + (t + 1 ) 2
| AD|| k$ |
λ −5 µ −8 = 2t 2 + 8t + 11
⇒ =1 =
2 2 1
Area of ∆ABC = | AB × AC |
⇒ λ = 7 and µ = 10 2
∴ 2 λ − µ = 14 − 10 = 4 1
= 2t 2 + 8t + 1
2
l Ex. 122 Three vectors a(| a | ≠ 0 ), b and c are such that 1
Let f (t ) = ∆2 = (2t 2 + 8t + 1 )
1 4
a × b = 3 a × c, also | a| = | b | =1 and | c | = . If the angle f (t ) = 0 ⇒t = − 2
3
between b and c is 60° and | b − 3c | = λ | a |, then the value of At t = − 2, f ′′(t ) > 0
λ is So, ∆ is minimum at t = − 2
114 Textbook of Vector & 3D Geometry

l Ex. 125 Let OA = a, OB =10 a + 2b and OC = b, where The above three equations will satisfy if the coefficients of α , β
and γ are zero because α , β and γ are three distinct real numbers
O, A and C are non-collinear points. Let p denote the area of
a − 2 = 0 or a = 2,
quadrilateral OACB, and let q denote the area of parallelo-
b − 3 = 0 or b = 3 and c = 0
gram with OA and OC as adjacent sides. If p = k q, then k is
equal to ∴ a 2 + b 2 + c 2 = 2 2 + 3 2 + 0 2 = 4 + 9 = 13
Sol. Here, OA = a, OB = 10a + 2 b, OC = b
l Ex. 127 Let v = 2 $i + $j − k$ and w = $i + 3k$ . If u$ is unit
q = Area of parallelogram with OA and OC as adjacent side.
∴ q = |a × b | …(i) vector and the maximum value of [u v w ] = λ , then the
value of ( λ − 51) is
C B
Sol. (8) We have, [ u v w ] = u . ( v × w )

2b ⇒ [ u v w ] ≤ | u| | v × w| [Qa . b ≤ | a|| b|]


b +
10a ⇒ [ u v w ] ≤ | v × w| [Q| u| = 1]
i$ $j k$
O a A
Now, v × w = 2 1 − 1 = 3 $i − 7 $j − k$
p = Area of quadrilateral OABC 1 0 3
= Area of ∆OAB + Area of ∆OBC
∴ | v × w| = 9 + 49 + 1 = 59
1 1
= | a × (10 a + 2 b )| + |(10 a + 2 b ) × b | Hence, maximum value of [ u v w ] = 59
2 2
= |a × b | + 5 |a × b| On comparing, we get λ = 59
∴ p = 6 |a × b | ∴ λ − 51 = 8
or p = 6q [From Eq. (i)]
l Ex. 128 Let a = α $i + 2 $j − 3k$ , b = $i + 2α$j − 2k$ and
∴ k =6
c = 2 $i − α$j + k$ . Then the value of 6α, such that
l Ex. 126. If x, y are two non-zero and non-collinear {( a × b ) × (b × c )} × (c × a ) = 0, is
vectors satisfying [(a − 2 )α 2 + (b − 3 )α + c ]x + [(a − 2 )β 2 Sol. (4) a = α$i + 2 $j − 3 k$ , b = $i + 2α$j − 2 k$ , c = 2 $i − α$j + k$
+ (b − 3 )β + c ]y + [(a − 2 ) γ 2 + (b − 3 ) γ + c ] ( x × y) = 0 {(a × b ) × ( b × c )} × ( c × a ) = 0
where α, β, γ are three distinct real numbers, then find the or {[a b c ]b − [a b b ]c } × ( c × a ) = 0
value of (a 2 + b 2 + c 2 − 4 ). or [a b c ]b × ( c × a ) = 0
Sol. (9) Since, x and y are non-collinear vectors, therefore or [a b c ]((a . b ) c − ( b . c )a ) = 0
x, y and x × y are non-coplanar vectors. or [a b c ] = 0 (Qa and c are not collinear)
So, [(a − 2 )α 2 + (b − 3 )α + c ]x + [(a − 2 ) β 2 α 2 −3
⇒ 1 2α − 2 = 0
+ (b − 3 )β + c ] y + [(a − 2 ) γ 2 + (b − 3 ) γ + c ] ( x × y ) = 0
2 −α 1
⇒ Coefficient of each vector x, y and x × y is zero.
or α (2α − 2α ) − 2(1 + 4 ) − 3 ( − α − 4α ) = 0 or 10 − 15α = 0
(a − 2 ) α 2 + (b − 3 )α + c = 0
2
∴ α=
(a − 2 ) β 2 + (b − 3 ) β + c = 0 3
(a − 2 ) γ 2 + (b − 3 ) γ + c = 0 6 ×2
6α = =4
3
Chap 02 Product of Vectors 115

Subjective Type Questions


l Ex. 129 Let a and b be two unit vectors such that a$ + b$ Then, AC 2 + BD 2 + AD 2 + BC 2
$
is also a unit vector. Then, find the angle between a$ and b. = ( c − a ) ⋅ ( c − a ) + (d − b ) ⋅ ( d − b )
Sol. Since, a$ + b$ is a unit vector. + (d − a ) . (d − a ) + (c − b) . (c − b)
= | c | 2 + | a | 2 − 2a ⋅ c + | d | 2 + | b | 2 − 2 d ⋅ b
⇒ | a$ + b$ | = 1 ⇒ | a$ + b$ | 2 = 1
+ | d | 2 + | a | 2 − 2a . d + | c | 2 + | b | 2 − 2 b . c
⇒ ($a + b$ ) ⋅ ($a + b$ ) = 1 = | a | + | b | 2 − 2a ⋅ b + | c | 2 + | d | 2 − 2 c ⋅ d
2

⇒ a$ ⋅ a$ + b$ ⋅ b$ + 2 a$ ⋅ b$ = 1 ⇒ 1 + 1 + 2 a$ ⋅ b$ = 1 + | a| 2 + | b| 2 + | c| 2 + | d| 2 − 2a ⋅ b + 2 c ⋅ d

1
a$ ⋅ b$ = −⇒ | a$| | b$ | cosθ = −
1 − 2a ⋅ c − 2 b . d − 2a ⋅ d − 2 b ⋅ c
2 2 = (a − b ) ⋅ (a − b ) + ( c − d ) ⋅ ( c ⋅ d ) +
1 (a + b − c − d ) ⋅ (a + b − c − d )
⇒ cosθ = − (Q| a$| = | b$ | = 1)
2 = AB 2 + CD 2 + (a + b − c − d ) ⋅ (a + b
⇒ θ = 120 ° − c − d ) ≥ AB 2 + CD 2
Hence, the angle between a$ and b$ = 120 °. ∴ AC 2 + BD 2 + AD 2 + BC 2 ≥ AB 2 + CD 2

l Ex. 130 Determine the values of c so that for all real x,


l Ex. 132 Using vector method, prove that the altitudes of
the vectors cx $i − 6 $j + 3 k$ and x $i + 2 $j + 2cxk$ make an obtuse a triangle are concurrent.
Sol. Let the point of intersection O of two altitudes BQ and CR be
angle with each other.
taken as origin and the position vectors of the vertices A, B, C
Sol. Given, a = cx $i − 6 $j + 3 k$ and b = xi$ + 2 $j + 2cxk$ be a, b, c respectively. Let AO produced meet BC at P. We
Q a and b make an obtuse angle with each other. will show that AP is perpendicular to BC , showing there by
that the three altitudes are concurrent.
a.b
∴ cosθ = <0 A(a)
| a|| b
cx 2 − 12 + 6cx
i.e., <0
c x + 36 + 9 x 2 + 4 + 4c 2x 2
2 2
R Q

⇒ cx 2 + 6cx − 12 < 0 …(i) O (0)


Now, two cases are possible.
B(b) P C(c)
Case I c≠0
∴ OB = b, BQ = µb
⇒cx 2 + 6cx− 12 is a quadratic equation which has real solution
as its collinear with OB.
‘‘iff A < 0 and B 2 − 4 AC < 0’’
Similarly, since OC = C
i.e. if c < 0 and 36c 2 + 48c < 0
∴ CR = νC
i.e. if c < 0 and 12c(3c + 4 ) < 0
Now, AC = c − a and AB = b − a
⇒ 3c + 4 > 0 [Qc > 0]
Since, BQ ⊥ AC , we have µb . ( c − a ) and so a . b = b . c
4
⇒ − <c < 0 …(ii) Again, since CR ⊥ AB, νc . ( b − a ) = 0
3
Case II c=0 ∴ b. c = c.a
⇒ − 12 < 0 which is an identity. ∴ a. b = b. c = c.a
∴ c = 0 satisfy Eq. (i) …(iii) or a . (c − b) = 0
4 ⇒ λa . ( c − b ) = 0
∴ From Eqs. (ii) and (iii), we get − < c ≤ 0
3 ∴ AP . BC = 0 ⇒ AP ⊥ BC

l Ex. 131 A, B, C and D are four points in space. Using l Ex. 133 Using vector method, prove that the angle in a
vector methods, prove that AC 2 + BD 2 + AD 2 + BC 2 semi-circle is a right angle.
Sol. Take the centre O as origin and AB is the diameter, so that
≥ AB 2 + CD 2 what is the implication of the sign of equality. OA = OB.
Sol. Let the position vector of A, B, C and D be a, b, c and d, If the point A is a, then B is − a and | a| = r = radius.
respectively.
116 Textbook of Vector & 3D Geometry

P(r)
∴Using Eq. (i),
| OA| 2 + | OB| 2 + | OC | 2 + 2( OA. OB + OB . OC + OC . OA ) ≥ 0
B A(a) ⇒ 3 R 2 + 2 R 2(cos2 A + cos2 B + cos2C ) ≥ 0
(–a) O
3
⇒ cos2 A + cos2 B + cos2C ≥ −
2

Let P be any point r on the circumference, so that | r | = OP = r


l Ex. 136 Let β = 4 $i + 3 $j and γ be two vectors
Then, AP = Position vector of P − Position vector of A = r − a perpendicular to each other in the XY -plane. Find all the
and BP = Position vector of P − Position vector of B = r + a vectors in the same plane having the projections 1, 2 along β
∴ AP . BP = (r − a ) . (r + a ) = r 2 − a 2 = r 2 − r 2 = 0 and γ, respectively.
Sol. Here, β = 4 $i + 3 $j
Since, γ is perpendicular to β i.e. β . γ = 0
l Ex. 134 The corner P of the square OPQR is folded up so
∴We can choose γ = 3 λi$ − 4 λ$j for all values of λ.
that the plane OPQ is perpendicular to the plane OQR, find
Let the required vector be α = l $i + m $j.
the angle between OP and QR.
α .β
Sol. After folding OPQ, PS ⊥ SR. Now, projection of α along β =
|β |
P
a
P Q 4l + 3m
1= ⇒ 4l + 3m = 5 …(i)
5
α.γ
a a Similarly, projection of α along γ =
S Q Q | γ|
S
3 λl − 4 λm
⇒ 2=
O a R 5λ
R ⇒ 3l − 4m = 10 …(ii)
Here, SQ ⊥ SR, SQ ⊥ PS On solving Eqs. (i) and (ii), we get
a $ a $ a $ l = 2 and m = − 1
Let SR = j, SQ = i, SP = j
2 2 2 α = 2 i$ − $j
a $ a $ a $ $
OP = − SO + SP = j+ i= (j + i)
2 2 2 l Ex. 137 If a, b and c are three coplanar vectors. If a is
a $ $ not parallel to b, show that
QR = SR − SQ = (k − i )
2 c.a a.b a.a c.a
a a+ b
| OP | = 2 = a ⇒ | QR | = a c.b b.b a.b c.b
2 c=
a.a a.b
Cosine of angle between OP and
a.b b.b
OP ⋅ QR a 2 ( $i + $j) ⋅ ( k$ − $i )
QR = = Sol. Since, a, b and c are coplanar, we may write
| OP|| QR | 2 a2
1 1 2π c = λ 1a + λ 2b
⇒ cosθ = ( − 1 ) = − ⇒ θ =
2 2 3 ⇒ a . c = λ 1a . a + λ 2a . b …(i)
and b . c = λ 1b . a + λ 2b . b …(ii)
l Ex. 135 In a ∆ABC, prove by vector method that On solving Eqs. (i) and (ii), by Cramer’s rule, we find that
3
cos 2 A + cos 2B + cos 2C ≥ − a.c a.b a.a a.c
2 b.c b.b b.a b.c
Sol. As we know, ( OA + OB + OC ) 2 ≥ 0 …(i) λ1 = and λ 2 =
a.a a.b a.a a.b
and | OA| = | OB| = | OC | = R
2 2 2 2
…(ii) a.b b.b a.b b.b
A On substituting λ 1 and λ 2, we get
c.a a.b a.a c.a
O a+ b
c.b b.b a.b c.b
2A
c=
a.a a.b
B C a.b b.b
Chap 02 Product of Vectors 117

l Ex. 138 In ∆ ABC , D is the mid-point of side AB and E a| PA | 2 + b| PB| 2 + c| PC | 2


is the centroid of ∆ CDA. If OE ⋅ CD =0, where O is the = (a + b + c ) | PI | 2 + a| IA| 2 + b| IB| 2 + c| IC | 2
circumcentre of ∆ ABC, using vectors prove that AB = AC . + 2 PI ⋅ | aIA + bIB + cIC | 2
Sol. Let us take O to be the origin and position vector of the ⇒ a| PA| 2 + b| PB| 2 + c| PC | 2 = (a + b + c )
vertices A, B and C be a, b and c , respectively.
| PI | 2 + a | IA| 2 + b| IB| 2 + c | IC | 2
We have,
(Q a IA + b IB + c IC = 0) shown as, since D be point of
| a| = | b| = | c|
intersection of AI with side BC , we have BD : DC = c : b and
a+b
Now, D= (Q mid-point of AB) AI : ID = b + c : a
2
c IC + b IB
A(a) ⇒ ID = and a AI = ( b + c ) ID
b+c
a AI = c IC + b IB ⇒ a IA + b IB + c IC = 0
D
O (Origin)
l Ex. 140 If two circles intersect, prove by using vector
B(b) C(c) method, that the line joining their centres is perpendicular to
3a + b + 2 c their common chord.
E=
2 Sol. Let O be the centre of the first circle and C be the centre of
a + b a + b − 2c second. Let a and b be the radii of the two circles. Position
∴ CD = = vector of C is c and AB be point of intersection of two circles.
2 2
3a + b + 2 c A
and OE =
2
E
∴ OE ⋅ CD = 0 O C
1 B
⇒ (3a + b + 2 c ) ⋅ (a + b − 2 c ) = 0
4
If r is the position vector of A.
3 | a | 2 + | b | 2 − 4| c | 2 + 4a ⋅ b − 4a ⋅ c = 0 ⇒ CA = OA − OC = r − c …(i)
⇒ a⋅b =a⋅c (Q OA = r and OC = c)
⇒ 3 | a | 2 + | b | 2 − 4| c | 2 = 0 Also, r ⋅ r = a 2 and ( r − c ) ⋅ ( r − c ) = b 2 …(ii)
⇒ | a | + | b | − 2a ⋅ b = | a | + | c | − 2a ⋅ c
2 2 2
(Q| a| = | b | = | c |) Hence, at the point of intersection of two circle
1
⇒ |a − b | = |a − c |
2 2
a 2 − 2 r ⋅ c + | c | 2 = b 2 ⇒ r ⋅ c = [b 2 − a 2 − | c| 2 ]
2
⇒ | AB| 2 = | AC | 2 If E is the point of intersection of OC andAB, then
⇒ | AB | = | AC | OA = OE + EA = λc + k1AB
OB = OE + EB = λc + k2AB
l Ex. 139 Let I be the incentre of ∆ABC. Using vectors ⇒ 2 OA ⋅ c = 2 r ⋅ c = 2 [ λc + k1AB] ⋅ c = a 2 − b 2 + | c| 2
prove that for any point P a (PA ) 2 + b(PB ) 2 + c (PC ) 2
and 2 OB ⋅ c = 2 r ⋅ c = 2 { λc + k2AB} ⋅ c = a 2 − b 2 + | c| 2
= a (IA ) 2 + b(IB ) 2 + c (IC ) 2 + (a + b + c )(IP ) 2
⇒ 2[ λc − k1AB] ⋅ c = 2 [ λc + k2AB] c ⇒ AB ⋅ c = 0
where a , b and c have usual meanings.
Hence, AB is perpendicular to OC.
Sol. We have, IP + IA = PA
A
P
l Ex. 141 Using vector method prove that
cos( A − B ) = cos A cos B + sin A sin B.
l Sol. Let OX and OY be two lines perpendicular to each other and
∠POX = A, ∠QOX = B. So that, ∠ POQ = A − B shown as,

B D C y
P(r cos A, sin A)
⇒ | PA| = | PI | + | IA| + 2 PI ⋅ IA
2 2 2
r
a| PA| 2 = a| PI| 2 + a| IA| 2 + 2 PI ⋅ (aIA ) …(i) Q(r1 cos B, r1sin B)
Similarly, b | PB| = b | PI| + b| IB| + 2 PI ⋅ (b IB)
2 2 2
…(ii) A–B r1
A
c| PC | = c| PI| + c | IC | + 2 PI ⋅ (c IC )
2 2 2 B x
O
On adding Eqs. (i), (ii) and (iii), we get
118 Textbook of Vector & 3D Geometry

Let $i and $j denote unit vectors along OX and OY so that, l Ex. 143 If a, b, c and d are the position vector of the
i$ ⋅ $i = $j ⋅ $j = 1 and i$ ⋅ $j = $j ⋅ i$ = 0 vertices of a cyclic quadrilateral ABCD, prove that
Also, let OP = r and OQ = r1 a × b + b × d + d × a| | b × c + c × d + d × b|
∴P (r cos A, r sin A ) and Q (r1 cos B, r1 sin B ) + =0
(b − a ) ⋅ (d − a ) (b − c ) ⋅ (d ⋅ a )
∴ OP = (r cos A ) $i + (r sin A ) $j …(i) |a × b + b × d + d × a|
and OQ = (r1 cos B ) $i + (r1 sin B ) $j Sol. Consider,
(b − a) ⋅ (d − a)
By definition (a − d) × (b − a) |a − d| | b − a| sin A
OP ⋅ OQ = | OP| | OQ| cos ∠ POQ = r1r cos( A − B ) = =
(b − a) ⋅ (d − a) | b − c| | d − c| cos A
∴ OP ⋅ OQ = rr1 cos( A − B ) …(ii)
= tan A …(i)
Also, from Eq. (i)
D C
OP ⋅ OQ = rr1 cos A cos B + rr1 sin A sin B
= rr1(cos A cos B + sin A sin B ) …(iii)
From Eqs. (ii) and (iii), we get
rr1 cos( A − B ) = rr1(cos A cos B + sin A sin B )
A B
⇒ cos( A − B ) = cos A cos B + sin A sin B
| b × c + c × d + d × b| | (b − c) × (c − d)|
l Ex. 142 A circle is inscribed in an n-sided regular Again, =
(b − c) ⋅ (d − c) (b − c) ⋅ (d − c)
polygon A1 , A 2 K, An having each side a unit for any arbi-
| b − c| | c − d| sin C
trary point P on the circle, prove that = = tanC
| b − c| | d − c| cosC
n
a 2  1 + cos 2 π / n 
Σ (PAi ) 2 = n   As cyclic quadrilateral
i =1 4  sin 2 π / n 
A = 180 ° − C
Sol. Let the centre of the incircle be the reference point. ⇒ tan A = tan(180 ° − C )
Then, PAi = OAi − OP ⇒ tan A + tan C = 0
O |a × b + b × d + d × a| | b × c + c × d + d × b|
⇒ + =0
(b − a) ⋅ (d − a) (b − c) ⋅ (d − c)
O R p
R r nr
l Ex. 144 In ∆ ABC, points D, E and F are taken on the
Ai Ai+1 Ai a/2 Ai+1 sides BC , CA and AB , respectively such that
BD CE AF
PAi ⋅ PAi = ( OAi − OP ) ⋅ ( OAi − OP ) = = = n.
DC EA AB
( PAi ) 2 = (| OAi | ) 2 + (| OP| ) 2 − 2 OAi ⋅ OP
n 2 − n +1
n n Prove that, ∆DEF = ∆ ABC .
Σ (PAi )2 = iΣ= 1(| OAi | )2 + (| OP| 2 ) − 2OAi ⋅ OP
i =1
(n + 1) 2
n Sol. Take A is the origin and let the position vectors of the points
= nR 2 + nr 2 − 2OP ⋅ Σ OAi …(i) B and C be b and c, respectively.
i =1
C
= n( R + r ) − 2 OP ⋅ ( 0 )
2 2
n l
a π a π E D
Now, R = cosec ,r = cot …(ii)
2 n 2 n l n
2π π
2
a 
∴ R2 + r2 =  cosec + cot 2  A n F l B
4  n n
a 2  1 + cos2 π / n  ∴The position vector of D, E and F are
=   …(iii) nc + b c nb
4  sin 2 π / n  , ,
n+1 n+1 n+1
∴ From Eqs. (i) and (iii), we get
n c + b − n b n c + (1 − n ) b
n a 2  1 + cos2 π / n  FD = AD − AF = =
⇒ Σ ( PAi ) = n 
2
 n+1 n+1
i =1 4  sin 2 π / n 
nb − c
and EF = AF − AE =
n+1
Chap 02 Product of Vectors 119

1
Now, vector area of ∆ABC = ( b × c ) and vector area of∆DEF c  b λ
For the point E, we have b + λ 1  − b = + 3 c
2 2  2 2
1
= ( FD × FE ) 1  c
2 ⇒ b  − λ 1 + ( λ 1 − λ 3 ) = 0
2  2
1
= {(nb − c ) × nc + (1 − n ) b } 1
2 (n + 1 ) 2 ⇒ λ1 = λ 3 =
2
1
= {n 2b × c + (1 − n ) b × c } 2b + c
2 (n + 1 ) 2 ∴ AE =
4
1 3c + b − 2b − c 2c − b
= [(n 2 − n + 1 ) ( b × c )] Now, EA 2 = =
2 (n + 1 ) 2 4 7
n2 − n + 1 b + c 3c + b 5b + c
= ∆ ABC DA1 = − =
2 (n + 1 ) 2 2 7 14
1
n2 − n + 1 Area of quadrilateral EA1A2 D = | EA 2 × DA1|
∴ Area of ∆ DEF = area of ∆ ABC 2
2 (n + 1 ) 2 1
= |(2 c − b ) × (5 b + c )|
112
l Ex. 145 Let the area of a given ∆ ABC be ∆. Points 1
= |10 c × b − b × c |
A1 , B1 and C 1 are the mid-points of the sides BC , CA and AB, 112
respectively. Point A 2 is the mid-point of CA, lines C 1 A1 and 11 11 1 11
= | c × b | = ⋅ | c × b| =
A A 2 meet the median BB1 at E and D, respectively. If ∆ is 112 56 2 56
the area of the quadrilateral A1 A 2 DE , using vectors prove 11
Thus, required ratio is .
∆ 11 56
that 1 = .
∆ 56 l Ex. 146 Let ABC be an acute angled triangle with
Sol. Let the position verctor of A, B and C be a, b and c,
centriod G and the internal bisectors of angles A, B and C
respectively.
meets BC , CA and AB in M , N and K respectively using
b c
We have, AC1 = , AB1 = vectors, prove that if G lies on one of the sides of ∆ MNK ,
2 2
b+c 3c + b
then one of the altitudes of ∆ ABC equals the sum of other
AA 2 = , AA 2 = two.
2 4
Sol. Let G be on MK .
Equation of the lines BB1, AA2 and C1A1 are
Let the position vectors of B and C with reference to origin A
c 
r = b + λ 1  − b be b and c, respectively.
2 
∴ BC = a, CA = b andAB = c
3c + b b  c
r = λ2 and r = + λ 3  BM c
=
4 2  2 Now,
MC b
C (c) A(a)
A2
N
B1 A1 K
D
E G
A (0) C1 B (b)
B(b) D M C(c)
For the point D, we have bb + cc
∴ Position vector of M =
c   3c + b b+c
b + λ 1  − b = λ 2  
2   4  bb
Similarly, position vector of K =
 λ  c a+b
b 1 − λ 1 − 2  + (2 λ 1 − 3 λ 2 ) = 0
 4 4 bb bb + cc
MK = PV of K − PV of M = −
6 4 a+b b+c
⇒ λ1 = , λ 2 = b+c
7 7 PV of G =
3c + b 3
∴ AD = bb b+c
7 GK = PV of K − PV of G = −
a+b 3
120 Textbook of Vector & 3D Geometry

Since, G lies on MK , MK × GK = 0 Sol. Let A′ , B′ and C′ be the tops of the poles at A, B and C,
 bb bb + c c   bb b + c respectively. Through A′ draw a ∆ A′ B1C1 congruent to
⇒  −  × −  =0 ∆ ABC and parallel to the horizontal plane of the park. Take
a + b b + c  a + b 3 
A′ B1 as the X -axis and a line perpendicular to it as the Y -axis
bb b + c bb × c c (in the plane of ∆ A′ B1C1 ) and a line through A′ and perpen-
⇒ − × − dicular to the plane A′ B1C1 as the Z-axis.
a+b 3 a+c
bb bb + cc b + c Z
× + × =0
a+b b+c 3 C¢
b( b × c ) bc ( c × b ) b (b × c) b (c × b)
⇒ − − + + =0 B¢
3(a + b ) (b + c ) (a + b ) 3 (b + c ) 3
 b bc b c 
3 (a + b ) − (b + c ) (a + b ) + 3(b + c ) + 3 (b + c )  C1
  b c
a
X
(c × b) = 0 A¢ B1
b bc b c
⇒ − − + =0 If $i , $j and k$ are the unit vectors along these axes, then
3 (a + b ) (b + c ) (a + b ) 3(b + c ) 3 (b + c )
A′ B1 = c $i
⇒ b(b + c ) − 3bc − b (a + b ) + c (a + b ) = 0 A′ C1 = (b cos A ) $i + (b sin A ) $j
⇒ b 2 + bc − 3bc − ab − b 2 − ac + bc A′ B′ = c$i + y k$
⇒ ac = ab + bc A′ C′ = (b cos A ) $i + (b sin A ) $j + zk$
1 1 1 Since, the planes A′ B′ C ′ is inclined at an angle θ to the plane
⇒ = +
b c a A′ B1C1, angle between the normals to the planes is ( π − θ ).
2∆ 2∆ 2∆ Obviously, the unit vector normal to the plane A′ B1C1 is k and
⇒ = + (where, ∆ denotes area of ∆ ABC)
b c a the normal vector to A′ B′ C ′ is
⇒ Pb = Pa + Pc denotes the altitudes drawn through A, B and [(b cos A ) i$ + (b sin A ) $j + (z ) k$ ] × (c i$ + y k$ )
C, respectively. = (yb sin A ) i$ − (yb cos A − zc ) $j − (bc sin A ) k$
Aliter
cos( π − θ ) =
g = α m + (1 − α ) k
{(yb sin A ) $i − (yb cos A − zc ) $j − (bc sin A ) k$ } ⋅ k$
b + c α (b b + cc ) (1 − α ) b b
∴ = + y 2b 2 sin 2 A + y 2b 2 cos2 A + z 2c 2 − 2yzbc cos A + b 2c 2 sin 2 A
3 b+c a+b
sin A
On comparing coefficients of b and c, we get ⇒ cosθ =
y 2 z 2 2yz
1 αc + − cos A + sin 2 A
= c2 b2 bc
3 b+c
b+c y 2 z 2 2yz
⇒ α= + − cos A
3c ⇒ tan θ = c2 b2 bc
αb (1 − α ) b 1 sin A
and + = substituting α, we get
b+c a+b 3
lEx. 148 If a, b and c are three vectors such that a × b = c,
ca = ab + bc
b × c = a and c × a = b, then prove that | a | = | b | = | c |
1 1 1
⇒ = + Sol. Here, a×b = c (given)
b c a
⇒ (a × b ) ⋅ c = c ⋅ c
l Ex. 147 Three poles of height x , x + y and x + z are ⇒ [a b c ] = | c | 2 …(i)
Also, b×c =a (given)
posted at the vertices A, B and C of a triangular park of
(b × c) ⋅ a = a ⋅ a
sides a , b and c , respectively. A plane sheet is mounted at the
tops of the poles. If the plane of the sheet is inclined at an ⇒ [ b c a ] = | a| 2 …(ii)
angle θ to the horizontal plane, prove using vector and c ×a = b
 y 2 z 2 2yz  ⇒ (c × a ) ⋅ b = b ⋅ b (given)
 2 + 2 − cos A  ⇒ [ c a b] = | b|2 (given)
 c b bc 
θ = tan −1   Since, [a b c ]= [ b c a ] = [ c a b ] …(iii)
 sin A  ∴From Eqs. (i), (ii) and (iii), we get
 
  | c | 2 = | a | 2 =| b | 2 ⇒ | c | = | a | = | b |
Chap 02 Product of Vectors 121

Ex. 149 If a, b, c and d are four coplanar points, then R .(α × β ) [ Rαβ ]
l
⇒ k3 = =
|α × β |2 |α × β| 2
show that [ a b c ] = [b c d]+ [c a d] + [ a b d]
Sol. Since, a, b, c and d are coplanar points. On taking dot product of Eq. (i) with α × (α × β )
⇒ R .α × (α × β) = k2 (α × (α × β )).β
We have, b − a, c − a and d − a are coplanar.
⇒ [ b − ac − ad − a ] = 0 k2[(α . β )α − (α . α )β ].β = k2[(α . β ) 2 − α 2β 2 ]
⇒ {( b − a ) × ( c − a )} ⋅ ( d − a ) = 0 = − k 2| α × β | 2
⇒ (b × c) ⋅ d − (b × c) ⋅ a − (b × c) ⋅ a = 0 − [ R.(α × (α × β ))]
⇒ k2 =
− (a × c ) ⋅ d + (a × c ) ⋅ a = 0 |α × β |2
⇒ [ b c d ] − [ b c a ] − [ b a d ] − [a c d ] = 0 [ R.(β × (β × α ))]
Similarly, k1 = −
⇒ [a b c ] = [ b c d ] + [a b d ] + [ c a d ] |α × β |2
− [ R.(β × (β × α ))] α [ R.(α × (α × β ))] β
l Ex. 150 Let u$ and v$ be unit vectors. If w is a vector such ⇒ R= −
|α × β |2 |α × β |2
1
that w + ( w × u) = v, then prove that |(u × v ) ⋅ w|≤ and +
[ R.(α × β )](α × β )
2 |α × β |2
that the equality holds if and only if u is perpendicular to v.
[ R.(β × (β × α ))]α [ R.(α × (α × β ))] β
Sol. w + (w × u ) = v ⇒ R+ +
|α × β |2 |α × β |2
⇒ w×u =v −w …(i)
⇒ (w × u )2 = v 2 + w 2 − 2v ⋅ w [ R.(α × β )] (α × β )
=
|α × β |2
⇒ 2v ⋅ w = 1 + w 2 − (u × w )2 …(ii)
Also, taking dot product of Eq. (i) with v, we get
w ⋅ v + (w × u ) ⋅ v = v ⋅ v
l Ex. 152 If a, b and c represents the sides of tetrahedron
⇒ v ⋅ (w × u ) = 1 − w ⋅ v …(iii) (Q v ⋅ v = | v | 2 = 1) and θ be an angle between a and b, φ be an angle between a
1 and c, ψ be an angle between b and c, then prove that the
Now, v ⋅ ( w × u ) = 1 − [1 + w 2 − ( u × w ) 2 ]
2 volume of the tetrahedron is given by
[using Eqs. (ii) and (iii)] 1 cos θ cos θ
1 w 2 (u × w )2 a 2b 2c 2
= − + (Q 0 ≤ cos2 θ ≤ 1) v =
2
cos θ 1 cos ψ
2 2 2
36
1
= (1 − w 2 + w 2 sin 2 θ ) …(iv) cos φ cos ψ 1
2
Sol. OABC represent a tetrahedron, where
As we know, 0 ≤ w 2 cos2 θ ≤ w 2
OA = a,OB = b,OC = c relative to O
1 1 − w 2 cos2 θ 1 − w 2
∴ ≥ ≥ 1
2 2 2 Volume of tetrahedron (v ) = [ a ⋅ ( b × c) ]
1 − w cos θ 1
2 2 6
⇒ ≤ …(v)
2 2 A a O
1 q
From Eqs. (iv) and (v), we get | v ⋅ ( w × u ) | ≤ y
2 b c
π
Equality holds only when cos2 θ = 0 ⇒ θ =
2
i.e., u ⊥ w = 0 ⇒ u⋅w = 0
w + (w × u ) = v B C
u ⋅ w + u ⋅ (w × u ) = u ⋅ v a ⋅a a ⋅ b a ⋅ c
0 + 0 = u⋅v ⇒ u⋅v = 0 ⇒ u ⊥ v 1 1
Also, v2 = [ a ⋅ ( b × c) ]2 = b ⋅a b ⋅ b b ⋅ c
36 36
l Ex. 151 Prove that c ⋅a c ⋅ b c ⋅ c
[R.(β × (β × α )]α R.(α × (α × β )]β [R α β ](α × β ) a2 ab cos θ ac cos φ
R+ + =
| α × β| 2
| α × β| 2
|α × β | 2 1
= ab cosθ b2 bc cos ψ
36
Sol. α , β and α × β are three non-coplanar vectors. Any vector R ac cos φ bc cos ψ c2
can be represented as a linear combination of these vectors.
1 cosθ cos φ
⇒ R = k1α + k2β + k3(α × β ) …(i) 1 2 2 2
= a b c cosθ 1 cos ψ
⇒ R. (α × β ) = k3(α × β ).(α × β ) = k3(α × β ) 2 36
cos φ cos ψ 1
122 Textbook of Vector & 3D Geometry

l Ex. 153 A pyramid with vertex at the point P, whose π| a$ (b$ × c$ )|


=
position vector is 4 $i + 2 $j + 2 3 k$ has a regular hexagonal | Σ sin α cos β cos γ η $|
base ABCDEF. Position vectors of points A and B are $i and $ $ $ $
$1 = b × c , η $ 2 = c × a and η$3 = a ×b
$ $
$i + 2 $j, respectively. Centre of the base has the position where, η
| b$ × c$ | | c$ × a$ | | a$ × b$ |
vector $i + $j + 3k$ . Altitude drawn from P on the base meets
Sol. We know from sine rule,
the diagonal AD at point G. Find all possible position vectors
AB AC BC
of G. It is given that volume of the pyramid is 6 3 cu units. = =
sin C sin B sin A
Sol. Let the centre of base be (0). ( AB )( BC )(CA )
AB = 2 $j ⇒ | AB| = 2 = 2R = …(i)
2( ∆ABC )
1
∆OAB = ⋅ 4 3 = 3 BC = | BC | | = | c$ cos γ − b$ cosβ |
4
= |($a ⋅ b$ ) c$ ⋅ ($c ⋅ a$ ) b$ | = | a$ × ( b$ × c$ )|
P(4i+2j+2√2 k)
Similarly, AC = | AC | = | b$ × ($c × a$ )|
E D
and AB = | AB| = | c$ × ($a × b$ )|
(i+j+√3 k)
1
F
G
C Also, ∆ABC = | BC × BA|
2
O 1
= |($c cos γ − b$ cosβ ) × ($a cosα − b$ cosβ)|
B( i+2j )
2
A( i )
1
= |($c × a$ ) cosα cos γ + ( b$ × c$ ) cos γ cosβ
⇒ Base are a = 6 3 sq unit. 2
+ ($a × b$ ) cosβ cosα |
Let height of the pyramid be h.
1

1
⋅ 6 3h = 6 3 = |η $ 1 sin α cosβ cos γ + η $ 2 sin β cosα cos γ
3 2
+η $ 3 sin γ cosα cosβ|
⇒ h = 3 units ⇒ 2 ∆ABC = | Ση $ 1 sin α cosβ cos γ|
AP = 3 i$ + 2 $j + 2 3 k$ ∴Eq. (i) reduces to
⇒ | AP | = 9 + 4 + 12 = 5 units | a$ × ( b$ × c$ )| | b$ × ($c × a$ )| | c$ × ($a × b$ )|
= =
⇒ AP = 9 + 4 + 12 = 5 units sin A sin B sin C
π | a$ × ( b$ × c$ )|
⇒ AG = 25 − 9 = 4 units =
| Σ sin α cos β cos γ η| $
| AG | = 4 units
Now, AQ and AO are collinear. l Ex. 155 Let a and b be given non-zero and non-collinear
⇒ AG = λAO
vectors, such that c × a = b − c. Express c in terms of a, b and
⇒ | AG | = | λ || AO| a×b
⇒ 2| λ | = 4 Sol. Let c = x1a + x 2b + x 3(a × b )
⇒ | λ| = 2
⇒ c × a = x 2( b × a ) − x 3a × (a × b )
⇒ AG = ± ( $i + $j + 3 k$ )
= x 2( b × a ) − x 3(a ⋅ b )a + x 3| a| 2 b
⇒ G = ± 2( $i + $j + 3 k$ ) + $i
We have been given, c × a = b − c
= − ( $i + 2 $j + 2 3 k$ ), −3 $i + 2 $j + 2 3 k$ ⇒ b − x1a − x 2b − x 3(a × b ) = − x 2(a × b ) − x 3(a ⋅ b ) a + x 3| a | 2 b
⇒ x 3 {(a ⋅ b ) − x1 }a + (1 − x 2 − x 3| a | 2 ) b + ( x 2 − x 3 )(a × b ) = 0
l Ex. 154 Let a$ , b$ and c$ be the non-coplanar unit vectors.
Now, a, b and a × b are linearly independent.
The angle between b$ and c$ be α and angle between c$ and a$
be β and between a$ and b$ be γ. If A( a$ cos α, 0 ), B(b$ cos β, 0 ) Hence, x 3(a ⋅ b ) = x1, 1 = x 2 + x 3| a | 2 , x 2 = x 3
1 a⋅b
and C( c$ cos γ , 0 ), then show that in ∆ABC. x2 = x3 = and x1 =
1 + |a |2 1 + |a |2
| a$ × (b$ × c$ )| | b$ × (c$ × a$ )| | c$ × ( a$ × $b)|
= = a⋅b 1
sin A sin B sin C c= a+ [ b + (a × b )]
(1 + | a | 2 ) (1 + | a | 2 )
#L
Product of Vectors Exercise 1 :
Single Correct Type Questions
1. If a has magnitude 5 and points North-East and vector b 10. If a and b are two vectors, then (a × b ) 2 is equal to
has magnitude 5 and points North-West, then | a − b | is a ⋅ b a ⋅a a ⋅a a ⋅ b
equal to (a) (b)
b ⋅ b b ⋅a b ⋅a b ⋅ b
(a) 25 (b) 5
a⋅b
(c) 7 3 (d) 5 2 (c) (d) None of these
b ⋅a
2. If | a + b | > | a − b |, then the angle between a and b is
(a) acute (b) obtuse
11. The moment of the force F acting at a point P, about the
(c) π /2 (d) π point C is
(a) F × CP
3. If a , b and c are three vectors such that a = b + c and (b) CP ⋅ F
π (c) a vector having the same direction as F
the angle between b and c is , then
2 (d) CP × F
(a) a 2 = b 2 + c 2 (b) b 2 = c 2 + a 2
12. The moment of a force represented by F = i$ + 2$j + 3k$
(c) c 2 = a 2 + b 2 (d) 2a 2 − b 2 = c 2
about the point 2$i − $j + k$ is equal to
Note Here, a = | a |, b = | b | and c = | c |
(a) 5 $i − 5 $j + 5 k$ (b) 5 $i + 5 $j − 6 k$
4. If the angle between the vectors a and b be θ and
a ⋅ b = cos θ, then the true statement is (c) −5 $i − 5 $j + 5 k$ (d) −5 $i − 5 $j + 2 k$
(a) a and b are equal vectors 13. A force of magnitude 6 acts along the vector (9, 6, − 2)
(b) a and b are like vectors
and passes through a point A ( 4, − 1, − 7 ). Then moment
(c) a and b are unlike vectors
of force about the point O (1, − 3, 2) is
(d) a and b are unit vectors
150 $ 6
(a) (2 i − 3 $j) (b) (50 i$ − 75 $j + 36 k$ )
5. If the vector i$ + $j + k$ makes angles α, β and γ with 11 11
vectors $i, $j and k$ respectively, then (c) 150 (2 $i − 3 $j) (d) 6 (50 $i − 75 $j + 36 k$ )
(a) α = β ≠ γ (b) α = γ ≠ β 14. A force F = 2$i + $j − k$ acts at a point A, whose position
(c) β = γ ≠ α (d) α = β = γ vector is 2i$ − $j . The moment of F about the origin is
6. ( r ⋅ $i ) 2 + ( r ⋅ $j) 2 + ( r ⋅ k$ ) 2 is equal to (a) i$ + 2 $j − 4 k$ (b) i$ − 2 $j − 4 k$
(a) 3 r 2 (b) r 2 (c) $i + 2 $j + 4 k$ (d) $i − 2 $j + 4 k$
(c) 0 (d) None of these 15. If a, b and c are any three vectors and their inverse are
7. Let a and b be two unit vectors inclined at an angle θ, a − 1 , b − 1 and c − 1 and [abc] ≠ 0, then [a − 1 b − 1 c − 1 ] will be
then sin(θ / 2) is equal to (a) zero (b) one
1 1 (c) non-zero (d) [abc ]
(a) |a − b| (b) | a + b |
2 2
(c) | a − b| (d) | a + b|
16. If a, b and c are three non-coplanar vectors, that
a ⋅ b × c b ⋅a × c
8. If a = 4 $i + 6$j and b = 3$j + 4 k$ , then the component of a + is equal to
c × a ⋅ b c ⋅a × b
along b is (a) 0 (b) 2
(a)
18 $
(3 j + 4 k$ )
18
(b) (3 $j + 4 k$ ) (c) − 2 (d) None of these
10 3 25
17. a × ( b × c) is coplanar with
18 $
(c) (3 j + 4 k$ ) (d) (3 $j + 4 k$ ) (a) b and c (b) c and a
3
(c) a and b (d) a, b and c
9. If vector a = 2$i − 3$j + 6k$ and vector b = − 2$i + 2$j − k$ ,
18. If u = $i × (a × $i ) + $j × (a × $j) + k$ × (a × k$ ), then
Projection of vector a on vector b
then is equal to (a) u = 0
Projection of vector b on vector a
(b) u = $i + $j + k$
3 7
(a) (b) (c) u = 2a
7 3
(d) u = a
(c) 3 (d) 7
124 Textbook of Vector & 3D Geometry

19. If a = $i + 2$j − 2k$ , b = 2$i − $j + k$ and c = $i + 3$j − k$ , then 28. Given a parallelogram ABCD. If | AB | = a, | AD | = b and
a × ( b × c) is equal to | AC | = c, then DB ⋅ AB has the value
(a) 20 i$ − 3 $j + 7 k$ 3a 2 + b 2 − c 2
(a)
(b) 20 $i − 3 $j − 7 k$ 2
(c) 20 i$ + 3 $j − 7 k$ a 2 + 3b 2 − c 2
(b)
(d) None of the above 2
a 2 − b 2 + 3c 2
20. If a × ( b × c) = 0, then (c)
2
(a) | a | = | b | ⋅ | c | = 1 (b) b || c (d) None of the above
(c) a || b (d) b ⊥ c
29. For two particular vectors A and B, it is known that
21. A vector whose modulus is 51 and makes the same A × B = B × A. What must be true about the two
$i − 2$j + 2k$ −4 $i − 3k$ vectors?
angle with a = ,b= and c = $j , will
3 5 (a) Atleast one of the two vectors must be the zero vector
be (b) A × B = B × A is true for any two vectors
(a) 5 $i + 5 $j + k$ (c) One of the two vectors is a scalar multiple of the other
(b) 5 i$ + $j − 5 k$ vector
(d) The two vectors must be perpendicular to each other
(c) 5 $i + $j + 5 k$
(d) ± (5 i$ − $j − 5 k$ ) 30. For some non-zero vector V, if the sum of V and the
vector obtained from V by rotating it by ∠ 2α equals to
22. The horizontal force and the force inclined at an angle the vector obtained from V by rotating it by ∠α, then
60° with the vertical, whose resultant is in vertical the value of α, is
direction of P kg, are π π
(a) 2nπ ± (b) nπ ±
(a) P , 2 P (b) P , P 3 3 3
(c) 2 P , P 3 (d) None of these 2π 2π
(c) 2nπ ± (d) nπ ±
3 3
23. If x + y + z = 0, | x| = | y| = | z| = 2 and θ is angle between
31. In the isosceles ∆ABC, | AB | = | BC | = 8, a point E
y and z, then the value of cosec 2 θ + cot 2 θ is equal to
divides AB internally in the ratio 1 : 3, then the cosine of
(a) 4 / 3 (b) 5 / 3
the angle between CE and CA is (where, | CA | = 12)
(c) 1 / 3 (d) 1
3 7 3 8
(a) − (b)
24. The value of x for which the angle between the vectors 8 17
a = − 3i$ + x $j + k$ and b = x $i + 2x $j + k$ is acute and the 3 7 −3 8
(c) (d)
angle between b and X -axis lies between π /2 and π 8 17
satisfy 32. Given an equilateral ∆ABC with side length equal to a.
(a) x > 0 (b) x < 0 Let M and N be two points respectively, on the side AB
(c) x > 1 only (d) x < − 1 only AB
and AC such that AN = kAC and AM = . If BN and
25. If a, b and c are non-coplanar vectors and 3
d = λa + µb + v c, then λ is equal to CM are orthogonal, then the value of k is
[ dbc ] [ bcd ] 1 1
(a) (b) (a) (b)
[ bac ] [ bca ] 5 4
[ bdc ] [ cbd ] 1 1
(c) (d) (c) (d)
[abc ] [abc] 3 2
26. If the vectors 3p + q, 5p − 3q and 2p + q, 4 p − 2q are pairs 33. In a quadrilateral ABCD, AC is the bisector of the

of mutually perpendicular vectors, then sin ( pq ) is ( AB, AD) which is , 15 | AC | = 3 | AB | = 5 | AD |, then
(a) 55 / 4 (b) 55 / 8 3
(c) 3/16 (d) 247 / 16 cos( BA, CD) is equal to
14 21
27. Let u = $i + $j ,v = $i − $j and w = $i + 2$j + 3k$ . If n$ is a unit (a) − (b) −
7 2 7 3
vector such that u ⋅ n$ = 0 and v ⋅ n$ = 0, then | w ⋅ n$ | is 2 2 7
equal to (c) (d)
7 14
(a) 1 (b) 2 (c) 3 (d) 0
Chap 02 Product of Vectors 125

34. If the distance from the point P(1, 1, 1) to the line passing 41. Given unit vectors m, n and p such that angle between
through the points Q(0, 6, 8) and R( − 1, 4, 7 ) is expressed in π
m and n = Angle between p and ( m × n ) = , then
the form p / q , where p and q are co-prime, then the 6
( p + q ) ( p + q − 1) [npm ] is equal to
value of is equal to (a) 3 / 4 (b) 3/4
2
(a) 4950 (b) 5050 (c) 1/4 (d) None of these
(c) 5150 (d) None of these 42. If a and b are unit vectors, then the vector defined as
35. Given the vectors V = (a + b ) × (a + b ) is collinear to the vector
u = 2 $i − $j − k$ (a) a + b (b) b − a
v = $i − $j + 2 k$ (c) 2a − b (d) a + 2 b
w = $i − k$ 43. If a and b are orthogonal unit vectors, then for any
If the volume of the parallelopiped having − cu,v and cw non-zero vector r, the vector ( r × a ) is equal to
as concurrent edges, is 8, then c is equal to (a) [ r a$ b$ ] ($a + b$ )
(a) ± 2 (b) 4 (b) [ r a$ b$ ] a$ + (r ⋅ a$ )($a × b$ )
(c) 8 (d) Cannot be determined (c) [ r a$ b$ ] b$ + ( r ⋅ b$ )( b$ × a$ )
36. The vector c is perpendicular to the vectors a = (2, − 3, 1), (d) [ r a$ b$ ] b$ + ( r ⋅ a$ )($a × b$ )
b = (1, − 2, 3) and satisfies the condition 44. If vector $i + 2$j + 2k$ is rotated through an angle of 90°, so
c ⋅ ( $i + 2$j − 7 k$ ) = 10. Then, the vector c is equal to
as to cross the positive direction of Y -axis, then the
(a) (7, 5, 1) (b) ( − 7, − 5, − 1 ) vector in the new position is
(c) (1, 1, − 1 ) (d) None of these 2 $ 4 $ 2 $ 4 $
(a) − i + 5 $j − k (b) i − 5 $j + k
37. Let a = i$ + $j, b = $j + k$ and c = αa + βb. If the vectors, 5 5 5 5
$i − 2$j + k$ , 3$i + 2$j − k$ and c are coplanar, then α is (c) 4 $i − $j − k$ (d) None of these
β 45. 10 different vectors are lying on a plane out of which
equal to four are parallel with respect to each other. Probability
(a) 1 (b) 2 that three vectors chosen from them will satisfy the
(c) 3 (d) − 3 equation λ 1a + λ 2 b + λ 3 c = 0, where λ 1 , λ 2 and
38. A rigid body rotates about an axis through the origin λ 3 ≠ 0 is
with an angular velocity 10 3 rad/s. If ω points in the
6
C 2 × 4C1 ( 6C 3 × 4C1 ) + 6C 3
(a) 10
(b) 10
direction of $i + $j + k$ , then the equation to the locus of C3 C3
the points having tangential speed 20 m/s is ( 6C 2 × 4C1 ) + ( 4 C 3 ) ( 6C 2 + 4C1 ) + ( 6C 2 × 4C1 )
(c) 10
(d) 10
(a) x 2 + y 2 + z 2 − xy − yz − zx − 1 = 0 C3 C3
(b) x 2 + y 2 + z 2 − 2 xy − 2yz − 2zx − 1 = 0 46. If a$ is a unit vector and projection of x along a$ is 2 units
(c) x 2 + y 2 + z 2 − xy − yz − zx − 2 = 0 and (a$ × x ) + b = x, then x is equal to
(d) x + y + z − 2 xy − 2yz − 2zx − 2 = 0
2 2 2 1
(a) [$a − b + ($a × b )]
2
39. A rigid body rotates with constant angular velocity ω 1
(b) [2a$ − b + ($a × b )]
about the line whose vector equation is, 2
r = λ( $i + 2$j + 2k$ ). The speed of the particle at the instant (c) [$a + ($a × b )]
it passes through the point with position vector (d) None of the above
(2i$ + 3$j + 5k$ ) is equal to 47. If a ⋅ b and c are any three non-zero vectors, then the
(a) ω 2 (b) 2ω component of a × ( b × c) perpendicular to b is
(c) ω / 2 (a × b ) ⋅ ( c × a )
(d) None of these (a) a × ( b × c ) + b
| b| 2
40. Consider ∆ABC with A = (a ), B = ( b ) and C = ( c). If (a × c ) ⋅ (a × b )
b ⋅ (a + c) = b ⋅ b + a ⋅ c, | b − a | = 3 and | c − b | = 4, then the (b) a × ( b × c ) + b
| b| 2
angle between the medians AM and BD is (b × c) ⋅ (b × a )
 1   1  (c) a × ( b × c ) + b
(a) π − cos− 1   (b) π − cos− 1   | b| 2
 5 13   13 5 
(a × b ) ⋅ ( b × c )
 1   1  (d) a × ( b × c ) + b
(c) cos− 1   (d) cos− 1   | b| 2
 5 13   13 5 
126 Textbook of Vector & 3D Geometry

48. The position vector of a point P is r = x$i + y$j + zk$ , where 54. The ∆ABC is such that the mid-points of the sides
x , y, z ∈ N and a = $i + 2$j + k$ . If r ⋅ a = 20, then the BC , CA, AB are (l , 0, 0), (0, m, 0), (0, 0, n ) respectively. Then,
number of possible position of P is AB 2 + BC 2 + CA 2
is equal to
(a) 81 (b) 49 l 2 + m2 + n 2
(c) 100 (d) 36 (a) 2 (b) 4
$i 4 $j 1 (c) 8 (d) 16
49. Let a, b > 0 and α = + + bk$ and β = b$i + a$j + k$ ,
a b b 55. The angle between the lines whose direction cosines are
10 given by 2l − m + 2n = 0, lm + mn + nl = 0 is
then the maximum value of is π π
5 + α ⋅β (a) (b)
6 4
(a) 1 (b) 2
π π
(c) 4 (d) 8 (c) (d)
3 2
50. If a, b and c are any three vectors forming a linearly
56. A line makes an angle θ both with X andY -axes. A
independent system, then ∀ θ ∈ R possible range of θ is
  2π   π  π
a cos θ + b sin θ + c cos 2θ, a cos  3 + θ (a) 0,
 4 
(b) 0,
 2 

 2π   2π  π π  π π 
+ b sin  + θ + c cos 2 + θ , (c) ,
 4 2 
(d) ,
 6 3 
3  3 
 2π   2π   2π   57. Let a, b and c be the three vectors having magnitudes 1,
a cos θ −  + b sin θ −  + c cos 2θ −   equals
 3  3  3  5 and 3 respectively such that the angle between a and b
is θ and a × (a × b) = c, then tan θ is equal to
(a) [a b c] cosθ
2
(b) [a b c ] cos2θ (a) 0 (b)
3
(c) [a b c ] cos3θ
3 3
(d) None of the above (c) (d)
5 4
51. Two adjacent sides of a parallelogram ABCD are given
58. The perpendicular distance of a corner of a unit cube
by AB = 2$i + 10$j + 11k$ and AD = − $i + 2$j + 2k$ . The side
from a diagonal not passing through it is
AD is rotated by an acute angle α in the plane of 3 2
parallelogram so that AD becomes AD′. If AD′ makes a (a) (b)
2 3
right angle with the side AB, then the cosine of angle α 3 4
is given by (c) (d)
4 3
8 17
(a) (b)
9 9 59. If p, q are two non-collinear vectors such that
1 4 5 (b − c ) p × q + (c − a )p + (a − b )q = 0 where a, b, c are
(c) (d)
9 9 lengths of sides of a triangle, then the triangle is
e f 2e (a) right angled (b) obtuse angled
52. If in a ∆ABC, BC = − and AC = ; | e | ≠ | f |, then (c) equilateral (d) right angled isosceles triangle
| e| | f | | e|
the value of cos 2A + cos 2B + cos 2C must be 60. Let a = $i + $j + k$ , b = − $i + $j + k$ , c = $i − $j + k$ and
(a) −1 (b) 0 d = $i + $j − k$ . Then, the line of intersection of planes one
−3 determined by a, b and other determined by c, d is
(c) 2 (d)
2 perpendicular to
53. a, b, c are three unit of vectors, a and b are (a) X-axis (b) Y-axis
(c) both X-axis and Y-axis (d) both Y-axis and Z-axis
perpendicular to each other and vector c is equally
inclined to both a and b at an angle θ. If 61. A parallelopiped is formed by planes drawn parallel to
c = α a + β b + γ (a × b ), where α, β, γ are constants , then coordinate axes through the points A = (1, 2, 3) and
(a) α = β = − cosθ, γ 2 = cos2θ B = (9, 8, 5). The volume of that parallelopiped is equal to
(b) α = β = cosθ, γ 2 = cos2θ (in cubic units)
(a) 192 (b) 48
(c) α = β = cosθ, γ 2 = − cos2θ
(c) 32 (d) 96
(d) α = β = − cosθ, γ 2 = − cos2θ
Chap 02 Product of Vectors 127

62. Let a, b and c be three non-coplanar vectors and d be a 68. The length of the edge of the regular tetrahedron ABCD
non-zero vector, which is perpendicular to a + b + c. is ‘a’. Points E and F are taken on the edges AD and BD
Now, if d = (sin x ) (a × b) + (cos y ) ( b × c) + 2( c × a ), respectively such that ‘E’ divides DA and ‘F ’ divides BD
then minimum value of x 2 + y 2 is equal to in the ratio 2 : 1 each. Then, area of ∆CEF is
5a a
π2 (a) sq units (b) sq units
(a) π 2
(b) 12 3 12 3
2
a2 5a 2
π2 5π 2 (c) sq units (d) sq units
(c) (d) 12 3 12 3
4 4
69. If two adjacent sides of two rectangles are represented
63. If α (a × b) + β( b × c) + γ ( c × a) = 0, then
by the vectors p = 5a − 3b, q = − a − 2b and
(a) a, b, c are coplanar if all of α , β, γ ≠ 0 r = − 4a − b; s = − a + b respectively, then the angle
(b) a, b, c are coplanar if any one α , β, γ = 0 1 1
(c) a, b c are non-coplanar for any α , β, γ between the vectors x = ( p + r + s) and y = ( r + s)
3 5
(d) None of the above
 19   19 
(a) π − cos−1   (b) cos−1  
64. Let area of faces,  5 43   5 43 
∆OAB = λ 1 , ∆OAC = λ 2 , ∆OBC = λ 3 , ∆ABC = λ 4  19   19 
(c) − cos−1   (d) π – cos–1  
 5 43   43 
andh 1 , h 2 , h 3 , h 4 be perpendicular height from 0 to face
∆ABC , A to the face ∆OBC , B to the face ∆OAC , C to the 70. Let a, b, c are three vectors along the adjacent edges of a
face ∆OAB, then the face tetrahedron, if |a| =| b| =| c | = 2 and a ⋅ b = b ⋅ c = c ⋅ a = 2,
1 1 1 1 then volume of tetrahedron is
λ 1 h 4 ⋅ λ 2 h 3 + λ 3 h 2 + λ 4 h1 1 2
3 3 3 3 (a) (b)
2 1 2 3
(a) |[ AB AC OA]| (b) |[ AB AC OA]|
3 3 3 2 2
(c) (d)
1 2 3
(c) |[OA OB OC]| (d) None of these
3 71. The angle θ between two non-zero vectors a and b
65. Given four vectors a, b, c and d. The vectors a, b, c are satisfies the relation
coplanar but not collinear pair by pair and the vector d cos θ = (a × $i ) ⋅ ( b × $i ) + (a × $j) ⋅ ( b × $j) + (a × k$ ) ⋅ ( b × k$ ),
is not coplanar with the vecotrs a, b and c. If it is known
that the angle between a and b is equal to that between then the least value of | a| + | b| is equal to (where θ ≠ 90°)
b and c, each being equal to 60°. The angle between d 1
(a) (b) 2
and a is α and between d and b is β. Then, the angle 2
between the vectors d and c. (c) 2 (d) 4
−1
(a) cos (cosβ − cosα ) 72. If the angle between the vectors a = $i + (cos x )$j + k$ and
−1
(b) sin (cosβ − cosα ) b = (sin 2 x − sin x )$i − (cos x )$j + (3 − 4 sin x )k$ is obtuse
−1
(c) sin (sin β − sin α )  π
and x ∈ 0,  , then the exhaustive set of values of ‘x’ is
(d) cos−1(tan β − tan α )  2
66. The shortest distance between a diagonal of a unit cube equal to
and a diagonal of a face skew to it is  π  π π
(a) x ∈  0,  (b) x ∈  , 
1 1  6  6 2
(a) (b)
2 2  π π  π π
(c) x ∈  ,  (d) x ∈  , 
1 1  6 3  3 2
(c) (d)
3 6
73. If position vectors of the points A, B and C are a, b, c
67. Let V = 2i$ + $j + k$ and W = $i + 3k$ . If U is a unit vector, respectively and the points D and E divides line
then the maximum value of the scalar triple product segments AC and AB in the ratio 2 : 1 and 1 : 3 ,
[U V W ] is respectively. Then, point of intersection of BD and EC
(a) −1 (b) 35 divides EC in the ratio
(c) 59 (d) 60 (a) 2 : 1 (b) 1 : 3
(c) 1 : 2 (d) 3 : 2
128 Textbook of Vector & 3D Geometry

#L
Product of Vectors Exercise 2 :
More than One Option Correct Type Questions
a b
74. If vectors a and b are non-collinear, than + is 82. b and c are non-collinear if a × ( b × c) + (a ⋅ b ) b =
(a) a unit vector | a | | b|
( 4 − 2x − sin y ) b + ( x 2 − 1) c and ( c ⋅ c) a = c. Then,
(b) in the plane of a and b
(c) equally inclined to a and b (a) x = 1 (b) x = − 1
π π
(d) perpendicular to a × b (c) y = ( 4n + 1 ) , n ∈I (d) y = (2n + 1 ) , n∈I
2 2
75. If a × b ( b × c) = (a × b ) × c, then u v 2u
(a) ( c × a ) × b = 0 (b) c × (a × b ) = 0 83. If in triangle ABC, AB = − and AC = , where
|u| |v| |u|
(c) b × ( c × a ) = 0 (d) ( c × a ) × b = b × ( c × a ) = 0
| u | ≠ | v |, then
76. Let a and b be two non-collinear unit vectors. If (a) 1 + cos2 A + cos 2 B + cos2C = 0
u = a − (a . b ) b and v = a × b , then | v | is (b) sin A = cosC
(a) | u | (b) | u | + | u . a | (c) projection of AC on BC is equal to BC
(d) projection of AB on BC is equal to AB
(c) | u | + | u . b | (d) | u | + u .(a + b )
84. If a, b and c be three non-zero vectors satisfying the
77. The scalars l and m such l a + m b = c, where a, b and c condition a × b = c and b × c = a, then which of the
are give vectors, are equal to following always hold(s) good?
( c × b ).(a × b ) ( c × a ).( b × a )
(a) l = (b) l = (a) a, b and c are orthogonal in pairs.
(a × b ) 2 (b ×a )2 (b) [a b c ] = | b |
( c × a ).( b × a ) ( c × a ).( b × a ) (c) [a b c ] = | c | 2
(c) m = (d) m =
(b ×a )2 (b ×a )2 (d) | b | = | c |
78. Let r$ be a unit vector satisfying r$ × a = b, where | a | = 3 85. Given the following information about the non-zero
and | b | = 2. Then, vectors A, B and C
2
(a) r$ = (a + a × b )
1
(b) r$ = (a + a × b ) (i) ( A × B) × A = 0 (ii) B ⋅ B = 4
3 3 (iii) A ⋅ B = − 6 (iv) B ⋅ C = 6
2 1
(c) r$ = (a − a × b ) (d) r$ = ( −a + a × b ) Which one of the following holds good?
3 3
(a) A × B = 0 (b) A ⋅ ( B × C ) = 0 (c) A ⋅ A = 8 (d) A ⋅ C = − 9
79. a1 , a 2 , a 3 ∈ R − {0} and a1 + a 2 cos 2x + a 3 sin 2 x = 0 for 86. Let a, b and c are non-zero vectors such that they are not
all x ∈ R, then orthogonal pairwise and such that V1 = a × ( b × c) and
(a) vectors a = a1$i + a 2$j + a 3k$ and b = 4 $i + 2 $j + k$ are
V2 = (a × b ) × c, then which of the following hold(s)
perpendicular to each other good?
(b) vectors a = a1$i + a 2$j + a 3k$ and b = − $i + $j + 2 k$ are (a) a and b are orthogonal (b) a and c are collinear
perpendicular to each other (c) b and c are orthogonal (d) b = λ (a × c ) when λ is a scalar
(c) if vectors a = a1$i + a 2$j + a 3k$ is of length 6 units, then
87. Given three vectors
one of the ordered triplet (a1, a 2, a 3 ) = (1, − 1, − 2 ) U = 2 i$ + 3 $j − 6 k$ , V = 6 $i + 2 $j + 3 k$ and W = 3 i$ − 6 $j − 2 k$
(d) if vectors 2a1 + 3a 2 + 6a 3, then | a1i$ + a 2$j + a 3k$ | is 2 6
which of the following hold good for the vectors U,V and W?
80. If a and b are two vectors and angles between them is θ, (a) U, V and W are linearly dependent
then (b) ( U × V ) × W = 0
(a) | a × b | 2 + (a ⋅ b ) 2 = | a | 2 | b | 2 (c) U, V and W form a triplet of mutually perpendicular vectors
(b) | a × b | = (a ⋅ b ), if θ = π / 4 (d) ( U × ( V × W ) = 0
(c) a × b = (a ⋅ b ) n$ (where n$ is a normal unit vector), if θ = π / 4 88. Let a = 2$i − $j + k$ , b = $i + 2$j − k$ and c = $i + $j − 2k$ be
(d) | a × b | ⋅ (a + b ) = 0 three vectors. A vector in the plane of b and c whose
81. If unit vectors a and b are inclined at an angle 2θ such 2
projection on a is of magnitude is
that | a − b | < 1 and 0 ≤ θ < = π , then θ lies in the interval 3
(a) [ 0, π / 6 ) (b) (5 π / 6 π ] (a) 2 $i + 3 $j − 3 k$ (b) 2 $i + 3 $j + 3 k$
(c) [ π / 6, π / 2 ) (d) ( π / 2, 5 π / 6 ] (c) − 2 i$ − $j + 5 k$ (d) 2 i$ + $j + 5 k$
Chap 02 Product of Vectors 129

89. Three vectors a (| a| ≠ 0), b and c are such that 96. Let the unit vectors a and b be perpendicular and unit
1 vector c is inclined at angle α to a and b. If
a × b = 3a × c ⋅ Also, | a| = | b| = 1 and | c| = . If the angle
3 c = la + mb + n(a × b), then
between b and c is 60°, then (a) l = m (b) n 2 = 1 − 2l 2
(a) b = 3 c + a (b) b = 3 c − a 1 + cos2α
(c) n 2 = − cos2α (d) m 2 =
(c) a = 6 c + 2 b (d) a = 6 c − 2 b 2
90. Let a, b, c be non-zero vectors and | a| = 1 and r is a 97. If a, b, c are three non-zero vectors, then which of the
non-zero vector such that r × a = b and r ⋅ c = 1, then following statement(s) is/are true?
(a) a ⊥ b (b) r ⊥ b (a) a × ( b × c), b × ( c × a), c × (a × b ) form a right handed
1 − [a b c ] system
(c) r ⋅ a = (d) [ra b] = 0
a⋅b (b) c, (a × b) × c,a × b form a right handed system
(c) a ⋅ b + b ⋅ c + c ⋅ a < 0 if a + b + c = 0
91. If a and b are two unit vectors perpendicular to each (a × b) ⋅ ( b × c)
other and c = λ 1a + λ 2 b + λ 3 (a × b), then the following (d) = − 1 if a + b + c = 0
( b × c) ⋅ (a × c)
is (are) true
(a) λ 1 = a ⋅ c 98. Let a and b be two given perpendicular vectors, which
r r
(b) λ 2 = | b × a| are non-zero. A vector r satisfying the equation
(c) λ 3 = |(a × b) × c| r × b = a, can be ..............
(d) λ 1 + λ 2 + λ 3 = (a + b + a × b) ⋅ c a×b (a × b)
(a) b − (b) 2 b −
| b|2 | b|2
92. Given three non-coplanar vectors
OA = a, OB = b, OC = c. a×b (a × b)
(c) | a | b − (d) | b | b −
Let S be the centre of the sphere passing through the | b| 2 | b| 2
points, O, A, B, C if OS = x, then 99. If a and b are any two vectors, then possible integers(s)
(a) x must be linear combination of a, b, c 3 | a + b|
(b) x must be linear combination of b × c, c × a and a × b in the range of + 2 | a − b| is
2
a 2( b × c) + b 2( c × a) + c 2(a × b) (a) 2 (b) 3
(c) x = , a = | a|,b = | b|, c = | c|
2[a b c ] (c) 4 (d) 5
(d) x = a + b + c 100. If a is perpendicular to b and p is non-zero scalar such
93. If a = $i + $j + k$ and b = i$ − $j , then the vectors that p r + ( r ⋅ b ) a = c, then r
(a ⋅ $i )$i + (a ⋅ $j)$j + (a ⋅ k$ )k$ ,( b ⋅ $i )$i + ( b ⋅ $j)$j + ( b ⋅ $j)k$ and (a) [ r a c ] = 0
$i + $j − 2k$ (b) p 2r = pa − ( c ⋅ a)b
(c) p 2r = pb − (a ⋅ b)c
(a) are mutually perpendicular
(b) are coplanar (d) p 2r = pc − ( b ⋅ c )a
(c) form a parallelopiped of volume 6 units 101. In a four-dimensional space where unit vectors along
(d) form a parallelopiped of volume 3 units
axes are $i, $j, k$ and $l and a 1 , a 2 , a 3 , a 4 are four non-zero
94. If a = xi$ + y$j + zk$ , b = y$i + z$j + xk$ , c = z$i + x$j + yk$ , then vectors such that no vector can be expressed as linear
a × ( b × c) is combination of others and
(a) parallel to (y − z ) i$ + (z − x ) $j + ( x − y ) k$ ( λ − 1)(a 1 − a 2 ) + µ(a 2 + a 3 ) + γ (a 3 + a 4 − 2a 2 )
(b) orthogonal to $i + $j + k$ + a 3 + δa 4 = 0, then
2
(c) orthogonal to (y + z ) i$ + (z + x ) $j + ( x + y ) k$ (a) λ = 1 (b) µ = −
3
(d) parallel to i$ + $j + k$ 2 1
(c) λ = (d) δ =
3 3
95. If a, b, c are three non-zero vectors, then which of the
following statement (s) is/are true? 102. A vector ( d ) is equally inclined to three vectors
(a) a × ( b × c), b × ( c × a),c × (a × b) form a right handed a = $i − $j + k$ , b = 2$i + $j and c = 3$j − 2k$ . Let x, y, z be
system three vectors in the plane of a, b; b, c; c, a respectively,
(b) c,(a × b) × c, a × b form a right handed system then
(c) a ⋅ b + b ⋅ c + c ⋅ a < 0, if a + b + c = 0 (a) x ⋅ d = 14 (b) y ⋅ d = 3
(a × b) ⋅ ( b × c)
(d) = − 1, if a + b + c = 0 (c) z ⋅ d = 0
( b × c) ⋅ (a × c) (d) r ⋅ d = 0, where r = λx + µy + δz
130 Textbook of Vector & 3D Geometry

103. If a, b, c are non-zero, non-collinear vectors such that a 107. Let a, b, c be three vectors such that each of them are
vctors such that a vector p = ab cos (2π − (a, c) ) c and non-collinear, a + b and b + c are collinear with c and a
a q = ac cos ( π –(a c)) then b + q is respectively and a + b + c = k. Then, (| k|, | k| ) lies on
(a) parallel to a (b) perpendicular to a (a) y 2 = 4ax (b) x 2 + y 2 − ax − by = 0
(c) coplanar with b and c (d) coplanar with a and c (c) x 2 − y 2 = 1 (d) | x| + | y | = 1
104. Given three vevtors a, b, c such that they are non-zero ,
108. If a, b, c are non-coplanar unit vectors also b, c are
non-coplanar vectors, then which of the following are
non-collinear and 2a × ( b × c) = b + c, then
coplanar.
(a) angle between a and c is 60°
(a) a + b, b + c, c + a (b) a − b, b + c, c + a
(b) angle between b and c is 30°
(c) a + b, b − c, c + a (d) a + b, b + c, c − a
(c) angle between a and b is 120°
105. If r = $i + $j + λ (2$i + $j + 4 k$ ) and r ⋅ ( $i + 2$j − k$ ) = 3 are the (d) b is perpendicular to c
equations of a line and a plane respectively, then which 1 1
109. If a = (2$i + 3$j + 6k$ ); b = (6$i + 2$j − 3k$ );
of the following is incorrect? 7 7
(a) line is perpendicular to the plane 2 3 6
(b) line lies in the plane 7 7 7
(c) line is parallel to the plane but does not lie in the plane
6 2 −3
c = c 1 i$ + c 2 $j + c 3 k$ and matrix A =  
(d) line cuts the plane obliquely 7 7 7
106. If vectors a and b are two adjacent sides of a c 1 c 2 c3
 
parallelogram, then the vector representing the altitude
of the parallelogram which is perpendicular to a is and AA = I, then c
T

b ×a a⋅b 3 i$ + 6 $j + 2 k$ 1
(a) b + (b) b (a) (b) (3 $i − 6 $j + 2 k$ )
7 7
| a|2 | b|2
1 1
b ⋅a a × (b × a ) (c) ( −3 i + 6 $j − 2 k$ )
$ (d) − (3 $i + 6 $j + 2 k$ )
(c) b − 2 a (d) 7 7
|a | |a |2

#L
Product of Vectors Exercise 3 :
Statement I & II Type Questions
n
Directions (Q. Nos. 110 to 121) Each of these questions and a 3 $i + b 3 $j + c 3 k$ , may be mutually perpendicular
contains two statements. unit vectors.
Statement I (Assertion) and Statement II (Reason) Statement II Value of determinant and its transpose
Each of these questions also has four alternatives are the same.
choices, only one of which is the correct answer. You
have to select the correct choice, as given below. 112. Consider the vector a, b and c.
(a) Statement I is true, Statement II is true and Statement II is Statement I a × b = ( $i × b ).( b ) $i
a correct explanation for Statement I. +( $j × a ).( b $j × ( k$ × a ). b ) k$
(b) Statement I is true, Statement II is true but Statement II is
not a correct explanation for Statement I. Statement II c = ( $i . c)$i + ( $j . c)$j + ( k$ . c) k$
(c) Statement I is true, Statement II is false. 113. Statement I Distance of point D(1, 0, − 1) from the plane
(d) Statement I is false, Statement II is true. 8
of points A (1, − 2, 0), B (3, 1, 2) and C ( − 1, 1, − 1) is .
110. Statement I A component of vector b = 4 i$ + 2$j + 3k$ in 229
the direction perpendicular to the direction of vector
Statement II Volume of tetrahedron formed by the
a = $i + $j + k$ is $i − $j . 229
points A, B, C and D is .
Statement II A component of vector in the direction 2
a = $i + $j + k$ is 2$i + 2$j + 2k$ .
114. Statement I A = 2$i + 3$j + 6k, $ B = $i + $j − 2k$ and
111. Statement I a1 $i + a 2 $j + a 3 k$ , b1 $i + b 2 $j + b 3 k$ , and $ $ $
C = i + 2 j + k, then | A × ( A × ( A × B)). C | = 243
c 1 $i + c 2 $j + c 3 k$ are three mutually perpendicular
Statement II | A × ( A × ( A × B)). C | = | A | 2 |[ABC]|
unit vector, then a1 $i + b1 $j + c 1 k$ , a 2 $i + b 2 $j + c 2 k$ ,
Chap 02 Product of Vectors 131

115. Statement I The number of vectors of unit length and 118. Statement I If a = 3$i − 3$j + k$ , b = − $i + 2$j + k$ and
perpendicular to both the vectors $i + $j and $j + k$ is zero. c = $i + $j + k$ and d = 2$i − $j, then there exist real numbers
Statement II a and b are two non-zero and non-parallel α, β, γ such that a = αb + βc + γd
vectors it is true that a × b is perpendicular to the plane Statement II a, b, c, d are four vectors in a
containing a and b. 3-dimensional space. If b, c, d are non-coplanar, then
116. Statement I (S 1 ) : If A( x 1 , y 1 ), B( x 2 , y 2 ), C ( x 3 , y 3 ) are there exist real numbers α, β, γ such that
non-collinear points. Then, every point ( x , y ) in the a = αb + βc + γd.
plane of ∆ABC, can be expressed in the form 119. Statement I Let a, b, c and d are position vector four
 kx 1 + lx 2 + mx 3 ky 1 + ly 2 + my 3  points A, B, C and D and 3a − 2b + 5c − 6d = 0, then
 , 
 k +l +m k +l +m  points A, B, C and D are coplanar.
Statement II (S 2 ) The condition for coplanarity of four Statement II Three non-zero, linearly dependent
points A(a ), B( b ), C ( c), D( d) is that there exists scalars, coinitial vectors ( PQ, PR and PS) are coplanar.
l , m, n, p not all zeros such that 120. If a = $i + $j − k$ , b = 2$i + $j − 3k$ and r is a vector satisfying
la + mb + nc + pd = 0 2r + r × a = b.
where l + m + n + p = 0 Statement I r can be expressed in terms of a, b and
a × b.
117. If a, b are non-zero vectors such that | a + b| = | a − 2b|, 1
then Statement II r = (7 $i + 5$j − 9 k$ + a × b)
7
4 π
Statement I Least value of a ⋅ b + is 2 2 − 1 121. Let a$ and c$ be units vectors at an angle with each
3
| b| + 2 2

4 other. If (a$ × ( b$ × c))


$ ⋅ (a$ × c$ ) = 5 then
Statement II The expression a ⋅ b + 2 is least
| b| + 2 Statement I [a$ b$ c$ ] = 10
 π because
when magnitude of b is 2 tan  
 8 Statement II [x y z] = 0, if x = y or y = z or x = z

#L
Product of Vectors Exercise 4 :
Passage Based Questions
Passage I Passage II
(Q. Nos. 122-124) (Q. Nos. 125-127)
Consider three vectors p = $i + $j + k$ , q = 2$i + 4$j − k$ and Consider the three vectors p, q and r such that
r = $i + $j + 3k$ and let s be a unit vector, then p = $i + $j + k$ and q = $i − $j + k$ ; p × r = q + cp and p⋅ r = 2

122. p, q and r are 125. The value of [p q r] is


5 2c 8
(a) linearly dependent (a) − (b) −
(b) can form the sides of a possible triangle | r| 3
(c) such that the vectors ( q − r ) is orthogonal to p (c) 0 (d) greater than 0
(d) such that each one of these can be expressed as a linear 126. If x is a vector such that [p q r] x = ( p × q) × r, then x is
combination of the other two (a) c ( i$ − 2 $j + k$ ) (b) a unit vector
123. If ( p × q) × r = u p + v q + w r, then (u + v + w ) is (c) indeterminate, as [ p q r ] (d) − ( $i − 2 $j + k$ ) / 2
equal to
127. If y is a vector satisfying (1 + c ) y = p × ( q × r), then the
(a) 8 (b) 2
(c) − 2 (d) 4 vectors x, y and r
(a) are collinear (b) are coplanar
124. The magnitude of the vector (c) represent the coterminus edges of a tetrahedron whose
( p ⋅ s) ( q × r) + ( q ⋅ s) ( r × p) + ( r ⋅ s) ( p × q ) is volume is c cu units
(a) 4 (b) 8 (d) represent the coterminus edge of a paralloepiped whose
(c) 18 (d) 2
volume is c cu units
132 Textbook of Vector & 3D Geometry

Passage III a b c
(Q. Nos. 128-130) 134. The value of a . p b . p c . p is
Let P and Q are two points on the curve a.q b.q c.q
y = log 1/ 2 ( x − 0.5) + log 2 4x 2 − 4x + 1 (a) ( p × q ) [a × b b × c c × a ]
(b) 2( p × q ) [a × b b × c c × a ]
and P is also on the circle x + y = 10 , Q lies inside the given
2 2
(c) 4( p × q )[a × b b × c c × a ]
circle such that its abscissa is an integer.
(d) ( p × q ) |[a × b b × c c × a ]|
128. The coordinates of P are given by
(a) (1, 2) (b) (2, 4) (c) (3, 1) (d) (3, 5) Passage V
129. OP ⋅ OQ, O being the origin is (Q. Nos. 135-136)
x
(a) 4 or 7 (b) 4 or 2 (c) 2 or 3 (d) 7 or 8 Let g ( x ) = ∫ ( 3 t + 2t + 9) dt and f ( x ) be a decreasing
2
0
130. Max {| PQ | } is
function, ∀ x ≥ 0 such that AB = f ( x )$i + g ( x )$j and
(a) 1 (b) 4 (c) 0 (d) 2
AC = g ( x )$i + f ( x )$j are the two smallest sides of a ∆ABC
Passage IV whose circumcentre lies outside the triangle, ∀ x > 0.
(Q. Nos. 131 to 134)
135. Which of the following is true (for x > 0) ?
If a, b, c are three given non-coplanar vectors and any arbitrary
(a) f ( x ) > 0, g ( x ) < 0
vector r is in space, where (b) f ( x ) < 0, g ( x ) < 0
r ⋅ a b⋅ a c ⋅ a a⋅ a r ⋅ a c⋅ a (c) f ( x ) > 0, g ( x ) > 0
∆ 1 = r ⋅ b b⋅ b c ⋅ b ; ∆ 2 = a ⋅ b r ⋅ b c ⋅ b (d) f ( x ) < 0, g ( x ) > 0
f (x )⋅ g (x )
r ⋅ c b⋅ c c ⋅ c a⋅ c r ⋅ c c⋅ c  π 
136. lim lim  cot  (1 – t 2 )  is equal to
a ⋅ a b⋅ a r ⋅ a a ⋅ a b⋅ a c ⋅ a t→ 0 x →∞ 
 4  
∆ 3 = a ⋅ b b⋅ b r ⋅ b ; ∆ = a ⋅ b b⋅ b c ⋅ b (a) 0 (b) 1
a ⋅ c b⋅ c r ⋅ c a⋅ c c⋅ c c⋅ c (c) e (d) does not exist

131. The vector r is expressible in the form Passage VI


∆ ∆ ∆ (Q. Nos 137-139)
(a) r = 1 a + 2 b + 3 c
2∆ 2∆ 2∆ Let x , y , z be the vector , such that | x | = | y | = | z | = 2 and
2∆ 2∆ 2 2∆ 3
(b) r = 1 a + b+ c x , y , z make angles of 60° with each other also,
∆ ∆ ∆
∆ ∆ ∆ x × ( y × z) = a
(c) r = a+ b+ c
∆1 ∆2 ∆3 y × ( z × x ) = b and x × y = c, then
∆ ∆ ∆
(d) r = 1 a + 2 b + 3 c 137. The value of x is
∆ ∆ ∆
(a) (a + b) × c − (a + b)
132. The vector r is expressible as (b) (a + b) − (a + b) × c
[ r b c] [ rca ] [ ra b ] 1
(a) r = a+ b+ c (c) {(a + b) × c − (a + b )}
2 [a b c ] 2 [a b c ] 2 [a b c ] 2
2[ r b c] 2[ r c b] 2 [ ra b ] (d) None of the above
(b) r = a+ b+ c
[a b c ] [a b c ] [a b c ] 138. The value of y is
1
(c) r = ([ r b c ]a + [ r ca ] b + [ ra b ] c ) 1
(a) [(a + b) + (a + b) × c] (b) 2[(a + b) + (a + b) × c ]
[a b c ] 2
(d) None of the above (c) 4[(a + b) + (a + b) × c ] (d) None of these
133. If vector is expressible as r = x a + y b + g c, then 139. The value of z is
1 1
(a) a = [(a . a )( b × c ) + ( b . b )( c × a ) + ( c . c ) (a × b )] (a) [( b − a) × c + (a + b)]
[a b c ] 2
1 1
(b) a = [(a . a )( b × c ) + ( b . a )( c × a ) + (a . a ) (a × b )] (b) [( b − a) + (a + b) × c ]
[a b c ] 2
(c) ( b − a) × c + (a + b)
(c) a = (a . a )( b × c ) + (a . b )( c × a ) + ( c . a ) (a × b )
(d) None of the above
(d) None of the above
Chap 02 Product of Vectors 133

Passage VII q
141. + 2 cos θ is equal to
(Q. Nos. 140-142) p
a, b, c are non-zero unit vectors inclined pairwise with the (a) 1 (b) 2 [a b c]
same angle θ. p, q, r are non-zero scalars satisfying (c) 0 (d) None of these
a × b + b × c = pa + qb + rc. Now, answer the following 142. |( p + q ) cos θ + r | is equal to
questions.
(a) (1 + cosθ ) 1 − 2 cos θ
140. Volume of parallelopiped with edges a, b and c is equal θ
(b) 2 sin 2 | 1 + 2 cosθ |
to 2
(a) p + (q + r ) cosθ (b) ( p + q + r ) cosθ (c) (1 − sin θ )| 1 + 2 cosθ |
(c) 2p − (r + q ) cosθ (d) None of these (d) None of the above

#L
Product of Vectors Exercise 5 :
Matching Type Questions
143. Given two vectors a = $i + 2$j + 2k$ and b = $i + $j + 2k$ . 145. Match the statement of Column I with values of
Match the Column I with Column II and mark the Column II.
correct option from the codes given below. Column I Column II
A. Let O be an interior point of ∆ABC such p. 0
Column I Column II
that OA + 2OB + 3OC = 0, then the
A. A vector coplanar with a p. − 3$i + 3$j + 4 k$ ratio of the area of ∆ABC to the area of
and b ∆AOC, with O as the origin
B. A vector which is q. 2$i − 2$j + 3k$ B. A ⋅ B = A ⋅ C = 0, B ⋅ C = 3 / 2 q. 1
perpendicular to both a A ⋅ A = B ⋅ B = C ⋅ C = 1, [A B C] = 2
and b
C. If a, b, c and d are non-zero vectors such r. 2
C. A vector which is equally r. $i + $j that no three of them are in the same
inclined to a and b plane and no two are orthogonal, then
D. A vector which forms a s. $i − $j + 5k$ the value of the scalar
triangle with a and b ( b × c) ⋅ (a × d) + (c × a ) ⋅ ( b × d)
is
(a × b) ⋅ (d × c)
144. Volume of parallelepiped formed by vectors a × b, b × c
146. Match the statement of Column I with values of
and c × a is 36 sq units.
Column II.
Column I Column II Column I Column II
A. Volume of parallelopiped formed by p. 0 sq units A. | a | = | b| = 2, x = a + b, y = a − b p. 4
vectors a , b and c is
If | x × y | = 2 {λ − (a ⋅ b)2}1/ 2, then
B. Volume of tetrahedron formed by q. 12 sq units value of λ is
vectors a, b and c is
B. The non-zero value of λ for which q. 42
C. Volume of parallelopiped formed by r. 6 sq units angle between λ$i + $j + k$ and
vectors a + b, b + c and c + a is
$i + λ$j + k$ is π
D. Volume of parallelopiped formed by s. 1 sq units 3
vectors a − b, b − c and c − a is
134 Textbook of Vector & 3D Geometry

Column I Column II B. If a 1 , a 2 , a 3 are vectors reciprocal to q. 9


C. The non-zero value of k for which the r. 16 the non-coplanar vectors b1 , b2 , b3
lines kx − 4 y + 7z + 16 = 0 then [ a 1 a 2 a 3 ][ b1 b2 b3 ] is
= 4 x + 3 y − 2z + 3 and equal to
x − 3 y + 4z + 6 = 0 = x − y + z + 1 C. ABCD is a quadrilateral with AB = a, r. 8
are coplanar is AD = b and AC = 2a + 3 b . If its area
D. If | a | = | b| = 1 and | c | = 2 , then s. 7 is α times the area of the
maximum value of parallelogram with AB , AD as its
| a − 2 b|2 + | b − 2c|2 + | c − 2a |2 is adjacent sides, then α is equal to
t. 5 D. If d = x (a × b) + y ( b × c) + z (c × a ) s. 1
1
and [a b c] = , then
147. Match the statement of Column I with values of 8
Column II. x + y + z = R (a + b + c) ⋅ d, where
Column I Column II
R = adjacent sides, then α is equal to
A. Let a , b, c be the three vectors such p. 5
thata ⋅ ( b + c) = b⋅ (c + a ) = c ⋅ (a + b) 2
= 0 and | a | = 1, | b| = 4 , | c| = 8, then
| a + b + c| is equal to

#L
Product of Vectors Exercise 6 :
Integer Type Questions
148. Let u,
$ v$ and w
$ are three unit vectors, the angle between 154. Let A(2$i + 3$j + 5k$ ), B( − $i + 3$j + 2k$ ) and C( λ$i + 5$j + µk$ )
u$ and v$ is twice that of the angle between u$ and w
$ and are vertices of a triangle and its median through A is
v$ and w,
$ then [u$ v$ w
$ ] is equal. to equally inclined to the positive directions of the axes.
149. If a, b and c are three vectors such that [a b c] = 1, then The value of 2λ − µ is equal to
find the value of [a + b b + c c + a ] + [a × b b × c c × a ] 155. If V is the volume of the parallelopiped having three
+ [a + ( b × c) b × ( c × a ) c × (a × b )] . coterminous edges as a, b and c are the volume of the
150. If a,$ b$ and c$ are the three unit vectors and α, β and γ are parallelopiped having three coterminous edges as
α = (a ⋅ a)a + (a ⋅ b)b + (a ⋅ c)c,
scalars such that c$ = αa$ + βb$ + γ (a$ × b$ ). If is given that β = ( b ⋅ a)a + ( b ⋅ b )b + ( b ⋅ c)c and
a$ ⋅ b$ = 0 and c$ makes equal angle with both a$ and b, $ then γ = ( c ⋅ a )a + ( c ⋅ b )b + ( c ⋅ c)c is V λ , then λ =
evaluate α 2 + β 2 + γ 2 . 156. If a, b are vectors perpendicular to each other and
151. The three vectors i$ + $j, $j + k$ and k$ + i$ taken two at a time | a| = 2,| b| = 3, c × a = b, then the least value of 2| c − a| is
form three planes. If V be the volume of the tetrahedron 157. If M and N are the mid-point of the diagonals AC and
having adjacent sides as the three unit vectors drawn per- BD, respectively of a quadrilateral ABCD, then
pendicular to those three planes, then find the value of AB + AD + CB + CD = kMN , where k = ...... .
9 3V.
158. If a × b = c, b × c = a, c × a = b. If vectors a, b and c are
152. Let c$ be a unit vector coplanar with a = $i − $j + 2k$ and forming a right handed system, then the volume of
b = 2$i − $j + k$ such that c$ is perpendicular to a. If P be tetrahedron formed by vectors 3a − 2b + 2c, − a − 2c and
11 2a − 3b + 4 c is
the projection of c$ along b, where P = then find k. π
k 159. Let a and c be unit vectors inclined at with each other.
3
153. Let a, b and c are three vectors having magnitudes 1, 2
If (a × ( b × c)) ⋅ (a × c) = 5, then −[abc] − 1 =
and 3, respectively satisfy the relation [a b c] = 6. If d$ is a
160. Volume of parallelopiped formed by vectors a × b, b × c
unit vector coplanar with b and c such that b ⋅ d$ = 1, then
and c × a is 36 sq units, then the volume of the
evaluate | (a × c) ⋅ d$ | 2 + | (a × c) × d$ | 2 . parallelopiped formed by the vectors.
Chap 02 Product of Vectors 135

161. If α and β are two perpendicular unit vectors such that 163. Volume of a tetrahedron whose coterminous edges are
x = β$ − (α × x ); then the value of 4| x | 2 is. a, b and c is 3 and volume of a parallelopiped whose
coterminous edges are a + b − c, a − b, b − c is V. Then,
162. The volume of the tetrahedron whose vertices are the units digit of V is.
points with position vectors $i + $j + k$ ,
− $i − 3$j + 7 k$ , $i + 2$j − 7 k$ and 3$i − 4 $j + λk$ is 22, then the
digit at unit place of λ is.

Product of Vectors Exercise 7 :


Subjective Type Questions
164. Prove Cauchy-Schwartz inequality ( a . b) 2 ≤ | a | 2 ⋅ | b | 2 173. Two forces F1 = {2, 3} and F 2 = {4, 1} are
165. Two points P and Q are given in the rectangular specified relative to a general cartesian form. Their
points of application are respectively, A = (1, 1) and
cartesian coordinates in the curve y = 2 x + 2 , such that B = (2, 4 ). Find the coordinates of the resultant and the
OP . $i = − 1 and OQ ⋅ $i = 2, where $i is a unit vector along equation of the straight line l containing it.
the X -axis. Find the magnitude of OQ − 4OP. 174. A non-zero vector a is parallel to the line of intersection
166. O is the origin and A is a fixed point on the circle of of the plane determined by the vectors, $i, $i + $j and the
radius a with centre O. The vector OA is denoted by a. A plane determined by the vectors $i − $j and $i + k$ . Find the
variable point P lie on the tangent at A and OP = r.
Show that a . r = a 2 . Hence, if P( x , y ) and A( x 1 , y 1 ) angle between a and $i − 2$j + 2k$
deduce the equation of tangent at A to this circle. 175. The vector $i + 2j$ + 2 k$ turns through a right angle while
167. If a is real constant and A, B and C are variable angles passing through the positive X -axis on the way. Find the
and a − 4 tan A + a tan B + a + 4 tan C = 6a, then
2 2 vector in its new position.

find the least value of tan 2 A + tan 2 B + tan 2 C . 176. Let u$ and v$ are unit vectors and w is a vector such that
u$ × v$ + u$ = w and w × u$ = v$ , then find the value of
168. Given, the angles A, B and C of ∆ABC. Find cos ∠BAM, [u$ v$ w ].
where M is mid-point of BC.
177. A, B and C are three vectors given by 2$i + k$ , $i + $j + k$
169. Find the perpendicular distance of A (1, 4, − 2) from the and 4 $i − 3$j + 7 k$ Then, find R, which satisfies the
segment BC, where B (2, 1, − 2) and C (0, − 5, 1). relation R × B = C × B and R ⋅ A = 0.
170. Given angles, A, B and C of ∆ ABC. Let M be the 178. If x ⋅ a = 0, x ⋅ b = 1, [x a b ] = 1 and a ⋅ b ≠ 0, then find x in
mid-point of segment AB and let D be the foot of the terms of a and b.
Area of ∆ CDM 179. Let x$ , y$ and z be unit vectors such that x$ + y$ + z$ = a,
bisector of ∠C. Find the ratio of and 3 7
Area of ∆ ABC x$ × ( y$ × z$ ) = b,( x$ × y$ ) × z$ = c, a ⋅ x$ = , a ⋅ y$ = and
also cos φ = cos ∠DCM. 2 4
a = 2. Find x, y and z in terms of a, b and c.
171. In the ∆ABC a point P is taken on the side AB such that 180. Let a, b and c be three mutually perpendicular vectors of
AP : BP = 1 : 2 and a point Q is taken on the side BC such equal magnitude. If the vector x satisfies the equation.
that CQ : BQ = 2 : 1. If R be the point of intersection of
a × {( x − b ) × a } + b × {( x − c) × b } + c × {( x − a ) × c } = 0
lines AQ and CP, using vector find the area of ∆ ABC, if
it is known that area of ∆ABC is one unit. then find x.

172. If one diagonal of a quadrilateral bisects the other, then 181. Given vectors CB = a, CA = b and CO = x, where O is
it also bisects the quadrilateral. the centre of circle circumscribed about ∆ ABC, then
find vector x.
136 Textbook of Vector & 3D Geometry

#L
Product of Vectors Exercise 8 :
Questions Asked in Previous Years' Exam
(i) JEE Advanced & IIT-JEE
182. Let O be the origin and let PQR be an arbitrary triangle. 189. Let a = $i + $j + k$ , b = $i − $j + k$ and c = $i − $j − k$ be three
The point S is such that vectors. A vector v in the plane of a and b, whose
OP ⋅ OQ + OR ⋅ OS = OR ⋅ OP + OQ ⋅ OS 1
projection on c is , is given by
= OQ ⋅ OR + OP ⋅ OS 3 [Single Correct Type, 2011 Adv.]
Then the triangle PQR has S as its (a) $i − 3 $j + 3 k$ (b) − 3 i$ − 3 $j − k$
[Single Correct Type, 2017 Adv.] (c) 3 $i − $j + 3 k$ (d) i$ + 3 $j − 3 k$
(a) centroid (b) orthocentre
(c) incentre (d) circumcentre
190. Two adjacent sides of a parallelogram ABCD are given
by AB = 2$i + 10$j + 11k$ and AD = − $i + 2$j + 2k$ . The side
Passage AD is rotated by an acute angle α in the plane of the
(Q. Nos. 183-184) parallelogram so that AD becomes AD’. If AD’ makes a
Let O be the origin and OX , OY , OZ be three unit vectors in the right angle with the side AB, then the cosine of the angle
directions of the sides, QR, RP, PQ respectively of a ∆PQR. α is given by [Single Correct Type, 2010 Adv.]

[Passage Type Question, 2017 Adv.] 8 17


(a) (b)
9 9
183. If the triangle PQR varies, then the minimum value of 1 4 5
cos ( P + Q ) + cos (Q + R ) + cos( R + P ) is (c)
9
(d)
9
3 3 5 5
(a) − (b) (c) (d) −
2 2 3 3 191. Let P, Q, R and S be the points on the plane with position
184. | OX × OY | = vectors − 2$i − $j, 4 $i, 3$i + 3$j and − 3$i + 2$j, respectively. The
(a) sin ( P + Q ) (b) sin ( P + R ) quadrilateral PQRS must be a
[Single Correct Type, IITJEE 2010]
(c) sin (Q + R ) (d) sin 2R
(a) parallelogram, which is neither a rhombus nor a rectangle
185. Let a, b and c be three unit vectors such that (b) square
3
a × ( b × c) = ( b + c). If b is not parallel to c, then the (c) rectangle, but not a square
2 (d) rhombus, but not a square
angle between a and b is [Single Correct Type, 2016 Adv.] $i − 2$j
3π π 2π 5π 192. If a and b are vectors in space given by a = and
(a) (b) (c) (d)
4 2 3 6 5
2$i + $j + 3k$
186. Let a, b and c be three non-zero vectors such that no two b= , then the value of
1 14
of them are collinear and (a × b ) × c = | b | | c | a. If θ is ( 2a + b ) ⋅ [(a × b ) × (a − 2b )] is [Integer Type Question, 2010]
3
the angle between vectors b and c, then a value of sin θ 193. If a , b , c and d are the unit vectors such that
is 1
[Single Correct Type, 2015 Adv.] (a × b ) ⋅ ( c × d ) = 1 and a ⋅ c = , then
2 2 − 2 2 −2 3 2
(a) (b) (c) (d) [More than One Option Correct Type, 2009]
3 3 3 3
187. If a, b and c are unit vectors satisfying (a) a, b, c are non-coplanar
(b) a, b, d are non-coplanar
| a − b | + | b − c | + | c − a | = 9, then | 2a + 5b + 5c | is
2 2 2
(c) b, d are non-parallel
equal to [Subjective Type Question, 2012] (d) a, d are parallel and b, c are parallel
188. The vector(s) which is/are coplanar with vectors 194. The edges of a parallelopiped are of unit length and are
$i + $j + 2k$ and $i + 2$j + k$ , are perpendicular to the vector parallel to non-coplanar unit vector a, $ c$ such that
$ b,
$i + $j + k$ is/are 1
[More than One Option Correct Type, 2011] a$ ⋅ b$ = b$ ⋅ c$ = c$ ⋅ a$ = . Then, the volume of the
(a) $j − k$ (b) − $i + $j 2
(c) $i − $j (d) − $j + k$
parallelopiped is [Single Correct Type, IIT-JEE 2008]
Chap 02 Product of Vectors 137

(a)
1
cu unit (b)
1
cu unit 200. Let, a = $i + 2$j + k$ , b = i$ − $j + k$ , c = $i + $j − k$ .A vector
2 2 2
coplanar to a and b has a projection along c of
3 1 1
(c) cu unit (d) cu unit
2 3 magnitude , then the vector is
3
195. Let two non-collinear unit vectors a$ and b$ form an [Single Correct Type, IIT-JEE 2006]
acute angle. A point P moves, so that at any time t the (a) 4 i$ − $j + 4 k$ (b) 4 i$ + $j − 4 k$
position vector OP (where, O is the origin) is given by
(c) 2 i$ + $j + k$ (d) None of these
a$ cos t + b$ sin t . When P is farthest from origin O, let M
be the length of OP and u$ be the unit vector along OP. 201. If a, b, c are three non-zero, non-coplanar vectors and
Then, [Single Correct Type, IIT-JEE 2008] b ⋅a b ⋅a
b1 = b − 2 a, b 2 = b + a,
a$ + b$ | a| | a| 2
(a) u$ = and M = (1 + a$ ⋅ b$ )1/2
| a$ + b$ |
c ⋅a c⋅b c ⋅a c ⋅ b1
a$ − b$ c1 = c − 2 a − b, c 2 = c − 2 a − b1,
(b) u$ = and M = (1 + a$ ⋅ b$ )1/2 |a | | b | 2
| a | | b| 2
| a$ − b$ |
a$ + b$ c ⋅a a ⋅b2 c ⋅a
(c) u$ = and M = (1 + 2 a$ ⋅ b$ )1/2 c3 = c − 2 a − b 2 , c 4 = a − 2 a.
| a$ + b$ | | a| |b2 |
2
|a |
a$ − b$ Then, which of the following is a set of mutually
(d) u$ = and M = (1 + 2 a$ ⋅ b$ )1/2
| a$ − b$ | orthogonal vectors? [Single Correct Type, IIT-JEE 2005]
196. Let the vectors PQ, QR, RS, ST, TU and UP represent (a) {a, b1, c1 } (b) {a, b1, c 2 }
(c) {a, b 2, a 3 } (d) {a, b 2, c 4 }
the sides of a regular hexagon.
Statement I PQ × ( RS + ST) ≠ 0. because 202. The unit vector which is orthogonal to the vector
Statement II PQ × RS = 0 and PQ × ST ≠ 0 3$i + 2$j + 6 k$ and is coplanar with the vectors 2$i + $j + k$
[Single Correct Type, 2007, 3M] and $i − $j + k$ is [Single Correct Type, IIT-JEE 2004]
(a) Statement I is true, Statement II is true and Statement II is 2 i$ − 6 $j + k$ 2 i$ − 3 $j
a correct explanation for Statement I. (a) (b)
41 13
(b) Statement I is true, Statement II is true but Statement II is
^
not a correct explanation for Statement I. 3 $j − k$ 4 i$ + 3 j − 3 k$
(c) (d)
(c) Statement I is true, Statement II is false. 10 34
(d) Statement I is false, Statement II is true. 203. The value of a, so that the volume of parallelopiped
197. The number of distinct real values of λ, for which the formed by i$ + a $j + k$ , $j + a k$ and a i$ + k$ become
vectors − λ $i + $j + k$ , $i − λ 2 $j + k$ and $i + $j − λ 2 k$ are
2
minimum, is [Single Correct Type, IIT-JEE 2003]
coplanar, is [Single Correct Type, IIT-JEE 2007] (a) −3 (b) 3
(a) 0 (b) 1 (c) 1 / 3 (d) 3
(c) ± 2 (d) 3 204. If a = ( $i + $j + k$ ), a ⋅ b = 1 and a × b = $j − k$ , then b is equal
198. Let a, b, c be unit vectors such that a + b + c = 0. Which to [Single Correct Type, IIT-JEE 2003]
one of the following is correct? ^
[Single Correct Type, IIT-JEE 2007] (a) i$ − $j + k$ (b) 2 $j − k
(a) a × b = b × c = c × a = 0 (c) $i (d) 2 $i
(b) a × b = b × c = c × a ≠ 0 205. If V = 2$i + $j − k$ and W = i$ + 3k$ . If U is a unit vector, then
(c) b × b = b × c = a × c = 0
(d) a × b, b × c, c × a are mutually perpendicular the maximum value of the scalar triple product [U V W]
is [Single Correct Type, IIT-JEE 2002]
199. Let A be vector parallel to line of intersection of
(a) −1 (b) 10 + 6
planes P1 and P2 through origin. P1 is parallel to the
(c) 59 (d) 60
vectors 2$j + 3k$ and 4 $j − 3k$ and P2 is parallel to $j − k$ and
206. If a and b 1 are two unit vectors such that a + 2b and
3$i + 3$j , then the angle between vector A and 2$i + $j − 2k$
5a − 4 b, are perpendicular to each other, then the angle
is [More than One Option Correct Type, 2006, 5M]
between a and b is [Single Correct Type, 2002, 1M]
π π
(a) (b) (a) 45° (b) 60°
2 4 −1  1  2
π 3π (c) cos   (d) cos−1  
(c) (d)  3  7
6 4
138 Textbook of Vector & 3D Geometry

(ii) JEE Main & AIEEE


207. Let a = 2i$ + $j − 2k$ , b = $i + $j and c be a vector such that 215. If the vectors a = i$ − $j + 2 k$ , b = 2 i$ + 4 $j + k$ and
| c − a | = 3, | (a × b ) × c | = 3 and the angle between c and c = λi$ + $j + µk$ are mutually orthogonal, then ( λ , µ ) is
a × b is 30°. Then, a ⋅ c is equal to [JEE Main 2017] equal to [AIEEE 2010]
25 (a) (– 3, 2) (b) (2, – 3)
(a) (b) 2
8 (c) (– 2, 3) (d) (3, – 2)
1
(c) 5 (d) 216. If u, v and w are non-coplanar vectors and p , q are real
8
numbers, then the equality [3u pv pw ] − [pv w qu ]
208. If [a × bb × cc × a ] = λ [a b c]2 , then λ is equal to − [2w qv qu ] = 0 holds for [AIEEE 2009]
(a) 0 (b) 1 [JEE Main 2014] (a) exactly two values of ( p, q )
(c) 2 (d) 3 (b) more than two but not all values of ( p, q )
(c) all values of ( p, q )
209. Let a and b be two unit vectors. If the vectors c = a + 2b
(d) exactly one value of ( p, q )
and d = 5a − 4a are perpendicular to each other, then the
angle between a and b is [AIEEE 2012]
217. The vector a = α$i + 2$j + βk$ lies in the plane of the
π π vectors b = $i + $j and c = $j + k$ and bisects the angle
(a) (b)
6 2 between b and c. Then, which one of the following gives
π π possible values of α and β? [AIEEE 2008]
(c) (d)
3 4 (a) α = 1, β = 1 (b) α = 2 , β = 2
210. Let ABCD be a parallelogram such that AB = q , AD = p (c) α = 1, β = 2 (d) α = 2 , β = 1
and ∠BAD be an acute angle. If r is the vector that 218. If u and v are unit vectors andθ is the acute angle between
coincides with the altitude directed from the vertex B to them, then 2u × 3v is a unit vector for [AIEEE 2007]
the side AD, then r is given by [AIEEE 2012] (a) exactly two values of θ
3( p ⋅ q )  p ⋅ q (b) more than two values of θ
(a) r = 3 q + p (b) r = − q +   p (c) no value of θ
(p ⋅ p )  p ⋅ p (d) exactly one value of θ
 p ⋅ q 3( p ⋅ q )
(c) r = q −   p (d) r = − 3 q + p 219. Let a = $i + $j + k$ , b = i$ − $j + 2k$ and c = x$i + ( x − 2)$j − k$ . If
 p ⋅ p (p ⋅ p )
the vector c lies in the plane of a and b, then x equal to
1 1 (a) 0 (b) 1
211. If a = (3$i + k$ ) and b = (2$i + 3$j − 6k$ ), then the [AIEEE 2007]
10 7 (c) – 4 (d) –2
value of (2 − b ) ⋅ [(a × b ) × (a + 2b )] is [AIEEE 2011] 220. If (a × b ) × c = a × ( b × c), where a, b and c are any three
(a) −3 (b) 5 vectors such that a ⋅ b ≠ 0, b ⋅ c ≠ 0, then a and c are
(c) 3 (d) −5
π
212. The vectors a and b are not perpendicular and c and d (a) inclined at an angle of between them [AIEEE 2006]
(b) perpendicular 6
are two vectors satisfying b × c = b × d and a ⋅ b = 0. (c) parallel
Then, the vector d is equal to [AIEEE 2011] π
(d) inclined at an angle of between them
 a ⋅ c  b ⋅ c 3
(a) c +   b (b) b +   c
 a ⋅ b  a ⋅ b 221. The value of a , for which the points, A, B, C with
 a ⋅ c  b ⋅ c
(c) c −   b (d) b −   c position vectors 2$i − $j + k$ , $i − 3 $j − 5 k$ and a$i − 3$j + k$
 a ⋅ b  a ⋅ b
respectively are the vertices of a right angled triangle
213. If the vectors p$i + $j + k$ , $i + q $j + k$ and $i + $j + r k$ π
with C = are [AIEEE 2006]
( Where, p ≠ q ≠ r ≠ 1) are coplanar, then the value of 2
pqr − ( p + q + r ) is [AIEEE 2011] (a) –2 and –1 (b) –2 and 1
(a) − 2 (b) 2 (c) 2 and –1 (d) 2 and 1
(c) 0 (d) −1 222. The distance between the line
214. Let a = $j − k$ and a = $i − $j − k$ . Then, the vector r = 2$i − 2$j + 3k$ + λ ( $i − $j + 4 k$ ) and the plane
b satisfying a × b + c = 0 and a ⋅ b = 3, is [AIEEE 2010] r ⋅ ( $i + 5$j + k$ ) = 5 is [AIEEE 2005]
(a) − i$ + $j − 2 k$ (b) 2 $i − $j + 2 k$ 10 3 10 10
(a) (b) (c) (d)
(c) $i − $j − 2 k$ (d) $i + $j − 2 k$ 3 10 3 3 9
Chap 02 Product of Vectors 139

223. For any vector a, the value of 228. A particle is acted upon by constant forces 4 $i + $j − 3k$
(a × $i ) 2 + (a × $j) 2 + (a × k$ ) 2 is equal to [AIEEE 2005] and 3i$ + $j − k$ which displace it from a point i$ + 2$j + 3k$
to the point 5$i + 4 $j + k$ . The work done in standard units
2 2
(a) 4a (b) 2a
(c) a 2 (d) 3a 2 by the forces is given by [AIEEE 2004]
224. If a, b, c are non-coplanar vectors and λ is a real number, (a) 40 units (b) 30 units
then [λ (a + b ) λ 2 b λc] = [a b + c b ] for (c) 25 units (d) 15 units
[AIEEE 2005]
(a) exactly two values of λ 229. If u, v and w are three non-coplanar vectors, then
(b) exactly three values of λ ( u + v − w ) ⋅ [( u − v ) × ( v − w )] equal to [AIEEE 2003]
(c) no value of λ (a) 0 (b) u ⋅ v × w
(d) exactly one value of λ (c) u ⋅ w × v (d) 3u ⋅ v × w
225. Let a = i$ − k$ , b = x$i + $j + (1 − x ) k$ and 230. a, b, c are three vectors, such that a + b + c = 0, | a | = 1,
c = yi$ + x $j + (1 + x − y )k$ . Then, [a b c] depends on | b | = 2 , | c | = 3, then a ⋅ b + b ⋅ c + c ⋅ a is equal to
[AIEEE 2005] (a) 0 (b) –7 [AIEEE 2003]
(a) Neither x nor y (c) 7 (d) 1
(b) Both x and y 231. A tetrahedron has vertices at O(0, 0, 0), A(1, 2 , 1), B (2, 1, 3)
(c) Only x
and C( −1, 1, 2). Then, the angle between the faces
(d) Only y
OAB and ABC will be [AIEEE 2003]
226. Let u, v, w be such that | u | = 1, | v | = 2 ,| w | = 3. If the  19  17
(a) cos−1   (b) cos−1  
projection v along u is equal to that of w along u and  35  31
v, w are perpendicular to each other, then | u − v + w | (c) 30° (d) 90°
equal to [AIEEE 2004]
232. Let u = i$ + $j , v = $i − $j and w = $i + 2$j + 3k$ . If n is a unit
(a) 2 (b) 7
(c) 14 (d) 14 vector such that u ⋅ n = 0 and v ⋅ n = 0, then | w ⋅ n | is
equal to [AIEEE 2003]
227. Let a, b and c be non-zero vectors such that (a) 0 (b) 1
1 (c) 2 (d) 3
(a × b ) × c = | b || c | a. If θ is an acute angle between the
3 233. Given, two vectors are $i − $j and i$ + 2$j, the unit vector
vectors b and c, then sin θ is equal to [AIEEE 2004]
coplanar with the two vectors and perpendicular to
1 2 first is [AIEEE 2002]
(a) (b)
3 3 1 $ $ 1 $ $
2 2 2 (a) ( i + j) (b) (2 i + j)
(c) (d) 2 5
3 3 1 $ $
(c) ± ( i + j) (d) None of these
2
140 Textbook of Vector & 3D Geometry

Answers
Exercise for Session 1 67. (b) 68. (d) 69. (b) 70. (d) 71. (c) 72. (b)
π
1. cos−1  
5 73. (d) 74. (b,c,d) 75. (a,c,d) 76. (a,c) 77. (a,c) 78. (b,d)
2. 4. r = ± 3($i + $j + k
$)
 7 4 79. (a,b,c,d) 80. (a,b,c,d) 81. (a,b) 82. (a,c)
π 60 5 83. (a,b,c) 84. (a,c) 85. (a,b,c) 86. (b,d)
5. 6. 7. −
2 114 2 87. (b,c,d) 88. (a,c) 89. (a,b)
2π 90. (a,b,c) 91. (a,d) 92. (a,b,c) 93. (a,c) 94. (a,b,c)
8. 0° 9. 10. a > 2
3 95. (b,c,d) 96. (a,b,c,d) 97. (c,d) 98. (a,b,c,d)
1 $ $ $ 19 $ $ 1
11. ( i + j + k) 12. (2 i + j + 2k$ ); (−20$i + 8$j + 16k$ ) 99. (b,c,d) 100. (a,d) 101. (a,b,d)
6 9 9 102. (c,d) 103. (b,c) 104. (b,c,d) 105. (a,c,d)
13. 40
106. (c,d) 107. (a,b)
Exercise for Session 2 108. (a,c) 109. (b,c) 110. (c) 111. (a) 112. (a) 113. (d)
27 114. (d) 115. (d) 116. (a) 117. (a) 118. (b) 119. (a)
1. 19 2 2. λ = 3 and µ =
2 120. (a) 121. (b) 122. (c) 123. (b) 124. (a) 125. (b)
π 126. (d) 127. (c) 128. (c) 129. (a) 130. (d) 131. (d)
3. − 74 6. 3 7.
6 132. (d) 133. (c) 134. (b) 135. (d) 136. (a) 137. (a)
π − $i $j k
8. 9. ± 7 10. + + 138. (a) 139. (b) 140. (a) 141. (c) 142. (b)
4 3 3 3 143. (A) → (p,r), (B) → (q), (C) → (s), (D) → (p)
5 $ ) 12. 1
11. (7$i − 4$j − 4k (160 i − 4 i − 70k)
$ $ $ 144. (A) → (r), (B) → (s), (C) → (q), (D) → (p)
3 3
1 145. (A) → (s), (B) → (r), (C) → (q)
13. ± 2 (b × c) 14. 65 sq. units 146. (A) → (r), (B) → (p), (C) → (s), (D) → (q)
2
61 147. (A) → (q), (B) → (s), (C) → (p), (D) → (r)
15. sq. units. 17. $i + 2$j + 4k
$
2 148. (0) 149. (3) 150. (1) 151. (2) 152. (6) 153. (9)
18. 2 i − 7$j − 2k
$ $ 19. − 20$i + 16$j + 12k
$ 154. (2) 155. (3) 156. (3) 157. (4) 158. (2) 159. (9)
160. (5) 161. (2) 162. (3) 163. (8) 165. (10)
Exercise for Session 3 166. xx1 + 2 yy1 = a 167. (12)
9 sin C + sin B ⋅ sin A
1. 4 2. 4 cubic unit 3. 168.
2 sin B + sin 2 C + 2 sin B ⋅ sin C ⋅ cos A
2

2 38
4. 4 cubic unit 5. 3 26 sin A – sin B 49
19 169. 170. 171. sq units
7 2(sin A + sin B ) 28
6. a, b, c form a right handed system. 9. 6 10. 1
π
173. {6, 4} and 4x – 6 y + 13 = 0 174. θ =
Exercise for Session 4 4
 $ 1 $ 1 $
(−7$i + 8$j − k) 175.  2 2 i – j– k 176. 1
1. 0 2. 3  2 2 
114
a⋅ b
1 1 $ ) and c′ = 1 (i + j) 177. − i$ − 8$j + 2k$ 178.
7. a ′ = (− i + k), b ′ = (− $j + k (a ⋅ b)2 – a 2 b2
2 2 2
1 1 1 4
9. r = y b = (a × b ) 10. r = (a × c + mb ) 179. x$ = (3a + 4 b + 8c), y$ = − 4c, z$ = (c − b)
b2 a⋅b 3 3
a+ b+ c
180. x =
Chapter Exercises 2
1 a 2 b2 − b2 (a ⋅ b) 1 a 2 b2 − a 2 (a ⋅ b)
1. (d) 2. (a) 3. (a) 4. (d) 5. (d) 6. (b) 181. x = a+ ⋅b
7. (a) 8. (b) 9. (b) 10. (d) 11. (d) 12. (d) 2 2(a b ) – (a ⋅ b)
2 2 2
2 (a 2 b2 ) – (a ⋅ b)2
13. (a) 14. (c) 15. (c) 16. (a) 17. (a) 18. (c) 182. (b) 183. (a) 184. (a) 185. (d) 186. (a) 187. (3)
19. (a) 20. (b) 21. (d) 22. (c) 23. (b) 24. (b) 188. (a) 189. (c) 190. (b) 191. (a) 192. (5) 193. (c)
25. (b) 26. (b) 27. (c) 28. (a) 29. (c) 30. (a) 194. (a) 195. (a) 196. (c) 197. (c) 198. (b) 199. (b,d)
31. (c) 32. (a) 33. (c) 34. (a) 35. (a) 36. (b) 200. (a) 201. (b) 202. (c) 203. (c) 204. (c) 205. (c)
37. (d) 38. (c) 39. (a) 40. (a) 41. (a) 42. (b) 206. (b) 207. (b) 208. (b) 209. (c) 210. (b) 211. (d)
43. (c) 44. (a) 45. (d) 46. (b) 47. (d) 48. (a) 212. (c) 213. (a) 214. (d) 215. (a) 216. (d) 217. (d)
49.(a) 50. (d) 51. (b) 52. (a) 53. (c) 54. (c) 218. (d) 219. (d) 220. (c) 221. (d) 222. (c) 223. (b)
55. (d) 56. (c) 57. (d) 58. (b) 59. (c) 60. (d) 224. (c) 225. (a) 226. (c) 227. (d) 228. (a) 229. (b)
61. (d) 62. (d) 63. (a) 64. (a) 65. (a) 66. (a) 230. (b) 231. (a) 232. (d) 233. (a)
6

Solutions
13. OA = 3$i + 2$j − 9k$ , F = (9$i + 6$j − 2k$ ) ×
11
i$ $j k$
6
∴ Moment = OA × F = 3 2 −9
11
9 6 −2
1. Since, a ⊥ b ⇒ a ⋅ b = 0 6 150 $
| a − b | 2 = (a − b ) 2 = a 2 + b 2 − 2 a ⋅ b = 25 + 25 = (50 i$ − 75 $j) = (2 i − 3 $j)
11 11
⇒ |a − b| =5 2
14. Force ( F ) = 2 $i + $j − k$ and its position vector = 2 i$ − $j. We
2. | a + b | > | a − b | know that the position vector of a force about origin
( r ) = (2 $i − $j) − ( 0 $i + 0 $j + 0 k$ ) or r = 2 i$ − $j Therefore, moment
On squaring both sides, we get
of the force about origin
a 2 + b2 + 2a ⋅ b > a 2 + b2 − 2a ⋅ b
$i $j k$
⇒ 4a ⋅ b > 0 ⇒ cosθ > 0
= r × F = 2 − 1 0 = $i + 2 $j + 4 k$ .
Hence, θ < 90 ° (acute)
π 2 1 −1
3. Given that, a = b + c and angle between b and c is .
2 b × c −1 a × b −1 c × a
So, a 2 = b 2 + c 2 + 2b ⋅ c 15. a −1 = ,c = ,b =
[abc] [abc ] [abc]
π
⇒ a 2 = b 2 + c 2 + 2| b | | c | cos ( b × c )  ( c × a ) (a × b )
⇒ a =b +c +0
2 2 2 2 ⇒ [a −1 b −1 c −1 ] = ⋅ × 
[abc]  [abc] [abc] 
∴ a 2 = b2 + c2 b×c a 1
= ⋅ = ≠0
i.e., a 2 = b2 + c2 [abc ] [abc] [abc ]
4. Obviously, a and b are unit vectors. a ⋅ b × c b ⋅ a × c [abc ] [ bac ]
16. + = +
c × a ⋅ b c ⋅ a × b [ cab ] [ cab ]
5. Angle between $i + $j + k$ and $i is equal to
[abc ] [abc ]
 ( $i + $j + k$ ) ⋅ $i  = − =0
− 1 1 
cos− 1   ⇒ a = cos   [ cab ] [ cab ]
| $i + $j + k| | $i | 
$ 3
17. b × c is a vector perpendicular to b, c. Therefore, a ×(b × c ) is a
Similarly, angle between $i + $j + k$ and $j is vector again in plane of b, c.
− 1
1  18. Let a = x $i + y $j + z k$
β = cos   and between $i + $j + k$ and k$ is
 3
u = $i × (a × $i ) + $j × (a × $i ) + k$ × (a × k$ )
− 1
1 
γ = cos   = ( $i ⋅ $i )a − i$(a ⋅ $i ) + ( $j ⋅ $j)a − $j(a ⋅ $j)
 3
+ ( k$ ⋅ k$ )a − k$ (a ⋅ k$ ) = 3a − a = 2a
Hence, a =β = γ
$i $j k$
6. Let r = xi + y$j + z$i ⇒ r ⋅ $i = x, r ⋅ $j = y, r ⋅ k$ = z
$
19. a × ( b × c ) = a × 2 − 1 1 = a × ( − 2 $i + 3 $j + 7 k$ )
⇒ ( r ⋅ $i ) 2 + ( r ⋅ $j) 2 + ( r ⋅ k$ ) 2 = x 2 + y 2 + z 2 = r 2
1 3 −1
7. | a − b | = 12 + 12 − 2 ⋅ 12 cosθ = 2(1 − cosθ )
i$ $j k$
θ θ θ |a − b|
= 2 × 2 sin = 2 sin ⇒ sin = = 1 2 − 2 = 20 $i − 3 $j + 7 k$
2 2 2 2
−2 3 7
8. The component of vector a along b is
(a ⋅ b ) b 18 $ 20. a × (b × c ) = 0
= (3 j + 4 k$ )
| b |2 25 ⇒ a || ( b × c ) or b × c = 0
b ⋅a a⋅b | a | 7 i.e., b || c or a = 0
9. Required value = = =
| b| |a| | b| 3 21. Let the required vector be α = d1$i + d 2$j + d 3k$
a ⋅a a ⋅ b where, d12 + d 22 + d 32 = 51 (given) …(i)
10. (a × b ) 2 = a 2b 2 − (a . b ) 2 =
b ⋅a b ⋅ b
Now, each of the given vectors a, b and c is a unit vectors.
11. Torque = r × F or CP × F d ⋅a d⋅b d ⋅c
cosθ = = =
| d | | a | | d | | b | | d| | c |
$i $j k$
12. r × F = 2 − 1 1 = − 5$i − 5$j + 5k$ or d ⋅a = d ⋅ b = d ⋅ c
1 2 3 | d | = 51 cancels out and | a | = | b | = | c | = 1
1 1
Hence, (d1 − 2d 2 + 2d 3 ) = ( − 4d1 + 0d 2 − 3d 3 ) = d 2
3 5
142 Textbook of Vector & 3D Geometry

⇒ d1 − 5d 2 + 2d 3 = 0 where, a12 + a 22 + a 32 = 1
and 4d1 + 5d 2 + 3d 3 = 0 u ⋅ n$ = 0 ⇒ a1 + a 2 = 0
d1 d 2 d v ⋅ n$ = 0 ⇒ a1 − a 2 = 0
On solving, we get = = 3 =λ (say) Also,
5 −1 −5 Hence, a1 = a 2 = 0
Putting d1, d 2 and d 3 in Eq. (i), we get λ = ± 1 ∴ a 3 = 1 or − 1
Hence, the required vectors are ± (5 i$ − $j − 5 k$ ) ∴ n$ = k$ or − k$
22. Let OA = P1$i, CB = − P1i$ and OB = − P1$i + P $j | w ⋅ n$ | = 3
OB ⋅ $j Y 28. To find (a − b ) ⋅ a D(b) C(c)
= cos60 °
OB i.e., | a |2 − a ⋅ b …(i)
( − P1i$ + P $j) ⋅ $j 1
⇒ = Now, a+b=c
B C
P12 + P 2 2
⇒ | a | + | b | 2 + 2a ⋅ b = | c| 2 …(ii) A(origin)
2
B(a)
⇒ 2 P = P 2 + P12 60°
A On substituting the value of a ⋅ b from Eq. (ii) in Eq. (i), we get
X
⇒ P1 = P 3 O 1
a 2 − (c 2 − a 2 − b 2 )
| OB | = P 2 + P12 = P 2 + 3 P 2 = 2 P 2
3a 2 + b 2 − c 2
23. x + y + z = 0 ⇒ x = − (y + z ) ⇒
2
| x | 2 = (y + z ) ⋅ (y + z ) C
29. A × B = − A × B B
⇒ | x | 2 = | y | 2 + | z | 2 + 2y ⋅ z
A ×B=0
⇒ | x | 2 = | y | 2 + | z | 2 + 2 | y | | z | cosθ either A=0
v1 v2
⇒ 4 = 4 + 4 + 2 × 2 × 2 cosθ or B=0
−1 or A and B are collinear
⇒ cosθ = ⇒ θ = 120 ° 2a
a
2 30. Given, V + V1 = V2
O A
∴ cosec 2120 ° + cot 2 120 ° Also, V ⋅ V1 = 2α
 2 
2
 1  4 1 5
2
( V1 ) 2 = ( V2 − V ) 2 and V ⋅ V2 = α
=   + −  = + =
 3  3 3 3 3 Also, | V | = | V1 | = | V2 | = λ (say)
24. For acute angle a ⋅ b > 0 Hence, λ2 = 2 λ2 − 2 λ2 cos α
1
i.e., − 3 x + 2 x 2 + 1 > 0 ⇒ cos α =
2
⇒ ( x − 1 ) (2 x − 1 ) > 0
For obtuse angle between b and X -axis b . i$ < 0 ⇒ x < 0 31. Given, | b | = | b − c | = 8 and | c | = 12 …(i)
b b
25. Since, d = λa + µb + vc AE = and EC = c −
4 4
∴ d ⋅ ( b × c ) = λa ⋅ (a × c ) + µb ⋅( b × c ) + νc ⋅( b × c )
= λ [a b c ] B
[ dbc ] [ bcd ]
⇒ λ= =
[abc ] [ bca ]
b
26. (3p + q ) ⋅(5p − 3q ) = 0 c–(b/4)
E
or 15 p 2 − 3 q 2 = 4 p ⋅ q …(i)
q
(2 p + q ) ⋅( 4 p − 2 q ) = 0 or 8 p 2 = 2 q 2 A c C
(Origin)
⇒ q = 4p
2 2
…(ii)
p ⋅q  b c⋅b
Now, cos θ = c ⋅ c −  c2 −
| p|| q|  4 4
cos θ = = …(ii)
On substituting q 2 = 4 p 2 in Eq. (i), we get b b
| c| c − 12 c −
4 4
⇒ 3p 2 = 4p ⋅ q
From Eq. (i), | b | = 8, | c | = 12
3 p2 3 55
cos θ = ⋅ = ⇒ sin θ = ∴ | b − c |2 = | b |2
4 | p|2| p| 8 8
⇒ | b | 2 + | c | 2 − 2b ⋅ c = | b | 2
27. n$ = a1$i + a 2$j + a 3k$ ,
Chap 02 Product of Vectors 143

⇒ b ⋅ c = 72 …(iii) P
2
b 2
| b| b⋅c
and c− = | c |2 + −
4 | 16 | 2 b d
= 144 + 4 − 36 = 112
b
∴ c− =4 7 …(iv)
4
Q(0, 6, 8) a R(–1, 4, 7)
From Eqs. (ii), (iii) and (iv)
144 − 10 3 7 | a × b | = a b − (a ⋅ b ) = (6 ) (75 )
2 2 2 2

cos θ = =
12 × 4 7 8 − ( − 1 + 10 + 7 ) 2 = 450 − 256 = 194
32. BN ⋅ CM = 0 | a × b | = 194
b  ∴ d =
194
=
97
(kc − b ) ⋅  − c = 0 A (origin)
3  6 3
k a2 a2 a2 M (b/3) N (kc) ∴ p + q = 100
⋅ − ka 2 − + =0 ( p + q ) ( p + q − 1 ) 100 × 99
3 2 3 2 ⇒ = = 4950
k 1 90° 2 2
−k + = 0
6 6 2 −1 −1
5k 1 1 B (b) C (c ) 35. V = − c 2 [u v w] = − c 2 1 − 1 2
= ⇒ k=
6 6 5 1 0 −1
33. Given, 15 | AC | = 3 | AB | = 5 | AD | D(d) = − c 2[2(1 − 0 ) − 1(1 ) + ( − 2 − 1 )]
C(c)
Let | AC | = λ > 0 = − c 2[2 − 1 − 3 ] = 8
∴ | AB | = 5λ ∴ 2c 2 = 8 ⇒ c = 2 or − 2
p/3
| AD | = 3λ
p/3
BA ⋅ CD 36. Let c = λ(a × b )
Now, cos ( BA ⋅ CD) = A(a) origin B(b)
| BA | | CD | Hence, λ(a × b ) ⋅ ( i$ + 2 $j − 7 k$ ) = 10
b ⋅ (d − c) 2 −3 1
= …(i)
| b|| d− c| λ 1 − 2 3 = 10
Now, numerator of Eq. (i), we get 1 2 −7
π 2π ⇒ λ = − 1 ⇒ c = − (a × b )
b ⋅ c − b ⋅ d = | b | | c | cos − | b | | d | cos
3 3
a = 2 i$ − 3 $j + k$ and b = i$ − 2 $j + 3 k$
1 1
= (5 λ) ( λ) + 5 λ(3λ) $i $j k$
2 2
5 λ + 15 λ
2 2 2 − 3 1 = ( − 9 + 2 ) $i − (5 ) $j + ( − 4 + 3 ) k$
= = 10 λ2
2 1 −2 3
Denominator of Eq. (i) ⇒ ( − 7, − 5, − 1 )
=| b|| d− c|
37. V1 = i − 2 j + k$
$ $
Now, | d − c | 2 = d 2 + c 2 − 2 c ⋅ d
V2 = 3 i$ + 2 $j − k$
1
= 9 λ2 + λ2 − 2( λ) (3 λ) V3 = c = αa + βb = α( $i + $j) + β( $j + k$ )
2
= 10 λ2 − 3 λ2 = 7 λ2 = α $i + (α + β)$j + βk$ = c
∴ | d − c | = 7λ Since, V1, V2, V3 are coplanar.
1 −2 1
Denominator of Eq. (i)
Now, 3 2 − 1 = 0, using C 2 → C 2 − (C1 + C 3 ), we get
= (5 λ) (7 λ) = 5 7 λ2
α α+β β
10 λ2 2
∴ cos ( BA ⋅ CD) = = 1 −4 1
5 7 λ2 7
3 0 − 1 = 0, hence 4 (3β + α ) = 0
34. a = − i$ − 2$j − k$ α 0 β
b = $i − 5 $j − 7 k$
⇒ 3β + α = 0
|a × b| α
|d|= ; |a| = 6 ⇒ = −3
|a| β
144 Textbook of Vector & 3D Geometry

$i + $j + k$ 0 = [ r a b] (r ⋅ b) + z [ r a b] ⇒ z = − (r ⋅ b)
38. n$ = ;
3 Hence, r × a = [ r a b ] b − ( r ⋅ b ) (a × b )
ω = | ω | n$ = 10( $i + $j + k$ ) r × a = [ r a b] b + (r ⋅ b) (b × a )
Now, v = ω × r = 10 ( $i + $j + k$ ) × ( x $i + y $j + z k$ ), 44. Since, i$ + 2 $j + 2 k$ is rotated so as to cross Y -axis, the vector in
where r is the position vector of the point whose locus is to be new position. Let the required vector be xi$ + y$j + zk$
determined. where, x2 + y 2 + z 2 = 9 …(i)
Hence, v = 10 [(z − y ) $i − (z − x ) $j + (y − x ) k]
$
x + 2y + 2z = 0 …(ii)
∴ | v | = 10 ( x − y ) 2 + (y − z ) 2 + (z − x ) 2 x y z
Hence, 2 ( x 2 + y 2 + z 2 − xy − yz − zx ) = 4 and 1 2 2 ⇒ 2x − z = 0 …(iii)
0 1 0
⇒ x 2 + y 2 + z 2 − xy − yz − zx − 2 = 0
which is the equation of a cylinder. On solving Eqs. (i), (ii) and (iii), we get
2 −4
$i + $j + k$ ω x=− , y = 5, z =
39. n$ = ; ω = ( i$ + 2 $j + 2 k$ ) 5 5
3 3
−2$ 4
$i $j k$ ∴ Required vector, is i + 5 $j − k$
ω ω 5 5
v =ω × r = 1 2 2 = ( 4 $i − $j − k$ )
3 3 45. Set A → Set B
2 3 5
(Parallel) (Non-parallel)
ω 4 6
∴ | v| = 18 = ω 2
3 ways →
40. b ⋅ a + b ⋅ c = b ⋅ b + a ⋅ c (i) 3 from B → 6C 3
y
(ii) 2 from B, 1 from A → 4C1 × 6C 2
A
(iii) 3 from A → 4C 3
D Total number of ways

3j = 6C 3 + ( 4 C1 × 6C 2 ) + 4C1
∴ n( E ) = 6C13 + 4C1 + ( 4 C1 × 6C12 )
M x
B ∧
and n(S ) = 10C 3
(origin) 4 C4 i
or b ⋅(a − b ) − c ⋅(a − b ) = 0
6
C 3 + 4C1 + ( 6C 2 × 4C1 )
⇒ P (E ) =
or ( b − c ) ⋅(a − b ) = 0 10
C3
⇒ BC and AB are perpendicular.
46. ($a × x ) + b = x
Now, find angle between AM and BD.
⇒ a$ × ($a × x ) + ($a × b ) = a$ × x
where, AM = 2 $i − 3 $j,
($a ⋅ x ) a$ − ($a ⋅ a$ ) x + ($a × b ) = x − b
4 $i + 3 $j
BD = Projection of x along a$ is 2 units
2
($a ⋅ x )
AM ⋅ BD −1 ⇒ = 2 ⇒ a$ ⋅ x = 2
∴ cos θ = = | a$ |
| AM | | BD | 5 13
1
 1  So, x = [2a$ − b + ($a × b )]
⇒ θ = π − cos− 1   2
 5 13 
47. We know, a × (b × c ) = (a ⋅ c ) b − (a ⋅ b ) c
π π 1 3 3
41. [ npm ] = sin θ cos φ = sin ⋅ cos = ⋅ = Component of a × ( b × c ) along b is
6 6 2 2 4
(a ⋅ c ) b − (a ⋅ b ) c    (a ⋅ c )( b ⋅ b ) − (a ⋅ b ) ( b ⋅ c )
42. V = (a × b ) × a + (a × b ) × b   ⋅ b b =   b
 | b | 2
   | b |2 
= b − (a ⋅ b )a + (a ⋅ b ) b − a = ( b − a ) + ( b − a ) (a ⋅ b )
V = ( b − a ) (1 + a ⋅ b ) = λ( b − a ) So, component of a × ( b × c ) perpendicular to b is
 (a ⋅ c ( b ⋅ b ) − (a ⋅ b ) ( b ⋅ c )
43. Since, a and b are perpendicular, hence a, b and a × b are a × (b × c) −   b
non-coplanar. Hence, any vector say ( r × a ) can be expressed  | b |2 
as  (a ⋅ b)( b ⋅ c ) − (a ⋅ c ) ( b ⋅ b )
∴ r × a = xa + y b + z(a × b ) …(i) = a × (b × c) +   b
 | b |2 
dot with a 0=x+0+0 ⇒ x=0
(a × b ) ⋅( b × c)
dot with b [ r a b ] = 0 + y + 0 ⇒ y = [ r a b ] = a × (b × c) + b
dot with r 0 = x a ⋅ r + y r ⋅ b + z [ r a b] | b |2
Chap 02 Product of Vectors 145

48. r ⋅ a = 20, ⇒ x + 2y + z = 20, x,y ,z ∈ N 54. The mid-points of sides are D (1, 0, 0), F (0, 0, n).
The number of non-negative integral solution are BC 2
EF 2 = ⇒ BC 2 = 4(m 2 + n 2 )
17
C1 + 15C1 + K 1C1 = 81 4
4a AB 2 + BC 2 + CA 2
49. α ⋅ β = +
b
+1 =8
a b l 2 + m2 + n2
b 4a A
as + + 1 ≥5
a b E
F
 10 
So,   =1
 5 + α ⋅ β max B C
D
50. The system of vectors is coplanar.
Q Their sum is zero. 55. Eliminating m,
2 (l + n ) 2 + nl = 0
51. D¢ D C
or (2l + n ) (l + 2n ) = 0
a n = − 2l ⇒ m = − 2l
q
A B
or l = − 2n ⇒ m = − 2n
The d.r’s 1, − 2, − 2 and − 2, − 2, 1. The lines are perpendicular.
AB ⋅ AD 8
cosθ = = 56. cos2 θ + cos2 θ + cos2 γ = 1
| AB| | AD| 9
⇒ cos2 γ = − cos2θ
–1  8  α
∴ cos   + α = by hypothesis π π 
 9 2 ⇒ cos2θ ≤ 0 ⇒ θ∈ ,
 4 2 
8
∴ sinα =
9 57. a × (a × b ) = c ⇒| a | | a × b| = | c |
∴ cosα = 1 −
64
=
17 1(1 × 5 )sin θ = 3
81 9 3 3
sinθ = gives tanθ = ⋅
52. BA + AC = BC 5 4
⇒ BA = BC − AC 58. From the figure the vector equation of OP is r = λ(i$ + $j + k$ )
e f 2e  e f ∴ OM = projection of OC on OP
⇒ − − = − +  1
| e| | f| | e|  | e| | f|  = OC ⋅ OP =
3
 e f  e f
Now, BA ⋅ BC =  +   –  = 0 1 2
 | e| | f |  | e| | e|  Now, CM = OC − OM = 1 − =
2 2 2
3 3
⇒ ∠ B = 90° 2
⇒ cos 2 B = − 1 ∴ CM =
3
and cos2 A + cos2C = 2 cos( A + C ) cos( A − C ) = 0 Z
(Q A + C = 90°)
C
⇒ cos2 A + cos2 B + cos2C = − 1
53. We have, c ⋅ a = c ⋅ b = cosθ,a ⋅ b = 0 P
Now, c = αa + βb + γ (a × b) M
Taking the dot product of both sides with a, we get Y
O
c ⋅ a = α = cosθ (Q| a | 2 = 1, a ⋅ b = 0)
A
Similarly, β = cosθ
Now, taking the dot product with a × b, we get X
[ c a b] = γ | a × b|2 = γ
59. p × q, p, q are non-coplanar vectors
Now, [ c a b]2 = [a b c ]2
⇒ b − c = 0, c − a = 0 , a − b = 0
a⋅a a⋅ b a⋅c 1 0 cosθ ⇒ a =b =c
= b⋅ a b⋅ b b⋅ c = 0 1 cosθ ⇒ ∆ is equilateral.
c⋅ a a ⋅ b c⋅ c cosθ cosθ 1 60.
r
(a × b) × ( c × d) = [abd ]c − [a b c ]d
= 1 − cos2 θ + cosθ ( − cosθ ) = 4c − 4d
Thus, α = β = cosθ, γ 2 = − cos2θ. = − 8 $i perpendicular to Y-axis , Z-axis
146 Textbook of Vector & 3D Geometry

61. Translating the axes through A(1, 2, 3). a2 a


⇒ ⇒ EF =
( EF ) 2 =
A changes to (0, 0, 0) B changes to (8, 6, 2). 3 3
∴Coterminous edges are of lengths 8, 6, 2. | CF| = | CE| =
7a
and | CM| =
5a
Volume of parallelopiped = 8 ⋅ 6 ⋅ 2 = 96 cu units 3 6
62. a, b, c are non-coplanar ⇒[a, b, c] ≠ 1 Also, a × b, b × c, c × a where ‘m’ is middle point of EF.
are non-coplanar given D (origin)
d = sin x (a × b) + cosy ( b × c) + 2 ( c × a).
m F
Taking dot product with a + b + c, we get E
O = sin x[a b c] + cosy [a b c] + 2[a b c ]
A C
⇒ sin x + cosy + 2 = 0 a c
⇒ sin x + cosy = − 2
π
⇒ x = ( 4n − 1 ) , y = (2n − 1 ) π ,n ∈ z Bb
2
1
−π 5π 2 Area of ∆CEF = | EF| | CM|
for least value of x 2 + y 2, x = , y = π and least value is . 2
2 4
1 a 5a 5a 2
63. We have, α(a × b ) + β(b × c ) + γ(c × a ) = 0 = × × = sq units
2 3 6 12 3
Taking dot product with c, we have 69. p ⋅ q = 0 and r ⋅ s = 0
α [a b c ] + β[ b c c ] + γ[ c a c ] = 0 ⇒ (5a − 3 b) ⋅ ( −a − 2 b) = 0
i.e. α[abc ] + 0 + 0 = 0
6 b 2 − 7a ⋅ b − 5a 2 = 0 ...(i)
α[a b c ] = 0
⇒ ( −4a − b) ⋅ ( −a + b) = 0
Similarly, taking dot product with b and c, we have
4a 2 − b 2 − 3a ⋅ b = 0 ...(ii)
γ [abc ] = 0, β [abc ] = 0
Now, even if one of α , β, γ ≠ 0, then we have [abc] = 0 1
Now, x = (p + r + s )
⇒ a, b, c are coplanar. 3
1
1 1 1 1 x = (5a − 3 b − 4a − b − a + b)
64. λ 1h4 + λ 2h3 + λ 3h2 + λ 4h1 = 4 area the tetrahedron 3
3 3 3 3
OABC. 1 1
∴ x = − b, y = ( r + s) = ( −5a ) = − a
5 5
65. θ = cos−1(cosβ − cosα )
x⋅y a⋅b
Angle between x and y i.e cosθ = =
66. Z | x|| y| | a| | b|
C From Eqs. (i) and (ii), we get
A′ 25 43
B′ |a | = ⋅ ab and | b | = ab
Q′ 19 19
O 25 × 43 r r 19 a⋅b
B
Y | a| | b | = (a ⋅ b ) = =
A 19 5 43 | a | | b |
X 19
⇒ cosθ =
x y z 5 43
Equation of OO′ ⇒ = = ⇒ r = a + tb
1 1 1  19 
∴ θ = cos−1  
x −1 y z  5 43 
Equation of AB : = = ⇒ (α ) r = c + sd,
−1 1 0 1
where a = 0, b = $i + $j + k$ , c = $i, d = − $i + $j 70. Volume of tetrahedron = [a b c ]
6
|( c − a) ⋅ ( b × d )| 1 a⋅a a⋅ b a⋅c 4 2 2
Shortest distance = =
| b × d| 6 Now, [a b c ]2 = b⋅ a b⋅ b b⋅ c = 2 4 2
67. v × w = 3i$ − 5$j − k$ c⋅ a c⋅ b c⋅ c 2 2 4
Maximum value of [uvw ] = | u| | v × w | = 1 ⋅ 35 = 35 = 4 (12 ) + 2( −4 ) + 2 ( −4 )
1 2 2
68. | a| = | b| = | c| Volume = × 4 2 =
6 3
| b − a| = | b − c| = | c − a| = a
a ⋅ b = b ⋅ c = c ⋅ a = AB ⋅ AC = CA ⋅ CB = BA ⋅ BC
71. Given, cos θ = (a × $i ) ⋅ (b × $i ) + (a + $j) ⋅ (b × $j)
π + (a × k$ ) ⋅ ( b × k$ ) ...(i)
= a 2 cos
3 Consider, (a × $i ) ⋅ ( b × $i ) = [(a × $i ) b $i ] = ((a × $i ) × b) ⋅ $i
Chap 02 Product of Vectors 147

((a ⋅ b)i$ ) − ( $i ⋅ b)a)i$ = (a ⋅ b ) ( $i ⋅ $i ) – ( $i ⋅ b$ ) (a ⋅ $i ) 75. a × ( b × c ) = (a × b ) × c


= a ⋅ b − a1b1 or (a ⋅ c ) b − (a ⋅ b ) c = (a ⋅ c ) b − ( c ⋅ b ) a
Similarly, (a × $j) ⋅ ( b × $j) = a ⋅ b − a 2b 2 or (a ⋅ b ) c − ( c ⋅ b ) a = 0
and (a × k$ ) ( b × k$ ) = a ⋅ b − a 3b3 or b × (c × a ) = 0
∴From Eq. (i), we get or (c × a ) × b = 0
cosθ = 3a ⋅ b − (a1b1 + a 2b 2 + a 3b 3 ) or b ×(c × a ) = (c × a ) × b = 0
= 3a ⋅ b − a ⋅ b 76. Let angle between a and b be θ
a⋅b 1 v = a × b = | a | | b | sin θ n$
⇒ = 2a ⋅ b ⇒ | a| | b| =
| a| | b| 2  (a × b ) v 
∴ | v | = sin θ, Q | a | = 1, | b | = 1, n = = 
Now, use AM ≥ GM on | a| ,| b|  | a × b | | v |
1
| a| + | b| 2 u = a − (a ⋅ b ) b = a − cosθ b
∴ ≥ (| a | ⋅| b| ) 2 ⇒ | a| + | b| ≥
2 2 (Qa ⋅ b = | a | | b | cosθ = cosθ)
⇒ | a| + | b| ≥ 2 u ⋅ u = | u | 2 = 1 + cos2 θ − 2 cosθ = sin 2 θ

72. Qa ⋅ b < 0 ∴ | u | = sin θ


u ⋅ a = a ⋅ a = − cos θ a ⋅ b = 1 − cos2 θ = sin 2 θ
⇒ (sin x − sin x ) − cos x + 3 − 4 sin x < 0
2 2

u ⋅ b = a ⋅ b − cosθ b ⋅ b = cosθ − cosθ = 0


⇒ 2 sin 2 x − 5 sin x + 2 < 0
u ⋅ (a + b ) = (a − cosθ b ) ⋅ (a + b )
⇒ (sin x − 2 ) (2 sin x − 1 ) < 0
1424 3 = 1 + cosθ − cos2 θ − cosθ
( − ) ve
= 1 − cos2 θ = sin 2 θ
1  π π
⇒ sin x > ⇒ x ∈ ,  77. Here, (la + mb ) × b = c × b
2  6 2
⇒ la × b = c × b
73. A(a) ⇒ l(a × b ) 2 = ( c × b ) ⋅ (a × b )
3a+b 1 2 ( c × b ) ⋅ (a × b )
E l a+2c ⇒ l =
4
1 D
3
(a × b ) 2
3 m F (c × a ) ⋅ (b × a )
1
1
C(c) Similarly, m=
(b × a )2
B(b)
3a + b 78. a × (r × a ) = a × b
Positive vector of point E =
4 3 r − (a ⋅ r ) a = a × b
a + 2c Also, | r × a | = | b|
Position vector of point D = 2
3 ⇒ sin 2 θ =
Let point F divides EC in λ : 1 and BD in µ : 1, 3
µ 3a + b 2
b + (a + 2 c) λc + or (1 − cos θ) =
2

3 4 3
then =
µ+1 λ+1 or
1
= cos θ ⇒ a ⋅ r = ± 1
2

 µ   3a + b  3
b + (a + 2 c ) ( λ + 1 ) =  λc +  (µ + 1 ) ⇒ 3r ± a = a × b
 3   4 
1
Comparing the coefficient of a b and c. or r = (a × b ± a )
3
µ( λ + 1 ) 3(µ + 1 )
⇒ = ...(i)
3 4 79. a1 + a 2 cos2x + a 3 sin 2 x = 0, ∀ x ∈ R
µ+1 or (a1 + a 2 ) + sin 2 x (a 3 − 2a 2 ) = 0
⇒ λ+1= ...(ii)
4 ⇒ a1 + a 2 = 0 and a 3 − 2a 2 = 0

( λ + 1 ) = λ (µ + 1 ) ...(iii) a1 a 2 a 3
3 = = = λ( ≠ 0 )
−1 1 2
3
On solving, we get λ= ⇒ a1 = − λ, a 2 = λ, a 3 = 2 λ
2
80. a × b = | a | | b | sin θ n$
a b | a × b| = | a | | b | sin θ
74. Obviously, + is a vector in the plane of a and b and or
|a| | b| |a × b |
hence perpendicular to a × b. It is also equally inclined to a and or sin θ = …(i)
|a | × | b|
b as it is along the angle bisector.
148 Textbook of Vector & 3D Geometry

a ⋅ b = | a || b | cosθ ⇒
|a| | c|
=
|a ⋅ b| | c| |a|
⇒ cosθ = …(ii)
| a || b | ⇒ | a | = | c | and | b | =1
From Eqs. (i) and (ii), ⇒ a × b ⋅ c = | a | | b | | c | = | a |2 = | c |2
sin 2 θ + cos2 θ = 1 (children will assume a = $i; b = $j and c = k$ but in this case all
⇒ | a × b | 2 + (a ⋅ b ) 2 = | a | 2 | b | 2 the four will be correct which will be wrong).
If θ = π / 4, then sin θ = cosθ = 1 / 2. Therefore, 85. Given, | A | | B | cos θ = − 6; | B | = 2 (given)
| a || b | | a || b | B ⋅ C = | B | | C | cos φ = 6
|a × b| = and a ⋅ b =
2 2 and ( A × B) × A = 0
|a × b| = a ⋅ b (A ⋅ A )B − (B ⋅ A )A = 0
| a || b | (A ⋅ A ) = − 6A …(i)
a × b = | a || b | sin θ n$ = n$
2 ∴ A and B are collinear and θ between A and B is π.
= (a ⋅ b ) n$ ⇒ A× B = 0
81. We have, | a − b | 2 = | a | 2 + | b | 2 − 2(a ⋅ b ) ⇒ (a) is correct.
⇒ A ⋅ ( B × C ) = ( A × B) ⋅ C = 0
or | a − b | 2 = | a | 2 + | b | 2 − 2 | a || b | cos2θ
⇒ (b) is correct.
or | a − b | 2 = 2 − cos2θ (Q | a | = | b | = 1 ) Also, A ⋅ B = − 6 and | B | = 2
= 4 sin θ or | a − b | = 2 |sin θ|
2
∴ | A | | B | cos π = − 6 | A | ⋅ (2 ) = 6
Now, |a − b | < 1 ⇒ | A | =3 ⇒ A⋅A =9
1 ⇒ (c) is not correct.
⇒ 2 |sin θ | < 1 or |sin θ | <
2 Again, A ⋅ C = ?
⇒ θ ∈[ 0, π / 6 ) or θ ∈(5 π / 6, π ) dot with C in the Eq. (i)
82. a × (b × c ) + (a ⋅ b ) b 9 (B ⋅ C) = − 6A ⋅ C
9 (6 ) = − 6 ( A ⋅ C ) ⇒ A ⋅ C = − 9
= ( 4 − 2 x − sin y ) b + ( x 2 − 1 ) c
⇒ (d) is correct.
or (a ⋅ c ) b − (a ⋅ b ) c + (a ⋅ b ) b
= ( 4 − 2 x − sin y ) b + ( x 2 − 1 ) c
86. V1 = V2
a × ( b × c ) = (a × b ) × c
Now, ( c ⋅ c ) a = c .
(a ⋅ c ) b − (a ⋅ b ) c = (a ⋅ c ) b − ( b ⋅ c ) a
Therefore, ( c ⋅ c ) (a ⋅ c ) = ( c ⋅ c ) or a . c = 1
∴ (a ⋅ b ) c = ( b ⋅ c ) a
⇒ 1 + a ⋅ b = 4 − 2 x − sin y , x 2 − 1 = − (a ⋅ b )
⇒ Either c and a are collinear or b is perpendicular to both a
or 1 = 4 − 2 x − sin y + x 2 − 1 and c
or sin y = x 2 − 2 x + 2 = ( x − 1 ) 2 + 1 ⇒ b = λ (a × c )
π 87. It may be observed that
But, siny ≤ 1 ⇒ x = 1, siny = 1 ⇒ y = ( 4n + 1 ) , n ∈I
2 2 3 −6
83. AB + BC = AC [U V W] = 6 2 3 = 343 ≠ 0
BC =
2u

u
+
v
=
u
+
v 3 −6 −2
| u| | u| | v| | u| | v| ⇒ U, V and W are non-coplanar, hence linearly independent
 u v  u v Further U × V = W and V × W = U
AB ⋅ BC =  −   + 
 | u | | v |  | u | | v | They form a right handed triplet of mutually perpendicular
= ( u$ − v$ ) ⋅ ( u$ + v$ ) = 1 − 1 = 0 vectors and of course!
⇒ ∠B = 90 ° ⇒ ( U × V ) × W = 0 and U × ( V × W )
⇒ 1 + cos 2 A + cos2 B + cos2C = 0 88. Let the required vector be d = x $i + y $j + zk$ . For this to be
84. Clearly, a ⋅ c = 0 and b ⋅ c = 0. Also, a ⋅ b = 0 coplanar wih b and c , we must have
a× b= c x y z
dot with b ⇒ b⋅ c = 0 1 2 −1 = 0
Similarly, b× c= a 1 1 −2
dot with b ⇒a ⋅ b = 0
⇒ x ( −4 + 1 ) + y ( −1 + 2 ) + z (1 − 2 ) = 0
dot with c ⇒a ⋅ c = 0
−3 x + y − z = 0
⇒ a ⋅ b = b ⋅ c = c ⋅a = 0 | a ⋅ d|
|a|| b| =| c| The projection of d on a is .
Again, | a|
| b|| c| =|a|
Chap 02 Product of Vectors 149

r
2 1 B → (a × b) × c,a × b,c = 0 in that order form RHS
So, = |2 x − y + z |
3 6 ⇒ c,(a × b) × c, a × b also form RHS as they are in same cyclic
⇒ 2x − y + z = ± 2 order.
The choices (a) and (c) satisfy the Eqs. (i) and (ii). C → a + b + c = 0 ⇒ (a + b + c)2 = 0

89. a × ( b − 3 c) = 0 ⇒ a 2 + b 2 + c 2 = − 2(a ⋅ b + b ⋅ c + c ⋅ a )
⇒ b − 3 c = λa Hence, a ⋅ b + b ⋅ c + c ⋅a < 0
⇒ | b − 3 c| = | λa| D→ a + b+ c= 0
1 1 ⇒ a × b = b × c = c ×a
⇒ 1 + 1 − 6.1 ⋅ ⋅ = | λ | ⇒ λ ± 1.
3 2 Using this we get result.
∴ b − 3c = ± a 96. a ⋅ b = 0, c ⋅ a = c ⋅ b = cosα
90. (a × c) ⋅ (r × a) = (a × c) ⋅ b Take dot products with a, b and c, respectively.
91. (a) is proved if we take dot product of both sides with a. l = m,n 2 + l 2 + m 2 = 1
(b) If we take dot product with b, we get 1 + cos2α
n 2 = − cos 2α , m 2 =
λ2 = b⋅ c 2
⇒ Option (b) is not true. 97. A → a × (b × c) + b(c × a) + c(a × b ) = 0
(c) If we take dot product of both sides with a × b, we get ⇒ Vectors are coplanar, so do not form RHS
[ c b a] = λ 3[a × b]2 B → ( a × b) × c, a × b, c in that order form RHS
⇒ λ 3 = [a b c] or c ⋅ (a × b ) ⇒ c, (a × b) × c, a × b also form RHS as they are in same
⇒ Option (c) is wrong. cyclic order
(d) is correct since λ 1 + λ 2 + λ 3 = c ⋅ a + b ⋅ c + [a b c ]. C → a + b+ c= 0
92. (a) Since, a, b, c, are non-coplanar, option (a) is true. ⇒ (a + b + c ) 2 = 0
Since, b × c, c × a, a × b are also non-coplanar. ⇒ a 2 + b 2 + c 2 = − 2(a ⋅ b + b ⋅ c + c ⋅ a )
(b) is also correct. Hence, a ⋅ b + b ⋅ c + c ⋅ a < 0
Since, x = λ ( b × c) + µ( c × a) + v(a × b) D→ a + b+ c= 0
a⋅x ⇒ a × b = b × c = c ×a
We have, λ = , (on taking dot product with a)
[a b c ] Using this we get result.
µ and v have similar values. 98. Since a, b, a × b are non-coplanar,
Also, | x| = | a − x| r = xa + yb + z (a × b), r × b = a
a2 ⇒ xa × b + z (a × b) × b = a
⇒ a ⋅ x = , etc.
2 ⇒ x(a × b) + z ( b ⋅ a)b − ( b ⋅ b )a = a
⇒ Option (c) is correct. ⇒ x(a × b ) − a(1 + | b| 2 z ) = 0
If (c) is correct (d) is ruled out. 1
⇒ x = 0, z = − 2
93. α = (a ⋅ $i ) $i + (a ⋅ $j) $j + (a ⋅ k$ ) k$ = a = (1, 1, 1 ) | b|
β = ( b ⋅ $i ) $i + ( b ⋅ $j) $j + ( b ⋅ k$ ) k$ = b = (1, − 1, 0 ) 1
∴ r = yb − 2 (a × b), where y is any scalar.
γ = (1, 1, − 2 ) | b|
∴ α ⋅β = β ⋅ γ = γ ⋅α = 0 99. Let angle between a and b be θ.
⇒α, β, γ are mutually perpendicular α , β, γ = 6 We have, | a| = | b | = 1
⇒α , β, γ form a parallelopiped of volume 6 units. θ θ
Now, | a + b| = 2 cos and | a − b| = 2 sin
94. a × (b × c) = (a ⋅ c)b − (a ⋅ b)c 2 2
= ( xz + yx + yz ) (y$i + z$j + xk$ ) 3 θ  θ 
Consider, F θ = 2 cos  + 2 2 sin 
2 2  2
− ( xy + yz + zx ) (zi$ + x$j + yk$ )
θ θ
= ( xz + yx + yz ) ((y − z ) $i + (z − x ) $j + ( x − y ) k$ ) ∴ F (θ ) = 3 cos + 4 sin ,θ ∈ [ 0, π ]
2 2
Clearly, perpendicular to $i + $j + k$ and also to −3 θ θ
F (θ ) = sin + 2 cos
(y + z ) $i + (z + x ) $j + ( x + y ) k$ as dot products are zeros. 2 2 2
Clearly, parallel to (y − z ) $i + (z − x ) $j + ( x − y ) k$ Now, F (θ ) = 0
θ 4
95. A → a × (b × c ) + b × (c × a ) + c × (a × b ) = O ⇒ tan =
2 3
⇒ Vectors are coplanar, so do not form RHS
150 Textbook of Vector & 3D Geometry

Y 2tan–1 4 , 5
3
105. Since, (2$i + $j + 4k$ ) ⋅ ($i + 2$j − k$ ) = 0
(0, 5) De and ( i$ + $j) ⋅ ( $i + 2 $j − k$ ), 1 + 2 = 3
ing cre
as
⇒ Line lies in the plane
s
ea
ing
r
Inc

(p, 4)
(a ⋅ b ) a
106. OD =
| a| 2
(0, 3)
B C
X
q=0 q=p
q=2tan–1 4 b
3

O D a A
Clearly, F (θ ) = 3 (a ⋅ b )a
 4  3  4 9 16 25 ⇒ DB = b − OD = b −
F θ = 2 tan −1  = 3   + 4   = + = =5 | a| 2
 3  5  5 5 5 5
(a ⋅ a)b − (a ⋅ b)a a × ( b × a)
F (θ = π ) = 4 = =
| a| 2 | a| 2
∴ Range = [3, 5 ]
Hence, possible integer(s) in the range of F (θ ) in 107. a + b = λc; b + c = µa
[ 0, π ] are 3 viz, 3, 4 and 5. a − c = λc − µa
100. Let r = xa + yb + z(a × b ) with b a(1 + µ ) = c(1 + λ )
r ⋅ b = 0 + y| b| 2 but a and c are non-collinear ⇒µ = – 1, λ = − 1
∴ a + b + c = 0 = k$ ⇒| k| = 0
a( r ⋅ b) = y ( b ) 2a
⇒(k, k ) ≡ ( 0, 0 ) all the given curves pass through (0, 0)
⇒ c − pr = y| b| 2 a
b + c r  r r 1  1
y | b| 2 108. (a ⋅ c)b − (a ⋅ b)c = ; b a ⋅ c −  − c a ⋅ b +  = 0
r=
1
c− a 2  2  2
p p 1 1
⇒ a ⋅ c = and a ⋅ c = −
∴ [ r ac ] = 0 2 2
1
Now, r⋅ b = c⋅ b 109. AAT = I ⇒a , b , c are orthogonal unit vectors
p
$i $j k$
b⋅ c 1 1
∴ y |b |2 = c =a × b = 2 3 6 = ( −3 i$ + 6 $j − 2 k$ )
p 49 7
6 2 –3
1 1
∴ r = c − 2 ( b ⋅ c )a 1
p p ⇒ c = ± (3 $i − 6 $j + 2 k$ )
7
101. (λ − 1) (a1 − a 2 ) + µ(a 2 + a 3 ) + γ(a 3 + a 4 − 2a 2 ) + a 3 + δa 4 = 0 110. Component of vector b = 4$i + 2$j + 3k$ in the direction of
i.e. ( λ − 1 )a1 + (1 − λ + µ − 2 γ )a 2 + (µ + γ + 1 )a 3 a ⋅ b ⋅a
a = $i + $j + k$ is or 3 $i + 3 $j + 3 k$ . Then, component in the
+ ( γ + δ )a 4 = 0 |a | |a |
Since, a1, a 2, a 3, a 4 are linearly independent direction perpendicular to the direction of a = $i + $j + k$ is
∴ λ − 1 = 0, 1 − λ + µ − 2 γ = 0, µ + λ + 1 = 0 b − 3 $i + 3 $j + 3 k$ = $i − $j
γ+δ=0 111. Le the three given unit vector be a$, b$ and c$ . Since, they are
i.e. λ = 1, µ = 2 γ, µ + γ + 1 = 0, γ + δ = 0 mutually perpendicular, a ⋅ ( b × c ) = 1.
2 1 1 a1 a 2 a 3
i.e. λ = 1,µ = − , γ = − ,δ =
3 3 3 Therefore, b1 b2 b3 = 1
102. Since [a b c ] = 0 c1 c 2 c 3
∴a, b and c are complanar vectors a1 b1 c1
Further since d is equally inclined to a, b and c a 2 b2 c 2 = 1
∴d ⋅ a = d ⋅ b = d ⋅ c = 0 a 3 b3 c 3
∴d ⋅ r = 0
Hence, a1i + b1$j + c1k$ , a 2$i + b2$j + c 2k$ and a 3$i + b3$j + c 3k$ may
$
103. p = ab cos(2π − θ )c, where θ is the angle between a and b and be mutually perpendicular.
q = ac cos( π − φ ) b where φ is the angle between a and b now 112. Statement II is true (see properties of dot product)
p + q = (ab cosθ ) c − ac cos φb = (a ⋅ b ) c − (a ⋅ c ) b = a × ( c × b )
Also, ( $i × b ) ⋅ b = $i ⋅ (a × b )
⇒ B and C
⇒ a × b = ( $i . (a × b )) $i + ( $j ⋅ (a × b )) $j + ( k$ ⋅ (a × b )) k$
104. Verify v1 + v 2 = v 3 in order to quickly answer
Chap 02 Product of Vectors 151

113. AD = 2$j − k$ , BD = − 2$i − $j − 3k$ and ∴AB, AC and AD are linearly dependent, hence by
statement II, the statement I is true.
CD = 2 $i − $j
1 120. r = x a + yb + z(a × b )
Volume of tetrahedron = [ AD BD CD]
6 121. ($a × (b × c$ )) ⋅ ($a × c$ ) = ($a ⋅ c$ )[ bac
$$ ] = 5
0 2 −1
1 8 ⇒ $$ ] = 10 ⇒[$abc$ ] = − 10.
[ bac
= − 2 −1 − 3 =
6 3 Solutions (Q.Nos. 122-124)
2 −1 0
1 i$ + 3 $j − 4 k$ ) ⋅ ( i$ + $j + k$ ) = 1 + 3 − 4 = 0
122. (1 44244 3
Also, area of the triangle ABC = | AB × AC | (q − r)
2
$i $j k$ 1 1 1
1 1 299 Since, [ q p r ] = 2 4 −1
= 2 3 2 = | − 9 i$ − 2 $j + 12 k$ | =
2 2 2 1 1 3
−2 3 −1
= (12 + 1 ) − 1 (6 + 1 ) + 1 (2 − 4 ) = 13 − 7 − 2 = 4
8 1
Then, = × (Distance of D from base ABC) ⇒ (a) (b) (d) are wrong.
3 3
× (Area of triangle ABC) 123. (p × r ) × r = (p ⋅ r ) q − (q ⋅ r ) p
Distance of D from base ABC = 16 / 229 ⇒ u = − ( q ⋅ r ) = − (2 + 4 − 3 ) = − 3
v = p ⋅ r = 1 + 1 + 3 = 5 and w = 0
114. A × (A ⋅ B) A − (A ⋅ A ) B) ⋅ C Hence, u = − 3, v = 5, w = 0 ⇒ u + v + w = 2
 
=  A
1×4
4(A2⋅44 A − (A ⋅ A) A × B ⋅ C = − | A | 2 [ A B C ]
B)3 124. Q p ,q and r are non-coplanar, therefore
 zero  q × r, r × p and p × q are also non-coplanar
Now, | A | 2 = 4 + 9 + 36 = 49 Hence, s = l (q × r ) + w (r × p ) + n (p × q )
s⋅p s⋅q s⋅r
2 3 6 ∴ l = ,w = ,n =
[ A B C ] = 1 1 − 2 = 2 (1 + 4 ) − 1 (3 − 12 ) + 1 ( − 6 − 6 ) [p q r] [p q r] [p q r]
1 2 1 Hence, s[ p q r ] = (s ⋅ p ) ( q × r ) + (s ⋅ q )
( r × p ) + (s ⋅ r ) ( p × q )
= 10 + 6 − 12 = 7 = 10 + 9 − 12 = 7
|(s ⋅ p ) ( q × r ) + (s ⋅ q ) ( r × p ) + (s ⋅ r ) ( p × q )
∴ | − | A | 2 [ A B C ]| = 49 × 7 = 343
= | s [ p q r ]| = [ p q r ] ( as |s | = 1 )
a×b
115. ± ⊥ a and b 1 1 1
a×b 2 4 −1 = (12 + 1 ) − 1 (6 + 1 ) + 1 (2 − 4 ) = 13 − 7 − 2 = 4
∴There are two such vectors 1 1 3
k l m
116. + + =1 Solutions (Q.Nos. 125-127)
k+l +m k+l +m k+l +m
⇒ Point lies in the plane of ∆ ABC. 125. Given, p = $i + $j + k$ ; q = $i − $j + k$
p × r = q + c p and p ⋅ r = 2
117. a 2 + b 2 + 2a ⋅ b = a 2 + 4b 2 − 4a ⋅ b ⇒ 6ab cosθ = 3b 2
∴ p × ( p × r ) = p × q ( q + c p ) and p ⋅ ( p × r ) = p ⋅ ( q + c p )
1
⇒ a ⋅ b = b2 ∴ (p ⋅ r )p − (p ⋅ p )r = p × q + c 0
2
0 = p ⋅ q + c (p ⋅ q)
b2 4 b2 + 2 4
GE = + 2 = + 2 −1 ∴ (p ⋅ p )r = (p ⋅ r )p − p × q …(i)
2 b +2 2 b +2 p⋅q
c=− …(ii)
b 2 + 2 2 2  p⋅p
= 2 + 2  − 1 = 2(≥ 2) – 1 ≥ 2 – 1
 2 2 b + 2 But p ⋅ p = | p| = p2 = 3 …(iii)
b +2
2
p⋅p =1 −1 + 1 =1 …(iv)
It is least when =1
2 2 $i $j k$
⇒ b = 2( 2 – 1 ) = 2 tan π / 8 p × q = 1 1 1 = 2 $i − 2 k$ …(v)
118. Both the statements are true and statement II is the not 1 −1 1
correct explanation of statement I. Because b , c , d in Using Eqs. (iii), (iv) in Eq. (i) and Eq. (ii), we get
statement I are coplanar. 1
3 r = 2 p − 2 $i + 2 k$ and c = − …(vi)
119. 3a − 2b + 5c − 6d = (2a − 2b ) 3
+ ( −5a + 5 c) + (6a − 6 d ) = − 2 AB + 5 AC − 6 AD = 0 1 $
∴ r = [2 i + 2 $j + 2 k$ − 2 $i + 2 k$ ]
3
152 Textbook of Vector & 3D Geometry

2 Solutions (Q.Nos. 131-134)


∴ r = ( $j + 2 k$ ) …(vii)
3 131. Since, a, b, c are non-coplanar vectors, then
1 1 1 [a b c ] ≠ 0 ⇒ [a b c ]2 ≠ 0
 −4 2   4  2
Now, [ p q r ] = 1 −1 1 =  −  − 1  + 1  a ⋅a b ⋅a c ⋅a
1 4  3 3   3   3
0 ⇒ [a b c ]2 = a ⋅ b b ⋅ b c ⋅ b ≠ 0
2 3
2 8 a⋅ c b⋅ c c⋅ c
∴ [p q r] = − 2 − = − …(viii)
5 3 Since, any vector r in space can be expressed as a linear
8 combination of three non-coplanar vectors.
126. [ p q r ] x = (p × q ) × r ⇒ x = (p ⋅ r ) q − (q ⋅ r ) p So, let r = la + mb + nc …(i)
3
 8 2p 2 taking dot product by a, b, c successively, we get
⇒  −  x = 2q − (q ⋅ r = , verify yourself)
 3 3 3 r ⋅ a = l a ⋅ a + m b ⋅ a + nc ⋅ a …(ii)
3 2 1 $ r ⋅ b = l a ⋅ b + m b ⋅ b + nc ⋅ b …(iii)
∴ x = − ⋅ (3 q − p ) = (3 i − 3 $j + 3 k$ − $i − $j − k$ ) r ⋅ c = l a ⋅ c + m b ⋅ c + nc ⋅ c …(iv)
8 3 4
1 $ Now, eliminating l, m and n from above 4 relations, we get
= − (2 i − 4 j + 2 k )
$ $
r a b c
4
1 r ⋅a a ⋅a b ⋅a c ⋅a
∴ x = − ( i$ − 2 $j + k$ ) =0
2 r⋅ b a⋅ b b⋅ b c⋅ b
1
127. As c = − from Eq. (vi) r⋅c a⋅c b⋅ c c⋅ c
3 Now, expanding along first row, we get
 1
∴ 1 −  y = (p ⋅ r ) q − (p ⋅ q ) r ∆  ∆  ∆ 
 3 r =  1 a +  2 b +  3 c
 ∆  ∆  ∆
2
y = 2 q − (1 ) r [As p ⋅ q = 1 from Eq. (iv)] 132. Since, a, b, c are three non-coplanar vectors, then
3
On three exists scalars x,y ,z , such that
3 2 4  3 (6 $i − 8 $j + k$ )
∴ y = 2 $i − 2 $j + 2 k$ − $j − k$  = r = x a + y b + zc …(i)
2 3 3  2 3
Taking dot product by b × c, c × a and a × b successively,
y = 3i − 4 j + k
$ $ $
we get
1
− 1 −
1 r ⋅ ( b × c ) = ( xa + yb + zc ) ⋅ ( b × c ) = x [a b c ]
2 2 1  16 2 1 r ⋅ (c × a ) = y [ b c a ]
∴ [x y r] = 3 −4 1 =  − −  − 1( 4 ) − (2 )
2 4 2  3 3 2 r ⋅ (a × b ) = z [ c a b ]
0 [ r b c] [r c a] [ r a b]
3 3 Q x= ,y = and z =
[a b c ] [a b c ] [a b c ]
∴ [x y r] = 3 − 4 − 1 = − 2
On substituting the values of x, y , z in Eq. (i), we get
1 1
∴ [x y r] = − = c [ r b c ]  [ r c a ]  [ r a b ] 
6 3 r = a +  b+  c
[a b c ]  [a b c ]   [a b c ] 
⇒ x, y, r are the coterminous edges of a tetrahedron whose
1
volume is | c |. or r = {[ r b c ]a + [ r c a ] b + [ r a b ] c }
[a b c ]
Solutions (Q.Nos. 128-130)
133. We know that,
128. y = log1/2  x −  + log 2 (2x − 1)2
1
 [a × b b × c c × a ] = [a b c ]2
2
1  1 Clearly, [a × b b × c c × a ] ≠ 0 {[a b c ] ≠ 0 }
But, x> = log1/2  x −  + log 2(2 x − 1 ) ⇒ a × b b × c c × a are non-coplanar.
2  2
We also know that any vector in space can be expressed as a
y =1
linear combination of any three non-coplanar vectors, so let.
P ≡ (3, 1 ) a = l ( b × c ) + m ( c × a ) + n (a × b ) …(i)
129. OP = 3 $i + $j On taking dot product on both sides by a, b, c successively,
we get
Q ≡ (1, 1 ) or (2, 1 )
a ⋅ a = l [a b c ]
OQ = i$ + $j and 2 i$ + $j
a ⋅ b = m [ c a b]
OP ⋅ OQ = 3 + 1 = 4 and 6 + 1 = 7
c ⋅ a = n [ c a b]
130. PQ = OQ − OP = − 2$i or $i a ⋅a a⋅b a⋅c
Q l = ,m = and n =
| PQ| = 2 or 1 [a b c ] [a b c ] [a b c ]
Chap 02 Product of Vectors 153

On substituting these values in Eq. (i), we get  1


and x ⋅ z = | x| | z| cos60 ° = 2( 2 )   = 1
a ⋅a a⋅b a⋅c  2
a= (b × c) + (c × a ) + (a × b )
[a b c ] [a b c ] [a b c x ⋅ x = | x| 2 = 2
1
or a = {a ⋅ a ( b × c ) + a ⋅ b ( c × a ) + a ⋅ c (a × b )} y ⋅ y = | y| 2 = 2 and z ⋅ z = | z | 2 = 2
[a b c ]
Now, x × ( y × z ) = a and y × ( z × x) = b (given)
134. Let a = a1$i + a 2$j + a 3k$ , b = b1$i + b2$j + b3k$ , ⇒ ( x ⋅ z) y − ( x ⋅ y ) z = a and ( y ⋅ x)z − ( y ⋅ z)x = b
c = c1$i + c 2$j + c 3k$ , p = p1i$ + p 2$j + p 3k$ , ⇒ y − z = a and z − x = b
and q = q1$i + q 2$j + q 3k$ , ⇒ y − x =a + b
a b c Thus, we have
Then, a ⋅ p b⋅ p c⋅ p y − z =a ...(i)
a⋅q b⋅ q c⋅ q z−x=b ...(ii)
y − x =a + b ...(iii)
a1i$ + a 2$j + a 3k$ b1$i + b2$j + b3k$ c1$i + c 2$j + c 3k$ Now, x×y=c (given)
a1p1 + a 2p 2 + a 3p 3 b1p1 + b2p 2 + b3p3 c1p1 + c 2p2 + c 3p3 ⇒ x × ( x × y) = x × c (taking cross-product with x)
a1q1 + a 2q 2 + a 3q 3 b1q1 + b2q 2 + b3q 3 c1q1 + c 2q 2 + c 3q 3 ⇒ ( x ⋅ y) × x − ( x ⋅ x)y = x × c
$i $j k$ a1 a 2 a 3 ⇒ x − 2y = x × c ...(iv)
Again, x×y=c
p1 p 2 p 3 ⋅ b1 b2 c 3
⇒ y × ( x × y ) = y × c (taking cross product with y)
q1 q 2 q 3 c1 c 2 c 3
⇒ (y ⋅ y )x − (y ⋅ x )y = y × c
= ( p × q ) [abc] ⇒ 2x − y = y × c …(iv)
= [abc ]2 ( p × q ) On subtracting Eqs. (iv) and (v), we get
= [a × bb × cc × a ] ( p × q ) x − y = ( y × c) − ( x × c)
⇒ x × y = ( y − x) × c
Solutions (Q.Nos. 135-136) ⇒ x + y = (a + b) × c ...(vi)
135. Q g′ (x ) = 3x 2 + 2x + 0 > 0, ∀ x ≥ 0 Adding Eqs. (iii) and (vi), we get
⇒ g( x ) is an ↑ ing function. 1
2 y = (a + b) + (a + b) × c, y = [(a + b) (a + b) × c
If circumcentre lies outside, then triangle is obtuse angle 2
triangle and angle containing the given sides is obtuse angle. Substituting the value of y in Eq. (iii) in Eq. (i), we get
Therefore, 1
x = [(a + b) + (a + b) × c] − (a + b)
( f ( x ) i$ + g( x ) $j) ⋅ ( g( x ) $i + f ( x ) $j) < 0 2
⇒ f ( x ) ⋅ g( x ) < 0 …(i) ⇒
1
x = [(a + b) × c − (a + b)]
⇒ g( x ) ↑ forx ≥ 0 2
1
⇒ g( x ) > g( 0 ) ∀x > 0, Also, g( 0 ) = 0 z = [(a + b) + (a + b) × c ] − a
2
⇒ g( x ) > 0 ∀x > 0, (i) ⇒ f ( x ) < 0
1
∴ f ( x ) < 0 and g( x ) > 0 ∀ x > 0 z = [( b − a) + (a + b) × c ].
2
136. If x → ∞ then g(x ) → ∞ and f (x ) is some negative number,
then
137. (a) 138. (a) 139. (b)
f ( x ) ⋅g ( x ) Solutions (Q.Nos. 140 to 142)
  14243
  →∞
Taking dot products with a, b, c respectively with given
  
 equation
 π 
lim lim  cot  (1 − t 2 )  =0 [abc] = p + (q + r ) cosθ ...(i)
t → 0 x→∞  4 123
 −  0 = ( p + r ) cosθ + q ...(ii)
  1421 4 3 
 1442443 
 [abc] = ( p + q ) cosθ + r ...(iii)
 1+  1 cosθ cosθ
Solutions (Q.Nos. 137 to 139) Also, [abc] = cosθ
2
1 cosθ
cosθ cosθ 1
We have | x| = | y| = | z| = 2 and x, y, z make angle of 60° with
each other. = 2 cos2 θ − 3 cos3 θ + 1 = (1 − cosθ ) 2 (1 + 2 cosθ )
1
∴ x ⋅ y = | x| | y| cos60 ° = 2( 2 ) ⋅ = 1 ∴ v = |[abc ]| = |1 − cosθ|| 1 + 2 cosθ|
2
θ
 1 = 2 sin 2 | 1 + 2 cosθ |
y ⋅ z = | y| | z| cos60 ° = 2( 2 )   = 1 2
 2
154 Textbook of Vector & 3D Geometry

From Eqs. (i) and (iii) p = r ; substituting in Eq. (ii), we get 3p p


+ +p
q |a × b + b × c + c × a | 2 2 3| p |
∴ 2 p cosθ + q = 0 ⇒ + 2 cosθ = 0 ∴ Ratio = = = =3
p | c × a| | p| | p|
140. (a) 141. (c) 142. (b)
A⋅A A⋅B A⋅C 1 0 0
143. (A) We know that 3 vectors are coplanar, if x a + y b + z c = 0 3
Clearly, − 3 i$ + 3 $j + 4 k$ and i$ + $j are two vectors lie in the (B) [ ABC ]2 = B ⋅ A B⋅ B B⋅ C = 0 1
2
plane (a + b and a − b). C⋅ A C⋅ B C⋅ C 3
(B) a × b is a vector which perpendicular to both a and b. 0 1
2
$i $j k$  3 1
1 = 1 −  =
∴ a × b = − 1 2 2 = $i ( 4 − 2 ) − $j ( − 2 + 4 ) + k$ ( − 1 + 4 )  4 4
−2 1 2 1
[ ABC ] =
2
= 2 $i − 2 $j + 3 k$
b ⋅a b ⋅a
(C) c is a vector which is equally inclined to a and b (C) ( b × c ) ⋅ (a × d ) =
c ⋅a c ⋅ d
∴ c ⋅a = c ⋅ b
Similarly, compute others which gives (i).
Clearly i − j + 5 k$ satisfies the condition.
$ $
(D) a, b and c are from the triangle 146. (A) x ⋅ y = | a| 2 − | b| 2 = 0
⇒ AB + BC = AC x is perpendicular to y.
C | x × y|2 = | x| 2 | y| 2
{| a|2 + | b|2 + 2a ⋅ b| } {| a|2 + | b| 2 − 2a ⋅ b}
A B  1

= 64 − 4(a ⋅ b ) 2 = 4 16 − (a ⋅ b ) 2 
144. Given, [a × b b × c c × a ] = 36  
[a b c ] = 6
λ = 16
⇒ Volume of tetrahedron from by vectors
2λ + 1 1
1 (B) =
a, b and c is [a b c ]] = 1 λ +2 λ +2 2
2 2
6
[a + b b + c c + a ] = 2 [a b c ] = 12 2(2 λ + 1 ) = λ2 + 2
a − b, b − c and c − a are coplanar λ2 − 4 λ = 0, λ = 0 or 4
⇒ [a − b b − c c − a ] = 0
λ = 4 is non-zero value.
1 (C) If the lines are coplanar, all the 4 planes will have a
|a × b + b × c + c × a |
Area of ∆ABC 2 common point.
145. (A) =
Area of ∆AOC 1
|a × c| Solving 4 x + 3y − 2z + 3 = 0
2 x − 3y + 4z + 6 = 0
Now, a + 2b + 3c = 0 x −y + z + 1 = 0
Cross with b, a × b + 3 c × b = 0 1 −11
⇒ a × b = 3 (b × c) We get x = − , y = − 3, z =
3 3
Cross with a, 2a × b + 3a × c = 0 Substituting in kx − 4y + 7z + 16 = 0
3
⇒ a × b = (c × a ) We get k = 7
2
(D) E = | a − 2 b| 2 + | b − 2 c| 2 + | c − 2a| 2
3
∴ a × b = (c × a ) = 3 (b × c) = 5(| a| 2 + | b| 2 + | c| 2 ) − 4[a ⋅ b + b ⋅ c + c ⋅ a ]
2
Let (c × a ) = p = 5 ⋅ 6 − 4[a ⋅ b + b ⋅ c + c ⋅ a ]
3p p 30 − E
a×b= ; b×c= ⇒a ⋅ b + b ⋅ c + c ⋅ a =
2 2 4
A (a) Also, | a + b + c| 2 ≥ 0
6 + 2[a ⋅ b + b ⋅ c + c ⋅ a ] ≥ 0
30 − E 
6+2 ≥0
O
 4 
12 + 30 − E ≥ 0
B (b ) C (c)
42 ≥ E
E ≥ 42
Chap 02 Product of Vectors 155

147. (A) a ⋅ b + a ⋅ c = b ⋅ c + b ⋅ a = 0 i$ + $j + k$ $i + $j − k$ − $i + $j + k$
i.e., , ,
17 3 3 3
⇒ b⋅ c = a ⋅ c = −a ⋅ b = −
2 1 −1 1
1 1 2
⇒ | a + b + c| = a 2 + b 2 + c 2 − 2(a ⋅ b ) = 9 V = × 1 1 −1 =
6 3 3 9 3
(B) [a b c ], write in terms of (a1 a 2 a 3 b1b 2b 3 ) −1 1 1
b × c c × a a × b 1 So, 9 3V = 2
[ def ] =   =
 [abc ] [abc ] [abc]  [abc]
152. Let c$ = x a + y b, where x and y are scalars.
⇒ [abc] [ def ] = 1
⇒ $c = x ( i$ − $j + 2k$ ) + y (2 $i − $j + k$ )
ar( ABCD )
(C) α = = ⇒ c$ = $i ( x + 2y ) + $j( − x − y ) + k$ (2 x + y )
ar( parallelogram )
1 1  But, c$ ⋅ a = 0
 | a × (a + 3 b)| + |(2a + 3 b ) × b  5 6x
 2 2  = 6 x + 5y = 0 ⇒ y = −
 | a × b|  2 5
  −7 x $ x $ 4 x $
So, c$ = i+ j+ k
d⋅c d ⋅a d⋅b 5 5 5
(D) x = ,y = ,z =
[abc ] (a b c ) [abc ] 49 x 2 + x 2 + 16 x 2 25
We have, = 1 ⇒ x2 =
d ⋅ (a + b + c ) 25 66
x+y +z =
[abc] 5  −7 $ 1 $ 4 $
∴ c$ = ±  i + j + k
4 66  5 5 5 
⇒ R= =8
[abc] 11
p = | c$ ⋅ b | =
148. Let the angle between u and v is θ and w and u is θ 6
 u$ ⋅ u$ u$ ⋅ v$ u$ ⋅ w
$ So,
11
=6 ⇒ k =6
[u v w] = v ⋅ u v ⋅ v v ⋅ w 
$ $ $ 2  $ $ $ $ $ $ p
 
w$ ⋅ u$ w$ ⋅ v$ w$ ⋅w$  143. Let the angle between a and b is α and a × b and c is β.
 1 cos2θ cosθ  ∴ |[a b c ]| = 6

= cos2θ 1 cosθ  = 0 ⇒ sin α cosβ = 1 ⇒ sinα = 1, cosβ = 1
  ⇒ α = 90 °, β = 0 °
 cosθ cosθ 1 
⇒ a, b and c are mutually perpendicular.
149. 2[a b c ] + [a b c ]2 + 0 = 2 × 1 + 12 = 3 Again, [ b c d$ ] = 0
150. a$ ⋅ c$ = b$ ⋅ c$ = cosθ 4 0 1
c$ = α a$ + βb$ + γ ($a + b$ ) ⇒ 0 9 c ⋅ d$ = 0
Taking dot product with a$ both sides cosθ = α 1 c ⋅ d$ 1
Taking dot product with b$ both sides cosθ = β 3 3
⇒ c ⋅ d$ = ±
Taking dot product with c both sides 2
1 = α cosα + β cosθ + γ [$a b$ c$ ] We have, a⋅b = 0
 1 0 cosθ 
[$a b$ c$ ]2 =  0 cosθ 
1 0 0
But 1
  |a × c ⋅ d | = 0
$ 2
9
3 3
cosθ cosθ 1  2
= 1 − 2 cos2 θ 3 3
0 1
2
So, 1 = cos2 θ + cos2 θ + γ 1 − 2 cos2 θ
27 9
=9 − =
⇒ γ = 1 − 2 cos2 θ 4 4
|(a × c ) × d$ | 2 = |(a ⋅ d$ ) c − ( c ⋅ d$ )a | 2
So, α 2 + β 2 + γ 2 = 1
2
3 3 27
151. The three adjacent sides of tetrahedron is given by = a =
( i$ + $j) × ( $j + k$ ) ( $j + k$ )( k$ + $i ) ( k$ + $i )( $i + $j) 2 4
, , 36
|( i + j) × ( j + k )| | ( j + k ) × ( k + i )| |( k + $i ) × ( $i + $j)|
$ $ $ $ $ $ $ $ $
So, | a × c ⋅ d$ | 2 + |(a × c ) × d$ | 2 = =9
4
156 Textbook of Vector & 3D Geometry

λ −1$ µ+2$
154. P.V. of D = i + 4 $j + k D
2 2
C
λ − 4 µ −8
D.R. of AD = 1,
2, 2 N
1 1 1
But directions of AD should be , , .
3 3 3 M
λ−4 µ −8
⇒ =1 = A B
2 2
A (2i+3j+5k)
158. | a| = | b| = | c| = 1; [abc ] = 1
1
Volume of the tetrahedron =
6
3 −2 2
1
B C = −1 0 −2 [a b c ] = 2
(–i+3j+2k) (λi+5j+µk) 6
2 −3 4
λ = 6, µ = 10
159. [a × ( b × c )] ⋅ (a × c ) = 5
2λ − µ = 2
⇒ [(a ⋅ c ) b − (a ⋅ b ) c ] ⋅ (a × c ) = 5 ⇒ (a ⋅ c) [ bac ] = 5
155. v = [abc] ⇒ [abc] = − 10
a ⋅a a ⋅ b a ⋅ c
⇒ −[abc] − 1 = + 10 − 1 = 9
[α β γ ] = b ⋅ a b ⋅ b b ⋅ c [a b c ] a ⋅a a ⋅ b a ⋅ c
c ⋅a c ⋅ b c ⋅ c 160. b ⋅ a b ⋅ b b ⋅ c = [abc]2 = [a × b b × c c × a ]2 = 36
= [abc ] [abc] [abc] = v 3 c ⋅a c ⋅ b c ⋅ c
∴ λ =3 = 4 × 9 = 22 × 32
156. c × a = b ⇒| c × a| = | b| ∴ P + q =5
⇒ | c | | a | sin θ = 3, $
161. α$ ⋅ x$ = α$ ⋅ β − α$ ⋅ (α$ × x ) = 0
3
| c| = | c − a| 2 = | c| 2 + | a|2 − 2 c ⋅ a Also, α$ × x$ = α$ × β$ − α$ × (α$ × x )
2 sin θ
= | c|2 + 4 − 2| c|⋅| a| cosθ = α × β$ − (α$ ⋅ x )α$ + | α | 2 x
9 3 ⇒ β$ − x = α × β$ + x or 2x = β$ − α × β$
= + 4 − 2⋅ ⋅ 2 ⋅ cosθ
4 sin 2 θ 2 sin θ ⇒ 4| x| 2 = | β$ | 2 + | α × β$ | 2 − 2β$ ⋅ (α$ × β$ ) = 2
9
= 4 + cosec 2θ − 6 cot θ ⇒
1
| x| 2 =
⇒ 4| x| 2 = 2
4 2
2
9 3  −2 −4 6
= +  cot θ − 2
4 2  1
162. 0 1 −8 = 22 ⇒ λ = 133
9 6
| c − a| 2 ≥ 2 −5 λ − 1
4
(1, 1, 1)
3
⇒ | c − a| ≥
2
⇒ 2| c − a| ≥ 3 (–1, –3, 7) (3, –4, λ)
∴Min. of 2 | c − a| = 3
157. In ∆ABD, N is the mid-point of BD.
∴ AB + AD = 2 AN ...(i) 1
163. [abc] = 3 ⇒ [abc] = 18
In ∆CBD, N is the mid-point of BD. 6
∴ CB + CD = 2 CN …(ii) V = [a + b − c a − b b − c ]
Adding Eqs. (i) and (ii), we have = (a + b − c) ⋅ (a − b) × ( b − c ) = a ⋅ ( b × c ) = [abc] = 18
AB + AD + CB + CD = 2( AN + CN ) ...(iii) 164. Let θ be the angle between vectors a and b. Then,
In ∆ ANC , M is the mid-point of AC a ⋅ b = | a || b | cosθ
∴ AN + CN = 2 MN ⇒ (a ⋅ b ) 2 = | a | 2 | b | 2 cos2 θ
From Eq. (iii), we get Now, cos2 θ ≤ 1 ⇒ | a | 2 | b | 2 cos2 θ ≤ | a | 2 | b | 2
AB + AD + CB + CD = 2 (2 MN ) = 4 MN
∴ (a ⋅ b ) 2 ≤ | a| 2 | b | 2
Chap 02 Product of Vectors 157

165. Let P (x1, y1 ) and Q (x 2, y 2 ) be the two points on y = 2x + 2 A


OP ⋅ $i = Projection of OP on the X -axis
f
⇒ x1 = − 1 (Q OP . $i = − 1)
Also, ( x1, y1 ) lies on y = 2 x + 2
C
∴ y1 = 2 x1 + 2 ⇒ y1 = 2 B M
Also, OQ ⋅ i$ = Projection of OQ on X -axis. Let angle BAM = φ
⇒ x2 = 2 (given OQ . i$ = 2) AB ⋅ ( AB + AC ) | AB| 2 + AB ⋅ AC
∴ cos φ = =
As ( x 2, y 2 ) lies on y = 2 x + 2 | AB|| AB + AC | c b 2 + c 2 + 2bc cos A
y 2= 2 x 2 + 2 ; y 2 = 16 c 2 + c ⋅ b cos A c + b cos A
= =
Thus, OP = x1$i + y1$j = − $i + 2 $j c c 2 + b 2 + 2bc cos A b 2 + c 2 + 2b cos A
and OQ = x 2$i + y 2$j = 2 i$ + 16 $j b sin B
⇒ OQ − 4 OP = 6 $i + 8 $j and = ⇒ cos φ
c sin C
⇒ | OQ − 4 OP | = 36 + 64 = 10 sin C + sin B sin A
=
166. Let A(x1, y1 ) in XY -plane. sin 2 B + sin 2 C + 2 sin B sin C cos A
∴ OA = a = x1$i + y1$j
169. Let p = BA and q = BC
OP = r = x $i + y $j
Now, required perpendicular distance
Q Point P lies on the tangent to the circle.
= AM = ( BA )sinθ
A (x1, y1) = | p | sin θ …(i)
Consider , | q × p | = | q || p | sin θ
a
On dividing by | q|
| q × p|
O r P(x, y) = | p |sin θ …(ii)
| q|
A (1, 4, –2)

∴OA is perpendicular to AP.


⇒ OA ⋅ AP = 0 ⇒ a ⋅ ( r − a ) = 0 P
i.e., a ⋅ r −a ⋅a = 0
or a ⋅ r =a ⋅a
q C
⇒ a ⋅ r = a2 B (2, 1, –2) M q (0, –5, 1)
⇒ ( x1$i + y1$j) ⋅ ( x $i + y $j) = a 2
From Eqs. (i) and (ii), required perpendicular distance
⇒ xx1 + yy1 = a 2 | q × p|
= …(iii)
which is the equation of the tangent to the circle at the | q|
point A. where, q = BC = OC − OB = − 2 $i − 6 $j + 3 k$
167. The given relation can be rewritten as, P = AB = OA − OB = − $i + 3 $j
( a 2 − 4 i$ + a $j + a 2 + 4 k$ ) . (tan A $i + tan B $j + tan C k$ ) = 6a ∴ | q | = 4 + 36 + 9 = 7 …(iv)
⇒ (a − 4 ) + a + (a + 4 )
2 2 2
$i $j k$
tan A + tan 2 B + tan 2 C . cosθ = 6a (Qa . b = | a|| b| cosθ)
2 and q × p = −2 −6 3 = − 9 $i − 3 $j − 12 k$
−1 3 0
3a . tan 2 A + tan 2 B + tan 2C . cosθ = 6a
| q × p| = 81 + 9 + 144 = 3 26 …(v)
⇒ tan 2 A + tan 2 B + tan 2 C = 12 sec 2 θ …(i)
From Eqs. (iii), (iv) and (v), we get
Also, 12 sec 2 θ ≥ 12 (Qsec 2 θ ≥ 1) …(ii)
3 26
From Eqs. (i) and (ii), tan A + tan B + tan C ≥ 12
2 2 2 Perpendicular distance =
7
∴ Least value of tan 2 A + tan 2 B + tan 2 C = 12
170. Here, AM = MD and CD is angle bisector of ∠C.
168. Here, M is the mid-point of BC. ab + ba
1 CD =
∴ AM = AB + ( AB + AC ) (using AB + BC = AC) a+b
2 a+b
AB || ( AB + AC ) and CM =
2
158 Textbook of Vector & 3D Geometry

R is the point of intersection of AQ and CP.


(a)
A ⇒ For point R , we have
 2b + c b 
D λ1   = c + λ 2  − c
b
 3  3 
M 2λ 2 λ 2
f ⇒ = (comparing coefficients of b and c)
3 3
B C λ1
(b ) (c ) and = 1 − λ2
a 3
Where, a = CB and b = CA On solving, we get
Consequently, λ 1 = 3 / 7, λ 2 = 6 / 7
1 1
Area of ∆ CDM = ( CD × CM ) ⇒ R = (2 b + c )
2 7
1 (ab + ba ) × (a + b ) 1 5b − c
= Now, RB = b − ⋅(2 b + c ) =
2 1 (a + b ) 7 7
a( b × a ) + a ( b × b ) + b (a × a ) + b (a × b ) 1 (6 c − 2 b )
= RC = c − (2 b + c ) =
4(a + b ) 7 7
1
(b − a ) (a × b ) ∴ | RB × RC | = (5 b − c ) × (6 c − 2 b )
= 49
4(a + b )
1 28
(using a × a = b × b = 0 and b × a = − a × b) = (30 b × c + 2 c × b ) = (b × c)
49 49
b − a  a × b
=   28 28
2 (a + b )  2  ⇒ | RB × RC | = | b × c| = [(area of ∆ ABC) ⋅2]
49 49
b −a
= (Area of ∆ ABC) (Q area of ∆ BRC = 1)
2(a + b ) 1 49 49
Area of ∆ CDM (a − b ) sin A − sin B ⇒ Area of ∆ ABC = | RB × RC | ⋅ = (Area of ∆ BRC ) ⋅
∴ = = 2 28 28
Area of ∆ ABC 2 (a + b ) 2 (sin A + sin B ) 49
∴ Area of ∆ABC = sq units.
Also, CD || (ab + ba ) and CM | | (a + b ) 28
(ab + ba ) ⋅ (a + b )
cos φ = 172. Let OABC be a given quadrilateral such that its diagonal OB
| ab + ba | | a + b| bisects the diagonal AC let OA = a, OB = b, OC = c.
a| b| + b| a| + | a| | b| (a + b ) cosC
2 2 a+c
= Since, the mid-point of AC lies on OB, there exits a
2
2a b + 2a 2b 2 cosC a 2 + b 2 + 2ab cosCA
2 2
scalar t such that,
[Where, a = a and b = b] a+c
Area of ∆ABC = = tb ⇒ a + c = 2t b
(a + b ) cos(C / 2 ) 2
=
a 2 + b 2 + 2ab cosC C B
(sin A + sin B ) cos(C / 2 )
=
sin 2 A + sin 2 B + 2 sin A sin B cosC

171. Let A (O), B (b ), C (c ), P (p ), Q (q ), R(r )


b
We have, p = O A
3
C(c) On multiplying both sides with b, we have
(a + c ) × b = 2t b × b
Q ⇒ a×b+c×b=0
2:1 ⇒ a×b=b×c
R 1 1
P B(b) ⇒ |a × b| = | b × c|
1:2 2 2
A(o) ⇒ Area of ∆ OAB = Area of ∆OBC
2b + c Hence, the diagonal OB bisects the quadrilateral.
and Q=
3
173. The coordinates of the resulting force F = F1 + F2 = {6, 4} i. e.,
 2b + c
Equation of the line AQ, r = λ 1   resultant F are 6 and 4. Now, let M (a, y ) be a arbitrary point of
 3  l. Then, the moment of the resultant about point M is equal to
b  zero.
Equation of the line CP, r = c + λ 2  − c
3 
Chap 02 Product of Vectors 159

This moment is equal to sum of the moments MA × F1 and ⇒


1
y2 = ,y = ±
1
MB × F2 of component forces (the cross product of vectors is 2 2
distributive.) ⇒ x=±2 2
Since, MA = (1 − x, 1 − y ), MB = {2 − x, 4 − y }, if follows that
It is given that the vector passes through the positive X -axis.
( MA × F1 ) ( $i + $j) = 3 (1 − x ) − 2 (1 − y ) 1
= 1 − 3 x + 2y ⇒ x = 2 2 and y = 0 − =z
2
( MB × F2 ) ⋅ ( $i + $j) = (2 − x ) − 4 ( 4 − y )
 1 $ 1 $
= −14 − x + 4y Hence, required vector is 2 2 i$ − j− k .
 2 2 
Hence, the equation of straight line l is
(1 − 3 x + 2y ) + ( −14 − x + 4y ) = 0 176. u$ + v$ + u$ + w and w × u$ = v$
⇒ −4 x + 6y − 13 = 0 ⇒ ( u$ + v$ + u$ ) × u$ = w × u$
⇒ 4 x − 6y + 13 = 0 ⇒ ( u$ + v$ ) × u$ + u$ × u$ = v$
⇒ ( u$ ⋅ u$ ) v$ − ( v$ ⋅ u$ ) u$ + u$ × u$ = v$
174. Let a = x$i + y $j + z k$
(using u$ ⋅ u$ = 1 and u$ × u$ = 0, since unit vectors)
Now, a, $i and $i + $j are coplanar and a, $i − $j and $i + k$ are
⇒ v$ − ( v$ ⋅ u$ ) u$ = v$
coplanar. ⇒ ( u$ ⋅ v$ ) u$ = 0 ⇒ u$ ⋅ v$ = 0
⇒ [a $i $i + $j] = 0 and [a $i − $j $i + k$ ] = 0 Now, [ u$ v$ w ]
x y z x y z ⇒ u$ ⋅ ( v$ × w ) ⇒ u$ ⋅ [ v$ × ( u$ × v$ + u$ )]
⇒ 1 0 0 = 0 and 1 −1 0 = 0 (given w = u$ × v$ + u$ )
1 1 0 1 0 1 ⇒ u$ ⋅ [ v$ × ( u$ × v$ ) + v$ × u$ ]
∴ = 0 and − x − y + z = 0
z ⇒ u$ ⋅ [( v$ ⋅ v$ ) u$ − ( v$ ⋅ u$ ) v$ + v$ × u$ ]
⇒ = 0 and x + y = 0
z [Q u$ ⋅ v$ = 0 from Eq. (i)]
⇒ = −x
y ⇒ | v$ | 2 ( u$ ⋅ u$ ) − u$ ⋅ ( v$ × u$ )
∴ = x $i − x $j
a ⇒ | v$ | 2 | u$ | 2 − 0 (Q[ u$ v$ w ] = 0)
$i − $j (Q| u$ | = | v$ | = 1)
⇒ a=
2 ⇒ [ u$ v$ w ] = 1
Let the angle between a and i$ − 2 $j + 2 k$ be θ. 177. We have,
i$ − 2 $j + 2 k$ R × B = C × B and R ⋅ A = 0
cosθ = a ⋅
| i$ + 2 $j + 2 k$ | ⇒ A × ( R × B) = A × ( C × B)
( i$ − $j) ( $i − 2 $j + 2 k$ ) ⇒ ( A ⋅ B) R − ( A ⋅ R ) B = ( A ⋅ B) C − ( A ⋅ C ) B …(i)
= ⋅ where, A = 2 i$ + k$ , B = $i + $j + k$
2 1+4+4
and C = 4 i$ − 3 $j + 7 k$
(1 − $j) ( $i − 2 $j + 2 k$ ) 1+2 1
= = = ∴ A ⋅ B = 2 + 1 = 3, A ⋅ C = 8 + 7 = 15
2 ⋅3 3 2 2
π Hence, Eq. (i) reduces to
∴ θ= 3 R − 0 ⋅ B = 3 C − 15 B
4
or R = C − 5 B = ( 4 $i − 3 $j + 7 k$ ) − 5( $i + $j + k$ )
175. In the new position, let the vector be x$i + y $j + z k$ . Since, it is
perpendicular to the given vector. ∴ R = − $i − 8 $j + 2 k$
( xi$ + y $j + z k$ ) ⋅ ( $i + 2 $j + 2 k$ ) = 0 178. Since, a, b and a × b are non-coplanar vectors.
⇒ x + 2y + 2z = 0 …(i) Let x = λ + µb + γ(a × b ) …(i)
The magnitude is the new position which also remains the x ⋅ a = λa ⋅ a + µb ⋅ a + γ(a × b ) ⋅ b
same.
⇒ 0 × λ | a | 2 + µa ⋅ b …(ii)
⇒ x2 + y 2 + z 2 = 1 + 4 + 4 = 9 …(ii)
Again from Eq. (i),
The given vector, the vector in new position and the X -axis are
x ⋅ b = λa ⋅ b + µb ⋅ b + γ(a × b ) ⋅ b
coplanar.
1 0 0 1 = λ a ⋅ b + µ| b | 2 …(iii)
⇒ 1 2 2 =0 From Eq. (i)
x y z x ⋅ (a × b ) = λa ⋅ (a × b ) + µb ⋅ (a × b ) + γ(a × b ) 2

⇒ y = z and x = − 4y (using x + 2y + 22 = 0) [ x a b ] = λ[a a b ] + µ[ b a b ] + γ(a × b )2


Hence, x2 + y 2 + z 2 = 9 1 = γ(a × b ) 2 …(iv)
2
⇒ 16y + y + y = 9
2 2 2
From Eq. (ii), µ(a ⋅ b ) = − λ | a | 2 ⇒ µ = − λ
| a|
a⋅b
160 Textbook of Vector & 3D Geometry

From Eq. (iii), − { b ⋅ ( x − c )} b + ( c ⋅ c ) ( x − a ) − { c ⋅ ( x − a )} c = 0


|a |2 | b|2 ⇒ λ 2 ( x − b ) − {a ⋅ x − 0 } a + λ 2 ( x − c ) − { b ⋅ x − 0 } b
1 = λa ⋅ b − λ
a⋅b + λ 2 ( x − a ) − ( c ⋅ x − 0 )} = c = 0
 (a ⋅ b ) 2 −| a| 2 | b | 2  (using a ⋅ b = b ⋅ c = c ⋅ a = 0 and | a | = | b | = | c | = λ)
⇒ 1 = λ 
 a⋅b  ⇒ λ {x − b + x − c + x − a }
2

a⋅b = {(a ⋅ x ) a + ( b ⋅ x ) b + ( c ⋅ x ) c }
⇒ λ=
(a ⋅ b ) 2 − a 2b 2 Let x = α a + β b + γ c
( x is linear combination of a, b and c )
179. x$ + y$ + z$ = a …(i)
2 ⇒ a ⋅ x = α a ⋅a ⇒ a ⋅ x = α λ 2
a ⋅ x$ + a ⋅ y$ + a ⋅ z$ = a ⋅ a = a = 4
b ⋅ x = βλ 2
3 7 3
+ + a ⋅ z$ = 4 ⇒ a ⋅ z$ = …(ii) c ⋅ x = γλ 2
2 4 4
3 From Eq. (i), λ 2 {3 x − (a + b + c )} (a ⋅ x )
From Eq. (i), x$ ⋅ ( x$ + y$ + z$ ) = x$ ⋅ a = a ⋅ x$ =
2 a + (b ⋅ x ) b + (c ⋅ x ) c
3
⇒ x$ ⋅ x$ + x$ ⋅ y$ + x$ ⋅ z$ = and from Eq. (iii)
2 a ⋅ x = λ 2 α, b ⋅ x = λ 2β, c ⋅ x = λ 2γ
3
⇒ 1 + x$ ⋅ y$ + x$ ⋅ z$ = Above equation reduces to
2
1 λ 2 (3 x − (a + b + c )} = λ2(aα + bβ + cγ )
⇒ x$ ⋅ y$ + x$ ⋅ z$ = …(iii) ⇒ 3x − (a − b + c )
2
7 ⇒ 2x = a + b + c
From Eq. (i), y$ ⋅ ( x$ + y$ + z$ ) = y$ ⋅ a = a+b+c
4 ⇒ x=
7 2
x$ ⋅ y$ + 1 + y$ ⋅ z$ =
4 181. Here, OC = x, CA = b, CB = a
3
x$ . y$ + y$ ⋅ z$ = …(iv) OA = ( b − x ) and OB = a − x
4 A
From Eq. (i), ( x$ + y$ + z$ ) 2 = (a ) 2
2 b
⇒ x$ ⋅ x$ + y$ ⋅ y$ + z$ ⋅ z$ + 2( x$ ⋅ y$ + y$ ⋅ z$ + z$ ⋅ x$ ) = a
3 + 2( x$ ⋅ y$ + y$ ⋅ z$ + z$ ⋅ x$ ) = 4 …(v) O
x
From Eqs. (iii), (iv) and (v), we get
1 3 B a C
y$ ⋅ z$ = 0, x$ ⋅ z$ = − , x$ ⋅ y$ =
4 4
Now, x$ × ( y$ × z$ ) = b Now, OA 2 = OB2 = OC 2
( x$ ⋅ z$ ) y$ − ( x$ ⋅ y$ )$z = b x 2 = (a − x ) 2 = ( b − x ) 2
1 3 ⇒ x ⋅ x = (a − x ) ⋅ (a − x ) = ( b − x )( b − x )
− y$ − z$ = b …(vi)
4 4 ⇒ x ⋅ x = a ⋅ a − 2a ⋅ x + x ⋅ x = b ⋅ b − 2 b ⋅ x + x ⋅ x
Again, ( x$ × y$ ) × z$ = c a2 b2
1 a⋅x = and b ⋅ x =
( x$ ⋅ z$ ) y$ − ( y$ ⋅ z$ ) x$ = c$ ⇒ − y$ = c 2 2
4
Now, if we take x = λa + µb, then from Eq. (i)
y$ = −4 c [from Eq. (vi)]
a2
4 λa 2 + µ ⋅ a ⋅ b =
z$ = ( c − b ) 2
3 2
b
From Eq. (i), x$ = a − y$ − z$ and λ a ⋅ b + µb 2 =
1 2
x$ = (3a + 4 b + 8 c ) ∴On solving Eqs. (ii) and (iii),
3
1 a 2b 2 − b 2 (a ⋅ b )
x$ = (3a + 4 b + 8 c ) λ=
3 2 (a 2b 2 ) − (a ⋅ b ) 2
4 a 2b 2 − a 2(a ⋅ b )
y$ = −4 c ; z$ = ( c − b ) and µ=
3 2 (a 2b 2 ) − (a ⋅ b ) 2
180. Here, 1 a 2b 2 − b 2 (a ⋅ b ) 1 a 2b 2 − a 2(a ⋅ b )
a × {( x − b ) × a } + b × {( x − c ) × b } + c × {( x − a ) × c } = 0 ⇒ x= a+ ⋅b
2 (a b ) − (a ⋅ b )
2 2 2
2 (a 2b 2 ) − (a ⋅ b ) 2
(a ⋅ a ) ( x − b ) − {a ⋅ ( x − b )} a + ( b ⋅ b ) ( x − c )
Chap 02 Product of Vectors 161

182. OP ⋅ OQ + OR ⋅ OS = OR ⋅ OP + OQ ⋅ OS −1
∴ a ⋅ b + b⋅ c + c⋅ a ≥ (| a | 2 + | b | 2 + | c | 2 )
⇒ OP( OQ − OR ) + OS( OR − OQ) = 0 2
⇒ ( OP − OS )( OQ − OR ) = 0 Given, | a − b | 2 + | b − c | 2 + | c − a | 2 = 9
⇒ SP ⋅ RQ = 0 ⇒ | a | 2 + | b | 2 −2a ⋅ b + | b |2 + | c |2 −2 b⋅ c + | c |2 + | a |2 −2 c ⋅ a = 9
Similarly SR ⋅ PQ = 0 and SQ ⋅ PR = 0 ⇒ 6 − 2 (a ⋅ b + b ⋅ c + c ⋅ a ) = 9 [Q| a | = | b | = | c | =1 ]
∴ S is orthocentre. 3
⇒ a ⋅ b + b⋅ c + c⋅ a = − …(i)
183. cos(P + Q ) + cos(Q + R ) + cos(R + P ) 2
−1
= − (cos R + cos P + cosQ ) Also, a ⋅ b + b ⋅ c + c ⋅ a ≥ (| a | 2 + | b | 2 + | c | 2 )
3 2
Max. of cos P + cosQ + cos R = 3
2 ≥− …(ii)
Min. of cos( P + Q ) + cos(Q + R ) + cos( R + P ) is 2
3 From Eqs. (i) and (ii), | a + b + c | = 0
=− as a ⋅b + b ⋅c + c ⋅a is minimum when | a + b + c | = 0
2
⇒ a+ b+ c = 0
184. sin R = sin(P + Q )
∴ |2 a + 5 b + 5 c | = | 2 a + 5 ( b + c ) | = | 2 a − 5 a| =3
185. Given, | a$ | = | b$ | = | c$ | = 1 188. Let a = $i + $j + 2 k$ , b = $i + 2 $j + k$
3 $ and c = $i + $j + k$
and a$ × ( b$ × c$ ) = ( b + c$ )
2
∴ A vector coplanar to a and b and perpendicular to c
3 $
Now, consider a$ × (b$ × c) $ = ( b + c$ ) = λ (a × b ) × c = λ {(a ⋅ c ) v − (b ⋅ c ) a }
2
= λ {(1 + 1 + 4 ) ( $i + 2 $j + k$ ) − (1 + 2 + 1 ) ( $i + $j + 2 k$ )}
3 $ 3
⇒ ($a ⋅ c$ ) b$ − ($a ⋅ b$ ) c$ = b+ c$ = λ {6 i$ + 12 $j + 6 k$ − 6 $i − 6 $j − 12 k$ }
2 2
= λ {6 $j − 6 k$ } = 6λ { $j − k$ }
On comparing, we get
1
3 3 For, λ = ⇒ Option (a) is correct.
a$ ⋅ b$ = − ⇒| a$ | | b$ | cos θ = − 6
2 2
1
3 and for λ = − ⇒ Option (d) is correct.
⇒ cos θ = − [Q | a$ | = | b$ | = 1 ] 6
2
 π 5π 189. Let v = a + λb
⇒ cos θ = cos  π −  ⇒ θ =
 6 6 v = (1 + λ ) $i + (1 − λ ) $j (1 + λ ) k$
1 1
186. Given, (a × b ) × c = | b | | c | a Projection of v on c =
3 3
1 v⋅c 1
⇒ − c × (a × b ) = | b | | c | a ⇒ =
3 | c| 3
1 (1 + λ ) − (1 − λ ) − (1 + λ ) 1
⇒ − (c ⋅ b) ⋅ a + (c ⋅ a ) b = | b | c | a ⇒ =
3 3 3
 1  ⇒ 1 + λ −1 + λ −1 − λ = 1
| b | | c | + (c ⋅ b) a = (c ⋅ a ) b
3  ⇒ λ −1 = 1
Since, a and b are not collinear. ⇒ λ =2
1
c ⋅ b + | b | | c | = 0 and c ⋅ a = 0 ∴ v = 3 $i − $j + 3 k$
3
1
190. AB = 2 i$ + 10 $j + 11 k$
⇒ | c | | b | cos θ + | b | | c | = 0 AD = − $i + 2 $j + 2 k$
3
 1 D
⇒ | b | | c |  cos θ +  = 0 C
 3
1
⇒ cos θ + = 0 (Q | b | ≠ 0, | c | ≠ 0 ) α
3
1 8 2 2 θ
⇒ cos θ = − ; sin θ = =
3 3 3 A B
187. If a, b,c are any three vectors Angle ‘θ’ between AB and AD is
Then, | a + b + c| ≥ 0
2
AB ⋅ AD
cos(θ ) =
⇒ | a | 2 + | b | 2 + | c | 2 + 2 (a ⋅ b + b ⋅ c + c ⋅ a ) ≥ 0 | AB| | AD|
162 Textbook of Vector & 3D Geometry

− 2 + 20 + 22 8 17 194. The volume of the parallelopiped with coterminous edges as


= = ⇒ sin(θ ) =
(15 )(3 ) 9 9 a$ , b$ , c$ is given by [$a b$ c$ ] = a$ ⋅ ( b$ × c$ )
Since, α + θ = 90 ° Z
17
∴ cos(α ) = cos(90 ° − θ ) = sin(θ ) =
9
1 1
191. mPQ = , mSR = , mRQ = −3, mSP = − 3 c
6 6
S (− 3, 2) R (3, 3) b
Y
a

X
a$ ⋅ a$ a$ ⋅ b$ a$ ⋅ c$ 1 1/2 1/2
P (− 2, − 1) Q (4, 0) Now, [$a b c$ ] = b$ ⋅ a$ b$ ⋅ b$
$ 2
b$ ⋅ c$ = 1 / 2 1 1 / 2
c$ ⋅ a$ c$ ⋅ b$ c$ ⋅ c$ 1/2 1/2 1
⇒ Parallelogram, but neither
PR = SQ nor PR ⊥ SQ.  1 1  1 1 1  1 1 1
⇒ [$a b$ c$ ]2 = 1 1 −  −  −  +  − =
∴ So, it is a parallelogram, which is neither a rhombus nor a  4 2  2 4 2  4 2 2
rectangle. Thus, the required volume of the parallelopiped
$i − 2 $j 1
192. From the given information, it is clear that a = = cu unit
5 2
⇒ | a | = 1, | b | = 1, a ⋅ b = 0 195. Given, OP = a$ cos t + b$ sin t
Now, (2a + b ) ⋅ [(a × b ) × (a − 2 b )]
⇒ | OP | = ($a ⋅ a$ ) cos2 t + ( b$ ⋅ b$ ) sin 2 t + 2a$ ⋅ b$ sin t cos t
= (2a + b ) ⋅ [a 2b − (a ⋅ b ) ⋅ a + 2 b 2 ⋅ a −2( b ⋅ a ) ⋅ b ]
= [2a + b ] ⋅ [ b + 2a ] = 4a 2 + b 2 ⇒ | OP | = 1 + a$ ⋅ $b sin 2t
= 4 ⋅1 + 1 = 5 [as a ⋅ b = 0] π
⇒ | OP | max = M = 1 + a$ ⋅ b$ at sin 2t = 1 ⇒ t =
4
193. Let angle between a and b be θ1, c and d be θ 2 and a × b and
π 1
b × d be θ. At t = , OP = ($a + b )
$
4 2
Since, (a × b ) ⋅ ( c × d ) = 1 ⇒ sin θ1 ⋅ sinθ 2 ⋅ cosθ = 1
 π a$ + b$
⇒ θ1 = 90 °, θ 2 = 90 °, θ = 0 ° Unit vector along OP at t =  =
 4  | a$ + b$ |
⇒ a ⊥ b, c ⊥ d , (a × b )||( c × d )
So, a × b = k( c × d ) and a × b = k( c × d ) 196. Since, PQ is not parallel to TR.
⇒ (a × b ) ⋅ c = k ( c × d ) ⋅ c and (a × b ) ⋅ d = k( c × d ) ⋅ d T S

⇒ [a b c ] = 0 and [a b d ] = 0
⇒ a, b, c and a, b, d are coplanar vectors, so U R
options (a) and (b) are incorrect.
Let b || d ⇒ b =± d
Q
As (a × b ) ⋅ ( c × d ) = 1 ⇒ (a × b ) ⋅ ( c × b ) = ± 1 P

⇒ [a × b c b ] = ± 1 ⇒ [ c ba × b ] = ± 1 Q TR is resultant of RS and ST vectors.


⇒ PQ × ( RS + ST ) ≠ 0.
⇒ c ⋅[ b × (a × b )] = ± 1 ⇒ c ⋅[a − ( b ⋅a ) b ] = ± 1
But for Statement II, we have PQ × RS = 0
⇒ c ⋅a = ± 1 [Q a ⋅ b = 0]

which is not possible as PQ not parallel to RS.
Which is a contradiction, so b Hence, Statement I is true and Statement II is false.
option (c) is correct.
Let option (d) is correct. 197. Since, given vectors are coplanar
⇒ d = ± a and c = ± b − λ2 1 1
α ←
a ∴ 1 − λ2 1 =0
As (a × b ) ⋅( c × d ) = 1 60º
⇒ (a × b ) ⋅( b × a ) = ± 1 1 1 − λ2
←←
Which is a contradiction, so option (d) is d c ⇒ λ6 − 3 λ2 − 2 = 0
incorrect. ⇒ (1 + λ ) ( λ − 2 ) = 0 ⇒ λ = ± 2
2 2 2

Alternatively options (c) and (d) may be observed from the


above figure.
Chap 02 Product of Vectors 163

198. Since, a, b, c are unit vectors and a + b + c = 0, then 203. We know that, volume of parallelopiped whose edges are a , b , c
a, b, c represent an equilateral triangle. = [a b c ].
∴ a × b = b × c = c × a ≠ 0. 1 a 1
∴ [a b c ] = 0 1 a = 1 + a 3 − a
199. Let vector AO be parallel to line of intersection of planes P1
and P2 through origin. a 0 1
Normal to plane p1 is Let f (a ) = a 3 − a + 1
n1 = [(2 $j + 3 k$ ) × ( 4 $j − 3 k$ )] = −18 $i ⇒ f ′ (a ) = 3a 2 − 1 ⇒ f ′′ (a ) = 6a
Normal to plane p 2 is For maximum or minimum, put f ′ (a ) = 0
^
n 2 = ( $j − k$ ) × (3 i + 3 $j) = 3 $i − 3 $j − 3 k$ 1 1
⇒a = ± , which shows f (a ) is minimum at a = and
3 3
So, OA is parallel to ± ( n1 × n 2 ) = 54 $j − 54 k$ . 1
maximum at a = − .
∴ Angle between 54 ( $j − k$ ) and (2 i$ + $j − 2 k$ ) is 3
 54 + 108  1 204. We know that, a × (a × b ) =(a ⋅ b )a −(a ⋅a ) b
cos θ = ±   =±
 3 ⋅ 54 ⋅ 2  2
∴ ( i$ + $j + k$ ) × ( $j − k$ ) = ( $i + $j + k$ ) − ( 3 ) 2 b
π 3π
∴ θ= , ⇒ − 2 i$ + $j + k$ = i$ + $j + k$ − 3 b ⇒ 3 b = + 3 $i
4 4
Hence, (b) and (d) are correct answers. ∴ b = $i
200. Let vector r be coplanar to a and b. 205. Given, V =2$i + $j − k$ and W = i$ + 3k$
∴ r =a + t b [ U V W ] = U ⋅[( 2 $i + $j − k$ ) × ( $i + 3 k$ )]
⇒ r = ( i$ + 2 $j + k$ ) + t ( $i − $j + k$ ) = U ⋅(3 $i − 7 $j − k$ ) = | U || 3 $i − 7 $j − k$ | cosθ
= (1 + t ) $i + (2 − t ) $j + (1 + t ) k$ Which is maximum, if angle between U and 3 i$ − 7 $j − k$ is 0
1 and maximum value
The projection of r on c = ⋅ [given]
3 = | 3 $i − 7 $j − k|
$ = 59
r⋅c 1 206. Since, (a + 2 b ) ⋅ (5a − 4a ) = 0
⇒ =
| c| 3
⇒ 5 | a | 2 + 6a ⋅ b − 8 | b | 2 = 0
| 1 ⋅ (1 + t ) + 1 ⋅ (2 − t ) − 1 ⋅ (1 + t ) | 1
⇒ = ⇒ 6a ⋅ b = 3 [Q| a | = | b | =1 ]
3 3
1
⇒ (2 − t ) = ± 1 ⇒ t = 1 or 3 ⇒ cosθ = ⇒ θ = 60 °
2
When, t = 1, we have r = 2 i$ + $j + 2 k$
207. We have, a = 2$i + $j − 2k$
When, t = 3, we have r = 4 i$ − $j + 4 k$
⇒ |a| = 4 + 1 + 4 =3
b ⋅a b ⋅a
201. Since, b1 = b − a, b1 = b + 2 a and b = $i + $j ⇒| b | = 1 + 1 = 2
| a| 2 | a|
c ⋅a c⋅b c ⋅a c ⋅ b1 Now, | c − a | = 3 ⇒| c − a | 2 = 9
and c = c − 2 a − 2 b, c 2 = c − 2 a − b1
|a | | b| |a | | b |2 ⇒ (c − a ) ⋅ (c − a ) = 9
c ⋅a c ⋅ b2 c ⋅a ⇒ | c | 2 + | a | 2– 2 c ⋅ a = 9 …(i)
c3 = c − a− b 2, c 4 = a − 2 a. Again, |a × b| × c| =3
|a |2 | b |2 |a |
which shows a ⋅ b1 = 0 = a ⋅ c = b1 ⋅ c 2 6
⇒ | a × b | | c | sin 30 ° = 3 ⇒| c | =
So, {a, b1, c 2 } are mutually orthogonal vectors. |a × b|
202. As we know that, a vector coplanar to a, b and orthogonal to c $i $j k$
is λ {(a × b ) × c} . But a × b = 2 1 −2 = 2 $i − 2 $j + k$
∴ A vector coplanar to (2i$ + $j + k$ ), ( $i − $j + k$ ) and orthogonal 1 1 0
to 3 $i + 2 $j + 6 k$ 6
∴ | c| =2 …(ii)
= λ [{(2i$ + $j + k$ ) × ( $i − $j + k$ )} × (3 $i + 2 $j + 6 k$ )] 4+4+1
= λ [(2 i$ − $j − 3 k$ ) × (3 $i + 2 $j + 6 k$ )] = λ (21 $j − 7 k)
$ From Eqs. (i) and (ii), we get
(21 $j − 7 k$ ) (2 ) 2 + (3 ) 2 − 2 c ⋅ a = 9 ⇒ 4 + 9 − 2 c ⋅ a = 9 ⇒ c ⋅ a = 2
∴ Unit vector = +
(21 ) 2 + (7 ) 2 208. Use the formulae, a × (b × c ) = (a ⋅ c )b − (a ⋅ b ) c,
(3 $j − k$ ) [a b c ] = [ b c a ] = [ c a b ]
=+
10 and [a a b ] = [a b b ] = [ a c c ] = 0
Further, simplify it and get the result.
164 Textbook of Vector & 3D Geometry

Now, [a × b b × c c × a ] 211. a =
1 1
(3 $i + k$ ) and b = (2 $i + 3 $j − 6 k$ )
= a × b ⋅ (( b × c ) × ( c × a )) 10 7
= a × b ⋅ (( k × c × a )) [Here, k = b × c] ∴ (2a − b ) ⋅ {(a × b ) × (a + 2 b )}
= a × b ⋅ [( k ⋅ a ) c − ( k ⋅ c ) a ] = (2a − b ) ⋅ {(a × b ) × a + (a × b ) × 2 b }
= (a × b ) ⋅ (( b × c ⋅ a ) c − ( b × c ⋅ c ) a ) = (2a − b ) ⋅ {(a ⋅ a ) b − ( b ⋅ a ) a
= (a × b ) ⋅ ([ b c a ] c ) − 0 [Q[ b × c ⋅ c ] = 0] + 2 (a ⋅ b ) b − 2 ( b ⋅ b ) a }
= a × b ⋅ c [ b c a ] = [a b c ] [ b c a ] = (2a − b ) ⋅ {1 ( b ) − ( 0 ) a + 2 ( 0 ) b − 2 (1 ) a }
= [a b c ] 2 {Q[a b c ] = [ b c a ]} [as a ⋅ b = 0 and a ⋅ a = b ⋅ b = 1]
= (2a − b ) ( b − 2a )
Hence, [a × b b × c c × a ] = λ [ a b c ] 2
= − (4 | a | 2 − 4 a ⋅ b + | b| 2 ) = − { 4 − 0 + 1 } = − 5
⇒ [a b c ] 2 = λ [a b c ] 2
212. Given, a ⋅ b ≠ 0, a ⋅ d = 0 …(i)
⇒ λ =1 and b×c=b×d
209. Given that, ⇒ b × (c − d ) = 0
(i) a and b are unit vectors, ∴ b || ( c − d )
i.e. | a | = | b | = 1 ⇒ c − d = λb
(ii) c = a + 2 b and d = 5 a − 4 b ⇒ d = c − λb …(ii)
(iii) c and d are perpendicular to each other. Taking dot product with a, we get
i.e. c ⋅ d = 0 a ⋅ d = a ⋅ c − λa ⋅ b
To find Angle between a and b. ⇒ 0 = a ⋅ c − λ (a ⋅ b )
Now, c ⋅ d = 0 ⇒ (a + 2 b ) ⋅ (5 a − 4 b ) = 0 a⋅c
∴ λ= …(iii)
a⋅b
⇒ 5 a ⋅ a − 4 a ⋅ b + 10 b ⋅ a − 8 b ⋅ b = 0
(a ⋅ c )
⇒ 6a⋅b =3 ∴ d=c− b
(a ⋅ b )
1
⇒ a⋅b = 213. Given, a = p i$ + $j + k$ , b = i$ + q$j + k$ and c = $i + $j + r k$ are
2
π coplanar and p ≠ q ≠ r ≠ 1.
So, the angle between a and b is . Since, a, b and c are coplanar.
3
⇒ [a b c ] = 0
210. Given, p 1 1
(i) A parallelogram ABCD such that AB = q and AD = p. ⇒ 1 q 1 =0
(ii) The altitude from vertex B to side AD coincides with a 1 1 r
vector r.
To find The vector r in terms of p and q. ⇒ p (qr − 1 ) − 1 (r − 1 ) + 1 (1 − q ) = 0
Let E be the foot of perpendicular from B to side AD. ⇒ pqr − p − r + 1 + 1 − q = 0
q⋅ p ∴ pqr − ( p + q + r ) = − 2
AE = Projection of vector q on p = q ⋅ p =
|p| 214. We have, a × b + c = 0
D
⇒ a × (a × b ) + a × c = 0
C
⇒ (a ⋅ b ) a − (a ⋅ a ) b + a × c = 0
⇒ 3a − 2 b + a × c = 0
E
p r ⇒ 2 b = 3a + a × c
⇒ 2 b = 3 $j − 3 k$ − 2 $i − $j − k$ = − 2 i$ + 2 $j − 4 k$
A B
q ∴ b = − i$ + $j −2 k$
AE = Vector along AE of length AE 215. Since, the given vectors are mutually orthogonal, therefore
 q ⋅ p (q ⋅ p ) p a ⋅b = 2 − 4 + 2 = 0
= | AE | AE =   p= a ⋅c = λ − 1 + 2µ = 0 ...(i)
|p| | p |2
and b ⋅c = 2 λ + 4 + µ = 0 ...(ii)
Now, applying triangles law in ∆ABE, we get On solving Eqs. (i) and (ii), we get
AB + BE = AE µ = 2 and λ = −3
(q ⋅ p ) p (q ⋅ p ) p ( λ , µ ) = ( −3, 2 )
⇒ q+ r = ⇒ r= −q Hence,
| p |2 | p |2
216. Since, [3u p v p w ] − [ p v w q u ] − [2w qv qu ] = 0
 q ⋅ p ∴ 3 p 2 [u ⋅ ( v × w )] − pq [ v ⋅ ( w × u )]
⇒ r=−q+  p
 p ⋅ p − 2q 2 [ w ⋅ ( v × u )] = 0
Chap 02 Product of Vectors 165

⇒ (3 p 2 − pq + 2q 2 ) [ u ⋅ ( v × w )] = 0 ⇒ {(a − 2 ) $i − 2 $j } ⋅ {(a − 1 ) $i + 6 k$ } = 0
But [u v w] ≠ 0 ⇒ (a − 2 )(a − 1 ) = 0
⇒ 3 p 2 − pq + 2q 2 = 0 ∴ a = 1 and a = 2
∴ p =q = 0 222. Line is parallel to plane as
217. Given that, b = $i + $j and c = $j + k$ . ( i$ − $j + 4 k$ ) ⋅ ( $i + 5 $j + k$ ) = 1 − 5 + 4 = 0
The equation of bisector of b and c is General point on the line is
 i$ + $j $j + k$  ( λ + 2 , − λ − 2 , 4 λ + 3 ).
r = λ( b + c ) = λ  + 
 2 2  For λ = 0, a point on this line is (2, –2, 3) and distance from
λ $ r ⋅ ( i$ + 5 $j + k$ ) = 5 or x + 5y + z = 5 is
= ( i + 2 $j + k$ ) …(i)
2 2 + 5 ( −2 ) + 3 − 5 −10  10
Since, vector a lies in plane of b and c. d =  ⇒ d = =
 1 + 25 + 1  3 3 3 3
∴ a = b + µc
λ $ 223. Let a = a1 $i + a 2$j + a 3 k$
⇒ ( i + 2 $j + k$ ) = ( i$ + $j) + µ( $j + k$ )
2 Then, a × $i = − a 2k$ + a 3 $j
On equating the coefficient of $i both sides, we get a × $j = a1 k$ − a 3 $i
λ
=1 ⇒ λ = 2 a × k$ = − a1 $j + a 2$i
2
On putting λ = 2 in Eq. (i), we get ∴ (a × $i ) 2 + (a × $j) 2 + (a × k$ ) 2
r = $i + 2 $j + k$ = a 22 + a 23 + a 12 + a 23 + a 12 + a 22
Since, the given vector a represents the same bisector equation = 2 (a 12 + a 22 + a 23) = 2 a 2
r$. 224. Given that, [ λ(a + b ) λ2b λ c ] = [a b + c b ]
∴ α = 1 and β = 1
λ (a1 + b1 ) λ (a 2 + b2 ) λ (a 3 + b3 )
218. Since, (2u × 3v ) is a unit vector. ∴ λ2b1 λ2b2 λ2b3
⇒ | 2u × 3v | = 1
λc1 λc 2 λc 3
⇒ 6 | u || v || sin θ | = 1
1 a1 a2 a3
⇒ sin θ = [Q | u | = | v| = 1 ]
6 = b1 + c1 b2 + c 2 b3 + c 3
Since, θ is an acute angle, then there is exactly one value of θ b1 b2 b3
for which (2 u × 3 v ) is a unit vector.
a1 a 2 a 3 a1 a 2 a 3
219. Since, given vectors v, b and c are coplanar. ⇒ λ4 b1 b2 b3 = − b1 b2 b3
1 1 1 c1 c 2 c 3 c1 c 2 c 3
∴ 1 −1 2 =0
⇒ λ4 = − 1
x x − 2 −1
So, no real value of λ exists.
⇒ 1 {1 − 2 ( x − 2 )} − 1 ( −1 − 2 x ) + 1 ( x − 2 + x ) = 0
225. Given, vectors are
⇒ 1 − 2x + 4 + 1 + 2x + 2x − 2 = 0
a = i$ − k$ , b = x i$ + $j + (1 − x ) k$
⇒ 2x = − 4 ⇒ x = −2
and c = y i$ + x $j + (1 + x − y ) k$
220. Since, (a × b ) × c = a × ( b × c )
1 0 −1
∴ (a ⋅c ) b − ( b ⋅c ) a = (a ⋅c ) b − (a ⋅b ) c
⇒ ( b ⋅ c ) a = (a ⋅ b ) c ∴ [a b c ] = x 1 1−x
(a ⋅ b ) y x 1 + x −y
⇒ a= ⋅c
(b ⋅ c) Applying C 3 → C 3 + C1, we get
Hence, a is parallel to c. 1 0 0
221. Since, position vectors of A, B, C are 2 i$ − $j + k$ , i$ − 3$j − 5k$ and = x 1 1 = 1(1 + x ) − x = 1
a i$ − 3 $j + k$ , respectively. y x 1+ x
Now, AC = (a $i − 3 $j + k$ ) − (2 $i − $j + k$ ) Thus, [a b c ] depends upon neither x nor y.
= (a − 2 ) i$ − 2 $j 226. Since, | u | = 1, | v | = 2, | w | = 3
and BC = (a $i − 3 $j + k$ ) − ( $i − 3 $j − 5 k$ ) v ⋅u
The projection of v along u =
= (a − 1 ) i$ + 6 k$ |u |
Since, the ∆ABC is right angled at C, then w ⋅u
and the projection of w along u =
AC ⋅ BC = 0 |u |
166 Textbook of Vector & 3D Geometry

According to given condition, 231. Vector perpendicular to face OAB is n1.


v ⋅u w ⋅u Y
=
| u| | u|
B(2, 1, 3)
⇒ v ⋅u = w ⋅u …(i)
Since, v, w are perpendicular to each other.
∴ v ⋅w = 0 …(ii)
Now, | u −v + w | 2 = | u | 2 + | v | 2 + | w | 2
X
−2 u ⋅v − 2 v ⋅w + 2 u ⋅w O A (1, 2, 1)
⇒ | u − v + w | 2 = 1 + 4 + 9 − 2 u ⋅ v + 2 v ⋅u
[from Eqs. (i) and (ii)]
C (–1, 1, 2)
⇒ | u −v + w | 2 = 1 + 4 + 9 Z
$i $j k$
⇒ | u − v + w | = 14
1 = OA × OB = 1 2 1 = 5 $i − $j − 3 k$
227. Given that, | b || c | a = (a × b ) × c 2 1 3
3
We know that, Vector perpendicular to face ABC is n 2
(a × b ) × c = (a ⋅ c ) b − ( b ⋅ c ) a $i $j k$
1
∴ | b || c | a = (a ⋅ c ) b − ( b ⋅c ) a = AB × AC = 1 −1 2 = i$ − 5 $j − 3 k$
3
−2 −1 1
On comparing the coefficients of a and b, we get
1
| b || c | = − b ⋅ c and a ⋅ c = 0 Since, angle between faces is equal to the angle between their
3 normals.
1
⇒ | b | | c | = − | b || c | cos θ n ⋅n 5 × 1 + ( −1 ) × ( −5 ) + ( −3 ) × ( −3 )
3 ∴ cos θ = 1 2 =
1 1 | n1|| n 2| 5 + ( −1 ) 2 + ( −3 ) 2 1 2 + ( −5 ) 2 + ( −3 ) 2
2
⇒ cos θ = − ⇒ 1 − sin 2 θ =
3 9 5 + 5 + 9 19  19
8 = = ⇒ θ = cos−1  
⇒ sin θ =
2
35 35 35  35
9
2 2  π 232. Given that, u = $i + $j , v = $i −$j , w = i$ +2 $j +3 k$ ,
∴ sin θ = Q 0 ≤θ ≤
3  2  u ⋅ n = 0 and v ⋅ n = 0
u× v
228. Total force, F = ( 4 i$ + $j − 3 k$ ) + (3 $i + $j − k$ ) i.e. n=
| u × v|
∴ F = 7 $i + 2 $j − 4 k$
$i $j k$
The particle is displaced from A( $i + 2 $j + 3 k$ ) to B (5 $i + 4 $j + k$ ).
Now, u × v = 1 1 0 = 0 $i − 0 $j − 2 k$ = −2 k$
Now, displacement,
1 −1 0
AB = (5 $i + 4 $j + k$ ) − ( $i + 2 $j + 3 k$ ) = 4 $i + 2 $j − 2 k$
∴Work done = F ⋅ AB | w ⋅ u × v | | − 6 k$ |
∴ | w ⋅ n| = = =3
= (7 i$ + 2 $j − 4 k$ ) ⋅ ( 4 $i + 2 $j − 2 k$ ) |u × v | | − 2 k$ |
= 28 + 4 + 8 = 40 units [Q w ⋅ ( u × w ) = ( $i + 2 $j + 3 k$ ) ⋅ ( − 2 k$ ) = − 6 k$ ]
229. (u + v − w ) ⋅ [(u − v ) × (v − w )] Hence, | w ⋅ n| = 3
= (u + v − w ) ⋅ [ u × v − u ×w − v × v + v ×w ] 233. Given two vectors lie in XY-plane. So, a vector coplanar with
= u ⋅ ( u × v )− u ⋅ ( u × w )+ u ⋅ ( v × w ) + v ⋅ ( u × v ) them is a = x $i + y $j
− v ⋅ ( u × w ) + v ⋅ ( v × w ) − w ⋅ ( u × v ) + w ⋅( u × w ) − w ⋅( v × w ) Since, a ⊥ ( i$ − $j)
= u ⋅v × w − v ⋅u × w − w ⋅u × v {[a, a b] = 0} ⇒ ( x $i + y $j) ⋅ ( $i − $j) = 0
= u ⋅v × w + w ⋅u × v − w ⋅u × v = u ⋅v × w ⇒ x −y = 0
230. Given that, | a | = 1,| b | = 2,| c | = 3 ⇒ x =y
and a +b +c = 0 ∴ a = x $i + x $j
Now, (a + b + c ) 2 = | a | 2 + | b | 2 + | c | 2 + 2(a ⋅ b + b ⋅ c + c ⋅ a ) and 4| a| = x 2 + x 2 = x 2
⇒ 0 = 1 + 2 + 3 + 2 (a ⋅ b + b ⋅ c + c ⋅ a )
2 2 2
∴ Required unit vector
⇒ 2 (a ⋅ b + b ⋅ c + c ⋅ a ) = − 14 a x( $i + $j ) 1 $ $
= = = ( i + j)
⇒ a ⋅ b + b ⋅ c + c ⋅ a = −7 |a | x 2 2
CHAPTER

03
Three Dimensional
Coordinate System
Learning Part
Session 1
● Introduction

● Position Vector of a Point in Space

● Shifting of Origin ● Distance Formula ● Section Formula

● Direction Cosines and Direction Ratios of a Vector

● Projection of the Line Segment Joining Two Points on a Given Line

Session 2
● Equation of a Straight Line in Space

● Angle between Two Lines ● Perpendicular Distance of a Point from a Line

● Shortest Distance between Two Lines

Session 3
● Plane

● Equation of Plane in Various Form

● Angles between Two Planes ● Family of Planes

● Two Sides of a Plane ● Distance of a Point from a Plane

● Equation of Planes Bisecting the Angle between Two Planes

● Line and Plane

Session 4
● Sphere

Practice Part
● JEE Type Examples

● Chapter Exercises

Arihant on Your Mobile !


Exercises with the #L
symbol can be practised on your mobile. See inside cover page to activate for free.
Session 1
Introduction, Position Vector of a Point in Space, Shifting
of Origin, Distance Formula, Section Formula,Direction
Cosines and Direction Ratios of a Vector, Projection of the
Line Segment Joining Two Points on a Given Line
Introduction Z

Let OY and OZ be two perpendicular lines which intersect C


N
at O and let a third straight line OX be perpendicular to k
the plane in which they lie. The three mutually L P (x, y, z)
perpendicular lines form a set of coordinate axis. They z r
j
determine three mutually perpendicular planes called x O y B
Y
coordinate planes. i
A M
Z X

C
N
Signs of Coordinates of a Point in Various Octants
k Octant/
OXYZ OX ′ YZ OXY ′ Z OXYZ OX ′ YZ OX ′ YZ ′ OXY ′ Z ′ OX ′ Y ′ Z ′
L Coordinates
P (x, y, z)
r x + − + + − − + −
z
j y + + − + − + − −
x O y Y
B z + + + − + − − −
i
A M
X Note
Any point on X-axis = ( x, 0, 0 )
Remarks Y-axis = ( 0, y, 0 )
1. The axes to coordinate form a right handed set (in the figure) Z-axis = ( 0, 0, z )
i.e. a right handed screw, driven from O to X would rotate in the XY-plane =( x, y, 0 )
sense from OY to OZ. YZ-plane = ( 0, y, z )
2. The points A, B and C are the orthogonal projections of P on the ZX-plane = ( x, 0, z )
X, Y and Z-axes. ∴ OP = x 2 + y2 + z2
3. Points L, M and N are (x, 0, z), ( x, y, 0 ), ( 0, y, z ) and A, B and C
are ( x, 0, 0 ), ( 0, y, 0 ), ( 0, 0, z ), respectively. y Example 1. Planes are drawn parallel to the coordinate
planes through the points (1, 2, 3) and ( 3, − 4, − 5). Find
Position Vector of a Point the lengths of the edges of the parallelopiped so formed.
Sol. Let P = (1, 2, 3), Q = (3,−4,−5) through which planes are
in Space drawn parallel to the coordinate planes shown as,
Z
Let $i, $j, k
$ be unit vector (called base vector) along OX, OY E D
and OZ, respectively.
F Q(3, –4, –5)
Let P ( x , y , z ) be a point in space, let the position of P be r.
P (1, 2, 3)
Then, r = OP = OM + MP
C
= (OA + AM) + MP = OA + OB + OC
A B
r = x$i + y$j + zk
$
O Y
Thus, the position vector of a point P is, x$i + y$j + zk
$.
X
Chap 03 Three Dimensional Coordinate System 169

∴PE = Distance between parallel planes ABCP and FQDE, Proof. Let O be the origin and let P ( x 1 , y 1 , z 1 ) and
i.e. (along Z-axis) Q ( x 2 , y 2 , z 2 ) be two given points.
= | –5 − 3 | = 8
Z
PA = Distance between parallel planes ABQF and PCDE,
i.e. (along X -axis) Q (x2, y2, z2)
=|3 −1| =2
PC = Distance between parallel planes BCDQ and APEF ,
i.e. (along Y -axis) P (x1, y1, z1)
=| − 4 −2| =6
∴ Lengths of edges of the parallelopiped are 2, 6, 8.
Y
O
Shifting of Origin X
Shifting the origin to another point without changing the
The, OP = x 1 $i + y 1 $j + z 1 k
$
directions of the axes is called the translation of axes.
Let the origin O(0, 0, 0) be shifted to another point OQ = x 2 $i + y 2 $j + z 2 k
$
O ′ ( x ′ , y ′ , z ′ ) without changing the direction of axes. Let Now, PQ = Position vector of Q − Position vector of P
the new coordinate frame be O ′ X ′ Y ′ Z ′. Let P ( x , y , z ) be a = OQ − OP
point with respect to the coordinate frame OXYZ. Then,
= ( x 2 $i + y 2 $j + z 2 k
$ ) − ( x 1 $i + y 2 $j + z 1 k
$)
coordinate of point P with respect to new coordinate
frame O ′ X ′ Y ′ Z ′ is ( x 1 , y 1 , z 1 ), where x 1 = x − x ′, = ( x 2 − x 1 ) $i + (y 2 − y 1 ) $j + (z 2 − z 1 ) k $
y 1 = y − y ′, z 1 = z − z ′ ∴ PQ = | PQ |

P (x, y, z)
= ( x 2 − x 1 ) 2 + (y 2 − y 1 ) 2 + (z 2 − z 1 ) 2
Z
Hence, PQ = ( x 2 − x 1 ) 2 + (y 2 − y 1 ) 2 + (z 2 − z 1 ) 2

Y¢ y Example 3. Find the distance between the points


O¢(x1¢ , y1¢ , z1¢ )
P( − 2, 4, 1) and Q(1, 2, − 5).
Y
O
Sol. We have, PQ = (1 + 2)2 + (2 − 4 )2 + ( − 5 − 1)2

PQ = 3 2 + ( − 2 ) 2 + ( − 6 ) 2
X = 9 + 4 + 36
= 49 = 7
y Example 2. If the origin is shifted (1, 2, − 3) without
changing the directions of the axis, then find the new y Example 4. Prove by using distance formula that the
coordinates of the point (0, 4, 5) with respect to new points P(1, 2, 3), Q( − 1, − 1, − 1) and R( 3, 5, 7 ) are collinear.
frame.
Sol. We have,
Sol. In the new frame x ′ = x − x 1, y ′ = y − y1, z ′ = z − z1, where
PQ = ( − 1 − 1 ) 2 + ( − 1 − 2 ) 2 + ( − 1 − 3 ) 2
( x 1, y1, z1 ) is shifted origin.
⇒ x′ = 0 − 1 = − 1, = 4 + 9 + 16 = 29
y′ = 4 − 2 = 2, z′ = 5 + 3 = 8
QR = (3 + 1 ) 2 + (5 + 1 ) 2 + (7 + 1 ) 2
Hence, the coordinates of the point with respect to the new
coordinates frame are ( − 1, 2, 8 ). = 16 + 36 + 64

= 116 = 2 29

Distance Formula and PR = (3 − 1 ) 2 + (5 − 2 ) 2 + (7 − 3 ) 2

The distance between the points P ( x 1 , y 1 , z 1 ) and = 4 + 9 + 16 = 29


Q ( x 2 , y 2 , z 2 ) is given by Since, QR = QP + PR
PQ = ( x 2 − x 1 ) 2 + (y 2 − y 1 ) 2 + (z 2 − z 1 ) 2 Therefore, the given points are collinear.
170 Textbook of Vector & 3D Geometry

⇒ nPR = mQR
Section Formula ⇒ n(OR − OP ) = m(OR − OQ )
1. Section Formula for Internal Division ⇒ n( r − r1 ) = m( r − r2 )
Let P and Q be two points whose position vectors are r1 ⇒ (m − n ) r = mr2 − nr1
and r2 respectively. Let R be a point on PQ dividing it in  mr − nr1 
the ratio m : n. Then, the position vector of R is given by ⇒ r = 2 
 m −n 
mr2 + nr1
r= .
m+ n R

R
m:n
P Q
r

r1 r Q
r2
r2
r1
P
O
O
Corollary 1. If R( x , y , z ) is a point dividing the join of
Proof. Let O be the origin. Then, OP = r1 , OQ = r2 and P ( x 1 , y 1 , z 1 ) and Q ( x 2 , y 2 , z 2 ) in the ratio m : n.
OR = r mx 2 + nx 1 my 2 + ny 1 mz + nz 1
Then, x = ,y = ,z = 2
Now,
PR m
= m +n m +n m +n
RQ n Corollary 2. The coordinates of the mid-point of the joint
⇒ nPR = mRQ of P ( x 1 , y 1 , z 1 ) and Q ( x 2 , y 2 , z 2 ) are
⇒ n(OR − OP ) = m (OQ − OR )  x 1 + x 2 y1 + y2 z1 + z2 
 , , .
⇒ n( r − r1 ) = m( r2 − r )  2 2 2 
⇒ (m + n ) r = mr2 + nr1 Corollary 3. The coordinates of a point R which divides
mr + nr1 the join of P ( x 1 , y 1 , z 1 ) and Q ( x 2 , y 2 , z 2 ) externally in the
⇒ r= 2 ratio m : n are
m +n
 mx 2 − nx 1 my 2 − ny 1 mz 2 − nz 1 
 , , 
Corollary  m −n m −n m −n 
Mid-point formula Let P and Q be two points whose Corollary 4. The coordinate of centroid of triangle with
position vectors are given by r1 and r2 respectively. Then, vertices ( x 1 , y 1 , z 1 ), ( x 2 , y 2 , z 2 ), ( x 3 , y 3 , z 3 ) is
the position vector of the mid-point R of PQ is given by,
 x 1 + x2 + x 3 y1 + y2 + y 3 z1 + z2 + z 3 
r + r2  , , .
r= 1 .  3 3 3 
2
Corollary 5. Centroid of tetrahedron with vertices
2. Section Formula for External Division ( x 1 , y 1 , z 1 ), ( x 2 , y 2 , z 2 ), ( x 3 , y 3 , z 3 ), ( x 4 , y 4 , z 4 ) is
Let P and Q be the points whose position vectors are r1  x1 + x2 + x 3 + x 4 y1 + y2 + y 3 + y 4
 , ,
and r2 respectively. Let R be a point on PQ dividing it  4 4
externally in the ratio m : n. Then, the position vector of R z1 + z2 + z 3 + z 4 
.
is given by, 4 
mr − nr1
r= 2 y Example 5. Find the ratio in which 2x + 3y + 5z = 1
m −n
divides the line joining the points (1, 0, − 3) and
Proof. Let O be the origin. Then, OP = r1 , OQ = r2 and (1, − 5, 7 ).
OR = r
Sol. Let 2x + 3y + 5z = 1 divides (1, 0, − 3) and (1, − 5, 7 ) in the
PR m
Now, = ratio of k : 1 at point P.
QR n
Chap 03 Three Dimensional Coordinate System 171

 k + 1 − 5k 7k − 3 Remark
Then, P =  , ,  which must satisfy
k + 1 k + 1 k + 1  Students are advised to learn above result as a formula i.e.
2 x + 3y + 5z = 1 ax + by + cz + d = 0 divides join of ( x1, y1, z1 ) and ( x 2, y2, z2 ) in
( ax1 + by1 + cz1 + d )
 k + 1  − 5k   7k − 3 ratio of − .
⇒ 2  +3  +5  =1 ( ax 2 + by2 + cz2 + d )
 k + 1  k + 1  k + 1
⇒ 2k + 2 − 15k + 35k − 15 = k + 1 y Example 8. Find the ratio in which the join of (2, 1, 5),
⇒ 21k = 14 ⇒ k =
2 ( 3, 4, 3) is divided by the plane 2x + 2y − 2z − 1 = 0.
3 Sol. Using above result,
∴ 2 x + 3y + 5z = 1, divides (1, 0, − 3 ) and (1, − 5, 7 ) in the ratio {(2 ( 2 ) + 2 (1 ) − 2 (5 ) − 1 }
of 2 : 3. Required ratio =
{2 (3 ) + 2 ( 4 ) − 2 (3 ) − 1 }
y Example 6. If A( 3, 2, − 4 ), B( 5, 4, − 6 ) and C(9, 8, − 10) {6 − 11 } –5
= =
{14 − 7 } 7
are three collinear points, then find the ratio in which
point C divides AB. ⇒2 x + 2y − 2z − 1 = 0 divides (2, 1, 5 ) and (3, 4, 5 ) externally in
ratio of 5 : 7.
Sol. Let C divide AB in the ratio λ : 1. Then,
 5λ + 3 4λ + 2 − 6λ − 4
C ≡  = (9, 8, − 10 )
, ,
 λ +1 λ + 1 λ + 1  Direction Cosines and
Comparing, 5 λ + 3 = 9 λ + 9 or 4 λ = − 6
∴ λ=−
3 Direction Ratios of a Vector
2
1. Direction Cosines (DC’s)
Also, from 4 λ + 2 = 8 λ + 8 and − 6 λ − 4 = − 10 λ − 10, we get
the same value of λ. If α, β and γ are the angles which a vector OP makes with
∴ C divides AB in the ratio 3 : 2 externally. the positive directions of the coordinate axes OX, OY and
OZ respectively. Then cos α, cos β and cos γ are known as
y Example 7. Show that the plane ax + by + cz + d = 0 direction cosines of OP and are generally denoted by
divides the line joining ( x 1 , y 1 , z 1 ) and ( x 2 , y 2 , z 2 ) in letters l, m and n, respectively.
 ax 1 + ay 1 + cz 1 + d  Z
the ratio of  − .
 ax 2 + by 2 + cz 2 + d 
Sol. Let the plane ax + by + cz + d = 0 divides the line joining P (x, y, z)
( x 1,y1, z1 ) and ( x 2 , y 2 , z 2 ) in the ratio k : 1 as shown in
figure.
g

b
k :1 O Y
a
A P B
(x1, y1, z1) (x2, y2, z2)
X
ax+by+cz+d = 0
Thus, l = cos α; m = cos β ; n = cos γ. The angles α, β and γ
 kx + x1 ky 2 + y1 kz 2 + z1  are known as direction angles and they satisfy the
∴ Coordinates of P  2 , , 
 k+1 k+1 k+1  condition 0 ≤ α, β, γ ≤ π.
must satisfy ax + by + cz + d = 0 It can be seen form the figure
 kx + x1   ky 2 + y1   kz 2 + z1  x
i.e., a  2  + b  + c +d =0 cos α =
 k+1   k+1   k+1  OP
⇒a(kx 2 + x1 ) + b(ky 2 + y1 ) + c(kz 2 + z1 ) + d (k + 1 ) = 0 y
Similarly, cos β =
⇒ k(ax 2 + by 2 + cz 2 + d ) + (ax1 + by1 + cz1 + d ) = 0 OP
(ax1 + by1 + cz1 + d ) z
⇒ k=− cos γ =
(ax 2 + by 2 + cz 2 + d ) OP
172 Textbook of Vector & 3D Geometry

P 1
⇒ λ =±
a + b2 + c2
2

a
So, l=± ,
a2 + b2 + c2
a
x X b
O
m=± ,
Where, OP is the modulus of positive vector of P. a + b2 + c2
2

OP = x 2 + y 2 + z 2 c
Clearly, n=±
a + b2 + c2
2
So, l 2 + m 2 + n 2 = cos 2 α + cos 2 β + cos 2 γ
For example, let the direction ratios of a point be (3, 1, − 2 ).
x2 +y2 +z2 x2 +y2 +z2
= = =1 ⇒ Direction cosines are
OP 2 x2 +y2 +z2  
 3 1 −2 
∴ l 2 +m 2 + n 2 = cos 2 α + cos 2 β + cos 2 γ = 1  3 2 + 12 + ( − 2 ) 2
, ,
2 
 3 2
+ 12
+ ( − 2 ) 2
3 2
+ 12
+ ( − 2 ) 
∴ If OP = r = x$i + y$j + zk
$
 3 1 2 
Then, r$ = l $i + m $j + n k
$ ⇒ , ,− 
 14 14 14 
By definition it follows that the direction cosine of the
axis x are cos 0°, cos 90°, cos 90°, i.e. (1, 0, 0). 3. Angle between Two Vectors in Terms of
Similarly, direction cosine of the axes Y and Z are (0, 1, 0 ) Direction Cosines and Direction Ratios
and (0, 0, 1), respectively. Let a and b be two given vectors with direction cosines l 1 ,
m 1 , n1 and l 2 , m 2 , n2 respectively. Then,
2. Direction Ratios (DR’s)
a = l 1 $i + m 1 $j + n1 k
$ and b = l 2 $i + m 2 $j + n2 k
$
Let l, m and n be the direction cosines of a vector r and a, b
and c be three numbers such that a, b and c are a⋅ b
∴cos θ = , where θ be the angle between a and b.
proportional to l, m and n | a | ⋅| b |
l m n
= = =k | l $i + m 1 $j + n1 k $ | ⋅ | l 2 $i + m 2 $j + n2 k
$|
i.e. ⇒ cos θ = 1
a b c | l 1 $i + m 1 $j + n1 k
$ | | l 2 $i + m 2 $j + n2 k $|
or (l, m, n ) = (ka, kb, kc )
l 1 l 2 + m 1m 2 + n1 n2
⇒ (a, b, c ) are direction ratios. ⇒ cos θ =
l 12 +m 12 + n12 l 22 + m 22 + n22
 1 −1 1 
If  , ,  are direction cosines of a vector r, then
 3 3 3 ⇒ cos θ = l 1 l 2 + m 1m 2 + n1 n2 [Ql 2 + m 2 + n 2 = 1]
its direction ratios are (1, − 1, 1) or ( − 1, 1, − 1) or (2, − 2, 2 ) or Also, sin2 θ = 1 − cos 2 θ
( λ, − λ, λ)... etc.
= (l 12 + m 12 + n12 ) (l 12 + m 22 + n22 )
It is evident from the above definition that to obtain
direction ratios of a vector from its direction cosines, we − (l 1 l 2 + m 1m 2 + n1 n2 ) 2
just multiply them by a common number.
⇒ sin2 θ = (m 1 n2 − m 2 n1 ) 2 + (n1 l 2 − n2 l 1 ) 2 + (l 1m 2 − l 2m 1 ) 2
“That shows there can be infinitely many direction ratios
for a given vector but the direction cosines are unique”. (m 1 n2 − m 2 n1 ) 2 + (n1 l 2 − n2 l 1 ) 2
⇒ sin θ = ±
To obtain direction cosines from direction ratios. + (l 1m 2 − l 2m 1 ) 2
Let a, b and c be direction ratios of a vector r having
direction cosines l, m and n. Then, Remarks
1. Acute angle θ between the two lines having direction cosines
l = λa, m = λb , n = λc (by definition)
l1, m1, n1 and l2, m2, n2 is given by
∴ l 2 + m 2 + n2 = 1 cos θ = | l1l2 + m1m2 + n1n2 |
sin θ = ( l1m2 − l2m1 ) 2 + ( m1n2 − m2n1 ) 2 + ( n1l2− n2l1 ) 2
⇒ a 2 λ2 + b 2 λ2 + c 2 λ2 = 1
Chap 03 Three Dimensional Coordinate System 173

2. If a1, b1, c1 and a2, b2, c2 are the direction ratios of two lines, then y Example 9. What are the direction cosines of a line
the acute angle θ between them is given by
| a1a2 + b1b2 + c1c2 | which is equally inclined to the coordinate axes?
cos θ =
a1 + b12 + c12 a22 + b22 + c22
2 Sol. If α, β and γ are the angles that a line makes with the
coordinate axes, then if they are equally inclined.
( a1b2 − a2b1 ) + ( b1c2 − b2c1 )
2 2

⇒ α =β = γ
+ ( c1a2 − c2a1 ) 2
sin θ = Also, l 2 + m2 + n2 = 1
a12 + b12 + c12 a22 + b22 + c22
⇒ cos2 α + cos2 β + cos2 γ = 1
3. The two lines with direction cosines l1, m1, n1 and
π ⇒ cos2 α + cos2 α + cos2 α = 1
l2, m2, n2 are perpendicular to each other if θ =
2
⇒ 3 cos2 α = 1
⇒ cos θ = l1l2 + m1m2 + n1n2 = 0
4. The two lines with direction cosines l1, m1, n1 and 1
⇒ cos α = ± = cos β = cos γ
l2, m2, n2 are parallel to each other if θ = 0 3
or π ⇒sin θ = 0  1 1 1 
⇒( m1n2 − m2n1 ) 2 + ( n1l2 − n2l1 ) 2 + ( l1m2 − l2m1 ) 2 = 0 ∴ Direction cosines are  , , 
 3 3 3
l1 m1 n1
⇒ = =  1 1 1 
l2 m2 n2 or − ,− ,− .
 3 3 3
5. The angle between two lines having direction ratios a1, b1, c1
and a2, b2, c2 is given by
a1a2 + b1b2 + c1c2 y Example 10. If a line makes angles α, β and γ with
cos θ =
( a1 + b12 + c12 a22 + b22 + c22 )
2 the coordinates axes, prove that
Thus, the two straight lines are perpendicular, if
sin 2 α + sin 2 β + sin 2 γ = 2.
a1a2 + b1b2 + c1c2 = 0 Sol. Let l, m and n be the direction cosines of the given vector.
a b c
The two straight lines are parallel if 1 = 1 = 1 Then, l = cos α, m = cos β, n = cos γ
a2 b2 c2
Now, l 2 + m2 + n2 = 1
⇒ cos2 α + cos2 β + cos2 γ = 1
Projection of the Line Segment ⇒ 1 − sin 2 α + 1 − sin 2 β + 1 − sin 2 γ = 1
Joining Two Points on a Given ⇒ sin 2 α + sin 2 β + sin 2 γ = 2

Line y Example 11. A line OP through origin O is inclined at


The projection of the line segment joining two given
30° and 45° to OX and OY , respectively. Find the angle
points ( x 1 , y 1 , z 1 ) and ( x 2 , y 2 , z 2 ) on the line having
at which it is inclined to OZ.
direction cosines l, m, n is given by Sol. Let l, m and n be the direction cosines of the given vector.
l( x 2 − x 1 ) + m(y 2 − y 1 ) + n(z 2 − z 1 ), which is clear l 2 + m2 + n2 = 1
from the vector. where, α = 30 °, β = 45 °
Q(x2, y2, z2) ∴ cos2 α + cos2 β + cos2 γ = 1
⇒ cos2 30 ° + cos2 45 ° + cos2 γ = 1
(x1, y1, z1)
q 2
P  3  1 
2
⇒   +   + cos γ = 1
2
 2   2
3 1
A l, m, n B ⇒ cos2 γ = 1 − −
4 2
Clearly, PQ = ( x 2 − x 1 ) $i + (y 2 − y 1 ) $j + (z 2 − z 1 ) k
$
⇒ cos2 γ =
4 −3 −2
4
and the line AB = l$i + m$j + nk $
1
⇒ cos γ = − which is not possible.
2
The projection of PQ on AB 4
PQ ⋅ AB l( x 2 − x 1 ) + m(y 2 − y 1 ) + n(z 2 − z 1 ) ∴ There exists no point which is inclined to 30° to X -axis and
= = 45° to Y -axis.
| AB | l 2 + m 2 + n2
= l( x 2 − x 1 ) + m(y 2 − y 1 ) + n(z 2 − z 1 )
174 Textbook of Vector & 3D Geometry

y Example 12. Find the direction cosines of a vector r as 49 λ2 = 1


which is equally inclined to OX, OY and OZ. If | r | is ⇒
1
λ=m
given, find the total number of such vectors. 7
Sol. Let l, m and n be the direction cosines of r.  2 3 6
∴ Direction cosines are  ± , m , ±  .
 7 7 7
Since, r is equally inclined with X , Y and Z-axes.
∴ l 2 + m2 + n2 = 1 But it makes obtuse angle with X -axis ⇒ l < 0.
 2 3 6
⇒ 3l 2 = 1 (Ql = m = n) ∴ Direction cosines are  − , , − 
 7 7 7
1
⇒ l =± Also, r = | r | (l i$ + m $j + n k$ )
3
1 1 1  2 3 6 
∴Direction cosines of r are ± ,± ,± . ⇒ r = 21  − $i + $j − k$  (given, | r | = 21)
3 3 3  7 7 7 
Now, r = | r | (l $i + m $j + n k$ ) r = 3 ( − 2 i$ + 3 $j − 6 k )
$

 1 $ 1 $ 1 $ So, the component of r along X , Y and Z-axes are − 6 i$, 9 $j and


⇒ r = | r | ± i± j± k − 18 k,
$ respectively.
 3 3 3 
Since, ‘+’ and ‘−’ signs can be arranged at three planes.
y Example 15. Find the angle between the lines whose
There are eight vectors (i.e. 2 × 2 × 2 ) which are equally
inclined to axes.
 3 1 3
direction cosines are  − , ,−  and
y Example 13. If the points (0, 1, − 2), ( 3, λ , − 1) and  4 4 2 
(µ, − 3, − 4 ) are collinear, verify whether the point  3 1 3
(12, 9, 2) is also on the same line. − , , .
 4 4 2 
Sol. Let the points be A, B and C, whose coordinates are
(0, 1, – 2), (3, λ , – 1) and (µ , – 3, – 4 ) respectively. Sol. Let θ be the required angle, then
Let D = (12, 9, 2 ) cos θ = l1l 2 + m1m2 + n1n2
⇒ DR’s of AB = (3 − 0, λ − 1, − 1 + 2 ) 3 1 3 1
= + − =−
16 16 4 2
= (3, λ − 1, 1 )
1
DR’s of AC = (µ − 0, − 3 − 1, − 4 − ( − 2 )) ∴ cos θ = − ⇒ θ = 120 °
2
= (µ, − 4, − 2 )
Since, A, B and C are collinear. y Example 16.
3 λ −1 1 (i) Find the angle between the lines whose direction
⇒ = =
µ −4 −2 ratios are 1, 2, 3 and − 3, 2, 1.
⇒ µ = − 6, λ = 3 (ii) Find the acute angle between two diagonal of a
∴ Direction ratios of AB are (3, 2, 1 ). cube.
Now, direction ratios of AD are (12 − 0, 9 − 1, 2 − (2 )) or (12, 8, 4 ) Sol. (i) Let θ be the required angle, then
3 2 1
= = 1 × −3 + 2 ×2 + 3 ×1 4 2
Here, cos θ = = =
12 8 4 1 + 4 + 9 1 + 4 + 9 14 7
∴ AB || AD  2
⇒ θ = cos− 1  
Since, AB and AD lie on same straight line.  7
Hence, the point (12, 9, 2 ) is on the same line. (ii) From the figure given below, the direction ratios of the
diagonals OP and CD of a given cube are given by
y Example 14. A vector r has length 21 and direction
a − 0, a − 0, a − 0
ratios 2, − 3, 6. Find the direction cosines and
and a − 0, a − 0, 0 − a
components of r, given that r makes an obtuse angle
and hence their respective direction cosines are
with X-axis.
a a –a
Sol. Here, direction ratio’s are 2, − 3, 6. , ,
a +a +a
2 2 2
a +a +a
2 2 2
a + a2 + a2
2

∴Direction cosines can be written as (2 λ, − 3 λ, 6 λ).


1 1 –1
where, (2 λ)2 + ( − 3 λ ) 2 + (6 λ ) 2 = 1 (Ql 2 + m 2 + n 2 = 1) i.e. , ,
3 3 3
Chap 03 Three Dimensional Coordinate System 175

Z y Example 18. If the direction cosines of a variable line


C (0, 0, a) in two adjacent points be l, m, n and l + δl , m + δm,
E
(0, a, a) n + δn, show that the small angle δθ between the two

a)
P positions, is given by δθ 2 = δl 2 + δm 2 + δn 2 .

a,
F

,
(a
(a, 0, a)
Sol. We have, l 2 + m 2 + n 2 = 1
and (l + δl ) 2 + (m + δm ) 2 + (n + δn ) 2 = 1
B (0, a, 0)
O Y
⇒l 2 + m 2 + n 2 + (δl ) 2 + (δm ) 2 + (δn ) 2 + 2(lδl + mδm + nδn ) = 1
D
A(a,0, 0) (a, a, 0) ⇒ 1 + (δl ) 2 + (δm ) 2 + (δn ) 2 + 2 (lδl + mδm + nδn ) = 1
⇒ (δl ) 2 + (δm ) 2 + (δn ) 2 = − 2 (lδl + mδm + nδn ) ...(i)
X
Let δθ be angle between the two positions.
a a −a
and , , cos δθ = l (l + δl ) + m (m + δm ) + n (n + δn )
a2 + a2 + a2 a2 + a2 + a2 a2 + a2 + a2 δθ
⇒ 1 − 2 sin 2 = 1 + lδl + mδm + nδn ...(ii)
1 1 −1 2
i.e. , ,
3 3 3 From Eqs. (i) and (ii), we get
Let θ be the angle between these diagonals, then δθ
(δl ) 2 + (δm ) 2 + (δn ) 2 = 4 sin 2
1 1 1 1 1 −1 1 1 1 1 2
cos θ = × + × + × = + − = 2
3 3 3 3 3 3 3 3 3 3  δθ 
⇒ 4   = lδl + mδm + nδn
 1  2
⇒θ = cos− 1  
 3
⇒ lδl + mδm + nδn = (δθ)2,

y Example 17. Find the angle between the lines whose  δθ δθ 


 since, sin → as δθ is very small
 
direction cosines are given by l + m + n = 0 and 2 2
2l 2 + 2m 2 − n 2 = 0.
y Example 19. If l 1 , m 1 , n1 and l 2 , m 2 , n 2 are the
Sol. l 2 + m 2 + n 2 = 1 direction cosines of two mutually perpendicular lines,
l + m +n = 0 …(i) shows that the direction cosines of the line
2l 2 + 2m 2 − n 2 = 0 …(ii) perpendicular to both of them are m 1n 2 − m 2n1 ;
2(l 2 + m 2 ) – n 2 = 0 n1 l 2 − n 2l 1 ; l 1 m 2 − l 2m 1 .
2 Sol. Let l, m and n be the direction cosines of the line
2(1 − n 2 ) = n 2 ⇒ 3n 2 = 2 ⇒ n = ± …(iii) perpendicular to both the given lines.
3
∴ ll1 + mm1 + nn1 = 0 and ll 2 + mm2 + nn2 = 0
2(l 2 + m 2 ) = n 2 = ( − (l + m )) 2 …(iv)
l m n
⇒ 2l 2 + 2m 2 = l 2 + m 2 + 2lm Solving them, we get = =
m1 n1 n1 l1 l1 m1
⇒ l 2 + m 2 − 2lm = 0 m2 n2 n2 l 2 l 2 m2
⇒ (l – m ) 2 = 0 ⇒ l = m ⇒
l
=
m
=
n
=k
2 m1n2 − m2n1 n1l 2 − n2l1 l1m2 − l 2m1
⇒ l +m=m
3 ∴ l = k(m1n2 − m2n1 ) , m = k(n1l 2 − n2l1 ), n = k(l1m2 − l 2m1 )
2 On squaring and adding, we get
⇒ 2l = m
3 l 2 + m 2 + n 2 = k 2 {(m1n2 − m2n1 ) 2 + (n1l 2 − n2l1 ) 2 } + (l1m2 − l 2m1 ) 2
1 1
∴ l =± ,m = ± ⇒ 1 = k 2 {sin 2 θ }
6 6
where, θ is the angle between the given lines as we know,
 1 1 2  1 1 2
Direction cosines are  , ,  and  , ,−  sin θ = (a1b2 − a 2b1 ) 2 + (b1c 2 − b2c1 ) 2 + (c1a 2 − c 2a1 ) 2
 6 6 3  6 6 3
 1 where, a1, b1, c1 and a 2, b2, c 2 are direction cosines.
1 2  1 1 2
or  − ,− ,  and  − ,− ,−  ⇒ 1 = k2 ⋅ 1 (Q θ = 90 °, given)
 6 6 3  6 6 3
 1 ⇒ k =1
The angle between these lines in both the cases is cos− 1  −  .
 3 Hence, direction cosines of a line perpendicular to both of
them are m1n2 − m2n1, n1l 2 − n1l 2, l1m2 − l 2m1.
176 Textbook of Vector & 3D Geometry

y Example 20. Find the direction cosines of the line y Example 21. Let A (− 1, 2, 1) and B (4, 3, 5) be two
which is perpendicular to the lines with direction given points. Find the projection of AB on a line which
cosines proportional to (1, − 2, − 2) and (0, 2, 1). makes angle 120° and 135° with Y and Z-axes
Sol. If l, m and n are the direction cosines of the line perpendicu- respectively, and an acute angle with X-axis.
lar to the given line, then
Sol. Let α be an acute angle that the given line make with
l ⋅(1 ) + m ⋅( − 2 ) + n ⋅ ( − 2 ) = 0 X -axis. Then, cos 2 α + cos 2 120° + cos 2 135° = 1
⇒ l − 2m − 2n = 0 ...(i) 1 1 4 −2 −1 1
and l ⋅ 0 + m ⋅2 + n ⋅ 1 = 0 ⇒ cos2 α = 1 − − = =
4 2 4 4
0 + 2m + n = 0 ...(ii) 1
⇒ cos α = ± but α is acute
Then, from Eqs. (i) and (ii) by cross multiplication, we get 2
l
=
m n
= ∴ cos α = + ve
2 −1 2 1
⇒ cos α = = cos 60 ° ⇒ α = 60 °
l m n 2
⇒ = =
2 −1 2 Thus, the direction cosines of the given straight line are
1 1 1
l 2 + m2 + n2 1 cos 60°, cos 120°, cos 135°, i.e. , − , −
= = (Q l 2 + m 2 + n 2 = 1 ) 2 2 2
4+1+4 3
Hence the projection of AB on the line
2 1 1 1 1 5 1
⇒ l = ,m = − , = ( 4 + 1 ) − (3 − 2 ) − (5 − 1 ) = − − 2 2
3 3 2 2 2 2 2
2
n= = (2 − 2 2 ) units
3

Exercise for Session 1


1. In how many disjoint parts does the three dimensional rectangular cartesian coordinate system divide the
space.
2. Find the distance between the points (k , k + 1, k + 2) and (0, 1, 2).

3. Show that the points (1, 2, 3), ( − 1, − 2, − 1), (2, 3, 2) and (4, 7, 6) are the vertices of a parallelogram.

4. If the mid-points of the sides of a triangle are (1, 5, − 1), (0, 4, − 2) and (2, 3, 4). Find its vertices.

5. Find the maximum distance between the points (3 sin θ, 0, 0) and (4 cos θ, 0, 0).

6. If A = (1, 2, 3), B = (4, 5, 6), C = (7, 8, 9) and D, E, F are the mid-points of the triangle ABC, then find the centroid of
the triangle DEF.
7. A line makes angles α, β and γ with the coordinate axes. If α + β = 90° , then find γ.

8. If α, β and γ are angles made by a line with positive direction of X-axis, Y-axis and Z-axis respectively, then find
the value of cos 2α + cos 2β + cos 2γ .
9. If cos α, cos β, cos γ are the direction cosine of a line, then find the value of cos 2 α + (cos β + sin γ)
(cos β − sin2 γ).

10. A line makes angles α, β, γ, δ with the four diagonals of a cube, then prove that
4
cos 2 α + cos 2 β + cos 2 γ + cos 2 δ = .
3

11. Find the direction cosine of line which is perpendicular to the lines with direction ratio[1, − 2, − 2] and [0, 2, 1].

12. The projection of a line segment on the axis 1, 2, 3 respectively. Then find the length of line segment.
Session 2
Equation of a Straight Line in Space, Angle between
Two Lines, Perpendicular Distance of a Point from
a Line, Shortest Distance between Two Lines

Equation of a Straight Line 2. Cartesian Equation of a Line Passing


Through a Given Point and Given
in Space Direction Ratios
A straight line in space is specified basically in two ways Let the coordinates of the given point A be ( x 1 , y 1 , z 1 ) and
(i) A line passing through a given point and parallel to a the direction ratios of the line be a, b and c. Consider the
given vector. coordinate of any point P be ( x , y , z ). Then,
(ii) A line passing through any two given points. r = x$i + y$j + zk $;

1. Vector Equation of a Line Passing Through a = x 1 $i + y 1 $j + z 1 k


$
a Given Point and Parallel to a Given Vector and b = a$i + b$j + ck
$
To find the vector equation of a straight line which passes Substituting these values in (i) and equating the
through a given point and is parallel to a given vector. coefficients of $i, $j and k,
$ we get
A lb P
x = x 1 + λa
y = y 1 + λb
a z = z 1 + λc
r These are parametric equations of the line.
b
Eliminating the parameter λ, we get
O x − x 1 y − y1 z − z1
E F
= = .
a b c
Let A be the given point and let EF be the given line, then
through A draw AP parallel to given line EF . Remarks
Let b any vector parallel to the given line. Take any point 1. Parametric equation of straight line
O as the origin of reference. Let a the position vector of x − x1 y − y1 z − z1
= = =λ
the given point A. a b c
⇒ x = x1 + λa, y = y1 + λb, z = z1 + λc
Let P be any point on the AP and let its position vector be (where, λ being parameter)
r. Then, we have 2. Since, X, Y and Z-axes pass through origin and have direction
r = OP = OA + AP = a + λb (where, AP = λb) cosines ( 1, 0, 0 ), ( 0, 1, 0 ) and ( 0, 0, 1).
∴Their equations are
Hence, the vector equation of straight line x −0 y −0 z −0
Equation of X-axis, = =
r = a + λb …(i) 1 0 0
⇒ y = 0 and z = 0
Remarks x −0 y −0 z −0
Equation of Y-axis, = =
1. Here, r is the position vector of any point P( x, y, z ) on the line 0 1 0
∴ r = x $i + y$j + zk$ . ⇒ x = 0 and z = 0
x −0 y − 0 z − 0
2. In particular, the equation of the straight line through origin Equation of Z-axis, = =
and parallel to b is r = λb. 0 0 1
⇒ x = 0 and y = 0
178 Textbook of Vector & 3D Geometry

y Example 22. Find the equation of straight line 3. Vector Equation of a Line Passing
parallel to 2$i − $j + 3k$ and passing through the point Through Two Given Points
( 5, − 2, 4 ). The vector equation of a line passing through two points
Sol. Vector form Let P = (5, − 2, 4 ), then OP = 5$i − 2$j + 4 k$ = a whose position vectors a and b is
Also, b = 2 $i − $j + 3 k$ r = a + λ( b − a )
So, equation of straight line passing through a and parallel to Let O be the origin and A and B be the given points with
straight line whose direction ratios are b is given as position vectors a and b, respectively.
r = a + λb Then, OP = r, OA = a and OB = b
⇒ r = (5 i$ − 2 $j + 4 k)
$ + λ(2 $i − $j + 3 k)
$
A B P
Cartesian form Here, ( x1, y1, z1 ) = (5, − 2, 4 ) and parallel to
straight line whose DR’s are (2, − 1, 3 ), so equation of the
x −5 y + 2 z − 4
straight line is = = . a b
r
2 −1 3

y Example 23. Find the vector equation of a line


passing through (2, − 1, 1) and parallel to the line whose O
x − 3 y +1 z −2 Since, AP is collinear with AB.
equation is = =
2 7 −3 ∴ AP = λ AB for some scalar λ
Sol. Since, the required line is parallel to ⇒ OP − OA = λ(OB − OA )
x −3 y +1 z −2 ⇒ r − a = λ( b − a )
= =
2 7 −3
⇒ r = a + λ( b − a )
it follows that the required line passing through
∴ Equation of straight line passing through a and b.
A(2i$ − $j + k$ ) has the direction of 2$i + 7 $j − 3k$ . Hence, the
vector equation of the required line is
⇒ r = a + λ( b − a )
r = 2$i − $j + k$ + λ(2 $i + 7 $j − 3k$ ) where λ is a parameter. 4. Cartesian Equation of a Line Passing
y Example 24. The cartesian equation of a line are Through Two Given Points
6 x − 2 = 3y + 1 = 2z − 2. Find its direction ratios and Equation of straight line passing through ( x 1 , y 1 , z 1 ),
also find the vector equation of the line. ( x 2 , y 2 , z 2 ).
Sol. We know that, (x1, y1, z1) (x2, y2, z2) (x, y, z)
x − x1 y − y1 z − z1 A B
= = is cartesian equation of straight line. P
a b c
∴ 6 x − 2 = 3y + 1 = 2z − 2
 1  1
⇒ 6  x −  = 3 y +  = 2(z − 1 )
 3  3
1 1
x− y + O
⇒ 3 = 3 = z −1
1 1 1 The direction ratios of
6 3 2 AB = ( x 2 − x 1 , y 2 − y 1 , z 2 − z 1 )
1 1
x− y + The direction ratios of
⇒ 3 = 3 = z −1
1 2 3 AP = ( x − x 1 , y − y 1 , z − z 1 )
1 1  Since, they are proportional
which shows given line passes through  , − , 1 and has
3 3  x − x1 y − y1
direction ratios (1, 2, 3 ). =
∴ Its vector equation is x2 − x1 y2 − y1
1 1  z − z1
r =  i$ − $j + k$  + λ ( $i + 2 $j + 3 k$ )
3 
=
3 z2 − z1
Chap 03 Three Dimensional Coordinate System 179

y Example 25. Find the vector equation of line passing Condition for Perpendicularity
through A( 3, 4, − 7 ) and B(1, − 1, 6). Also, find its
cartesian equations. The lines are perpendicular, then
b ⋅ b′ = 0
Sol. Since, the line passes through A (3$i + 4 $j − 7 k$ )
and B( $i − $j + 6k$ ), its vector equation is ⇒ a 1a 2 + b 1b 2 + c 1c 2 = 0
r = 3 i$ + 4 $j − 7 k$ + λ[( i$ − $j + 6 k$ ) − (3 i$ + 4 $j − 7 k$ )]
or r = 3 i$ + 4 $j − 7 k$ − λ(2 $i + 5 $j − 13 k$ ) …(i) Condition for Parallelism
where λ is a parameter. The lines are parallel, then b = ( b ′ ) λ, for some scalar λ
x −3 y − 4 z + 7
The cartesian equivalent of (i) is = = . a1 b1 c 1
2 5 − 13 ⇒ = =
a2 b2 c 2
y Example 26. Find the equation of a line which
passes through the point (2, 3, 4 ) and which has equal y Example 27. Find the angle between the pair of lines
intercepts on the axes. r = 3$i + 2$j − 4 k$ + λ ( $i + 2$j + 2 k$ )
Sol. Since, lines has equal intercepts on axes, it is equally
inclined to axes. and r = 5 $i − 2 k$ + µ ( 3$i + 2$j + 6 k$ )
⇒ line is along the vector a( i$ + $j + k$ ) Sol. Given line are
x −2 y − 3 z − 4 r = (3 $i + 2 $j − 4 k$ ) + λ( $i + 2 $j + 2 k$ )
⇒ Equation of line is = =
1 1 1 and r = (5 $i − 2 k$ ) + µ(3 $i + 2 $j + 6 k$ )
We know that, angle between r = a1 + λb1 and r = a 2 + µb 2,
Angle between Two Lines b ⋅b
cos θ = 1 2
| b1 | | b 2 |
Vector Form
( i$ + 2 $j + 2 k$ ) ⋅(3 $i + 2 $j + 6 k$ ) 3 + 4 + 12 19
Let r = a + λb ...(i) ∴ cos θ = = =
1 +2 +2
2 2 2
3 +2 +6
2 2 2 9 ⋅ 49 21
and r = a ′ + µb ′ ...(ii)
 19
be two straight line in space. ∴ θ = cos−1  
 21
Clearly, Eqs. (i) and (ii) are straight line in the directions
of b and b′, respectively. y Example 28. Prove that the line x = ay + b , z = cy + d
Let θ be the between the straight lines (i) and (ii). and x = a ′ y + b ′, z = dy + d ′ are perpendicular,
Then, θ is the angle between the vectors b and b′ also if aa ′ + cc ′+ 1 = 0.
b ⋅ b ′ = | b | | b ′ | cos θ Sol. We can write the equations of straight line as
x − b′ z − d′
b ⋅ b′ = y, y =
⇒ cos θ = a′ c′
| b | | b′ | x − b′ y − 0 z − d ′
⇒ = = …(i)
Cartesian Form a′ 1 c′
x − x1 y − y1 z − z1 x −b z −d
and = y, y =
Let = = ...(i) a c
a1 b1 c1
x − b y − 0 z − d2
⇒ = = ...(ii)
x − x2 y − y2 z − z2 a 1 c
and = = ...(ii)
a2 b1 c2 x − x1 y − y1 z − z1
We know that, = =
a1 b1 c1
be two straight lines. Then, b = a 1 $i + b 1 $j + c 1 k
$
x − x2 y − y 2 z − z 2
b ′ = a 2 $i + b 2 $j + c 2 k
$ and = =
a2 b2 c2
So, that b ⋅ b′ = a 1a 2 + b 1b 2 + c 1c 2 are perpendicular, if a1a 2 + b1b2 + c1c 2 = 0
and |b|= a 12 + b 12 + c 12 ; | b′ | = a 22 + b 22 + c 22 ∴ For the straight lines given by Eqs. (i) and (ii), to be
perpendicular.
a 1a 2 + b 1b 2 + c 1c 2 a′ a + 1 ⋅ 1 + c ′ c = 0
∴ cos θ =
a 12 + b 12 + c 12 a 22 + b 22 + c 22 ⇒ aa′+ cc′+1 = 0
180 Textbook of Vector & 3D Geometry

− (a − α ) ⋅ b
Perpendicular Distance ⇒ λ=
| b |2
of a Point from a Line ⇒ Position vector of L is a − 
 (a − α ) ⋅ b 
 b, which is
1. Foot of Perpendicular from a Point on  | b |2 
the Given Line the foot of the perpendicular.
(i) Cartesian Form Here, the equation of line AB is (iii) The distance of the point ( x 2 , y 2 , z 2 ) from the line
x − x 1 y − y1 z − z1 x − x1 y − y1 z − z1
= = = = , (where l, m and n are
a b c l m n
direction cosines of the line), is
Let L be the foot of the perpendicular drawn from
x − x1 y − y1 z − z1 [( x 2 − x 1 ) 2 + (y 2 − y 1 ) 2 + (z 2 − z 1 ) 2
P(α, β, γ ) on the line = = .
a b c − {l( x 2 − x 1 ) + m(y 2 − y 1 ) + n(z 2 − z 1 )} 2 ]1 /2
Let the coordinates of L be (x 1 + aλ, y 1 + bλ, z 1 + cλ). Let r1 = ( x 2 − x 1 ) $i + (y 2 − y 1 ) $j + (z 2 − z 1 ) k
$
Then, the direction ratios of PL are (x 1 + aλ − α,
r2 = l$i +m$j + nk
$
y 1 + bλ − β, z 1 + cλ − γ).
r2 ⋅ r1
P (a, b, g) ∴ cos θ =
| r2 | ⋅ | r1 |
(x2, y2, z2)

A L B
d
Direction ratios of AB are (a, b, c ).
Since PL is perpendicular to AB.
q
a( x 1 + aλ − α) + b(y 1 + bλ − β) + c (z 1 + cλ − γ) = 0 (x1, y1, z1) DC’s
a(α − x 1 ) + b(β − y 1 ) + c ( γ + z 1 ) (l , m , n )
λ=
a2 + b2 +c2 Also, d = | r1 | sin θ
Putting the value of λ in (x 1 + aλ, y 1 + bλ, z 1 + cλ), d 2 = | r1 |2 sin2 θ
we get the foot of the perpendicular. Now, we can get
distance PL using distance formula. = | r1 |2 (1 − cos 2 θ)
(ii) Vector Form Let L be the foot of the perpendicular  ( r1 ⋅ r2 ) 2 
= | r1 |2  1 − 
drawn from P(α ) on the line r = a + λb.  | r1 |2 ⋅ | r2 |2 
Since, r denotes the position vector of any point on
d 2 = | r1 |2 − ( r1 ⋅ r2 ) 2 (where, | r2 | = 1)
the line r = a + λb, the position vector of L will be
(a + λb ) ⇒ d = | r1 |2 − ( r1 ⋅ r2 ) 2
Directions ratios of PL = a − α + λb
{( x 2 − x 1 ) 2 + (y 2 − y 1 ) 2 + (z 2 − z 1 ) 2
P (a) d=
− {l ( x 2 − x 1 ) + m (y 2 − y 1 ) + n (z 2 − z 1 )} 2

2. Reflection or Image of a Point


A B
in a Straight Line
(r=a+lb) L(a+lb) (i) Cartesian Form To find the reflection or image of a
point in a straight line in cartesian form.
Since, PL is perpendicular to b,
Let P(α, β, γ ) be the point and
(a − α + λb ) ⋅ b = 0 x − x 1 y −y1 z − z1
⇒ (a − α ) ⋅ b + λb ⋅ b = 0 = = be the given line.
a b c
Chap 03 Three Dimensional Coordinate System 181

P (a, b, g)
[(a − α ) ⋅ b ]
⇒ λ =−
| b |2
A B ∴ Position vector of L,
L
 (a − α ) ⋅ b 
a + λb = a −  b
 | b |2 
Q(Image) (a´, b´, g´)
Let Q be the image of point P and α′ be the position
Let L be the foot of perpendicular from P to AB and vector.
let Q be the image of the point in the given line, Since, L is mid-point of PQ.
where PL = LQ .
α + α′  (a − α ) ⋅ b 
Let the coordinates of L be ⇒ =a −  b
2  | b |2 
( x 1 + aλ, y 1 + bλ, z 1 + cλ )
Then, direction ratios of PL are  2(a − α) ⋅ b 
⇒ α ′ = 2a −   b −α
(x 1 + aλ − α, y 1 + bλ − β, z 1 + cλ − γ )  | b |2 
Since, PL is perpendicular to the given line, whose which is image of P on r.
direction ratios are a, b and c.
∴ ( x 1 + aλ − α ) ⋅ a + (y 1 + bλ − β) ⋅ b y Example 29. Find the foot of perpendicular drawn
+ (z 1 + cλ − γ) ⋅ c = 0 from the point 2$i − $j + 5k$ to the line
{a(α − x 1 ) + b(β − y 1 ) + c ( γ − z 1 ) r = (11$i − 2$j − 8k$ ) + λ (10$i − 4 $j − 11k$ ). Also, find the
⇒ λ=
a2 + b2 + c2 length of the perpendicular.
Substituting λ, we get L, (foot of perpendicular) Sol. Let L be the foot of the perpendicular drawn from
P(2$i − $j + 5k$ ) on the line
Let coordinates of Q(α ′ , β′ , γ ′ ) be image.
∴ Mid-point of PQ is L. P (2i – j + 5k)

α + α′ β + β′ γ + γ′
∴ = x 1 + aλ , = y 1 + bλ, = z 1 + cλ
2 2 2 (11i – 2j – 8k)
∴ α ′ = 2(x 1 + aλ) − α, β′ = 2(y 1 + bλ) − β, +l(10i – 4j – 11k)

γ ′ = 2(z 1 + cλ ) − γ L

(ii) Vector Form To find the reflection or image of a r = (11 $i − 2 $j − 8 k$ ) + λ(10 $i − 4 $j − 11 k$ )


point in a straight line in vector form. Let P(α ) be the Let the position vector of L is
given point and r = a + λb be the given line. (11 i$ − 2 $j − 8 k$ ) + λ(10 $i − 4 $j − 11 k$ )
P (a) = (11 + 10 λ)i$ + ( − 2 − 4 λ)$j + ( − 8 − 11 λ ) k$
∴PL = Position vector of L − Position vector of P
r=a+lb = (9 + 10 λ)$i + ( − 1 − 4 λ)$j + ( − 13 − 11 λ ) k$
A
L Since, PL is perpendicular to the given line and parallel to
b = 10 $i − 4 $j − 11 k$ . ⇒ PL ⋅ b = 0

Q(Image) (a´) ⇒ {(9 + 10 λ)i$ + ( −1 − 4 λ)$j + ( −13 − 11 λ ) k$ } ⋅ (10 $i − 4 $j − 11 k$ ) = 0


⇒ 10(9 + 10 λ ) − 4( − 1 − 4 λ) − 11( − 13 − 11 λ) = 0
Let Q be the image of P in r = a + λb ⇒ λ = −1
∴ PL = a + λb − α On putting λ = − 1, we get L as ( i + 2 j + 3 k )
$ $ $
Since, PL is perpendicular to the given line, Now, PL = ( $i + 2 $j + 3 k$ ) − (2 $i − $j + 5 k$ )
∴ PL ⊥ b = ( − $i + 3 $j − 2 k$ )
⇒ PL ⋅ b = 0
Hence, the length of perpendicular from P on the given line
⇒ (a + λb − α ) ⋅ b = 0 = | PL | = 1 + 9 + 4 = 14
182 Textbook of Vector & 3D Geometry

y Example 30. Find the coordinates of the foot of the y Example 32. Find the length of the perpendicular
perpendicular drawn from point A (1, 0, 3) to the join of 4 − x y 1−z
points B (4, 7, 1) and C(3, 5, 3). drawn from point (2, 3, 4) to line = = ⋅
2 6 3
Sol. Let D be the foot of the perpendicular and let it divide BC Sol. Let P be the foot of the perpendicular from A(2, 3, 4 ) to the
3λ + 4
in the ratio λ : 1. Then, the coordinates of D are , given line l whose equation is
λ +1
4 − x y 1 −z
5λ + 7 3λ + 1 = =
and . 2 6 3
λ +1 λ +1
x − 4 y z −1
or = = = k (say). ...(i)
A (1, 0, 3) −2 6 −3
Therefore, x = 4 − 2k, y = 6k, z = 1 − 3k
As P lies on (i), coordinates of P are ( 4 − 2k, 6k, 1 − 3k) for some
value of k.
D
The direction ratios of AP are
B (4, 7, 1) C (3, 5, 3)
( 4 − 2k − 2, 6k − 3, 1 − 3k − 4 )
Now, AD ⊥ BC ⇒ AD ⋅ BC = 0 or (2 − 2k, 6k − 3, − 3 − 3k ).
7 Also, the direction ratios of l are − 2, 6 and − 3.
⇒ (2 λ + 3 ) + 2(5 λ + 7 ) + 4 = 0 ⇒ λ = −
4 Since, AP ⊥ l
5 7 17 ⇒ − 2(2 − 2k ) + 6(6k − 3 ) − 3( − 3 − 3k ) = 0
⇒ Coordinates of D are , and ⋅
3 3 3 ⇒ − 4 + 4k + 36k − 18 + 9 + 9k = 0
13
y Example 31. Find the length of perpendicular from or 49k − 13 = 0 or k =
x+1 y − 3 z +2 49
P(2, − 3, 1) to the line = = We have, AP 2 = ( 4 − 2k − 2 ) 2 + (6k − 3 ) 2 + (1 − 3k − 4 ) 2
2 3 −1
= (2 − 2k ) 2 + (6k − 3 ) 2 + ( − 3 − 3k ) 2
x +1 y −3 z +2
Sol. Given line is = = =r ...(i) = 4 − 8k + 4k 2 + 36k 2 − 36k + 9 + 9 + 18k + 9k 2
2 3 −1
and P(2, − 3, 1 ) = 22 − 26k + 49k 2
2
P (2, –3, 1)  13   13 
= 22 − 26   + 49  
 49  49
22 × 49 − 26 + 13 + 13 2 909
= =
90° 49 49
A B 3
Q AP = 101
Coordinates of any point on line (i) may be taken as 7
(2r − 1, 3r + 3, − r − 2 ) A(2, 3, 4)
Let Q = (2r − 1, 3r + 3, − r − 2 )
Direction rati o’s of PQ are(2r − 3, 3r + 6, − r − 3 ).
Direction ratio’s of AB are (2, 3, −1).
Since, PQ ⊥ AB l P
2(2r − 3 ) + 3(3r + 6 ) − 1( − r − 3 ) = 0 Aliter
15 We know that the distance of the point ( x 2, y 2, z 2 ) from the
⇒ r =−
14 x − x1 y − y1 z − z1
line = = is
 22 3 13 l m n
∴ Q = − , − , − 
 7 14 14 ( x 2 − x1 ) 2 + (y 2 − y1 ) 2 + (z 2 − z1 ) 2
2 2 2
 22  3  13 531 − (l ( x 2 − x1 ) + m(y 2 − y1 ) + n(z 2 −z1 )) 2
PQ 2 = 2 +  +  − 3 +  + 1 +  =
 7   14   14  14
Here, ( x 2, y 2, z 2 ) are (2, 3, 4 ) and ( x1, y1, z1 ) are ( 4, 0, 1 ) and
PQ =
531  − 2 6 − 3
units (l , m, n ) =  , , .
14  7 7 7 
Chap 03 Three Dimensional Coordinate System 183

(2 − 4 ) 2 + (3 − 0 ) 2 + ( 4 − 1 ) 2 x − 1 y + 2 z −3
Sol. Here, = = ,
∴ d = 2 2 3 6
− 2 6 3 
− (2 − 4 ) + (3 − 0 ) − ( 4 − 1 )
 7 7 7  P(1, –2, 3)
2
 4 + 18 − 9
= 4+9+9− 
 7  3 units

169 1078 − 169


= 22 − = Q(2r+1, 3r–2, 6r+3)
49 49
909 3 is the given straight line ...(i)
= = 101
7 7 Let P = (1, − 2, 3 ) on the straight line.
Here, direction ratios of line (i) are (2, 3, 6 ).
y Example 33. Find the image of the point (1, 6, 3) in 2 3 6
x y −1 z −2 ∴Direction cosines of line (i) are , , .
the line = = . 7 7 7
1 2 3 Equation of line (i) may be written as
Sol. Let P be the given point and let L be the foot of perpendicu- x −1 y + 2 z − 3
= = ...(ii)
lar from P to the given line. 2/7 3/7 6/7
P (1, 6, 3) Coordinates of any point on the line (ii) may be taken as
2 3 6 
 r + 1, r − 2, r + 3
7 7 7 
2 3 6 
A L B Let Q  r + 1, r − 2, r + 3
7 7 7 
Given, | r| =3
Q ∴ r =±3
 13 −5 39  1 −23 3
The coordinates of a general point on the given line are given Putting the value of r, we have Q =  , ,  or  , , 
by 7 7 7  7 7 7
x − 0 y −1 z −2
= = =λ

i.e.
1 2 3
x = λ, y = 2 λ + 1, z = 3 λ + 2
Shortest Distance between
Let the coordinates of L be
( λ, 2 λ + 1, 3 λ + 2 ) . ...(i)
Two Lines
If two lines in space intersect at a point, then the shortest
So, direction ratios of PL are
distance between them is zero. Also, if two lines in space
( λ − 1, 2 λ − 5, 3 λ − 1 )
are parallel, then the shortest distance between them will
Direction ratios of the given line are (1, 2, 3 ) which is
be the perpendicular distance, i.e. the length of the
perpendicular to PL
perpendicular drawn from any point on one line onto the
( λ − 1 ) ⋅ 1 + (2 λ − 5 ) ⋅ 2 + (3 λ − 1 ) ⋅ 3 = 0
other line. Further, in a space, there are lines which are
⇒ λ =1
neither intersecting nor parallel. In fact, such pair of lines
So, coordinates of L are (1, 3, 5). are non-coplanar and are called skew lines.
Let Q( x1, y1, z1 ) be the image of P(1, 6, 3 ) on given line.
Z
Since, L is mid-point of PQ.
x +1 y +6 z +3
∴ 1= 1 ,3 = 1 ,5 = 1
2 2 2
G F
⇒ x1 = 1, y1 = 0, z1 = 7
∴ Image of P(1, 6, 3 ) in the given line is (1, 0, 7 ).
D E
y Example 34. Find the coordinates of those points on
x − 1 y +2 z − 3 O Y
the line = = which are at a distance of C
2 3 6
A B
3 units from points (1, − 2, 3).
X
184 Textbook of Vector & 3D Geometry

Line GE goes diagonally across the ceiling and line DB Condition for Lines to Intersecting
passes through one corner of the ceiling directly above A The two lines are intersecting, if
and goes diagonally down the wall. These lines are skew
( b1 × b2 ) ⋅ (a 2 − a 1 )
because they are not parallel and also never meet. =0
| b1 × b2 |
By the shortest distance between two lines, we mean the
join of a point in one line with one point on the other line ⇒ ( b1 × b2 ) ⋅ (a 2 − a 1 ) = 0
so that the length of the segment so obtained is the ⇒ [ b1 b2 (a 2 − a 1 )] = 0
smallest.
Cartesian Form
1. Shortest Distance between Two Let the two skew lines be
Skew Straight Lines x − x1 y − y1 z − z1
= =
Line of Shortest Distance a1 b1 c1
If l 1 and l 2 are two skew lines, then there is one and only x − x2 y − y2 z − z2
and = =
one line perpendicular to each of lines l 1 and l 2 which is a2 b2 c2
known as the line of shortest distance.
Vector equations for these two lines are
P
r = ( x 1 $i + y 1 $j + z 1 k
$ ) + λ(a 1 $i + b 1 $j + c 1 k
$)
l1 and r = ( x 2 $i + y 2 $j + z 2 k
$ ) + µ(a 2 $i + b 2 $j + c 2 k
$)
Line of (a 2 − a 1 ) ⋅ ( b1 × b2 )
shortest distance Q Shortest distance n =
| b1 × b2 |
l2
Q x2 − x1 y2 − y1 z2 − z1
Here, distance PQ is called to be shortest distance. a1 b1 c1
Vector Form a2 b2 c2
⇒ d=
Let l 1 and l 2 be two lines whose equations are (b 1 c 2 − b 2 c 1 ) + (c 1 a 2 − a 1 c 2 ) 2
2

r = a 1 + λb1 and r = a 2 + µb2 , respectively. + (a 1 b 2 − a 2 b 1 ) 2


Clearly, l 1 and l 2 pass through the points A and B with a 1
and a 2 , respectively and are parallel to the vectors b1 and Conditions for Lines to Intersect
b2 , respectively. The lines are intersecting, if shortest distance = 0
Since, PQ is perpendicular to both l 1 and l 2 which are x 2 − x 1 y2 − y1 z2 − z1
parallel to b1 and b2 . ⇒ a1 b1 c1 =0
∴ PQ is parallel to b1 × b2 . a2 b2 c2
b × b2
Let n$ be a unit vector along PQ, then n$ = ± 1
| b1 × b2 | 2. Shortest Distance between Parallel Lines
∴ PQ = Projection of AB on PQ Let l 1 and l 2 be two parallel lines whose equations are
⇒ PQ = AB ⋅ n$ r = a 1 + λb or r = a 2 + µb, respectively.
( b × b2 )
= ± (a 2 − a 1 ) ⋅ 1 l2 B(a2)
| b1 × b2 |
( b1 × b2 ) (a 2 − a 1 )

| b1 × b2 |
a2
( b1 × b2 ) ⋅ (a 2 − a 1 ) l1 q
Hence, distance PQ =
| b1 × b2 | M A(a1)
a1
[ b1 b2 (a 2 − a 1 )] p–q
= q
| b1 × b2 | O
Chap 03 Three Dimensional Coordinate System 185

Clearly, l 1 and l 2 pass through the points A and B with $i $j k$


position vectors a 1 and a 2 , respectively and both are and b1 × b 2 = 1 2 − 3 = 2 $i − $j + 0 k$
parallel to the vector b, where BM is the shortest distance 2 4 −5
between l 1 and l 2 .
∴ (a 2 − a1 ) ⋅( b1 × b 2 ) = ( − 3 i$ + 0 $j + 2 k$ ) ⋅ (2 i$ − $j + 0 k$ ) = − 6
Let θ be the angle between AB and l 1 . and | b1 × b 2 | = 4 + 1 + 0 = 5
BM (a 2 − a1 ) ⋅ ( b1 × b 2 ) −6 6
∴ sin θ = ∴ Shortest distance, d = = =
AB | b1 × b 2 | 5 5
⇒ BM = AB sin θ = | AB | sin θ
Now, | AB × b | = | AB | | b | sin( π − θ)
y Example 37. Find the shortest distance between
| AB | | b | sin θ
the lines
x −1 y −2 z −3 x − 2 y −4 z − 5
= (| AB | sin θ ) | b | = BM | b | = = and = = .
2 3 4 3 4 5
| AB × b | | (a 2 − a 1 ) × b |
∴ BM = = Sol. Given lines are
|b| |b| x −1 y −2 z −3
= = ...(i)
∴ Shortest distance between parallel lines 2 3 4
r = a 1 + λb1 and r = a 2 + µb is x −2 y − 4 z −5
and = = ...(ii)
3 4 5
| (a − a 1 ) × b |
d= 2 Here, x1 = 1, y1 = 2, z 3 = 3; x 2 = 2, y 2 = 4, z 2 = 5
|b|
l1 = 2, m1 = 3, n1 = 4; l 2 = 3, m2 = 4, n2 = 5
y Example 35. Show that the two lines Shortest distance between the lines (i) and (ii) are modulus of
x − 1 y −2 z − 3 x −4 y −1 x 2 − x1 y 2 − y1 z 2 − z1
= = and = = z , intersect.
2 3 4 5 2 l1 m1 n1
Also, find the point of intersection of these lines. l2 m2 n2
= ...(iii)
x −1 y −2 z −3 (l1m2 − l 2m1 ) + (m1n2 − m2n1 ) 2
Sol. Here, = = ...(i)
2 3 4 + (l1n2 − l 2n1 ) 2
x −4 y −1 z − 0
and = = ...(ii) x 2 − x1 y 2 − y1 z 2 − z1 1 2 2
5 2 1
Now, = l1 m1 n1 = 2 3 4
Any point on line (i) is P (2r + 1, 3r + 2, 4r + 3 ) and any point
on the line (ii) is Q(5 λ + 4, 2 λ + 1, λ ). l2 m2 n2 3 4 5
They intersect if and only if = 1(15 − 16 ) − 2(10 − 12 ) + 2(8 − 9 ) = 1
2r + 1 = 5 λ + 4, 3r + 2 = 2 λ + 1, 4r + 3 = λ
Also, (l1m2 − l 2m1 ) 2 + (m1n2 − m2n1 ) 2 + (n1l 2 − n2l1 ) 2
On solving, r = − 1, λ = − 1
= (8 − 9 ) 2 + (15 − 16 ) 2 + (10 − 12 ) 2
Clearly, for these values of λ and r P ( − 1, − 1, − 1 )
Hence, lines (i) and (ii) intersect at ( − 1, − 1, − 1 ). =6
From Eq. (iii) shortest distance between lines (i) (ii), we get
y Example 36. Find the shortest distance between the 1 1
= =
lines r = (4 $i − $j ) + λ ( $i + 2$j − 3 k$ ) 6 6
and r = ( $i − $j + 2 k$ ) + µ (2$i + 4 $j − 5 k$ ).
y Example 38. Find the shortest distance and the
Sol. We know, the shortest distance between the lines vector equation of the line of shortest distance
r = a1 + λb1 and r = a1 + λb 2 between the lines given by
(a 2 − a1 ) ⋅ ( b1 × b 2 ) r = ( 3 $i + 8 $j + 3 k$ ) + λ ( 3$i − $j + k$ )
⇒ d =
| b1 × b 2 |
On comparing the given equation with the equations
and r = ( − 3$i − 7 $j + 6 k$ ) + µ (− 3$i + 2$j + 4 k$ )
r = a1 + λb1 and r = a1 + λb 2 respectively, we have Sol. Given lines are
a1 = 4 $i − $j, a 2 = $i − $j + 2 k$ , b1 = $i + 2 $j − 3 k$ and b 2 = 2 $i + 4 $j − 5 k$ r = (3 $i + 8 $j + 3 k$ ) + λ(3$i − $j + k$ ) ...(i)
Now, a 2 − a1 = − 3 $i + 0 $j + 2 k$ and r = ( − 3 $i − 7 $j + 6 k$ ) + µ( − 3 $i + 2 $j + 4 k$ ) ...(ii)
186 Textbook of Vector & 3D Geometry

Equation of lines (i) and (iii) in cartesian form, y Example 40. Find the equation of a line which
x −3 y − 8 z − 3 passes through the point (1, 1, 1) and intersects the lines
AB : = = =λ ...(iii)
3 −1 1 x −1 y −2 z −3 x +2 y −3 z +1
x + 3 y + 7 z −6 = = and = = ⋅
and CD : = = =µ ...(iv) 2 3 4 1 2 4
−3 2 4
Sol. Any line passing through the point (1, 1, 1) is
B
x −1 y −1 z −1
L = = ...(i)
a b c
A x − 1 y −2 z −3
This line intersects the line = = .
2 3 4
1 −1 2 −1 3 −1
If a : b : c ≠ 2 : 3 : 4 and a b c =0
C D
M 2 3 4
Let L(3 λ + 3 ), − λ + 8, λ + 3), M( − 3µ − 3, 2µ − 7, 4µ + 6 ) ⇒ a − 2b + c = 0
Direction ratios of LM are Again, line (i) intersects line
(3 λ + 3µ + 6, − λ − 2µ + 15, λ − 4µ − 3) x −(− 2) y − 3 z −(− 1)
= = ...(ii)
Since, LM ⊥ AB 1 2 4
∴ 3(3 λ + 3µ + 6 ) − 1( − λ − 2µ + 15 ) + 1( λ − 4µ − 3 ) = 0 −2 −1 3 −1 −1 −1
or 11 λ + 7µ = 0 ...(v) If a : b : c ≠ 1 : 2 : 4 and a b c =0
Again, LM ⊥ CD 1 2 4
− 3(3 λ + 3µ + 6 ) − 2( − λ − 2µ + 15 ) + 4( λ − 4µ − 3 ) = 0 ⇒ 6a + 5b − 4c = 0 ...(iii)
or − 7 λ − 29µ = 0 ...(vi) From (ii) and (iii) by cross multiplication, we have
Solving Eqs. (v) and (vi), we get a b c
= =
λ = 0 =µ 8 − 5 6 + 4 5 + 12
∴ L ≡ (3, 8, 3 ), a b c
⇒ = =
M ≡ ( − 3, − 7, 6 ) 3 10 17
Hence, the shortest distance, x −1 y −1 z −1
So, the required lines is = =
LM = (3 + 3 ) 2 + (8 + 7 ) 2 + (3 − 6 ) 2 = 3 30 units 3 10 17

∴Vector equation of LM is
r = 3 i$ + 8 $j + 3 k$ + t(6 $i + 15 $j − 3 k$ ) y Example 41. If the straight lines x = − 1 + s ,
Also, the cartesian equation of LM is t
y = 3 − λs , z = 1 + λs and x = , y = 1 + t, z = 2 − t, with
x −3 y −8 z −3 2
= =
6 15 −3 parameters s and t, respectively, are coplanar, then
find λ.
y Example 39. Find the shortest distance between
x +1 y −3 z − 1
lines r = ( $i + 2$j + k$ ) + λ (2$i + $j + 2 k$ ) Sol. The given lines = = =s
1 −λ λ
and r = 2$i − $j − k$ + µ (2$i + $j + 2 k$ ). x −0 y −1 y −2
= = = t are coplanar if
y2 1 −1
Sol. Here lines (i) and (ii) are passing through the points
a 1 = $i + 2$j + k$ and a 2 = 2$i − $j − k$ , respectively, and are 0 + 1 1 −3 2 −1
parallel to the vector b = 2i$ + $j + 2k$ . 1 −λ λ =0
1/2 1 −1
Hence, the distance between the lines using the formula
1 −2 1
$i $j k$
1 −λ λ =0
2 1 2
1/2 1 −1
| b × (a 2 − a1 ) | 1 −3 −2
=  λ  λ
| b| 3 ⇒ 1(λ − λ) + 2 1 −  + 1 1 +  = 0
 2  2
| 4 i$ − 6 $j − 7 k$ | 16 + 36 + 49 101
= = = ⇒ λ = −2
3 3 3
Chap 03 Three Dimensional Coordinate System 187

Exercise for Session 2


x −3 y + 1 z −3
1. The cartesian equation of a line is = = . Find the vector equation of the line.
2 −2 5

2. A line passes through the point with position vector 2$i − 3$j + 4k$ and is in the direction of 3$i + 4$j − 5k$ . Find the
equation of the line is vector and cartesian forms.

3. Find the coordinates of the point where the line through (3, 4, 1) and (5, 1, 6) crosses XY-plane.

4. Find the angle between the pairs of line r = 3$i + 2$j − 4k$ + λ( $i + 2$j + 2k$ ), $r = 5$i − 2$j + µ(3$i + 2$j + 6k$ )
x −1 y −2 z −3 x −4 y −1
5. Show that the two lines = = and = = z intersect. Find also the point of intersection
2 3 4 5 2
of these lines.
x y z x − 2 y −1 z + 2
6. Find the magnitude of the shortest distance between the lines = = and = = .
2 −3 1 3 −5 2
x −2 y + 3 z
7. Find the perpendicular distance of the point (1, 1, 1) from the line = = .
2 2 −1
8. Find the equation of the line drawn through the point (1, 0, 2) to meet at right angles the line
x + 1 y −2 z +1
= = .
3 −2 −1
x y z x y z
9. Find the equation of line through (1, 2, − 1) and perpendicular to each of the lines = = and = = .
1 0 −1 3 4 5
x − 6 y −7 z − 7
10. Find the image of the point (1, 2, 3) in the line = = .
3 2 −2
Session 3
Plane, Equation of Plane in Various Form, Angle between
Two Planes, Family of Planes, Two Sides of a Plane,
Distance of a Point from a Plane, Equation of Planes
Bisecting the Angle between Two Planes, Line and Plane

Plane which shows that the point R lies on Eq. (i). Since, R is an
arbitrary point on the line segment joining P and Q.
A plane is a surface such that if any two points are taken
on it, the line segment joining them lies completely on the ∴ Every point on PQ lies on the surface represented by
surface. Eq. (i).
General Form General equation of the first degree in x, Hence, ax + by + cz + d = 0 is equation of plane.
y, z always represents a plane. Equation of a Plane Passing Through a Given Point
The general equation of plane is ax + by + cz + d = 0. The general equation of a plane passing through a given
Proof. Let first degree equation in x, y and z be point (x 1 , y 1 , z 1 ) is a( x − x 1 ) + b(y − y 1 ) + c (z − z 1 ) = 0,
where a, b and c are constants.
ax + by + cz + d = 0 ...(i)
Proof. The general equation of plane is
In order to prove that Eq. (i) is the equation of plane, it is
sufficient to show that every point on the line joining two ax + by + cz + d = 0 ...(i)
points lies on the surface represented by Eq. (i). If it passes through ( x 1 , y 1 , z 1 )
Let P ( x 1 , y 1 , z 1 ) and Q ( x 2 , y 2 , z 2 ) be two points on the ⇒ ax 1 + by 1 + cz 1 + d = 0 ...(ii)
surface represented by Eq. (i). On subtracting Eq. (i) from Eq. (ii), we get
Then, ax 1 + by 1 + cz 1 + d = 0 ...(ii) a( x − x 1 ) + b(y − y 1 ) + c (z − z 1 ) = 0
and ax 2 + by 2 + cz 2 + d = 0 ...(iii) which is the equation of a plane passing through
Let R be any arbitrary point on the line segment joining P ( x 1 , y 1 , z 1 ).
and Q. Suppose R divides PQ in the ratio λ : 1 . y Example 42. Show that the four points (0, − 1, − 1),
 x + λx 2 y 1 + λy 2 z 1 + λz 2  ( − 4, 4, 4 ), (4, 5, 1) and ( 3, 9, 4 ) are coplanar. Find the
∴ R is  1 , , 
 1+ λ 1+ λ 1+ λ  equation of the plane containing them.
We are to show that R lies on the surface represented by Sol. We shall find the equation of a plane passing through any
Eq. (i) for all values of λ. For this, it is sufficient to prove three out of the given four points and show that the fourth
point satisfies the equation.
that R satisfy Eq. (i)
Now, any plane passing through ( 0, − 1, − 1 ) is
On putting this value of R in LHS of Eq. (i), we obtain
a( x − 0 ) + b(y + 1 ) + c(z + 1 ) = 0 ...(i)
 x + λx 2   y 1 + λy 2   z 1 + λz 2  If it passes through ( − 4, 4, 4 ), we have
a 1  +b   +c   +d
 λ +1   λ +1   λ +1  a( − 4 ) + b(5 ) + c(5 ) = 0 ...(ii)
1 Also, if plane passes through (4, 5, 1),we have
= {(ax 1 + by 1 + cz 1 ) + λ(ax 2 + by 2 + cz 2 )} a( 4 ) + b(6 ) + c(2 ) = 0
λ +1
⇒ 2a + 3b + c = 0 ...(iii)
1
= [0 + 0 ] [using Eqs. (ii) and (iii)] On solving Eqs. (ii) and (iii) by cross multiplication method, we
λ +1 obtain
a b c
=0 = = =k
−5 7 −11
Chap 03 Three Dimensional Coordinate System 189

On putting in Eq. (i), we get Cartesian Form


− 5kx + 7k(y + 1 ) − 11k(z + 1 ) = 0
Equation (ii) gives the vector equation of a plane, where n$
⇒ − 5 x + 7y − 11z − 4 = 0
is the unit vector normal to the plane. Let P ( x , y , z ) be any
(required equation of plane)
point on the plane. Then
Clearly, the fourth point namely (3, 9, 4) satisfies this equation.
Hence, the given points are coplanar and the equation of plane OP = r = x$i + y$j + zk
$
containing those points, is 5 x − 7y + 11z + 4 = 0 Let l, m and n be the direction cosines of n.
$
Then, n$ = (l i + m j + nk )
$ $ $

Equation of Plane Therefore, (ii) gives


( x$i + y$j + zk
$ ) ⋅ (l$i + m$j + nk
$) =d
in Various Form or lx + my + nz = d ...(iii)
A plane is determined uniquely if
This is the cartesian equation of the plane in the normal
(i) The normal to the plane and its distance from the form.
origin is given, i.e. the equation of a plane in normal
form. Note
(ii) It passes through a point and is perpendicular to a Equation (iii) shows that if r ⋅( a $i + b$j + ck$ ) = d is the vector
given direction. equation of a plane, then ax + by + cz = d is the cartesian
equation of the plane, where a, b and c are the direction ratios of
(iii) It passes through three given non-collinear points. the normal to the plane.

The equation r ⋅ n = d is in normal form, if n is a unit


Equation of Plane in Normal Form vector and d is the distance of the plane from the origin. If
Vector Form n is not a unit vector, then to reduce the equation r ⋅ n = d
to normal form, we reduce the equation r ⋅ n = d to normal
The vector equation of a plane normal to unit vector n$ form by dividing both sides by | n |, we get
and at a distance d from the origin is r ⋅ n$ = d.
r⋅ n d d
p = ⇒ r⋅ n = = p (distance from the origin)
|n| |n| |n|
N P
(r) y Example 43. Find the vector equation of plane which
dn
d is at a distance of 8 units from the origin and which is
normal to the vector 2$i + $j + 2 k$ .
r

O
Sol. Here, d = 8 and n = 2$i + $j + 2k$
n 2 $i + $j + 2 k$ 2 $i + $j + 2 k$
Proof. Let O be the origin and let ON be the perpendicular n= = =
| n| 22 + 12 + 22 3
from O to the given plane π such that ON = dn$ , where d is
perpendicular distance of plane from origin. Hence, the required equation of plane is, r ⋅ n = d
Let P be a point on the plane, with position vector r so  2 $i + $j + 2 k$ 
⇒ r⋅  =8
that OP = r  3 
Now, NP ⊥ ON ⇒ r ⋅ (2 $i + $j + 2 k$ ) = 24
⇒ NP ⋅ ON = 0 ...(i)
y Example 44. Reduce the equation
⇒ (OP − ON ) ⋅ ON = 0
r ⋅( 3$i − 4 $j + 12k$ ) = 5 to normal form and hence find the
⇒ ( r − dn$ ) ⋅ dn$ = 0
length of perpendicular from the origin to the plane.
⇒ r ⋅ dn$ − d 2 n$ ⋅ n$ = 0
Sol. The given equation of plane is
⇒ dr ⋅ n$ − d 2 | n |2 = 0 (Qd ≠ 0) r ⋅(3 $i − 4 $j + 12 k$ ) = 5 or r ⋅ n = 5
⇒ r ⋅ n$ − d = 0 (Q| n$ | = 1) where, n = 3 $i − 4 $j + 12 k$
⇒ r ⋅ n$ = d ...(ii) Since, | n | = 9 + 16 + 144 = 13 ≠ 1, therefore the given
Thus, the required equation of the plane is r ⋅ n$ = d. equation is not the normal form. To reduce to normal form we
190 Textbook of Vector & 3D Geometry

r⋅ n 5 Since, AP lies in the plane and n is a normal to the plane


divide both sides by | n | i.e. = or
| n| | n| π.
3 4 12  5
r ⋅  $i − $j + k$  = . This is the normal form of the A (a)
 13 13 13  13
5 N
equation of given plane and length perpendicular = .
13 P a
(r)
y Example 45. Find the distance of the plane r

2x − y − 2z − 9 = 0 from the origin. O


Sol. The plane can be put in vector form as r ⋅ (2$i − $j − 2k$ ) = 9.
∴ AP ⊥ n
Here, n = 2 $i − $j − 2 k$ ⇒ AP ⋅ n = 0 ⇒ ( r ⋅ a ) ⋅ n = 0 (Q AP = r − a)
n 2 $i − $j − 2 k$ Hence, the required equation of the plane is
⇒ =
| n| 3
(r − a) ⋅ n = 0
Dividing equation throughout by 3, we have equation of plane
(2 $i − $j − 2 k$ ) Note
in normal form as r ⋅ = 3, in which 3 is the distance
3 The above equation can be written as r ⋅ n = d, where d = a ⋅ n
of the plane from the origin. (known as scalar product form of plane).

y Example 46. Find the vector equation of a line Cartesian Form


passing through 3$i − 5$j + 7k$ and perpendicular to the
If r = x$i + y $j + zk
$ , a = x 1 $i + y 1 $j + z 1 k
$ and n = a$i + b$j + ck
$,
plane 3x − 4 y + 5z = 8.
then ( r − a ) = ( x − x 1 ) $i + (y − y 1 ) $j + (z − z 1 ) k
$
Sol. The given plane 3x − 4y + 5z = 8.
Then equation of the plane can be written as
or (3 i$ − 4 $j + 5 k$ ) ( xi$ + y$j + zk$ ) = 8.
( x − x 1 ) $i + (y − y 1 ) $j + (z − z 1 ) k
$ ⋅ (a$i + b$j + ck
$) =0
This shows that d = 3 $i − 4 $j + 5 k$ is normal to the given plane.
⇒ a( x − x 1 ) + b(y − y 1 ) + c (z − z 1 ) = 0
Therefore, the required line is parallel to 3 i$ − 4 $j + 5 k$ .
Thus, the coefficients of x, y and z in the cartesian
Since, the required line passes through 3 $i − 5 $j + 7 k$ , its equation equation of a plane are the direction ratios of the normal
is given by r = 3 $i − 5 $j + 7 k$ + λ(3 $i − 4 $j + 5 k$ ), where λ is a to the plane.
parameter.
y Example 48. Find the equation of the plane passing
y Example 47. Find the unit vector perpendicular to through the point (2, 3, 1) having (5, 3, 2) as the
the plane r ⋅ (2$i + $j + 2k$ ) = 5. direction ratios of the normal to the plane.
Sol. Vector normal to the plane is n = 2$i + $j + 2k$ Sol. The equation of the plane passing through ( x 1, y1, z1 ) and
perpendicular to the line with direction ratios a, b and c is
Hence, unit vector perpendicular to the plane is
given by a( x − x 1 ) + b(y − y1 ) + c (z − z1 ) = 0.
n 2 i$ + $j + 2 k$
= Now, since the plane passes through (2, 3, 1 ) and is
| n| 22 + 12 + 22 perpendicular to the line having direction ratios (5, 3, 2 ), the
equation of the plane is given by 5( x − 2 ) + 3(y − 3 ) + 2(z − 1 )
1 $ $
= (2 i + j + 2 k$ ) = 0 or 5 x + 3y + 2z = 21.
3
y Example 49. The foot of the perpendicular drawn
Vector Equation of a Plane Passing Through from the origin to a plane is (12, − 4, 3). Find the
a Given Point and Normal to a Given Vector equation of the plane.
The vector equation of a plane passing through a point Sol. Since P(12, − 4,3) is the foot of the perpendicular from the
having position vector a and normal to vector n is origin to the plane OP is normal to the plane π. Thus, the
( r − a ) ⋅ n = 0. direction ratios of narmal to the plane are 12, –4 and 3.
Proof Suppose the planer π passes through a point having Now, since the plane passes through (12,− 4 , 3), its
equation is given by
position vector a and is normal to the vector n. Let O be
12 ( x − 12 ) − 4(y + 4 ) + 3(z − 3 ) = 0
the origin and r be the position vector of any point P on
the plane π. Then, OP = r. or 12 x − 4y + 3z − 169 = 0
Chap 03 Three Dimensional Coordinate System 191

y Example 50. A vector n of magnitude 8 units is x − x1 y − y1 z − z1


inclined to X-axis at 45°, Y-axis at 60° and an acute ⇒ x2 − x1 y2 − y1 z 2 − z 1 = 0, which is the required
angle with Z-axis. If a plane passes through a point x 3 − x1 y 3 − y1 z 3 − z1
( 2, − 1, 1) and is normal to n, then find its equation in
vector form. equation of the plane
Sol. Let γ be the angle made by n with Z-axis, then direction Vector Form
cosines of n are
Vector form of the equation of the plane passing through
1 1
l = cos 45 ° = , m = cos 60 ° = and n = cos γ three points A, B and C having position vectors a, b and c,
2 2
respectively.
1 1
∴ l 2 + m2 + n2 = 1 ⇒ + + n2 = 1 Let r be the position vector of any point P in the plane.
2 4
1 Hence, vector AP = r − a AB = b − a and AC = c − a are
⇒ n2 =
4 coplanar.
1 1
n = (neglecting n = − as γ is acute : n > 0) Hence, ( r − a ) ⋅ {( b − a ) × (c − a )} = 0
2 2
⇒ (r − a) ⋅ (b × c − b × a − a × c + a × a) = 0
We have, | n | = 8
⇒ ( r − a ) ⋅( b × c + a × b + c × a ) = 0
n = | n | (li$ + m$j + nk$ )
⇒ r ⋅( b × c + a × b + c × a )
 1 $ 1 $ 1 $
n =8  i + j + k = 4 2 i$ + 4 $j + 4 k$ = a ⋅ ( b × c ) + a ⋅ (a × b ) + a ⋅ (c × a )
 2 2 2 
The required plane passes through the point ( 2, − 1, 1 ) having ⇒ [ r b c ] + [ r a b ] + [ r c a ] = [a b c ]
position vector which is the required equation of the plane.
a = 2 $i − $j + k$
Note
So, its vector equation is ( r − a ) ⋅ n = 0 1. If p is the length of perpendicular from the origin on this plane,
⇒ r$ ⋅ n$ = a$ ⋅ n$ then p = [ a b c ] / n, where n = | a × b + b × c + c × a |.
⇒ r ⋅( 4 2 $i + 4 $j + 4 k$ ) = ( 2 $i − $j + k$ ) ⋅ ( 4 2 $i + 4 $j + 4 k$ ) 2. Four points a, b, c and d are coplanar if d lies on the plane
containing a, b and c.
⇒ r ⋅( 4 2 $i + 4 $j + 4 k$ ) = 8 or d ⋅ [a × b + b × c + c × a ] = [a b c ]
⇒ r ⋅( 2 $i + $j + k$ ) = 2 or [ d a b] + [ d b c ] + [ d c a ] = [ a b c ]

y Example 51. Find the equation of the plane such y Example 52. Find the equation of the plane passing
that image of point (1, 2, 3) in it is (− 1, 0, 1). through A(2, 2, − 1), B( 3, 4, 2) and C(7, 0, 6 ). Also find a
unit vector perpendicular to this plane.
Sol. Since, the image of A(1, 2, 3) in the plane is B( − 1, 0, 1), the
plane passes through the mid-point (0, 1, 2) of AB and is Sol. Here, (x 1, y1, z1 ) ≡ (2, 2, − 1), ( x 2 , y 2 , z 2 ) ≡ (3, 4, 2) and
( x 3 , y 3 , z 3 ) ≡ (7, 0, 6).
normal to the vector AB = − 2$i − 2$j − 2k$ .
Then, the equation of the plane is
Hence, the equation of the plane is − 2 ( x − 0 ) − 2 (y − 1 )
x − x1 y − y1 z − z1
− 2 (z − 2 ) = 0
x 2 − x1 y 2 −y1 z 2 − z1 = 0
or x + y + z = 3.
x 3 − x1 y 3 − y1 z 3 − z1
x − 2 y − 2 z − (− 1)
Equation of a Plane Passing through or 3 − 2 4 − 2 2 − ( − 1) = 0
Three Given Points 7 − 2 0 − 2 6 − (− 1)
or 5 x + 2y − 3z = 17
Cartesian Form
A normal vector to this plane is d = 5 $i + 2 $j − 3 k$ ...(i)
Let the plane be passing through points A( x 1 , y 1 , z 1 ),
Therefore, a unit vector normal to (i) is given by
B( x 2 , y 2 , z 2 ) and C ( x 3 , y 3 , z 3 ).
d 5 i$ + 2 $j − 3 k$
Let P ( x , y , z ) be any point on the plane. n$ = =
|d| 25 + 4 + 9
Then, vectors PA, BA and CA are coplanar.
1
[PA BA CA] = 0 = (5 $i + 2 $j − 3 k$ )
38
192 Textbook of Vector & 3D Geometry

y Example 53. Find equation of plane passing through Cartesian Form


the points P(1, 1, 1), Q (3, − 1, 2) and R ( − 3, 5, − 4 ). From ( r − a ) ⋅ ( b × c ) = 0,we have [ r − a b c ]
Sol. Let the equation of plane passing through (1, 1, 1) be x − x 1 y − y1 z − z1
a( x − 1) + b(y − 1) + c (z − 1)= 0, as it passes through the
points Q and R. ⇒ x2 y2 z2 = 0, which is the required
∴ 2a − 2b + c = 0 x3 y3 z3
and − 4a + 4b − 5c = 0 equation of the plane, where b = x 2 $i + y 2 $j + z 2 k
$ and
Hence, solving by cross multiplication method, we get c = x 3 i + y 3 j + z 3 k.
$ $ $
a b c
= = =k
10 − 4 − 4 + 10 8 − 8 y Example 54. Find the vector equation of the
∴ a = 6k, b = 6k, c = 0 following planes in cartesian form r = $i − $j + λ( $i + $j + k$ )
On substituting in Eq. (i), we get
+ µ( $i − 2$j + 3 k$ ).
6( x − 1 ) + 6(y − 1 ) + 0 = 0
i.e. x + y = 2; which is the required equation. Sol. The equation of the plane is
Aliter Equation of plane passing through ( x1, y1, z1 ), ( x 2, y 2, z 2 ) r = $i − $j + λ( $i + $j + k$ ) + µ( $i − 2$j + 3k$ ).
and ( x 3, y 3, z 3 ) is
Let r = x$i + y$j + zk$
x − x1 y − y1 z − z1
= x 2 − x1 y 2 − y1 z 2 − z1 = 0 Hence, the equation is
x 3 − x1 y 3 − y1 z 3 − z1 ( xi$ + y$j + zk$ ) − ( i$ − $j) = λ( i$ + $j + k$ ) + µ( i$ − 2 $j + 3 k$ )

x −1 y −1 z −1 Thus, vectors ( xi$ + y$j + zk$ ) − ( $i − $j), i$ + $j + k$ , i$ − 2 $j + 3 k$ are


i.e. 3 − 1 −1 − 1 2 − 1 = 0 coplanar.
−3 − 1 5 − 1 −4 − 1 Therefore, the equation of the plane is
x − 1 y − (− 1) z − 0
On solving, we get x + y = 2
1 1 1 =0
1 −2 3
Equation of a Plane Passing Through or 5 x − 2y − 3z − 7 = 0
a Given Point and Parallel to Two Intercept Form of a Plane
Given Vectors The equation of a plane having intercepting lengths a, b
Let a plane pass through A (a ) and is parallel to the plane and c with X-axis, Y -axis and Z-axis, respectively is
formed by two vectors b and c. Since, AP lies in the plane x y z
and b and c are two non-collinear vectors, + + =1
a b c
AP = λb + µc
Proof Let O be the origin and let OX, OY and OZ be the
⇒ r − a = λb + µc coordinate axes.
⇒ r = a + λb + µc Let the plane meets the coordinate axes at the points
P, Q and R, respectively such that
b c OP = a, OQ = b and P = c . Then, the coordinates of the
P(r) points are P (a, 0, 0 ), Q (0, b, 0 ) and R(0, 0, c ).
A(a)
Let the equation of plane be
Ax + By + Cz + D = 0 ...(i)
O
Since, Eq. (i) passes through (a, 0, 0 ), (0, b, 0 ) and (0, 0, c ), we
Here, λ and µ are arbitrary scalars. have
This form is also called the parametric form of the plane. −D
Aa + D = 0 ⇒ A =
It can also be written in the non-parametric form as a
( r − a ) ⋅ ( b × c) = 0 −D
Bb + D = 0 ⇒ B =
or [ r b c ] = [a b c ] b
Chap 03 Three Dimensional Coordinate System 193

−D
Cc + D = 0 ⇒ C =
b Angle between Two Planes
On putting these values in Eq. (i),we get required equation Vector Form
of plane as
The angle between two planes is defined as the angle
−D D D
x − y − z =−D between their normals.
a b c n1
x y z
⇒ + + =1
a b c
90°–q
y Example 55. A plane meets the coordinates axes in
q
A, B and C such that the centroid of the ∆ABC is the 90°–q

point (p , q, r ), show that the equation of the plane is


x y z n2
+ + =3
p q r
Sol. Let the required equation of plane be Let θ be the angle between planes r ⋅ n 1 = d 1 and
x y z
+ + =1 ...(i) n1 ⋅ n2
a b c r ⋅ n 2 = d 2 then cos θ =
| n1 | | n 2 |
Then, the coordinates of A, B and C are A(a, 0, 0 ), B( 0, b, 0 ) and
C ( 0, 0, c ), respectively. So, the centroid of the ∆ABC,
a b c
Condition for Perpendicularity
 , , 
 3 3 3 If the planes r ⋅ n 1 = d 1 and r ⋅ n 2 = d 2 are perpendicular,
But the coordinate of the centroid are ( p, q, r ). then n 1 and n 2 are perpendicular. Therefore, n 1 ⋅ n 2 = 0
a b c
∴ = p, = q, = r Condition for Parallelism
3 3 3
On putting the values of a, b and c in Eq. (i), we get If the planes r ⋅ n 1 = d 1 and r ⋅ n 2 = d 2
The required plane as are parallel, there exists the scalar λ such that n 1 = λn 2 .
x y z
+ + =1 Cartesian Form
3 p 3q 3r
x y z If the planes are a 1 x + b 1 y + c 1 z + d = 0
⇒ + + =3
p q r and a 2 x + b 2y + c 2z + d 2 = 0
a 1a 2 + b 1b 2 + c 1c 2
y Example 56. A variable plane moves in such a way ⇒ cos θ =
that the sum of the reciprocals of its intercepts on the a 12 + b 12 + c 12 a 22 + b 22 + c 22
three coordinate axes is constant. Show that the plane Condition for parallelism
passes through the fixed point. a1 b1 c 1
x y z
= = =λ
Sol. Let the equation of the plane be + + = 1. Then, the a2 b2 c 2
a b c
intercepts made by the plane with axes area, b and c. Condition for perpendicularity
1 1 1 a 1a 2 + b 1b 2 + c 1c 2 = 0
∴ + + = constant (k ) ...(i) (given)
a b c
1 1 1 x y z y Example 57. Find the angle between the planes
⇒ + + = 1 comparing with + + =1
ak bk ck a b c 2x + y − 2x + 3 = 0 and r ⋅(6 $i + 3$j + 2k$ ) = 5.
1 1
x = ,y = Sol. Normals along the given planes are 2$i + $j − 2k$ and
k k
6$i + 3$j + 2k$
1
and z=
k Then angle between planes,
 1 1 1 (2 i$ + $j − 2 k$ ) ⋅(6 $i + 3 $j + 2 k$ ) 11
This shows Eq. (i) passes through the fixed point  , ,  . θ = cos− 1 = cos−1
 k′ k′ k′  (2 ) + (1 ) + ( − 2 ) (6 ) + (3 ) + (2 )
2 2 2 2 2 2 21
194 Textbook of Vector & 3D Geometry

y Example 58. Show that ax + by + r = 0, Equation of any Plane Passing


by + cz + p = 0 and cz + ax + q = 0 are perpendicular to
XY , YZ and ZX planes, respectively.
Through the Line of Intersection
Sol. The planes a1x + b1y + c 1z + d1 = 0 and of Two Plane
a 2 x + b 2y + c 2z + d 2 = 0 are perpendicular to each other if The equation of the plane passing through the line of
and only if a1a 2 + b1b 2 + c 1c 2 = 0. intersection of the planes
The equation of XY , YZ and ZX planes are z = 0, x = 0 and
a 1 x + b 1 y + c 1 z + d 1 = 0 and a 2 x + b 2 y + c 2 z + d 2 = 0 is
y = 0, respectively.
(a 1 x + b 1 y + c 1 z + d 1 ) + k(a 2 x + b 2 y + c 2 z + d 2 ) = 0
Now, we have to show that z = 0 is perpendicular to
ax + by + r = 0.
Proof Let the given plane be
It follows immediately, since a( 0 ) + b( 0 ) + ( 0 ) (1 ) = 0, other parts a 1 x + b 1y + c 1z + d 1 = 0 ...(i)
can be done similarly. and a 2 x + b 2y + c 2z + d 2 = 0 ...(ii)
∴ Required plane is (a 1 x + b 1 y + c 1 z + d 1 )
Family of Planes + k(a 2 x + b 2 y + c 2 z + d 2 ) = 0
Clearly, plane (iii) represents the equation of plane.
...(iii)

Plane Parallel to a Given Plane Let (α, β, γ) be a point on the line of intersection of planes
Since parallel planes have the same normal vector, so (i) and (ii), then P lies on planes (i) and (ii).
equation of a plane parallel to r ⋅ n = d 1 is of the form ∴ a 1α + b 1β + c 1 γ + d 1 = 0 ...(iv)
r ⋅ n = d 2 , where d 2 is determined by the given conditions. and a 2α + b 2β + c 2 γ + d 2 = 0 ...(v)
In cartesian form, if ax + by + cz + d = 0 be the given plane Now, multiply by k in plane (v) and then adding planes
then the plane parallel to this plane is ax + by + cz + k = 0. (iv) and (v), we get
y Example 59. Find the equation of the plane through (a 1 α + b 1 β + c 1 γ + d 1 )
the point (1, 4, − 2) and parallel to the plane + k(a 2 α + b 2 β + c 2 γ + d 2 ) = 0
− 2x + y − 3z = 7. ⇒ P(α, β, γ ) lies on plane (iii).
Sol. Let the equation of a plane parallel to the plane Hence, plane (iii) passes through each point on the line of
− 2x + y − 3z = 7 be intersection of planes (i) and (ii).
− 2 x + y − 3z + k = 0 ...(i)
Thus, plane (iii) is the equation of plane passing through
This passes through (1, 4, − 2), therefore the line of intersection of planes (i) and (ii).
( − 2 ) (1 ) + 4 − 3 ( − 2 ) + k = 0
⇒ −2 + 4 + 6 + k = 0 ⇒ k = −8 Vector Form
Putting k = − 8 in Eq. (i), we obtain Equation of planes passing through the line of intersection
− 2 x + y − 3z − 8 = 0 or − 2 x + y − 3z = 8 of planes
This is the equation of the required plane. r ⋅ n 1 = d 1 and r ⋅ n 2 = d 2 is
y Example 60. Find the equation of the plane passing ( r ⋅ n 1 − d 1 ) + k( r ⋅ n 2 − d 2 ) = 0
through ( 3, 4, − 1), which is parallel to the plane or r ⋅ ( n 1 + kn 2 ) = d 1 + kd 2 , k being any scalar.
r ⋅(2$i − 3$j + 5k$ ) + 7 = 0.
y Example 61. Find the equation of the plane
Sol. The equation of any plane which is parallel to containing the line of intersection of the plane
r ⋅ (2$i − 3$j + 5k$ ) + 7 = 0 is x + y + z − 6 = 0 and 2x + 3y + 4z + 5 = 0 and passing
r ⋅(2 $i − 3 $j + 5 k$ ) + λ = 0 through the points (1, 1, 1).
or 2 x − 3y + 5z + λ = 0 Sol. The equation of a plane through the line of intersection of
the given plane is
Further (i) will pass through (3, 4, − 1 )
( x + y + z − 6 ) + λ(2 x + 3y + 4z + 5 ) = 0 ...(i)
if (2 ) (3 ) + ( − 3 ) ( 4 ) + 5 ( − 1 ) + λ = 0
If line (i) passes through (1, 1, 1), we have
or − 11 + λ = 0 ⇒ λ = 11
− 3 + 14 λ = 0
Thus, equation of the required plane is
3
r ⋅ (2 $i − 3 $j + 5 k$ ) + 11 = 0 ⇒ λ= .
14
Chap 03 Three Dimensional Coordinate System 195

Putting λ =
3
in line (i), we obtain the equation of the  mx + nx 1 my + ny 1 mz 2 + nz 1 
14 Then, R  2 , , 
required plane as
 m +n m +n m +n 
3 Since, R lies on the plane (i).
( x + y + z − 6 ) + (2 x + 3y + 4z + 5 ) = 0
14
 mx + nx 1   my 2 + ny 1   mz 2 + nz 1 
⇒ 20 x + 23y + 26z − 69 = 0 ∴a  2  +b   +c   +d =0
 m +n   m +n   m +n 
y Example 62. Find the planes passing through the ⇒ a(mx 2 + nx 1 ) + b(my 2 + ny 1 )
intersection of planes r ⋅(2$i − 3$j + 4k$ ) = 1 and + c (mz 2 + nz 1 ) + d (m + n ) = 0
r ⋅( $i − $j ) + 4 = 0 and perpendicular to planes ⇒ m(ax 2 + by 2 + cz 2 + d )
r ⋅(2$i − $j + k$ ) = − 8. + n(ax 1 + by 1 + cz 1 + d ) = 0
Sol. The equation of any plane through the line of intersection m (ax 1 + by 1 + cz 1 + d )
⇒ =− ...(ii)
of the given planes is n (ax 2 + by 2 + cz 2 + d )
{ r ⋅(2 $i − 3 $j + 4 k$ ) − 1 } + λ { r ⋅( i$ − $j) + 4 } = 0
Now, if ax 1 + by 1 + cz 1 + d and ax 2 + by 2 + cz 2 + d
r ⋅ {(2 + λ)$i − (3 + λ)$j + 4 k$ } = 1 − 4λ ...(i) m
are of same sign <0 (external division)
If it is perpendicular to r ⋅(2 i$ − $j + k$ ) + 8 = 0, then n
{(2 + λ)$i − (3 + λ)$j + 4 k$ } ⋅ (2 i$ − $j + k$ ) = 0 m
are of opposite sign >0 (internal division)
2(2 + λ) + (3 + λ) + 4 = 0 n
− 11 ax 1 + by 1 + cz 1 + d
λ= ∴ If >0 (same side)
3 ax 2 + by 2 + cz 2 + d
11
Putting λ = − in line (i), we obtain the equation of the ax 1 + by 1 + cz 1 + d
3 <0 (opposite side)
required plane as r ⋅( − 5 $i + 2 $j + 12 k$ ) = 47 ax 2 + by 2 + cz 2 + d

y Example 63. Find the interval of α for which


Two Sides of a Plane (α, α 2 , α) and ( 3, 2, 1) lies on same side of
x + y − 4z + 2 = 0.
Let ax + by + cz + d = 0 be the plane, then the points
( x 1 , y 1 , z 1 ) and ( x 2 , y 2 , z 2 ) lie on the same side or opposite Sol. (α, α 2 , α ) and (3, 2, 1) lies on same side of x + y − 4z + 2 = 0
side according as ∴ (α + α 2 − 4α + 2 ) (3 + 2 − 4 + 2 ) > 0
ax 1 + by 1 + cz 1 + d ⇒ α 2 − 3α + 2 > 0
> 0 or < 0
ax 2 + by 2 + cz 2 + d (α − 1 ) (α − 2 ) > 0 ⇒ α ∈ ( − ∞, 1 ] ∪ (2, ∞ ]
Proof Here equation of plane is,
ax + by + cz + d = 0
Let Eq. (i) divide the line segment joining P and Q at R
...(i)
Distance of a Point from a Plane
internally in the ratio m : n. Vector Form
P (x1, y1, z1) The length of the perpendicular from a point having
position vector a to the plane r ⋅ n = d is given by
| a⋅ n − d |
P=
p |n|
m
n Proof. Let π be the given plane and P(a ) be the given
R
point. Let PM be the length of perpendicular from P to the
plane π.
Since, line PM passes through P(a ) and is parallel to the
vector n which is normal to the plane π. So, vector
Q (x2, y2, z2) equation of line PM is
r = a + λn ...(i)
196 Textbook of Vector & 3D Geometry

P (a)
= (a 2 + b 2 + c 2 ) r 2 = a 2 + b 2 + c 2 | r |
− (ax 1 + by 1 + cz 1 + d )
= a2 + b2 + c2
p a2 + b2 + c2
[from Eq. (ii)]
M | (ax 1 + by 1 + cz 1 + d ) |
∴ PM =
a2 + b2 + c2
Point M is the intersection of Eq. (i) and the given plane π.
y Example 64. Find the distance of the point (2, 1, 0)
∴ (a + λn ) ⋅ n = d
from the plane 2x + y + 2z + 5 = 0.
d − (a ⋅ n )
⇒ a ⋅ n + λn ⋅ n = d ⇒ λ = Sol. We know that the distance of the point ( x 1, y1, z1 ) from the
| n |2
plane ax + by + cz + d = 0 is
On putting the value of λ in Eq. (i), we obtain the position | ax1 + by1 + cz1 + d |
vector of M given by a2 + b2 + c2
 d − a⋅ n  |2 ×2 + 1 + 2 × 0 + 5| 10
r= a + n So, required distance = = .
 | n |2  2 +1 +2
2 2 2 3

PM = Position vector of M − Position vector of P


 d − (a ⋅ n )  Distance between the Parallel Planes
=a +   n− a
 | n |2  The distance between two parallel planes
 d − (a ⋅ n )  ax + by + cz + d 1 = 0
PM =  n
 | n |2  and ax + by + cz + d 2 = 0

(d − a ⋅ n ) n (d 2 − d 1 )
⇒ PM = | PM | = is given by d=
| n |2 a2 + b2 + c2
| d − (a ⋅ n ) | | n | | d − (a ⋅ n ) | Proof. Let d = Difference of the length of perpendicular
= 2
=
|n| |n| from origin to the two planes.
Thus, the length of perpendicular from a point having | d1 | | d2 |
| a⋅ n − d | = − .
position vector a on the plane r ⋅ n = d is a +b +c
2 2 2
a + b2 + c2
2
|n|
d1 − d2
Cartesian Form if d 1 and d 2 are of same side =
a2 + b2 + c2
The length of perpendicular from a point P ( x 1 , y 1 , z 1 ) to
the plane ax + by + cz + d = 0. Then, the equation of PM is
Vector Form
x − x 1 y − y1 z − z1
= = ...(i) The distance between two parallel plane r ⋅ n = d 1
a b c
and r ⋅ n = d 2 is given by
The coordinates of any point on this line are
| d − d2 |
( x 1 + ar , y 1 + br , z 1 + cr ) d= 1
|n|
Thus, the point coincides with M iff it lies on plane.
i.e. a( x 1 + ar ) + b(y 1 + br ) + c (z 1 + cr ) + d = 0 y Example 65. Find the distance between the parallel
(ax 1 + by 1 + cz 1 + d ) planes x + 2y − 2z + 1 = 0 and 2x + 4 y − 4z + 5 = 0.
i.e. r =− ...(ii)
a2 + b2 + c2 Sol. We know that, distance between parallel planes
ax + by + cz + d1 = 0 and ax + by + cz + d 2 = 0 is,
Now, PM = ( x 1 + ar − x 1 ) 2 + (y 1 + br − y 1 ) 2 | d1 − d 2 |
+ (z 1 + cr − z 1 ) 2 a2 + b2 + c2
Chap 03 Three Dimensional Coordinate System 197

∴ Distance between x + 2y − 2z + 1 = 0
5
Bisector of the Angle between the
and x + 2y − 2z + = 0 is
2 Two Planes Containing the Origin
5
−1 Let the equation of the two planes be
2 1
= a 1 x + b 1y + c 1z + d 1 = 0 ...(i)
1+4+4 2
and a 2 x + b 2y + c 2z + d 2 = 0 ...(ii)
where, d 1 and d 2 are positive, then equation of the
Equation of Planes Bisecting bisector of the angle between the planes (i) and (ii)
containing the origin is
the Angle between Two Planes a 1 x + b 1y + c 1z + d 1 a 2 x + b 2y + c 2z + d 2
=
Equation of the planes bisecting the angle between the a 12 + b 12 + c 12 a 22 + b 22 + c 22
planes.
a 1 x + b 1 y + c 1 z + d 1 = 0 and a 2 x + b 2 y + c 2 z + d 2 = 0 is
a 1 x + b 1y + c 1z + d 1 a x + b 2y + c 2z + d 2
Bisector of the Acute and Obtuse
=± 2 Angles between Two Planes
a1 + b1 + c 1
2 2 2
a 22 + b 22 + c 22
Let the two planes be
Proof. Given planes are
a 1 x + b 1y + c 1z + d 1 = 0 ...(i)
a 1 x + b 1y + c 1z + d 1 = 0 ...(i)
and a 2 x + b 2y + c 2z + d 2 = 0 ...(ii)
and a 2 x + b 2y + c 2z + d 2 = 0 ...(ii)
where, d 1 and d 2 > 0
Let P ( x , y , z ) be a point on the plane bisecting the angle
between planes (i) and (ii). (a) If a 1 a 2 + b 1 b 2 + c 1 c 2 > 0, the origin lies in the obtuse
angle between the two planes and the equation of
Let PL and PM be the length of perpendiculars from P to bisector of the acute angle is,
planes (i) and (ii).
a 1 x + b 1y + c 1z + d 1 a x + b 2y + c 2z + d 2
∴ PL = PM =− 2
a1 + b1 + c 1
2 2 2
a 22 + b 22 + c 22
a 1 x + b 1y + c 1z + d 1 a 2 x + b 2y + c 2z + d 2
⇒ = (b) If a 1 a 2 + b 1 b 2 + c 1 c 2 < 0, then origin lies in the acute
a 12 + b 12 + c 12 a 22 + b 22 + c 22 angle between the two planes and the equation of
a 1 x + b 1y + c 1z + d 1 a 2 x + b 2y + c 2z + d 2 bisector of the acute angle between two planes is
=± a 1 x + b 1y + c 1z + d 1 a x + b 2y + c 2z + d 2
a 12 + b 12 + c 12 a 22 + b 22 + c 22 =+ 2
a1 + b1 + c 1
2 2 2
a 22 + b 22 + c 22
This is equation of planes bisecting the angles between the
planes (i) and (ii). y Example 66. Find the equation of the bisector
Vector Form planes of the angles between the planes
2x − y + 2z + 3 = 0 and 3x − 2y + 6z + 8 = 0 and specify
Equation of planes bisecting the angle between planes the plane which bisects the acute angle and the plane
r ⋅ n 1 = d 1 and r ⋅ n 2 = d 2 are which bisects the obtuse angle.
r⋅ n1 − d 1 r ⋅ n2 − d2 Sol. The two given planes are
=
n1 n2 2 x − y + 2z + 3 = 0 and 3 x − 2y + 6z + 8 = 0
where, d1 and d 2 > 0
r⋅ n1 − d 1 r⋅ n2 − d 2
⇒ =± and a1a 2 + b1b2 + c1c 2 = 6 + 2 + 12 > 0
n1 n2
a1x + b1y + c1z + d1 a x + b2y + c 2z + d 2
∴ =− 2
n1 n d d a12 + b12 + c12 a 22 + b22 + c 22
⇒ r⋅ ±r⋅ 2 = 1 ± 2
| n1 | | n1 | | n1 | | n2 |
(obtuse angle bisector)
d d a1x + b1y + c1z + d1 a 2x + b2y + c 2z + d 2
⇒ r ⋅ ( n$ 1 ± n$ 2 ) = 1 ± 2 and =
| n1 | | n2 | a12 + b12 + c12 a 22 + b22 + c 22
(acute angle bisector)
198 Textbook of Vector & 3D Geometry

2 x − y + 2z + 3 3 x − 2y + 6z + 8
i.e.
4+1+4

9 + 4 + 36 Angle between a Line and a Plane
n
⇒ (14 x − 7y + 14z + 21 ) = ± (9 x − 6y + 18z + 24 )
Taking positive sign on the right hand side, we get Normal
b
5 x − y − 4z − 3 = 0 (obtuse angle bisector) Line
and taking negative sign on the right hand side, we get
23 x − 13y + 32z + 45 = 0 (acute angle bisector)
q
90°– q
Line and Plane
Line of Intersection of Two Planes
Let two non-parallel planes are r ⋅ n 1 = d 1 and r ⋅ n 2 = d 2 Plane

n2 The angle between a line and a plane is the complement of


the angle between the line and the normal to the plane.
n1×n2
If the equation of the line is r = a + λb and that of the
plane is r ⋅ n = d, then angle θ between the line and the
b⋅n
normal to the plane is cos θ = .
| b || n |
So, the angle φ between the line and the plane is given by
n1 90° −θ.
b⋅ n b⋅ n
Now line of intersection of planes is perpendicular to sin φ = or φ = sin − 1
| b|| n | | b|| n |
vector n 1 and n 2 .
∴ Line of intersection is parallel to vector n 1 × n 2 . Line r = a + λb and plane r ⋅ n = d are perpendicular if
b = λn or b × n = 0 and parallel if b ⊥ n or b ⋅ n = 0.
If we wish to find the equation of line of intersection of
planes a 1 x + b 1 y + c 1 z − d 1 = 0 and y Example 68. Find the angle between the line
a 2 x + b 2 y + c 2 z − d 2 = 0, then we find any point on the r = $i + 2$j − k$ + λ( $i − $j + k$ ) and the plane
line by putting z = 0 (say), then we can find corresponding
values of x and y be solving equations a 1 x + b 1 y − d 1 = 0 r ⋅(2$i − $j + k$ ) = 4.
and a 2 x + b 2 y − d 2 = 0. Thus, by fixing the value of z = λ, Sol. We know that if θ is the angle between the lines r = a + λb
we can find the corresponding value of x and y in terms of b⋅ n
λ. After getting x, y and z in terms of λ, we can find the and r ⋅ n = p, then sin θ =
| b || n |
equation of line in symmetric form.
Therefore, if θ is the angle between r = i$ + 2 $j − k$ + λ( $i − $j + k$ )
y Example 67. Reduce the equation of line and r ⋅(2 i$ − $j + k$ ) = 4, then
x − y + 2z = 5 and 3x + y + z = 6 in symmetrical form. ( $i − $j + k$ ) ⋅ (2 $i − $j + k$ )
Or sin θ =
| i$ − $j + k$ | | 2 i$ − $j + k$ |
Find the line of intersection of planes x − y + 2z = 5
2+1+1 4 4
and 3x + y + z = 6. = = =
1+1+1 4+1+1 3 6 3 2
Sol. Given x − y + 2z = 5, 3x + y + z = 6.
 4 
Let z =λ ⇒ θ = sin − 1  
3 2
Then, x − y = 5 − 2λ
and 3 x + y = 6 − λ.
Solving these two equations, 4 x = 11 − 3 λ Intersection of a Line and a Plane
and 4y = 4 x − 20 + 8 λ = − 9 + 5 λ To find the point of intersection of the line
4 x − 11 4y + 9 z − 0
= =
x − x 1 y − y1 z − z1
The equation of the line is
−3 5 1
. = = and the plane
l m n
Chap 03 Three Dimensional Coordinate System 199

ax + by + cz + d = 0. Putting r = 0 in (i), we obtain (2, − 1, 2 ) as the coordinates of the


x - x1 y - y1 z - z1 point of intersection of the given line and plane.
= =
l m n
Required distance = distance between points ( − 1, − 5, − 10 ) and
(2, − 1, 2 ) = (2 + 1 ) 2 + ( − 1 + 5 ) 2 + (2 + 10 ) 2
= 9 + 16 + 144 = 169 = 13.
P ax+by+cz+d=0

Coplanarity of Two Lines


x − x 1 y − y1 z − z1
Let = = =r x − x 1 y − y1 z − z1
l m n The straight line = = lies in a given
l m n
∴ (x = rl + x 1 , y = mr + y 1 , z = nr + z 1 ) ...(i) plane ax + by + cz + d = 0 if ax 1 + by 1 + cz 1 + d = 0
be a point in the plane say P. and al + bm + cn = 0
It must satisfy the equation of plane.
∴ a( x 1 + lr ) + b(y 1 + mr ) + c (z 1 + nr ) + d = 0
⇒ (ax 1 + by 1 + cz 1 + d ) + r (al + bm + cn ) = 0
(ax 1 + by 1 + cz 1 + d )
⇒ r =−
al + bm + cn
On substituting the value of r Eq. (i), we get the (x1, y1, z1)

coordinates of the required point of intersection.


ax+by+az+d=0
(i) Condition for a Line to be Parallel to a
Thus, the general equation of the plane containing a
Plane
straight line
x − x1 y − y1 z − z1
Let line = = be parallel to plane x − x 1 y − y1 z − z1
l m n = = is
l m n
ax + by + cz + d =0 iff;
a( x − x 1 ) + b(y − y 1 ) + c (z − z 1 ) = 0
θ = 0 or π or sin θ = 0 ⇒ al + bm + cm = 0
where, al + bm + cn = 0
(ii) Condition for a Line to Lie in the Plane The equation of the plane containing a straight line
x − x 1 y − y1 z − z1 x − x 1 y − y1 z − z1
Condition for = = to lie in the = = and parallel to the straight line
l m n l m n
plane ax + by + cz + d = 0 are x − x2 y − y2 z − z2
= = is
al + bm + cn = 0 and ax 1 + by 1 + cz 1 + d = 0 l1 m1 n1
x − x1 y − y1 z − z1
Note
l m n =0
A line will be in a plane iff
(i) the normal to the plane is perpendicular to the line. l1 m1 n1
(ii) a point on the line in the plane.
Hence, the equation of the plane containing two given
y Example 69. Find the distance between the point straight lines
with position vector − $i − 5$j − 10 k$ and the point of x − x 1 y − y1 z − z1
= =
x −2 y + 1 z −2 l m n
intersection of the line = = with the x − x2 y − y2 z − z2
3 4 12 and = =
plane x − y + z = 5. l1 m1 n1
Sol. The coordinates of any point on the line x − x1 y − y1 z − z1
x −2 y +1 z −2
= = = r (say) are (3r + 2, 4r − 1, 12r + 2) l m n =0
3 4 2
If it lies on the plane x − y + z = 5, then
l1 m1 n1
3r + 2 − 4r + 1 + 12r + 2 = 5 ⇒11r = 0 ⇒r = 0.
200 Textbook of Vector & 3D Geometry

x − x2 y − y2 z − z2 x − x1 y − y1 z − z1
l1 m1 n1 =0
or l m n =0
l2 m2 n2
l1 m1 n1
Here, x1 = − 1, y1 = − 3, z1 = − 5,
If the lines r = a 1 + λb1 and r = a 2 + λb2 are coplanar, x 2 = 2, y 2 = 4, z 2 = 6, l1 = 3,
then
m1= 5, n1= 7, l 2 = 1, m2 = 4, n2 = 7.
[a 1 b 1 b 2 ] = [a 2 b 1 b 2 ] x 2 − x1 y 2 − y 1 z 2 − z1 3 7 11
and the equation of plane containing them is ∴ l1 m1 n1 = 3 5 7 =0
[ r b 1 b 2 ] = [a 1 b 1 b 2 ] l2 m2 n2 1 4 7
or [ r b1 b2 ] =[a 2 b1 b2 ] so, the given lines are coplanar.
The equation of the plane containing the lines is
y Example 70. Find the equation of plane passing x+1 y +3 z +5
through the point (0, 7, –7) and containing the line
3 5 7 =0
x+1 y − 3 z +2
= = . 1 4 7
−3 2 1
ar ( x + 1 )(35 − 28 ) − (y + 3 ) (21 − 7 ) + (z + 5 ) (12 − 5 ) = 0
Sol. Let the equation of the plane passing through the point
(0, 7, − 7 ) be a( x − 0) + b(y − 7 ) + c (z + 7 ) = 0 ...(i) or x − 2y + z = 0.

x + 1 y −3 z +2
The line = = passes through the point
−3 2 1 Image of a Point in a Plane
( − 1, 3, − 2) and has direction ratios − 3, 2, 1. If (i) contains To find the image of the point (α, β, γ ) in the plane
this line, it must pass through ( − 1, 3, − 2) and must be
parallel to the line. Therefore, ax + by + cz + d = 0 ...(i)
a( − 1 ) + b(3 − 7 ) + c( − 2 + 7 ) = 0 Let Q ( x 1 , y 1 , z 1 ) be the image of point P in the plane (i).
i.e. a( − 1 ) + b( − 4 ) + c(5 ) = 0 ...(ii)
P (a, b, g)
and − 3a + 2b + 1c = 0 ...(iii)
On solving Eqs. (ii) and (iii) by cross multiplication, we get
a b c a b c
= = ⇒ = = =λ (say)
− 14 − 14 − 14 1 1 1
L
⇒ a = λ, b = λ, c = λ
Putting the values of a, b, c in (i), we obtain
λ( x − 0 ) + λ(y − 7 ) + λ(z + 7 ) = 0
⇒ x+ y + z = 0
Q (x1, y1, z1)
This is the equation of the required plane.

x+1 y + 3 z + 5 Let PQ meet plane (i) at L, direction ratios of normal to


y Example 71. Prove that the lines = =
3 5 7 plane (i) are (a, b, c), since PQ perpendicular of plane (i).
x −2 y −4 z −6 So, direction ratios of PQ are a, b, c.
and = = are coplanar. Also, find the
1 4 7 ⇒ Equation of line PQ is,
plane containing these two lines.
x −α y −β z − γ
x − x 1 y − y 1 z − z1 = = =r (say)
Sol. We know that, the line = = a b c
l1 m1 n1
Coordinate of any point on line PQ may be taken as
x − x2 y − y 2 z − z 2
and = = are coplanar if
l2 m2 n2 (ar + α, br + β, cr + γ )
x 2 − x1 y 2 − y1 z 2 − z1 Let Q (ar + α, br + β, cr + γ )
l1 m1 n1 =0 Since, L is the middle point of PQ
l2 m2 n2
 ar br cr 
∴ L = α + , β + , γ + 
and the equation of the plane containing these two lines is  2 2 2
Chap 03 Three Dimensional Coordinate System 201

Since, L lies on plane (i), we get Let the coordinates of the foot of the perpendicular from the
point P(7, 14, 5 ) be M(α, β, γ ).
 ar   br   cr 
a  + α  + b  + β + c  + γ  + d = 0 Then, the direction ratios of PM are α − 7, β −14 and γ − 5.
2  2  2 
Therefore, the direction ratios of the normal to the plane are
r α −7, β − 14 and γ −5.
⇒ (a 2 + b 2 + c 2 ) = − (aα + bβ + cγ + d )
2 But the direction ratios of normal to the given plane
2 x + 4y − z = 2 are 2, 4 and − 1.
− 2(aα + bβ + cγ + d )
⇒ r= Hence,
α − 7 β − 14 γ − 5
= = =k
a2 + b2 + c2 2 4 −1
∴ α = 2k + 7, β = 4k + 14 and γ = − k + 5 ...(i)
y Example 72. Find the image of the point P( 3, 5, 7 ) in
Since, α, β and γ lie on the plane 2 x + 4y − z = 2,
the plane 2x + y + z = 0. 2α + 4β − γ = 2
Sol. Given plane is 2x + y + z = 0 ...(i) ⇒ 2(7 + 2k ) + 4(14 + 4k ) − (5 − k ) = 2
and the point P(3, 5, 7 ) ⇒ 14 + 4k + 56 + 16k − 5 + k = 2
DR’s of normal to the plane (i) are 2, 1, 1. ⇒ 21k = − 63 ⇒ k = − 3
Let Q be the image of a point P in plane (i). Now, putting k = − 3 in (i), we get α = 1, β = 2, γ = 8
Hence, the foot of the perpendicular is (1, 2, 8).
P (3, 5, 7)
x − 1 y −2
y Example 74. Find the image of the line =
9 −1
z+3
= in the plane 3x − 3y + 10z − 26 = 0.
R
−3
Sol.
A(1, 2, –3)

x −3 y −5 z −7 B A¢
∴Equation of line PR is = = =r
2 1 1
Let R(2r + 3, r + 5, r + 7 )
Since, R lies on plane (i). x −1 y − 2 z + 3
= = …(i)
∴ 2(2r + 3 ) + (r + 5 ) + (r + 7 ) = 0 ; 6r + 18 = 0 9 −1 −3
∴ r = − 3 ∴ R ≡ ( − 3, 2, 4 ) 3 x − 3y + 10z − 26 = 0 …(ii)
Let Q ≡(α, β, γ ) The direction ratios of the line are 9, − 1 and − 3 and direction
Since, R is mid-point of PQ. ratios of the normal to the given plane are 3, − 3 and 10.
α+3 Since, 9 ⋅ 3 + ( − 1 ) ( − 3 ) + ( − 3 ) 10 = 0 and the point (1, 2, − 3 ) of
∴ −3 = ⇒ α = −9
2 line (i) does not lie in plane (ii) for
β+5 3 ⋅ 1 − 3 ⋅ 2 + 10 ⋅ ( − 3 ) − 26 ≠ 0, line (i) is parallel to plane (ii). Let
2= ⇒ β = −1 A′ be the image of point A(1, 2, − 3 ) in plane (ii). Then the
2 image of the line (i) in the plane (ii) is the line through A′ and
γ+7 parallel to the line (i).
4= ⇒ γ =1
2 Let point A′ be (p, q, r). Then
∴ Q ≡ ( − 9, 1, 1 ) p −1 q −2 r + 3
= =
3 −3 10
y Example 73. Find the length and the foot of the
perpendicular from the point (7, 14, 5) to the plane (3 (1 ) − 3 (2 ) + 10 ( − 3 ) − 26 ) 1
=− =
2x + 4 y − z = 2. 9 + 9 + 100 2
2(7 ) + 4(14 ) − (5) − 2 5 1 
Sol. The required length = The point A′  , , 2
2 2 
2 + 4 +1
2 2 2
 5  1
x −  y − 
14 + 56 − 5 − 2 63  2  2 z −2
= = The equation of line BA′ is = =
4 + 16 + 1 21 9 −1 −3
202 Textbook of Vector & 3D Geometry

Exercise for Session 3


1. Find the equation of plane passing through the point (1, 2, 3) and having the vector r = 2$i − $j +3k$ normal to it.

2. Find a unit vector normal to the plane through the points (1, 1, 1), (− 1, 2, 3) and (2, − 1, 3).

3. Show that the four points (0, − 1, 0), (2, 1, − 1), (1, 1, 1) and (3, 3, 0) are coplaner. Also, find equation of plane
through them.

4. Find the equation of plane passing through the line of intersection of planes 3x + 4y − 4 = 0 and
x + 7y + 3z + z = 0 and also through origin.
5. Find equation of angle bisector of plane x + 2y + 3z − z = 0 and 2x − 3y + z + 4 = 0.

6. Find image of point (1, 3, 4) in the plane 2x − y + z + 3 = 0.


x + 1 y z −3
7. Find the angle between the line = = and the plane 3x + y + z = 7.
2 3 6
8. Find the equation of plane which passes through the point (1, 2, 0) and which is perpendicular to the plane
x − y + z = 3 and 2x + y − z + 4 = 0.
x −2 y + 1 z −2
9. Find the distance of the point ( − 1, − 5, − 10) from the point of intersection of the line = = and
3 4 12
plane x − y + z = 5.
x −5 y +7 z + 3 x − 8 y −4 z − 5
10. Find the equation of a plane containing the lines = = and = = .
4 4 −5 7 1 3
x +1 y −1
11. Find the equation of the plane which passes through the point (3, 4, − 5) and contains the line =
2 3
z +2
= .
−1

12. Find the equation of the planes parallel to the plane x − 2y + 2z − 3 = 0. Which is at a unit distance from the
point (1, 2, 3).

13. Find the equation of the bisector planes of the angles between the plane x + 2y + 2z = 19 and
4x − 3y + 12z + 3 = 0 and specify the plane which bisects the acute angle and the plane which bisects the
obtuse angle.

14. Find the equation of the image of the plane x − 2y + 2z = 3 in the plane x + y + z = 1.

15. Find the equation of a plane which passes through the point (1, 2, 3) and which is at the maximum distance
from the point (− 1, 0, 2).
Session 4
Sphere
Sphere Proof. Let C be the centre of the sphere.
A sphere is the locus of a point which Then, coordinates of C are (a, b, c ). Let P ( x , y , z ) be any
P(r)
moves in space in such a way that its point on the sphere, then
distance from a fixed point always CP = R
C(a)
remains constant. The fixed point is called ⇒ CP 2 = R 2
the centre of the sphere and the fixed
distance is called the radius of sphere. ⇒ ( x − a ) 2 + (y − b ) 2 + (z − c ) 2 = R 2
Shown as in adjoining figure. Since, P ( x , y , z ) is an arbitrary point on the sphere,
therefore required equation of the sphere is
Equation of Sphere whose
Centre c and Radius is a ( x − a ) 2 + (y − b ) 2 + (z − c ) 2 = R 2
Let O be the origin of reference and C be the centre of Remarks
sphere whose position vector is c. Let P be any point on 1. The above equation is called the central form of a sphere. If the
the surface of the sphere whose position vector is r. centre is at the origin, then equation of sphere is,
Thus, OP = r and OC = c x 2 + y 2 + z 2 = R2

∴ CP = OP − OC = r − c (known as the standard form of the sphere)


2. Above equation can also be written as
P x 2 + y 2 + z 2 − 2ax − 2by − 2cz + ( a2 + b2 + c 2 − R2 ) = 0
r a which has the following characteristics of the equation of
sphere
O c C (i) It is a second degree equation in x, y and z.
(ii) The coefficient of x 2, y 2 and z 2 are all equal.
(iii) The term containing the product of xy, yz and zx are
absent.
Now, | r− c| =a [radius of sphere]
⇒ | r− c| =a2 2 y Example 75. Find the vector equation of a sphere
with centre having the position vector $i + $j + k$ and
⇒ ( r − c ) ⋅( r − c ) = a 2
radius 3.
⇒ r 2 − 2r ⋅ c + c 2 = a 2
Sol. We know that equation of sphere is
⇒ r 2 − 2r ⋅ c + (c 2 − a 2 ) = 0 | r− c| =a (vector form)
which is the required equation of sphere. ⇒ | r − ( i + j + k$ ) | = 3
$ $
which is the required equation of sphere.
Cartesian Equation of a Sphere
y Example 76. Find the equation of sphere whose
The equation of sphere with centre (a, b, c ) and radius R is
centre is ( 5, 2, 3) and radius is 2 in cartesian form.
( x − a ) 2 + (y − b ) 2 + (z − c ) 2 = R 2 Sol. The required equation of the sphere is
P(x, y, z) ( x − 5 ) 2 + (y − 2 ) 2 + (z − 3 ) 2 = 2 2
⇒ x 2 + y 2 + z 3 − 10 x − 4y − 6z + 34 = 0
R

C(a,b,c)
y Example 77. Find the equation of a sphere whose
centre is (3, 1, 2) and radius is 5.
204 Textbook of Vector & 3D Geometry

Sol. The equation of the sphere whose centre is (3, 1, 2) and y Example 80. Find the equation of a sphere which
radius is 5, is passes through (1, 0, 0), (0, 1, 0) and (0, 0, 1) and has
( x − 3 ) 2 + (y − 1 ) 2 + (z − 2 ) 2 = 5 2 radius as small as possible.
x 2 + y 2 + z 2 − 6 x − 2y − 4z − 11 = 0 Sol. Let the equation of the required sphere be
x 2 + y 2 + z 2 + 2ux + 2vy + 2wz + d = 0 ...(i)
General Equation of Sphere As the sphere passes through (1, 0, 0 ), ( 0,1, 0 ) and ( 0, 0, 1 ), we
The equation x 2 + y 2 + z 2 + 2ux + 2vy + 2wz + d = 0 get
represents a sphere with centre ( − u , − v , − w ) i.e. 1 + 2u + d = 0, 1 + 2v + d = 0 and 1 + 2w + d = 0
1
 1 1 ⇒ u = v = w = − (d + 1 )
 − coefficient of x , − coefficient of y , 2
 2 2
If R is the radius of the sphere, then R 2 = u 2 + v 2 + w 2 − d
1 
− coefficient ofz  3
2  ⇒ R2 = (d + 1 ) 2 − d
4
and radius = u 2 + v 2 + w 2 − d . 3  2 4 
= d + 2d + 1 − d
4  3 
Note 3  2 2 
= d + d +1
The equation x 2 + y 2 + z 2 + 2ux + 2vy + 2wz + d = 0 represents 4  3 
a real sphere, if u 2 + v 2 + w 2 − d > 0. If u 2 + v 2 + w 2 − d = 0, then 3  1
2
1
it represents a point sphere. The sphere is imaginary, if = d +  + 1 − 
4  3 9
u 2 + v 2 + w 2 − d < 0.
3  1
2
8
y Example 78. Find the centre and radius of the = d +  + 
4  3 9
sphere 2x 2 + 2y 2 + 2z 2 − 2x − 4 y + 2z + 3 = 0.
The last equation shows that R 2 (and thus R) will be the least if
Sol. The given equation 1
an only if d = − .
3 3
x 2 + y 2 + z 2 − x − 2y + z + = 0;
2 1 1 1
Therefore, u = v = w = − 1 −  = −
where cetnre is 2  3  3
 1 1 1  2
 − coefficient of x, − coefficient of y , − coefficient of z  Hence, the equation of the required sphere is x 2 + y 2 + z 2 −
 2 2 2  3
1
1 1 (x + y + z ) − = 0 or 3 ( x 2 + y 2 + z 2 ) − 2 ( x + y + z ) − 1 = 0.
∴ Centre =  , − 1, −  3
2 2

and
 1
2
 1
Radius =   + ( − 1 ) 2 +  −  −
3
2
Diameter Form of the Equation of a Sphere
 2  2 2 If the position vectors of the extremities of a diameter of a
1 1 3 sphere are a and b, then its equation is
= +1+ − =0
4 4 2 ( r − a ) ⋅( r − b) = 0
1 1
∴ Given sphere represents a point sphere  , − 1, −  . ⇒ | r |2 − r ⋅ (a + b ) + a ⋅ b = 0
2 2
Proof. Let a and b be the position vectors of the
y Example 79. Find the equation of the sphere passing extremities A and B of a diameter AB of sphere. Let r be
through (0, 0, 0), (1, 0, 0), (0, 1, 0) and (0, 0, 1). the position vector of any point P on the sphere.
Sol. Let the equation of the sphere be Then,
P(r)
x 2 + y 2 + z 2 + 2ux + 2vy + 2wz + d = 0 …(i)
As (i) passes through ( 0, 0, 0 ), (1, 0, 0 ), ( 0, 1, 0 ) and ( 0, 0, 1 ), we
must have d = 0, 1 + 2u + d = 0 B A
(b ) (a)
1 + 2v + d = 0 and 1 + 2w + d = 0
Since, d = 0, we get 2u = 2v = 2w = − 1
Thus, the equation of the required sphere is
AP = r − a and BP = r − b
x 2+ y 2 + z 2 − x − y − z = 0.
Chap 03 Three Dimensional Coordinate System 205

Since, the diameter of a sphere subtends a right at any Section of a Sphere by a Plane
point on the sphere, therefore
Consider a sphere intersected by a
π plane. The set of points common to C
⇒ ∠APB =
2 both sphere and plane is called a P
M
⇒ AP ⋅ BP = 0 plane section of a sphere.
⇒ ( r − a ) ⋅( r − b) = 0 It can be easily seen the plane
r⋅ r − r⋅ b − r⋅ a + a⋅ b = 0 section of sphere is a circle.
Let C be the centre of the sphere and M be the foot of the
| r |2 − (a + b ) ⋅ r + a ⋅ b = 0
perpendicular from C on the plane. Then, M is the centre
This is the required equation of sphere. of the circle and radius of circle is given by PM.
Vector Form i.e. PM = CP 2 − CM 2
If the position vectors of the extremities of a diameter of a The centre M of the circle is the point of intersection of
sphere are a and b, then its equation is the plane and line CM, which passes through C and is
| r − a | 2 + | r − b | 2 = | a − b |2 perpendicular to given plane.

Proof. Let a and b be the position vectors of the y Example 82. Find the radius of the circular section
extremities A and B of a diameter of a sphere. Let r be the in which the sphere | r | = 5 is cut by the plane
position vector of any point P on the sphere, then
r ⋅( $i + $j + k$ ) = 3 3.
AP = r − a
and BP = r − b Sol. Let A be the foot of the perpendicular from the centre O to
the plane r ⋅( $i + $j + k$ ) − 3 3 = 0
Since, ∆APB is a right angled triangle.
∴ AP 2 + BP 2 = AB 2
A P
⇒ | AP |2 + | BP |2 = | AB |2
O
⇒ | r − a | 2 + | r − b | 2 = | a − b |2
This is the required equation of the sphere.
0 ⋅ ( i$ + $j + k$ ) − 3 3 3 3
Cartesian Form Then, | OA | = = = 3 (perpendicular
| $i + $j + k$ | 3
If ( x 1 , y 1 , z 1 ) and ( x 2 , y 2 , z 2 ) are the coordinates of the distance of a point from the plane)
extremities of a diameter of a sphere, then its equation is, If P is any point on the circle, then P lies on the plane as well
( x − x 1 ) ( x − x 2 ) + (y − y 1 ) (y − y 2 ) + (z − z 1 ) (z − z 2 ) = 0 as on the sphere. Therefore, OP = radius of the sphere = 5
Now, AP 2 = OP 2 − OA 2 = 5 2 − 3 2 = 16
y Example 81. Find the equation of the sphere
⇒ AP = 4
described on the joint of points A and B having
Position position vectors 2$i + 6 $j − 7k$ and − 2$i + 4 $j − 3k,
$ y Example 83. Find the centre of the circle given by
respectively, as the diameter. Find the centre and the r ⋅( $i + 2$j + 2 k$ ) = 15 and | r − ( $j + 2 k$ )| = 4.
radius of the sphere.
Sol. The equation of a line through the centre $j + 2k$ and normal
Sol. If point P with position vector r = x $i + y $j + z k$ is any to the given plane is
point on the sphere, then AP ⋅ BP = 0
r = ( $j + 2 k$ ) + λ( $i + 2 $j + 2 k$ ) ...(i)
( x − 2 ) ( x + 2 ) + (y − 6 ) (y − 4 ) + (z + 7 ) (z + 3 ) = 0
This meets the plane at a point for which we must have
⇒( x 2 − 4 ) + (y 2 − 10y + 24 ) + (z 2 + 10z + 21 ) = 0
[( $j + 2 k$ ) + λ( $i + 2 $j + 2 k$ )] ⋅( i$ + 2 $j + 2 k$ ) = 15
⇒ x 2 + y 2 + z 2 − 10y + 10z + 41 = 0 ⇒ 6 + 9 λ = 15
The centre of this sphere is (0, 5, − 5) and its radius is ⇒ λ =1
On putting λ = 1 in Eq. (i), we obtain the position vectors of the
5 2 + ( − 5 ) 2 − 41 = 9 = 3
centre as i$ + 3 $j + 4 k$ , Hence, the coordinates of the centre of
the circle are (1, 3, 4).
206 Textbook of Vector & 3D Geometry

Condition of Tengency of a Plane to a Sphere Sol. Let (α, β, γ) be any point on the locus. Then according to
the given condition, (α, β, γ ) is the centre of the sphere
A plane touches a given sphere, if the perpendicular through the origin. Therefore, its equation is given by
distance from the centre of the sphere to the planes is ( x − α)2 + (y − β ) 2 + (z − γ ) 2 = ( 0 − α ) 2 + ( 0 − β ) 2 + ( 0 − γ ) 2
equal to the radius of the sphere.
x 2 + y 2 + z 2 − 2 αx − 2βy − 2 γz = 0
Vector Form To obtain its point of intersection with the X -axis, we put
The plane r ⋅ n = d touches the sphere | r − a | = R, y = 0 and z = 0, so that
| a⋅ n − d | x 2 − 2αx = 0
if =R ⇒ x ( x − 2α ) = 0
|n|
⇒ x = 0 or x = 2α
Cartesian Form Thus, the plane meets X -axis at O( 0, 0, 0 ) and A(2α, 0, 0 ).
Similarly, it meets Y -axis at O ( 0, 0, 0 ) and B ( 0, 2β, 0), and
The plane lx + my + nz = p touches the sphere Z-axis at O( 0, 0, 0 ) and C( 0, 0, 2γ ).
x 2 + y 2 + z 2 + 2ux + 2vy + 2wz + d = 0, The equation of the plane through A, B and C is
if (ul + vm + wn + p ) 2 = (l 2 + m 2 + n 2 ) (u 2 + v 2 + w 2 − d ) x
+
y
+
z
=1 (intercept form)
2α 2β 2 γ
y Example 84. Show that the plane 2x − 2y + z + 12 = 0 Since, it passes through (a, b, c ), we get
touches the sphere x 2 + y 2 + z 2 − 2z − 4 y + 2z − 3 = 0. a b c
+ + =1
2α 2β 2 γ
Sol. The given plane will touch the given sphere if the
perpendicular distance from the centre of the sphere to the a b c
or + + =2
plane is equal to the radius of the sphere. The centre of the α β γ
given sphere x 2 + y 2 + z 2 − 2x − 4y + 2z − 3 = 0 is (1, 2, − 1) a b c
Hence, locus of (α, β, γ) is + + = 2.
and its radius is 12 + 22 + ( − 1)2 − ( − 3) = 3. x y z
Length of the perpendicular from (1, 2, − 1 ) to the plane
2 x − 2y + z + 12 = 0 is y Example 87. A sphere of constant radius k passes
through the origin and meets the axis at A, B and C.
2(1 ) − 2(2 ) + ( − 1 ) + 12 9
= =3 Prove that the centroid of triangle ABC lies on the
2 + (− 2) + 1
2 2 2 3
sphere 9( x 2 + y 2 + z 2 ) = 4k 2 .
Thus, the given plane touches the given sphere.
Sol. Let the equation of any sphere passing through the origin
y Example 85. Find the equation of the sphere and having radius k be
whose centre has the position vector 3$i + 6 $j − 4k$ x 2 + y 2 + z 2 + 2ux + 2vy + 2wz = 0

and which touches the plane r ⋅ (2$i − 2$j − k$ ) = 10. As the radius of the sphere is k, we get
u 2 + v 2 + w 2 =k 2
Sol. Let the radius of the required sphere be R. Then, its
equation is Note that (i) meets the X -axis at O( 0, 0, 0 ) and A( − 2u, 0, 0 );
Y -axis at O( 0, 0, 0 ) and B( 0, − 2v, 0 ), and Z-axis at O( 0, 0, 0 ) and
| r − (3 $i + 6 $j − 4 k$ )| = R ...(i) C ( 0, 0, − 2w ).
Since, the plane r ⋅(2 i$ − 2 $j − k$ ) = 10 touches the sphere (i), Let the centroid of the triangle ABC be (α, β, γ), Then
therefore length of perpendicular from the centre to the plane 2u 2v 2w
α= − ,β = − , γ = −
r ⋅(2 $i − 2 $j − k$ ) = 10 is equal to R. 3 3 3
| (3 $i − 6 $j − 4 k$ ) ⋅ (2 $i − 2 $j − k$ ) − 10 | 3α 3β 3v
i.e. =R ⇒ R=4 ⇒ u=− ,v = − ,w = −
| 2 i$ − 2 $j − k$ | 2 2 2
Putting this in (ii), we get
On putting R = 4 in Eq. (i), we obtain | r − (3 $i + 6 $j − 4 k$ ) | = 4
2 2 2
as the equation of the required sphere. −3  −3  −3 
 α +  β +  γ = k 2
 2   2   2 
y Example 86. A variable plane passes through a fixed 4 2
⇒ α 2 + β2 + γ 2 = k
point (a, b , c ) and cuts the coordinate axes at points 9
A, B and C. Show that the locus of the centre of the This shows that the centroid of triangle ABC lies on
a b c
sphere OABC is + + = 2. 4
x 2 + y 2 + z 2 = k 2.
x y z 9
Chap 03 Three Dimensional Coordinate System 207

Exercise for Session 4


1. Find the centre and radius of sphere 2( x − 5) ( x + 1) + 2( y + 5) ( y − 1) + 2(z − 2) (z + 2) = 7

2. Obtain the equation of the sphere with the points (1, − 1, 1) and (3, − 3, 3) as the extremities of a diametre and
find the coordinates of its centre.

3. Find the equation of sphere which passes through (1, 0, 0) and has its centre on the positive direction of Y-axis
and has radius 2.

4. Find the equation of sphere if it touches the plane r ⋅ (2$i − 2$j − k$ ) = 0 and the position vector of its centre is
3$i + 6$j − 4k$ .

5. Find the value of λ for which the plane x + y + z = 3λ touches the sphere x 2 + y 2 + z 2 − 2x − 2y − 2z = 6.

6. Find the equation of sphere concentric with sphere 2x 2 + 2y 2 + 2z 2 − 6x + 2y − 4z = 1 and double its radius.

7. A sphere has the equation | r − a |2 + | r − b |2 = 72, where a = $i + 3$j − 6k$ and b = 2$i + 4$j + 2k$
Find
(i) The centre of sphere
(ii) The radius of sphere
(iii) Perpendicular distance from the centre of the sphere to the plane r ⋅ (2$i + 2$j − k$ ) + 3 = 0.
JEE Type Solved Examples :
Single Option Correct Type Questions
l Ex. 1 If a line makes angle α, β and γ with the y1 = − m, y 2 = m, y 3 = m
coordinates axes, then and z1 = n, z 2 = − n, z 3 = n
(a) cos 2α + cos 2β + cos 2γ − 1 = 0 ∴Coordinates are A (l , − m, n ), B (l , m, − n ) and C ( − l , m, n )
(b) cos 2α + cos 2β + cos 2γ − 2 = 0 AB 2 + BC 2 + CA 2

(c) cos 2α + cos 2β + cos 2γ + 1 = 0 l 2 + m2 + n2
(d) cos 2α + cos 2β + cos 2γ + 2 = 0 ( 4m 2 + 4n 2 ) + ( 4l 2 + 4n 2 ) + ( 4l 2 + 4m 2 )
= =8
Sol. (c) If cos α, cos β and cos γ are the DC’s of a line, then l 2 + m2 + n2
2 cos2 α + 2 cos2 β + 2 cos2 γ = 2
⇒ 1 + cos 2α + 1 + cos 2β + 1 + cos 2 γ = 2
l Ex. 4 The angle between a line with direction ratios
⇒ cos 2α + cos 2β + cos 2 γ + 1 = 0 proportional to 2, 2, 1 and a line joining (3, 1, 4 ) to (7, 2, 12 ), is
 2  2
(a) cos −1   (b) cos −1  − 
l Ex. 2 The points (5, − 4, 2 ) ( 4, − 3, 1), (7, − 6, 4 ) 3  3
and (8, − 7, 5 ) are the vertices of  2
(c) tan−1   (d) None of these
(a) a rectangle (b) a square 3
(c) a parallelogram (d) None of these Sol. (a) A line with direction ratios proportional to 2, 2, 1 is parallel
Sol. (c) Let A(5, − 4, 2 ), B( 4, − 3, 1 ), C(7, − 6, 4 ) and D(8, − 7, 5 ) to the vector a = 2 $i + 2 $j + k$ .
AB = − $i + $j − k$ Line joining P(3, 1, 4 ) to Q (7,2, 12 ) is parallel to the vector
PQ = 4 $i + $j + 8 k$ .
BC = 3 $i − 3 $j + 3 k$
Let θ be the required angle. Then,
CD = $i − $j + k$
a ⋅ PQ 8+2+8
and DA = −3 $i + 3 $j − 3 k$ cos θ = =
| a | | PQ | 4 + 4 + 1 16 + 1 + 64
Clearly AB || CD and BC || DA 18 2  2
AB ⋅ BC = − 9 ≠ 0 ⇒ cos θ = = ⇒ θ = cos−1  
Also, 3 ×9 3  3
∴ ABCD is a parallelogram.
l Ex. 5 The angle between the lines 2 x = 3y = − z and
l Ex. 3 In ∆ABC the mid-point of the sides AB, BC and CA
are respectively (l , 0, 0 ), (0, m, 0 ) and (0, 0, n ). Then, 6x = − y = − 4z is
(a) 30° (b) 45°
AB 2 + BC 2 + CA 2
is equal to (c) 60° (d) 90°
l 2 +m2 + n 2
Sol. (d) Given, equation of lines can be rewritten as
(a) 2 (b) 4 x y z
= =
(c) 8 (d) 16 1/2 1/3 −1
Sol. (c) From the figure, x y z
C (x3, y3, z3) and = =
1/6 −1 −1/ 4
a1a 2 + b1b2 + c1c 2
∴ cos θ =
(0, m, 0) (0, 0, n) a1 + b12 + c12 a 22 + b22 + c 22
2

1 1 1  1
× + × (− 1) − 1 ×  − 
2 6 3  4
=
B A 1 1 1 1
(x2, y2, z2) (l, 0, 0) (x1, y1, z1) + +1 +1+
4 9 36 16
x1+ x 2 = 2l , y1 + y 2 = 0, z1 + z 2 = 0 1 1 1
− +
x 2 + x 3 = 0, y 2 + y 3 = 2m, z 2 + z 3 = 0 = 12 3 4
1 1 1 1
and x1 + x 3 = 0, y1 + y 3 = 0, z1 + z 3 = 2n + +1 +1 +
4 9 36 16
On solving, we get
x1 = l , x 2 = l , x 3 = − l ⇒ cos θ = 0 ⇒ θ = 90 °
Chap 03 Three Dimensional Coordinate System 209

l Ex. 6 A line makes the same angle θ with X-axis and l Ex. 9 The coordinates of the foot of the perpendicular
Z-axis. If the angle β, which it makes with Y-axis, is such drawn from the point A(1, 0, 3 ) to the join of the points
that sin 2 β = 3 sin 2 θ, then the value of cos 2 θ is B( 4, 7, 1) and C(3, 5, 3 ) are
1 2  5 7 17 
(a) (b) (a)  , ,  (b) (5, 7, 17 )
5 5 3 3 3 
3 2  5 7 17   5 7 17 
(c) (d) (c)  , − ,  (d)  − , , − 
5 3 7 3 3  3 3 3
Sol. (c) Since, cos2 θ + cos2 β + cos2 θ = 1 [Ql 2 + m 2 + n 2 = 1] Sol. (a) Let D be the foot of the perpendicular and let it divide BC in
⇒ 2 cos θ + 1 − 3 sin θ = 1
2 2
[Qsin β = 3 sin θ]
2 2 the ratio λ : 1. Then, the coordinates of D are
 3λ + 4 5λ + 7 3λ + 1
⇒ 2 cos θ − 3(1 − cos θ) = 0
2 2
 , , 
 λ +1 λ +1 λ +1
3
⇒ 5 cos2 θ = 3 ⇒ cos2 θ = Now, AD ⊥ BC ⇒ AD ⋅ BC = 0
5
7
⇒ − (2 λ + 3 ) − 2(5 λ + 7 ) − 4 = 0 ⇒ λ = −
l Ex. 7 The projection of a line segment on the coordinate 4
axes are 2, 3, 6. Then, the length of the line segment is  5 7 17
So, the coordinates of D are  , , 
3 3 3 
(a) 7 (b) 5
(c) 1 (d) 11
Sol. (a) Let the length of the line segment be r and its direction
l Ex. 10 A mirror and a source of light are situated at the
cosines be l, m, n. Then, its projections on the coordinate axes origin O and at a point on OX, respectively. A ray of light
are lr, mr, nr. from the source strikes the mirror and is reflected. If the
∴ lr = 2, mr = 3 and nr = 6 direction ratios of the normal to the plane are proportional
⇒ l 2r 2 + m 2r 2 + n 2r 2 = 4 + 9 + 36 to 1,−1 , 1, then direction cosines of the reflected ray are
⇒ r 2 (l 2 + m 2 + n 2 ) = 49 1 2 2 1 2 2
(a) , , (b) − , ,
3 3 3 3 3 3
⇒ r 2 = 49 ⇒ r = 7 [Ql 2 + m 2 + n 2 = 1]
1 2 2 1 2 2
(c) − , − , − (d) − , − ,
3 3 3 3 3 3
l Ex. 8 The equation of the straight line through the origin
Sol. (d) Let the source of light be situated at A(a, 0, 0 ), where, a ≠ 0.
and parallel to the line (b + c ) x + (c + a )y
+ (a + b ) z = k = (b − c ) x + (c − a )y + (a − b ) z are Let OA be the incident ray, OB be the reflected ray and ON be
the normal to the mirror at O.
x y z
(a) = = θ
b2 −c2 c 2 − a2 a2 − b 2 ∴ ∠AON = ∠NOB = (say)
2
x y z Direction ratios of OA are proportional to a, 0, 0 and so its
(b) = =
b b a direction cosines are 1, 0, 0.
x y z 1 −1 1
(c) 2 = 2 = 2 Direction cosines of ON are , ,
a − bc b − ca c − ab 3 3 3
θ 1
(d) None of the above ∴ cos =
2 3
Sol. (c) Equations of straight line through the origin are
x−0 y −0 z −0 A(a, 0, 0) N
= = 1, –1,1 B
l m n
where, l (b + c ) + m (c + a ) + n (a + b ) = 0 q q
2
and l (b − c ) + m (c − a ) + n (a − b ) = 0 2
1 m
On solving, =
2(a − bc ) 2(b − ca )
2 2 O (0, 0, 0)

n Let l, m, n be the direction cosines of the reflected ray OB.


= 2
2(c − ab ) l +1 1 m+0 1
Then, = , =−
θ 3 θ 3
Equations of the straight line are 2 cos 2 cos
x y z 2 2
= = . n+0 1
a 2 − bc b 2 − ca c 2 − ab and =
θ 3
2 cos
2
210 Textbook of Vector & 3D Geometry

⇒ l =
2 2
− 1, m = − , n =
2 Sol. (a) Let AB = a, AD = b, then AC = a + b
3 3 3 Given, AB1 = λ 1a, AD1 = λ 2b, AC1 = λ 3(a + b )
1 2 2 B1D1 = AD1 − AB1 = λ 2b − λ 1a
⇒ l = − ,m = − ,n =
3 3 3
1 2 2 D C
Hence, direction cosines of the reflected ray are − , − , .
3 3 3

D1
l Ex. 11 Equation of plane passing through the points
C1
( 2, 2, 1), ( 9, 3, 6 ) and perpendicular to the plane A B1 B
2 x + 6y + 6 z −1 = 0, is
(a) 3 x + 4y + 5z = 9
Since, vectors D1C1 and B1D1 are collinear, we have
(b) 3 x + 4y − 5z + 9 = 0
D1C1 = k B1D1 for some k ∈R.
(c) 3 x + 4y − 5z − 9 = 0
⇒ AC1 − AD1 = k ⋅ B1D1
(d) None of the above
⇒ λ 3(a + b ) − λ 2b = k ⋅ ( λ 2b − λ 1a )
Sol. (c) Equation of a plane passing through (2, 2, 1 ) is ⇒ λ 3a + ( λ 3 − λ 2 ) b = k ⋅ λ 2b − k ⋅ λ 1a
a( x − 2 ) + b(y − 2 ) + c(z − 1 ) = 0 ...(i) Thus, λ 3 = − kλ 1 and λ 3 − λ 2 = kλ 2
This passes through (9, 3, 6 ) and is perpendicular to −λ 3 λ 3 − λ 2
⇒ k= = ⇒ λ 1λ 2 = λ 1λ 3 + λ 2λ 3
2 x + 6y + 6z − 1 = 0 λ1 λ2
∴ 7a + b + 5c = 0 and 2a + 6b + 6c = 0 1 1 1
⇒ = +
Solving these two by cross-multiplication, we get λ 3 λ1 λ 2
a b c
= =
− 24 − 32 40 l Ex. 14 If the direction cosines of two lines are such that

a
=
b
=
c l + m + n =0, l 2 + m 2 − n 2 = 0, then the angle between them is
−3 − 4 5
π
Substituting the values of a, b, c in Eq. (i), we get (a) π (b)
3
3 x + 4y − 5z − 9 = 0 as the required plane. π π
(c) (d)
4 6
l Ex. 12 If the position vectors of the points A and B are
Sol. (b) If l, m, n are direction cosines of two lines are such that
3 i + $j + 2k$ and $i − 2 $j − 4k$ respectively, then the equation of
$
l +m + n = 0 ...(i)
the plane through B and perpendicular to AB is and l 2 + m2 − n2 = 0 ...(ii)
(a) 2x + 3y + 6 z + 28 = 0
⇒ l 2 + m 2 − (− l −m )2 = 0
(b) 3 x + 2y + 6 z = 28
⇒ 2lm = 0 ⇒ l = 0 or m = 0
(c) 2x − 3y + 6 z + 28 = 0
If l = 0, then n = − m
(d) 3 x − 2y + 6 z = 28
⇒ l :m:n = 0:1: −1
Sol. (a) We have, AB = − 2 $i − 3 $j − 6 k$
and if m = 0, then n = − l
So, vector equation of the plane is ⇒ l :m:n =1: 0: −1
| r − ( $i − 2 $j − 4 k$ ) | ⋅ AB = 0 0+0+1 1
∴ cos θ = =
⇒ r ⋅( − 2 $i − 3 $j − 6 k$ ) = ( $i − 2 $j − 4 k$ ) ⋅ ( − 2 $i − 3 $j − 6 k$ ) 0+1+1 0+1+1 2
⇒ − 2 x − 3y − 6z = − 2 + 6 + 24 π
⇒ θ=
⇒ 2 x + 3y + 6z + 28 = 0 3

l Ex. 13 A straight line ‘L’ cuts the lines AB, AC and AD of


l Ex. 15 The equation of the plane passing through the
a parallelogram ABCD at points B1 , C 1 and D 1 , respectively. mid-point of the line points (1, 2, 3 ) and (3, 4, 5 ) and perpen-
1 dicular to it is
If AB 1 , = λ 1 AB, AD1 = λ 2 AD and AC1 = λ 3 AC, then is (a) x + y + z = 9 (b) x + y + z = − 9
λ3
(c) 2x + 3y + 4 z = 9 (d) 2x + 3y + 4 z = − 9
equal to
Sol. (a) The DR’s of the joining of the points (1,2, 3 ) and (3, 4, 5 ) and
1 1 1 1
(a) + (b) − (3 − 1, 4 − 2, 5 − 3 ), ie. (2, 2, 2 )
λ1 λ 2 λ1 λ 2
Also, the mid-point of the join of the points (1, 2, 3) and (3, 4, 5 )
(c) − λ 1 + λ 2 (d) λ 1 + λ 2 is (2, 3, 4).
Chap 03 Three Dimensional Coordinate System 211

∴ Equation of plane which passes through (2, 3, 4 ) and the Sol. (b) Clearly, the given line passes through the point
DR’s of its normal are (2, 2, 2) is a = 2 $i − 2 $j + 3 k$ and is parallel to the vector b = $i − $j + 4k$ .
2( x − 2 ) + 2(y − 3 ) + 2(z − 4 ) = 0 The plane is normal to the vector n = i$ + 5 $j + k$ .
⇒ x + y + z −9 = 0
We have, b⋅ n = 1 − 5 + 4 = 0
⇒ x + y + z =9
So, the line is parallel to the plane.
∴Required distance
l Ex. 16 Equation of the plane that contains the lines = Length of the perpendiculars from a point on
r = ( $i + $j) + λ( $i + 2 $j − k
$ ) and, r = ( $i + $j) + µ( − $i + $j − 2 k
$ ) is the line to the given plane.
(a) r ⋅( 2i$ + j$ − 3k$ ) = − 4 = Length of the perpendicular from (2 i$ − 2 $j + 3 k$ )
to the given plane.
(b) r × ( − $i + $j + k$ ) = 0
(2 $i − 2 $j + 3 k$ ) ⋅( $i + 5 $j + k$ ) − 5
=
(c) r ⋅ ( − i$ + j$ + k$ ) = 0 1 + 25 + 1
(d) None of the above 2 − 10 + 3 − 5 10
= =
Sol. (c) The lines are parallel to the vectors b1 = $i + 2 $j − k$ and 3 3 3 3
b 2 = − $i + $j − 2 k$ . Therefore, the plane is normal to the vector
x y z
$i $j k$ l Ex. 19 If the plane + + = 1, cuts the coordinate
n = b1 × b 2 = 1 2 − 1 = − 3 $i + 3 $j + 3 k$
2 3 4
axes in A, B, C, then the area of ∆ABC is
−1 1 −2
(a) 29 sq. units (b) 41 sq. units
The required plane passes through ( $i + $j) and is normal to the
(c) 61 sq. units (d) None of these
vector n. Therefore, its equation is
r⋅ n = a⋅ n Sol. (c) The given plane cuts the coordinate axes in A(2, 0, 0 ),
⇒ r ⋅ ( − 3 i$ + 3 $j + 3 k$ ) = ( $i + $j) ⋅( − 3 $i + 3 $j + 3 k$ ) B( 0, 3, 0 ) and C( 0, 0, 4 ).
1
⇒ r ⋅ ( − 3 i$ + 3 $j + 3 k$ ) = −3 + 3 ∴ Area of ∆ABC = AB × AC × sin ∠BAC
2
⇒ r ⋅ ( − $i + $j + k$ ) = 0
Now, AB = 4 + 9 + 0 = 13, AC = 4 + 0 + 16 = 20.

x − 2 y +1 z −1 AB ⋅ AC ( − 2 $i + 3 $j) ⋅ ( − 2 $i + 4 k$ )
l Ex. 17 The line = = intersects the curve cos ∠BAC = =
| AB | | AC | 4 + 9 4 + 16
3 2 −1
4+0+0 4 2
xy = c 2 , z = 0, if c is equal to ⇒ cos ∠BAC = = =
13 20 13 20 65
1
(a) ± 1 (b) ±
3 4 61
⇒ sin ∠BAC = 1 − =
(c) ± 5 (d) None of these 65 65
1 61
Sol. (c) At the point on the line where it intersects the given curve, Hence, Area of ∆ABC = × 13 × 20 × = 61 sq. units.
we have z = 0, so that 2 65
x −2 y + 1 0 −1
= =
3 2 −1
l Ex. 20 The distance of the point (1, − 2, 3 ) from the plane
x −2 y +1 x y z −1
⇒ = 1 and =1 x − y + z = 5 measured parallel to the line = = is
3 2 2 3 −6
⇒ x = 5 and y = 1. (a) 1 (b) 2
Putting these values of x and y in xy = c 2, we get (c) 4 (d) None of these
c 2 = 5 ⇒c = ± 5. Sol. (a) The equation of the line passing through P(1, − 2, 3 ) and
parallel to the given line is
l Ex. 18 The distance between the line r = 2 $i − 2 $j + 3k$ x −1 y + 2 z − 3
= =
+ λ( $i − $j + 4k$ ) and the plane r ⋅ ( $i + 5 $j +k$ ) = 5, is 2 3 −6
10 10 Suppose it meets the plane x − y + z = 5 at the point Q given
(a) (b) by
9 3 3
x − 1 y +2 z − 3
10 = =
(c) (d) None of these 2 3 −6
3
= λ i.e. (2 λ + 1, 3 λ − 2, − 6 λ + 3 )
212 Textbook of Vector & 3D Geometry

This lies on x − y + z = 5. Therefore, l Ex. 23 The equation of the plane through the intersection
2λ + 1 − 3λ + 2 − 6λ + 3 = 5 of the planes x + y + z =1 and 2 x + 3y − z + 4 = 0 and
1
⇒ − 7λ = − 1 ⇒ λ = parallel to X-axis, is
7
(a) y − 3 z + 6 = 0 (b) 3y − z + 6 = 0
 9 11 15
So, the coordinates of Q are  , − ,  . (c) y + 3 z + 6 = 0 (d) 3y − 2z + 6 = 0
7 7 7
Sol. (a) The equation of the plane through the intersection of the
4 9 36 planes x + y + z = 1 and 2 x + 3y − z + 4 = 0 is
Hence, required distance = PQ = + + = 1.
49 49 49 ( x + y + z − 1 ) + λ(2 x + 3y − z + 4 ) = 0
or, (2 λ + 1 ) x + (3 λ + 1 )y + (1 − λ)z + 4 λ − 1 = 0 ...(i)
l Ex. 21 The length of the perpendicular from the origin to x y z
the plane passing through the point a and containing the line It is parallel to X -axis, i.e. = =
1 0 0
r = b + λc is ∴ 1(2 λ + 1 ) + 0 × (3 λ + 1 ) + 0 (1 − λ) = 0
[a b c ] 1
(a) ⇒ λ=−
| a× b+ b× c+ c× a | 2
[a b c ] 1
(b) Substituting λ = − in Eq. (i), we get
| a× b+ b× c | 2
[a b c ] y − 3z + 6 = 0 as the equation of the required plane.
(c)
|b× c+ c× a |
l Ex. 24 A plane passes through the point (1, 1, 1). If b, c, a
[a b c ]
(d) are the direction ratios of a normal to the plane where a, b,
| a× b+ c × a |
c (a < b < c ) are the prime factors of 2001, then the equation
Sol. (c) The plane passing through a and containing the line of the plane II is
r = b + λc also passes through the point b and is parallel to the
vector c. So, it is normal to the vector (a − b ) × c. (a) 29x + 31y + 3 z = 63
Thus, the equation of the plane is (b) 23 x + 29y − 29z = 23
( r − a ) ⋅| (a − b ) × c | = 0 (c) 23 x + 29y + 3 z = 55
⇒ ( r − a ) ⋅(a × c − b × c ) = 0 (d) 31x + 37y + 3 z = 71
⇒ r ⋅(a × c − b × c ) = a ⋅ (a × c − b × c ) Sol. (c) The equation of the plane is
⇒ r ⋅(a × c − b × c ) = − a ⋅( b × c ) b ( x − 1 ) + c (y − 1 ) + a (z − 1 ) = 0 ...(i)
⇒ r ⋅( b × c + c × a ) − [a b c ] = 0 Now, 2001 = 3 × 23 × 29
∴Length of the perpendicular from the origin to this plane ∴ a < b < c ⇒ a = 3, b =23 and c = 29.
0 ⋅ ( b × c + c × a ) − [a b c ] Substituting the values of a, b, c in Eq. (i), we obtain
= 23 x + 29y + 3z = 55 as the equation of the required plane.
| b× c+ c× a |
[a b c ]
= l Ex. 25 If the direction ratios of two lines are given by
| b× c+ c× a |
a + b + c = 0 and 2ab + 2ac − bc = 0, then the angle between
the lines is
l Ex. 22 If P(0, 1, 0 ) and Q(0, 0, 1) are two points, then the

projection of PQ on the plane x + y + z = 3 is (a) π (b)
3
(a) 2 (b) 3 π π
(c) 2 (d) 3 (c) (d)
2 3
Sol. (c) The projection of PQ on the given plane is PQ cos θ, where θ Sol. (b) We have,
is the angle between PQ and the plane. a + b + c = 0 and 2ab + 2ac − bc = 0
Let n be a vector normal to the plane. ⇒ a = − (b + c ) and 2a(b + c ) − bc = 0
We have, PQ = − $j + k$ and n = $i + $j + k$ ⇒ − 2(b + c ) 2 − bc = 0
PQ ⋅ na ⇒ 2b 2 + 5bc + 2c 2 = 0
∴ sin θ = =0
| PQ | | n | ⇒ (2b + c ) (b + 2c ) = 0
⇒ PQ is parallel to the plane. ⇒ 2b + c = 0 or, b + 2c = 0
Hence, projection of PQ on the given plane If 2b + c = 0, then a = − (b + c ) ⇒ a =b
= | PQ| cos θ a b c
∴ a = b and c = − 2b ⇒ = =
= | PQ | = 1 + 1 = 2 1 1 −2
Chap 03 Three Dimensional Coordinate System 213

If b + 2c = 0, then a = − (b + c ) ⇒ a = c l Ex. 28 The vector equation of the plane through the point
∴ a = c and b = − 2c ⇒
a
=
b
=
c $i + 2 $j − k$ and perpendicular to the line of intersection of the
1 −2 1
plane r ⋅ (3 $i − $j + k$ ) = 1 and r ⋅ ( $i + 4 $j − 2k$ ) = 2, is
Thus, the direction ratios of two lines are proportional to 1, 1,
− 2 and 1, − 2, 1, respectively. So, the angle θ between them is (a) r ⋅ ( 2$i + 7 $j − 13k$ ) = 1 (b) r ⋅( 2$i − 7 $j − 13k$ ) = 1
given by
1 −2 −2 −1 2π
(c) r ⋅ ( 2i$ + 7 j$ + 13k$ ) = 0 (d) None of these
cos θ = = ⇒ θ=
1+1+4 1+4+1 2 3 Sol. (b) The line of intersection of the planes
r ⋅ (3 $i − $j + k$ ) = 1
l Ex. 26 A tetrahedron has vertices at O (0, 0, 0 ), A(1, 2, 1), and r ⋅ ( i$ + 4 $j − 2 k$ ) = 2
B( 2, 1, 3 ) and C( −1, 1, 2 ). Then, the angle between the faces is common to both the planes. Therefore, it is perpendicular to
normals to the two planes, i.e.
OAB and ABC will be
n1 = 3 $i − $j + k$
 19
(a) 90° (b) cos − 1   and n 2 = $i + 4 $j − 2 k$
 35
Hence, it is parallel to the vector n1 × n 2 = − 2 $i + 7 $j + 13 k$ .
 17 
(c) cos − 1   (d) 30°
 31 Thus, we have to find the equation of the plane passing
through a = i$ + 2 $j − k$ and normal to the vector n = n1 × n 2.
Sol. (b) Let n1 and n 2 be the vectors normal to the faces OAB and
The equation of the required plane is
ABC. Then,
(r − a ) ⋅ n = 0
$i $j k$
⇒ r⋅ n = a⋅ n
n1 = OA × OB = 1 2 1 = 5 $i − $j − 3 k$
⇒ r ⋅ ( − 2 $i + 7 $j + 13 k$ ) = ( $i + 2 $j − k$ ) ⋅ ( − 2i$ + 7 $j + 13 k$ )
2 1 3
⇒ r ⋅ (2 $i − 7 $j − 13 k$ ) = 1
$i $j k$
and n 2 = AB × AC = 1 − 1 2 = $i − 5 $j − 3 k$ l Ex. 29 The cartesian equation of the plane
−2 −1 1 r = (1 + λ − µ)$i + ( 2 − λ)$j + (3 − 2 λ + 2µ )k$ , is
If θ is the angle between the faces OAB and ABC, then (a) 2x + y = 5 (b) 2x − y = 5
n1 ⋅ n 2 (c) 2x + z = 5 (d) 2x − z = 5
cos θ =
| n1 | | n 2 | Sol. (c) We have,
5+5+9 19 r = (1 + λ − µ)$i + (2 − λ)$j + (3 − 2 λ + 2µ ) k$
⇒ cos θ = =
25 + 1 + 9 1 + 25 + 9 35 ⇒ r = ( i$ + 2 $j + 3 k$ ) + λ( $i − $j − 2 k$ ) + µ( − i$ + 2 k$ )
 19 which is a plane passing through a = i$ + 2 $j + 3 k$ and parallel to
⇒ θ = cos− 1  
 35 the vectors b = $i − $j − 2 k$ and c = − i$ + 2 k$ .
Therefore, it is normal to the vector
l Ex. 27 The vector equation of the plane through the point n = b × c = − 2 $i − k$
( 2, 1, − 1) and passing through the line of intersection of the
Hence, its vector equation is
plane r ⋅ ( $i + 3 $j − k$ ) = 0 and r ⋅ ( $j + 2k$ ) = 0, is (r − a ) ⋅ n = 0
(a) r ⋅( i$ + 9j$ + 11k$ ) = 0 (b) r$ ⋅ ( i$ + 9j$ + 11k$ ) = 6 ⇒ r⋅ n = a⋅ n
⇒ r ⋅( − 2 $i − k$ ) = − 2 − 3
(c) r$ ⋅( $i − 3 $j − 13k$ ) = 0 (d) None of these
⇒ r ⋅(2 i$ + k$ ) = 5
Sol. (a) The vector equation of a plane through the line of
intersection of the plane r ⋅ ( i$ + 3 $j − k$ ) = 0 and r ⋅( $j + 2 k$ ) = 0 So, the cartesian equation of the plane is
can be written as ( x $i + y $j + z k$ ) ⋅ (2 $i + k$ ) = 5 ⇒2 x + z = 5
r ⋅ ( $i + 3 $j + − k$ ) + λ { r ⋅( $j + 2 k$ )} = 0 ...(i)
l Ex. 30 A variable plane is at a distance, k from the origin
This passes through 2 i$ + $j − k$ .
and meets the coordinates axis in A, B, C. Then, the locus of
∴ (2 i$ + $j − k$ ) ⋅( $i + 3 $j − k$ ) + λ(2 i$ + $j − k$ ) ⋅( $j + 2 k$ ) = 0 the centroid of ∆ABC is
⇒ (2 + 3 + 1 ) + λ( 0 + 1 − 2 ) = 0 ⇒ λ = 6. (a) x −2 + y −2 + z −2 = k −2
Putting the value of λ in Eq. (i), we get (b) x −2 + y −2 + z −2 = 4k −2
the equation of the required plane as
(c) x −2 + y −2 + z −2 = 16k −2
r ⋅( i$ + 9 $j + 11 k$ ) = 0
(d) x −2 + y −2 + z −2 = 9k −2
214 Textbook of Vector & 3D Geometry

x y z
Sol. (d) Let the equation of the variable plane be + + = 1. 0 0 0
+ + −1
a b c a b c
∴ =k
This meets the coordinates axes at A(a, 0, 0 ), B( 0, b, 0 ) and 1 1 1
C ( 0, 0, c ). + +
a2 b2 c2
Let (α, β, γ) be the coordinates of the centroid of ∆ABC. Then, 1 1 1 1
a b c ⇒ + 2 + 2 = 2
α = ,β = , γ = a 2
b c k
3 3 3
⇒ α − 2 + β − 2 + γ − 2 = 9k − 2
⇒ a = 3α, b = 3β, c = 3γ ...(i)
The plane is at a distance, k from the origin. Hence, the locus of (α, β, γ) is x − 2 + y − 2 + z − 2 = 9k − 2

JEE Type Solved Examples :


More than One Correct Option Type Questions
l Ex. 31 The direction ratios of the line x − y + z − 5 = 0 l Ex. 33 The direction cosines of the lines bisecting the
= x − 3y − 6 are angle between the line whose direction cosines are l1 , m1 , n1
(a) 3, 1, − 2 (b) 2, − 4, 1 and l 2 , m 2 , n 2 and the angle between these lines is θ, are
3 1 −2 2 −4 1 l1 + l 2 m1 + m 2 n 1 + n 2
(c) , , (d) , , (a) , ,
14 14 14 21 21 21 θ θ θ
cos cos cos
Sol. (a, c) Let the DR’s of line are a, b and c. 2 2 2
As the line is perpendicular to both the planes
l1 + l 2 m1 + m 2 n 1 + n 2
(b) , ,
θ θ θ
⇒ a −b + c = 0 2 cos 2 cos 2 cos
a − 3b + 0 ⋅ c = 0 2 2 2
a b c l1 + l 2 m1 + m 2 n 1 + n 2
= = (c) , ,
3 1 −2 θ θ θ
sin sin sin
Hence, (a) and (c) are correct answers.
2 2 2
l1 − l 2 m1 − m 2 n 1 − n 2
(d) , ,
θ θ θ
l Ex. 32 The equation of the line x + y + z − 1 = 0 , 2 sin 2 sin 2 sin
2 2 2
4 x + y − 2 z + 2 = 0 written in the symmetrical form is
Sol. (b, d) Distance ratio of the bisector are
x +1 y − 2 z − 0
(a) = = < l1 + l 2, m1 + m2, n1 + n2 >
1 −2 1
= (l1 + l 2 ) 2 + (m1 + m2 ) 2 + (n1 + n2 ) 2
x y z −1
(b) = = = 2 + 2(l1l 2 + m1m2 + n1n2 )
1 −2 1
x +1 z −1 θ
= 2 + 2 cos θ = 2 cos
2 y −1 2
(c) = = 2
1 −2 1
l1 + l 2 m1 + m2 n1 + n2
x −1 y + 2 z − 2 ∴ Direction cosines are , ,
(d) = + θ θ θ
2 −1 2 2 cos 2 cos 2 cos
2 2 2
Sol. (a, b, c, d) x + y + z − 1 = 0 Distance ratio of the other bisector are
4 x + y − 2z + 2 = 0 < l1 − l 2, m1 − m2, n1 − n2 > (l1 − l 2 ) 2 + (m1 − m2 ) 2 + (n1 − n2 ) 2
∴Direction ratios of the line are ( − 3, 6, − 3 ). θ
= 2 sin
i.e. < 1, − 2, 1 > 2
Let z = k, then x = k − 1, y = 2 − 2k ∴ Direction cosines of the bisector are
i.e. (k − 1, 2 − 2k, k ) is any point on the line.
 1 1 l1 − l 2 m1 − m2 n1 − n2
∴ ( − 1, 2, 0 ), ( 0, 0, 1 ),  − , 1,  and (1, − 2, 2 ) are points on the θ
,
θ
,
θ
 2 2 2 sin 2 sin 2 sin
line. 2 2 2
Hence, (a), (b), (c) and (d) are the correct answers. Hence, (b) and (d) are correct answers.
Chap 03 Three Dimensional Coordinate System 215

l Ex. 34 Consider the planes 3 x − 6y + 2 z + 5 = 0 and  9 


∴ Coordinates of Q are 3, − , 9 .
 2 
4 x −12y + 3 z = 3. The planes 67 x +162y + 47 z + 44 = 0
x −1 y −2 z −5
bisects the angle between the planes which Equation of PN is = = .
3 − 176 − 89
(a) contains origin (b) is acute
(c) is obtuse (d) None of these
l Ex. 36 The equation of a plane is 2 x − y − 3 z = 5 and
Sol. (a,b) 3 x − 6y + 2z + 5 = 0 ...(i)
A(1,1, 1), B( 2, 1, − 3 ), C(1, − 2, − 2 ) and D( − 3, 1, 2 ) are four
− 4 x + 12y − 3z + 3 = 0 ...(ii)
3 x − 6y + 2z + 5 − 4 x + 12y − 3z + 3 points. Which of the following line segment are intersected
= by the plane?
9 + 36 + 4 16 + 144 + 9
(a) AD (b) AB
Bisects the angle between the planes that contains the origin.
(c) AC (d) BC
13(3 x − 6y + 2z + 5 ) = 7( − 4 x + 12y − 3z + 3 )
Sol. (b, c) For A(1, 1, 1 ), 2 x − y − 3z − 5 = 2 − 1 − 3 − 5 < 0
39 x − 78y + 26z + 65 = − 28 x + 84y − 21z + 21
67 x − 162y + 47z + 44 = 0 ...(iii) For B (2, 1, −3), 2 x − y − 3z − 5 = 0 − 1 + 9 − 5 > 0
Let θ be the angle between Eqs. (i) and (iii), then find cos θ and For C (1, − 2, − 2 ), 2 x − y − 3z − 5 = 2 + 2 + 6 − 5 > 0. A, D
then we obtain | tan θ | < 1. For D ( −3, 1, 2 ), 2 x − y − 3z − 5 = −6 − 1 − 6 − 5 = −18 < 0
Hence, (a) and (b) are the correct answer. are on one side of the plane and B, C are on the other side, the
line segments AB , AC, BD, CD intercept the plane.
x y z
l Ex. 35 Consider the equation of line AB is = = .
2 −3 6 l Ex. 37 The coordinates of a point on the line
x −1 y +1
Through a point P(1, 2, 5 ) line PN is drawn perpendicular to = = z at a distance 4 14 from the point (1, − 1, 0)
AB and line PQ is drawn parallel to the plane 2 −3
3 x + 4y + 5 z = 0 to meet AB is Q. Then, are
 52 78 156  (a) ( 9, − 13, 4 )
(a) coordinate of N are  , − , 
 49 49 49  (b) (8 14 + 1, − 12 14 − 1, 4 14 )
 9  (c) ( − 7, 11, − 4 )
(b) the coordinates of Q are 3, − , 9
 2  (d) ( − 8 14 + 1, 12 14 − 1, − 4 14 )
x −1 y − 2 z −5 Sol. (a, c) The coordinates of any point on the given line are
(c) the equation of PN is = =
3 − 176 − 89 (2r + 1, − 3r − 1, r )
The distance of this point from the point (1, − 1, 0 ) is given to
 156 52 78 
(d) coordinates of N are  , −  be 4 14.
 49 49 49
⇒ (2r ) 2 + ( − 3r ) 2 + (r ) 2 = ( 4 14 ) 2
x y z
Sol. (a,b,c) Equation of line AB is = = ⇒ 14r 2 = 16 × 14
2 −3 6
⇒ r =±4
Its DR’s are < 2, − 3, 6 >
So, the coordinate of the required point are
Let the coordinates be < 2r, − 3r, 6r >
(9, − 13, 4 ) or ( − 7, 11, − 4 ).
DR’s of PN are < 2r − 1, − 3r − 2, 6r − 5 >
It is perpendicular to AB
l Ex. 38 The line whose vector equation are
∴ 2(2r − 1 ) − 3 ( − 3r − 2 ) + 6(6r − 5 ) = 0
4r − 2 + 9r + 6 + 36r − 30 = 0 r = 2 $i − 3 $j + 7k$ + λ( 2 $i + p $j + 5k$ )
49r = 26 i.e. r =
26 and r = $i + 2 $j + 3k$ + µ(3 $i −p$j + pk$ )
49
are perpendicular for all values of λ and µ if p equals to
 52 78 156  (a) − 1
(a)∴ Coordinates of N are  , − ,  (b) 2
 49 49 49 
(c) 5 (d) 6
(b) Let the coordinates of Q be ( 2r , − 3r , 6r ), then DR’s of Sol. (a, d) The given lines are perpendicular for all values of λ and
PQ are < 2r − 1 , − 3r − 2 , 6r − 5 >. Since, PQ is parallel to µ if the vectors.
the plane. 2 $i + p$j + 5 k$ and 3 $i − p$j + pk$ are perpendicular
∴ 3(2r − 1 ) + 4( − 3r − 2 ) + 5(6r − 5 ) = 0
⇒ 2 × 3 + p (− p ) + 5p = 0
6r − 3 − 12r − 8 + 30r − 25 = 0
3 ⇒ p 2 − 5p − 6 = 0
24r = 36, r =
2 ⇒ p = − 1 or 6
216 Textbook of Vector & 3D Geometry

l Ex. 39 Equation of a plane passing through the lines l Ex. 40 The plane passing through the point ( − 2, − 2, 2 )
2 x − y + z − 3 = 0, 3 x + y + z − 5 = 0 and which is at a and containing the line joining the points (1, 1, 1) and
1 (1, −1, 2 ) makes intercepts of lengths a, b, c respectively on
distance of from the point ( 2, 1, − 1) is
6 the axes of x, y and z respectively, then
(a) 2x − y + z − 3 = 0 (a) a = 3b (b) b = 2c
(b) 3 x + y + z − 5 = 0 (c) a + b + c = 12 (d) a + 2b + 2c = 0
(c) 62x + 29y + 19z − 105 = 0 Sol. (a,b,c) Equation of any plane passing through ( − 2, − 2, 2 ) is
(d) x + 2y − 2 = 0 A( x + 2 ) + B(y + 2 ) + C (z − 2 ) = 0
Sol. (a,c) Equation of a plane through the given line is Since it contains the line joining (1, 1, 1 ) and (1, − 1, 2 ) these
2 x − y + z − 3 + λ(3 x + y + z − 5 ) = 0 points also lie on this plane.
⇒ (2 + 3 λ)x + ( λ − 1 )y + ( λ + 1 )z − (3 + 5 λ) = 0 ⇒ 3 A + 3 B − C = 0 and 3 A + B + 0 = 0
1 2(2 + 3 λ) + ( λ − 1 ) − ( λ + 1 ) − 3 − 5 λ A B C
So, = ⇒ = = .
6 (2 + 3 λ)2 + ( λ − 1 ) 2 + ( λ + 1 ) 2 1 −3 −6
⇒ 11 λ2 + 12 λ + 6 = 6( λ − 1 ) 2 So, the equation of the plane is
( x + 2 ) − 3(y + 2 ) − 6(z − 2 ) = 0
= 6( λ − 2 λ + 1 )
2
or x − 3y − 6z + 8 = 0
⇒ 5 λ + 24 λ = 0
2
x y z 8 8
24 or + + = 1 ⇒ a = 8, b = , c =
⇒ λ = 0 or λ = − . −8 8 8 3 6
5 3 6
Thus, the equation of the required planes are ⇒ a = 3b, b = 2c, a + b + c = 12
2 x − y + z − 3 = 0 or 62 x + 29y + 19z − 105 = 0. and a + 2b + 2c = 16

JEE Type Solved Examples :


Statement I and II Type Questions
n
Directions (Q.Nos. 41-45) For the following questions, l Ex. 42 The equation of two straight line are
choose the correct answers from the codes (a), (b), (c) and x −1 y + 3 z − 2 x − 2 y −1 z + 3
(d) defined as follows: = = and = =
2 1 −3 1 −3 2
(a) Statement I is true, Statement II is also true; Statement II is
the correct explanation of Statement I. Statement I The given lines are coplanar.
(b) Statement I is true, Statement II is also true; Statement II is Statement II The equation 2 x 1 − y 1 = 1, x 1 + 3y 1 = 4,
not the correct explanation of Statement I. 3 x 1 + 2y 1 = 5 are consistent.
(c) Statement I is true, Statement II is false. Sol. (a) Any point on the first line is (2 x1 + 1, x 2 − 3, − 3 x1 − 2 ).
(d) Statement I is false, Statement II is true.
Any point on the second line is (y1 − 2, 3y1 + 1, 2y1 − 3 ).
l Ex. 41 Statement I A line L is perpendicular to the
If two lines are coplanar, then 2 x1 − y1 = 1, x1 + 3y1 = 4,
3 x1 + 2y1 = 5 are consistent.
plane 3 x − 4y + 5 z = 10.
Statement II Direction cosines of L be l Ex. 43 Statement I The distance between the planes
3 4 1 3
< ,− , >. 4 x − 5y + 3 z = 5 and 4 x − 5y + 3 z + 2 = 0 is .
5 2 5 2 2 5 2
Sol. (a) lx + my + nz = P be the equation of a plane in the normal Statement II The distance between ax + by + cz + d 1 = 0
form.
∴DR of the plane
d1 − d 2
and ax + by + cz + d 2 = 0 is .
3 x − 4y + 5z = 10 be < 3, − 4, 5 >. a +b2 +c2
2

⇒ Direction cosines
5+2 7
3 −4 1 Sol. (d) Distance = =
< , , >. 50 5 2
5 2 5 2 2
Chap 03 Three Dimensional Coordinate System 217

x −1 y − 2 z +1
x −1 y +1 z − 3 l Ex. 45 Statement I Line = = lies in the
l Ex. 44 Given the line L : = = and the 3 11 11
3 2 −1 plane 11x − 3 z − 14 = 0.
plane π : x − 2y − z = 0 Statement II A straight line lies in a plane, if the line is
Statement I L lies in π. parallel to plane and a point of the line in the plane.
Sol. (a) Statement I (1, 2, − 1 ) is a point on the line and
Statement II L is parallel to π.
11 + 3 − 14 = 0.
Sol. (c) x = 1 + 3r , y = − 1 + 2r , t = 3 − r ∴The point lies on the plane 11 x − 3z − 14 = 0.
∴ 1 + 3r − 2 ( − 1 + 2r ) − 3 + r = 0 Further 3 × 11 + 11 ( − 3 ) = 0.
3 ×1 −2 ×2 + 1 ×1 = 0 ∴The line lies in the plane.
Hence, L is parallel to π. Statement II is also true.

JEE Type Solved Examples :


Passage Based Questions
Passage I y = 2 + 3r1
(Ex. Nos. 46 to 48) z = 1 + 4r1
x − 3 y − 2 z −1 x −2 y −3 z −2
Two line whose equation are = = and It will lie on = = ⇒ r1 = 1
3 2 3
2 3 λ
x −2 y−3 z −2 So, point of intersection is (5, 5, 5).
= = lie in the same plane, then. 48. (b) Equation of plane contains both lines
3 2 3
x −3 y −2 z −1
l Ex. 46 The value of sin − 1 sin λ is equal to 2 3 4 =0
3 2 3
(a) 3 (b) π − 3
(c) 4 (d) π − 4 ( x − 3 ) (1 ) + (y − 2 ) (12 − 6 ) + (z − 1 ) ( 4 − 9 ) = 0
x + 6y − 5z = 10
π
l Ex. 47 Point of intersection of the lines lies on Thus, the angle is .
2
(a) 3 x + y + z = 20 (b) 2x + y + z = 25
(c) 3 x + 2y + z = 24 (d) x = y = z Passage II
l Ex. 48 Angle between the plane containing both the lines
(Ex. Nos. 49 to 51)
and the plane 4 x + y + 2 z = 0 is equal to Let a1 x + b1 y + c1 z + d1 = 0 and a 2 x + b2 y + c2 z = d 2 = 0 be
π π
two planes, where d1 , d 2 > 0. Then, origin lies in acute
(a) (b) angle, if a1 a 2 + b1 b2 + c1 c2 < 0 and origin lies in obtuse
3 2
angle, if a1 a 2 + b1 b2 + c1 c2 > 0.
π 2
(c) (d) cos − 1 Further point ( x1 , y1 , z1 ) and origin both lie either in acute
6 186
angle or in obtuse angle. If ( a1 x1 + b1 y1 + c1 z1 + d1 )
Sol. (Ex. 46-48)
( a 2 x1 + b2 y1 + c2 z1 + d 2 ) > 0.
46. (d) Both lines and coplanar
2 3 λ One of ( x1 , y1 , z1 ) and origin lie in acute and the other in
3 2 3 =0 obtuse angle; if ( a1 x1 + b1 y1 + c1 z1 + d1 )
1 −1 −1
( a 2 x1 + b2 y1 + c2 z1 + d 2 ) < 0.
⇒ 2 ( − 2 + 3 ) + 3(3 + 3 ) + λ( − 3 − 2 ) = 0 l Ex. 49 Given that planes 2 x + 3y − 4 z + 7 = 0 and
⇒ λ=4
x − 2y + 3 z − 5 = 0. If a point P is (1, − 2, 3 ), then
sin − 1 sin 4 = sin − 1 sin ( π − 4 ) = π − 4
(a) O and P both lie in acute angle between the planes
x −3 y −2 z −1
47. (d) Let = = = r1 (b) O and P both lie in obtuse angle
2 3 4
(c) O lies in acute angle, P lies in obtuse angle
⇒ x = 3 + 2r1
(d) O lies in obtuse angle, P lies an acute angle
218 Textbook of Vector & 3D Geometry

l Ex. 50 Given the planes x + 2y − 3 z + 5 = 0 and lEx. 54 The equation of plane containing line AC and at a
2 x + y + 3 z +1 = 0. If a point P is ( 2, − 1, 2 ), then maximum distance from B is
(a) O and P both lie in acute angle between the planes (a) r ⋅ ( i$ + j$ ) = 7 (b) r ⋅ ( $i − $j ) = 7
(b) O and P both lie in obtuse angle (c) r ⋅ ( 2$i − $j ) = 7 (d) r ⋅(3 i$ + 4 j$ ) = 7
(c) O lies in acute angle, P lies in obtuse angle
(d) O lies in obtuse angle, P lies an acute angle (4i+3j) C E B(7i+7j)

l Ex. 51 Given the planes x + 2y − 3 z + 2 = 0 and P F

x − 2y + 3 z + 7 = 0, if the point P is (1, 2, 2 ) then


(a) O and P both lie in acute angle between the planes O A(3i+4j)
(b) O and P both lie in obtuse angle
(c) O lies in acute angle, P lie in obtuse angle Sol. (Ex 52-54)
13 21
(d) O lies in obtuse angle, P lies in acute angle 52. (d) OB = 7 $i + 7 $j , OE = 5 $i + $j , OP = ( $i + $j)
3 5
Sol. (Ex. 49-51)
53. (b) Direction ratio of CP is (1, 6, 0), then equation of line
49. (b) Equation of the second plane is − x + 2y − 3z + 5 = 0
passing through (2, 3, 4) and parallel to CP is
2( − 1 ) + 3 ⋅ 2 + ( − 4 ) ( − 3 ) > 0
x −2 y − 3 z − 4
∴Origin lies in obtuse angle. = =
1 6 0
(2 × 1 + 3( − 2 ) − 4 × 3 + 7 ) ( − 1 + 2 ( − 2 ) − 3 × 3 + 5 )
= (2 − 6 − 12 + 7 ) ( − 1 − 4 − 9 + 5 ) > 0 54. (a) The plane containing line AC and at a maximum distance
from B must be perpendicular to the plane OABC.
∴P lies in obtuse angle.
Since, OABC is rhombus, so OB must normal to the plane. So,
50. (c) 1 × 2 + 2 × 1 − 3 × 3 < 0 equation of required plane is
∴ Origin lies in acute angle. [ r − 4 i$ − 3 $j] ⋅ ( $i + $j) = 0
Also, (2 + 2( − 1 ) − 3(2 ) + 5 ) (2 × 2 − 1 + 3 × 2 + 1 ) = ( − 1 ) (10 ) < 0 ⇒ ( $i + $j) = 7
∴ P lies in obtuse angle.
51. (a) 1 − 4 − 9 < 0 Passage IV
∴Origin lies in acute angle. (Ex. Nos. 55 to 57)
Further (1 + 4 − 6 + 2 ) (1 − 4 + 6 + 7 ) > 0 A ray of light comes along the line L = 0 and strikes the
∴The point P lies in acute angle. plane mirror kept along the plane P = 0 at B. A(2, 1, 6) is a
Passage III point on the line L = 0 whose image about P = 0 is A′. It is
x − 2 y −1 z − 6
(Ex. Nos. 52 to 54) given that L = 0 is = = and P = 0 is
3 4 5
In a parallelogram OABC with position vectors of A is
x + y − 2 z = 3.
3$i + 4$j and C is 4$i + 3$j with reference to O as origin. A
point E is taken on the side BC which divides it in the ratio l Ex. 55 The coordinates of A′ are
of 2 :1. Also, the line segment AE intersects the line (a) (6, 5, 2) (b) (6, 5, − 2)
bisecting the ∠AOC internally at P. CP when extended
(c) (6, − 5, 2) (d) None of these
meets AB at point F .
l Ex. 56 The coordinates of B are
l Ex. 52 The position vector of P is
(a) (5, 10, 6) (b) (10, 15, 11)
2 $ $
(a) i$ + j$ (b) ( i + j) (c) (− 10, − 15, − 14) (d) None of these
3
13 $ $ 21
(c) ( i + j) (d) ( $i + $j ) l Ex. 57 If L1 = 0 is the reflected ray, then its equation is
3 5
x + 10 y − 5 z + 2
(a) = =
l Ex. 53 The equation of line parallel to CP and passing 4 4 3
x + 10 y + 15 z + 14
through ( 2, 3, 4 ) is (b) = =
x −2 y −3 x − 2 y −3 3 5 5
(a) = ,z =4 (b) = ,z =4 x + 10 y + 15 z + 14
1 5 1 6 (c) = =
x −2 y −3 x −2 y − 3 4 5 3
(c) = ,z =3 (d) = ,z =3 (d) None of the above
2 5 3 5
Chap 03 Three Dimensional Coordinate System 219

Sol. (Ex 55-57) l Ex. 59 The equation of a line of greatest slope can be
55. (b) Let Q( x 2, y 2, z 2 ) be the image of A(2, 1, 6 ) about mirror x y z x y z
x + y − 2z = 3. Then,
(a) = = (b) = =
3 1 −1 3 −1 1
x 2 −2 y 2 −1 z 2 − 6
= = (c)
x y z
= = (d)
x y
= =
z
1 1 −2
−3 1 1 1 3 −1
− 2 (2 + 1 − 12 − 3 )
= =4
1 2+ 1 2 + 2 2 l Ex. 60 The coordinates of a point on the plane
⇒ ( x 2, y 2, z 2 ) ≡ (6, 5, − 2 ) 2 x + y − 5 z = 0, 2 11 unit away from the line of intersection
x−2 y −1 z −6 of 2 x + y − 5 z = 0 and 4 x − 3y + 7 z = 0 are
56. (c) Let = = =λ
3 4 5 (a) (6, 2, − 2) (b) (3, 1, − 1)
x = 2 + 3λ, y = 1 + 4λ, z = 6 + 5λ lies on plane x + y − 2z = 3 (c) (6, − 2, 2) (d) (1, 3, − 1)
⇒ 2 + 3 λ + 1 + 4 λ − 2(6 + 5 λ) = 3 Sol. (Ex. 58-60)
⇒ 3 + 7 λ − 12 − 10 λ = 3 58. (a) Plane P1 is of the form r ⋅ n1 = 0, where n1 = ( 4, − 3, 7 )
⇒ − 3 λ = 12
Plane P2 is of the form r ⋅ n 2 = 0,
⇒ λ=−4
where n 2 = (2, 1, − 5 )
Point B ≡ ( − 10, − 15, − 14 )
The vector b along the line of intersection of planes is
57. (c) The equation of the reflected ray L1 = 0 is the line joining
n1 × n1 = ( 4, 17, 5 ) = n 3
Q( x 2, y 2, z 2 ) and B( − 10, − 15 , − 14 ).
x + 10 y + 15 z + 14 Since the line of greatest slope is perpendicular to n 3 and n 2
= = the vector along the line of greatest slope
16 20 12
= n 2 × n 3 = (3, − 1, 1 ) = n 4
x + 10 y + 15 z + 14
or = = and the unit vector
4 5 3
 3 −1 1 
n 4 = n$ =  , , 
Passage V  11 11 11 
(Ex. Nos. 58 to 60) 59. (b) Since, ( 0, 0, 0 ) is a point on both planes, it lies on the line of
The line of greatest slope on an inclined plane P1 is the line intersection.
in the plane P1 which is perpendicular to the line of Hence, the equation a line of greatest slope can be
intersection of the plane P1 and a horizontal plane P2 . x y z
= =
3 −1 1
l Ex. 58 Assuming the plane 4 x − 3y + 7 z = 0 to be x y z
60. (c) The point on the line = = at a distance 2 11 unit
horizontal, the direction cosines of the line of greatest slope 3 −1 1
in the plane 2 x + y − 5 z = 0 are from the origin is given by
3 −1 1 3 1 −1 x y z
(a) , , (b) , , = = = 2 11
11 11 11 11 11 11 3 −1 1
−3 1 1 11 11
(c) , , (d) None of these
11 11 11 The point is (6, − 2, 2 ) .
220 Textbook of Vector & 3D Geometry

JEE Type Solved Examples :


Matching Type Questions
l Ex. 61 Match the entries between following two columns. ∴The lines are coincident.
Column I Column II
(c) < 5, 4, − 2 > are direction ratios of both the lines.
∴The lines are parallel.
A. If the line x − 1 = y + 1 = z + 1 lies in the 6
p. sin − 1 Also, x = 2 + 5λ, y = − 3 + 4λ, z = 5 − 2λ
1 −2 λ 25
2 + 5λ − 7 − 3 + 4λ − 1 5 − 2λ − 2
plane 3 x − 2y + 5z = 0, then λ is equal to ∴ = =
5 4 −2
B. If ( 3 , λ, µ) is a point on the line q. −
7
2 x + y + z − 3 = 0 = x − 2y + z − 1, then 3 − 2λ
5 i.e. λ −1 = λ − 1 =
λ + µ is equal to −2
C. The angle between the line x = y = z and r. −3 ∴No value of λ.
the plane 4 x − 3y + 5z = 2 is
Thus, the lines are parallel and different.
D. The angle between the planes 8
s. cos − 1 (D) < 2, 3, 5 > and <3, 2, 5 > are direction ratios of first and
x + y + z = 0 and 3 x − 4y + 5z = 0 is 75 second line, respectively.
Sol. (A) → (q), (B) → (r), (C) → (p), (D) → (s)
7
l Ex. 63 Match the followings
(A) 3 ⋅ 1 − 2 ( − 2 ) + 5(λ) = 0 ⇒ λ = −
5 Column I Column II
(B) Point (3, λ, µ) lies on 2 x + y + z − 3 = x − 2y + z − 1 A. The coordinates of a point on the line x = 4y + 5 , p. ( − 1, − 2 , 0 )
⇒ 3 ⋅ 2 + λ + µ − 3 = 0 and 3 − 2 λ + µ − 1 = 0 z = 3y − 6 at a distance 3 from the point (5, 3, − 6)
is/are
⇒ λ + µ + 3 = 0 and 2 λ − µ − 2 = 0
So, λ + µ = −3 B. The plane containing the lines x − 2 = y + 3 q. (5, 0 , − 6 )
1 ⋅ 4 + 1( − 3 ) + 1 ⋅ 5 6 3 5
(C) sin θ = = z+5
= and parallel to $i + 4 $j + 7k$ has the point
1 2 + 1 2 + 1 2 16 + 9 + 25 3 50 7
6 C. A line passes through two points A(2 , − 3 , − 1) r. (2 , 5, 7 )
θ = sin − 1 and B( 8 , − 1, 2 ). The coordinates of a point on this
25
line nearer to the origin and at a distance of 14
1 ⋅ 3 + 1( − 4 ) + 1 ⋅ 5 4
(D) cos θ = = units from A is/are
3 16 + 9 + 25 3 50 D. The coordinates of the foot of the perpendicular s. (14 , 1, 5)
−1 8 from the point ( 3 , − 1, 11) on the line
θ = cos x y −2 z −3
75 = = is/are
2 3 4

l Ex. 62 Match the following Sol. (A) → (q), (B) → (p), (C) → (s), (D) → (r)
Column I Column II (A) The given line is x = 4y + 5, z = 3y − 6,
x −5 z +6
A. x − 1 = y − 2 = z − 3 and x − 1 = y − 3 = z − 5 p. coincident or = y, =y
4 3
2 3 4 3 4 5
x−5 y z + 6
are or = = =λ (say)
B. x − 1 = y − 2 = z − 3 and x − 3 = y − 5 = z − 7 q. parallel and 4 1 3
2 3 4 2 3 4 different Any point on the line is of the form (4 λ + 5, λ, 3 λ − 6 ).
are The distance between (4 λ + 5, λ, 3 λ − 6 ) and (5, 3, − 6) is 3 units
C. x − 2 = y + 3 = 5 − z and x − 7 = y − 1 = z − 2 r. skew (given).
5 4 2 5 4 −2 Therefore, ( 4 λ + 5 − 5 ) 2 + ( λ − 3 ) 2 + (3 λ − 6 + 6 ) 2 = 9
are
⇒ 16 λ2 + λ2 + 9 − 6 λ + 9 λ2 = 9
D. x − 3 = y + 2 = z − 4 and x − 3 = y − 2 = z − 7 s. intersecting
2 3 5 3 2 5 in a point ⇒ 26 λ2 − 6 λ = 0
are 3
⇒ λ = 0,
13
Sol. (A) → (s), (B) → (p), (C) → (q), (D) → (r)
The point is (5, 0, − 6)
(A) Both the lines pass through the point (7, 11 , 15).
(B) The equation of the plane containing the lines
(B) < 2, 3, 4 > are direction ratios of both the lines. Also, x−2 y + 3 z + 5
the point (1, 2, 3) is common to both. = = and parallel to $i + 4 $j + 7 k$ .
3 5 7
Chap 03 Three Dimensional Coordinate System 221

x −2 y + 3 z + 5 x −2 y − 1 z − 3
(B) Any point on the line = = = λ is
1 4 7 = 0 ⇒ x − 2y + z − 3 = 0 −3 2 2
3 5 7 ( − 3 λ + 2, 2 λ + 1, 2 λ + 3 ), which lies on plane 2 x + y − z = 3.
Therefore,
Point ( − 1, − 2, 0 ) lies on this plane.
− 6λ + 4 + 2λ + 1 − 2λ − 3 = 3
(C) The line passing through points A(2, − 3, − 1 )
− 6λ = 1
x −2 y +3 z +1
and B(8, − 1, 2 ) is = = 1
8 −2 −1 + 3 2 + 1 λ=−
6
x −2 y + 3 z + 1
or = = =λ (say)  5 2 8
6 2 3 Therefore, the point is  , , 
 2 3 3
Any point on this line is of the form P(6 λ + 2, 2 λ − 3, 3 λ − 1 ),
(C) If ( x, y , z ) is required foot of the perpendicular, then
whose distance from point A(2, − 3, − 1 ) is 14 units. Therefore,
x − 1 y − 1 z − 2 (2 − 2 + 8 + 5 )
⇒ PA = 14 ⇒ PA 2 =(14 ) 2 = = = 2
2 −2 4 2 + (− 2)2 + 42
⇒ (6 λ)2 + (2 λ)2 + (3 λ)2 = 196
 − 1 25 − 2
or ( x, y , z ) ≡  , , 
⇒ 49 λ2 = 196 ⇒ λ2 = 4 ⇒ λ = ± 2  12 12 12 
Therefore, the required points are (14, 1, 5) and ( − 10, − 7, − 7 ). x − 1 y − 2 z −3
The point nearer to the origin is (14, 1, 5). (D) Any point on the line = = = λ is
2 3 4
x y −2 z −3 P(2 λ + 1, 3 λ + 2, 4 λ + 3 ), which satisfies the line
(D) Any point on line AB , = = = λ is
2 3 4 x −4 y − 1 z
M(2 λ, 3 λ + 2, 4 λ + 3 ). Therefore, the direction ratios of PM are = =
5 2 1
2 λ − 3, 3 λ + 3 and 4 λ − 8. 2λ + 1 − 4 3λ + 2 − 1 4λ + 3
But PM ⊥ AB or = =
5 2 1
P (3, –1, 11)
⇒ λ = −1
The required point is ( − 1, − 1, − 1 ).

x − 2 y −3 z − 4
l Ex. 65 = =
A
M
B 3 4 5
Column I Column II
∴ 2(2 λ − 3 ) + 3(3 λ + 3 ) + 4( 4 λ − 8 ) = 0
4 λ − 6 + 9 λ + 9 + 16 λ − 32 = 0 A. Point on the line at a distance 10 2 from p. ( − 1, − 1, − 1)
(2, 3, 4)
29 λ − 29 = 0 ; λ = 1
Therefore, foot of the perpendicular is M(2, 5, 7 ). B. Point on the line common to the plane q. (2, 3, 4)
x+y+z+ 3=0
l Ex. 64 Match the followings C. Point on the line at a distance 29 from the origin. r. (8, 11, 14)
D. Point on the line common to the plane s. (− 4 , − 5, − 6)
Column I Column II x + y −z + 3 = 0
A. Image of the point (3, 5, 7) in the plane p. ( − 1, − 1, − 1)
2 x + y + z = − 18 is Sol. (A) → (r, s), (B) → (p), (C) → (q), (D) → (s)
B. The point of intersection of the line q. ( − 21, − 7 , − 5) Any point on the line is (3r + 2, 4r + 3, 5r + 4
x −2 y −1 z − 3 (A) (3r + 2 − 2 ) 2 + ( 4r + 3 − 3 ) 2 + (5r + 4 − 4 ) 2 = 200
= = and the plane
−3 −2 2
⇒ (9 + 16 + 25 )r 2 = 200 ⇒ r = ± 2
2 x + y − z = 3 is
For r = 2, the point is (8, 11, 14), For r = − 2 it is ( − 4, − 5, − 6 )
C. The foot of the perpendicular from the point r.  5 2 8 
(1, 1, 2 ) to the plane 2 x − 2y + 4z + 5 = 0 is  , ,  (B) 3r + 2 + 4r + 3 + 5r + 4 + 3 = 0
2 3 3
⇒ 12r + 12 = 0 ⇒r = − 1
D. The intersection point of the lines s.  1 25 2 
x −1 y − 2 z − 3 x − 4 y −1 − , ,  and the point on the line common to the plane is ( − 1, − 1, − 1 ).
= = and = = z is  12 12 12 
2 3 4 5 2 (C) (3r + 2 ) 2 + ( 4r + 3 ) 2 + (5r + 4 ) 2 = 29
76
Sol. (A) → (q), (B) → (r), (C) → (s), (D) → (p) 50r 2 + 76r = 0 ⇒r = 0, r = −
50
(A) If the required image is ( x, y , z ), then
For r = 0, the point is (2, 3, 4).
x − 3 y − 5 z −7 2(6 + 5 + 7 + 18 )
= = =− (D) 3r + 2 + 4r + 3 − 5r − 4 + 3 = 0
2 1 1 22 + 12 + 12
⇒ 2r + 4 = 0 ⇒ r = −2
= − 12 or ( − 21, − 7, − 5 ). ∴ The point on the line common to the plane is ( −4, − 5, − 6 )
222 Textbook of Vector & 3D Geometry

JEE Type Solved Examples :


Single Integer Answer Type Questions
l Ex. 66 If the perpendicular distance of the point (6, 5, 8) Shortest distance
from the Y-axis is 5λ unit, then λ is equal to =
[( −3 − 3 ) $i + (7 − 8 ) $j + (6 − 3 ) k$ ]
Sol. (2) Foot of perpendicular from (6, 5, 8) on Y-axis is (0, 5, 0). [(3 $i − $j + k$ ) × ( −3 $i + 2 $j + 4 k$ )]
Required distance = (6 − 0 ) 2 + (5 − 5 ) 2 + (8 − 0 ) 2 ( −6 i$ − 15 $j + 3 k)
$ × ( −6 $j − 15 k$ + 3 k)$
=
= 10 unit 36 + 225 + 9
10 = 270 = 3 30 unit
⇒ 5 λ = 10 ⇒ λ = =2
5
l Ex. 69 If the planes x − cy − bz = 0, cx − y + az = 0 and
l Ex. 67 A parallelopiped is formed by planes drawn bx + ay − z = 0 pass through a line, then the value of
through the points (2, 4, 5) and (5, 9, 7) parallel to the coordi-
a 2 + b 2 + c 2 + 2abc is
nate planes. The length of the diagonal of the parallelopiped
is Sol. (1) Given, planes are
x − cy − bz = 0 ...(i)
Sol. (7) The length of the edges are given by a = 5 − 2 = 3,
cx − y + az = 0 ...(ii)
b = 9 − 3 = 6, c = 7 − 5 = 2, so length of the diagonal
bx + ay − z = 0 (iii)
= a2 + b2 + c2 Equation of planes passing through the line of intersection of
= 9 + 36 + 4 planes (i) and (ii) may be taken as
( x − cy − bz ) + λ(cx − y + az ) = 0 ...(iv)
= 7 units
Now, planes (iii) and (iv) are same
1 + cλ − (c + λ) − b + aλ
l Ex. 68 If the shortest distance between the lines ∴ = =
b a −1
x −3 y − 8 z − 3 x +3 y +7 z −6
= = and = = is λ 30 unit, By eliminating λ, we get a 2 + b 2 + c 2 + 2abc = 1
3 −1 1 −3 2 4
then the value of λ is x − 4 y − 2 z −k
Sol. (3) Given, lines are
l Ex. 70 If the line = = lies exactly on the
1 1 2
r = 3 $i + 8 $j + 3 k$ + λ(3i$ − $j + k$ )
plane 2 x − 4y + z = 7, the value of k is
r = ( −3 $i − 7 $j + 6 k$ ) + µ( −3 $i + 2 $j + 4 k$ ) Sol. (7) The point ( 4, 2, k ) must satisfy the plane.
where λ, µ are parameters. So, 8 − 8 +k = 7 ⇒ k = 7

Subjective Type Questions


l Ex. 71 The equation of motion of rockets are Thus, the path of the rocket represents a straight line passing
x = 2t , y = − 4t , z = 4t , through the origin.
where the time ‘t’ is given in second and the coordinates of a For t = 10 s
moving point in kilometres. We have, x = 20, y = − 40, z = 40
What is the path of the rocket? At what distance will the and | r | = | OM | = x 2 + y 2 + z 2
rocket be from the starting point O (0, 0, 0 ) in 10s. = 400 + 1600 + 1600 = 60 km
Sol. Eliminating ‘t’ from the given equations, we get the equation of
the path, l Ex. 72 Write the equation of a tangent to the curve x = t ,
x y z
= = y = t 2 , z = t 3 at its point M(1, 1, 1); (t =1).
2 −4 4
x y z Sol. Here, r = t$i + t 2$j + t 3k$
or = =
1 −2 2 dr $
= i + 2t$j + 3t 2k$
dt
Chap 03 Three Dimensional Coordinate System 223

Hence, the direction of the tangent at the point M is (a 2 + b 2 ) sin 2 α


determined by the vector. k2 =
cos2 α
 dr 
  = $i + 2 $j + 3 k$ On putting in Eq. (iii) k = ± a 2 + b 2 tan α, we get equation of
 dt  M
plane as,
Thus, the equation of the desired tangent is,
x − 1 y − 1 z −1 ax + by ± z a 2 + b 2 tan α = 0.
= =
1 2 3
l Ex. 75 Assuming the plane 4 x − 3y + 7 z = 0 to be hori-
l Ex. 73 Find the locus of a point, the sum of squares of zontal, find the equation of the line of greatest slope through
whose distances from the planes x − z =0, x − 2y + z = 0 and the point ( 2, 1, 1) in the plane 2 x + y − 5 z = 0.
x + y + z = 0 is 36. Sol. The required line passing through the point (2, 1, 1) in the
Sol. Given planes are x − z = 0, x − 2y + z = 0 plane 2 x + y − 5z = 0 and is having greatest slope, so it must
and x + y + z = 0. be perpendicular to the line of intersection of the planes
Let the point whose locus is required be P(α, β, γ). According 2 x + y − 5z = 0 ...(i)
to question, and 4 x − 3y + 7z = 0 ...(ii)
| α + γ | 2 | α − 2β + γ | 2 | α + β + γ | 2 Let the DR’s of the line of intersection of Eqs. (i) and (ii) are a,
+ + = 36 b, c.
2 6 3
2a + b − 5c = 0
or 3(α 2 + γ 2 − 2αγ) + α 2 + 4β 2 + γ 2 − 4αβ − 4βγ + 2αγ
and 4a− 3b + 7c = 0
+ 2(α 2 + β 2 + γ 2 + 2αβ + 2βγ + 2αγ ) = 36 × 6 (as DR’s of straight line (a, b, c ) is perpendicular to DR’s of
or 6α 2 + 6β 2 + 6 γ 2 = 36 × 6 normal to both the planes)
a b c
or α 2 + β 2 + γ 2 = 36 = =
4 17 5
Hence, the required equation of locus is
Now, let the direction ratio of required line be proportional to
x 2 + y 2 + z 2 = 36 l, m and n, then its equation be
x −2 y −1 z −1
= =
l Ex. 74 The plane ax + by = 0 is rotated through an angle l m n
α about its line of intersection with the plane z = 0. Show where, 2l + m − 5n = 0 and 4l + 17m + 5n = 0
that the equation to the plane in new position is l m n
So, = =
3 −1 1
ax + by ± z a 2 + b 2 tan α = 0
x −2 y − 1 z −1
Sol. Given planes are Thus, the required line is = =
3 −1 1
ax + by = 0 ...(i)
and z =0 ...(ii) a b c
∴Equation of any plane passing through the line of
l Ex. 76 Does + + = 0 represents a pair
intersection of planes (i) and (ii) may be taken as,
x −y y − z z − x
ax + by + kz = 0 ...(iii) of planes?
The direction cosines of a normal to the plane (iii) are a b c
Sol. Here, given equation is + + =0
a b k x −y y −z z − x
, ,
a2 + b2 +k2 a2 + b2 + k2 a2 + b2 +k2 ⇒ a(y − z )(z − x ) + b( x − y ) (z − x ) + c( x − y ) (y − z ) = 0
The direction cosines of a normal to the plane (i) are ⇒ − axy + ayz − az 2 + axz + bxz − bx 2 − byz
a b + byx + cxy − cxz − cy 2 + cyz = 0
, ,0
a +b
2 2
a + b2
2
⇒ bx 2 + cy 2 + az 2 − (b + c − a ) xy −
Since, the angle between the planes (i) and (iii) is α. (c + a − b )yz − (a + b − c )zx = 0
a ⋅ a + b ⋅b + k ⋅ 0 ∴Value of determinant;
∴ cos α =
a + b2 + k2 a2 + b2
2 1 1
b − (b + c − a ) − (a + b − c )
2 2
a +b2 2
1 1
= − (b + c − a ) c − (c + a − b )
a + b2 + k2
2
2 2
1 1
k 2 cos2 α = α 2(1 − cos2 α) + b 2(1 − cos2 α) − (a + b − c ) (c + a − b ) a
2 2
224 Textbook of Vector & 3D Geometry

l Ex. 79 A line makes angle, α, β, γ and δ with the four


0 0 0
1 1 diagonals of cube, prove that
= − (b + c − a ) c − (c + a − b )
2 2 4
1 1 cos 2 α + cos 2 β + cos 2 γ + cos 2 δ = ⋅
− (a + b − c ) − (c + a − b ) a 3
2 2
Sol. Let the cube be shown in the figure, where four diagonals are
[ R1 → R1 + R2 + R3 ] OP, AL, BM and CN and A(a, 0, 0 ) , B( 0, a, 0 ), C ( 0, 0, a ), L( 0, a, a ),
=0 M(a, 0, a ), N (a, a, 0 ) and P (a, a, a ), hence direction cosines of OP
Hence, the given equation represents a pair of planes. are
z
x −α y − β z − γ
l Ex. 77 If the straight line = = intersect C (0, 0, a) M (a, 0, a)
l m n
the curve ax 2 + by 2 = 1, z = 0, then prove that L P ( a , a , a ) (l , m , n )
a (αn − γl ) 2 + b(βn − γm ) 2 = n 2 . (0, a, a) a

z
x −α y −β z − γ
Sol. Here, = = =λ
l m n x
O A(a,0, 0)
∴Any point on the given line
( λl + α, λm + β + λn + y ) B(0, a, 0) N(a, a, 0)
If it lies on the curve ax 2 + by 2 = 1, z = 0
y
(as the point of intersection)
a a a
a(α + lλ) + b(β + mλ)2 = 1
2
...(i) , ,
a +a +a
2 2 2
a +a +a
2 2 2
a + a2 + a2
2
and λn + γ = 0 ...(ii)
−γ  1 1 1 
From Eq. (ii), λ = must satisfy Eq. (i), we get = , , 
n  3 3 3

 lγ 
2
 mγ 
2
 1 1 1 
a α −  + b β −  =1 The DC’s of AL are  − , , .
 n  n   3 3 3
a(nα − lγ)2 + b(nβ − mγ)2 = n 2  1
The DC’s of BM are  , −
1 1 
, .
 3 3 3
l Ex. 78 Prove that the three lines from O with direction  1 1 1 
The DC’s of CN are  , ,− .
cosines l1 , m1 , n1 ; l 2 , m 2 , n 2 and l3 , m3 , n3 are coplanar, if  3 3 3
l1 (m 2 n3 − n 2 m3 ) + m1 (n 2 l3 − l 2 n3 ) + n1 (l 2 m3 − l3 m 2 ) = 0. Let the DC’s of required line be (l , m, n ).
Sol. Here, three given lines are coplanar, if they have common l +m+n −l + m + n
∴ cos α = , cos β = ,
perpendicular. 3 3
Let DC’s of common perpendicular be l, m and n. l −m + n l + m −n
cos γ = and cos δ =
ll1 + mm1 + nn1 = 0 ...(i) 3 3
ll 2 + mm2 + nn2 = 0 ...(ii) cos2 α + cos2 β + cos2 γ + cos2 δ
and ll 3 + mm3 + nn3 = 0 ...(iii)
1
Solving Eqs. (ii) and (iii) by cross multiplication method, we = {(l + m + n ) 2 + ( − l + m + n ) 2 + (l − m + n ) 2
3
get
+ (l + m − n ) 2 }
l m n
= =
4 4
m2 n2 l2 m2 = (l 2 + m 2 + n 2 ) =
3 3

m3 n3 l3 m3 l Ex. 80 Let PM be the perpendicular from the point


P(1, 2, 3 ) to XY-plane. If OP makes an angle θ with the posi-
l m n
⇒ = = =k tive direction of the Z-axis and OM makes an angle φ with
m2n3 − n2m3 n2l 3 − n3l 2 l 2m3 − l 3m2
the positive direction of X-axis, where O is the origin, then
⇒ l = k(m2n3− n2m3 ), m = k(n2l 3 − n3l 2 ), n = k(l 2m3 − l 3m2 )
find θ and φ.
Substituting in Eq. (i), we get
Sol. Here, P be ( x, y , z ) shown as,
k(m2n3 − n2m3 )l1 + k(n2l 3 − n3l 2 )m1 + k(l 2m3 − l 3m2 )n2 = 0
⇒ l1(m2n3 − n2m2 ) + m1(n2l 3− n3l 2 ) + n1(l 2m3 − l 3m2 ) then, x = r sin θ ⋅ cos φ, y = r sin θ sin φ, z = r cos θ ...(i)
Chap 03 Three Dimensional Coordinate System 225

z 5r = 5
⇒ r =1
P (1, 2, 3) ∴Coordinate of P(3, 3, − 9 ).
⇒ Distance between (1, 0, − 3 ) and (3, 3, − 9 )
r
= (3 − 1 ) 2 + (3 − 0 ) 2 + ( − 9 + 3 ) 2
q
O y = 4 + 9 + 36 = 7
f
l Ex. 82 Find the equation of the plane which passes
through a 1 x + b1y + c 1 z + d 1 = 0, a 2 x + b 2 y + c 2 z + d 2 = 0
x M x −a y − β z − γ
and which is parallel to the line = = .
⇒ 1 = r sin θ ⋅ cos φ, 2 = r sin θ sin φ, 3 = r cos θ l m n
⇒ 1 2 + 2 2 + 3 2 = r 2 sin 2 θ cos2 φ + r 2 sin 2 θ sin 2 φ + r 2 cos2 θ Sol. Given, a1x + b1y + c1z + d1 = 0

= r sin θ (cos φ + sin φ ) + r cos θ


2 2 2 2 2 2 a 2x + b2y + c 2z + d 2 = 0 ...(i)
x − α y −β z − γ
= r 2 sin 2 θ + r 2 cos2 θ = r 2 and = = ...(ii)
l m n
⇒ r = ± 14 Equation of plane through the intersection of plane (i) is given
∴ From Eq. (i), we have by
1 (a1x + b1y + c1z + d1 ) + λ(a 2x + b2y + c 2z + d 2 ) = 0
sin θ cos φ = + ,
14 or (a1 + λa 2 ) x + (b1 + λb2 )y + (c1 + λc 2 )z
2 3 + (d1 + λd 2 ) = 0 ...(iii)
sin θ sin φ = , cos θ =
14 14 DR’s of normal to Eq. (iii) are
(neglecting − ve sign as acute angles) (a1 + λa 2 ), (b1 + λb2 ), (c1 + λc 2 )
sin θ sin φ 2 ∴ Eq. (iii) is parallel to Eq. (ii).
∴ =
sin θ cos φ 1 ⇒ Normal to plane (iii) should be perpendicular to line (ii).
sin θ 5 ∴ (a1 + λa 2 )l + (b1 + λb2 )m + (c1 + λc 2 )n = 0
and tan θ = = (a l + b1m + c1n )
cos θ 3 ⇒ λ=− 1 , putting in (iii), we get
(a 2l + b2m + c 2n )
5
⇒ tan φ = 2 and tan θ = (a1x + b1y + c1z + d1 ) (a 2l + b2m + c 2n ) − (a1l + b1 + c1n )
3
 5 (a 2x + b2y + c 2z + d 2 ) = 0
⇒ φ = tan − 1 2 and θ = tan − 1   Hence, the equation of required plane.
 3 
l Ex. 83 Find the perpendicular distance of a corner of a
l Ex. 81 Find the distance of the point (1, 0, − 3 ) from the unit cube from a diagonal not passing through it.
plane x − y − z = 9 measured parallel to the line, Sol. Let the edges OA, OB, OC of the unit cube be along OX , OY and
x − 2 y + 2 2 −6 OZ, respectively. Since, OA = OB = OC = 1 unit
= = .
2 3 −6 ∴ OA = $i, OB = $j , OC = k$
Sol. Given plane is z
x −y −z =9 ...(i)
x −2 y + 2 z −6 C(0, 0, 1)
Given line AB is = = ...(ii)
2 3 −6
P(1, 1, 1)
Equation of line passing through (1, 0, − 3) and parallel to
x − 2 y + 2 z −6 k
= = M
2 3 −6
y
x −1 y − 0 z + 3 x
O j B (0, 1, 0)
is = = =r ...(iii) i
2 3 −6
A(1, 0, 0) D
Coordinate of any point on Eq. (iii) may be given as
x
P (2r + 1, 3r, − 6r − 3)
If P is intersection of Eqs. (i) and (iii), then it must lie on Eq. (i). Let CM be perpendicular from the corner C on the diagonal OP.
(2r + 1 ) − (3r ) − ( − 6r − 3 ) = 9 The vector equation of OP is
2r + 1 − 3r + 6r + 3 = 9 r = λ( i$ + $j + k$ )
226 Textbook of Vector & 3D Geometry

∴ OM = Projection of OC on OP − λ + 4 µ + 2 − 2 λ + 7 2µ + 3
⇒ = , =
= OC ⋅ OP λ+1 µ +1 λ +1 µ+1
( i$ + $j + k$ ) 1 λ + 8 5µ + 4
= k$ = =
3 3 λ +1 µ + 1
Now, OC 2 = OM 2 + CM 2 − ( λ + 1 ) + 5 (µ + 1 ) + 1
⇒ =
1 2 λ +1 µ+1
⇒ CM 2 = | OC | 2 − OM 2 = 1 − =
3 3 − 2( λ + 1 ) + 9 2(µ + 1 ) + 1
=
2 λ+1 µ +1
⇒ CM =
3 ( λ + 1 ) + 7 5(µ + 1 ) − 1
=
λ +1 µ+1
l Ex. 84 If a variable plane forms a tetrahedron of constant 5 1
⇒ −1 + =1 +
volume 64k 3 with the coordinate planes, then find the locus λ+1 µ +1
of the centroid of the tetrahedron. 9 1
−2 + =2 + ;
Sol. Let the variable plane intersects the coordinate axes at λ+1 µ+1
A(a, 0, 0 ), B( 0, b, 0 ) and C ( 0, 0, c ). Then, the equation of the 7 1
plane will be 1+ =5 −
λ+1 (µ + 1 )
x y z
+ + =1 ...(i) 1 1
a b c Let = x and =y
λ +1 µ+1
Let P(α, β, γ) be the centroid of tetrahedron OABC, then
a b c ⇒ 5 x − y = 2; 9 x − y = 4; 7 x + y = 4
α= ,β = , γ = 1 1
4 4 4 On solving, x = , y =
or a = 4α, b = 4β, c = 4γ 2 2
1 ⇒ λ + 1 = 2, µ + 1 = 2
⇒ Volume of tetrahedron = (Area of ∆AOB ) ⋅ OC
3 λ = 1, µ = 1
1 1  abc Clearly, if λ = 1 and µ = 1,
⇒ 64k =  ab c =
3
3 2  6 AB and CD bisects each other.
( 4α ) ( 4β) ( 4 γ)  3 5 9
⇒ 64k =
3 ∴ P = , , 
6  2 2 2
αβγ 2 2 2
⇒ k =
3  3  5  9
Now, AP =  4 −  + 7 −  + 8 − 
6  2  2  2
∴Required locus of P(α, β, γ ) is xyz = 6k 3.
155
= = PB
2
l Ex. 85 Show that the line segments joining the points
2 2 2
 3  5  9
( 4, 7, 8 ), ( − 1, − 2, 1) and (2, 3, 4) (1, 2, 5) intersect. Verify Also, CP = 2 −  + 3 −  +  4 − 
 2  2  2
whether the four points concyclic.
Sol. Here, A( 4, 7, 8 ), B ( − 1, − 2, 1 ), C(2, 3, 4 ) and D(1, 2, 5 ). If the lines 3
== PD
AB and CD intersect at P, then let 2
We know four points A, B, C and D are concylic, if
C B AP ⋅ PB = PC ⋅ PD
m
1
A
P
C
1 P
l
A D D

AP λ CP µ B
= and =
PB 1 PD 1
But here,
 − λ + 4 − 2λ + 7 λ + 8 155 3
Then, P  , ,  AP ⋅ PB = and PC ⋅ PD =
 λ +1 λ + 1 λ + 1 4 4
 µ + 2 2µ + 3 5µ + 4 ∴ Points are non-concyclic.
= , , 
µ +1 µ +1 µ +1 
Chap 03 Three Dimensional Coordinate System 227

l Ex. 86 If P be any point on the plane lx + my + nz = p ⇒ a′ ( p + α) + b′ (q + β) + c′ (r + γ) + 2d ′ = 0 ...(iii)


DR’s of PQ are α − p, β − q, γ − r.
and Q be a point on the line OP such that OP ⋅ OQ = p 2 ,
Since, PQ perpendicular on plane Eq. (iii), we get
show that the locus of the point Q is α − p β −q γ −r
= = =k (say)
p(lx + my + nz ) = x 2 + y 2 + z 2 . a′ b′ c′
Sol. Let P(α, β, γ) and Q ( x1, y1, z1 ) ∴ α = p + a′ k, β = q + b′ k, γ = r + c′ k
DR’s of OP are (α, β, γ ) and DR’s of OQ are ( x1, y1, z1 ). Putting the values of α, β and γ in Eq. (iii), we get
QO, Q and P are collinear. a′ (2 p + ka′ ) + b′ (2q + kb′ ) + c(2r + kc′ ) + 2d ′ = 0
α β γ
∴ = = =k (say) …(i) ⇒ 2(a′ p + b′ q + c′ r + d ′ ) = − k(a′ 2 + b′ 2 + c′ 2 ) ...(iv)
x1 y1 z1
Since, P(α, β, γ ) lies on plane (i), we get
Since, P(α, β, γ ) lie on the plane
aα + bβ + cγ + d = 0
lx + my + nz = p,
lα + mβ + nγ = p ⇒ a( p + ka′ ) + n(q + kb′ ) + c(r + kc′ ) + d = 0
Since, P(α , β, γ ) lie on the plane lx + my + nz = p, (ap + bq + cr + d )
∴ k=−
lα + mβ + nγ ± p (aa′ + bb′ + cc′ )
⇒ klx1 + kmy1 + knz1 = p [using Eq. (i)] …(ii) Putting the value of k in Eq. (iv), we get
Since, OP ⋅OQ = p 2 2(a′ p + b′ q + c′ r + d ′ )
∴ α +β +γ ⋅
2 2 2
x12 + y12 + z12 =p 2 (a′ 2 + b′ 2 + c′ 2 ) (ap + bq + cr + d )
=
aa′ + bb′ + cc′
k 2x12 + k 2y12 + k 2z12 ⋅ x12 + y12 + z12 = p 2
∴Locus of Q ( p, q, r ).
⇒ k( x12 + y12 + z12 ) = p 2 ...(iii)
i.e. equation reflection of plane (i) in plane (ii) is,
From Eqs. (ii) and (iii), we get 2(aa′ + bb′ + cc′ ) (a′ x + b′ y + c′ z + d ′ )
lx1 + my1 + nz1 1
= = (a′ 2 + b′ 2 + c′ 2 ) (ax + by + cz + d )
x12 + y12 + z12 p
⇒ p(lx1 + my1 + nz1 ) = ( x12 + y12 + z12 ) x y z
l Ex. 88 A point P moves on a plane + + =1. A plane
Hence, locus of Q a b c
⇒ p(lx + my + nz ) = x 2 + y 2 + z 2 through P and perpendicular to OP meets the coordinate
axes in A, B and C. If the planes through A, B and C parallel
l Ex. 87 Find the reflection of the plane to the planes x = 0, y =0 and z = 0 intersect in Q, then find
ax + by + cz + d = 0 in the plane a ′ x + b ′ y + c ′ z + d ′ = 0 the locus of Q.
Sol. Given planes are x y z
Sol. Given plane is + + =1 ...(i)
ax + by + cz + d = 0 ...(i) a b c
and a′ x + b ′ y + c ′ z + d ′ = 0 ...(ii) Let P (h, k, l ) be the point on plane.
Let P(α, β, γ ) be an arbitrary point in the plane (i) and Q ( p, q, r ) h k 1
∴ + + =1 ...(ii)
be the reflection of the point P in plane (ii). Locus of Q will be a b c
the required reflection of plane (i) in plane (ii), let L be the ⇒ OP = h 2 + k 2 + l 2
mid-point of PQ.
 h k l 
DC’s of OP =  , , 
 h2 + k 2 + l 2 h2 + k 2 + l 2 h2 + k 2 + l 2 
 
P ∴ Equation of the plane through P and normal to OP is,
hx ky lz
+ +
L h +k + l
2 2 2
h +k +l
2 2 2
h +k2 + l 2
2

Q
= h2 + k 2 + l 2
⇒ hx + ky + lz = h 2 + k 2 + l 2
 p + α q + β r + γ  h2 + k 2 + l 2 
Then, L , ,  ∴ A≡ , 0, 0
 2 2 2   h 
L lies in plane (ii), we get  h2 + k 2 + l 2 
 p + α  q + β  r + γ B ≡  0, , 0
∴ a′   + b′   + c′   + d′ = 0  k 
 2   2   2 
228 Textbook of Vector & 3D Geometry

 h2 + k 2 + l 2  Let the equation of one of the pair of opposite edges OA and


C ≡  0, 0,  BC be
 l 
y + z = 0, x + z = 0 ...(i)
Let Q(α, β, γ ), then
and x + y = 0 , x + y + z = 3a ...(ii)
h2 + k 2 + l 2
α= , A
h (0, 0, 0) P
h2 + k 2 + l 2 O
β= ,
k
h2 + k 2 + l 2
γ= ...(iii)
l C
1 1 1 h2 + k 2 + l 2 (0, 0, √3a) Q B
Now, 2 + 2 + 2 = 2
α β γ (h + k 2 + l 2 ) 2 Eqs. (i) and (ii) can be expressed in symmetrical form as
= 2
1 x − 0 y −0 z − 0
...(iv) = = ...(iii)
h + k2 +l 2 1 1 −1
From Eq. (iii), we get x − 0 y − 0 z − 3a
and = = ...(iv)
h2 + k 2 + l 2 1 −1 0
h=
α DR’s of OA and BC are (1, 1, − 1 ) and ( − 1, 1, 0 ).
h h + k2 +l 2
2
⇒ = Let PQ be the shortest distance between OA and BC having
a aα direction cosine (l , m, n ).
k h2 + k 2 + l 2 ∴ PQ is perpendicular to both OA and BC.
Similarly, =
b bβ l +m+n=0 ...(v)
l h2 + k 2 + l 2 l − m =0 ...(vi)
and =
c cγ On solving Eqs. (v) and (vi), we get
h2 + k 2 + l 2 h2 + k 2 + l 2 h2 + k 2 + l 2 O P A
+ +
aα bβ cγ
h k 1
= + + =1 [from Eq. (ii)]
a b c B
C Q
1 1 1 1
⇒ + + =
aα bβ cγ h 2 + k 2 + l 2 l m n
= = =k
1 1 1 1 1 2
= 2 + 2 + 2 [from Eq. (iv)]
α β γ Also, l 2 + m 2 + n 2 =1
∴Required equation of locus is 1
∴ k 2 + k 2 + 4k 2 = 1 ⇒ k =
1 1 1 1 1 1 6
+ + = + + .
ax by cz x 2 y 2 z 2 ∴ l =
1
= m and n =
2
6 6
l Ex. 89 Prove that the shortest distance between any two Shortest distance between OA and BC,
opposite edges of a tetrahedron formed by the planes i.e. PQ = Length of projection of OC and PQ
y + z =0, x + z = 0, x + y = 0, x + y + z = 3a is 2a. = | ( x 2 − x1 ) l + (y 2 − y1 ) m + (z 2 − z1 ) n |
1 1 2
Sol. Here, planes = 0⋅ + 0⋅ + 3a ⋅
6 6 6
y + z = 0, z + x = 0, x + y = 0 meet at O( 0, 0, 0 ).
Let the tetrahedron be OABC. = 2a
#L
Three Dimensional Coordinate System Exercise 1 :
Single Option Correct Type Questions
1. The xy-plane divides the line joining the points 8. The equation of the plane perpendicular to the line
( −1, 3, 4 ) (2, − 5, 6). x −1 y −2 z +1
, , and passing through the point (2, 3, 1),
(a) Internally in the ratio 2 : 3 1 −1 2
(b) externally in the ratio 2 : 3 is
(c) internally in the ratio 3 : 2 (a) r. ( $i + $j + 2 k$ ) = 1 (b) r. ( i$ − $j + 2 k$ ) = 1
(d) externally in the ratio 3 : 2 (c) r. ( $i − $j + 2 k$ ) = 7 (d) None of these
2. Ratio in which the zx-plane divides the join of (1, 2, 3) 9. The locus of a point which moves so that the difference
and (4, 2, 1).
of the squares of its distances from two given points is
(a) 1 : 1 internally (b) 1 : 1 externally
constant, is a
(c) 2 : 1 internally (d) 2 : 1 externally
(a) straight line (b) plane
3. If P (3, 2, − 4 ), Q (5, 4, − 6) and R (9, 8, − 10) are collinear, (c) sphere (d) None of these
then R divides PQ in the ratio 10. The position vectors of points a and b are $i − $j + 3k$ and
(a) 3 : 2 internally (b) 3 : 2 externally
(c) 2 : 1 internally (d) 2 : 1 externally 3$i + 3$j + 3k$ respectively. The equation of a plane is
r. (5$i + 2$j − 7 k$ ) + 9 = 0. The points a and b
4. A (3, 2, 0), B (5, 3, 2) and C ( −9, 6, − 3) are the vertices of a
(a) lie on the plane
triangle ABC. If the bisector of ∠ABC meets BC at D,
(b) are on the same side of the plane
then coordinates of D are (c) are on the opposite side of the plane
 19 57 17  19 57 17
(a)  , ,  (b)  − , ,  (d) None of the above
 8 16 16  8 16 16
 19 57 17 11. The vector equation of the plane through the point
(c)  , − ,  (d) None of these 2$i − $j − 4 k$ and parallel to the plane
8 16 16
r. ( 4 $i − 12$j − 3k$ ) − 7 = 0, is
5. A line passes through the points (6, − 7, − 1) and (2, − 3, 1).
(a) r. ( 4 $i − 12 $j − 3 k$ ) = 0 (b) r. ( 4 $i − 12 $j − 3 k$ ) = 32
The direction cosines of the line so directed that the
angle made by it with the positive direction of x-axis is (c) r. ( 4 $i − 12 $j − 3 k$ ) = 12 (d) None of these
acute, are 12. Let L 1 be the line r1 = 2 $i + $j − k$ + λ ( $i + 2k$ ) and let L 2
2 2 1 2 2 1
(a) , − , − (b) − , , be the another line r = 3$i + $j + µ ( i$ + $j − k$ ). Let π be
3 3 3 3 3 3 2
2 2 1 2 2 1 the plane which contains the line L 1 and is parallel to
(c) , − , (d) , ,
3 3 3 3 3 3 L 2 . The distance of the plane π from the origin is
2 1
6. If P is a point in space such that OP is inclined to OX at (a) (b)
7 7
45° and OY to 60° then OP is inclined to OZ at
(a) 75° (c) 6 (d) None of these
(b) 60° and 120° x −1 y −2 z −3
13. For the line = = , which one of the
(c) 75° and 105° 1 2 3
(d) 255° following is incorrect ?
7. l 1 , m1 , n 1 and l 2 , m 2 , n 2 are direction cosines of the two (a) it lies in the plane x − 2y + z = 0
x y z
lines inclined to each other at an angle θ, then the (b) it is same as line = =
1 2 3
direction cosines of the internal bisector of the angle
(c) it passes through (2, 3, 5)
between these lines are
(d) it is parallel to the plane x − 2y + z − 6 = 0
l1 + l 2 m1 + m2 n1 + n2 l + l 2 m1 + m2 n1 + n2
(a) , , (b) 1 , ,
θ θ θ θ θ θ 14. The value of m for which straight line
2 sin 2 sin 2 sin 2 cos 2 cos 2 cos
2 2 2 2 2 2 3x − 2y + z + 3 = 0 = 4 x − 3y + 4z + 1 is parallel to the
l1 − l 2 m1 − m2 n1 − n2 l1 − l 2 m1 − m2 n1 − n2 plane 2x − y + mz − 2 = 0 is
(c) , , (d) , ,
θ θ θ θ θ θ (a) −2 (b) 8
2 sin 2 sin 2 sin 2 cos 2 cos 2 cos
2 2 2 2 2 2 (c) −18 (d) 11
230 Textbook of Vector & 3D Geometry

15. The length of projection of the line segment joining the x + 6 y + 10 z + 14


22. The line = = is the hypotenuse of an
points (1, 0, −1) and ( −1, 2, 2) on the plane x + 3y − 5z = 6, 5 3 8
is equal to isosceles right angled triangle whose opposite vertex is
271 (7, 2, 4). Then which of the following is not the side of
(a) 2 (b)
53 the triangle ?
472 474 x −7 y −2 z − 4
(c) (d) (a) = =
31 35 2 −3 6
x −7 y −2 z − 4
16. The number of planes that are equidistant from four (b) = =
3 6 2
non-coplanar points is x −7 y −2 z − 4
(c) = =
(a) 3 (b) 4 3 5 −1
(c) 7 (d) 9 (d) None of these
17. In a three dimensional co-ordinate system, P, Q and R 23. Consider the following 3 lines in space
are images of a point A (a, b, c ) in the xy, yz and zx L1 : r = 3 i − j + 2 k + λ (2 i + 4 j − k )
planes, respectively. If G is the centroid of triangle PQR, L2 : r = i + j − 3 k + µ ( 4 i + 2 j + 4 k )
then area of triangle AOG is (O is the origin) L3 : r = 3 i + 2 j − 2 k + t (2 i + j + 2 k )
(a) 0 (b) a 2 + b 2 + c 2 Then, which one of the following pair(s) is/are in the
2 same plane ?
(c) (a 2 + b 2 + c 2 ) (d) None of these
3 (a) Only L1L2 (b) Only L2L3
(c) Only L3L1 (d) L1L2 and L2L3
18. A plane passing through (1, 1, 1) cuts positive direction
of coordinate axes at A, B and C, then the volume of 24. Let r = a + λl and r = b + µm be two lines in space,
tetrahedron OABC satisfies where a = 5i + j + 2k, b = − i + 7 j + 8k, l = − 4 i + j − k,
(a) V ≤
9
(b) V ≥
9 and m = 2i − 5 j − 7 k, then the position vector of a point
2 2 which lies on both of these lines, is
(c) V =
9
(d) None of these (a) i + 2 j + k
2 (b) 2i + j + k
y z (c) i + j + 2 k
19. If lines x = y = z and x = = and third line passing (d) non-existent as the lines are skew
2 3
through (1, 1, 1) form a triangle of area 6 units, then 25. L 1 and L 2 are two lines whose vector equations are
point of intersection of third line with second line will L1 : r = λ [(cos θ + 3 ) i + ( 2 sin θ ) j + (cos θ − 3 ) k ]
be and L2 : r = µ (a i + bj + ck )
(a) (1, 2, 3) (b) (2, 4, 6) where, λ and µ are scalars and α is the acute angle
 4 8 12 between L 1 and L 2 . If the angle α is independent of θ,
(c)  , ,  (d) None of these
3 3 3  then the value of α is
π π
20. The point of intersection of the line passing through (a) (b)
6 4
(0, 0, 1) and intersecting the lines x + 2y + z = 1,
π π
− x + y − 2z = 2 and x + y = 2, x + z = 2 with xy plane is (c) (d)
3 2
5 1 
(a)  , − , 0 (b) (1, 1, 0)
3 3  26. The vector equations of two lines L 1 and L 2 are
2 1   5 1  respectively,
(c)  , − , 0 (d)  − , , 0
3 3   3 3  r = 17 i − 9 j + 9 k + λ (3i + j + 5k )
21. Two systems of rectangular axes have the same origin. If and r = 15i − 8 j − k + µ ( 4 i + 3 j)
a plane cuts them at distance a, b, c and a′, b ′ , c ′ from the I. L 1 and L 2 are skew lines.
origin, then : II. (11, −11, −1) is the point of intersection of L 1 and L 2 .
1 1 1 1 1 1
(a) + + + + + =0 III. (−11, 11, 1) is the point of intersection of L 1 and L 2 .
a2 b2 c 2 a′ 2 b ′ 2 c ′ 2
 3 
1 1
− 2
1 1 1
− 2 + 2 − 2 − 2 =0
1 IV. cos −1   is the acute angle between, L 1 and L 2 .
(b) 2
a b c a′ b′ c′  35 
1 1 1 1 1 1 Then, which of the following is true ?
(c) 2 + 2 + 2 − 2 − 2 − 2 =0
a b c a′ b′ c′ (a) II and IV (b) I and IV
1 1 1 1 1 1 (c) Only IV (d) III and IV
(d) 2 − 2 + 2 − 2 + 2 − 2 =0
a b c a′ b′ c′
Chap 03 Three Dimensional Coordinate System 231

27. Consider three vectors p = i + j + k, q = 2i + 4 j − k and 35. Equation of the line which passes through the point
r = i + j + 3k. If p, q and r denotes the position vector of with position vector (2, 1, 0) and perpendicular to the
three non-collinear points, then the equation of the plane containing the vectors i + j and j + k is
plane containing these points is (a) r = (2, 1, 0 ) + t (1, − 1, 1 )
(a) 2 x − 3y + 1 = 0 (b) x − 3y + 2z = 0 (b) r = (2, 1, 0 ) + t ( −1, 1, 1 )
(c) 3 x − y + z − 3 = 0 (d) 3 x − y − 2 = 0 (c) r = (2, 1, 0 ) + t (1, 1, − 1 )
(d) r = (2, 1, 0 ) + t (1, 1, 1 )
28. The intercept made by the plane r. n = q on the x-axis is Where, t is a parameter.
q i. n
(a) (b) 36. Which of the following planes are parallel but not
i. n q
identical ?
q
(c) ( i. n ) q (d)
| n| P1 : 4 x − 2y + 6z = 3
P2 : 4 x − 2y − 2z = 6
29. If the distance between the planes P3 : − 6x + 3y − 9z = 5
8x + 12y − 14z = 2 and 4 x + 6y − 7z = 2 P4 : 2x − y − z = 3
1 (a) P2 and P3 (b) P2 and P4
can be expressed in the form , where N is natural,
N (c) P1 and P3 (d) P1 and P4
N ( N + 1)
then the value of is 37. A parallelopiped is formed by planes drawn through the
2 points (1, 2, 3) and (9, 8, 5) parallel to the coordinate
(a) 4950 (b) 5050
planes, then which of the following is not the length of
(c) 5150 (d) 5151
an edge of this rectangular parallelopiped ?
30. A plane passes through the points P ( 4, 0, 0) and Q (0, 0, 4 ) (a) 2 (b) 4
and is parallel to the Y -axis. The distance of the plane (c) 6 (d) 8
from the origin is 38. vector equation of the plane
(a) 2 (b) 4 r = i − j + λ ( i + j + k ) + µ ( i − 2 j + 3k ) in the scalar dot
(c) 2 (d) 2 2 product form is
31. If from the point P ( f , g , h ) perpendiculars PL and PM be (a) r. (5 i − 2 j + 3 k ) = 7
(b) r. (5 i + 2 j − 3 k ) = 7
drawn to yz and zx-planes, then the equation to the
(c) r.(5 i − 2 j − 3 k ) = 7
plane OLM is (d) r.(5 i + 2 j + 3 k ) = 7
x y z x y z
(a) + − =0 (b) + + =0
f g h f g h 39. The vector equations of the two lines L 1 and L 2 are
x y z x y z given by L 1 : r = (2i + 9 j + 13k ) + λ ( i + 2 j + 3k )
(c) − + = 0 (d) − + + = 0
f g h f g h and L2 : r = ( −3 i + 7 j + pk ) + µ ( − i + 2 j − 3 k ).
Then, the lines L1 and L2 are
32. The plane XOZ divides the join of (1, − 1, 5) and (2, 3, 4) in (a) skew lines for all p ∈ R
the ratio λ : 1, then λ is (b) intersecting for all p ∈ R and the point of intersection is
1 (−1, 3, 4)
(a) −3 (b) −
3 (c) intersecting lines for p = − 2
(c) 3 (d)
1 (d) intersecting for all real p ∈ R
3
40. Consider the plane
33. A variable plane forms a tetrahedron of constant volume ( x , y, z ) = (0, 1, 1) + λ (1, − 1, 1) + µ (2, − 1. 0). The distance
64 K 3 with the coordinate planes and the origin, then of this plane from the origin is
locus of the centroid of the tetrahedron is 1 3
(a) (b)
(a) x 3 + y 3 + z 3 = 6k 3 (b) xyz = 6k 3 3 2
(c) x 2 + y 2 + z 2 = 4k 2 (d) x −2 + y −2 + z −2 = 4k −2 (c)
3
(d)
2
2 3
34. Let ABCD be a tetrahedron such that the edges AB, AC
x − 2 y − 9 z − 13
and AD are mutually perpendicular. Let the area of 41. The value of a for which the lines = =
∆ABC , ∆ACD and ∆ADB be 3, 4 and 5 sq units, 1 2 3
respectively. Then, the area of the ∆BCD, is x −a y −7 z +2
and = = intersect, is
(a) 5 2 (b) 5 −1 2 −3
5 (a) −5 (b) −2
(c) 5 / 2 (d) (c) 5 (d) −3
2
232 Textbook of Vector & 3D Geometry

x −1 y −2 z −3 49. A straight line is given by r = (1 + t ) i + 3t j + (1 − t ) k,


42. For the line = = , which one of the
1 2 3 where t ∈ R. If this line lies in the plane x + y + cz = d ,
following is incorrect ? then the value of (c + d ) is
(a) It lies in the plane x − 2y + z = 0. (a) −1 (b) 1
x y z
(b) It is same as line = = . (c) 7 (d) 9
1 2 3
(c) It passes through (2, 3, 5). 50. The distance of the point ( −1, − 5, − 10) from the point of
(d) It is parallel to the plane x − 2y + z − 6 = 0. x −2 y +1 z −2
intersection of the line = = and the
43. Given planes P1 : cy + bz = x ; 2 4 12
plane x − y + z = 5 is
P2 : az + cx = y (a) 2 11 (b) 126
P3 : bx + ay = z (c) 13 (d) 14
P1 , P2 and P3 pass through one line, if 51. P( p) and Q( q) are the position vector of two fixed points
(a) a 2 + b 2 + c 2 = ab + bc + ca and R( r) is the position vector of a variable point. If R
(b) a 2 + b 2 + c 2 + 2abc = 1 moves such that ( r − p) × ( r − q ) = 0, then the locus of R is
(c) a 2 + b 2 + c 2 = 1 (a) a plane containing the origin O and parallel to two
(d) a + b + c + 2ab + 2bc + 2ca + 2abc = 1
2 2 2 non-collinear vector OP and OQ.
(b) the surface of a sphere described on PQ as its diameter.
x −2 y −3 z − 4 x −1 y − 4 z −5
44. The lines = = and = = (c) a line passing through the points P and Q.
1 1 −k k 2 1 (d) a set of lines parallel to the line PQ.
are coplanar, if
(a) k = 0 or − 1 (b) k = 1 or − 1
52. The three vectors i + j, j + k, k + i taken two at a time
(c) k = 0 or − 3 (d) k = 3 or − 3 form three planes. The three unit vectors drawn
x −2 y +1 z −1 perpendicular to these three planes form a
45. The line = = intersects the curve parallelopiped of volume
3 2 −1 1
xy = c 2 , in xy-plane, if c is equal to (a)
3
(b) 4
1 3 4
(a) ± 1 (b) ± (c) 3 (d)
3 4 3 3
(c) ± 5 (d) None of these
53. The orthogonal projection A′ of the point A with
46. The line which contains all points ( x , y, z ) which are of position vector (1, 2, 3) on the plane 3x − y + 4z = 0 is
the form ( x , y, z ) = (2, − 2, 5) + λ (1, − 3, 2) intersects the  1 5 
(a) ( −1, 3, − 1 ) (b)  − , , 1
plane 2x − 3y + 4z = 163 at P and intersects the YZ-plane  2 2 
at Q. If the distance PQ is a b, where a, b ∈ N and a > 3, 1 5 
(c)  , − , − 1 (d) (6., − 7, − 5 )
then (a + b ) is equal to 2 2 
(a) 23 (b) 95
(c) 27 (d) None of these 54. The equation of the line passing through (1, 1, 1) and
perpendicular to the line of intersection of the planes
47. If the three planes r. n 1 = p 1 , r. n 2 = p 2 and r. n 3 = p 3 x + 2y − 4z = 0 and 2x − y + 2z = 0 is
have a common line of intersection, then x −1 1 −y z −1 x −1 1 −y z −1
(a) = = (b) = =
p 1 ( n 2 × n 3 ) + p 2 ( n 3 × n 1 ) + p 3 ( n 1 × n 2 ) is 5 1 2 −5 1 2
equal to x −1 1 −y z −1 x −1 y −1 z −1
(c) = = (d) = =
(a) 1 (b) 2 0 −10 −5 −10 0 −5
(c) 0 (d) −1
55. A variable plane at a distance of 1 unit from the origin
48. The equation of the plane which passes through the line cuts the axes at A, B and C. If the centroid D ( x , y, z ) of
of intersection of the planes r. n 1 = q 1 , r. n 2 = q 2 and is 1 1 1
∆ABC satisfies the relation 2 + 2 + 2 = K, then the
parallel to the line of intersection of the planes x y z
r. n 3 = q 3 and r. n 4 = q 4 , is
value of K is
(a) [ n 2 n 3 n 4 ] ( r. n1 − q1 ) = [ n1 n 3 n 4 ] ( r. n 2 − q 2 )
(a) 3 (b) 1
(b) [ n1 n 2 n 3 ] ( r. n 4 − q 4 ) = [ n 4 n 3 n1 ] ( r. n 2 − q 2 )
1
(c) [ n 4 n 3 n1 ] ( r. n 4 − q 4 ) = [ n1 n 2 n 3 ] ( r. n 2 − q 2 ) (c) (d) 9
3
(d) None of the above
Chap 03 Three Dimensional Coordinate System 233

56. The angle between the lines AB and CD, where 64. Through the point P (h, k , l ) a plane is drawn at right
A = (0, 0, 0), B = (1, 1, 1), C = ( −1, − 1, − 1) and D = (0, 1, 0) is angles to OP to meet co-ordinate axes at A, B and C. If
given by OP = p then the area of the ∆ABC is
1 4 p5 p5
(a) cos θ = (b) cos θ = (a) (b)
3 3 2 2hkl hkl
1 1 p3 p3
(c) cos θ = (d) cos θ = (c) (d)
5 2 2 2hkl hkl
57. The shortest distance of a point (1, 2, − 3) from a plane 65. The volume of the tetrahedron included between the
making intercepts 1, 2 and 3 units on position X , Y and plane 3x + 4y − 5z − 60 = 0 and the co-ordinate planes is
Z-axes respectively, is (a) 60 (b) 600
(a) 2 (b) 0 (c) 720 (d) 400
13 12
(c) (d) 66. The angle between the lines whose direction cosines are
12 7
given by the equations l 2 + m 2 − n 2 = 0, l + m + n = 0 is
58. A tetrahedron has vertices O (0, 0, 0), A (1, 2, 1), B (2, 1, 3) (a) cos−1 (2 3 ) (b) cos−1 3
and C ( −1, 1, 2). Then the angle between the faces OAB π π
and ABC will be (c) (d)
3 2
 19  17
(a) cos−1   (b) cos−1   67. The distance between the line
 35  31
(c) 30° (d) 90°
r = 2$i − 2$j + 3k$ + λ ( $i − $j + 4 k$ ) and the plane
r ⋅ ( $i + 5$j + k$ ) = 5 is
59. The direction ratios of line I 1 passing through P (1, 3, 4 )
10 10
x −1 y −2 z −3 (a) (b)
and perpendicular to line I 2 = = 3 3 3
2 3 4 10 10
(c) (d)
(where, I 1 and I 2 are coplanar) is 9 3
(a) 14, 8, 1 (b) −14, 8, −1
68. The Cartesian equation of the plane perpendicular to the
(c) 14, −8, −1 (d) −14, − 8, 1
x −1 y −3 z − 4
line = = and passing through the origin
60. Equation of the plane through three points A, B and C 2 −1 2
with position vectors −6i + 3 j + 2k, 3i − 2 j + 4 k and is
5i + 7 j + 3k is equal to (a) 2 x − y + 2z − 7 = 0 (b) 2 x + y + 2z = 0
(a) r.( i − j + 7 k ) + 23 = 0 (b) r.( i + j + 7 k ) = 23 (c) 2 x − y + 2z = 0 (d) 2 x − y − z = 0
(c) r.( i + j − 7 k ) + 23 = 0 (d) r.( i − j − 7 k ) = 23
69. Let P (3, 2, 6) be a point in space and Q be a point on the
61. OABC is a tetrahedron. The position vectors of A, B and line r = ( $i − $j + 2k$ ) + µ ( −3$i + $j + 5k$ ). Then the value of
C are i, i + j and j + k, respectively. O is origin. The µ for which the vector PQ is parallel to the plane
height of the tetrahedron (taking plane ABC as base) is x − 4y + 3z = 1 is
1 1 1 1
(a) (b) (a) (b) −
2 2 4 4
1 1 1
(c) (d) None of these (c) (d) −
2 2 8 8
62. The plane x − y − z = 4 is rotated through an angle 90° 70. A plane makes intercepts OA, OB and OC whose
about its line of intersection with the plane measurements are a, b and c on the OX , OY and OZ axes.
x + y + 2z = 4. Then the equation of the plane in its new The area of triangle ABC is
position is 1 1
(a) (ab + bc + ca ) (b) abc (a + b + c )
(a) x + y + 4z = 20 (b) x + 5y + 4z = 20 2 2
(c) x + y − 4z = 20 (d) 5 x + y + 4z = 20 1 2 2 1
(c) (a b + b c + c a ) (d) (a + b + c ) 2
2 2 2 2 1/ 2

63. Let A xy , A yz , A zx be the area of the projection of a 2 2

plane area A on the xy, yz, zx plane respectively 71. The radius of the circle in which the sphere
Then A 2 = x 2 + y 2 + z 2 + 2x − 2y − 4z − 19 = 0 is cut by the plane
2
(a) Axy + Ayz
2
+ Azx
2 2
(b) Axy + Ayz
2
+ Azx
2 x + 2y + 2z + 7 = 0 is
(a) 2 (b) 3
(c) Axy + Ayz + Axz (d) Axy + Ayz + Azx (c) 4 (d) 1
234 Textbook of Vector & 3D Geometry

72. Let a = i$ + $j and b = 2$i − k$ , then the point of 80. In a three dimensional coordinate system P, Q and R are
intersection of the lines r × a = b × a and r × b = a × b is images of a point A(a, b, c ) in the XY they YZ and the ZX
(a) (3, − 1, 1 ) (b) (3, 1, − 1 ) planes respectively. If G is the centroid of triangle PQR,
(c) ( −3, 1, 1 ) (d) ( −3, − 1, − 1 ) then area of triangle AOG is (O is the origin)
73. The co-ordinates of the point P on the line (a) 0 (b) a 2 + b 2 + c 2
r = ( i$ + $j + k$ ) + λ ( − $i + $j − k$ ) which is nearest to the 2
(c) (a 2 + b 2 + c 2 ) (d) None of these
3
origin is
 2 4 2  2 4 2 81. A plane 2x + 3y + 5z = 1 has a point P which is at
(a)  , ,  (b)  − , − , 
 3 3 3  3 3 3 minimum distance from line joining
 2 4 2
(c)  , , −  (d) None of these
A(1, 0, − 3), B(1, − 5, 7 ), then distance AP is equal to
 3 3 3 (a) 3 5 (b) 2 5
(c) 4 5 (d) None of these
74. The 3-dimensional vectors v 1 , v 2 , v 3 satisfying
v 1 . v 1 = 4, v 1 . v 2 = − 2, v 1 . v 3 = 6, v 2 . v 2 = 2, v 2 . v 3 = − 5, 82. The locus of a point which moves in such a way that its
v 3 . v 3 = 29, then v 3 may be x y z
distance from the line = = is twice the distance
(a) −3 $i + 2 $j ± 4 k$ (b) 3 $i − 2 $j ± 4 k$ 1 1 −1
(c) −2 i$ + 3 $j ± 4 k$ (d) 2 i$ + 3 $j ± 4 k$ from the plane x + y + z = 0 is
(a) x 2 + y 2 + z 2 − 5 x − 3y − 3z = 0
75. The points $i − $j + 3k$ and 3$i + 3$j + 3k$ are equidistant
(b) x 2 + y 2 + z 2 + 5 x + 3y + 3z = 0
from the plane r.(5i$ + 2$j − 7 k$ ) + 9 = 0, then they are
(c) x 2 + y 2 + z 2 − 5 xy − 3yz − 3zx = 0
(a) on the same sides of the plane
(d) x 2 + y 2 + z 2 + 5 xy + 3yz + 3zx = 0
(b) parallel of the plane
(c) on the opposite sides of the plane 83. A cube C = {( x , y, z ) | 0 ≤ x , y, z ≤ 1} is cut by a sharp knife
(d) None of the above
along the plane x = y, y = z, z = x . If no piece is moved
76. A, B, C , D are four points in space. Then, until all three cuts are made, the number of pieces is
AC 2 + BD 2 + AD 2 + BC 2 ≥ (a) 6 (b) 7
1 1 (c) 8 (d) 27
(a) AB 2 + CD 2 (b) −
AB 2 CD 2 84. A ray of light is sent through the point P(1, 2, 3) and is
1 1
(c) − (d) None of these reflected on the XY -plane. If the reflected ray passes
CD 2 AB 2
through the point Q(3, 2, 5), then the equation of the
77. If | x 1 | > | y 1 | + | z 1 |, | y 2 | > | x 2 | + | z 2 |, | z 3 | > | x 3 | + | y 3 |, reflected ray is
then x 1 i$ + y 1 $j + z 1 k$ , x 2 $i + y 2 $j + z 2 k$ and x −3 y −2 z −5 x −3 y −2 z −5
(a) = = (b) = =
1 0 1 1 0 −4
x $i + y $j + z k$ are
3 3 3
x −3 y −2 z −5 x −1 y −2 z −3
(a) perpendicular (b) collinear (c) = = (d) = =
1 0 4 1 0 4
(c) coplanar (d) non-coplanar
85. A plane cutting the axes in P, Q, R passes through
78. The position vector of the point of intersection of three
planes r. n 1 = q 1 , r. n 2 = q 2 , r. n 3 = q 3 , where n 1 , n 2 (α − β, β − γ, γ − α ). If O is the origin, then locus of
and n 3 are non-coplanar vectors, is centre of sphere OPQR is
1 (a) αx + βy + γz = 4
(a) [q 3 ( n1 × n 2 ) + q1 ( n 2 × n 3 ) + q 2 ( n 3 × n1 )] (b) (α − β ) x + (β − γ )y + ( γ − α ) z = 0
[ n1 n 2 n 3 ]
1 (c) (α − β )yz + (β − γ )zx + ( γ − α ) xy = 2 xyz
(b) [q1 ( n1 × n 2 ) + q 2 ( n 2 × n 3 ) + q 3 ( n 3 × n1 )]  1 1 1
[ n1 n 2 n 3 ] (d)  2 + 2 + 2 ( x 2 + y 2 + z 2 ) = xyz
1 α β γ 
(c) – [q1 ( n1 × n 2 ) + q 2 ( n 2 × n 3 ) + q 3 ( n 3 × n1 )]
[ n 3 n1 n 2 ]
86. The shortest distance between any two opposite edges
(d) None of the above of the tetrahedron formed by planes x + y = 0, y + z = 0,
79. A pentagon is formed by cutting a triangular corner z + x = 0, x + y + z = a is constant, equal to
from a rectangular piece of paper. The five sides of the 2a
(a) 2a (b)
pentagon have length 13, 19, 20, 25 and 31 not 6
necessarily in that order. The area of the pentagon is a 2a
(c) (d)
(a) 459 sq units (b) 600 sq units 6 3
(c) 680 sq units (d) 745 sq units
Chap 03 Three Dimensional Coordinate System 235

87. The angle between the pair of planes represented by (a) the direction cosines of three mutually perpendicular
equation 2x − 2y + 4z + 6xz + 2yz + 3xy = 0 is
2 2 2 lines
(b) the direction ratios of three mutually perpendicular lines
 1  4
(a) cos−1   (b) cos−1   which are not direction cosines
 3  21
(c) the direction cosines of three lines which need not be
 4  7 
(c) cos−1   (d) cos−1   perpendicular
 9  84 
(d) the direction ratios but not the direction cosines of three
88. Let ( p , q , r ) be a point on the plane 2x + 2y + z = 6, then lines which need not be perpendicular

the least value of p 2 + q 2 + r 2 is equal to 93. If ABCD is a tetrahedron such that each ∆ABC, ∆ABD
(a) 4 (b) 5 (c) 6 (d) 8 and ∆ACD has a right angle at A. If ar( ∆ABC ) = k 1 ,
ar ( ∆ABD ) = k 2 , ar ( ∆BCD ) = k 3 , then ar ( ∆ACD ) is
89. The four lines drawing from the vertices of any
k1k2k3
tetrahedron to the centroid of the opposite faces meet in (a) k12 + k22 + k32 (b)
k1 + k2 + k3
a point whose distance from each vertex is ‘k’ times the
distance from each vertex to the opposite face, where k (c) | k12 + k22 − k32 | (d) | k22 − k12 − k32 |
is
1 1 3 5 94. In a regular tetrahedron, if the distance between the
(a) (b) (c) (d) mid-points of opposite edges is unity, its volume is
3 2 4 4
1 1
90. The shortest distance from (1, 1, 1) to the line of (a) (b)
3 2
intersection of the pair of planes xy + yz + zx + y 2 = 0 is 1 1
(c) (d)
8 2 1 2 2 6 2
(a) (b) (c) (d)
3 3 3 3 95. A variable plane makes intercepts on X , Y and Z-axes
91. The shortest distance between the two lines L 1 : x = k 1 ; and it makes a tetrahedron of volume 64 cu. u. The locus
y = k 2 and L 2 : x = k 3 ; y = k 4 is equal to of foot of perpendicular from origin on this plane is
(a) ( x 2 + y 2 + z 2 ) 3 = 384 xyz
(a) k12 + k22 − k32 + k42 (b) k1k3 + k2k4

(c) (k1 + k3 ) 2 + (k2 + k4 ) 2 (d) (k1 − k3 ) 2 + (k2 − k4 ) 2 (b) xyz = 681


2
 1 1 1
l 1 m1 n1  p1 q1 r1  (c) ( x + y + z )  + +  = 16
 x y z
92. A = l 2 m2 n 2  and B = p 2
 
q2 r 2  , where
 (d) xyz ( x + y + z ) = 81
l 3 m3 n 3  p 3 q3 r 3 
96. If P, Q, R, S are four coplanar points on the sides AB, BC,
p i , q i , ri are the cofactors of the elements l i , mi , n i for
AP BQ CR DS
i = 1, 2, 3. If (l 1 , m1 , n 1 ), (l 2 , m 2 , n 2 ) and (l 3 , m 3 , n 3 ) are CD, DA of a skew quadrilateral, then ⋅ ⋅ ⋅
the direction cosines of three mutually perpendicular PB QC RD SA
lines, then ( p 1 , q 1 , r1 ), ( p 2 , q 2 , r 2 ) and ( p 3 , q 3 , r 3 ) are equals
(a) 1 (b) − 1 (c) 3 (d) − 3

#L
Three Dimensional Coordinate System Exercise 2 :
More than One Correct Option Type Questions
97. Given the equations of the line 3x − y + z + 1 = 0 and (c) Equation of the plane through (2, 1, 4) and perpendicular
to the given lines is 2 x − y + z − 7 = 0.
5x + y + 3z = 0. Then, which of the following is correct ?
(d) Equation of the plane through (2, 1, 4) and perpendicular
(a) Symmetrical form of the equations of line is
1 5 to the given lines is x + y − 2z + 5 = 0.
y − z+
x
= 8 = 8. 98. Consider the family of planes x + y + z = c where c is a
2 −1 1 parameter intersecting the coordinate axes at P, Q and R
(b) Symmetrical form of the equations of line is and α, β and γ are the angles made by each member of
1 5
x+ y − this family with positive x , y and z-axes. Which of the
8 = 8 = z .
following interpretations hold good for this family?
1 −1 −2
236 Textbook of Vector & 3D Geometry

(a) Each member of this family is equally inclined with A B C


(a) = = is true for the line to be perpendicular to the
coordinate axes. a b c
(b) sin 2 α + sin 2 β + sin 2 γ = 1 plane
(c) cos2 α + cos2 β + cos2 γ = 2 (b) A (a + 3 ) + B (b − 1 ) + C (c − 2 ) = 0
(c) 2aA + 3bB + 4cC = 0
(d) For c = 3 area of the ∆PQR is 3 3 sq units.
(d) Aa + Bb + Cc = 0
99. Equation of the line through the point (1, 1, 1) and x −2 y +1 z −1
intersecting the lines 2x − y − z − 2 = 0 = x + y + z − 1 105. The line = = intersects the curve
3 2 −1
and x − y − z − 3 = 0 = 2x + 4y − z − 4 x 2 + y 2 = r 2 , z = 0 then
(a) x − 1 = 0, 7 x + 17y − 3z − 134 = 0
(b) x − 1 = 0, 9 x + 15y − 5z − 19 = 0 (a) Equation of the plane through (0, 0, 0) perpendicular to
y −1 z −1 the given line is 3 x + 2y − z = 0
(c) x − 1 = 0, = (b) r = 26 (c) r = 6
1 3
(d) x − 2y + 2z − 1 = 0, 9 x + 15y − 5z − 19 = 0 (d) r = 7

100. Through a point P (h, k , l ) a plane is drawn at right 106. A vector equally inclined to the vectors $i − $j + k$ and
angles to OP to meet the coordinate axes in A, B and C. If i$ + $j − k$ then the plane containing them is
OP = p , A xy is area of projection of ∆ABC on xy-plane, i$ + $j − k$
A yz is area of projection of ∆ABC on yz-plane, then (a) (b) $j − k$ (c) 2 $i (d) $i
3
 p5   p 5  Axy  l  Axy h 
(a) ∆ =   (b) ∆ = (c) = (d) = 107. Consider the plane through (2, 3, − 1) and at right angles
hkl  2hkl  Ayz h  Ayz l 
to the vector 3i$ − 4 $j + 7 k$ from the origin is
101. Which of the following statements is/are correct? (a) The equation of the plane through the given point is
(a) If n.a = 0, n. b = 0 and n. c = 0 for some non-zero vector n, 3 x − 4y + 7z + 13 = 0
then [a b c ] = 0 1
(b) perpendicular distance of plane from origin
(b) There exist a vector making angles 30° and 45° with x-axis 74
and Y -axis. 13
(c) Locus of point for which x = 3 and y = 4 is a line parallel to (c) perpendicular distance of plane from origin
74
the Z-axis whose distance from the Z-axis is 5 21
(d) The vertices of regular tetrahedron are O, A, B, C where (d) perpendicular distance of plane from origin
74
‘O’ is the origin. The vector OA + OB + OC is
perpendicular to the plane ABC 108. A plane passes through a fixed point (a, b, c ) and cuts the
102. Which of the following is/are correct about a axes in A, B, C . The locus of a point equidistant from
tetrahedron ? origin, A, B and C must be
a b c
(a) Centroid of a tetrahedron lies on lines joining any vertex (a) ayz + bzx + cxy = 2 xyz (b) + + =1
to the centroid opposite face x y z
(b) Centroid of a tetrahedron lies on the lines joining the mid a b c a b c
(c) + + =2 (d) + + = 3
point of the opposite faces x y z x y z
(c) Distance of centroid from all the vertices are equal
109. Let A be vector parallel to line of intersection of planes
(d) None of the above
P1 and P2 . Plane P1 is parallel to the vectors 2$j + 3k$ and
103. A variable plane cutting coordinate axes in A, B, C is at a 4 $j − 3k$ and that P is parallel to $j – k$ and 3$i + 3$j , then
2
constant distance from the origin. Then the locus of
the angle between vector A and a given vector
centroid of the ∆ABC is
2$i + $j − 2k$ is
(a) x −2 + y −2 + z −2 = 16 (b) x −2 + y −2 + z −2 = 9
π π π 3π
−1 (a) (b) (c) (d)
1 1 1 1 2 4 6 4
(c)  2 + 2 + 2 = 0 (d) X + Y = 0
9  x y z  110. Consider the lines x = y = z and the line
x − x1 2x + y + z − 1 = 0 = 3x + y + 2z − 2, then
104. Equation of any plane containing the line = 1
a (a) the shortest distance between the two lines is
2
y − y1 z − z1
= is A ( x − x 1 ) + B (y − y 1 ) + C (z − z 1 ) = 0, (b) the shortest distance between the two lines is 2
b c
(c) plane containing 2nd line parallel to 1st line is y − z + 1 = 0
then pick correct alternatives
3
(d) the shortest distance between the two lines
2
Chap 03 Three Dimensional Coordinate System 237

111. If p 1 , p 2 , p 3 denote the perpendicular distances of the 119. Let a plane pass through origin and is parallel to the line
plane 2x − 3y + 4z + 2 = 0 from the parallel planes. x −1 y +3 z +1
(a) p1 + 8 p 2 − p 3 = 0 (b) p 3 = 16 p 2
= = such that distance between plane
2 −1 −2
(c) 8 p 2 = p1 (d) p1 + 2 p 2 + 3 p 3 = 29
5
and the line is . Then, equation of the plane is ........ .
112. A line segment has length 63 and direction ratios are 3, 3
−2,6. The components of the line vector are (a) x − 2y + 2z = 0 (b) x − 2y − 2z = 0
(a) −27, 18, 54 (b) 27, −18, −54 (c) 2 x + 2y + z = 0 (d) x + y + z = 0
(c) 27, −18, 54 (d) −27, 18, −54
120. OABC is a regular tetrahedron of side unity, then
x −2 y −3 z − 4 x −1 y − 4 z −5
113. The lines = = and = = (a) the length of perpendicular from one vertex to opposite
1 1 −k k 2 1 face is 2 / 3
are coplanar if (b) the perpendicular distance from mid-point of OA to the
(a) k = 0 (b) k = − 1 plane ABC is 1 / 6
(c) k = 2 (d) k = − 3
(c) the angle between two skew edges is π / 2
114. The points A ( 4, 5, 10), B (2, 3, 4 ) and C (1, 2, − 1) are three (d) the distance of centroid of the tetrahedron form any
vertices of a parallelogram ABCD, then vertex is 3 / 8
(a) Vector equation of AB is 2 i + 3 j + 4 k + λ ( i + j + 3 k )
x −2 y −3 z − 4 121. If OABC is a tetrahedron such that
(b) Cartesian equation of BC is = =
1 1 5 OA 2 + BC 2 = OB 2 + CA 2 = OC 2 + AB 2 , then
(c) Coordinates of D are (3, 4, 5) (a) OA ⊥ BC (b) OB ⊥ AC
(d) ABCD is a rectangle (c) OC ⊥ AB (d) AB ⊥ AC
115. The line x = y = z meets the plane x + y + z = 1 at the 122. If the line
x y z
= = intersects the line
point P and the sphere x + y + z = 1 at the points R
2 2 2 1 2 3
1
and S, then 3β 2 x + 3(1 − 2α )y + z = 3 = − {6α 2 x + 3(1 − 2β )y + 2z }
2 2
(a) PR + PS = 2 (b) PR × PS = then point (α, β, 1) lie on the plane
3
(c) PR = PS (d) PR + PS = RS (a) 2 x − y + z = 4 (b) x + y − z = 2
(c) x − 2y = 0 (d) 2 x − y = 0
116. A rod of length 2 units whose one end is (1, 0, − 1) and
other end touches the plane x − 2y + 2z + 4 = 0, then 123. Let PM be the perpendicular from the point P(1, 2, 3) to
(a) The rod sweeps the figure whose volume is π cubic units. XY plane. If OP makes an angle θ with the positive
(b) The area of the region which the rod traces on the plane is direction of Z-axis and OM makes an angle φ with the
2π. positive direction of X -axis, where O is the origin and θ
(c) The length of projection of the rod on the plane is 3 units. and φ are acute angles, then
(d) The centre of the region which the rod traces on the plane 5 2
 2 2 −5  (a) tanθ = (b) sin θ sin φ =
is  , ,  . 3 14
3 3 3  1
(c) tan φ = 2 (d) cos θ cos φ =
117. Consider the planes P1 : 2x + y + z + 4 = 0 14
P2 : y − z + 4 = 0 and P3 : 3x + 2y + z + 8 = 0 124. A variable plane which remains at a constant distance P
Let L 1 , L 2 , L 3 be the lines of intersection of the planes from the origin (0) cuts the coordinate axes in A, B, C
P2 and P3 , P3 and P1 , and P1 and P2 respectively. Then, (a) Locus of centroid of tetrahedron OABC is
(a) at least two of the lines L1, L2 and L3 are non-parallel 16
(b) at least two of the lines L1, L2 and L3 are parallel x 2y 2 + y 2z 2 + z 2x 2 = 2 x 2y 2z 2
p
(c) the three planes intersect in a line
(b) Locus of centroid of tetrahedron OABC is
(d) the three planes form a triangular prism
4
x 2y 2 + y 2z 2 + z 2x 2 = 2 x 2y 2z 2
118. The volume of a right triangular prism ABCA 1 B1C 1 is p
equal to 3. Find the coordinates of the vertex A 1 , if the (c) Parametric equation of the centroid of the tetrahedron is
coordinates of the base vertices of the prism are P P P 
of the form  sec α sec β, sec α cosec β, cosec α  ,
A(1, 0, 1), B (2, 0, 0) and C (0, 1, 0). 4 4 4 
(a) ( − 2, 0, 2 ) (b) ( 0, − 2, 0 ) α , β ∈ ( 0, 2 π ) − { π / 2, π , 3 π / 2 }
(c) (0, 2, 0) (d) (2, 2, 2) (d) None of the above
238 Textbook of Vector & 3D Geometry

#L
Three Dimensional Coordinate System Exercise 3 :
Statement I and II Type Questions
Directions (Q. Nos. 125 to 138) For the following x − 4 y +5 z −1 x −2 y +1 z
= = = =
n
130. Given lines and
questions, choose the correct answers from the codes (a), 2 4 −3 1 3 2
(b), (c) and (d) defined as follows :
Statement I The lines intersect.
(a) Statement I is true, Statement II is also true; Statement II
Statement II They are not parallel.
is the correct explanation of Statement I
(b) Statement I is true, Statement II is also true; Statement II 131. Consider the lines L 1 : r = a + λ b and L 2 : r = b + µ a,
is not the correct explanation of Statement I where a and b are non-zero and non-collinear vectors.
(c) Statement I is true, Statement II is false Statement I L 1 and L 2 are coplanar and the plane
(d) Statement I is false, Statement II is true containing these lines passes through origin.
125. Statement I Let A ( $i + $j + k$ ) and B ( $i − $j + k$ ) be two Statement II (a − b ).(b × a ) = 0 and the plane containing
points, P (2$i + 3$j + k$ ) lies exterior to the sphere with AB L 1 and L 2 is [r a b ] = 0 which passes through origin.
as one of its diameters. 132. Statement I P is a point (a, b, c ). Let A, B, C be the
Statement II If A and B are any two points and P is a images of P in yz, zx and xy planes respectively, then
point in space such that PA. PB > 0, then the point P equation of the plane passing through the points
x y z
lies exterior to the sphere with AB as one of its A, B and C is + + = 1
diameters. a b c
126. Statement I If r = x$i + y$j + zk$ , then equation Statement II The image of a point P in a plane is the
foot of the perpendicular drawn from P on the plane.
r × (2$i − $j + 3k$ ) = 3$i + k$ represents a straight line.
133. Statement I The locus of a point which is equidistant
Statement II If r = x$i + y$j + zk$ , then equation from the points whose position vectors are 3$i − 2$j + 5k$
r × ( $i + 2$j − 3k$ ) = 2$i − $j represents a straight line. and i$ + 2$j − k$ is r ( $i − 2$j + 3k$ ) = 8.
127. Statement I Let θ be the angle between the line Statement II The locus of a point which is equidistant
x −2 y −1 z +2 from the points whose position vectors are a and b is
= = and the plane x + y − z = 5.
2 −3 −2 r − a + b
 .(a − b ) = 0
 1   2 
Then, θ = sin −1  
 51  134. Statement I If the vectors a and c are non-collinear
Statement II Angle between a straight line and a plane then the lines r = 6a − c + λ (2c − a ) and
is the complement of angle between the line and normal r = a − c + µ (a + 3c) are coplanar.
to the plane. Statement II There exist λ and µ such that the two
values of r in Statement I becomes same.
128. Statement I A point on the straight line 2x + 3y − 4z = 5
x −1 y z +1
and 3x − 2y + 4z = 7 can be determined by taking x = k 135. Statement I The lines = = and
and then solving the two equations for y and z, where k 1 −1 1
x −2 y +1 z
is any real number. = = are coplanar and equation of the plane
1 2 3
Statement II If c ′ ≠ kc , then the straight line
containing them is 5x + 2y − 3z − 8 = 0
ax + by + cz + d = 0, Kax + Kby + c ′ z + d ′ = 0, does not
x −2 y+1 z
intersect the plane z = α, where α is any real number. Statement II The line = = is
1 2 3
x −1 y −3 z −1
129. Let the line L having equation = = , perpendicular to the plane 3x + 6y + 9z − 8 = 0 and
2 5 3 parallel to the plane x + y − z = 0
intersects the plane P, having equation x − y + z = 5 at
the point A.
136. The equation of two straight lines are
x −1 y +3 z −2 x −2 y −1 z +3
Statement I Equation of the line L′ through the point = = and = = .
2 1 −3 1 −3 2
A, lying in the plane P and having minimum inclination
with line L is 8x + y − 72 − 4 = 0 = x − y + z − 5 Statement I The given lines are coplanar.
Statement II Line L′ must be the projection of the line Statement II The equations 2x 1 − y 1 = 1, x 1 + 3y 1 = 4
L in the plane P. and 3x 1 + 2y 1 = 5 are consistent.
Chap 03 Three Dimensional Coordinate System 239

137. Statement I A plane passes through the point 138. Statement I At least two of the lines L 1 , L 2 and L 3 are
A (2, 1, − 3). If distance of this plane from origin is non-parallel.
maximum, then its equation is 2x + y − 3z = 14.
Statement II The three planes do not have a common
Statement II If the plane passing through the point point.
A (a ) is at maximum distance from origin, then normal
to the plane is vector a.

#L
Three Dimensional Coordinate System Exercise 4 :
Passage Based Questions
Passage I Passage III
(Q. Nos. 139 to 142) (Q. Nos. 145 to 148)
Let A (1, 2, 3), B (0, 0, 1) and C (−1, 1, 1) are the vertices of Consider a triangular pyramid ABCD the position vector of
∆ ABC. whose angular points are A(3, 0, 1), B ( −1, 4, 1), C (5, 2, 3) and
139. The equation of internal angle bisector through A to side D(0, − 5, 4). Let G be the point of intersection of the
BC is medians of the ∆BCD.
(a) r = i$ + 2 $j + 3 k$ + µ (3 $i + 2 $j + 3 k$ ) 145. The length of the vector AG is
(b) r = i$ + 2 $j + 3 k$ + µ (3 $i + 4 $j + 3 k$ ) 51 51 59
(a) 17 (b) (c) (d)
(c) r = $i + 2 $j + 3 k$ + µ (3 $i + 3 $j + 2 k$ ) 3 9 4
(d) r = $i + 2 $j + 3 k$ + µ (3 $i + 3 $j + 4 k$ ) 146. Area of the ∆ABC (in sq units) is
(a) 24 (b) 8 6
140. The equation of altitude through B to side AC is
(c) 4 6 (d) None of these
(a) r = k + t (7 $i − 10 $j + 2 k$ )
(b) r = k + t ( −7 $i + 10 $j + 2 k$ ) 147. The length of the perpendicular from the vertex D on
(c) r = k + t (7 $i − 10 $j − 2 k$ ) the opposite face is
14 2
(d) r = k + t (7 $i + 10 $j + 2 k$ ) (a) (b)
6 6
141. The equation of median through C to side AB is 3
(c) (d) None of these
(a) r = − $i + $j + k$ + p (3 $i − 2 k$ ) 6
(b) r = − i$ + $j + k$ + p (3 $i + 2 k$ ) 148. Equation of the plane ABC is
(c) r = − $i + $j + k$ + p ( −3 $i + 2 k$ ) (a) x + y + 2z = 5 (b) x − y − 2z = 1
(d) r = − $i + $j + k$ + p (3 $i + 2 $j) (c) 2 x + y − 2z = 4 (d) x + y − 2z = 1

142. The area of ( ∆ABC ) is equal to Passage IV


9 17 17 7 (Q. Nos. 149 to 151)
(a) (b) (c) (d)
2 2 2 2 A line L1 passing through a point with position vector
Passage II p = i + 2 j + 3k and parallel a = i + 2 j + 3k, Another line L2
passing through a point with position vector = 2i + 3 j + 3k
(Q. Nos. 143 to 144)
and parallel to b = 3i + j + 2k .
Consider a plane x + y − z =1 and the point A (1, 2, − 3).
A line L has the equation x = 1 + 3r, y = 2 − r, z = 3 + 4r 149. Equation of plane equidistant from line L 1 and L 2 is
(a) r$ . ( i − 7 j − 5 k ) = 3 (b) r$ . ( i + 7 j + 5 k ) = 3
143. The coordinate of a point B of line L, such that AB is (c) r$ . ( i − 7 j − 5 k ) = 9 (d) r$ . ( i + 7 j − 5 k ) = 9
parallel to the plane, is
(a) (10, − 1, 15 ) (b) ( −5, 4, − 5 ) 150. Equation of a line passing through the point (2, − 3, 2) and
(c) ( 4, 1, 7 ) (d) ( −8, 5, − 9 ) equally inclined to the line L 1 and L 2 may be equal to
144. Equation of the plane containing the line L and the point x −2 y −3 z −2 x −2
(a) = = (b) =y + 3 =z −2
A has the equation 2 −1 1 −2
(a) x − 3y + 5 = 0 (b) x + 3y − 7 = 0 x −2 y + 3 z −5 x + 2 y + 3 z −2
(c) = = (d) = =
(c) 3 x − y − 1 = 0 (d) 3 x + y − 5 = 0 −4 3 2 4 3 −5
240 Textbook of Vector & 3D Geometry

151. The minimum distance of origin from the plane passing 158. A point P moves in the space such that 3PA = 2PB, then
through the point with position vector p and the locus of P is
perpendicular to the line L 2 is (a) x 2 + y 2 + z 2 + 28 x − 12y + 10z − 247 = 0
7
(a) 14 (b) (b) x 2 + y 2 + z 2 − 28 x + 12y + 10z − 247 = 0
14
11 (c) x 2 + y 2 + z 2 + 28 x − 12y − 10z + 247 = 0
(c) (d) None of these
14 (d) x 2 + y 2 + z 2 − 28 x + 12y − 10z + 247 = 0

Passage V 159. Coordinates of the point P which divides the join of A


and B in the ratio 2 : 3 internally are
(Q. Nos. 152 to 154)
 33 2 
For positive l, m and n, if the planes x = ny + mz, y = lz + nx, (a)  , − , 9 (b) ( 4, 0, 7 )
5 5 
z = mz + ly intersect in a straight line, then  32 12 17
(c)  , − ,  (d) (20, 0, 35 )
152. l , m and n satisfy the equation 5 5 5
(a) l 2 + m 2 + n 2 = 2 (b) l 2 + m 2 + n 2 + 2lmn = 1 160. Equation of a line L, perpendicular to the line AB is
(c) l 2 + m 2 + n 2 = 1 (d) None of these x+2 y −2 z −3
(a) = =
15 −5 10
153. cos −1 l + cos −1 m + cos −1 n is equal to x −2 y +2 z +3
(a) 90° (b) 50° (b) = =
3 13 2
(c) 180° (d) None of these x+2 y −2 z −3
(c) = =
x y z 3 13 2
154. The equation of the straight line is = = , where the x −2 y +2 z +3
a b c (d) = =
ordered traid (a, b, c ) is 15 −5 10
(a) 1 − l 2 , 1 − m 2 , 1 − n 2
Passage VIII
(b) l , m and n (Q. Nos. 161 to 163)
1 m n
(c) , and The vector equation of a plane is a relation satisfied by
1 − l 2 1 − m2 1 − n2
position vectors of all the points on the plane. If P is a
(d) None of the above plane and n$ is a unit vector through origin which is
Passage VI perpendicular to the plane P then vector equation of the
(Q. Nos. 155 to 157) plane must be r. n$ = d where d represents perpendicular
distance of plane p from origin.
If a = 6 i + 7 j + 7k, b = 3$i + 2$j − 2k$ , P (1, 2, 3)
$ $ $
161. If A is a point vector a then perpendicular distance of A
155. The position vector of L, the foot of the perpendicular from the plane r. n$ = d must be
from P on the line r = a + λb is ^
(a) | d + an |
^
(b) | d − an |
(a) 6 i$ + 7 $j + 7 k$ (b) 3 i$ + 2 $j − 2 k$
(c) | a | − d | (d) | d − a$ |
(c) 3 i$ + 5 $j + 9 k$ (d) 9 i$ + 9 $j + 5 k$
162. If b be the foot of perpendicular from A to the plane
156. The image of the point P in the line r = a + λb is r. n$ = d then b must be
(a) (11, 12, 11 ) (b) (5, 2, − 7 ) (a) a + (d − a. n$ ) n$ (b) a − (d − a. n$ ) n$
(c) (5, 8, 15 ) (d) (17, 16, 7 ) (c) a + a. n$ (d) a − a. n$
157. If A is the point with position vector a then area of the 163. The position vector of the image of the point a in the
triangle ∆PLA is sq. units is equal to plane r. n$ = d must be (d ≠ 0)
7 17 (a) −a. n$ (b) a − 2(d − a. n$ ) n$
(a) 3 6 (b)
2 (c) a + 2(d − a. n$ ) n$ (d) a + d( − a. n$ )
7
(c) 17 (d)
2 Passage IX
(Q. Nos. 164 to 166)
Passage VII A circle is the locus of a point in a plane such that its
(Q. Nos. 158 to 160) distance from a fixed point in the plane is constant.
A( −2, 2, 3) and B (13, − 3, 13) and L is a line through A. Anologously, a sphere is the locus of a point in space such
that its distance from a fixed point in space is constant.
Chap 03 Three Dimensional Coordinate System 241

The fixed point is called the centre and the constant Passage XI
distance is called the radius of the circle/sphere. (Q. Nos. 169 to 171)
In anology with the equation of the circle | z − c | = a, the The line of greatest slope on an inclined plane P1 is that
equation of a sphere of radius a is | r − c | = a, where c is the line in the plane which is perpendicular to the line of
position vector of the centre and r is the position vector of intersection of plane P1 and a horizontal plane P2 .
any point on the surface of the sphere. In Cartesian system,
169. Assuming the plane 4 x − 3y + 7z = 0 to be horizontal, the
the equation of the sphere, with centre at ( − g, − f , − h) is
direction cosines of the line of greatest slope in the
x 2 + y 2 + z 2 + 2gx + 2 fy + 2hz + c = 0 and its radius is
plane 2x + y − 5z = 0 are
f 2
+ g 2 + h 2 − c.  3
(a) 
−1 1   3 1 −1
, ,  (b)  , , 
 11 11 11   11 11 11 
164. Radius of the sphere, with (2, − 3, 4 ) and ( − 5, 6, − 7 ) as
 −3 1 1   1 3 −1
extremities of a diameter, is (c)  , ,  (d)  , , 
 11 11 11   11 11 11 
251 251
(a) (b)
2 3 170. The equation of a line of greatest slope can be
x y z x y z
251 251 (a) = = (b) = =
(c) (d) 3 1 −1 3 −1 1
4 5
x y z x y z
165. The centre of the sphere (c) = = (d) = =
−3 1 1 1 3 −1
( x − 4 ) ( x + 4 ) + (y − 3) (y + 3 ) + z 2 = 0 is
(a) (4, 3, 0) (b) ( − 4, − 3, 0 ) 171. The coordinates of a point on the plane 2x + y − 5z = 0 is
(c) (0, 0, 0) (d) None of these 11 units away from the line of intersection of the given
two planes are
166. Equation of the sphere having centre at (3, 6, − 4 ) and
(a) (3, 1, − 1 ) (b) ( − 3, 1, 1 )
touching the plane r ⋅ (2i$ − 2$j − k$ ) = 10, is (c) (3, − 1, 1 ) (d) (1, 3, − 1 )
( x − 3) 2 + (y − 6) 2 + (z + 4 ) 2 = k 2 , where k is equal to
(a) 3 (b) 4
Passage XII
(c) 6 (d) 17 (Q. Nos. 172 to 174)
Given four points A(2, 1, 0), B (1, 0, 1), C (3, 0, 1) and
Passage X D(0, 0, 2). Point D lies on a line L orthogonal to the plane
(Q. Nos. 167 to 168) determined by the points A, B and C.
Let A (2, 3, 5), B ( − 1, 3, 2), C ( λ, 5, µ ) are the vertices of a 172. The equation of the plane ABC is
triangle and its median through A (i.e.) AD is equally (a) x + y + z − 3 = 0 (b) y + z − 1 = 0
inclined to the coordinates axes. (c) x + z − 1 = 0 (d) 2y + z − 1 = 0
On the basis of the above information answer the following 173. The equation of the line L is
167. The value of 2λ − µ is equal to (a) r = 2 k$ + λ ( $i + k$ )
(a) 13 (b) 4 (b) r = 2 k$ + λ (2 $j + k$ )
(c) 3 (d) None of these (c) r = 2 k$ + λ ( $j + k$ )
168. Projection of AB on BC is (d) None of the above
8 3 −8 3 174. The perpendicular distance of D from the plane ABC is
(a) (b)
11 11 (a) 2 (b) 1/2
(c) − 48 (d) 48 (c) 2 (d) 1 / 2
242 Textbook of Vector & 3D Geometry

#L
Three Dimensional Coordinate System Exercise 5 :
Matching Type Questions
175. Consider the following four pairs of line in Column I Match the statement of Column I with values of Column
and match them with one or more entries in Column II. II.

Column I Column II Column I Column II


(A) L1: x = 1 + t , y = t , z = 2 − 5t (p) non-coplanar (A) L1 , L2 and L3 are concurrent, if (p) k = − 9, − 6 , 5
L2: r = (2, 1, − 3) + λ (2, 2, − 10) lines 5
(B) L : x − 1 = y − 3 = z − 2 (q) lines lie in a (B) One of L1 , L2 and L3 is parallel to (q) k = − 6 , − 9
1
2 2 −1 unique plane atleast one of the other two, if 5
x − 2 y− 6 z + 2 (C) L1 , L2 and L3 form a triangle, if (r) k = 5
L2: = =
1 −1 −1 6
(C) L1: x = −6t , y = 1 + 9t , z = −3t (r) infinite planes (D) L1 , L2 and L3 do not form a (s) k = 5
L2: x = 1 + 2s, y = 4 − 3s, z = s containing both triangle, if (t) k = 0
the lines
(D) L : x = y − 1 = z − 2 (s) lines are not 179. A variable plane cuts the x, y and z-axes at the points,
1
1 2 3 intersecting at a A, B and C , respectively such that the volume of the
x − 3 y− 2 z −1 unique point tetrahedron OABC remain constant equal to 32 cu units
L2: = =
−4 −3 2 and O is the origin of the coordinate system.

176. P(0, 3, − 2), Q(3, 7, − 1) and R(1, − 3, − 1) are 3 given points. Column I Column II

Let L 1 be the line passing through P and Q and L 2 be the (A) The locus of the centroid of the (p) xyz = 24
tetrahedron is
line through R and parallel to the vector V = $i + k$ .
(B) The locus of the point equidistant (q) (x 2 + y2 + z2 )
Column I Column II from O , A , B and C is = 192 xyz
(A) Perpendicular distance of P from L1 (p) 7 3 (C) The locus of the foot of (r) xyz = 3
perpendicular from origin to the
(B) Shortest distance between L1 and L2 (q) 2
plane is
(C) Area of the ∆PQR (r) 6
(D) If PA, PB and PC are mutually (s) (x 2 + y2 + z2 )3
(D) Distance from (0, 0, 0) to the plane (s) 19 perpendicular, then the locus of P is = 1536 xyz
PQR 147
180. Match the statements of Column I with values of
177. Match the statements of Column I with values of Column II.
Column II.
Column I Column II
Column I Column II
(A) The area of the triangle whose vertices (p) 0
(A) If the line − 1 = y + 1 = z + 1 lies in
x (p) 6
sin −1 are (0, 0, 0) (3, 4 , 7) and (5, 2, 6) is
1 −2 λ 25 (B) The smallest radius of the sphere (q) 70
the plane 3x − 2 y + 5z = 0, then λ is
passing through (1, 0, 0), (0, 1, 0) and 3
equal to
(0, 0, 1) is
(B) If (3, λ, µ) is a point on the line (q) − 7
(C) The value(s) of λ for which the (r) 2
2x + y + z − 3 = 0 = x − 2 y + z − 1,then 5 triangle with vertices
λ + µ is equal to 3
A (6., 10, 10) B (1, 0, − 5) and
(C) The angle between the line x = y = z (r) −3 C (6, − 10, λ ) will be a right angled
and the plane 4 x − 3 y + 5z = 2 is triangle (right angled at A) is/are
(D) The angle between the planes (s) 8 (D) d is the perpendicular distance from (s) 3 65
cos−1
x + y + z = 0 and 3x − 4 y + 5z = 0 75 x −1 y−1 z 2
(1, 3, 4) to = = , then
−1 1 1
178. Consider the lines given by L 1 : x + 3y − 5 = 0, value of
d
2 3
L 2 : 3x − ky − 1 = 0 and L 3 : 5x + 2y − 12 = 0.
Chap 03 Three Dimensional Coordinate System 243

181. Match the statements of Column I with values of Column II. Consider the cube
Column I Column II
(A) Angle between any two solid diagonal (p) cos−1 2
6
(B) Angle between a solid diagonal and a plane (q)  1
cos−1  + 
 2
(C) Angle between plane diagonals of adjacent faces (r) cos−1 1
3
(D) The values of | a × b| (s) 1
2

#L
Three Dimensional Coordinate System Exercise 6 :
Single Integer Answer Type Questions
182. In a tetrahedron OABC, if OA = $i, OB = $i + $j and 192. If the centroid of the tetrahedron OABC where A, B, C
OC = i$ + 2$j + k$ , if shortest distance between edges OA are the points (a, 2, 3), (1, b, 2) and (2, 1, c ) be (1, 2, 3), then
the point (a, b, c ) is at distance 5 λ from origin, then λ
and BC is m, then 2m is equal to … (Where O is the must be equal to ...........
origin)
193. If the circumcentre of the triangle whose vertices are
183. A parallelopiped is formed by planes drawn through the (3, 2, − 5), ( −3, 8, − 5) and ( −3, 2, 1) is ( −1, λ , − 3) the integer
points (2, 4, 5) and (5, 9, 7) parallel to the coordinate λ must be equal to ...........
planes. The length of the diagonal of the parallelopiped
is ……… 194. If P1 P2 is perpendicular to P2 P3 , then the value of k is,
where P1 (k , 1, − 1), P2 (2k , 0, 2) and P3 (2 + 2k , k , 1) is ...........
184. If the perpendicular distance of the point (6, 5, 8) from
the Y -axis is 5λ units, then λ is equal to ……… 195. Let the equation of the plane containing line
x − y − z − 4 = 0 = x + y + 2z − 4 and parallel to the line
185. If the shortest distance between the lines of intersection of the planes 2x + 3y + z = 1 and
x −3 y −8 z −3 x +3 y +7 z −6 x + 3y + 2z = 2 be x + Ay + Bz + C = 0. Then the values
= = and = = is λ 30
3 −1 1 −3 2 2 of | A + B + C − 4| is ............ .
units, then the value of λ is ………
196. Let P(a, b, c ) be any point on the plane 3x + 2y + z = 7,
186. If the planes x − cy − bz = 0, cx − y + az = 0 and then find the least value of 2(a 2 + b 2 + c 2 ).
bx + ay − z = 0 pass through a line then the values of
197. The plane denoted by P1 : 4 x + 7y + 4z + 81 = 0 is rotated
a 2 + b 2 + c 2 + 2abc is ...........
through a right angle about its line of intersection with
187. If xz-plane divide the join of point (2, 3, 4 ) and (1, − 1, 5) in the plane P2 : 5x + 3y + 10z = 25. If the plane in its new
the ratio λ : 1, then the integer λ should be equal to position be denoted by P, and the distance of this plane
k 
188. If the triangle ABC whose vertices are from the origin is d, then the value of   (where [.]
A ( −1, 1, 1), B (1, − 1, 1) and C (1, 1, − 1) is projected on 2
xy-plane, then the area of the projected triangles is .......... represents greatest integer less than or equal to k) is ......
189. The equation of a plane which bisects the line joining 198. The distance of the point P( −2, 3, − 4 ) from the line
(1, 5, 7) and (−3, 1, −1) is x + y + 2z = λ, then λ must x + 2 2y + 3 3z + 4
= = measured parallel to the plane
be ........... 3 4 5
4 x + 12y − 3z + 1 = 0 is d, then find the value of (2d − 8).
190. The shortest distance between origin and a point on the
is ...........
space curve x = 2 sin t , y = 2 cos t , z = 3t is ...........
199. The position vectors of the four angular points of a
191. The plane 2x − 2y + z + 12 = 0 touches the surface tetrahedron OABC are (0, 0, 0), (0, 0, 2), (0, 4, 0) and (6, 0, 0),
x 2 + y 2 + z 2 − 2x − 4y + 2z − 3 = 0 only at point respectively. A point P inside the tetrahedron is at the
( −1, λ , − 2). The value of λ must be same distance ‘r’ from the four plane faces of the
tetrahedron. Then, the value of 9r is .............. .
244 Textbook of Vector & 3D Geometry

200. Value of λ do the planes x − y + z + 1 = 0, denotes the volume of tetrahedron OABC and V 2 that of
λx + 3y + 2z − 3 = 0, 3x + λy + z − 2 = 0 form a triangular the parallelepiped with OG 1 ,OG 2 and OG 3 as three
prism must be concurrent edges, then the value of 4V1 / V 2 is (where O
201. If the lattice point P( x , y, z ); x , y, z > 0 and x , y, z ∈ I with is the origin)
least value of z such that the ‘P’ lies on the planes 205. A variable plane which remains at a constant distance p
7 x + 6y + 2z = 272 and x − y + z = 16, then the value of from the origin cuts the coordinate axes in A, B, C . The
( x + y + z − 42) is equal to locus of the centroid of the tetrahedron OABC is
202. If the line x = y = z intersect the line k
x 2 y 2 + y 2 z 2 + z 2 x 2 = 2 x 2 y 2 z 2 , then 5 2k is
x sin A + y sin B + z sin C − 2d 2 = 0 p
= x sin 2A + y sin 2B + z sin C − d 2 , where A, B, C are the
206. If (l 1 , m1 , n 1 ); (l 2 , m 2 , n 2 ) are D.C’s of two lines, then
A B C
internal angles of a triangle and sin sin sin = k , (l 1m 2 − l 2 m1 ) 2 + (m1n 2 − m 2 n 1 ) 2
2 2 2
then the value of 64k is equal to + (n 1l 2 − n 2 l 1 ) 2 + (l 1l 2 + m1m 2 + n 1n 2 ) 2 =
203. The number of real values of k for which the lines 207. If the coordinates ( x , y, z ) of the point S which is
x y −1 z x −k y −k z −2
= = and = = are coplanar, is equidistant from the points O (0, 0, 0), A (n 5 , 0, 0)
1 k −1 2k 3k − 1 k
B (0, n 4 , 0), C (0, 0, n ) obey the relation 2( x + y + z ) + 1 = 0.
204. Let G 1 , G 2 and G 3 be the centroids of the triangular If n ∈ Z , then | n | = ________ (| ⋅ | is the modulus
faces OBC, OCA and OAB of a tetrahedron OABC. If V1 function).

Three Dimensional Coordinate System Exercise 7 :


Subjective Type Questions
208. Find the angle between the lines whose direction 213. Vertices B and C of ∆ABC lie along the
cosines has the relation l + m + n = 0 and x +2 y −1 z −0
line = = . Find the area of the triangle
2l 2 + 2m 2 − n 2 = 0. 2 1 4
given that A has coordinates (1, − 1, 2) and line segment
209. Prove that the two lines whose direction cosines are BC has length 5.
connected by the two relations al + bm + cn = 0 and
x y z
ul 2 + vm 2 + wn 2 = 0 are perpendicular if 214. A point P moves on the plane + + = 1 which is
a b c
a 2 (v + w ) + b 2 (w + u ) + c 2 (u + v ) = 0 fixed. The plane through P perpendicular to OP meets
the axes in A, B and C. The planes through A, B, C
a2 b2 c 2
and parallel if + + = 0. parallel to yz, zx , xy planes intersect in Q. Prove that if
u v w the axis bc rectangular, then the locus of Q is
x +2 y +1 z −3 1 1 1 1 1 1
210. Find the point on the line = = at a + 2 + 2 = + +
3 2 2 x 2
y z ax by cz
distance of 3 2 from the point (1, 2, 3).
215. Prove that the distance of the point of intersection of the
211. A line passes through (2, − 1, 3) and is perpendicular to x −2 y +1 z −2
line = = and the plane x − y + z = 5
the lines r⋅ ( $i + $j − k$ ) + λ (2$i − 2$j + k$ ) and 3 4 12
from the point ( −1, − 5, − 10) is 13.
r = (2i$ − $j − 3k$ ) + µ( $i + 2$j + 2k$ ) obtain its equation.
216. Find the equation of the plane through the intersection
212. Find the equations of the two lines through the origin of the planes x + 3y + 6 = 0 and 3x − y − 4z = 0, whose
x −3 y −3 z π
which intersect the line = = at angle of perpendicular distance from the origin is unity.
2 1 1 3
each. 217. Find the equation of the image of the plane
x − 2y + 2z − 3 = 0 in the plane x + y + z − 1 = 0.
Chap 03 Three Dimensional Coordinate System 245

#L
Three Dimensional Coordinate System Exercise 8 :
Questions Asked in Previous Years Exam
(i) JEE Advanced & IIT JEE  7 7 5
(a)  , ,  (b) ( −1, − 1, 0 )
 3 3 3
218. Consider a pyramid OPQRS located in the first octant  7 7 8
(c) (1 , 1 , 1 ) (d)  , , 
( x ≥ 0, y ≥ 0, z ≥ 0) with O as origin and OP and OR along  9 9 9
the X-axis and the Y-axis, respectively. The base OPQR of
the pyramid is a square with OP = 3. The point S is
223. Perpendicular are drawn from points on the line
x +2 y +1 z
directly above the mid-point T of diagonal OQ such that = = to the plane x + y + z = 3. The feet of
2 −1 3
TS = 3. Then,
perpendiculars lie on the line
[More than One Correct Type Question, 2016 Adv.]
[Single Option Correct Type Question, 2013 Adv.]
π x y −1 z −2 x y −1 z −2
(a) the acute angle between OQ and OS is (a) = = (b) = =
3 5 8 −13 2 3 −5
(b) the equation of the plane containing the ∆OQS is x y −1 z −2 x y −1 z −2
(c) = = (d) = =
x −y = 0 4 3 −7 2 −7 5
(c) the length of the perpendicular from P to the plane x −1 y z +3
3 224. Consider the lines L 1 : = = ,
containing the ∆OQS is 2 −1 1
2
x − 4 y +3 z +3
(d) the perpendicular distance from O to the straight line L2 : = = and the planes
1 1 2
15
containing RS is P1 : 7 x + y + 2z = 3, P2 : 3x + 5y − 6z = 4. Let
2
ax + by + cz = d the equation of the plane passing
219. Let P be the image of the point (3, 1, 7) with respect to through the point of intersection of lines L 1 and L 2 and
the plane x − y + z = 3. Then, the equation of the plane perpendicular to planes P1 and P2 .
passing through P and containing the straight line Match List I with List II and select the correct answer
x y z using the code given below the lists.
= = is
1 2 1 [Single Option Correct Type Question, 2013 Adv.]
[Single Option Correct Type Question, 2016 Adv.]
List I List II
(a) x + y − 3z = 0 (b) 3 x + z = 0
(c) x − 4y + 7z = 0 (d) 2 x − y = 0 P. a= 1. 13
Q. b= 2. –3
220. From a point P( λ , λ , λ ), perpendiculars PQ and PR are
R. c= 3. 1
drawn respectively on the lines y = x , z = 1 and
S. d= 4. –2
y = − x , z = −1. If P is such that ∠QPR is a right angle,
then the possible value(s) of λ is (are) Codes
[Single Option Correct Type Question, 2014 Adv.] P Q R S P Q R S
(a) 2 (b) 1 (c) −1 (d) − 2 (a) 3 2 4 1 (b) 1 3 4 2
(c) 3 2 1 4 (d) 2 4 1 3
y z y z
221. Two lines L 1 : x = 5, = and L 2 : x = α, = x −1 y +1 z
3 − α −2 −1 2 − α 225. If the straight lines = = and
2 k 2
are coplanar. Then, α can take value(s) x +1 y +1 z
[More than One Correct Type Question, 2013 Adv.]
= = are coplanar, then the plane(s)
5 2 k
(a) 1 (b) 2
containing these two lines is/are
(c) 3 (d) 4
[More than One Correct Type Question, IIT-JEE 2012]
222. A line l passing through the origin is perpendicular to (a) y + 2 z = − 1 (b) y + z = − 1
the lines [More than One Correct Type Question, 2013 Adv.] (c) y − z = − 1 (d) y − 2 z = − 1

l1 : (3 + t ) $i + ( −1 + 2t ) $j + ( 4 + 2t ) k$ , − ∞ < t < ∞ 226. If the distance between the plane A x − 2y + z = d and


l : (3 + 2s ) $i + (3 + 2s ) $j + (2 + s ) k$ , − ∞ < s < ∞ x −1 y −2 z −3
2 the plane containing the lines = = and
2 3 4
Then, the coordinate(s) of the point(s) on l 2 at a x −2 y −3 z − 4
distance of 17 from the point of intersection of l and = = is 6, then d is equal to….
l 1 is (are) 3 4 5
[Single Option Correct Type Question, IIT-JEE 2010]
246 Textbook of Vector & 3D Geometry

Passage 230. Consider three planes


(Q. Nos. 227-229) P1 : x − y + z = 1 , P2 : x + y − z = − 1
Read the following passage and answer the questions. and P3 : x − 3y + 3z = 2
Consider the lines Let L 1 , L 2 , L 3 be the lines of intersection of the planes
x +1 y +2 z +1 P2 and P3 , P3 and P1 , P1 and P2 , respectively.
L1 : = =
3 1 2 Statement I Atleast two of the lines L 1 , L 2 and L 3 are
x −2 y +2 z −3 non-parallel.
L2 : = = Statement II The three planes do not have a common
1 2 3
point. [Assertion and Reason Type Question, IIT-JEE 2008]
[Passage Type Question, IIT-JEE 2008]

227. The distance of the point (1, 1, 1) from the plane passing 231. Consider the planes
through the point ( −1, − 2, − 1) and whose normal is 3x − 6y − 2z = 15 and 2x + y − 2z = 5.
perpendicular to both the lines L 1 and L 2 , is Statement I The parametric equations of the line of
(a) 2 / 75 unit (b) 7 / 75 unit intersection of the given planes are x = 3 + 14t ,
(c) 13 / 75 units (d) 23 / 75 units y = 1 + 2t , z = 15 t .
^ ^ ^
228. The shortest distance between L 1 and L 2 is Statement II The vectors 14 i + 2 j + 15 k is parallel to
(a) 0 unit (b) 17 / 3 units the line of intersection of the given planes.
[Assertion and Reason Type Question, IIT-JEE 2007]
(c) 41 /5 3 units (d) 17 /5 3 units
232. Consider the following linear equations
229. The unit vector perpendicular to both L 1 and L 2 is ax + by + cz = 0, bx + cy + az = 0, cx + ay + bz = 0
− $i + 7 $j + 7 k
$ − $i − 7 $j + 5 k$ [Matching Type Question, IIT-JEE 2007]
(a) (b)
99 5 3
− i + 7 $j + 5 k$
$ 7 i − 7 $j − k$
$ Column I Column II
(c) (d)
5 3 99 A. a + b + c ≠ 0 and p. The equations represent
a2 + b2 + c2 = ab + bc + ca planes meeting only at
n Directions (Q. Nos. 230-231) For the following questions, a single point
choose the correct answer from the codes (a), (b), (c) and B. a + b + c = 0 and q. The equations represent the
(d) defined as follows. line x = y = z
a2 + b2 + c2 ≠ ab + bc + ca
(a) Statement I is true, Statement II is also true; Statement II
is the correct explanation of Statement I C. a + b + c ≠ 0 and r. The equations represent
(b) Statement I is true, Statement II is also true; Statement II a + b + c ≠ ab + bc + ca
2 2 2 identical planes
is not the correct explanation of Statement I
D. a + b + c = 0 and r. The equations represent the
(c) Statement I is true; Statement II is false whole of the
a2 + b2 + c2 = ab + bc + ca
(d) Statement I is false; Statement II is true three-dimensional space

(ii) JEE Main and AIEEE


233. If the image of the point P(1, − 2, 3) in the plane 235. The distance of the point (1, − 5, 9) from the plane
2x + 3y − 4z + 22 = 0 measured parallel to the line x − y + z = 5 measured along the line x = y = z is
x y z (a) 3 10 (b) 10 3 [2016 JEE Main]
= = is Q, then PQ is equal to 10 20
1 4 5 [2017 JEE Main] (c) (d)
(a) 3 5 ` (b) 2 42 3 3
(c) 42 (d) 6 5 x −3 y +2 z + 4
236. If the line, = = lies in the plane,
234. The distance of the point (1, 3, −7 ) from the 2 −1 3
plane passing through the point (1, − 1, − 1) lx + my − z = 9, then l 2 + m 2 is equal to [2016 JEE Main]
having normal perpendicular to both the lines (a) 26 (b) 18 (c) 5 (d) 2
x −1 y +2 z − 4 x −2 y +1 z +7
= = and = = , is 237. The distance of the point (1, 0, 2) from the point of
1 −2 3 2 −1 −1 x −2 y +1 z −2
intersection of the line = = and the
20 10 3 4 12
(a) units (b) units
74 83 [2017 JEE Main] plane x − y + z = 16, is [2015 JEE Main]
5 10
(c) units (d) units (a) 2 14 (b) 8
83 74 (c) 3 21 (d)13
Chap 03 Three Dimensional Coordinate System 247

238. The equation of the plane containing the line 246. Statement I The point A (1, 0, 7 ) is the mirror image of
2x − 5y + z = 3, x + y + 4z = 5 and parallel to the plane x y − 1 z −2
x + 3y + 6z = 1, is [2015 JEE Main]
the point B (1, 6, 3) in the line = = .
1 2 3
(a) 2 x + 6y + 12z = 13 (b) x + 3y + 6z = − 7
(c) x + 3y + 6z = 7 (d) 2 x + 6y + 12z = − 13 x y − 1 z −2
Statement II The line = = bisects the line
1 2 3
239. The angle between the lines whose direction cosines
segment joining A (1, 0 , 7 ) and B (1, 6, 3). [AIEEE 2011]
satisfy the equations l + m + n = 0 and l 2 = m 2 + n 2 is
(a) Statement I is true, Statement II is true; Statement II is not a
π π
(a) (b) correct explanation for Statement I
3 4 [2014 JEE Main]
π π (b) Statement I is true, Statement II is false
(c) (d) (c) Statement I is false, Statement II is true
6 2
(d) Statement I is true, Statement II is true; Statement II is a
x −1 y −3 z − 4
240. The image of the line = = in the plane correct explanation for Statement I
3 1 −5
2x − y + z + 3 = 0 is the line [2014 JEE Main]
247. The length of the perpendicular drawn from the point
x + 3 y −5 z −2 x + 3 y −5 z + 2 x y –2 z –3
(a) = = (b) = = (3,–1, 11) to the line = = is
3 1 −5 −3 −1 5 2 3 4 [AIEEE 2011]
x −3 y + 5 z −2 x −3 y + 5 z −2 (a) 66 (b) 29
(c) = = (d) = =
3 1 −5 −3 −1 5 (c) 33 (d) 53
241. Distance between two parallel planes 2x + y + 2z = 8 and 248. The distance of the point (1,–5, 9 ) from the plane
4 x + 2y + 4z + 5 = 0 is [ 2013 JEE Main] x – y + z = 5 measured along a straight line x = y = z, is
3 5 [AIEEE 2010]
(a) (b)
2 2 (a) 3 5 (b) 10 3
7 9 (c) 5 3 (d) 3 10
(c) (d)
2 2 249. A line AB in three-dimensional space makes angles 45°
242. If the lines and 120° with the positive X-axis and the positive Y-axis,
x −2 y −3 z − 4 x −1 y − 4 z −5 respectively. If AB makes an acute angle θ with the
= = and = = positive Z-axis, then θ equals [AIEEE 2010]
1 1 −k k 2 1
(a) 30° (b) 45°
are coplanar, then k can have [2013 JEE Main] (c) 60° (d) 75°
(a) any value (b) exactly one value
(c) exactly two values (d) exactly three values 250. Statement I The point A(3, 1, 6) is the mirror image of
the point B(1, 3, 4) in the plane x − y + z = 5.
243. An equation of a plane parallel to the plane
Statement II The plane x − y + z = 5 bisects the line
x − 2y + 2z − 5 = 0 and at a unit distance from the origin
segment joining A(3, 1, 6) and B(1, 3, 4). [AIEEE 2010]
is [AIEEE 2012]
(a) x − 2y + 2z − 3 = 0 (b) x − 2y + 2z + 1 = 0 (a) Statement I is true, Statement II is true;
(c) x − 2y + 2z − 1 = 0 (d) x − 2y + 2z + 5 = 0 Statement II is the correct explanation of Statement I
x −1 y +1 z −1 x −3 y −k z (b) Statement I is true, Statement II is true;
244. If the line = = and = = Statement II is not the correct explanation of Statement I
2 3 4 1 2 1
(c) Statement I is true, Statement II is false
intersect, then k is equal to [AIEEE 2012]
(d) Statement I is false, Statement II is true
2
(a) –1 (b) x −2 y −1 z +2
9 251. Let the line = = lies in the plane
9 3 −5 2
(c)
2
(d) 0 x + 3y − αz + β = 0. Then, (α, β ) equals [AIEEE 2009]
y −1 z −3 (a) (6, − 17 ) (b) ( − 6, 7 ) (c) (5, − 15 ) (d) ( − 5, 15 )
245. If the angle between the line x = = and the
plane 2 λ 252. The projections of a vector on the three coordinate axes
−1  5 are 6, − 3, 2, respectively. The direction cosines of the
x + 2y + 3z = 4 is cos   , then λ equals
 14  [AIEEE 2011]
vector are [AIEEE 2009]
6 3 2
3 2 (a) 6, − 3, 2 (b) ,− ,
(a) (b) 5 5 5
2 5
6 3 2 6 3 2
(c)
5
(d)
2 (c) ,− , (d) − , − ,
3 3 7 7 7 7 7 7
248 Textbook of Vector & 3D Geometry

253. The line passing through the points (5, 1 a ), and (3, b, 1) a c
(a) aa ′ + cc ′ = 1 (b) + = −1
a ′ c′
 17 − 13
crosses the YZ-plane at the point 0, ,  . Then, a c
 2 2  (c) + =1 (d) aa ′ + cc ′ = − 1
a ′ c′
[AIEEE 2008]
(a) a = 8, b = 2 (b) a = 2, b = 8 259. The image of the point ( −1, 3, 4 ) in the plane x − 2y = 0 is
(c) a = 4, b = 6 (d) a = 6, b = 4  17 19 
(a) (15, 11, 4 ) (b)  − , − , 1
 3 3  [AIEEE 2006]
254. If the straight lines
 9 13 
x −1 y −2 z −3 x −2 y −3 z −1 (c) (8, 4, 4 ) (d)  , – , 4
= = and = =  5 5 
k 2 3 3 k 2
260. If the plane 2ax − 3ay + 4az + 6 = 0 passes through the
intersect at a point, then the integer k is equal to
[AIEEE 2008] mid-point of the line joining the centres of the spheres
(a) – 2 (b) – 5 x 2 + y 2 + z 2 + 6x − 8y − 2 z = 13 and
(c) 5 (d) 2 x 2 + y 2 + z 2 − 10x + 4y − 2 z = 8, then a equals
[AIEEE 2005]
255. Let L be the line of intersection of the planes
(a) 2 (b) − 2
2 x + 3y + z = 1 and x + 3y + 2 z = 2. If L makes an angle α
(c) 1 (d) − 1
with the positive X-axis, then cos α equals [AIEEE 2007]
x +1 y −1 z −2
(a) 1 / 3 (b) 1 / 2 261. If the angle θ between the line = = and
(c) 1 (d) 1 / 2 1 2 2
1
π the plane 2 x − y + λ z + 4 = 0 is such that sin θ = . The
256. If a line makes an angle of with the positive directions 3
4 value of λ is [AIEEE 2005]
of each of X-axis and Y-axis, then the angle that the line 4 3
makes with the positive direction of the Z-axis is (a) − (b)
3 4
[AIEEE 2007]
3 5
(a) π /6 (b) π /3 (c) − (d)
5 3
(c) π / 4 (d) π /2
262. The angle between the lines 2x = 3y = − z and
257. If (2, 3, 5) is one end of a diameter of the sphere
x + y + z − 6x − 12y − 2 z + 20 = 0, then the coordinates
2 2 2 6x = − y = − 4z is [AIEEE 2005]
(a) 30° (b) 45°
of the other end of the diameter are [AIEEE 2007]
(c) 90° (d) 0°
(a) ( 4, 9, − 3 ) (b) (4, –3, 3)
(c) (4, 3, 5) (d) (4, 3, –3) 263. The plane x + 2y − z = 4 cuts the sphere
258. The two lines x = ay + b, z = cy + d and x 2 + y 2 + z 2 − x + z − 2 = 0 in a circle of radius
x = a ′ y + b ′ , z = c ′ y + d ′ are perpendicular to each [AIEEE 2005]
other, if [AIEEE 2006,2003]
(a) 2 (b) 2
(c) 1 (d) 3
Chap 03 Three Dimensional Coordinate System 249

Answers
Exercise for Session 1 43. (c) 44. (c) 45. (c) 46. (a) 47. (b) 48. (a)
1. 8 2. 3 | k | 4. (1, 2, 3), (3, 4, 5), (− 1, 6, − 7) 49. (d) 50. (c) 51. (c) 52. (d) 53. (b) 54. (a)
55. (d) 56. (b) 57. (b) 58. (a) 59. (c) 60. (a)
5. 5 6. (4, 5, 6) 7. 90°
61. (b) 62. (d) 63. (a) 64. (a) 65. (b) 66. (c)
2 − 1 2  − 2 1 − 2
8. −1 9. 0 11.  , ,  or  , ,  67. (a) 68. (c) 69. (a) 70. (c) 71. (b) 72. (b)
 3 3 3  3 3 3  73. (a) 74. (b) 75. (c) 76. (a) 77. (d) 78. (a)
12. 14 79. (d) 80. (a) 81. (b) 82. (c) 83. (a) 84. (c)
85. (c) 86. (b) 87. (c) 88. (a) 89. (c) 90. (a)
Exercise for Session 2 91. (d) 92. (a) 93. (c) 94. (a) 95. (a) 96. (a)
1. r = 3$i − $j + 3k$ + λ(2 $i − 2$j + 5k$ ) 97. (b,d) 98. (a,b,c) 99. (b,c)
x −2 y+ 3 z−4 100. (b,e) 101. (a,c,d) 102. (a,b)
2. r = 2$i − 3$j + 4k$ + λ(3$i + 4$j − 5k$ ), = = 103. (b,c) 104. (a,b) 105. (a,b)
3 4 −5
106. (c,d) 107. (b,c) 108. (a,c)
 13 23 
3.  , , 0 − 1  19 
4. cos   5. (− 1, − 1, − 1) 109. (b,d) 110. (a,c) 111. (a,b,c,d)
 5 5   21
112. (c,d) 113. (a,d) 114. (a,b,c)
1 41
6. 7. 115. (a,b,d) 116. (a,c,d) 117. (b,c)
3 10 118. (b,d) 119. (a,c) 120. (a,b,c,d)
x − 1 y z −2 x−1 y z−2
8. = = or = = 121. (a,b,c) 122. (a,b,c) 123. (a,b,c)
−1 2 −7 1 −2 7 124. (a,b) 125. (d) 126. (d)
x −1 y − 2 z + 1 127. (a) 128. (b) 129. (b) 130. (d) 131. (a) 132. (c)
9. = = 10. (5, 8, 15)
1 −2 1 133. (a) 134. (a) 135. (b) 136. (a) 137. (a) 138. (d)
139. (d) 140. (b) 141. (b) 142. (b) 143. (d) 144. (b)
Exercise for Session 3 145. (b) 146. (c) 147. (a) 148. (d) 149. (d) 150. (b)
1 $
1. 2x − y + 3 z = 9 (2i + 2$j + k$ )
2. ± 151. (c) 152. (b) 153. (c) 154. (a) 155. (c) 156. (c)
3 157. (b) 158. (a) 159. (b) 160. (c) 161. (b) 162. (a)
3. 4x − 3 y + 2 z = 3 4. 5x + 18 y + 6 z = 0 163. (c) 164. (c) 165. (c) 166. (b) 167. (b) 168. (b)
5. x − 5 y − 2 z + 6 = 0, 3x − y + 4 z − 2 = 0 169. (a) 170. (b) 171. (c) 172. (b) 173. (c) 174. (d)
6. (− 3, 5, 2)
 15 
7. sin − 1  175. (A) → r (B) → (C) → (q, s)(D) → ( p, s)
 q,
 7 11  176. (A) → r (B) → q, (C) → p, (D) → s
8. y + z = 2 9. 13 177. (A) → q (B) → r, (C) → p (D) → s
10. 17x − 47 y − 24 z + 172 = 0 178. (A) → s (B) → q, (C) → (r , t ) (D) → ( p, s)
11. 3x − y + 3 z + 10 = 0 179. (A) → r (B) → q, (C) → q, (D) → (s)
12. x − 2 y + 2 z = 0 and x − 2 y + 2 z − 6 = 0 180. (A) → s, (B) → r, (C) → q, (D) → (s)
13. 25x + 17 y + 62 z = 238 (acute angle bisector) 181. (A) → r (B) → p, (C) → q
x + 35 y − 10 z = 256 (obtuse angle bisector) 182. (1) 183. (7) 184. (2) 185. (3) 186. (1) 187. (3)
14. x − 8 y + 4 z = 7 188. (2) 189. (8) 190. (2) 191. (4) 192. (3) 193. (4)
15. 2x + 2 y + z = 9 194. (3) 195. (7) 196. (7) 197. (7) 198. (9) 199. (6)
200. (4) 201. (4) 202. (4) 203. (1) 204. (9) 205. (2)
Exercise for Session 4 206. (1) 207. (1)
51
1. Centre (2, − 2, 0), Radius =
208. cos–1 
–1
210. (–2, – 1, 3) and  ,
56 43 111
2  , 
 3  17 17 17 
2. x2 + y2 + z 2 − 4x + 4 y − 4 z + 9 = 0, Centre (2, − 2, 2) $ $ $ $ $ $
211. r = (2i – j + 3k ) + µ (2 j + i - 2k )
3. x2 + y2 + z 2 − 2 3 y − 1 = 0 x y z x y z
212. = = and = =
4. 9x2 + 9 y2 + 9 z 2 − 54x − 108 y + 72 z + 545 = 0 1 2 –1 –1 1 –2
5. λ = 3 ± 3 213.
1775
sq units 216. 2x + y – 2 z + 3 = 0 and x – 2 y – 2 z – 3 = 0
6. 2x2 + 2 y2 + 2 z 2 − 6x + 2 y − 4 z = 25 28
217. x – 8 y + 4 z – 7 = 0
7. (i)  , , − 2
3 7 78
(ii) (iii) 5
2 2  2 218. (b, c, d) 219. (c) 220. (c) 221. (a,d) 222. (b,d)
223. (d) 224. (a) 225. (b, c) 226. (b) 227. (c) 228. (d)
Chapter Exercises 229. (b) 230. (d) 231. (d)
1. (b) 2. (b) 3. (b) 4. (a) 5. (a) 6. (b) 232. (A) → (r); (B) → (q); (C) → (b); (D) → (s)
7. (b) 8. (b) 9. (b) 10. (c) 11. (b) 12. (a) 233. (b) 234. (b) 235. (b) 236. (d) 237. (d) 238. (c)
13. (c) 14. (a) 15. (d) 16. (c) 17. (a) 18. (b) 239. (a) 240. (a) 241. (c) 242. (c) 243. (a) 244. (c)
19. (b) 20. (a) 21. (c) 22. (c) 23. (d) 24. (a) 245. (d) 246. (d) 247. (d) 248. (b) 249. (c) 250. (a)
25. (a) 26. (b) 27. (d) 28. (a) 29. (d) 30. (d) 251. (b) 252. (c) 253. (d) 254. (b) 255. (a) 256. (d)
31. (a) 32. (d) 33. (b) 34. (a) 35. (a) 36. (c) 257. (a) 258. (d) 259. (d) 260. (b) 261. (d) 262. (c)
37. (b) 38. (c) 39. (c) 40. (c) 41. (d) 42. (c) 263. (c)
Solutions
6. We have, α = 45° and β = 60°
Suppose OP makes angle γ with OZ. Then,
cos2 α + cos2 β + cos2 γ = 1
2 2
 1   1
⇒   +   + cos γ = 1
2
.  2  2
1. Suppose xy-plane divides the line joining the given points in 1 1
⇒ cos2 γ = ⇒ cos γ = ±
the ratio λ : 1. The coordinate of the point of division are 4 2
 2 λ − 1 −5 λ + 3 6 λ + 4 
 , , . ⇒ y = 60 °, 120 °
λ +1 λ +1 λ +1
7. Let OA and OB be two lines with direction l1, m1, n1and
This point lies on xy-plane. l 2, m2, n2.
6λ + 4 2 Let OA = OB = 1. Then, the coordinates of A and B are
∴ =0⇒λ =−
λ+1 3 (l1, m1, n1 ) and (l 2, m2, n2 ) respectively. Let OC be the bisector of
Hence, xy-plane divides externally in the ratio 2 : 3. ∠ AOB. Then, C is the mid point of AB and so its coordinates
are
Aliter We know that the xy-plane divides the line segment
joining P ( x1, y1, z1 ) and Q ( x 2, y 2, z 2 ) in the ratio −z1 : z 2.  l1 + l 2 m1 + m2 n1 + n2 
 , , 
Therefore, xy-plane divides the segment joining ( −1, 3, 4 ) and  2 2 2 
(2, − 5, 6 ) in the ratio −4 : 6 i.e. 2 : 3 externally. Y
B (– l2,m2,n2)
2. Suppose zx-plane divides the join of (1, 2, 3) and (4, 2, 1) in the l1+l2 ,m1+m2 , n1+n2
ratio λ : 1. Then, the co-ordinates of the point of division are C
2 2 2
 4λ + 1 2λ + 2 λ + 3 A (-l1,m1,n1)
 , , 
 λ + 1 λ + 1 λ + 1 X′
O
X
This point lies on zx-plane
l1+l2 ,m1+m2 , n1+n2
2λ + 2 E
∴ y-coordinate = 0 ⇒ = 0 ⇒ λ = −1 2 2 2
λ+1 D (-l2,-m2,-n2)
Hence, zx-plane divides the join of (1, 2, 3) and (4, 2, 1)
externally in the ratio 1 : 1. Y′
Aliter We know that the zx-plane divides the segment joining
P ( x1, y1, z1 ) and Q ( x 2, y 2, z 2 ) in the ratio −y1 : y 2. l1 + l 2 m1 + m2 n1 + n2
∴Direction ratios of OC are , ,
∴zx-plane divides the join of (1, 2, 3 ) and ( 4, 2, 1 ) in the ratio 2 2 2
−2 : 2 i.e. 1 : 1 externally. 2 2
 l + l2  m1 + m2   n1 + n2 
3. Suppose R divides PQ in the ratio λ : 1. Then, the coordinates Now, OC =  1  +  + 
 2   2   2 
of R are
1
 5 λ + 3 4 λ + 2 −6λ − 4 OC =
 , ,  2
 λ+1 λ+1 λ+1 
(l12 + m12 + n12 ) + (l 22 + m22 + n22 ) + 2(l1l 2 + m1m2 + n1n2 )
But, the coordinates of R are given as (9, 8, − 10 ).
5λ + 3 4λ + 2 1
∴ = 9, =8 ⇒ OC = 2 + 2 cosθ [Q cosθ = l1l 2 + m1m2 + n1n2 ]
λ+1 λ+1 2
−6λ − 4
1  θ
3 ⇒ OC = 2 + 2 cosθ = cos 
and = − 10 ⇒ λ = − 2  2
λ+1 2
∴ Direction cosines of OC are
Hence, R divides PQ externally in the ratio 3 : 2.
l1 + l 2 m1 + m2 n1 + n2
4. D divides BC in the ratio AB : AC i.e. 3 : 13. Therefore, , ,
2 (OC ) 2 (OC ) 2 (OC )
coordinates of D are
 3 × −9 + 13 × 5 3 × 6 + 13 × 3 3 × −3 + 13 × 2 l1 + l 2 m1 + m2 n1 + n2
 19 57 17 or, , ,
 , ,  or  , ,  . θ θ θ
 3 + 13 3 + 13 3 + 13   8 16 16 2 (cos ) 2 (cos ) (2 cos )
2 2 2
5. Let l ,m,n be the direction cosines of the given line. Then, as it 8. The given line is parallel to the vector n = $i − $j + 2k$ . The
makes an acute angle with x-axis. Therefore, l > 0. The line
required plane passes through the point (2, 3, 1 ) i.e., 2 $i + 3 $j + k$
passes through (6, − 7, − 1 ) and (2, − 3, 1 ). Therefore, its direction
ratios are and is perpendicular to the vector n = $i − $j + 2 k$ . So, its
6 − 2, − 7 + 3, − 1 − 1 or 4, − 4, − 2 or 2, − 2, − 1 equation is
2 2 1 { r − (2 $i + 3 $j + k$ )} ⋅ ( $i − $j + 2 k$ ) = 00000
Hence, direction cosines of the given line are , − , −
3 3 3 ⇒ r ⋅ ( i$ − $j + 2 k$ ) = 1
Chap 03 Three Dimensional Coordinate System 251

9. Le the position vectors of the given points A and B be a and b 15. Let A(1, 0,−1),B(−1,2,2)
respectively and that of the variable point P be r. It is given
that Direction ratios of segment AB are < 2, − 2, − 3 >.
2 × 1 + 3 ( − 2 ) − 5( − 3 ) 11 11
PA 2 − PB 2 = k (constant) cos θ = = =
1 + 9 + 25 4 + 4 + 9 17 35 595
⇒ | AP| 2 − | BP| 2 = k
Length of projection = ( AB )sin θ
⇒ | r − a| 2 − | r − b| 2 = k
121
⇒ {| r| 2 + | a| 2 − 2 r ⋅ a } − {| r| 2 + | b| 2 − 2 r ⋅ b } = k = (2 ) 2 + (2 ) 2 + (3 ) 2 × 1 −
595
⇒ 2 r ⋅ ( b − a ) = k + | b| 2 − | a| 2 474 474
= 17 = units
1 17 35 35
⇒ r ⋅ ( b − a ) = λ , where λ = {k + | b| 2 − | a| 2 }
2
16. Let the point be A, B, C and D
Clearly, it represents a plane.
The number of planes which have three points on one side and
10. The position vectors of two given points are a = $i − $j + 3 k$ and the fourth point on other side is 4. The number of planes
b = 3 $i + 3 $j + 3 k$ and the equation of the given plane is which have two points on each side of the plane is 3.
r = (5 $i + 2 $j − 7 k$ ) + 9 = 0 ⇒ Number of planes is 7.

or r. n + d = 0 17. Point A is (a, b, c ) ⇒ Points P , Q, R are (a, b, − c ), (−a, b, c ) and


(a, − b, c ) respectively.
We have a. n + d = ( $i − $j + 3 k$ ) ⋅ (5 $i + 2 $j − 7 k$ ) + 9
a b c a b c
= 5 − 2 − 21 + 9 < 0 ⇒ Centroid of triangle PQR is  , ,  ⇒G ≡  , , 
 3 3 3  3 3 3
and b. n + d = (3 $i + 3 $j + 3 k$ ) ⋅(5 $i + 2 $j − 7 k$ ) + 9
⇒ A, O, G are collinear ⇒ area of triangle AOG is zero.
= 15 + 6 − 21 + 9 > 0 x y z
18. Let the equation of the plane be + + = 1
So, the points a and b are on the opposite sides of the plane. a b c
11. The equation of a plane parallel to the plane ⇒
1 1 1
+ + =1
r ⋅( 4 $i − 12 $j − 3 k$ ) − 7 = 0 is, a b c
1
r ⋅( 4 $i − 12 $j − 3 k$ ) + λ = 0 ⇒ Volume of tetrahedron OABC = V = (a b c )
6
This passes through 2 $i − $j − 4 k$ 3
Now, (abc ) ≥
1/ 3
≥ 3 (G.M ≥ H.M)
∴ (2 $i − $j − 4 k$ ) ⋅ ( 4 $i − 12 $j − 3 k$ ) + λ = 0 1 1 1
+ +
⇒ 8 + 12 + 12 + λ = 0 a b c
⇒ λ = −32 9
⇒ a b c ≥ 27 ⇒ V ≥
So, the required plane is r ⋅( 4 $i − 12 $j − 3 k$ ) − 32 = 0 2
19. A (1, 1, 1)
12. Equation of the plane containing L1,
A( x − 2 ) + B(y − 1 ) + C (z + 1 ) = 0
where A + 2C = 0; A + B − C = 0
⇒ A = −2C , B =3C , C = C
⇒ Plane is − 2 ( x − 2 ) + 3 (y − 1 ) + z + 1 = 0 q
or 2 x − 3y − z − 2 = 0 O B (l, 2l, 3l)
−2 2
Hence, p= = Let any point of second line be ( λ , 2 λ , 3 λ )
14 7
6 6
13. (1, 2, 3) satisfies the plane x −2y + z = 0 and also cos θ = , sin θ =
42 42
( i$ + 2 $j + 3 k$ ) ⋅( $i − 2 $j + k$ ) = 0 1
∆OAB = (OA )OB sin θ
x −1 y −2 z −3 2
Since the lines = = and
1 2 3 1 6
= 3 λ 14 × = 6 ⇒ λ =2
x y z 2 42
= = both satisfy ( 0, 0, 0 ) and (1, 2, 3), both are same. Given
1 2 3 So, B is (2, 4, 6).
line is obviously parallel to the plane x − 2y + z = 6
20. Equation of line x + 2y + z − 1 + λ(−x + y − 2z − 2) = 0 …(i)
14. Vector (3 i$ − 2 $j + k$ ) × ( 4 $i − 3 $j + 4 k$ ) is perpendicular to 2 i$ − $j + mk$
x + y − 2 + µ( x + z − 2 ) = 0 …(ii)
3 −2 1 (0, 0, 1) lies on it
⇒ 4 −3 4 = 0 ⇒ m = −2 ⇒ λ = 0, µ = − 2
2 −1 m For point of intersection, z = 0 and solve (i) and (ii).
252 Textbook of Vector & 3D Geometry

x y z x y z 26. Given lines are skew lines and angle between them
21. The planes are + + = 1 and + + =1
a b c a′ b ′ c ′ 12 + 3 + 0 
= cos–1  
Since the perpendicular distance of the origin on the planes is  35 25 
same, therefore
5
    = cos–1
35
 −1   −1 
 1 1 1 =  1 1 1  27. Equation of plane
 + 2 + 2  2 + 2 + 2  x − 1 y − 1 z − 1
 a 2
b c   a′ b′ c′ 
 1 −2 
1 1 1 1 1 1  3 = 0
⇒ + + − − − =0  0 0 2 
a 2 b 2 c 2 a′ 2 b ′ 2 c ′ 2
22. Given one vertex A(7, 2, 4) and line P (1, 1, 1) S (x, y, z)
x + 6 y + 10 z + 14
= =
5 3 8
General point on above line, B ≡ (5 λ − 6, 3 λ − 10, 8 λ − 14 ) R
(1, 1, 3)
Direction ratios of line AB are < 5 λ − 13, 3 λ − 12, 8 λ − 18 > Q (2, 4, –1)
Direction ratios of line BC are < 5, 3, 8 >
⇒ 2(3 x − 3 − y + 1 ) = 0
π
Since, angle between AB and BC is . ⇒ 3x − y = 2
4
q
π (5 λ − 3 )5 + 3(3 λ − 12 ) + 8(8 λ − 18 ) 28. x-intercept =
cos = $i ⋅ n
4 (5 λ − 13 ) 2 q
∴ x1i ⋅ n = q ⇒ x1 =
5 2 + 3 2 + 8 2 ⋅ + (3 λ − 12 ) 2 i⋅n
+ (8 λ − 18 ) 2 29. P1 = 4 x + 6y − 7z − 1 = 0
Squaring and solving, we have λ = 3, 2 P2 = 4 x + 6y − 7z − 2 = 0
x −7 y −2 z − 4 1 1
Hence, equation of lines are = = d = =
2 −3 6 16 + 36 + 49 101
x −7 y −2 z − 4 101 × 102
and = = Hence, = 5151
3 6 2 2
23. L1L2 intersecting; L2L3 parallel; L3L1 skew. 30. x and z-intercept of the plane is 4 and it is parallel to y-axis,
24. λ = µ = 1 (point of intersection of two lines) hence equation of the plane is x + z = 4.
⇒ r = a + l or b + m, i.e., r = i + 2 j + k Its distance from ( 0, 0, 0 ) is 2 2.

25. Both the lines pass through origin. 31. Coordinate of L(0, g, h ) and M( f , 0, h ). Now, to find the
Line L1 is parallel to the vector equation of OLM.
V1 = (cosθ + 3 )i + ( 2 sin θ ) j + (cosθ − 3 )k and L2 is parallel O (0, 0, 0) P (x, y, z)
to the vector
V2 = ai + bj + ck
V ⋅V
∴ cos α = 1 2 I (0, g, –h) M (f, 0, h)
| V1 || V2 |
a (cos θ + 3 ) + (b 2 )sin θ + c(cos θ − 3 )
=  x y z
(cos θ + 3 ) 2 + 2 sin 2 θ ⇒  0 g h = 0
a2 + b2 + c2  
+ (cos θ − 3 ) 2  f 0 h
(a + c ) cos θ + b 2 sin θ + (a − c ) 3 ⇒ ( gh ) x + ( fh )y − ( gf )z = 0
=
a +b +c
2 2 2
2+6 x y z
or + − =0
f g h
In order that cos α in independent of θ
a+c=0 32. y-coordinate of P is zero.
and b=0 3 λ + ( −1 )
⇒ = P
2a 3 3 λ+1 2, 3, 4
∴ cos α = = 1, –1, 5
a 2 ⋅2 2 2 1
⇒ λ=
π 3
⇒ α=
6
Chap 03 Three Dimensional Coordinate System 253

x1 y z 38. Plane through a and parallel to two non-collinear vector


33. = u, 1 = v, 1 = w
4 4 4 (r − a ) ⋅ (b × c) = 0 (takes dot with b × c both sides)
x1 = 4u, y1 = 4v, z1 = 4w b
 4u 0 0 
1  64 c
V =  0 4v 0  =   uvw
6 6 A (a)
 0 0 4w 
 uvw  i.e., ( r − ( i$ − $j) ⋅ (5 $i − 2 $j − 3 k$ ) = 0
∴ 64   = 64k
3
 6  ⇒ r ⋅ (5 i$ − 2 $j − 3 k$ ) = 7
z1 39. Intersecting, if
 5 2 13 − p  5 2 13 − p
1 2 3 
G (u, v, w)  = 0 2 0
 −1 2 −3  −1 2 −3
Q x1
−4 ( −15 + 13 − p ) = 0
p = −2
Aliter
y1 ( λ + 2 ) = − (µ + 3 ) …(i)
xyz = 6k 3 2 λ + p = 2µ + 7 …(ii)
3 λ + 13 = p − 3µ …(iii)
34. From Eq. (i) µ = ( − λ + 5 )
z1 D(0, 0, d)
On putting in Eq. (ii), 2 λ + 9 = − 2( λ + 5 ) + 7
λ = −3
Now, from Eq. (iii), −9 + 13 = p + 6
4 A p = −2
3 B(b, 0, 0)
40. r = a + γb + µc
Taking dot with b × c
C(0, c, 0) [ r b c ] = [a b c ] [where, a = ( 0, 1, 1 )]
1 b = (1, − 1, 1 ) and c = (2, − 1, 0 )
Area of ∆BCD = | BC × BD | 0 1 1
2
1 $ 1 [a b c ] =  
 1 −1 1  = 0 − ( 0 − 2 ) + 1( −1 + 2 ) = 3
= | (bi − c $j) × (b$i − dk$ ) | = | bd $j + bc k$ + dc $i |
2 2  2 −1 0 
1 2 2 x y z
= b c + c 2d 2 + d 2b 2 …(i)
2 and [ r b c ] = 
1 −1 1 
 = x ( 0 + 1 ) − y ( 0 − 2 ) + z ( −1 + 2 )
Now, 6 = bc, 8 = cd, 10 = bd −1 0 
2
b 2c 2 + c 2d 2 + d 2b 2 = 200
= x + 2y + z
On substituting the value in Eq. (i), we get Hence, equation of plane is x + 2y + z = 3
1
A= 200 = 5 2 −3 
p =
3
2 ∴ =
 6 2
35. r = 2 $i + $j + 0 k$ + t( $i + $j) × ( $j + k$ )
 2 − a 9 − 7 13 − ( −2 ) 
= (2, 1, 0 ) + t( k$ − $j + $i ) = (2, 1, 0 ) + t(1, − 1, 1 ) 41.  1 2 3 = 0

36.
2
Option (a), − = − ≠
2 2  −1 2 −3 
3 3 9
⇒ a = −3
Option (b), 2 = 2 = 2 = 2 identical
−2 −2 −2 3
42. On (1, 2, 3) satisfies the plane x − 2y + z = 0 and also
Option (c), = = ≠ ( i$ + 2 $j + 3 k$ ) ⋅ ( $i − 2 $j + k$ ) = 0
3 3 3 5
Option (d), 2 = 2 ≠ − 6 x −1 y −2 z −3
Since, the lines = =
x = 9; x = 1  1 2 3
 x y z
37. y = 8; y = 2  and = = both satisfy t + 1 and 3t + 3.
1 2 3
z = 5; z = 3 
Hence, both are same.
Edges of the cuboid are 8, 6 and 2. Given line is obviously parallel to the plane x − 2y + z = 6
254 Textbook of Vector & 3D Geometry

 1 −c −b  ( p1 + λp 2 )( n 2 × n 3 ) = kp3( n 2 × n 3 )
43. Infinite solution  c −1 a  = 0 = p 3( n 2 × n1 ) …(iv)
 
 b a −1  p1( n 2 × n 3 ) + p 2λ ( n 2 × n 3 ) + p 3( n1 + n 2 ) = 0
⇒ a2 + b2 + c2 = 1 ⇒ p1( n 2 × n 3 ) + p 2( n 3 × n1 ) + p 3( n1 × n 2 ) = 0
[Using Eq. (iii)]
Note that 3 such planes can meet only at one point i.e. (0, 0, 0)
or they may have the same line of intersection i.e., at infinite 48. Equation of any plane through the intersection of r ⋅ n1 = q1
solution. and r ⋅ n 2 = q 2 is of the form
44. The given lines are coplanar, if r ⋅ n1 + λr ⋅ n 2 = q1 + λq 2 …(i)
2 − 1 3 − 4 4 − 5 1 −1 −1 where λ is a parameter.
 1 −k  So, n1 + λn 2 is normal to the plane (i). Now, any plane parallel
 1 = 0 ⇒ 1 1 −k = 0
to the line of intersection of the planes r ⋅ n 3 = q 3 and
 k 2 1  k 2 1 r ⋅ n 4 = q 4 is of the form.
1 0 0 r ⋅ ( n 3 + µn 4 ) = q 3 + µq 4 , hence we must have
⇒ 1 1 1 −k = 0 n1 + λn 2 = k( n 3 + µn 4 ) for some k
kk + 21 + k ⇒ [ n1 + λn 2 ] ⋅ [ n 3 × n 4 ] = 0
⇒ 2(1 + k ) − (k + 2 )(1 − k ) = 0 ⇒ [ n1 n 3 n 4 ] + λ [ n 2 n 3 n 4 ] = 0
[n n n ]
if k 2 + 3k = 0 ⇒ k = 0 or −3 ⇒ λ=− 1 3 4
[n2 n3 n4 ]
45. Put z = 0 in the line given x = 5 and y = 1 On putting this value in Eq. (i), we have the equation of
⇒ 5 ⋅1 = c 2 required plane as
x −2 y + 2 z −5 [n n n ]
r ⋅ n1 − q1 = 1 3 4 (r ⋅ n 2 − q 2 )
46. Equation of the line is = = =λ …(i)
1 −3 2 [n2 n3 n4 ]
Hence, any point on the line (i) can be taken as ⇒ [ n 2 n 3 n 4 ]( r ⋅ n1 − q1 )
x=λ+2 = [ n1 n 3 n 4 ]( r ⋅ n 2 − q 2 )
y = − (3 λ + 2 ) 49. Equation of line is
z = (2 λ + 5 ) r = $i + 0 $j + k$ + t( $i + 3 $j − k$ ) …(i)
From some λ point lies on the plane
Eq. (i) lies in x + y + cz = d
2 x − 3y + 4z = 163 …(ii)
∴ 1 + 0 + c =d
2( λ + 2 ) + 3(3 λ + 2 ) + 4(2 λ + 5 ) = 163
1 + c =d
19 λ = 133
Also, 1 ⋅ 1 + 1 ⋅ 3 + c( −1 ) = 0
⇒ λ =7
c=4
Hence, P ≡ (9, − 23, 19 )
∴ 1 + 4 =d ⇒ d =5
Also, Eq. (i) intersect YZ-plane i.e., x = 0
⇒ (c + d ) = 4 + 5 = 9
λ+2=0 x −2 y + 1 z −2
Hence, λ = −2 50. Any point on = = can be
2 4 12
∴ Q( 0, 4, 1 ) (2r + 2, 4r − 1, 12r + 2 )
So, PQ = 9 2 + 27 2 + 18 2 which lies on x − y + z = 5
∴ (2r + 2 ) − ( 4r − 1 ) + 12r + 2 = 5
= 9 1 + 3 2 + 2 2 = 9 14
r =0
⇒ a = 9 and b = 14 ∴Point on the plane ≡ (2, − 1, 2 )
Hence, a + b = 9 + 14 = 23 Distance between (2, −1, 2) and (−1, −5, −10)
47. Equation of the plane passing through the line of intersection = (2 + 1 ) 2 + ( −1 + 5 ) 2 + (2 + 10 ) 2
of the first two planes is r ⋅ ( n1 + λn 2 ), = p1 + λp 2. where λ is a
parameter = 13
Since, three planes have a common line of intersection the 51. R(r ) moves on PQ,
above equation should be identical with r ⋅ n 3 = p 3 for some λ.
R(r )
That is for some λ, ←
n1 + λn 2 = kn 3 …(i) P (p ) Q( q)
and p1 + λp 2 = kp 3 …(ii)
52. ($i + $j) × ($j + k$ ) = $i − $j + k$ ⇒ Unit vector perpendicular as to
From Eq. (i) 1 $ $ $
n1 × n 3 + λn 2 × n 3 = 0 the plane of $i + $j and $j + k$ is ( i − j + k ).
3
and n1 × n 2 = kn 3 × n 2 …(iii) Similarly, other two unit vectors are
From Eq. (ii)
Chap 03 Three Dimensional Coordinate System 255

1 1 $ $ $ x y z
and ( − i + j + k ). 55. Let + + = 1 be the variable.
3( i + j − k )
$ $ $ 3 a b c
 1 −1 1 
1  4
V = [ n$ 1, n$ 2, n$ 3 ] = 1 1 −1 
= 3 3 So that,
1
=1
3 3 1 1 1
 −1 1 1  + +
Aliter a2 b2 c2
Let a = i$ + $j, b = $j + k$ and c = k$ + $i. Then, the coordinates of ∆ABC are A(a, 0, 0 ), B( 0, b, 0 ) and
C ( 0, 0, c ).
Now, [a × b, b × c, c × a] = [a b c ]2
a b c
2 The centroid of ∆ABC is  , , 
1 1 0  3 3 3
= 
 0 1 1  = [1(1 ) − 1( 0 − 1 )] = 4
2
a b c
x = ,y = ,z =
1 0 1  3 3 3
Hence, actual volume with unit vectors 1 1 1
+ + =9
4 x2 y 2 z 2
=
| a × b| | b × c | | c × a | 56. Direction ratios of AB are 1 : 1 : 1.
Now, | a × b | = a 2b 2 − (a ⋅ b ) 2 = 4 − 1 = 3 etc. Direction ratios of CD are 1 : 2 : 1.
4 Angle between AB and CD,
Vactual =
3 3 1 ×1 + 1 ×2 + 1 ×1 4
cos θ = =
53. n = 3 $i − $j + 4 k$ 3 6 3 2
x y z
Line through A are parallel to n is 57. Equation of plane is + + = 1
1 2 3
r = $i + 2 $j + 3 k$ + λ (3 $i − $j + 4 k$ )
= 3 λ + 1, 2 − λ, 3 + 4 λ …(i) 1 2 3
+ – –1
n A (1, 2, 3) ∴ Required distance 1 2 3 =0
2 2 2
 1  1  1
  +  + 
 1  2  3

58. Angle between the faces = Angle between the normals


A¢ n1 = Vector normal to face OAB
(–1/2, 5/2, 1)
$i $j k$
Hence, Eq. (i) must satisfy the plane = OA × OB = 1 2 1 = 5 $i − $j − 3 k$
3 x − y + 4z = 0 2 1 3
3(3 λ + 1 ) − (2 − λ ) + 4(3 + 4 λ ) = 0
n 2 = Vector normal to face ABC
26 λ + 13 = 0
1 $i $j k$
λ=−
2 = AB × AC = 1 −1 2
 1 5  −2 −1 1
Hence, A′ is  − , , 1 which is the foot of the perpendicular
 2 2 
from A on the given plane. = i$ − 5 $j − 3 k$

54. On solving x + 2y − 4z = 0 and 2x − y + 2z = 0, we get n1 . n 2  19


Angle between faces = cos−1 = cos−1  
[n1 ] [ n 2 ]  35
x y z
= =
0 −10 −5 59. Q ≡ (1 + 2λ,2 + 3λ,3 + 4λ )
One point P on line is ( 0, − 10t, − 5t ) and Q ≡ (1, 1, 1 ) Direction ratio of PQ = 2 λ , 3 λ − 1, 4 λ − 1
Direction ratio of PQ ≡ (1, 1 + 10t, 1 + 5t ) Now, (2 λ )2 + (3 κ − 1 )3 + ( 4 λ − 1 ) 4 = 0)
0 − 10 − 100t − 5 − 25t = 0 29 λ = 7
3 7
⇒ t=− λ=
25 29
 6 3 Direction ratio of line PQ is (14, − 8, − 1 ).
⇒ P ≡  0, , 
 5 5
60. Equation of the plane passing through three points A, B and C
x −1 1 −y z −1 with position vector a, b and c is
Hence, required equation = = .
5 1 2 r ⋅ (a × b + b × c + c × a ) = a ⋅ b × c
256 Textbook of Vector & 3D Geometry

So that, if a, b, c represent the given vectors, then 1 p2 p2 1 p2 p2


Axy = ⋅ ⋅ , Ayz = ⋅ ⋅ ,
 i$ $j k$  2 h k 2 k l
a × b + b × c + c × a =  −6 3 2  1 p2 p2
  Azx = ⋅ ⋅
 3 −2 4  2 l h

 i$ $j k$   $i $j k$  A = Axy
2
+ Ayz
2
+ Azx
2

+  3 −2 4  +  5 7 3  p4 l 2 + h2 + k 2 p 4 p2 p5
    = = =
 5 7 3   −6 3 2  h 2k 2l 2 2 2 2
4 2 hkl 2hkl
= − 13 $i + 13 $j − 912 k$ p5
Hence Ar ( DABC ) =
 −6 3 2  2hkl
and a ⋅ a × c =  3 −2 4  = 299 65. Equation of the given plane can be written as
 
5 7 3 x y z
+ + =1
20 15 −12
So, the required equation of the plane is
r.( −13 i$ + 13 $j − 91 k$ ) = 299 or r ⋅ ( i$ − $j + 7 k$ ) + 23 = 0 which meets the co-ordinates axes in points A(20, 0, 0 ),
B( 0, 15, 0 ) and C( 0, 0, − 12 ) and the co-ordinates of the origin
61. The volume of tetrahedron are ( 0, 0, 0 ).
1 0 0 ∴The volume of the tetrahedron OABC is
1 1 1
= ( OA OB OC ) =  1 1 0
= 6
units   20 0 0 
6 6
0 1 1  1  0 15 0  =  1 × 20 × 15 × ( −12 )  = 600
6   6 
1
Area of the base = | ( i$ + $j − i$ ) × ( $j + k$ − $i ) |   0 0 −12 
2
66. l + m + n = 0, l 2 + m 2 − n 2 = 0
1 $ $ 1
= |i + k | = ⇒ l 2 + m 2 − ( −l − m ) 2 = 0
2 2
3 × Volume 3 2 1 ⇒ 2lm = 0 i.e., l = 0 or m = 0
Height = = =
Area of base 6 2 If l = 0, m = − n and if m = 0, l = − n
Since d.r.’s of the two lines are 0, 1, − 1 and 1, 0, − 1
62. x − y − z − 4 = 0, x + y + 2z − 4 = 0 0 × 1 + 1 × 0 + ( −1 ) × ( −1 ) 1
Required plane is of the form cos θ = =
2 2 2
( x − y − z − 4 ) + λ ( x + y + 2z − 4 ) = 0 π
⇒ θ=
Since, this plane is perpendicular to the plane x − y − z − 4 = 0 3
3
∴ 1 + λ + ( λ − 1 )( −1 ) + (2 λ − 1 )( −1 ) = 0, λ = 67. ( i$ − $j + 4 k$ ) ⋅ ( $i + 5 $j + k$ ) = 0
2
Therefore the line and the plane are parallel. A point on the
∴ Required plane 5 x + y + 4z = 20
line is (2, − 2, 3 ). Required distance is equal to distance of
63. Let l , m, n be the direction cosines of the normal to the plane | 2 + 5( −2 ) + 3 − 5 | 10
(2, − 2, 3 ) from the given plane = =
on which lies the plane area A. 12 + 52 + 12 27
Then, Axy = projection of A on the xy-plane
= A cos α, where α is the angle between the plane 68. Q Plane is perpendicular to the line
and xy-plane. ∴ Equation of plane is of the form 2 x − y + 2z + k = 0
l .0 + m.0 + n.1 Q If passes through origin ∴ k = 0
∴ cos α =
1 ∴ 2 x − y + 2z = 0
Since, the normal to the xy-plane has direction cosines ( 0, 0, 1 ) 69. PQ = i ( −2 − 3µ ) + $j(µ − 3 ) + k$ (5µ − 4 )
$
∴ Axy = An PQ is parallel to x − 4y + 3z = 1
Similarly, Ayxz = A1 ⇒ 1( −2 − 3µ ) − 4(µ − 3 ) + 3(5µ − 4 ) = 0
Azx = Am 1
⇒ µ=
∴Axy
2
+ Ayz2
+ Azx
2
= A2 4
64. Equation of the plane through P (h, k, l ) perpendicular to OP is 70. Plane meets axes at A(a, 0, 0), B(0, b, 0) and C (0, 0, c ).
xh + yk + zl = h + k + l = p
2 2 2 2
1
Then area of ∆ABC = | AB × AC |
where, p =h + k
2 2 2
+l 2 2
1
x y z = | ( −ai$ + bj$ ) × ( −ai$ × ck$ ) |
⇒ + 2 + =1 2
p2 p p2
1
h k l = (a 2b 2 + b 2c 2 + c 2a 2
2
Chap 03 Three Dimensional Coordinate System 257

71. Centre of the sphere is (−1, 1, 2) and its radius ⇒ β = −2


= 1 + 1 + 4 + 19 = 5 and v 3 ⋅ v 2 = 29 = α 2 + β 2 + γ 2

CL, perpendicular distance of C from plane, is ⇒ γ=±4


 −1 + 2 + 4 + 7  ∴ v 3 = 3 $i − 2 $j ± 4 k$
 = 4
 1+4+4  75. The given plane is r ⋅ (5i + 2 j − 7k ) = − 9
Length of the perpendicular from $i − $j + 3 k$ to it is
C −9 − ( $i − $j + 3 k$ ) ⋅ (5 $i + 2 $j − 7 k$ ) −9 − 5 + 2 + 21 9
= =
5 + 4 + 49 78 78
A L B
Length of the perpendicular from 3 i + 3 j + 3 k $ $ $
−9 − (3 $i + 3 $j + 3 k$ ) ⋅ (5 $i + 2 $j − 7 k$ ) −9 − 15 − 6 + 21 9
Now, AL2 = CA 2 − CL2 = 25 − 16 = 9 = =−
78 78 78
Hence, radius of the circle = 9 = 3 Thus, the length of the two perpendiculars are equal in
magnitude but opposite in sign. Hence, they are located on
72. Let r ×a = b ×a
opposite side of the plane.
⇒ (r − b) × a = 0
76. Let the position vector of A, B, C , D be a, b, c and d
⇒ r = b + ta respectively.
Similarly, other line r = a + kb, where t and k are scalars.
Then, AC 2 + BD 2 + AD 2 + BC 2
Now a + k b = b + ta
= (c − a ) ⋅ (c − a ) + (d − b) ⋅ (d − b)
⇒ t = 1, k = 1 (equation the coefficients of a and b)
+ (d − a ) ⋅ (d − a ) + (c − b) ⋅ (c − b)
∴ r = a + b = $i + $j + 2 $i − k$
= | c | + | a | − 2a ⋅ c + | d | 2 + | b | 2
2 2
= 3 $i + $j − k$
− 2 d ⋅ b + | d | 2 + | a | 2 − 2a ⋅ d + | c | 2
i.e. (3, 1, −1)
+ | b | 2 − 2b ⋅ c
73. Let the point P be (x, y , z ), then the vector x$i + y$j + zk$ will lie
on the line = | a | 2 + | b | 2 − 2a ⋅ b + | c | 2 + | d | 2
⇒ ( x − 1 ) i$ + (y − 1 ) $j + (z − 1 ) k$ − 2c ⋅ d + | a | 2 + | b | 2 + | c | 2 + | d | 2
= − λ$i + λ$j − λk$ + 2 a ⋅ b + 2 c ⋅ d − 2a ⋅ c − 2 b ⋅ d
⇒ x = 1 − λ, y = 1 + λ and z = 1 − λ − 2a ⋅ d − 2 d ⋅ c
Now point P in nearest to the origin. = (a − b ) ⋅ (a − d ) + ( c − d ) ⋅ ( c − d )+
⇒ D = (1 − λ ) 2 + (1 + λ ) 2 + (1 − λ ) 2 + (a + b − c − d ) ⋅ (a + b − c − d )
dD = AB 2 + CD 2 + (a + b − c − d ) ⋅(a + b − c − d )
⇒ = − 4(1 − λ ) + 2(1 + λ ) = 0
dλ ≥ AB 2 + CD 2
1  x1 y1 z1 
⇒ λ=
3 77. If the given vectors are coplanar, then x 2 y 2 z 2  = 0
 2 4 2  x3 y 3 z 3 
⇒ The point is  , ,  .
 3 3 3
The set of equation
74. We have, | v1 | = 2, | v 2 | = 2 and | v 3 | = 29 x1x + y1y + z1z = 0
If θ is the angle between v1 and v 2, then x 2x + y 2y + z 2z = 0
2 2 cos θ = − 2 x 3x + y 3y + z 3z = 0
1 has a non-trivial solution.
⇒ cos θ = − ⇒ θ = 135 °
2 Let the given set has a non-trivial solution x, y , z without loss
of generality, we can assume that x ≥ y ≥ z .
For the given equation x1x + y1y + z1z = 0, we have
v2
x1x = − y1y − z1z
⇒ | x1x | = | y1y + z1z | ≤ | y1y | + | z1z |
v2
⇒ | x1x | ≤ | y1x | + | z1x |
Let v1 = 2 i, v 2 = − $i + $j and v 3 = α$i + β$j + γk$
⇒ | x1 | < | y1 | + | z1 |
Since v 3 ⋅ v1 = 6 = 2α Which is a contradiction to the given inequality.
⇒ α =3 | x1 | > | y1 | + | z1 |
Also, v3 ⋅ v2 = − 5 = − α + β
258 Textbook of Vector & 3D Geometry

Similarly, the other inequalities rule out the possibility of a 82. Q = (r , r , − r )


non-trivial solution. x y z
Therefore, the given equations have only a trivial solution. PQ perpendicular = =
1 1 −1
So, the given vectors are non-coplanar.
∴ (α − r ) ⋅ 1 + (β − r ) ⋅ 1 + ( γ + r ) ( − 1 ) = 0
78. The vectors n1 × n 2, n 2 × n 3 and n 3 × n1 are non-coplanar α+β−γ
vectors, so every vector can be written as r=
3
r = a( n1 × n 2 ) + b( n 2 × n 3 ) + c( n 3 × n1 ) 2 2
Substituting this value in r ⋅ n1 = q1, a, b, we get ∴ PQ = {α + β 2 + γ 2 − αβ + βγ + γ }
2
3
a( n1 × n 2 ) ⋅ n1 + b( n 2 × n 3 ) ⋅ n1 + c( n 3 × n1 ) ⋅ n1 = q1 But PQ = 2
q1 {perpendicular distance from P(α , β, γ ) to plane x + y + z = 0}
b( n1 n 2 n 3 ) = q1 ⇒ b =
n1 n 2 n 3 2
2 2 α + β + γ 
∴ (α + β 2 + γ 2 − αβ + βγ + γα ) = 4  
Since, the required point of intersection will have the position 3  3 
vector,
∴ α 2 + β 2 + γ 2 + 5αβ + 3βγ + 3 γα = 0
1
r= [q 3( n1 × n 2 ) + q1( n 2 × n 3 ) + q 2( n 3 × n1 )]
( n1 n 2 n 3 ) 83. The cut x = y separates the cube into points with x < y and
those with x > y .
79. Since r + s = e
2 2 2
∴ So, number of pieces equals to the number of ways of
⇒e = 31 or e = 19 is not possible. arrangements of x, y and z which is 3 ! = 6.
Therefore, e equals 13, 20 or 25. Aliter Since in each coordinate there is inequality x > y > z .
The possibility for triplet {r , s , e } are {5, 12, 13}, {12, 16, 20}, So, number of pieces = number of ways of arranging x, y , z = 6
{15, 20, 25}, {7, 24, 25}.
84. Here P and Q lie on the same side of XY plane
b Image P(1, 2, 3 ) on the XY plane is P′ (1, 2, − 3 )
x −3 y −2 z −5
a Reflected ray is P ′ Q ⇒ = =
2 0 8
c
x −3 y −2 z −5
e ⇒ = =
s 1 0 4
x y z
d r 85. Let + + = 1 be required plane.
a b c
Since 16, 15 and 24 do not appear among any of pair wise Let the sphere be x 2 + y 2 + z 2 + 2ux + 2vy + 2wz + d = 0
differences of 13, 19, 20, 25, 31
d = 0 if it passes through origin.
⇒a = 19, b = 25, c = 31, d = 20, e = 13
Also, a = − 2u, b = − 2v, c = − 2w
Hence, required area = 745 sq units.
α −β β − γ γ −α
80. Point A is (a, b, c ) and + + =1
− 2u − 2v − 2w
⇒ Points P , Q, R are (a, b, − c ), ( − a, b, c ) and (a, − b, c ) Locus of centre ( − u, − v, − w ) is Σ(α − β )yz = 2 xyz
respectively.
a b c
86. On solving the given planes, the vertices are O(0, 0, 0),
⇒ Centroid of triangle PQR is  , ,  A( − a, a, a ), B(a, − a, a ), C (a, a, − a ).
 3 3 3
x y z
a b c Consider the edges OA, BC whose equations are = = ;
⇒G ≡  , ,  −1 1 1
 3 3 3
x −a y + a z −a
⇒ A, O, G are collinear ⇒ Area of triangle AOG is zero. = =
0 1 −1
81. Let line joining AB meet plane 2x + 3y + 5z = 1 at P. Now, find S.D. between the lines.
 λ + 1 − 5λ 7λ − 3  AP  87. The angle between the pair of planes is
Let P = , ,  =λ
λ + 1 λ + 1 λ +1  PB 
ax 2 + by 2 + cz 2 + 2 fyz + 2 gzx + 2hxy = 0 is
 − 5λ   7λ − 3  2 f 2 + g 2 + h 2 − ab − bc − ca 
2.1 + 3   +5  =1 θ = tan − 1  
 λ + 1  λ + 1  a+b+c 
 
2 ( + 1 ) − 15 λ + 35 λ − 15 = λ + 1 88. 2 p + 2q + r = 6
2
⇒ λ= ⇒(2 $i + 2 $j + k$ ) ⋅ ( p$i + q $j + r k$ ) = 6
3
⇒ P = (1, − 2, 1 ) (a ⋅ b ) 2 ≤ | a | 2 | b | 2
⇒ AP = 2 5 ⇒ 6 2 ≤ 9( p 2 + q 2 + r 2 )
p + q + r2 ≤ 4
2 2
Chap 03 Three Dimensional Coordinate System 259

89. Let A(x1, y1, z1 ), B (x 2, y 2, z 2 ), C (x 3, y 3, z 3 ), D (x 4 , y 4 , z 4 ) be the x y z


95. The plane equation in the intercept forms is + + = 1.
vertices of tetrahedron. If E is the centroid of face BCD and G is a b c
the centroid of ABCD then AG = 3 / 4( AE ) Volume of tetrahedron OABC is
∴ K = 3 / 4. abc
v= = 64 ⇒ abc = 384
6
90. y (x + y ) + z (x + y ) = 0
Foot of perpendicular from (0, 0, 0) on this plane is
x + y = 0 ⇒ dr’s are b1 = (1, 1, 0 ) x y z 1
and y + z = 0 ⇒ dr’s are b 2 = ( 0, 1, 1 ) = = = =k
1/a 1/b 1/c 1 1 1
+ +
i$ $j k$ a2 b2 c2
Now, b1 × b 2 = 1 1 0 = (1, − 1, 1 ) k k k
⇒ x = ,y = ,z =
0 1 1 a b c
1 1 1 1
and a1 − a1 = (1, 1, 1 ) − ( 0, 0, 0 ) = (1, 1, 1 ) and = + +
k a2 b2 c2
|(1, 1, 1 ) × (1, − 1, 1 )| |( −2, 0, 2 )|
∴ SD = = 1 x2 + y 2 + z 2
| (1, − 1, 1 ) | |(1, − 1, 1 )| ⇒ =
k k2
4+0+4 8
= = ⇒ x + y + z =k
2 2 2
1+1+1 3
∴( x 2 + y 2 + z 2 ) 3 = k 3 = abc xyz = 384 xyz is the required
91. L1 || L2 Then required distance = distance between (k1, k2, 0), locus.
(k3, k4 , 0 )
96. Let A (x1, y1, z1 ), B (x 2, y 2, z 2 )
= (k1 − k3 ) 2 + (k2 − k4 ) 2
C ( x 3, y 3, z 3 ), D ( x 4 , y 4 , z 4 )
92. Let a = l1i$ + m1$j + n1k$ , b = l 2$i + m2$j + n2k$ and the equation of the plane containing P , Q, R and S is
and c = l 3$i + m3$j + n3k$ ax + by + cz + d = 0 and kR = axr + byr + czr + d
AP BQ CR DS − K1 − K 2 − K 3 − K 4
Given that a, b, c are three mutually perpendicular unit ∴ ⋅ ⋅ ⋅ = ⋅ ⋅ ⋅ =1
PB QC RD SA K2 K3 K4 K1
vectors.
Then, p1$i + q1$j + r1k$ = b × c = a 97. Let a, b and c be the DR’s of the given line. Then,
(Q b × c parallel to a and b × c, a are unit vectors) we have 3a − b + c = 0
Similarly, p 2i$ + q 2$j + r2k$ = c × a = b 5a + b + 3c = 0
a b c
and p 3i$ + q 3$j + r3k$ = a × b = c On solving, we get = =
1 1 −2
These vectors also mutually perpendicular unit vectors.
Again, suppose the given line intersect the plane z = 0 at
93. Let us suppose A be origin. ( x1, y1, 0 ), then 3 x1 − y1 + 1 = 0 and 5 x1 + y1 = 0
ar( ∆ABC ) 2 + ar( ∆ACD ) 2 + ar( ∆ABD ) 2 = ar( ∆BCD ) 2 On solving, we get x1 = − , y1 =
1 5
Hence the result follows. 8 8
Hence, the symmetrical form of the line is
94. a, b, c be P.V. of A, B, C , | a | = | b | = | c | = K
1 5
a b+c x+ y −
OD = | OE| = 8 = 8 = z
2 2 1 1 −2
b + c −a Equation of plane through (2, 1, 4) is
Given, | DE | = 1 ⇒ =1 ⇒ | b + c −a| =2
2 a( x − 2 ) + b(y − 1 ) + c(z − 4 ) = 0
 1 1 1 when a = 1, b = 1 and c = − 21
⇒ 3k 2 + 2k 2  − −  = 4
 2 2 2 x − 2 + y − 1 − 2(z − 4 ) = 0
x + y − 2z + 5 = 0
O
98. Vector normal to the plane
n = $i + $j + k$
D
Vx = $i, Vy = $j, Vz = k$
A
Vx ⋅ n
cos (90° − α ) =
B E C | n|
1
⇒ K = 2 ⇒ sinα =
3
1 1 3 2  1 1 1
Volume = (base area × height) =  (k ) 2 × k = Similarly, sin β = and sin γ =
3 3 4 3  3 3 3
260 Textbook of Vector & 3D Geometry

Hence, ∑ sin2α = 1 = {(a + b + c ) ⋅ c } b − {(a + b + c ) ⋅ a } b


+ {(a + b + c ) ⋅ c } b − {(a + b + c ) ⋅ b } × c
and ∑ cos2α = 2 + {(a + b + c ) ⋅ a } c − {(a + b + c ) ⋅ a }a
Also, plane is equally inclined with the coordinate axes. = (a + b + c ) ⋅ ( b − c )a + (a + b + c )
1 2 9 3 ⋅( c − a ) b + {(a + b + c ) ⋅ (a − b )} c
Also, A = 9 + 92 + 92 =
2 2 = (a ⋅ b − a ⋅ c + b 2 − c 2 )a + ( b ⋅ c − b ⋅ a
99. 2x − y − z − 2 + λ(x + y + z − 1) = 0 satisfies (1, 1, 1) + ( c 2 − a 2 ) b + (a 2 − b 2 + c ⋅ a− c − b ) c = 0
2 − 1 − 1 − 2 + λ(3 − 1 ) = 0 Thus, the statement is true.
λ =1
102. Let A(a ), B(d ), C (c ) and D(d ) be the vertices of a tetrahedron,
x =1 …(i)
then centroid of the tetrahedron is
x − y − z − 3 + µ(2 x + 4y − z − 4 ) = 0 a+b+c+d
−4 + µ(1 ) = 0 4
µ=4 b+c+d
x − y − z − 3 + 4(2 x + 4y − z − 4 ) = 0 centroid G1 of the face BCD is
3
9 x + 15y − 5z − 19 = 0 …(ii) Now, centroid of the tetrahedron G1 divides AG1 in the ratio
From Eqs. (i) and (ii), we get 3 : 1.
x − 1 = 0, 9 x + 15y − 5z − 19 = 0 3( b + c + d ) + a a + b + c + a
i.e. =
a = 0 and c = 3b 3+1 4
y −1 z −1 ∴ C lies on AC1.
x − 1 = 0, =
1 3 (b) The edges AB and CD. Let E be the mid point of AB and F
100. OP = h 2 + k 2 + l 2 be the mid point of CD
a+b
h k l  ∴Positive vector of E is
Direction ratios of OP are  , ,  2
 p p p
c+a
hx ky lz Positive vector of F is
Equation of plane is + + =p 2
p p p A

p   p
2  
2
p2
A  , 0, 0 , B  0, , 0 , C  0, 0, 
h   k   l 
Z

C
C

P
Y B D
O B a+ b+ c+ d
Mid-point of EF is which is the centroid of the
4
A tetrahedron ABCD.
X
x y z
103. Let the plane be + + = 1
101. (a) Since, n ⋅ a = n ⋅ b = n ⋅ c = 0 a b c
∴ a, b and c are coplanar  
∴ [a, b, c ] = 0  
1 1 1 1
  = 1 or 2 + 2 + 2 = 1
(b) cos3 30 ° + cos2 45 ° + cos2 γ = 1 
2 2 2
  1  +  1  +  1 
a b c

3 1 5
∴ sin 2 r = + = which is not possible.  a b c 
4 2 4
The plane cuts the coordinates axes at A(a, 0, 0 ), B( 0, b, 0 ),
(c) Obvious c( 0, 0, c ). The centroid of ∆ABC is
(d) AB × BC is perpendicular to the plane ABC. a b c
AB × BC = ( OB − OA ) × ( OC − OB)  , ,  = ( x, y , z ) (let)
 3 3 3
= OB × OC − OA × OC + OA × OB x −2 + y −2 + z −2 = 9
= OA × OB + OB × OC + OA × OB –1
i.e. a × b + b × c + c × a is perpendicular the plane ABC. 1 1 1 1
or  2 + 2 + 2 =0
(a + b + c ) × (a × b + b × c + c × a) 9 x y z 
Chap 03 Three Dimensional Coordinate System 261

104. When a line lies in a plane, then it is at right angles to the ∴ Angle between A and 2 $i + $j − 2 k$ is
normal to the plane. Here, d.r’s of the line are < a, b, c > and
altitude numbers of the plane are being taken as < A, B, C >. ( −54 $j + 54 k$ ) ⋅ (2 $i + $j − 2 k$ ) 1
cos θ = ± =±
So, we must aA + bB + cC = 0. 54 2.3 2
105. For the given curve z = 0, therefore, the line and the curve π 3π
θ = or
x −2 y + 1 0 −1 4 4
meet where = =
3 2 −1 110. Any plane through the second line is
x −2 y +1
i.e. where = 1, = 1 i.e., where x = 5, y = 1 2 x + y + z − 1 + k(3 x + y + 2z − 2 ) = 0
3 z If this is parallel to the first line, then
So, the given line and the given curve meet in the point (2 + 3 k ) + (1 + k ) + (1 + 2 k ) = 0
(5, 1, 0). Since, this point lies on the curve also, therefore,
2
52 + 12 = r 2 ⇒ k=−
3
⇒ r 2 = ( 26 ) 2 2
⇒ Plane is 2 x + y + z − 1 − (3 x + y + 2z − 2 ) = 0
⇒ r = ± 26 3
106. A vector coplanar with given vectors is or y − z + 1 = 0. The required SD must be distance of this
(1 + λ ) i$ + ( λ − 1 ) $j + (1 − λ ) k.
$ Since it is equally inclined to plane from any point on the line x = y = z say (1, 1, 1)
the two given vectors 1 −1 + 1 1
⇒ SD = =
(1 + λ ) i$ + ( λ − 1 ) $j + (1 − λ ) k$ ( i$ − $j + k$ ) 0 2 + 1 2 + ( −1 ) 2 2

| (1 + λ ) $i + ( λ − 1 ) $j + (1 − λ ) k$ | 3 4 1/2 1 8
111. p1 = , p2 = = , p3 =
(1 + λ ) i$ + ( λ − 1 ) $j + (1 − λ ) k$ ( i$ + $j − k$ ) 29 29 2 29 29
= ⋅
| (1 + λ ) $i + ( λ − 1 ) $j + (1 − λ ) k$ | 3 For these values all the choices are easily verified.
∴ λ =1 112. Let the components of the line segment vector be a, b, c, then
Required vector is 2 $i or $i
a 2 + b 2 + c 2 = (63 ) 2 …(i)
107. The equation of the plane through (2, 3, −1) and perpendicular a b c
to the vector 3 i$ − 4 $j + 7 k$ is also = = = λ (say) then
3 −2 6
3( x − 2 ) + ( −4 )(y − 3 ) + 7(z − ( −1 )) = 0 a = 3 λ , b = − 2 λ and c = 6λ
or 3 x − 4y + 7z + 13 = 0 and from (i), we have
Distance of this plane from the origin
9 λ2 + 4 λ2 + 36 λ2 = (63 ) 2
| 3 × 0 − 4 × 0 + 7 × 0 + 13 | 12
= = ⇒ 49 λ2 = (63 ) 2
3 + ( −4 ) + 7
2 2 2 74
63
⇒ λ=± = ± 9.
108. Let A, B, C be (α, 0, 0), (0, β, 0) and (0, 0, γ ) then the plane ABC 7
x y z The required components are 27, −18, 54 or −27 , 18, −54
is + + =1
α β γ
113. The given lines are coplanar if
Since it always passes through a, b, c 2 − 1 3 − 4 4 − 5 
a b c
+ + =1 …(i) 0 =
 1 1 −k 

α β γ
 k 2 1 
If p is (u, v, w ) then OP 2 = AP 2 = BP 2 = CP 2
 1 −1 −1   1 0 0 
⇒ u 2 + v 2 + w 2 = (u − α ) 2 + v 2 + w 2
u v w =
 1 1 −k =  1
  2 1 − k

=… ⇒ α = , β = , γ = 1  k k + 2 1 + k 
2 2 2 k 2
On putting α , β, γ in (i), we get or if 2(1 + k ) − (k + 2 )(1 − k ) = 0
a b c or if k 2 + 3k = 0 or if k = 0, − 3.
+ + =2
u v w
a b c 114. Direction ratios of AB are 4 − 2, 5 − 3, 10 − 4 or 1, 1, 3. So AB in
⇒ Locus of (u, v, w ) is + + = 2 parallel to the vector i$ + $j + 3 k$ and passes through B (2, 3, 4 ),
x y z
the vector 2 i$ + 3 $j + 4 k$ , its equation is
109. Normal of plane P1 is 2 i$ + 3 $j + 4 k$ + λ ( $i + $j + 3 k$ )
n1 = (2 $j + 3 k$ ) × ( 4 $j − 3 k$ ) = − 18 $i
Similarly, BC passes through the points B (2, 3, 4 ) and its
Normal to plane P2 is direction ratios are 2 − 1, 3 − 2, 4 + 1 or 1, 1, 5.
n 2 = ( $j − k$ ) × (3 $i + 3 $j) = 3 $i − 3 $j − 3 k$ So its cartesian equation is
∴ A is parallel to ( n1 < n 2 ) = ± ( −54 $j − 54 k$ ) x −2 y −3 z − 4
= =
1 1 5
262 Textbook of Vector & 3D Geometry

Next, if D is (a, b, c ), then since ABCD is a parallelogram mid 118. Volume = Area of base × height
point of AC and BD is same. (diagonals of a parallelogram
A1(a, b, c)
bisect each other)
 a + 2 b + 3 c + 4  5 7 9
⇒  , , = , ,  B1 C1
 2 2 2   2 2 2
⇒ (a, b, c ) = (3, 4. 5 )
AB is not perpendicular to BC because A (1, 0, 1)
1 × 1 + 1 × 1 + 3 × 5 ≠ 0. √2 √3
ABCD is not a rectangle.
B √5 C
115. The coordinates of P where the line x = y = z meets the plane (2, 0, 0) (0, 1, 0)
x + y + z = 1 are (1/3, 1/3, 1/3 ) and the co-ordinates of R and S 1
where the line meets the sphere x 2 + y 2 + z 2 = 1 are 3= × 2 × 3 ×h
2
(1 / 3, 1 / 3, 1 / 3 ) and ( −1 / 3, − 1 / 3, − 1 / 3 )
h= 6
 1 1   1 1
So that PR = 3 −  = 3  −  ( A1A ) 2 = h 2 = 6
 3 3  3 3
A1A ⋅ AB = 0
1 1 
and PS = 3  +  A1A ⋅ AC = 0
3 3
AA1 ⋅ BC = 0
 1 1 2
⇒ PR ⋅ PS = 3  −  = On solving, we get position vector of A1 are ( 0, − 2, 0 ) or (2, 2, 2 ).
 3 9 3
119. Let the equation of plane be lx + my + nz = 0, where l, m, n be
PR + PS = 2
d.c’s ⇒l 2 + m 2 + n 2 = 1 → (i)
and RS = (1 / 3 + 1 / 3 ) 2 × 3 = 2
x −1 y + 3 z + 1
So that, PR + PS = RS Given line = = ⇒ 2l − m − 2n = 0 → (ii);
2 −1 −2
116. The rod sweeps out the figure which is a cone. l − 3m − n 5
Also, =
A (1, 0, –1) l 2 + m2 + n2 3
5
⇒ l − 3m − n = → (iii)
3
Solving (i), (ii) and (iii), we get equation of plane as
r = √3
x − 2y + 2z = 0 or 2 x + 2y + z = 0.
1 2
120. (a) Height = h = 1 − =
3 3
The distance of point A(1, 0, − 1 ) from the plane is 1  2 1
(b) Required distance =   =
|1 − 2 + 4| 2  3 6
= 1 unit.
9 π
(c) Angle =
The slant height l of the cone is 2 units. 2
Then the radius of the base of the cone is 3 3  2 3
(d) Required distance = (h ) =   =
l 2 − 1 = 4 − 1 = 3. 4 4  3 8
π 121. Let OA = a, OB = b, OC = c then
Hence, the volume of the cone is ( 3 ) 2 ⋅ 1 = π cubic units.
3
a ⋅ a + (b − c ) ⋅ (b − c ) = b ⋅b + (c − a ) ⋅ (c − a )
Area of the circle on the plane which the rod traces is 3π.
⇒ − 2b ⋅ c = − 2c ⋅ a ⇒(a ⋅ b ) ⋅ c = 0
Also, the centre of the circle is Q( x, y , z ).
or BA ⋅ OC = 0
x −1 y −0 z + 1
Then = = Hence, AB ⊥ OC similarly BC ⊥ OA and CA ⊥ OB.
1 −2 2
−(1 − 0 − 2 + 4 ) 122. Intersection of line with both the planes are the same
= 2 3 −6
l + ( −2 ) 2 + 2 2 ⇒ =
3β 2 + 6(1 − 2α ) + 3 6α 2 + 6(1 − 2β ) + 6
 2 2 −5 
or Q ( x, y , z ) ≡  , ,  .
3 3 3  ⇒ 2(β − 1 ) 2 + 3(α − 2 ) 2 = 0 ⇒α = 2, β = 1

117. Observe that the lines L1, L2 and L3 are parallel to the vector 123. If P be (x, y , z ) then from the figure,
(1, − 1, − 1). x = r sin θ cos φ, y = r sin θ sin φ, z = r cosθ
Also, ∆ = 0 = ∆1 and b1c 2 − b2c1 ≠ 0 ⇒ 1 = r sin θ cos φ, 2 = r sin θ sin φ, 3 = r cosθ
Chap 03 Three Dimensional Coordinate System 263

⇒1 2 + 2 2 + 3 2 = r 2 ⇒r = ± 14 i$( −3y − 2z ) − $j( −3 x − z ) + k$ (2 x − y )


1 2 3 ∴ −3y − 2z = 2, 3 x + z = − 1, 2 x − y = 0
∴ sin θ cos φ = , sin θ sin φ = , cos θ =
14 14 14 i.e. −6 x − 2z = 2, 3 x + z = 1.
z ∴Straight line 2 x − y = 0, 3 x + z = − 1
Statement I
P(x, y, z)
 $i $j k$ 
r r × (2 i − $j + 3 k ) =  x y z 
$
 
θ  2 −1 3 
y = $i(3y + z ) − $j(3 x − 2z ) + k$ ( − x − 2y )
φ
∴ 3y + z = 3, 3 x − 2z = 0, − x − 2y = 1
3 x − 2(3 − 3y ) = 0
x M 3 x + 6y = 6
⇒ x + 2y = 2
(neglecting negative sign as θ and φ are acute)
Now, x + 2y = − 1, x + 2y = 2 are parallel planes.
sin θ sin φ 2
∴ = ⇒ tan φ = 2 ∴r × (2 $i − $j + 3 k$ ) = 3 $i + k$ is not a straight line.
sin θ cos φ 1
5  2 −3 + 2  1
Also, tanθ = 127. sinθ = =
3  4 + 9 + 4 3  51
124. Let (l , m, n ) be the direction cosines of the line perpendicular to ∴Statement I is true, Statement II is true by definition.
the plane. 128. Statement I
⇒ Equation of the plane lx + my + nz = p 3y − 4z = 5 − 2k
x y z −2y + 4z = 7 − 3k
+ + =1
 p  p  p 31 − 13k
      ∴ x = k, y = 12 − 5k, z =
l  m  n is a point on the line for all
4
A ( p / l , 0, 0 ), B ( 0, p / m, 0 ), C ( 0, 0, p / n ) real value of k.
Centroid of tetrahedron OABC is Statement I is true.
p p p Statement II Direction ratios of the straight line are
( x, y , z ) =  , , 
 4l 4m 4n  < bc′ − kbc, kac − ac′, 0 > direction ratios of normal to be plane
< 0, 0, 1 >
Using, l 2 + m 2 + n 2 = 1
Now, 0 × (bc′ − kbc ) + 0 × (kac − ac′ ) + 1 × 0 = 0
16
x 2y 2 + y 2z 2 + z 2x 2 = 2 x 2y 2z 2 ∴The straight line is parallel to the plane.
p Statement II is the true but does not explain Statement I.
1 1 1 16
⇒ + + = 129. Equation of the plane, perpendicular to the plane P and
x2 y 2 z 2 p2 containing line L is 8 x + y − 7z = 4.
p p p 130. L1 and L2 are obviously not parallel.
Put x = sec α sec β, y = secα cosec β , z = cosec α
4 4 4 Consider the determinent
1 4 1 4 1 4
= cosα cosβ, = cosα sin β, = sinα  2 −4 1 
x p y p z p
D =
 2 4 −3 

2
 1
2
 1  1
2
16 1 3 2 
  +  +  = 2
 x y   z  p = 2(8 + 9 ) + 4( 4 + 3 ) + 1(6 − 4 )
[cos2 α cos2 β + cos2 α sin 2 β + sin 2 α ] = 34 + 28 + 2
16 16 D ≠ 0 ⇒ Skew
= 2 [cos2 α + sin 2 α ] = 2
p p Hence, Statement I is false.

125. Statement I PA ⋅ PB = 9 > 0 131. n = a × b. Equation of the plane


n
∴P is exterior to the sphere a a
Statement II is true (standard result)
R (r)
 i$ $j k$  b
126. Statement II r × ( i + 2 j − 3 k ) =  x y z  b
 
−3  ( r − a ) ⋅ (a × b ) = 0
1 2
[ r a b] = 0
264 Textbook of Vector & 3D Geometry

132. Statement II is not true because image of P in a plane is a point 138. If l , m, n denote the direction ratios of L1 and l + m − n = 0 and
M such that PM is perpendicular to the plane and the l − 3m + 3n = 0 ⇒l = 0, m = n
mid-point of PM lies on the plane.
⇒ direction ratios of L1 are 0, 1, 1 similarly for L2 and L3, we
The point A, B, C are respectively ( −a, b, c ), (a, − b, c ) and find that the direction ratios of both are 0, 1, 1 showing that L1,
(a, b, − c ) L2, L3 are parallel, thus Statement I is False.
x y z
which lie on the plane + + = 1 and thus Statement I is Statement II is True, because solving the given equation we get
a b c −2
true. x = 0, y − z = − 1 and y − z = which is not possible.
3
133. In Statement II, let r be the position vector of the point on the Solution (Q. Nos. 139-142)
locus, then
| r − a | = | r − b | ⇒ (r − a )2 = (r − b)2 139. Here, ∆ABC is an isosceles with AB = AC
⇒ | r | 2 + | a | 2 − 2r ⋅ a = | r | 2 + | b | 2 − 2r ⋅ b So, internal bisector of A is perpendicular to BC.
A (1, 2, 3)
⇒ 2 r ⋅ (a − b ) + | b | 2 − | a | 2 = 0
⇒ 2 r ⋅ (a − b ) + ( b − a ) ⋅ ( b + a ) = 0
 a + b
⇒ r −  ⋅ (a − b ) = 0 3 3
 2 
Showing that Statement II is true using it for Statement I.
we get the required locus as B(0, 0, 1) M √2 C(–1, 1, 1)
 3 $i − 2 $j + 5 k$ + $i + 2 $j + k$ 
r − 2  ∆AMB ≅ ∆AMC (RHS rule)
 
M is mid-point of BC.
⋅(3 $i − 2 $j + 5 k$ − ( $i + 2 $j − k$ ) = 0
 −1 −1 
⇒ [ r − (2 $i + 2 k$ ) ⋅ (2 $i − 4 $j + 6 k$ )] = 0 So, M≡ , , 1
 2 2 
⇒ r ⋅ ( i$ − 2 $j + 3 k$ ) = 2 × 1 − 0 × 2 + 2 × 3 = 8
∴Equation of internal bisector through A to side BC is
and thus Statement I is also true. 3 3 
r = ( i$ + 2 $j + 3 k$ ) + µ  $i + $j + 2 k$ 
134. Since a and c are non-collinear. Equating the coefficients of a 2 2 
and c in the two values of r we get.
⇒ r = ( i + 2 j + 3 k ) + µ(3 i + 3 j + 4 k )
$ $ $ $ $ $
6 − λ = 1 + µ, 2 λ − 1 = 3µ − 1 ⇒ λ = 3, µ = 2
So there exist values for λ and µ such that the two values of r Aliter Equation of BC is r = k$ + λ ( $i − $j)
are same showing that the lines intersect and hence they are Let position vector of M on BC be r.
coplanar. Thus, statement I and statement II both are true and Now, AM = Position vector of M − Position vector of A
the first follows from the second.
= ( λ$i − λ$j + k$ ) − ( $i + 2 $j + 3 k$ )
 1 − 2 0 + 1 −1 − 0   −1 1 −1 
Since, −1 1    = ( λ − 1 ) $i − ( λ + 2 ) $j − 2 k$
135.  1 =  1 −1 1  = 0
−1
 1 2 3  1 2 3  Since, AM ⋅ ( i$ − $j) = 0 ⇒ λ =
2
The lines in Statement I are coplanar and equation of the plane −1 $ 1 $ $
containing them is Position vector of point, M = i+ j+ k
2 2
x − 1 y z + 1
Equation of internal bisector through A to the side BC is
 1 −1 1 
  = − (5 x + 2y − 3z − 8 ) = 0 3 3 
 1 2 3  r = ( i$ + 2 $j + 3 k$ ) + µ  $i + $j + 2 k$ 
2 2 
So Statement I is true. ⇒ r = ( i$ + 2 $j + 3 k$ ) + µ(3 $i + 3 $j + 4 k$ )
1 2 3
Also, Statement II is true because = = and 1 + 2 − 3 = 0 140. Now, equation of AC is
3 6 9
But does not lead to Statement I. r = ( i$ + 2 $j + 3 k$ ) + λ (2 $i + $j + 2 k$ )
136. Any point on the first line is (2x1 + 1, x1 − 3, − 3x1 + 2). A (1, 2, 3)

Any point on the second line is (y1 + 2, − 3y1 + 1, 2y1 − 3 )


N
If two lines are coplanar, then 2 x1 − y1 = 1, x1 + 3y1 = 4 and 3
3 x1 + 2y1 = 5 are consistent.
2 1 −3
137. The direction cosines of segment OA are , and .
14 14 14 B(0, 0, 1) C(–1, 1, 1)
OA = 14
Also, BM = (1 + 2 λ ) $i + (2 + λ ) $j + 2(1 + λ ) k$
This means OA will be normal to the plane and the equation of
the plane is 2 x + y − 3z = 14. BM ⋅ (2 $i + $j + 2 k$ ) = 0
Chap 03 Three Dimensional Coordinate System 265

⇒ 2(1 + 2 λ ) + (2 + λ ) + 4(1 + λ ) = 0 Solution (Q. Nos. 145-148)


−8 2 2
145. | AG | 2 =   + +   =
⇒ λ= 5 1 5 51
9  3   9 3 9
−7 i$ + 10 $j + 11 k$
Position vector of N = | AG | =
51
9 3
Equation of altitude through B to side AC is
146. AB = − 4$i + 4$j + 0k$
 7 10 11 
r = k$ + t  − $i + $j + k$ − k$  AC = 2 $i + 2 $j + 2 k$
 θ θ θ 
D(0, –5, 4) 4, 1 , 8
r = k$ + t( −7 $i + 10 $j + 2 k$ ) 3 3 3
2
141. Clearly, mid-point L of AB is  , 1, 2 .
1 h
2  G

Equation of median through C to AB is A


(3, 0, 1) C (5, 2, 3)
1
A (1, 2, 3)

1 ,1,2
2 B (–1, 4, 1)

 $i $j k$ 
∴ AB × AC = − 8 1 −1 0 
B(0, 0, 1) C(–1, 1, 1)  
1 1 1
3 
r = ( − $i + $j + k$ ) + p  $i + k$  = − 8( − i$ − $j + 2 k$ ) = 8( i$ + $j − 2 k$ ) = n
2 
1
⇒ r = ( − $i + $j + k ) + p(3 $i + 2 k )
$ $ ∴Area of ∆ABC = | AB × AC | = 4 6
2
32 + 32 − ( 2 )2
142. We have, cos A = 147. h = | Projection of AD on n |
2(3 )(3 )
AD = − 3 $i − 5 $j + 3 k$
16 8
∴ cos A = =
= 
 AD ⋅ n  $ 
 ( −3 i − 5 j + 3 k )( i + j − 2 k ) 
$ $ $ $ $
18 9 =
1  | n|   6 
Now, area ( ∆ ABC ) = (3 )(3 ) −3 − 5 − 6  14
2 = =
9 64  6  6
sin A = 1−
2 81 148. Equation of the plane ABC
9 17 17 A( x − 3 ) + By + (z − 1 ) = 0
= × = sq units
2 9 2 where, A = 1, B = 1, C = − 2
Solution (Q. Nos. 143-144) ∴ x − 3 + y − 2z + 2 = 0
x −1 y −2 z −3 x + y − 2z = 1
143. Line = = =r
3 −1 4 Solution (Q. Nos. 149-151)
Any point B ≡ 3r + 1, 2 − r, 3 + 4r (on the line L) 149. Line L1 is parallel to a = $i + 2$j + 3k$
AB = 3r, −r , 4r + 6 Line L2 is parallel to b = 3 i$ + $j + 2 k$
Hence, AB is parallel to x + y − z = 1.
Normal to the plane perpendicular to line L1 and L2 is
Hence, 3r − r − 4r − 6 = 0
a × b = ( i$ + 7 $j − 5 k$ )
2r = − 6 ; r = − 3
Hence, B is ( −8, 5, − 9 ) and plane passes through the point with positive vector
3 5
144. Equation of plane containing the line L is = $i + $j + 2 k$
2 2
A( x − 1 ) + B(y − 2 ) + C (z − 3 ) = 0
Equation of plane is r ⋅ ( i$ + 7 $j − 5 k$ ) = 9
where, 3 A − B + 4C = 0 …(ii)
∴Eq. (i) also contains the point A(1, 2, − 1 ) 150. Angle bisector of vector a and b is,
Hence, C = 0, 3A = B 1 $ $ $
r1 = (2 i − j − k )
Equation of plane x − 1 + 3(y − 2 ) = 9 14
x + 3y − 7 = 0 1
and r2 = ( 4 $i + 3 $j + 5 k$ )
14
266 Textbook of Vector & 3D Geometry

Hence, the plane with either (2, − 1, − 1 ) or ( 4, 3, 5 ) as the x m + nl sin θ1


= =
direction ratio of normal and passing through (2, − 3, 2 ) is the y l + mn sin θ 2
required plane. x y z
x −2 y + 3 3 −2 = =
. ∴ Equation of line is = = sin θ1 sin θ 2 sin θ 3
2 −1 −1
x y z
x −2 y + 3 3 −2 = =
and = = 1 −l 2
l −m 2
1 − n2
4 3 5
x −2 x −2 y + 3 z −2 152. (b) 153. (c) 154. (a)
= y + 3 = z − 2 or = =
2 4 3 5 Solution (Q. Nos. 155-157)
151. ∴ Equation of required plane is 155. Let the position vector of L be a + λb
r ⋅ (3 $i + $j + 2 k$ ) = ( $i + 2 $j + 3 k$ )
= (6 + 3 λ ) i$ + (7 + 2 λ ) $j + (7 − 2 λ ) k$
⇒ r ⋅ (3 $i + $j + 2 k$ ) – 11 = 0
So, PL = (6 + 3 λ ) $i + (7 + 2 λ ) $j + (7 − 2 λ ) k$ − ( $i + 2 $j + 3 k$ )
11 11
∴ Required distance = = = (5 + 3 λ ) $i + (5 + 2 λ ) $j + ( 4 − 2 λ ) k$
9+1+ 4 14
Since, PL is perpendicular to the given line which is parallel to
n Solutions (Q. Nos. 152-154) b = 3 i$ + 2 $j − 2 k$
The three plane intersect in a straight line. All three plane pass ⇒ 3 (5 + 3 λ ) + 2 (5 + 2 λ ) − 2 ( 4 − 2 λ ) = 0
through origin (clearly). ⇒ λ = − 1 and thus the position vector of L is 3 $i + 5 $j + 9 k$
 1 −n −m 
 n −1 l  = 1(1 − l 2 ) + n( −n − lm ) − m(nl + m ) 156. Let the position vector of Q, the image of P in the given line be
  x1$i + y1$j + z1k$ , then L is the mid-point of PQ.
m l −1 
$i + 2 $j + 3 k$ + x1$i + y1$j + z1k$
1 = l 2 + m 2 + n 2 + 2lmn ⇒ 3 i$ + 5 $j + 9 k$ =
2
Let l = cos θ1, m = cos θ 2, n = cos θ 3 [since, l , m, n ∈ ( 0, 1 )] x1 + 1 y1 + 2 z1 + 3
⇒ = 3, = 5, =9
cos2 θ1 + cos2 θ 2 + cos2 θ 3 + 2 cosθ1 cosθ 2 cosθ 3 = 1 2 2 2
cos2 θ1 + (2 cosθ 2 cosθ 3 ) cosθ1 + cos2 θ 2 + cos2 θ 3 − 1 = 0 ⇒ x1 = 5, y1 = 8, z1 = 15
cos θ1 ⇒ Image of P in the line is (5, 8, 15)
−2 cosθ 2 cosθ 3 ± 4 cos2 θ 2 cos2 θ 3 − 4 cos2 θ 2 − 4 cos2 θ 3 + 4 1
=
157. Area of the ∆PLA = PL AL
2
2
1 $
= − cosθ 2 cosθ 3 ± 1 − cosθ 2 1 − cos2 θ 3 = 2 i + 3 $j + 6 k$ − 3 $i − 2 $j + 2 k$
2
⇒ cosθ1 = (cosθ 2 cosθ 3 − sin θ 2 sin θ 3 ) 1 7 17
= 4 + 9 + 36 9 + 4 + 4 = sq units.
⇒ cosθ1 = − cos(θ 2 + θ 3 ) 2 2
θ1 + θ 2 + θ 3 = π Solution (Q. Nos. 158-160)
y − lz
ny + mz = 158. Let P (x, y , z ) be any point on the locus then 3PA = 2PB
n
n y + mnz = y − lz
2 ⇒ 9 ( PA ) 2= 4 ( PB ) 2

(1 + mn )z = (1 − n 2 )y ⇒ 9 [( x + 2 ) 2 + (y − 2 ) 2 + (z − 3 ) 2 ]

z 1 − n2 = 4 [( x − 13 ) 2 + (y + 3 ) 2 + (z − 13 ) 2 ]
=
y 1 + mn ⇒5 ( x 2 + y 2 + z 2 ) + 140 x − 60y + 50z − 1235 = 0
x − ny y − nx ⇒ x 2 + y 2 + z 2 + 28 x − 12y + 10z − 247 = 0
=
m 1 159. The required coordinates are
lx − nly = my − mnx  2 × 13 + 3 ( −2 ) 2 × ( −3 ) + 3 (2 ) 2 × 13 + 3 × 3
(l + mn ) x = (m + nl )y  , ,  = ( 4, 0, 7 )
 2+3 2+3 2+3 
y l + mn
=
x m + nl 160. Direction ratios of AB are 13 + 2, − 3 − 2, 13 − 3
y − lz z − ly i.e. 15, − 5, 10
=
n m Let the equation of the required line L be
my − mlz = nz − nly ⇒ (m + nl )y = (n + ml )z x + 2 y −2 z −3
= =
z m + nl sin θ1 sin θ 3 sin θ 3 l m n
= = =
y n + ml sin θ 2 sin θ1 sin θ 2 then 15l − 5m + 10n = 0
which satisfied by (c)
Chap 03 Three Dimensional Coordinate System 267

Solution (Q. Nos. 161-163) x y z


= =
161. Equation r = a + tn$ is line passing through a and parallel to n$ . 3 −1 1
This will meet the plane r ⋅ n$ = d at point for which 11 11 11
(a + tn$ ) . n$ = d ⇒ t = d − a . n$ 171. The point on the line at a distance 11 from the origin is the
Required distance = (a + (d − a. n$ ) n$ − a = d − a. n$ required point and it is (3, − 1, 1 )
162. Foot of the perpendicular from the point A to plane r. n$ = d Solution (Q. Nos. 172-174)
A (2, 1, 0)
= a + (d − a. n$ ) n$
163. Let b be position vector of image of a B (1, 0, 1)
b+a
∴ = a + (d − a. n$ ) n$
2 P (x, y, z)
b = a + 2(d − a. n$ ) n$
Solution (Q. Nos. 164-166) C (3, 0, 1)
164. The centre of the sphere is at the mid-point of the extremities x −2 y −1 z
 3 3 3
of a diameter ⇒ the centre  − , , −  −1 −1 1 = 0
 2 2 2
2 2 2
1 −1 1
 7  9  11
and hence the radius =   +   +   ( x − 2 )[( − 1 ) − (1 )] − (y − 1 ) [( − 1 ) − 1 ] + z [1 + 1 ] = 0
 2  2 2
2(y − 1 ) + 2z = 0
165. Equation of the circle can be written as ⇒ y + z −1 = 0
x 2 − 16 x + y 2 − 9 + z 2 = 0 The vector normal to the plane is r = 0 i$ + $j + k$
or x 2 + y 2 + z 2 = 25 The equation of the line through (0, 0, 2) and parallel to n is
166. Distance of the point (3, 6, − 4) from the given plane is equal to r = 2 k$ + λ ( $j + k$ )
the radius of the sphere ⇒ the radius of the sphere The perpendicular distance of D( 0, 0, 2 ) from plane.
(3 $i + 6 $j − 4 k$ ) ⋅ (2 $i − 2 $j − k$ ) − 10 6 − 12 + 4 − 10 172. (b) 173. (c) 174. (d)
= = =4
4+4+1 3 x −1 y − 0 z −2
175. (A) L1 : = = ; V1 = $i + $j − 5 k$ ;
1 1 −5
Solution (Q. Nos. 167-168)
λ − 1 µ + 2 x −2 y −1 z + 3
167. Mid-point of BC =  , 
L2 :
2
=
2
=
− 10
; V2 = 2( $i + $j − 5 k$ )
 2 2 
 λ − 5 µ − 8 Hence, lines are parallel and both contains the points (1, 0, 2)
DR’s AD =  , 1,  and (2, 1, − 3) Coincident lines both L1 and L2 may lie in an
 2 2 
infinite number of planes.
AD is equally inclined to axes ⇒ λ = 7, µ = 10, 2 λ − µ = 4
V = 2 i$ + 2 $j − k$ 
168. A(2, 3, 5) B (− 1, 2, 3) C (7, 5, 10) (B) 1  ⇒ Lines not parallel
V2 = i$ − 2 $j + 3 k$ 
Projection of AB = − 3 $i − 3 k$ on BC = 8 $i + 2 $j + 8 k$ Also, both intersect at (3, 5, 1)
AB ⋅ BC − 8 3 Hence, lines are intersecting, hence they lie on a unique plane.
=
| BC | 11 x − 0 y −1 z − 0
(C) L1 : = = =t
Solution (Q. Nos. 169-171) −6 9 −3
169. Horizontal plane P1 is of the form x −1 y − 4 z − 0
L2 : = = =s
2 −3 1
r ⋅ n1 = 0, where n1 = ( 4, − 3, 7 ) L1 is parallel to − 6 i$ + 9 $j − 3 k$ 
Plane P2 is of the form r ⋅ n 2 = 0, where n2 = (2, 1, − 5 ) 
L2 is parallel to 2i$ − 3 $j + k$ 
The vector b along the line of interaction ⇒ Lines parallel but not coincident.
= n1 × n 2 = ( 4, 17, 5 ) = n3 (say) Since, (0, 1, 0) does not lie on L2, not intersecting.
Since the line of greatest slope is perpendicular to n 2 and n 3, Hence L1, L2 lies in unique planes.
the vector along the line of greatest slope
(D) Lines are skew can be verified.
= n 2 × n 3 = (3, − 1, 1 ) = n 4
x y −3 z + 2
 3 −1 1  176. L1 : = = …(i) (passing through P and Q)
and n4 =  , ,  3 4 1
 11 11 11 
x −1 y −3 z + 1
L2 : = = …(ii)
170. Since (0, 0, 0) is a point on both the planes, it is a point on the 1 0 1
line of intersection and hence the equation of a line of greatest (passing through R and parallel to v = $i + k$ )
slope is
268 Textbook of Vector & 3D Geometry

(A) Distance of P( 0, 3, − 2 ) from L2 Distance from (0, 0, 0) of the plane


PN = (t + 1 ) $i − 6 $j + 2(t − 1 ) k$ 19 19
d = =
Now, PN ⋅ V = 0 25 + 1 + 121 147
[(t + 1 ) $i − 6 $j + (t + 1 ) k]
$ ⋅ ( $i + k$ ) = 0
177. (A) 3 ⋅ 1 − 2(−2) + 5(λ ) = 0
P (0, 3, –2) 7
⇒ λ=−
5
(B) Point (3, λ , µ ) lies on
N 2x + y + z − 3 = 0
L2
[(t+1),–3, t–1] = x − 2y + z − 1
v=i+0j + k 3 ⋅ 2 + λ + µ − 3 = 0 and 3 − 2 λ + µ − 1 = 0
(t + 1 ) + (t + 1 ) = 0 ; t = − 1 λ + µ + 3 = 0 and 2 λ − µ − 2 = 0
Hence, PN = 6 $j So, λ + µ = −3
1 ⋅ 4 + 1( −3 ) + 1 ⋅ 5 6
PN = | − 6 $j | = 6 (C) sin θ = =
(B) Distance between L1 and L2 1 + 1 + 1 16 + 9 + 25
2 2 2 3 50
6
P (0, 3, –2) θ = sin −1
25
1 ⋅ 3 + 1( −4 ) + 1 ⋅ 5 4
n=5i – j – 11k (D) cos θ = =
b 3 16 + 9 + 25 3 50
8
θ = cos−1
75
Q (3, 7, –1) a R (1, –3, –1) 1 3 −5
Equation of plane containing L1 and parallel to L2 178. 3 −k −1 = 1(12k + 2 ) − 3( −36 + 5 ) − 5(6 + 5k )
Ax + B(y − 3 ) + C (z + 2 ) = 0 5 2 −12
where 3A + 4B + C = 0 = 12k + 2 + 108 − 15 − 30 − 25k = 0
And A+B+C =0 k =5
A+C =0 L1, L2 and L3 are concurrent for k = 5.
C = λ, A = − λ, B = + λ /2 1 3
Slope of L1 = − , Slope of L2 = ,
∴ Equation of plane 3 k
λ 5
− λx + (y − 3 ) + λ (z + 2 ) = 0 Slope of L3 = −
2 2
2 x − y + 3 − 2z − 4 = 0 3 1
=− ⇒ k = −9
2 x − y − 2z = 1 …(i) k 3
Now, distance of the point (1, − 3, − 1 ) lying on the line L1 from 3 5 6
the plane (i) =− ⇒k=−
k 2 5
2 + 3 + 2 −1
d = =2 L1, L2 and L3 form a triangle, if they are non-concurrent or any
3
(C) Area of ∆PQR two out or three are not parallel.
6
QR = a = 2 i$ + 10 $j + 0 k$ k ≠ − 9, − , 5
5
QP = b = 3 i$ + 4 $j + k$ 5
k = and 0 will be the values for which L1, L2 and L3 form a
$i $j k$ 6
a × b =2 1 5 0 triangle.
abc
3 4 1 179. Given, = 32, where A, B and C are respectively, (a, 0, 0 ),
6
= 2 [ i$(5 ) − $j(1 ) + k$ ( 4 − 15 )] = 2[5 i$ − $j − 11 k$ ] ( 0, b, 0 ), ( 0, 0, c ).
|a×b| a b c 
= 25 + 1 + 121 = 147 = 3 ⋅ 49 = 7 3 (A) Centroid of tetrahedron [α , β, γ ] =  , , 
2  4 4 4
(D) Distance of ( 0, 0, 0 ) from PQR a = 4α , b = 4β, c = 4 γ
Equation of plane PQR is (r − p ) ⋅ n 64αβγ = 32 × 6
= [ x$i + (y − 3 ) $j + (z + 2 ) k$ ] ⋅ [5 $i − $j − 11 k$ ] αβγ = 3
= 5 x − (y − 3 ) − 11(z + 2 ) = 0 a b c
(B) Equidistant point (α , β, γ ) ≡  , , 
= 5 x − y − 11z − 19 = 0  2 2 2
Chap 03 Three Dimensional Coordinate System 269

a = 2α , b = 2β, c = 2 γ (B) Let the required sphere be


8 αβγ = 32 × 6 x 2 + y 2 + z 2 + 2ux + 2vy + wz + d = 0 …(1) substituting given
αβγ = 24 points then we get 1 + 2u + d = 0
x y z 1 + 2v + d = 0 and 1 + 2w + d = 0
(C) The equation of the plane is + + = 1
a b c 1+d
⇒ u =v =w =
∴Foot of the perpendicular from the origin 2
 1 /a 1 /b 1 / c  If R be the radius of the sphere,
≡ (α , β, γ ) ≡  , , 2 then R 2 = u2 + v 2 + w 2 − d
 ∑1 / a ∑1 / b ∑1 / c 
2 2

covert above equation in terms of d differentiate, equate to


1 1 1
= = =t zero solve for d.
aα bβ c γ (C) Let the given points be A, B and C respectively.
1 1 1 1
where, t= 2 + 2 + 2 =∑ 2 Then find AB, AC , BC and then apply AB 2 + AC 2 = BC 2 then
a b c a solve for the λ.
or t = (α 2 + β 2 + γ 2 )t 2 (D) Any point on the line is (1 − r , r + 1, r )
t= 2
1 The direction ratio of the line joining (1, 3, 4) & (1 − r , r + 1, r )
α + β2 + γ 2 is − r , r − 2, r − 4
α 2 + β2 + γ 2 α 2 + β2 + γ 2 ∴ ( −1 )( −r ) + 1 ⋅ (r − 2 ) + (r − 4 ) = 0
and α= ,b = , r + r − 2+ r − 4 = 0
α β
3r = 6 ⇒ r = 2
α 2 + β2 + γ 2
c= ∴Foot of the perpendicular is ( − 1, 3, + 2 )
γ
∴Distance = (2 ) 2 + 0 + 4 = 2 2
Now, abc = 6 × 32
∴ d =2 2
(α 2 + β 2 + γ 2 ) = 192 αβγ
(D) Let P be (α , β, γ ), then PA ⊥ PB d 2 2 2
= =
⇒ a (α − a ) + β(β − b ) + γγ = 0 2 3 2 3 3
⇒ aα + bβ = α 2 + β 2 + γ 2 181. The solid diagonals may be taken as the lines join (0, 0, 0),
(a, a, a ) and (a, a, 0 ) and ( 0, 0, a ). The direction ratios will be
PB ⊥ PC
a, a, a,; a, a, − a.
⇒ aα + b(β − b ) + c( γ − c ) = 0
a2 + a2 − a2 1 1
⇒ bβ + cγ = α 2 + β 2 + γ 2 ⇒ cosθ = ⇒ θ = cos−1
2 3 3
3a × 3a
2
a b c
∴ = =
1 /α 1 /β 1 / γ Let us take the solid diagonal as the one joining (0, 0, 0) (a, a, a )
and plane diagonal as joining (0, 0, 0) and (a, a, 0 ). We easily
α 2 + β2 + γ 2 α 2 + β2 + γ 2 2
a= ,b = get the angle as cos−1 .
2α 2β 6
α 2 + β2 + γ 2  1
and c= The third part is easily found as cos−1  
2γ  2
∴ abc = 6 × 32 Hence, matching follows ( A ) → ( r ) ; ( B ) → ( p ) ; ( C) → ( q )
⇒ (α 2 + β 2 + γ 2 ) = 192 × 8αβγ 182. (i) Shortest distance
= 1536αβγ OB ⋅ OA × BC ( i$ + $j) ⋅ i$ × ( $j × k$ ) 1
= = =
180. Let O(0, 0, 0), A(3, 4, 7) and B(5, 2, 6) be the given point OA × BC $i × ( $j + k$ ) 2
1
Area of ∆OAB = OA ⋅ OB sin( ∠AOB ) ⇒ 2m = 1
2
183. The length of the edges are given by a = 5 − 2 = 3, b = 9 − 3 = 6
Now, OA = 3 2 + 4 2 + 7 2 = 74 and c = 7 − 5 = 2, so length of the diagonal
OB = 5 + 2 + 6 = 65
2 2 2
= a 2 + b 2 + c 2 = 9 + 36 + 4 = 7 units
Also dc’s of the line OA and OB are 184. Foot of perpendicular r from (6, 5, 8) on y-axis is (0, 5, 0).
3 4 7 5 2 6
= , , and , , Required distance
74 74 74 65 65 65
1 3 3 = (6 − 0 ) 2 + (5 − 5 ) 2 + (8 − 0 ) 2 = 10
∴ Required area × 74 × 65 × = 65 10
2 74 2 ⇒ 5 λ = 10 ⇒ λ ⇒ =2
5
270 Textbook of Vector & 3D Geometry

185. Given lines are ⇒ Distance of centroid from origin is


r = (3, 8, 3 ) + λ ( −3, 1, 1 ) 1 2 + 25 + 49 = 75 = 5 3 ⇒ λ = 3
and r = ( −3, − 7, 6 ) + µ( −3, 2, 4 )
193. Equating the distances of circumcentre (−1, λ, − 3) from
where, λ and µ are parameters
(3, 2, − 5 ) and ( −3, 8, − 5 ) we get
Shortest distance
2 2 + ( λ + 2 ) 2 + ( −3 + 5 ) 2 = ( −1 + 3 ) 2 + ( λ − 8 ) 2 + ( −3 + 5 ) 2
( −3 − 3, − 7 − 8, 6 − 3 ) ⋅ [(3, − 1, 1 ) × ( −3, 2, 4 )]
= ⇒ λ=4
(3, − 1, 1 ) × ( −3, 2, 4 )
Note : Verify
( −6, − 15, 3 ) ⋅ ( −6, − 15, 3 )
= (i) ( −1, λ , − 3 ) is at the same distance from third vertex.
36 + 225 + 9
(ii) ( −1, λ , − 3 ) lies on the plane containing three points
= 270 = 3 30 units = λ 30 (3, 2, − 5 ); ( −3, 8, − 5 ) and ( −3, 2, 1 ).
∴ λ =3 194. D.R’s of P1P2 = (k, − 1, 3)
186. Given planes are D.R’s of P2P3 = (2, k, − 1 )
x − cy − bz = 0 …(i) Q P1P2 ⊥ P2P3
cx − y + az = 0 …(ii) ∴ k(2 ) − k − 3 = 0 : k = 3
bx + acy − z = 0 …(iii) 195. A plane containing line of intersection of the given planes is
Equation of plane passing through the line of intersection of x − y − z − 4 + λ( x + y + 2z − 4 ) = 0
planes (i) and (ii) may be taken as
i.e., ( λ + 1 ) x + ( λ − 1 )y + (2 λ − 1 )z − 4( λ + 1 ) = 0
( x − cy − bz ) + λ (cx − y + az ) = 0 …(iv)
vector normal to it
Now, eliminating λ we get
V = ( λ + 1 ) i$ + ( λ − 1 ) $j + (2 λ − 1 ) k$
a 2 + b 2 + c 2 + 2abc = 1
Now the vector along the line of intersection of the planes
λ ( −1 ) + 1 × 3
187. We must have = 0 ⇒ λ =3 2 x + 3y + z − 1 = 0
λ+1
and x + 3y + 2z − 2 = 0 is given by
∴ λ =3 $i $j k$
188. The coordinates of vertices of projected triangle will be n = 2 3 1 = 3( $i − $j + k$ )
A′ ( −1, 1, 0 ), B′ (1, − 1,0 ), C ′ (1, 1, 0 )
−1 1 1 1 3 2
1
area of triangle = 1 − 1 1 (Two dimension area formula) As n is parallel to the plane (i), therefore n ⋅ V = 0
2
1 1 1 ( λ + 1 ) − ( λ − 1 ) + (2 λ − 1 ) = 0
−1
= 2 square units. 2 + 2λ − 1 = 0 ⇒ λ =
2
189. Plane must pass through Hence, the required plane is −
x 3y
− 2z − 2 = 0
 1 − 3 5 + 1 7 − 1 2 2
 , ,  or ( −1, 3, 3 )
 2 2 2  x − 3y − 4z − 4 = 0
⇒ −1 + 3 + 2 × 3 = λ ⇒ λ = 8 Hence | A + B + C–4| =7
190. x + y + z = square of distance from origin
2 2 2 196. Clearly, minimum value of a 2 + b 2 + c 2
4 sin 2 t + 4 cos2 t + 9t 2 = 4 + 9t 2  3( 0 ) + 2( 0 ) + ( 0 ) − 7  49 7
= = = units
which is shortest at t = 0  (3 ) 2 + (2 ) 2 + (1 ) 2  14 2
 
⇒ Shortest distance = 2
197. 4 x + 7y + 4z + 81 = 0 …(i)
191. The point (−1, λ, − 2) must be lie on the plane 5 x + 3y + 10z = 25 …(ii)
2 x − 2y + z + 12 = 0 Equation of plane passing through their line of intersection is
− 2 − 2 λ − 2 + 12 = 0 ( 4 x + 7y + 4z + 81 ) + λ (5 x + 3y + 10z − 25 ) = 0
λ=4 or ( 4 + 5 λ ) x + (7 + 3 λ )y + ( 4 + 10 λ )z + 81 − 25 λ = 0 …(iii)
We can easily show that the distance of ( −1, 4, − 2 ) from centre plane (iii) ⊥ to (i), so
of the sphere (1, 2, − 1 ) is equal to its radius. 4( 4 + 5 λ ) + 7(7 + 3 λ ) + 4( 4 + 10 λ ) = 0
∴ λ = −1
a+1+2+0 2 + b +1 + 0
192. 1 = ,2 = , From (iii), equation of plane is
4 4
− x + 4y − 6z + 106 = 0 …(iv)
3+2+c+0
3= Distance of (iv) from (0, 0, 0)
4 106 106
⇒ a = 1, b = 5, c = 7 = =
1 + 16 + 36 53
Chap 03 Three Dimensional Coordinate System 271

198. Line through point P (−2, 3, − 4) and parallel to the given line Applying C 2 → C 2 + C1 and C 3 → C 3 + C1, then
x + 2 2y + 3 3z + 4 −1 0 0
= =
3 4 5 ∆1 = 3 5 0 = − 5( λ − 2 ) … (ii)
y +
3
z+
4 λ λ + 1 λ −2
x+2 2 3
is = = =λ 1 1 1
3 2 5
3 ∆2 = λ 2 − 3
 3 5 4 3 1 −2
Any point on this line is Q 3 λ − 2, 2 λ − , λ − .
 2 3 3  Applying C 2 → C 2 − C1 and C 3 → C 3 − C1, then
 4λ − 9 5λ + 8  1... 1... 0
Direction ratios of PQ are 3 λ , ,
 2 3  ∆ 2 = λ 2 − λ − 3 − λ = 3 λ − 16 … (iii)
Now, PQ is parallel to the given plane 4 x + 12y − 3z + 1 = 0 3 −2 −5
⇒ line is perpendicular to the normal to the plane 1 −1 1
 4λ − 9  5λ + 8 ∆3 = λ 3 −3
⇒ 4(3 λ ) + 12   − 3  =0
 2   3  3 λ −2
 5  Applying C 2 → C 2 + C1 and C 3 → C 3 − C1, then
⇒ λ = 2 ⇒ Q  4, , 2
 2 
1... 1... 0
2
5  17 ∆ 3 = λ 3 + λ − 3 − λ = ( λ + 3 ) ( λ − 2 ) … (iv)
⇒ PQ = (6 ) 2 +  − 3 + (6 ) 2 =
2  2 3 3+λ −5
199. The given points are O(0, 0, 0), A(0, 0, 0), B(0, 4, 0) and C(6, 0, 0) If the given planes form a triangular prism, then we know that
∆ 4 = 0 and none of ∆1, ∆ 2, ∆ 3 is zero. Here from Eqs. (i), (ii),
Here, three faces of tetrahedron are xy , yz , zx plane.
(iii) and (iv) we find that if λ = 4, then ∆ 4 = 0 and none of ∆1,
Since point P is equidistance from zx , xy and yz planes, its ∆ 2, ∆ 3 is zero. Consequently for λ = 4, then given planes form
coordinates are P (r , r , r ) a triangular prism.
Equation of plane ABC is
201. 7x + 6y + 2z = 272 and x − y + z = 16
2 x + 3y + 6z = 12 (from intercept form)
8
P is also at distance r from plane ABC ⇒ 5 x + 8y = 240 ⇒ x = 48 − y
5
2r + 3r + 6r − 12
⇒ = r ⇒ | 11r − 12 | = 7r Let y = 5λ, λ ∈ I ⇒ x = 48 − 8λ
4 + 9 + 36
and z = 16 + y − x = 13 λ − 32
12
⇒ 11r − 12 = ± 7r ⇒ r = ,3 But x > 0, y > 0 and z > 0 ⇒ 48 − 8 λ > 0 ⇒ λ >
48
18 8
∴ r = 2 /3 (as r < 2) 32
⇒ λ ≤ 5 and 13 λ − 32 > 0 ⇒ λ>
200. The equation of the given planes can be written as 13
x −y + z + 1 = 0 ⇒ λ ≥3
λxz + 3y + 2z − 3 = 0 ∴ λ ∈[3, 5 ]
3 x + λy + z − 2 = 0 ∴ Z min = 39 − 32 = 7 ⇒ x = 24, y = 15
The rectangular array is ∴ x + y + z − 42 = 4
1 −1 1 1 202. The given two lines are intersect each other, then
λ 3 2 −3 = 0 a1α + b1β + c1γ + d1 a 2α + b2β + c 2γ + d 2
=
3 λ 1 −2 a1l + b1m + c1n a 2l + b2m + c 2n
1 −1 1 − 2d 2 −d2
⇒ =
∴ ∆4 = λ 3 2 sin A + sin B + sin C sin 2 A + sin 2 B + sin 2C
3 λ 1 A B C 1
⇒ sin sin sin =
Applying C 2 → C 2 + C1 and C 3 → C 3 + C1, then 2 2 2 16
1 ...0 ...0 k k −1 2
∆ 4 = λ 3 + λ 2 − λ = (λ − 4) (λ + 3) … (i) 203. [ c − a b c] = 0 ⇒ 1 k −1 = 0
3 3 + λ −2 2k 3k − 1 k
⇒ k 3 − 4k 2 + 8k − 2 = 0
−1 1 1
Here f ′ (k ) = 3k 2 − 8k + 8 > 0 ∀ k ∈ R
Also, ∆1 = 3 2 − 3
(Q Its discriminate is negative)
λ 1 −2
∴ The equation has only one real root.
272 Textbook of Vector & 3D Geometry

204. Taking O as the origin, let the position vectors of A, B and C ∴ Direction cosines are
be a, b, c respectively. Then the position vectors of G1, G2 and  1 1 2  1 1 2
 , , ,  , ,− 
G3 are  6 6 3  6 6 3
b+c c+a a+b  1 1 2
, and
3 3 3 or − , − , 
 6 6 3
1
V1 = [a b c ] and V2 = [ OG1 OG 2 OG 3 ]  1 1 2
6 and − , − ,− 
1 2  6 6 3
⇒ V2 = [ b + c c + a a + b ] = [a b c ]
27 27  −1 
The angle between in both the cases is cos−1   .
2  3
⇒ V2 = × 6V1 ⇒9V2 = 4V1
27 209. Elimination n between the given relations, we get
205. Let the equation of planes is lx + my + nz = p  al + bm 
2

p   p   p ul 2 + vm 2 + w   =0
∴A =  , 0, 0 B =  0, , 0 C =  0, 0,  respectively  −c 
l   m   n
l2 l
p p p ⇒ (c 2u + a 2w )+ 2abw . + (b 2w + c 2v ) = 0 …(i)
Centroid of OABC =  , ,  = ( x1, y1,z1 ) (say) m 2
m
 4l 4m 4n 
l l b 2w + c 2v
Q l 2 + m2 + n2 = 1 ∴ 1 ⋅ 2 = product of roots = 2
m1 m2 c u + a 2w
p2 p2 p2
∴ + + =1 ll mm nn
or 2 1 2 2 = 2 1 22 = 2 1 2 2 (by symmetry)
16 x1 16y1 16z12
2 2
bw +cv cu+aw av+bu
16
⇒ x12y12 + y12z12 + z12x12 = 2 x12y12z12 If lines are perpendicular, then
p l1l 2 + m1m2 + n1n2 = 0
∴ k = 16 ⇒ 2k = 32 ⇒ 5
2k = 2 ⇒ a 2(v + w ) + b 2(w + u ) + c 2(u + v ) = 0
206. l12 + m12 + n12 = 1, l 22 + m22 + n22 =1 Again, if the lines be parallel, then their d ′ c are equal so that
the roots of Eq. (i) should be equal, i.e. discriminate = 0
(l12 + m12 + n12 ) (l 22 + m22 + n22 ) − (l1l1 + m1m2 + n1n2 ) 2
∴ 4a 2b 2w 2 − 4(c 2u + a 2w ) (b 2w + c 2v ) = 0
= (m1n2 − m2n1 ) + (n1l 2 − n2l1 ) 2 + (l1m2 − l 2m1 ) 2
2
⇒ a 2c 2vw + b 2c 2uw + c 4uv = 0
(l1m2 − l 2m1 ) + (m1n2 − m2n1 ) 2 + (n1l 2 − n2l1 ) 2
2
a2 b2 c2
+ (l1l 2 + m1m2 + n1n2 ) 2 = 1 ⇒ + + =0
u v w
 n5 n4 n 210. The coordinates of any point on the line
207. Coordinates of the point, S =  , ,  x + 2 y + 1 z −3
 2 2 2 = = = λ are given by
3 2 2
 n5 n4 n
⇒ 2× + +  = −1 (3 λ − 2, 2 λ − 1, 2 λ + 3 )
2 2 2 The distance between the above point and (1, 2, 3) is 3 2.
⇒ n(n 4 + n 3 + 1 ) = − 1 ∴ (3 λ − 2 − 1 ) 2 + (2 λ − 1 − 2 ) 2 + (2 λ + 3 − 3 ) 2 = 3 2
n = − 1 is the only solution. 30
⇒ λ= ,0
208. We have, l +m+n=0 …(i) 17
 56 43 111
2l 2 + 2m 2 − n 2 = 0 ∴ Required points are ( −2, − 1, 3 ) and  , , 
and …(ii)  17 17 17 
Now, 2(l + m ) − n = 0
2 2 2
211. The required line is perpendicular to the lines which are
⇒ 2(1 − n 2 ) − n 2 = 0 [Ql 2 + m 2 + n 2 = 1] parallel to vectors b1 = 2 $i − 2 $j + k$ and b 2 = $i + 2 $j + 2 k$
⇒ 3n 2 = 2 respectively. So, it is parallel to the vector b = b1 × b 2.
$i $j k$
2
⇒ n=± Now, b = b1 × b 2 = 2 − 2 1 = − 6 i$ − 3 $j + 6 k$
3
Again, 2(l 2 + m 2 ) = n 2 1 2 2
⇒ 2[(l + m ) − 2lm ] = ( − (l + m ))
2 2
Thus, the required line passes through the point (2, − 1, 3 ) and
⇒ l =m is parallel to the vector b = − 6 i$ − 3 $j + 6 k$ .
2 2 So, its vector equation is
∴ l +m=± ⇒ 2l = ±
3 3 r = (2 $i − $j + 3 k$ ) + λ ( −6 $i − 3 $j + 6 k$ )
1 r = (2 $i − $j + 3 k$ ) + µ(2 $i + $j − 2 k$ ),
⇒ l =± =m or
6
where µ = − 3λ.
Chap 03 Three Dimensional Coordinate System 273

x −3 y −3 z PA × b
212. The coordinates of any point on the line = = are ∴ h=
2 1 1 | b|
x −3 y −3 z
given by = = = λ. Now, PA = − 3 $i + 2 $j − 2 k$ and b = 2 $i + $j + 4 k$
2 1 1
So, let the coordinates of A be (2 λ + 3, λ + 3, λ ). $i $j k$
π ∴ PA × b = −3 2 − 2 = 10 i$ + 8 − 2 k$
Let the line through O( 0, 0, 0 ) and making an angle with the
3 2 1 4
given line be along OA. Then, its d ′ r are proportional to
2 λ + 3 − 0, λ + 3 − 0, λ − 0 ∴ PA × b = 10 2 + 8 2 + ( −7 ) 2 = 213
or 2 λ + 3, λ + 3, λ and | b | = 2 2 + 1 2 + 4 2 = 21
The direction ratios of the given line are proportional to 2, 1, 1.
| PA × b|
It is given that the angle between the given line and the line ∴ h=
π | b|
along OA is .
3 213 71
= =
π (2 λ + 3 ) × 2 + ( λ + 3 ) × 1 + λ × 1 21 7
∴ cos =
3 (2 λ + 3 ) 2 + ( λ + 3 ) 2 + λ2 2 2 + 1 2 + 1 2 It is given that the length of BC is 5 units.
6λ + 9 1
= ∴ Area of ∆ABC = ( BC × h )
2
6 λ + 18 λ + 18 6
2
1 71 1775
∴ λ = − 1, − 2. = ×5 × = sq units.
2 7 28
O( 0, 0, 0 )
214. If the coordinates of the point P be (α, β, γ ).
α β γ
Then, + + =1 …(i)
a β c
Again d′ c of OP are proportional to α , β, γ and hence these are
π/3 π/3 also the d′ r of the normal to the plane which is perpendicular
A B to OP and since it passes through P, its equation is
α( x − α ) + β(y − β ) + γ(z − γ ) = 0
x –3 y –3 z
= = or αx + βy + γz = α 2 + β 2 + γ 2 …(ii)
2 1 1
It meets the axes in A, B, C and hence the coordinates of these
Putting these values of λ in the coordinates of A i.e.
(2 λ + 3, λ + 3, λ ), we find the coordinates of A and B i.e.  α 2 + β2 + γ 2 
points are  , 0, 0 etc.
A(1, 2, − 1 ) and B( −1, 1, − 2 ).  α 
So, the equations of OA and OB are The equation of the plane through A and parallel to the YZ
x−0 y −0 z −0 α 2 + β2 + γ 2
= = plane is x = .
1 − 0 2 − 0 −1 − 0 α
x–0 y–0 z–0 Similarly the equations of other planes are
= = α 2 + β2 + γ 2 α 2 + β2 + γ 2
–1 – 0 1 – 0 –2 – 0 y = and z = .
x y z β γ
or = =
1 2 –1 The locus of their point of intersection is obtained by
x y z elimination α , β, γ between the three equations of the planes
and = = and relation (i)
−1 1 −2
1 1 1 α 2 + β2 + γ 2
213. Clearly, height h of ∆ABC is the length of perpendicular from + + =
x + 2 y −1 z − 0 x 2 y 2 z 2 (α 2 + β 2 + γ 2 ) 2
A(1, − 1, 2 ) to the line = = which passes
2 1 4 1
= 2
through P( −2, 1, 0 ) and is parallel to b = 2 $i + $j + 4 k$ . α + β2 + γ 2
 α   β  γ 
A(1, –1, 2)   +  + 
1 1 1  a  b  c 
Again, + + =
ax by cz α 2 + β2 + γ 2
h 1
= 2 [from Eq. (i)]
α + β2 + γ 2
B C 1 1 1 1 1 1
P(–2,1, 0) M ∴ 2
+ 2 + 2 = + +
x y z ax by cz
274 Textbook of Vector & 3D Geometry

215. Any point on the line is (3r + 2, 4r − 1, 12r + 2). Since, S is directly above the mid-point T of diagonal OQ and
ST = 3.
If it lies on the plane x − y + z = 5, then
3 3 
(3r + 2 ) – ( 4r – 1 ) + (12r + 2 ) = 5 i.e. S  , , 3
2 2 
⇒r = 0
3 3 
Hence, point of intersection is (2, − 1, 2 ) . Here, DR’s of OQ (3, 3, 0) and DR’s of OS  , , 3 .
2 2 
Its distance from ( −1, − 5, − 10 ) is
9 9
(2 + 1 ) 2 + ( −1 + 5 ) 2 + (2 + 10 ) 2 +
2 2 9 1
∴ cosθ = = =
= 9 + 16 + 144 = 169 = 13 9+9+ 0
9 9
+ +9 18 ⋅
27 3
4 4 2
216. Any plane through the intersection of given planes is
( x + 3y + 6 + λ (3 x − y − 4z ) = 0 ∴ Option (a) is incorrect.
or (1 + 3 λ ) x + (3 − λ )y − 4 λz + 6 = 0 …(i) Now, equation of the plane containing the ∆OQS is
Its perpendicular distance from ( 0, 0, 0 ) is 1. x y z
3 3 0 =0
6
∴ =1 3/2 3/2 3
(1 + 3 λ ) 2 + (3 − λ ) 2 + ( −4 λ ) 2
x y z
⇒λ = ±1
⇒ 1 1 0 =0
∴Required planes are 2 x + y − 2z + 3 = 0 and
x − 2y − 2z − 3 = 0. 1 1 2

217. The image of the plane ⇒ x(2 − 0 ) − y (2 − 0 ) + z (1 − 1 ) = 0


x − 2y + 2z − 3 = 0 …(i) ⇒ 2 x − 2y = 0 or x − y = 0
in the plane x + y + z −1 = 0 …(ii) ∴ Option (b) is correct.
passes through the line of intersection of the given planes. Now, length of the perpendicular from P(3, 0, 0 ) to the plane
containing ∆OQS is
Therefore, the equation of such a plane is
|3 − 0| 3
( x − 2y + 2z − 3 ) + t( x + y + z − 1 ) = 0 =
1+1 2
⇒ (1 + t ) x + ( − 2 + t )y + (2 + t )z − 3 − t = 0 …(iii)
Now, plane (ii) makes the same angle with plane (i) and image ∴ Option (c) is correct.
plane (iii). Thus, Here, equation of RS is
1 −2 + 2 1 + t −2 + t + 2 + t x − 0 y −3 z − 0
=± = = =λ
3 3 3 (t + 1 ) 2 + (t − 2 ) 2 + (2 + t ) 2 3/2 −3 / 2 3
3 3
2 ⇒ x = λ , y = − λ + 3, z = 3λ
⇒ t = 0, − 2 2
3
2 To find the distance from O( 0, 0, 0 ) to RS.
For t = 0, we get plane (i); hence for image plane, t = − Let M be the foot of perpendicular.
3
The equation of the image plane O (0, 0, 0)
3( x − 2y + 2z − 3 ) − 2( x + y + z − 1 ) = 0
⇒ x − 8y + 4z − 7 = 0.
218. Given, square base OP = OR = 3
∴ P (3, 0, 0 ), R = ( 0,3, 0 ) R M S
(0,3,0)  3λ 3λ   3 , 3 3 
S  ,3− ,3λ  2 2
 2 2   

Q OM ⊥ RS ⇒ OM ⋅ RS = 0
θ R(0,3,0)
O Y 9λ 3  3λ 
⇒ − 3 −  + 3(3 λ ) = 0
4 2 2
1
T ⇒ λ=
P 3
(3,3,0)
Q(3,3,0)  5 
1
∴ M  , , 1
2 2 
X 1 25 30 15
⇒ OM = + +1= =
3 3  4 4 4 2
Also, mid-point of OQ is T  , , 0 .
2 2  ∴ Option (d) is correct.
Chap 03 Three Dimensional Coordinate System 275

219. Let image of Q(3, 1, 7) w.r.t. x − y + z = 3 be P (α, β, γ). Direction ratios of PQ are λ − α , λ − α , λ − 1.
α − 3 β − 1 γ − 7 − 2 (3 − 1 + 7 − 3 ) Now, PQ ⊥ L1
∴ = = = 2
1 −1 1 1 + ( − 1 ) 2 + (1 ) 2 ∴ 1( λ − α ) + 1 ⋅ ( λ − α ) + 0 ⋅ ( λ − 1 ) = 0
⇒ λ =α
⇒ α −3 =1 −β = γ −7 = − 4
Hence, Q( λ , λ ,1 )
∴ α = − 1, β = 5, γ = 3
Direction ratios of PR are λ − β, λ + β, λ + 1.
Q (3,1,7) Now, PR ⊥ L 2
∴ 1( λ − β ) + ( −1 )( λ + β ) + 0( λ + 1 ) = 0
λ −β − λ −β = 0 ⇒ β = 0
Hence, R ( 0, 0, − 1 )
Now, as ∠QPR = 90 °
x–y+z=3 [as a1a 2 + b1b2 + c1c 2 = 0, if two lines with DR’s a1,b1,c1;a 2 ,b2,c 2
are perpendicular]
∴ ( λ − λ )( λ − 0 ) + ( λ − λ )( λ − 0 ) + ( λ − 1 )( λ + 1 ) = 0
P(α, β, γ)
⇒ (λ − 1) ( λ + 1 ) = 0 ⇒ λ = 1 or λ = −1
Hence, the image of Q(3, 1, 7 ) is P( − 1, 5, 3 ). λ = 1, rejected as P and Q are different points.
To find equation of plane passing through ⇒ λ = −1
x y z 221. If two straight lines are coplanar,
P ( − 1, 5, 3 ) and containing = =
1 2 1 x − x1 y − y1 z − z1
i.e. = =
DR’s (1, 2, 1) a1 b1 c1
x − x2 y − y 2 z − z 2
and = = are coplanar
(0, 0, 0) (x, y, z) a2 b2 c2

A (x1, y1, z1)

P (–1, 5, 3) (a1,b1,c1) DR’s

x−0 y −0 z −0
⇒ 1−0 2−0 1−0 =0 B (x ,y
2 2 ,z
2)
−1 − 0 5 − 0 3 − 0 DR’s
(a2,b2,c2)
⇒ x (6 − 5 ) − y (3 + 1 ) + z (5 + 2 ) = 0
Then, ( x 2 − x1, y 2 − y1, z 2 − z1 ), (a1, b1, c1 ) and (a 2, b2, c 2 ) are
∴ x − 4y + 7z = 0
coplanar,
220. (i) Direction ratios of a line joining two points ( x1,y1, z1 ) and
x 2 − x1 y 2 − y1 z 2 − z1
( x 2,y 2,z 2 ) are x 2 − x1, y 2 − y1, z 2 − z1.
i.e. a1 b1 c1 =0
(ii) If the two lines with direction ratios a1,b1,c1;a 2,b2,c 2 are
perpendicular, then a1 a 2 + b1 b2 + c1 c 2 = 0 a2 b2 c2
Line L1 is given by y = x ;z = 1 can be expressed Here, x = 5,
y
=
z
x y z −1 3 − α −2
L 1: = = =α [say]
1 1 0 x −5 y −0 z −0
⇒ = = …(i)
⇒ x = α,y = α,z = 1 0 − (α − 3 ) −2
Let the coordinates of Q on L1 be (α ,α ,1 ). y z
and x = α, =
Line L 2 given by y = − x, z = −1 can be expressed as −1 2 − α
x y z +1
L 2: = = =β [say] x −α y − 0 z − 0
1 −1 0 ⇒ = = …(ii)
0 −1 2 −α
⇒ x = β,y = − β,z = −1
Let the coordinates of R on L 2 be ( β, − β, − 1 ). 5 −α 0 0
⇒ 0 3 −α −2 = 0
Q (λ, λ, 1)
0 −1 2 − α
⇒ (5 − α ) [(3 − α ) (2 − α ) − 2 ] = 0
⇒ (5 − α ) [α 2 − 5 α + 4 ] = 0
⇒ (5 − α ) (α − 1 ) (α − 4 ) = 0
P (λ, λ, λ) R (0, 0, –1) ∴ α = 1, 4, 5
276 Textbook of Vector & 3D Geometry

x − x1 y − y1 z − z1 x − 1 y − 0 z − ( −3 )
222. Equation of straight line is l : = = 224. L1 : = =
a b c 2 −1 1
Since, l is perpendicular to l1 and l 2 . $i $jk$
So, its DR’s are cross-product of l1 and l 2 . Normal of plane P : n = 7 1 2
Now, to find a point on l 2 whose distance is given, assume a 3 5 −6
point and find its distance to obtain point.
x−0 y −0 z −0 = i$( −16 ) − $j( −42 − 6 ) + k
$ (32 )
Let l: = =
a b c
= − 16 i$ + 48 $j + 32 k$
which is perpendicular to
DR’s of normal n = i$ − 3 $j − 2 k
$
l : (3 $i − $j + 4 k
1
$ ) + t( $i + 2 $j + 2 k
$)
Point of intersection of L1 and L2.
l 2 : (3 $i + 3 $j + 2 k
$ ) + s (2 $i + 2 $j + k
$)
⇒ 2K1 + 1 = K 2 + 4
$i $j k $
and −k1 = k2 − 3
∴ DR’s of l is 1 2 2 = −2 i$ + 3 $j − 2 k $ ⇒ k1 = 2 and k2 = 1
2 2 1 ∴ Point of intersection (5, − 2, − 1 )
x y z Now equation of plane,
l: = = = k1, k2 1 ⋅ ( x − 5 ) − 3(y + 2 ) − 2(z + 1 ) = 0
−2 3 −2
Now, A( −2k1, 3k1, − 2k1 ) and B( −2k2, 3k2, − 2k2 ). ⇒ x − 3y − 2z − 13 = 0
Since, A lies on l1. ⇒ x − 3y − 2z = 13
∴ ( −2k1 ) i$ + (3k1 ) $j − (2k1 ) k
$ = (3 + t ) $i + ( −1 + 2 t ) $j + ( 4 + 2t ) k
$ ∴ a = 1, b = − 3, c = − 2, d = 13
⇒ 3 + t = − 2k1,−1 + 2 t = 3k1, 4 + 2 t = − 2k1 x −1 y + 1 z
225. Since, = =
∴ k1 = − 1 2 K 2
⇒ A(2, − 3, 2 ) x+1 y +1 z
and = = are coplanar.
Let any point on l 2 (3 + 2s , 3 + 2s , 2 + s ) 5 2 k
2 0 0
(2 − 3 − 2s ) 2 + ( −3 − 3 − 2s ) 2 + (2 − 2 − s ) 2 = 17
⇒ 2 K 2 =0
⇒ 9s 2 + 28s + 37 = 17
5 2 K
⇒ 9s 2 + 28s + 20 = 0
⇒ K2 = 4 ⇒ K = ± 2
⇒ 9s 2 + 18s + 10s + 20 = 0
∴ n1 = b1 × d1 = 6 $j − 6 k$ , for k = 2
⇒ ( 9s + 10 ) (s + 2 ) = 0
−10 ∴ n 2 = b 2 × d 2 = 14 $j + 14 k$ , for k = − 2
∴ s = − 2, ⋅
9 So, equation of planes are ( r − a ) ⋅ n1 = 0
 7 7 8 ⇒ y − z = − 1 and ( r − a ) ⋅ n 2 = 0
Hence, ( −1, − 1, 0 ) and  , ,  are required points. ⇒ y + z = −1
 9 9 9
x+2 y +1 z 226. Equation of the plane containing the lines
223. Any point on = = =λ x −2 y − 3 z − 4
2 −1 3 = =
⇒ x = 2 λ − 2, y = − λ − 1, z = 3 λ 3 4 5
Let foot of perpendicular from ( 2 λ − 2, − λ − 1, 3 λ ) x −1 y − 2 z − 3
and = =
to x + y + z = 3 be ( x 2, y 2, z 2 ). 2 3 4
x 2 − (2 λ − 2 ) y 2 − ( − λ − 1 ) z 2 − ( 3 λ ) is a( x − 2 ) + b(y − 3 ) + c (z − 4 ) = 0 ...(i)
∴ = =
1 1 1 where, 3a + 4b + 5c = 0 ...(ii)
(2 λ − 2 − λ − 1 + 3 λ − 3) 2a + 3b + 4c = 0 ...(iii)
=−
1+1+1 and a(1 − 2 ) + b( 2 − 3 ) + c( 2 − 3 ) = 0
4λ i.e. a+b+c=0 …(iv)
⇒ x 2 − 2λ + 2 = y 2 + λ + 1 = z 2 − 3λ = 2 − a b c
3 From Eqs. (ii) and (iii), = = , which satisfy Eq. (iv).
2λ 7λ 5λ 1 −2 1
∴ x2 = , y2 = 1 − , z2 = 2 + Plane through lines is x − 2y + z = 0.
3 3 3
x2 − 0 y 2 − 1 z 2 − 2 Given plane is A x − 2y + z = d is 6.
⇒ λ= = =
2/3 −7 / 3 5/3 ∴ Planes must be parallel, so A = 1 and then
Hence, foot of perpendicular lie on d
= 6
x y −1 z −2 x y −1 z −2 6
= = ⇒ = =
2 / 3 −7 / 3 5 / 3 2 −7 5 ⇒ d =6
Chap 03 Three Dimensional Coordinate System 277

227. The equation of the plane passing through the point Then the DR’s of line of intersection of planes is < 14, 2, 15 >
( −1, − 2, − 1 ) and whose normal is perpendicular to both the and line is
given lines L1 and L2 may be written as x −3 y + 1 z − 0
= = =λ [say]
( x + 1 ) + 7 (y + 2 ) − 5 (z + 1 ) = 0 ⇒ x + 7y − 5z + 10 = 0 14 2 15
The distance of the point (1, 1, 1) from the plane ⇒ x = 14 λ + 3, y = 2 λ − 1, z = 15 λ
1 + 7 − 5 + 10 13
= = units Hence, Statement I is false.
1 + 49 + 25 75 But Statement II is true.
228. The shortest distance between L1 and L2 is a b c
232. Let ∆ = b c a
{(2 − ( −1 )) i$ + (2 − 2 ) $j + (3 − ( −1 )) k$ } ⋅ ( − $i − 7 $j + 5 k$ )
c a b
5 3
1
(3 $i + 4 k$ ) ⋅ ( − $i − 7 $j + 5 k$ ) = − (a + b + c )[(a − b ) 2 + (b − c ) 2 + (c − a ) 2 ]
= 2
5 3
(A) If a + b + c ≠ 0 and a 2 + b 2 + c 2 = ab + bc + ca
17 ⇒ ∆ = 0 and a = b = c ≠ 0
= units
5 3
⇒ The equations represent identical planes.
229. The equations of given lines in vector form may be written as (B) a + b + c = 0 and a 2 + b 2 + c 2 ≠ ab + bc + ca

L1 : r = ( − $i − 2 $j − k$ ) + λ (3 $i + $j + 2 k$ ) ⇒ ∆=0

and L2 : r = (2 $i − 2 $j + 3 k$ ) + µ ( $i + 2 $j + 3 k$ ) ⇒ The equations have infinitely many solutions.
ax + by = (a + b ) z , bx + cy = (b + c ) z
Since, the vector is perpendicular to both L1 and L2 .
$i $j k$ ⇒ (b 2 − ac ) y = (b 2 − ac ) z ⇒ y = z
⇒ ax + by + cy = 0 ⇒ ax = ay ⇒ x = y = z
3 1 2 = − $i − 7 $j + 5 k$
1 2 3 (C) a + b + c ≠ 0 and a 2 + b 2 + c 2 ≠ ab + bc + ca
⇒ ∆≠0
∴ Required unit vector
The equations represent planes meeting at
( − $i − 7 $j + 5 k$ )
= only one point.
( −1 ) 2 + ( −7 ) 2 + (5 ) 2
(D) a + b + c = 0 and a 2 + b 2 + c 2 = ab + bc + ca
1 ⇒ a =b =c = 0
= ( − $i − 7 $j + 5 k$ )
5 3
⇒ The equations represent whole of the
230. Given three planes are three-dimensional space.
P1 : x − y + z = 1 ...(i) x y z
233. Any line parallel to = = and passing through P(1, − 2, 3 )
P2 : x + y − z = − 1 ...(ii) 1 4 5
and P3 : x − 3y + 3z = 2 ...(iii) is
P (1, –2, 3)
On solving Eqs. (i) and (ii), we get x y z <1, 4, 5>
= =
x = 0, z = 1 + y 1 4 5
which does not satisfy Eq. (iii).
As x − 3y + 3z = 0 − 3y + 3 (1 + y ) = 3 ( ≠ 2 ) R 2x + 3y – 4z + 22 = 0
So, Statement II is true.
Next, since we know that direction ratios of line of intersection
of planes a1x + b1y + c1z + d1 = 0 Q

a 2 x + b2y + c 2z + d 2 = 0 is x −1 y + 2 z −3
and = = =λ (say)
b1c 2 − b2c1, c1a 2 − a1c 2, a1b2 − a 2b1 1 4 5
Using above result, Any point on above line can be written as
Direction ratios of lines L1, L2 and L3 are ( λ + 1, 4 λ − 2, 5 λ + 3 ).
0, 2, 2 ; 0, − 4, − 4 ; 0, − 2, − 2 ∴Coordinates of R are ( λ + 1, 4 λ − 2, 5 λ + 3 ).
Since, all the three lines L1, L2 and L3 are parallel pairwise. Since, point R lies on the above plane.
Hence, Statement I is false. ∴ 2( λ + 1 ) + 3( 4 λ − 2 ) − 4(5 λ + 3 ) + 22 = 0 ⇒ λ = 1
231. Given planes are 3x − 6y − 2z = 15 and 2x + y − 2z = 5. So, point R is (2, 2, 8).
For z = 0, we get x = 3, y = − 1 Now, PR = (2 − 1 ) 2 + (2 + 2 ) 2 + (8 − 3 ) 2 = 42
Since, direction ratios of planes are
∴ PQ = 2 PR = 2 42
< 3, − 6, − 2 > and < 2, 1, − 2 >
278 Textbook of Vector & 3D Geometry

234. Given, equations of lines are Any point on the line (i) is
x −1 y + 2 z − 4 x −2 y + 1 z + 7 (3 λ + 2, 4 λ − 1, 12 λ + 2)
= = and = =
1 −2 3 2 −1 −1 Let this point be point of intersection of the line and plane.
∴ (3 λ + 2 ) − ( 4 λ − 1 ) + (12 λ + 2 ) = 16
Let n1 = $i − 2 $j + 3 k$ and n 2 = 2 $i − $j − k$
⇒ 11 λ + 5 = 16
∴Any vector n perpendicular to both n1, n 2 is given by ⇒ 11 λ = 11
n = n1 × n 2 ⇒ λ =1
$i $j k$ ∴Point of intersection is (5, 3, 14).
⇒ n = 1 − 2 3 = 5 $i + 7 $j + 3 k$ Now, distance between the points (1, 0, 2) and (5, 3, 14)
2 −1 −1 = (5 − 1 ) 2 + (3 − 0 ) 2 + (14 − 2 ) 2
= 16 + 9 + 144
∴Equation of a plane passing through (1, − 1, − 1 ) and
perpendicular to n is given by = 169
5( x − 1 ) + 7(y + 1 ) + 3(z + 1 ) = 0 = 13
⇒ 5 x + 7y + 3z + 5 = 0 238. Let equation of plane containing the lines 2x − 5y + z = 3 and
x + y + 4z = 5 be
5 + 21 − 21 + 5 10 (2 x − 5y + z − 3 ) + λ ( x + y + 4z − 5 ) = 0
∴Required distance = = units
52 + 72 + 32 83 ⇒ (2 + λ ) x + ( λ − 5 )y + ( 4 λ + 1 )z − 3 − 5 λ = 0 …(i)
235. Equation of line passing through (1, − 5, 9) and parallel to This plane is parallel to the plane x + 3y + 6z = 1.
x = y = z is 2 + λ λ − 5 4λ + 1
∴ = =
x −1 y + 5 z −9 1 3 6
= = =λ (say)
1 1 1 On taking first two equalities, we get
Thus, any point on this line is of the form 6 + 3λ = λ − 5
( λ + 1, λ − 5, λ + 9 ). ⇒ 2 λ = − 11
Now, if P( λ + 1, λ − 5, λ + 9 ) is the point of intersection of line 11
⇒ λ=−
and plane, then 2
(λ + 1) − (λ − 5) + λ + 9 = 5 On taking last two equalities, we get
⇒ λ + 15 = 5 6 λ − 30 = 3 + 12 λ
⇒ λ = − 10 ⇒ −6 λ = 33
11
∴Coordinates of point P are ( − 9, − 15, − 1 ). ⇒ λ=−
Hence, the required distance 2
So, the equation of required plane is
= (1 + 9 ) 2 + ( − 5 + 15 ) 2 + (9 + 1 ) 2
 11  −11   44  11
2 −  x +  − 5 y +  − + 1 z − 3 + 5 × = 0
= 10 2 + 10 2 + 10 2 = 10 3  2  2   2  2
x −3 y + 2 z + 4 7
⇒ − x− y − z+
21 42 49
=0
236. Since, the line = = lies in the plane
2 −1 3 2 2 2 2
lx + my − z = 9, therefore we have ⇒ x + 3y + 6z − 7 = 0
2l − m − 3 = 0 239. Given, l + m + n = 0 ⇒ l = − ( m + n )
[Q normal will be perpendicular to the line] ⇒ (m + n ) 2 = l 2
⇒ 2l − m = 3 ...(i)
⇒ m 2 + n 2 + 2mn = m 2 + n 2 [Q l 2 = m 2 + n 2, given]
and 3 l − 2m + 4 = 9
⇒ 2mn = 0
[Q point (3, − 2, − 4 ) lies on the plane]
⇒ 3l − 2m = 5 ...(ii)
Case I When m = 0, then
On solving Eqs. (i) and (ii), we get
l = −n
l = 1 and m = − 1
Hence, ( l, m, n ) is (1, 0, − 1 ).
∴ l 2 + m2 = 2
Case II When n = 0, then
237. Given equation of line is l = −m
x −2 y + 1 z −2 Hence, ( l, m, n ) is (1, 0, –1).
= = =λ (say) …(i)
3 4 12 1+0+0 1
∴ cosθ = =
and equation of plane is 2× 2 2
x − y + z = 16 …(ii) π
⇒ θ=
3
Chap 03 Three Dimensional Coordinate System 279

240. Plane and line are parallel to each other. Equation of normal to Note : If 0 appears in the denominator, then the correct way of
the plane through the point (1, 3, 4) is representing the equation of straight line is
x −1 y −3 z − 4 x −2 y −3 y − 4 z −5
= = =k [Say] = ; z = 4 and x = l ; =
2 −1 1 1 1 2 1
Any point in this normal is 243. Given A plane P : x − 2y + 2z − 5 = 0
(2k + 1 , − k + 3, 4 + k ).
To find The equation of a plane parallel to given plane P and
 2k + 1 + 1 3 − k + 3 4 + k + 4
⇒  , ,  lies on plane. at a distance of 1 unit from origin. Equation of family of planes
 2 2 2  parallel to the given plane P is
6 − k 8 + k Q : x − 2y + 2z + d = 0
⇒ 2 (k + 1 ) −   +  +3=0
 2   2  Also, perpendicular distance of Q from origin is 1 unit.
⇒ k = −2 0 − 2( 0 ) + 2( 0 ) + d
⇒ =1
Hence, point through which this image pass is 12 + 22 + 22
(2k + 1 , 3 − k, 4 + k ) d
⇒ =1 ⇒ d = ± 3
i.e. (2( −2 ) + 1 , 3 + 2, 4 − 2 ) = ( −3, 5 , 2 ) 3
Hence, equation of image line is Hence, the required equation of the plane parallel to P and at
x + 3 y −5 z −2 unit distance from origin is
= =
3 1 −5 x − 2y + 2z ± 3 = 0
241. Given planes are Hence, out of the given equations, option (a) is the only correct
2 x + y + 2z − 8 = 0 option.
5 x −1 y + 1 z −1
and 2 x + y + 2z + = 0 244. Given Two lines L1 : = =
2 2 3 4
Distance between two parallel planes x −3 y −k z − 0
and L2 : = =
5 1 2 1
−8 −
| d1 − d 2 | 2 To find The value of ‘k’ of the given lines L1 and L2 are
= =
a2 + b2 + c2 22 + 12 + 22 intersecting each other.
x −1 y + 1 z −1
Let L1 : = = =p
21 2 3 4
7 x −3 y −k z − 0
= 2 = and L2 : = = =q
3 2 1 2 1
242. The given line are ⇒ Any point P on line L1 is of type
x −2 y −3 z − 4 P (2 p + 1, 3 p − 1, 4 p + 1 ) and any point Q on line L2 is of type
= = …(i) Q(q + 3, 2q + k, q ).
1 1 −k
x −1 y −k z −5 Since, L1 and L2 are intersecting each other, hence both points
and = = …(ii) P and Q should coincide at the point of intersection, i.e.,
k 2 1
Condition for two lines are coplanar. corresponding coordinates of P and Q should be same.
x1 − x 2 y1 − y 2 z1 − z 2 2 p + 1 = q + 3, 3 p − 1 = 2q + k and 4 p + 1 = q
l1 m1 n1 =0 On solving 2 p + 1 = q + 3 and 4 p + 1 = q, we get the values of p
and q as
l2 m2 n2 −3
p= and q = − 5
wher,e ( x1, y1, z1 ) and ( x 2, y 2 z 2 ) are any points on the lines (i) 2
and (ii), respectively and < l1, m1, n1 > and < l 2, m2, n2 > are On substituting the values of p and q in the third equation
direction cosines of lines (i) and (ii), respectively. 3 p − 1 = 2q + k, we get
2 −1 3 − 4 4 −5  −3 
∴ −k = 0 3   − 1 = 2( −5 ) + k
1 1  2
k 2 1 9
⇒ k=
1 −1 −1 2
⇒ 1 1 −k = 0 245. Angle between straight line r = a + λb and plene r ⋅ n$ = d is
k 2 1 b ⋅ n$
sin θ =
⇒ 1 (1 + 2k ) + 1 (1 + k 2 ) − (2 − k ) = 0 | b | | n$ |
⇒ k 2 + 2k + k = 0 ( $i + 2 $j + λ k$ ) ⋅ ( $i + 2 $j + 3 k)
$
∴ sin θ =
⇒ k 2 + 3k = 0 1 + 4 + λ2 1+4+9
⇒ k = 0, − 3
280 Textbook of Vector & 3D Geometry

5 + 3λ Hence, coordinates of Q are (2, 5, 7 )


=
λ2 + 5 14 ∴ | PQ| = (3 – 2 ) 2 + (–1 – 5 ) 2 + (7 – 11 ) 2
5 = 1 + 36 + 16 = 53
Given, cos θ =
14 248. Let Q be any point on the plane.
3 3 5 + 3λ
∴ sin θ = ⇒ = Then equation of PQ is
14 14 λ2 + 5 ⋅ 14 x –1 y + 5 z –9
= = =λ
⇒ 9 ( λ2 + 5 ) = 9 λ2 + 30 λ + 25 1 1 1
⇒ 9 λ2 + 45 = 9 λ2 + 30 λ + 25 where P = (1, – 5, 9 )
2
⇒ 30 λ = 20 ⇒ λ =
3
Q
246. Mid-point of AB is M (1, 3, 5).
A (1, 0, 7)

∴ x = λ + 1, y = λ – 5, z = λ + 9 lies on the plane


x –y + z = 5
M x y–1 z–2 ⇒ λ + 1– λ + 5 + λ + 9 = 5
1 2 3 ∴ λ = –10
Hence, coordinate of Q is Q(–9, – 15, – 1 )
B (1, 6, 3) ∴ | PQ| = (10 ) 2 + (10 ) 2 + (10 ) 2 = 10 3
x y −1 z −2
which lies on = = 249. We know that, cos2 45°+ cos2 120°+ cos2 θ = 1
1 2 3 1 1 1
1 3 −1 5 −2 ⇒ + + cos2 θ = 1 ⇒ cos2 θ =
as = = ⇒1 = 1 = 1 2 4 4
1 2 3 1
Hence, Statement II is true. ⇒ cosθ = ± ⇒ θ = 60 ° or 120°
2
Also, directions ratios of AB is
250. The image of the point (3, 1, 6) with respect to the plane
(1 − 1, 6 − 0, 3 − 7 ) x − y + z = 5 is
i.e. ( 0, 6, − 4 ) …(i) x − 3 y − 1 z − 6 − 2 (3 − 1 + 6 − 5 )
= = =
and direction ratios of straight line is 1 −1 1 1+1+1
(1, 2, 3 ) …(ii)  x − x1 y − y1 z − z1 −2(ax1 + by1 + cz1 + d ) 
The two lines are perpendicular, if Q a = b = c = 
 a2 + b2 + c2 
0 (1 ) + 6 (2 ) − 4 (3 ) = 12 − 12 = 0
x −3 y −1 z −6
Hence, Statement I is true and statement II is a correct ⇒ = = = −2
1 −1 1
explanations of statement II.
⇒ x = 3 −2 = 1
247. Let the coordinates of Q be (2λ, 3λ + 2, 4λ + 3) which is any
y =1+2 =3
point on the straight line AB.
and z = 6 −2 = 4
P(3, –1, 11) which shows that Statement I is true.
We observe that the line segment joining the points
A(3, 1, 6) and B(1, 3, 4) has direction ratios 2, – 2, 2 which are
proportional to 1, – 1, 1. The direction ratios of the normal to
the plane. Hence, Statements II is true. Thus, the Statements I
and II are true and Statement II is correct explanation of
Statement I.
A B
Q 251. Dr’s of given line are (3, − 5, 2) .
∴ DR’s of PQ is (2 λ – 3, 3 λ + 3, 4 λ – 8 ) Dr’s of normal to the plane = (1, 3, − α )
x y –2 z –3 ∴ Line is perpendicular to the normal.
Also, perpendicular to straight line AB = = =λ
2 3 4 ⇒ 3(1 ) − 5(3 ) + 2( − α ) = 0
having DR’s (2, 3, 4 ). ⇒ 3 − 15 − 2α = 0
Thus, 2(2 λ – 3 ) + 3(3 λ + 3 ) + 4( 4 λ – 8 ) = 0 ⇒ 2α = − 12 ⇒ α = − 6
⇒ 4 λ – 6 + 9 λ + 9 + 16 λ – 32 = 0 Also, point (2, 1, − 2 ) lies on the plane.
⇒ 29 λ – 29 = 0 ∴ 2 + 3 + 6 ( − 2 ) + β = 0 ⇒β = 7
∴ λ =1 ⇒ (α , β ) = ( − 6, 7 )
Chap 03 Three Dimensional Coordinate System 281

252. Projection of a vector on coordinate axes are From Eqs. (i) and (ii), we get
x 2 − x1, y 2 − y1, z 2 − z1 l m n
= = =k [say]
x 2 − x1 = 6, 3 –3 3
y 2 − y1 = − 3, We know that, l 2 + m 2 + n 2 = 1
z 2 − z1 = 2 ∴ (3k ) 2 + ( −3k ) 2 + (3k ) 2 = 1
Now, ( x 2 − x1 ) + (y 2 − y1 ) + (z 2 − z1 )
2 2 2
1
⇒ 27k 2 = 1 ⇒ k =
= 36 + 9 + 4 = 7 3 3
1 1
6 3 2
So, the DC’s of the vector are , − , . ∴ l = ⇒ cos α =
3 3
7 7 7
π
253. Equation of line passing through (5, 1, a) and (3, b, 1) is 256. Since, a line makes an angle of with positive direction of
4
x −3 y −b z −1 each of X-axis and Y-axis, therefore
= = ... (i)
5 −3 1 −b a −1 π π
α = ,β =
 x − x1 y − y1 z − z1  4 4
Q x − x = y − y = z − z  We know that, cos2 α + cos2 β + cos2 γ = 1
 2 1 2 1 2 1

 17 13 π π
Point  0, , −  satisfies Eq. (i), we get ⇒ cos2 + cos2 + cos2 γ = 1
 2 2 4 4
17 13 1 1
−b − −1 ⇒ + + cos γ = 12
3 2 2 2
− = = 2
2 1 −b a −1 ⇒ cos2 γ = 0 ⇒ γ = 90 °
 15  257. Given, equation of sphere is
− 
 2
⇒ a −1 = =5 ⇒ a =6 x 2 + y 2 + z 2 − 6 x − 12y − 2 z + 20 = 0
 3
−  whose coordinates of centre are (3, 6, 1).
 2
Since, one end of diameter are (2, 3, 5 ) and the other end of
 17 
Also, − 3 (1 − b ) = 2  − b diameter be (α, β, γ),
2 
α +2 β+3 γ+5
then = 3, = 6, =1
⇒ 3b − 3 = 17 − 2b 2 2 2
⇒ 5b = 20 ⇒b = 4 ⇒ α = 4, β = 9
x −1 y −2 z −3 and γ = −3.
254. Given, = = ... (i) Hence, the coordinates of other point are (4, 9, – 3).
k 2 3
x −2 y −3 z −1 258. Given equations of lines are
and = = ... (ii)
3 k 2 x = ay + b, z = cy + d
Since, lines intersect at a point. Then, shortest distance and x = a ′ y + b′ , z = c ′ y + d ′
between them is zero. These equations can be rewritten as
k 2 3 x −b y − 0 z −d
= =
∴ 3 k 2 =0 a 1 c
1 1 −2 x −b′ y − 0 z −d ′
and = =
⇒ k( − 2k − 2 ) − 2( − 6 − 2 ) + 3(3 − k ) = 0 a′ 1 c′
⇒ − 2k 2 − 5k + 25 = 0 These lines will perpendicular, if aa′ + 1 + cc′ = 0
⇒ 2k 2 + 5k − 25 = 0 Q l1l 2 + m1m2 + n1n2 = 6
⇒ 2k 2 + 10k − 5k − 25 = 0 259. We know that, the image (x, y , z ) of a point (x1, y1, z1 ) in a
⇒ 2k(k + 5 ) − 5(k + 5 ) = 0 plane ax + by + cz + d = 0 is given by
5 x − x1 y − y1 z − z1
⇒ k = , −5 = =
2 a b c
Hence, integer value of k is − 5. −2 (ax1 + by1 + cz1 + d )
= .
a2 + b2 + c2
255. Let the drection cosines of line L be l, m and n. Since, the line
intersect the given planes, then the normal to the planes are Thus, the image of point ( −1, 3, 4 ) in a plane x − 2y = 0 is given
perpendicular to the line L. by
∴ 2l + 3m + n = 0 …(i) x + 1 y −3 z − 4
= =
and l + 3m + 2n = 0 …(ii) 1 −2 0
282 Textbook of Vector & 3D Geometry

−2 [1 × ( −1 ) + ( −2 ) × 3 + 0 × 4 ] 262. If a1, b1, c1 and a 2,b2,c 2 are DR’s of two lines, then the angle
=
1+4 between them is given by
x + 1 y − 3 z − 4 −2 ( −7 )
⇒ = = = a1a 2 + b1b2 + c1 c 2
1 −2 0 5 cosθ =
14 9 28 13 a12 + b12 + c12 a 22 + b22 + c 22
⇒ x= −1 = ,y = − +3=−
5 5 5 5 The given equations can be rewritten as
z =4 x y z x y z
and = = and = =
 9 13  3 2 −6 2 −12 −3
Hence, the image of point ( −1, 3, 4 ) is  , − , 4 .
5 5  ∴ Angle between the lines is given by
6 − 24 + 18
260. Centre of sphere x 2 + y 2 + z 2 + 2 ux + 2vy + 2wz + d = 0 is cos θ =
9 + 4 + 36 4 + 144 + 9
( −u, − v, − w ).
0
Given equation of first sphere is = =0
49 157
x 2 + y 2 + z 2 + 6 x − 8y − 2 z = 13 …(i)
⇒ θ = 90°
whose centre is ( −3, 4, 1 )
and equation of second sphere is 263. Since, the centre of sphere
1 1
x 2 + y 2 + z 2 − 10 x + 4y − 2 z = 8 …(ii) x 2 + y 2 + z 2 − x + z − 2 = 0 is  , 0, −  and radius of sphere
2 2
whose centre is (5, –2, 1).
1 1 10
Mid-point of ( −3, 4, 1 ) and (5 , − 2 , 1 ) is (1, 1, 1 ). = + +2=
4 4 2
Since, the plane passes through (1, 1, 1).
∴ 2a − 3a + 4a + 6 = 0
⇒ 3a = − 6 ⇒ a = − 2
O
261. Direction ratios of line normal are
(a1, b1, c1 ) = (1, 2, 2 ) B A
and direction ratios of a plane are
(a 2, b2, c 2 ) = (2, − 1, λ ) Distance of plane from centre of sphere
a1a 2 + b1b2 + c1c 2 1 1
Since, sinθ = + −4
a12 + b12 + c12 a 22 + b22 + c 22 3
= 2 2 =
1+4+1 6
1 × 2 + 2( −1 ) + 2 × λ
=
(1 ) + (2 ) 2 + (2 ) 2 (2 ) 2 + (1 ) 2 + ( λ ) 2 10 9
So, radius of circle = −
1 2 λ 4 6
⇒ = ⇒5 + λ = 4 λ
3 3 5+ λ 30 − 18 12
= = =1
5 12 12
⇒ λ=
3

You might also like